{ "cells": [ { "cell_type": "markdown", "metadata": {}, "source": [ "We shall work on several concepts of statistics in this notebook, always in the context of data coming from independent Bernoulli trials with an unknown parameter $p$." ] }, { "cell_type": "markdown", "metadata": {}, "source": [ "Assume that we have a coin flipping experiment with a coin, chosen from a pool of coins, whose success probability is $p$. Let us consider $1$ as success and $0$ a fail. The law which determines $p$ has a density $\\pi_0(p)$. Let us then flip the coin $k$ times with result $x_1,\\ldots,x_k$. What is the probability law for $p$ given $x_1,\\ldots,x_k$." ] }, { "cell_type": "code", "execution_count": 122, "metadata": {}, "outputs": [ { "name": "stdout", "output_type": "stream", "text": [ "0.8\n", "[1 1 1 1 1 1 1 0 1 0 1 1 0 1 1 1 0 1 1 1 1 0 1 1 1 1 0 1 1 1 1 0 1 1 1 1 1\n", " 0 1 1 1 1 0 1 1 1 1 1 1 1 1 1 1 1 1 1 1 1 1 1 1 1 0 1 1 0 0 1 1 1 1 1 1 0\n", " 1 0 1 0 1 1 1 1 1 1 0 1 1 1 1 1 1 1 1 0 1 0 1 1 1 1]\n" ] } ], "source": [ "import numpy\n", "\n", "p0 = 0.8 #numpy.random.uniform(0,1,1)\n", "def pi0(p):\n", " return 1\n", "\n", "k=100\n", "x = numpy.random.binomial(1,p0,k)\n", "y = numpy.cumsum(x)\n", "print(p0)\n", "print(x)\n", "#print(y)" ] }, { "cell_type": "markdown", "metadata": {}, "source": [ "# Bayesian Approach\n", "\n", "This a typical question for conditional distribution. We consider the random variables $X_1,\\ldots,X_k$ which are Bernoulli distributed with success parameter $p$. Then\n", "$$\n", "P(X_1=x_1,\\ldots,X_k=x_k|p) = p^{(x_1+\\ldots+x_k)}(1-p)^{(k-(x_1+\\ldots+x_k))}\n", "$$\n", "holds true. Bayes' theorem says\n", "$$\n", "P(p|x_1,\\ldots,x_k)=\\frac{p^{(x_1+\\ldots+x_k)}(1-p)^{(k-(x_1+\\ldots+x_k))}\\pi_0(p)}{\\int_0^1p^{(x_1+\\ldots+x_k)}(1-p)^{(k-(x_1+\\ldots+x_k))}\\pi_0(p)dp} \\, .\n", "$$" ] }, { "cell_type": "code", "execution_count": 123, "metadata": {}, "outputs": [ { "data": { "image/png": "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\n", "text/plain": [ "
" ] }, "metadata": { "needs_background": "light" }, "output_type": "display_data" }, { "name": "stdout", "output_type": "stream", "text": [ "0.6670003336670003\n" ] }, { "data": { "image/png": "iVBORw0KGgoAAAANSUhEUgAAAXcAAAD8CAYAAACMwORRAAAABHNCSVQICAgIfAhkiAAAAAlwSFlzAAALEgAACxIB0t1+/AAAADl0RVh0U29mdHdhcmUAbWF0cGxvdGxpYiB2ZXJzaW9uIDMuMC4zLCBodHRwOi8vbWF0cGxvdGxpYi5vcmcvnQurowAAIABJREFUeJzt3Xl8lNW9x/HPj4RAwhZCgmBICLsiyBbWWnfrWmktKlpUkIpotbZWe6221nrbXpdWq8WrcoWyiGxqFeuuaBUXIGELEJawJhAIEEiAkP3cP5J6uTGQCZmZJzP5vl+vvDKTOcn8nizfOTnPec4x5xwiIhJemnldgIiI+J/CXUQkDCncRUTCkMJdRCQMKdxFRMKQwl1EJAwp3EVEwpDCXUQkDCncRUTCUKRXTxwfH+9SUlK8enoRkZCUnp6+3zmXUFc7z8I9JSWFtLQ0r55eRCQkmdkOX9ppWEZEJAwp3EVEwpDCXUQkDCncRUTCUJ3hbmYtzWyZma02s3Vm9vta2rQws/lmlmVmS80sJRDFioiIb3zpuZcAFzrnBgADgcvMbESNNhOBg865nsDTwOP+LVNEROqjznB3VY5U321e/VZz+6bRwMzq268CF5mZ+a1KERGpF5/G3M0swsxWAXnAh865pTWaJALZAM65cqAA6ODPQkVEwsEzH21m3e6CgD+PT+HunKtwzg0EugDDzKxfjSa19dK/tTmrmU0yszQzS9u3b1/9qxURCWEL0rJ5+qNNvL0mN+DPVa/ZMs65Q8CnwGU1HsoBkgDMLBJoB+TX8vlTnXOpzrnUhIQ6r54VEQkba3cV8Js31jKqRwfuvaR3wJ/Pl9kyCWYWW307GrgY2FCj2SLglurbY4DFzrlv9dxFRJqi/KOl3D47nfhWUfzthkFERgR+Frova8t0BmaaWQRVLwYLnHP/NLNHgTTn3CJgGjDbzLKo6rGPDVjFIiIhpLyikrvnrmDfkRIW3j6SDq1bBOV56wx359waYFAtH3/4uNvFwLX+LU1EJPQ98f5Gvsg6wJNjzmZAUmzQnldXqIqIBMibq3Yx9bOt3DyyK9emJgX1uRXuIiIBsH53If/x2hqGpcTx26v6Bv35Fe4iIn528Ggpk2anERsdxXM/HkzzIJxArcmzzTpERMJR1QnUleQVlrBg8kgS2gTnBGpNCncRET968v2NLMnazxM/OpuBQTyBWpOGZURE/OSt1bt58bOtjBuRzHVDg3sCtSaFu4iIH2TmFvKrV9eQ2rU9D191ltflKNxFRBrqUFHVCdS20ZH897jBREV6H60acxcRaYB/n0DdW1DCvNtH0LFNS69LAhTuIiIN8l/vbuDzzft57Jr+DE5u73U53/D+fwcRkRC1YHk205ZsY/yoFMYOS/a6nP9H4S4icgrStufz0BsZnNMznt9ceabX5XyLwl1EpJ52HTrG5JfTSYyNZsqNwVnCt7405i4iUg9FpeXcNjONkrJK5k0aSmxMlNcl1UrhLiLio8pKx30LV5O5p5Dp44fSs2Nrr0s6ocb3v4SISCP17OLNvJOxh19ffgYX9OnodTknpXAXEfHBuxm5/PWjzVwzOJHbvtvd63LqpHAXEanDut0F3LtgNYOSY/nTD/tjZl6XVCeFu4jISew7XMJtM9NoF92cF8cNoWXzCK9L8olOqIqInEBJeQV3vJxOflEpC28fRce2jWNpAV8o3EVEauGc44HXMkjbcZC/3TCI/l3aeV1SvWhYRkSkFlMWZ/GPlbu495LefH/A6V6XU28KdxGRGhat3s1fPtzEDwclcveFPb0u55Qo3EVEjpO+4yD3LVzN0JT2PPaj0JgZU5s6w93MkszsEzPLNLN1ZnZPLW3ON7MCM1tV/fZwYMoVEQmc7PwiJs1Ko3O7lrx4UyotIkNjZkxtfDmhWg780jm3wszaAOlm9qFzbn2Ndp87567yf4kiIoFXWFzGrTOWU1ZRyfTxQ4lr1TjXjPFVnT1351yuc25F9e3DQCaQGOjCRESCpayikp/OWcG2/Ud5YdwQeiQ03jVjfFWvMXczSwEGAUtreXikma02s3fNzPvdYUVEfOCc43eL1vH55v388Yf9GNUz3uuS/MLnee5m1hp4Dfi5c66wxsMrgK7OuSNmdgXwBtCrlq8xCZgEkJzcuHYtEZGmadqSbbyydCeTz+vB9UPDJ5d86rmbWXOqgn2Oc+71mo875wqdc0eqb78DNDezb738OeemOudSnXOpCQkJDSxdRKRhPlq/lz++k8llZ3XiV5f28bocv/JltowB04BM59xTJ2jTqbodZjas+use8GehIiL+tHZXAT+bt5L+ie14+vqBNGsWmlMeT8SXYZnvADcBGWa2qvpjDwLJAM65F4AxwB1mVg4cA8Y651wA6hURabDs/CImzFhO+5goXro5leio0J3yeCJ1hrtzbglw0pc059wUYIq/ihIRCZSCojImzFhOSVkFr/xkeEgtBlYfWjhMRJqMkvIKbpudxs4DRcyaOIxep7XxuqSAUbiLSJNQWen45YLVLNuWz7M3DGJE9w5elxRQWltGRJqEx9/bwD/X5PLA5WdwdQiu8lhfCncRCXuzvtrOi59t5aYRXbn93Ma//6k/KNxFJKx9sG4Pjyxax8VnnsYjV58Vsqs81pfCXUTC1sqdB6vmsneJ5W83DCIizOayn4zCXUTC0vb9R5k4M42ObVoy7ZbwnMt+Mgp3EQk7+4+UMP7vy3DOMfPWYcS3buF1SUGnqZAiElaOlJRz64zl7CksZs5PRtAtvpXXJXlC4S4iYaO0vJLJs9NZt7uQqTcNYUjX9l6X5BkNy4hIWKisdPxy4WqWZO3nsWv6c9GZp3ldkqcU7iIS8pxzPPrP9by1ejcPXH4G16YmeV2S5xTuIhLy/vvTLcz4cjsTz+nWZC5SqovCXURC2rxlO3ny/Y38YODpPHTFmU3mIqW6KNxFJGR9sG4PD/4jg3N7J/DEmAFht+FGQyjcRSQkLduWz91zq64+ff7Hg4mKVJwdT98NEQk5G/YUMnHmchLbR/P38UNp1UKzumtSuItISNl5oIibpy0jJiqCWbcOI65VlNclNUp6uRORkLGnoJgfT/ua0opK5k8aSZf2MV6X1Gip5y4iISH/aCnjpi0l/0gpMycMo0+n8N0izx/UcxeRRq+wuIxbpi8jO7+IGROGMSAp1uuSGj313EWkUTtWWsFPZqSRmVvI8+MGM7JHeO996i/quYtIo1VaXskdc9JZviOfZ8YO4sIzmvZ6MfVRZ8/dzJLM7BMzyzSzdWZ2Ty1tzMyeNbMsM1tjZoMDU66INBUVlY5fzF/Fpxv38ccf9G8Sm1r7ky8993Lgl865FWbWBkg3sw+dc+uPa3M50Kv6bTjwfPV7EZF6c87x4OsZvJ2Ry4NXnMGNw5O9Link1Nlzd87lOudWVN8+DGQCiTWajQZmuSpfA7Fm1tnv1YpI2HPO8Ye3M5mfls3dF/Zk0rk9vC4pJNXrhKqZpQCDgKU1HkoEso+7n8O3XwBEROr0zMebmbZkG+NHpXDvJb29Lidk+RzuZtYaeA34uXOusObDtXyKq+VrTDKzNDNL27dvX/0qFZGw9/ynW/jrR5v50eAuPHxVX63w2AA+hbuZNacq2Oc4516vpUkOcPzq+F2A3TUbOeemOudSnXOpCQkJp1KviISplz7fyuPvbeDqAafzxJiztcJjA/kyW8aAaUCmc+6pEzRbBNxcPWtmBFDgnMv1Y50iEsZmfbWdP7ydyeX9OvHUdQOIULA3mC+zZb4D3ARkmNmq6o89CCQDOOdeAN4BrgCygCJggv9LFZFwNHfZTh5+cx0Xn9mRZ8YOIjJC11b6Q53h7pxbQu1j6se3ccBP/VWUiDQNr6bn8OA/MjivdwLPaU12v9J3UkQ8sWj1bn716mpG9ejAizcNoUVkhNclhRWFu4gE3bsZufxi/ipSU+L4n5tTadlcwe5vCncRCaqP1u/l7rkrGdClHdPHDyUmSktcBYLCXUSC5pMNedw5ZwVnnd6WGbcOo7W2xwsYhbuIBMXHmXu5fXY6vTu1Zuatw2jbsrnXJYU1hbuIBNyH6/cy+eV0+nRqw5yJI4iN0b6ngaZwF5GAen/dHu6ck07fzm15+SfDaRejHnswaMBLRALmvbV7uOuVFfRLbMesiRqKCSaFu4gExLsZudw9dyX9u7TTGLsHNCwjIn739ppc7pq7kgFJscxSsHtCPXcR8au3Vu/m5/NXMSgpVtMdPaSeu4j4zZurdnHPvJUMSW6vYPeYwl1E/GLB8mx+Pn8VQ1Pi+PuEoQp2j+m7LyINNvPL7fxu0Tq+2yueqTelEh2ltWK8pnAXkQZ5/tMtPP7eBi7pexpTbhyk1R0bCYW7iJwS5xxPf7iJZxdn8f0Bp/PUdQNoro02Gg2Fu4jUm3OOP76dyUtLtnFdahf+65qztTVeI6NwF5F6qax0/ObNtbyydCfjR6Xw8FV9tZl1I6RwFxGflVdU8qtX1/D6yl3ccX4PfnVpH8wU7I2Rwl1EfFJaXsk981by7to93Pe93tx1YS+vS5KTULiLSJ2OlpQz+eV0Pt+8n99e1ZeJ53TzuiSpg8JdRE7q4NFSJsxYzpqcQzwx5myuS03yuiTxgcJdRE4ot+AYN01bxs78Ip4fN4RLz+rkdUniI4W7iNRqy74j3DxtGQXHypg5YRgje3TwuiSphzqvODCz6WaWZ2ZrT/D4+WZWYGarqt8e9n+ZIhJMGTkFXPvCV5SUVzBv0ggFewjypec+A5gCzDpJm8+dc1f5pSIR8dSXW/Zz28w0YmOiePknw+kW38rrkuQU1Nlzd859BuQHoRYR8dh7a3MZP305ie2jee2OUQr2EOavhSBGmtlqM3vXzM7y09cUkSCat2wnd85ZQb/Etiy4fSSd2rX0uiRpAH+cUF0BdHXOHTGzK4A3gFqvbjCzScAkgOTkZD88tYg0lHOOv360mWc+3sx5vRN4ftxgYqI01yLUNbjn7pwrdM4dqb79DtDczOJP0Haqcy7VOZeakJDQ0KcWkQYqq6jkP15bwzMfb+baIV146ZZUBXuYaPBP0cw6AXudc87MhlH1gnGgwZWJSEAdLSnnzjkr+Nemffzsol784uJeWicmjNQZ7mY2FzgfiDezHOB3QHMA59wLwBjgDjMrB44BY51zLmAVi0iD5R0u5tYZy8nMPcxj1/Rn7DANk4abOsPdOXdDHY9PoWqqpIiEgC37jnDL9GUcOFLKSzencsEZHb0uSQJAg2siTUja9nx+MiuNCDPmTRrBgKRYr0uSAFG4izQR763dwz3zVnJ6bDQzJgylawfNYQ9nCneRMOec46XPt/GndzMZmBTLtFuGEtcqyuuyJMAU7iJhrKyikoffXMfcZTu5vF8nnrpuINFREV6XJUGgcBcJUwXHyvjpnBUsydrPnef34L7v9dFep02Iwl0kDO08UMSEGVXrsGuDjaZJ4S4SZtK25zNpdjoVlY7ZE4czoruW622KFO4iYeTNVbu4f+EaEttHM+2WVLontPa6JPGIwl0kDDjneObjzfz1o80M6xbHi+OG0F4zYpo0hbtIiCsqLef+hWt4OyOXMUO68Kcf9icq0l+reUuoUriLhLCcg0VMmpVO5p5CHrj8DG4/t7sW/xJA4S4SspZuPcAdc1ZQVlHJ9PFDuaCP1oiR/6NwFwlBL3+9g0cWrSO5Qwz/c3MqPXTiVGpQuIuEkNLySh55ax2vLN3JBX0SeOaGQbRt2dzrsqQRUriLhIj9R0q48+UVLNuez+TzenD/pX2I0BWncgIKd5EQsHZXAbfPTmf/kRKeGTuQ0QMTvS5JGjmFu0gjtzAtm9+8sZa4VlEsnDySs7toDXapm8JdpJEqKa/g92+t55WlOxnZvQN/u3EQ8a1beF2WhAiFu0gjtOvQMe58OZ3VOQVMPq8H932vN5ERujBJfKdwF2lkvsjaz91zV1JaXskL4wZzWb/OXpckIUjhLtJIOOd4/l9b+PP7G+mR0JoXbhqi+etyyhTuIo1AYXEZ9y1YzQfr93LV2Z15/Edn06qF/jzl1Om3R8Rja3cVcNcrK8g+eIzfXtWXW7+TovVhpMHqPENjZtPNLM/M1p7gcTOzZ80sy8zWmNlg/5cpEn6cc8z8cjvX/PeXFJdVMm/SCCae003BLn7hS899BjAFmHWCxy8HelW/DQeer34vIidQcKyMB15bw7tr93BBnwT+ct1A4rT+uvhRneHunPvMzFJO0mQ0MMs554CvzSzWzDo753L9VKNIWFmdfYi75q5g96Fifn35Gdz23e7auFr8zh9j7olA9nH3c6o/pnAXOY5zjr9/sZ3/ejeTjm1asuD2kQzp2t7rsiRM+SPca+tyuFobmk0CJgEkJyf74alFQkNBURn3v1o1G+biM0/jz9eeTWyMhmEkcPwR7jlA0nH3uwC7a2vonJsKTAVITU2t9QVAJNws25bPL+avIu9wsWbDSND4I9wXAXeZ2TyqTqQWaLxdBMoqKnn2480890kWSXExLJw8ioFJWvRLgqPOcDezucD5QLyZ5QC/A5oDOOdeAN4BrgCygCJgQqCKFQkV2/cf5Z75q1idfYhrh3Thd1efRWtdlCRB5MtsmRvqeNwBP/VbRSIhzDnHq+k5PLJoHRHNjOduHMyVZ2ttGAk+dSVE/KSgqIwH/5HB2xm5DO8Wx9PXD+T02Givy5ImSuEu4gdfbTnAvQtWse9wCb+6rA+3n9tDW+CJpxTuIg1QXFbBE+9tZPoX2+gW34rX7xylnZKkUVC4i5yilTsP8suFq9m67yg3j+zKA5efQUyU/qSkcdBvokg9lZRX8OzHm3n+0y10atuSlycO55xe8V6XJfL/KNxF6mH97kLuXbCKDXsOc+2QLvz2+31p27K512WJfIvCXcQH5RWVvPCvLTzz8WZiY6KYdksqF515mtdliZyQwl2kDpm5hfzHa2tYk1PA9weczqNXn0V7Lc8rjZzCXeQEissqmLI4ixf+tYV20c11QZKEFIW7SC2Wb8/ngdfWsGXfUa4ZnMhvr+yr3rqEFIW7yHEOF5fxxHsbmf31DhJjo5l56zDO653gdVki9aZwF6m2eMNeHvrHWvYUFjPhOync970+tNJiXxKi9JsrTV5eYTF/eDuTRat306tja16dPEo7JEnIU7hLk1VR6Zj91Xb+8sEmSsorueeiXtx5QQ9aREZ4XZpIgyncpUlanX2Ih97IYO2uQr7bK55HR/ejW3wrr8sS8RuFuzQpBUVlPPnBBuYs3UlC6xb87YZBXHV2Z217J2FH4S5NgnOOf6zcxZ/eyST/aCnjR6Vw7yW9aaOlAyRMKdwl7K3bXcDv31rPsm35DEyKZcaEYfRLbOd1WSIBpXCXsHXgSAl//mAT85bvpH1MFH/6YX/GDk2imTbRkCZA4S5hp6yikllf7eCvH23iWGkFE0Z1456Le9EuWkMw0nQo3CWsfLoxj//853q27DvKub0TePiqM+nZsY3XZYkEncJdwsKWfUf449uZLN6QR7f4Vkwfn8oFfTpqFow0WQp3CWn7DpfwzMebmLssm+jmETx0xZncMiqFqMhmXpcm4imFu4SkoyXlvPT5NqZ+toWS8krGDU/m7ot6Ed+6hdeliTQKPoW7mV0GPANEAC855x6r8fh44ElgV/WHpjjnXvJjnSJA1Y5I89Oy+etHm9l3uIQr+nfi/kvP0NWlIjXUGe5mFgE8B1wC5ADLzWyRc259jabznXN3BaBGEZxzfLh+L4+/t4Et+44yNKU9L940hMHJWuBLpDa+9NyHAVnOua0AZjYPGA3UDHcRv3POsSRrP3/+YBOrsw/RPaEVU28awiV9T9PJUpGT8CXcE4Hs4+7nAMNrafcjMzsX2AT8wjmXXbOBmU0CJgEkJyfXv1ppUpZuPcBfPtzEsm35JMZG89g1/RkzpAuRETpZKlIXX8K9tu6Rq3H/LWCuc67EzCYDM4ELv/VJzk0FpgKkpqbW/BoiAKzceZCnPtzE55v307FNCx4dfRbXD03SUrwi9eBLuOcAScfd7wLsPr6Bc+7AcXf/B3i84aVJU7N2VwFPf7iJjzfkEdcqit9ceSbjRnSlZXOFukh9+RLuy4FeZtaNqtkwY4Ebj29gZp2dc7nVd68GMv1apYS19B35TFmcxScb99G2ZST3X9qH8aNStMWdSAPU+dfjnCs3s7uA96maCjndObfOzB4F0pxzi4CfmdnVQDmQD4wPYM0SBpxzfLnlAFMWZ/HV1gO0j2nOfd/rzc2jUmirZXhFGsyc82boOzU11aWlpXny3OId5xyLN+Qx5ZMsVu48RMc2LZh0bnduHJ5MTJR66iJ1MbN051xqXe301yRBUVZRyTsZubzwr61k5hbSpX00f/hBP8YM6aIxdZEAULhLQB0uLmPesmz+/sU2dhcU0z2hFX++dgCjB55Oc01pFAkYhbsExO5Dx/j7F9uYtyybwyXlDO8Wx3/+oB8X9OmozTJEgkDhLn6VkVPAS0u28s81VZOnruzfmdu+253+XbStnUgwKdylwUrKK3gnI5fZX+1gxc5DtG4RyYRRKUw4pxuJsdFelyfSJCnc5ZTtOnSMOV/vYP7ybA4cLaV7fCsevqovY1K7aDqjiMcU7lIv/17Ia9ZXO/g4cy8AF515GjeP7Mp3esRrPF2kkVC4i0/2FhbzanoOC9Oy2X6giLhWUdx+Xg9+PDyZLu1jvC5PRGpQuMsJlVVUsnhDHguWZ/PJxjwqHQzrFsfPLurFFf07a366SCOmcJdvyco7woK0bF5fkcP+I6V0bNOCyef14NrUJO14JBIiFO4CQN7hYt5ancubq3axJqeAyGbGhWd05PqhSZzXO0FrqIuEGIV7E3a4uIz31u7hzVW7+XLLfiod9Etsy2+uPJPRAxNJaKPNpkVClcK9iTlWWsG/Nu1j0epdfJSZR2l5JclxMdx1QU+uHphIz46tvS5RRPxA4d4EHC4uY/GGPN5bu4dPN+7jWFkFHVpFccPQJEYPSmRQUqz2IxUJMwr3MHXwaCkfZu7lvbV7WLJ5P6UVlSS0acGPhiRyeb/ODO8Wp3F0kTCmcA8Tzjk27T3C4g15fLIhj/SdB6modCTGRnPTyK5c3q8Tg5Pb6yIjkSZC4R7CjpVW8OWW/SzekMenG/ex69AxAPp2bssd5/Xg0rM60S+xrYZcRJoghXsIqah0rN1VwBdb9vNl1gGWb8+npLySmKgIzukZz90X9uT8Ph3p1K6l16WKiMcU7o2Yc46svCN8kbWfL7Yc4OutBzhcXA5An9Pa8OPhXbngjASGdYujRaSuFhWR/6Nwb0RKyytZu7uA9O0HSduRT/qOg+w/UgpAUlw0V/bvzKie8Yzs3kFz0EXkpBTuHso7XExGTgHpOw6Stv0gq3MOUVJeCUByXAzn9qrqlX+nZzxJcVqcS0R8p3APkrzDxazdVUBGTiEZuwpYu6uAPYXFAEQ2M85KbMe4EV1J7dqeIV3b07Gtxs1F5NQp3P2sqLSczXuPsGnvYTbnHWHjnsNs2FPI3sISAMyge3wrRnSPo3+XWPontqN/YjuiozRmLiL+41O4m9llwDNABPCSc+6xGo+3AGYBQ4ADwPXOue3+LbXxKK+oJLegmB0Hith+4Cg784vYkneEjXsPk3Pw2DftoiKb0TOhNaN6xNOvOsT7nt6W1i30mioigVVnyphZBPAccAmQAyw3s0XOufXHNZsIHHTO9TSzscDjwPWBKDgYjpVWsKewmNyCY+wpKK66faiYnflF7MwvIudgEWUV7pv2UZHNSOkQw8CkWK5PTaLXaW3ofVprunZoRYQuGhKp1Rsrd/Hk+xvZfegYp8dGc/+lffjBoESvywobvnQhhwFZzrmtAGY2DxgNHB/uo4FHqm+/CkwxM3POOTzinKOkvJKi0gqOlpRXvS8tp/BYGYeKysg/WsqholLyi0o5eLSMg0WlHDhSyp7CYgqOlX3r67VtGUlSXAx9O7flsn6d6BoXQ9cOrejaIYZObVvqyk+Renhj5S5+/XoGx8oqgKr9eH/9egaAAt5PfAn3RCD7uPs5wPATtXHOlZtZAdAB2O+PIo+3ZPN+nnh/A+UVjopKR3llZfV798374uogr6zjpcUM2sdEERvTnLiYKJI7xDC8exyd2rWkU9uW/+99TJSGUkT85cn3N34T7P92rKyCJ9/fqHD3E18Sq7Yuac3Y9KUNZjYJmASQnJzsw1N/W1RkMzq0iiKiWTMimxkREVb1vtn/vW/ZPIJWUZHEtIggpnkEMS0iq+5HRdCmZSTtW0URFxNF2+jmGjYR8cDuQ8fq9XGpP1/CPQdIOu5+F2D3CdrkmFkk0A7Ir/mFnHNTgakAqamppzRkM6xbHMO6DTuVTxWRRuL02Ohv1kKq+XHxD1/WfF0O9DKzbmYWBYwFFtVoswi4pfr2GGCxl+PtItK43X9pH6JrbLAe3TyC+y/t41FF4afOnnv1GPpdwPtUTYWc7pxbZ2aPAmnOuUXANGC2mWVR1WMfG8iiRSS0/XtcXbNlAse86mCnpqa6tLQ0T55bRCRUmVm6cy61rnbaikdEJAwp3EVEwpDCXUQkDCncRUTCkMJdRCQMKdxFRMKQwl1EJAwp3EVEwpBnFzGZ2T5gxyl+ejwBWHGykdMxNw065qahIcfc1TmXUFcjz8K9IcwszZcrtMKJjrlp0DE3DcE4Zg3LiIiEIYW7iEgYCtVwn+p1AR7QMTcNOuamIeDHHJJj7iIicnKh2nMXEZGTaNThbmaXmdlGM8syswdqebyFmc2vfnypmaUEv0r/8uGY7zWz9Wa2xsw+NrOuXtTpT3Ud83HtxpiZM7OQn1nhyzGb2XXVP+t1ZvZKsGv0Nx9+t5PN7BMzW1n9+32FF3X6i5lNN7M8M1t7gsfNzJ6t/n6sMbPBfi3AOdco36ja9WkL0B2IAlYDfWu0uRN4ofr2WGC+13UH4ZgvAGKqb9/RFI65ul0b4DPgayDV67qD8HPuBawE2lff7+h13UE45qnAHdW3+wLbva67gcd8LjAYWHuCx68A3gUMGAEs9efzN+ae+zAgyzm31TlXCswDRtdoMxqYWX37VeAiM7Mg1uhvdR6zc+4qDj7RAAACjElEQVQT51xR9d2vqdqwPJT58nMG+E/gCaA4mMUFiC/HfBvwnHPuIIBzLi/INfqbL8fsgLbVt9sBu4NYn9855z6jatvRExkNzHJVvgZizayzv56/MYd7IpB93P2c6o/V2sY5Vw4UAB2CUl1g+HLMx5tI1St/KKvzmM1sEJDknPtnMAsLIF9+zr2B3mb2hZl9bWaXBa26wPDlmB8BxplZDvAOcHdwSvNMff/e66XODbI9VFsPvObUHl/ahBKfj8fMxgGpwHkBrSjwTnrMZtYMeBoYH6yCgsCXn3MkVUMz51P139nnZtbPOXcowLUFii/HfAMwwzn3FzMbCcyuPubKwJfniYDmV2PuuecAScfd78K3/037po2ZRVL1r9zJ/g1q7Hw5ZszsYuAh4GrnXEmQaguUuo65DdAP+NTMtlM1NrkoxE+q+vq7/aZzrsw5tw3YSFXYhypfjnkisADAOfcV0JKqNVjClU9/76eqMYf7cqCXmXUzsyiqTpguqtFmEXBL9e0xwGJXfaYiRNV5zNVDFC9SFeyhPg4LdRyzc67AORfvnEtxzqVQdZ7haudcmjfl+oUvv9tvUHXyHDOLp2qYZmtQq/QvX455J3ARgJmdSVW47wtqlcG1CLi5etbMCKDAOZfrt6/u9RnlOs42XwFsouos+0PVH3uUqj9uqPrhLwSygGVAd69rDsIxfwTsBVZVvy3yuuZAH3ONtp8S4rNlfPw5G/AUsB7IAMZ6XXMQjrkv8AVVM2lWAd/zuuYGHu9cIBcoo6qXPhGYDEw+7mf8XPX3I8Pfv9e6QlVEJAw15mEZERE5RQp3EZEwpHAXEQlDCncRkTCkcBcRCUMKdxGRMKRwFxEJQwp3EZEw9L9SdaJTFFGrGgAAAABJRU5ErkJggg==\n", "text/plain": [ "
" ] }, "metadata": { "needs_background": "light" }, "output_type": "display_data" }, { "name": "stdout", "output_type": "stream", "text": [ "0.750375187593797\n" ] }, { "data": { "image/png": "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\n", "text/plain": [ "
" ] }, "metadata": { "needs_background": "light" }, "output_type": "display_data" }, { "name": "stdout", "output_type": "stream", "text": [ "0.8004001333331997\n" ] }, { "data": { "image/png": "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\n", "text/plain": [ "
" ] }, "metadata": { "needs_background": "light" }, "output_type": "display_data" }, { "name": "stdout", "output_type": "stream", "text": [ "0.8337500693400401\n" ] }, { "data": { "image/png": "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\n", "text/plain": [ "
" ] }, "metadata": { "needs_background": "light" }, "output_type": "display_data" }, { "name": "stdout", "output_type": "stream", "text": [ "0.8575714283567853\n" ] }, { "data": { "image/png": "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\n", "text/plain": [ "
" ] }, "metadata": { "needs_background": "light" }, "output_type": "display_data" }, { "name": "stdout", "output_type": "stream", "text": [ "0.8754374267547519\n" ] }, { "data": { "image/png": "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\n", "text/plain": [ "
" ] }, "metadata": { "needs_background": "light" }, "output_type": "display_data" }, { "name": "stdout", "output_type": "stream", "text": [ "0.8893331847402168\n" ] }, { "data": { "image/png": "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\n", "text/plain": [ "
" ] }, "metadata": { "needs_background": "light" }, "output_type": "display_data" }, { "name": "stdout", "output_type": "stream", "text": [ "0.7999987975959133\n" ] }, { "data": { "image/png": "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\n", "text/plain": [ "
" ] }, "metadata": { "needs_background": "light" }, "output_type": "display_data" }, { "name": "stdout", "output_type": "stream", "text": [ "0.8181804518136747\n" ] }, { "data": { "image/png": "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\n", "text/plain": [ "
" ] }, "metadata": { "needs_background": "light" }, "output_type": "display_data" }, { "name": "stdout", "output_type": "stream", "text": [ "0.7499999999875754\n" ] }, { "data": { "image/png": "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\n", "text/plain": [ "
" ] }, "metadata": { "needs_background": "light" }, "output_type": "display_data" }, { "name": "stdout", "output_type": "stream", "text": [ "0.7692307692137785\n" ] }, { "data": { "image/png": "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\n", "text/plain": [ "
" ] }, "metadata": { "needs_background": "light" }, "output_type": "display_data" }, { "name": "stdout", "output_type": "stream", "text": [ "0.7857142856917244\n" ] }, { "data": { "image/png": "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\n", "text/plain": [ "
" ] }, "metadata": { "needs_background": "light" }, "output_type": "display_data" }, { "name": "stdout", "output_type": "stream", "text": [ "0.7333333333422666\n" ] }, { "data": { "image/png": "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\n", "text/plain": [ "
" ] }, "metadata": { "needs_background": "light" }, "output_type": "display_data" }, { "name": "stdout", "output_type": "stream", "text": [ "0.75000000001142\n" ] }, { "data": { "image/png": "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\n", "text/plain": [ "
" ] }, "metadata": { "needs_background": "light" }, "output_type": "display_data" }, { "name": "stdout", "output_type": "stream", "text": [ "0.7647058823672719\n" ] }, { "data": { "image/png": "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\n", "text/plain": [ "
" ] }, "metadata": { "needs_background": "light" }, "output_type": "display_data" }, { "name": "stdout", "output_type": "stream", "text": [ "0.777777777795477\n" ] }, { "data": { "image/png": "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\n", "text/plain": [ "
" ] }, "metadata": { "needs_background": "light" }, "output_type": "display_data" }, { "name": "stdout", "output_type": "stream", "text": [ "0.7368421052631586\n" ] }, { "data": { "image/png": "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\n", "text/plain": [ "
" ] }, "metadata": { "needs_background": "light" }, "output_type": "display_data" }, { "name": "stdout", "output_type": "stream", "text": [ "0.7500000000000011\n" ] }, { "data": { "image/png": "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\n", "text/plain": [ "
" ] }, "metadata": { "needs_background": "light" }, "output_type": "display_data" }, { "name": "stdout", "output_type": "stream", "text": [ "0.7619047619047634\n" ] }, { "data": { "image/png": "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\n", "text/plain": [ "
" ] }, "metadata": { "needs_background": "light" }, "output_type": "display_data" }, { "name": "stdout", "output_type": "stream", "text": [ "0.7727272727272746\n" ] }, { "data": { "image/png": "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\n", "text/plain": [ "
" ] }, "metadata": { "needs_background": "light" }, "output_type": "display_data" }, { "name": "stdout", "output_type": "stream", "text": [ "0.7826086956521764\n" ] }, { "data": { "image/png": "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\n", "text/plain": [ "
" ] }, "metadata": { "needs_background": "light" }, "output_type": "display_data" }, { "name": "stdout", "output_type": "stream", "text": [ "0.7499999999999992\n" ] }, { "data": { "image/png": "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\n", "text/plain": [ "
" ] }, "metadata": { "needs_background": "light" }, "output_type": "display_data" }, { "name": "stdout", "output_type": "stream", "text": [ "0.7599999999999992\n" ] }, { "data": { "image/png": "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\n", "text/plain": [ "
" ] }, "metadata": { "needs_background": "light" }, "output_type": "display_data" }, { "name": "stdout", "output_type": "stream", "text": [ "0.7692307692307683\n" ] }, { "data": { "image/png": "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\n", "text/plain": [ "
" ] }, "metadata": { "needs_background": "light" }, "output_type": "display_data" }, { "name": "stdout", "output_type": "stream", "text": [ "0.7777777777777767\n" ] }, { "data": { "image/png": "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\n", "text/plain": [ "
" ] }, "metadata": { "needs_background": "light" }, "output_type": "display_data" }, { "name": "stdout", "output_type": "stream", "text": [ "0.7857142857142841\n" ] }, { "data": { "image/png": "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\n", "text/plain": [ "
" ] }, "metadata": { "needs_background": "light" }, "output_type": "display_data" }, { "name": "stdout", "output_type": "stream", "text": [ "0.7586206896551725\n" ] }, { "data": { "image/png": "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\n", "text/plain": [ "
" ] }, "metadata": { "needs_background": "light" }, "output_type": "display_data" }, { "name": "stdout", "output_type": "stream", "text": [ "0.7666666666666666\n" ] }, { "data": { "image/png": "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\n", "text/plain": [ "
" ] }, "metadata": { "needs_background": "light" }, "output_type": "display_data" }, { "name": "stdout", "output_type": "stream", "text": [ "0.7741935483870968\n" ] }, { "data": { "image/png": "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\n", "text/plain": [ "
" ] }, "metadata": { "needs_background": "light" }, "output_type": "display_data" }, { "name": "stdout", "output_type": "stream", "text": [ "0.7812499999999999\n" ] }, { "data": { "image/png": "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\n", "text/plain": [ "
" ] }, "metadata": { "needs_background": "light" }, "output_type": "display_data" }, { "name": "stdout", "output_type": "stream", "text": [ "0.787878787878788\n" ] }, { "data": { "image/png": "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\n", "text/plain": [ "
" ] }, "metadata": { "needs_background": "light" }, "output_type": "display_data" }, { "name": "stdout", "output_type": "stream", "text": [ "0.7647058823529412\n" ] }, { "data": { "image/png": "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\n", "text/plain": [ "
" ] }, "metadata": { "needs_background": "light" }, "output_type": "display_data" }, { "name": "stdout", "output_type": "stream", "text": [ "0.7714285714285715\n" ] }, { "data": { "image/png": "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\n", "text/plain": [ "
" ] }, "metadata": { "needs_background": "light" }, "output_type": "display_data" }, { "name": "stdout", "output_type": "stream", "text": [ "0.7777777777777778\n" ] }, { "data": { "image/png": "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\n", "text/plain": [ "
" ] }, "metadata": { "needs_background": "light" }, "output_type": "display_data" }, { "name": "stdout", "output_type": "stream", "text": [ "0.7837837837837837\n" ] }, { "data": { "image/png": "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\n", "text/plain": [ "
" ] }, "metadata": { "needs_background": "light" }, "output_type": "display_data" }, { "name": "stdout", "output_type": "stream", "text": [ "0.7894736842105262\n" ] }, { "data": { "image/png": "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\n", "text/plain": [ "
" ] }, "metadata": { "needs_background": "light" }, "output_type": "display_data" }, { "name": "stdout", "output_type": "stream", "text": [ "0.7948717948717948\n" ] }, { "data": { "image/png": "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\n", "text/plain": [ "
" ] }, "metadata": { "needs_background": "light" }, "output_type": "display_data" }, { "name": "stdout", "output_type": "stream", "text": [ "0.7750000000000001\n" ] }, { "data": { "image/png": "iVBORw0KGgoAAAANSUhEUgAAAW4AAAD8CAYAAABXe05zAAAABHNCSVQICAgIfAhkiAAAAAlwSFlzAAALEgAACxIB0t1+/AAAADl0RVh0U29mdHdhcmUAbWF0cGxvdGxpYiB2ZXJzaW9uIDMuMC4zLCBodHRwOi8vbWF0cGxvdGxpYi5vcmcvnQurowAAIABJREFUeJzt3Xl0XGl95vHvr0qlfV9sWZYs2W4vbbu9tdt0083WQHfTEBpmWBMISycOCcmQkxlmwnAmJyGTSTKEAJMhgIedgRDosDQMdLP03vQmb23J+25JliVZu2St9c4fVTJuuySV7Kq6dauezzk61nJV9VxJfvTqve+915xziIiIfwS8DiAiIguj4hYR8RkVt4iIz6i4RUR8RsUtIuIzKm4REZ9RcYuI+IyKW0TEZ1TcIiI+k5OMB62urnZNTU3JeGgRkYy0a9euHudcTTzbJqW4m5qaaG5uTsZDi4hkJDM7He+2mioREfEZFbeIiM+ouEVEfEbFLSLiMypuERGfUXGLiPiMiltExGeSso5bRLLTvrP9PH28h8Ul+bxx4xLyQ0GvI2UkFbeIXLdw2PH3Dx/ii4+fuPS+zz12jG98cDv1FYUeJstMmioRkev2z48d44uPn+C3X7aMfX9xF1//4Ha6h8b5wFdfYGR8yut4GUfFLSLXZd/Zfj71iyPct7mOv3nLBsoKQ7xqdQ2f/52bOdo1zGd/ddTriBlHxS0i18w5x1882Ep1cR7//S0bMLNLH7tjVTXvuqWBLz91klM9Ix6mzDwqbhG5Zo8e7mLf2X4+etcaSvJDV338z16/mmDA2PnkiRifLdcqruI2s3Ize8DMDpnZQTO7LdnBRCT9ff6x49SV5fPWrUtjfnxRaT5vu7meB5rb6BuZSHG6zBXviPuzwEPOubXAJuBg8iKJiB/sOdPHC6f6+L1XrCAUnL1K3vOyRiamw/z4xY4Upsts8xa3mZUCrwS+DOCcm3DO9Sc7mIikt+82n6UgFOQdtzTMud26ulLWLSnlgV1tKUqW+eIZca8AuoGvmtkeM/uSmRVduZGZ7TCzZjNr7u7uTnhQEUkfFyem+fG+c7zhplqK8+Y/HeRtN9fzYtsAR88PpSBd5ounuHOArcDnnXNbgBHgz6/cyDm30zm3zTm3raYmrrvviIhPPdzayfD4FG+/ee7R9ow3bVwCwM8PnE9mrKwRT3G3AW3Oueeibz9ApMhFJEv9v/3nqC3N52XLK+PaflFpPpsayvmFijsh5i1u51wncNbM1kTf9VrgQFJTiUjaGp2Y4okj3dyzoZZAwOb/hKjX37iIvWf76RoaS2K67BDvqpI/Ab5lZi8Cm4H/kbxIIpLOHj/czfhUmLvX1y7o815742IAHjnYlYxYWSWu4nbO7Y3OX290zr3FOdeX7GAikp4eau2ksiiXW5oqFvR5a2tLWFSSx9PHLyQpWfbQmZMiErfJ6TCPHOzidTcuImeOtduxmBkvX1nFM8d7cM4lKWF2UHGLSNz2nu1naHyKO9cuuqbPf/nKanqGJzjaNZzgZNlFxS0icXvySDcBg9tWVl/T59+2sgqAXx/rSWSsrKPiFpG4PXmsh00N5ZQVXH1BqXg0VBZSX1HAMyc0z309VNwiEpeB0Un2ne3nFauu7wS7bY0V7D7Tr3nu66DiFpG4PHOih7CDV6y6tmmSGVsbK+geGqe9/2KCkmUfFbeIxOWJoz0U5+WwuaH8uh5nS0NkGeGeM7pW3bVScYtIXH59rIdbV1TOeQnXeKxdUkJ+KKDivg4qbhGZV9fQGKcujLI9zmuTzCUUDLBxaTm7z+g8vmul4haReTWfipTsLU3XX9wAWxrLOdAxyPjUdEIeL9uouEVkXs+f7CU/FGDD0rKEPN6WhgompsO0dgwm5PGyjYpbROb1wqletjRUXPf89owNS0sBVNzXSMUtInMaGpvk4LlBbknA/PaMpeUFlBWEONAxkLDHzCYqbhGZ0+4z/YQdC74a4FzMjA1LSzXivkYqbhGZ0wsnewkGjK3LElfcAOvryjh0bojJ6XBCHzcbqLhFZE4vnOplfV0pRXHcFHgh1teVMjEd5piuFLhgKm4RmdXkdJh9bf3c3JjY0TZEiht0gPJaqLhFZFaHO4cYmwyzJcHTJADLq4spCAVpadcByoVScYvIrPa1RU5L31x/fdcniSUYMNYuKeHAOY24F0rFLSKz2ne2n4rCEA2VBUl5/LW1JRw9P6RLvC6QiltEZrXv7ACbGsoxs6Q8/urFJfSNTtI9PJ6Ux89UKm4RiWl4fIojXUPXfRnXuaxZXALAkU6tLFmIuIrbzE6Z2X4z22tmzckOJSLe2982gHOwKYnFvbo2Wtznh5L2HJloIQszX+Oc0x0+RbLEzIHJTUk4MDmjujiPyqJcFfcCaapERGLad7afZZWFVBblJvV5Vi8u5rCKe0HiLW4H/NzMdpnZjmQGEpH0sO9sf1KnSWasWVzCkU6tLFmIeIv7dufcVuANwIfN7JVXbmBmO8ys2cyau7u7ExpSRFKra3CMjoExNtUn5vrbc1ldW8LIxLRuHrwAcRW3c64j+m8X8ANge4xtdjrntjnnttXU1CQ2pYik1N6zkfntLctSM+IGOHpeK0viNW9xm1mRmZXMvA7cBbQkO5iIeGdfWz/BgLG+Lvkj7lXR4j7UqXnueMWzqmQx8IPoAvwc4NvOuYeSmkpEPPVi2wBrFpeQHwom/bnKCkIsLs3TVQIXYN7ids6dADalIIuIpAHnHK0dg7zuxkUpe84V1cWc6FFxx0vLAUXkJToHx+gdmUjJNMmMlYuKON41rJUlcVJxi8hLtLRHrtY3c73sVFhZU8zg2BQ9wxMpe04/U3GLyEu0dgxgBjcuSV1xr6gpBuBEt6ZL4qHiFpGXaO0YZHl1UcJvVTaXlTVFABzvHknZc/qZiltEXqK1fSCl89sAdWUF5IcCGnHHScUtIpf0jUzQMTDGhhTObwMEAsby6mKOq7jjouIWkUtmbtyb6hE3RKZLNFUSHxW3iFzS0hG5cW8qV5TMWFFTTFvfKGOT0yl/br9RcYvIJa0dgywtL6AiyZdyjWVlTRFhB6cvjKb8uf1GxS0il7R2DLDOg9E2RNZyA5rnjoOKW0QAGBmf4mTPiCfTJAAroksCtbJkfipuEQHg4LlBnIMNHhyYBCjMzaGuLF8HKOOg4hYR4LIVJUu9GXEDLK8p4mSPins+Km4RASLz25VFudSW5nuWobGqiFMXVNzzUXGLCBC5uNT6ulKi1973xPKqIvpHJ+kf1cWm5qLiFhEmpsIc7Rry5MSbyzVWFQJaEjgfFbeIcOT8EJPTzrMVJTOWV0dWlmi6ZG4qbhGhNXrG5Ial3o64GyoLMYNTPRpxz0XFLSK0dgxSnJdDY2WhpznyQ0GWlOZrxD0PFbeI0NoxyI1LSggEvDswOaOpWitL5qPiFsly02HHgY5Bzw9MzmisKuKU1nLPScUtkuVO9oxwcXLa8wOTM5ZXF9I3OsnA6KTXUdJW3MVtZkEz22NmP0lmIBFJrdZLl3JNnxE3wOlejbpns5AR90eAg8kKIiLeONAxSG4wwKrFxV5HAX6zJFCnvs8uruI2s3rgjcCXkhtHRFKtpWOANbUlhILpMXO6rFIn4cwn3u/UZ4D/DISTmEVEUsw5R2vHYNrMb0NkSWBdWb4OUM5h3uI2szcBXc65XfNst8PMms2subu7O2EBRSR5OgbG6B+dTKviBl1saj7xjLhvB95sZqeA7wB3mtn/vXIj59xO59w259y2mpqaBMcUkWRoaY8emPT4jMkrRdZya6pkNvMWt3PuY865eudcE/Au4BHn3HuSnkxEkq61Y5CAwY216TXibqoqpHdkgoGLWhIYS3ocjRARTxzoGGBFTTEFuUGvo7zEpSWBmi6JaUHF7Zx7zDn3pmSFEZHUamkfZEOazW/D5VcJ1HRJLBpxi2SpC8PjdA6Opc2JN5e7tCRQK0tiUnGLZKl0uMfkbApygywpy+ekpkpiUnGLZKmWmVPdl6TfiBsid8PRWu7YVNwiWaq1Y5CGygLKCkNeR4mpqaqIM72a445FxS2SpVrbB9J2tA2RlSU9wxMMj095HSXtqLhFstDQ2CSnLoym3RmTl2u6dONgTZdcScUtkoUORA9Men2Pybn8Zi23pkuupOIWyUKXVpSk8Yi7MTri1jVLrqbiFslCLR0D1JTksag03+sosyrKy6G6OI/TuuP7VVTcIlnoQJpdynU2TVWFGnHHoOIWyTJjk9Mc7RpmQxqeMXmlxqoizXHHoOIWyTKHO4eYDjvfjLg7B8cYm5z2OkpaUXGLZJlWH6womdEYvdiUTsR5KRW3SJZp6RigND+H+ooCr6PMa2Ytt059fykVt0iWidxjsgwz8zrKvBortZY7FhW3SBaZmg5z6Jw/VpQAlBWGKC8MaWXJFVTcIlnkePcI41PhtLyU62y0suRqKm6RLDJzc2A/LAWcobXcV1Nxi2SR1o5B8kMBVtQUex0lbo1VRXT0X2RiKux1lLSh4hbJIi0dA9y4pJRgIP0PTM5oqiok7KCtT9MlM1TcIlkiHHYc9Mmp7pfTVQKvpuIWyRKnLowwND7lq/lt0FUCY5m3uM0s38yeN7N9ZtZqZn+VimAiklj7owcmN9aXe5xkYaqKcinOy9GI+zI5cWwzDtzpnBs2sxDwlJn9zDn3bJKziUgC7Ts7QF5OgNWL/XNgEsDMIjcO1oj7knlH3C5iOPpmKPrikppKRBJuf3s/6+tKyQn6b4a0SWu5XyKu76CZBc1sL9AF/MI591xyY4lIIk1Nh2lpH/TdNMmMxqpCzvaOMjWtJYEQZ3E756adc5uBemC7mW24chsz22FmzWbW3N3dneicInIdjnePcHFymo31/jowOaOpqoipsOPcwJjXUdLCgv5mcs71A48B98T42E7n3Dbn3LaampoExRORRHixrR/w34HJGcu0suQl4llVUmNm5dHXC4DXAYeSHUxEEufFtgGK83JYEb2+td80Rddyn9I8NxDfqpIlwNfNLEik6L/rnPtJcmOJSCK92D7AhqWlBHx0xuTlFpXkkR8KcFrX5QbiKG7n3IvAlhRkEZEkmJgKc7BjkA/c3uR1lGsWCBiNlUUacUf5b12QiCzIkfNDTEyHucmnByZnNFYVclpz3ICKWyTj7YsemNzk0wOTM5qqizjdO0o4rNNIVNwiGW5/2wAVhSFf3GNyLssqC5mYCnN+SEsCVdwiGW5f2wA31Zf74h6Tc7m0sqRH89wqbpEMNjY5zZHzQ2xc6u/5bfjNVQI1z63iFslo+9sHmA47Njf4e34boK68gFDQtLIEFbdIRtt9ug+ALcv8X9zBgNFQqZUloOIWyWi7z/TRWFVIVXGe11ESoqlKa7lBxS2SsZxz7D7Tz9ZlFV5HSZiZtdzOZfeSQBW3SIZq779I99A4WzNgmmRGY2UhoxPT9AxPeB3FUypukQy1+0zkxJstmTTirp65cXB2z3OruEUy1O7TfRSEgqytLfE6SsLoKoERKm6RDLXnbD8b68t8eauy2SwtLyAYMI24vQ4gIok3NjnNgY4BtjZmzjQJQG5OgKXlBRpxex1ARBKvpX2AyWnHlgw48eZKukqgilskI+0+EznxJtNG3BAp7lNZfkMFFbdIBtpzpp9llYVUZ8iJN5drqipicGyK/tHsXRKo4hbJMM45mk/3ZdT67cs1amWJilsk05y6MEr30Djbl1d5HSUpmnSVQBW3SKZ5/uQFALYvr/Q4SXI0VBZiBse7VdwikiGeO9lLVVEuK2uKvI6SFPmhIA0VhZzoHvY6imdU3CIZ5vmTvWxfXun7O97MZWVNkUbcczGzBjN71MwOmlmrmX0kFcFEZOHa+y/S1ncxY6dJZqysKeZE93DW3jg4nhH3FPAfnXM3ArcCHzazdcmNJSLX4oWTvUDmzm/PWLmomPGpMO39F72O4ol5i9s5d845tzv6+hBwEFia7GAisnDPneylJD+HtbWlXkdJqpU1xQAcz9J57gXNcZtZE7AFeC4ZYUTk+jx/8gK3NFUSDGTu/DZw6cBrts5zx13cZlYM/Bvwp865wRgf32FmzWbW3N3dnciMIhKHnuFxjnePZPw0CUBlUS7lhSGNuOdiZiEipf0t59z3Y23jnNvpnNvmnNtWU1OTyIwiEodnT2T2+u3LmRk31BRzrEvFHZNF1hR9GTjonPvH5EcSkWvx9LEeSvJy2Li0zOsoKTGzsiQbxTPivh14L3Cnme2Nvtyb5FwiskBPHevh1pVVGXXjhLmsXFREz/BEVl5sKme+DZxzTwGZfaRDxOfOXBjlbO9Ffu+OFV5HSZnfrCwZ4ebGXI/TpFZ2/GoWyXBPHesB4PYbqj1OkjrZvCRQxS2SAZ4+1kNtaX7GXp8klvqKAnKDARW3iPhPOOx4+ngPt99QndHXJ7lSTjBAU3Uhx7uyby23ilvE5w6cG6R/dJI7VmXm9bfnsrKmWCNuEfGfx49ETni7fWX2zG/PWLW4hNMXRhibnPY6SkqpuEV87tFDXayvK2VRab7XUVJubW0JYQdHz2fXqFvFLeJjfSMT7D7Tx51rF3kdxRNraksAONR51VU4MpqKW8THnjjaTdjBa7K0uJuqisjLCXC4c8jrKCml4hbxsUcOdVFZlMum+sy8o/t8ggFj9eISDp9XcYuID0yHHY8f6ebVq2sy/jKuc1lTW8LBcypuEfGBvWf76B+dzNppkhlra0voGR7nwvC411FSRsUt4lO/OthFMGC8clV2X0Z55gBlNs1zq7hFfMg5x89aOrltRRVlhSGv43jqNytLVNwiksYOnx/iZM8I92yo9TqK52qK86gsytWIW0TS28/2d2IGd61f7HUUz5kZaxaXcCiLVpaouEV86KGWTm5pqmRRSfadLRnL2iUlHOkcIhx2XkdJCRW3iM8c7x7m8Pkh3qBpkkvWLSnl4uQ0J3qy40qBKm4Rn3mopRNA89uX2Rg9AWl/e7/HSVJDxS3iI845HtzbwdZl5SwpK/A6TtpYWVNEQSjI/rbsuGaJilvERw6cG+Tw+SHeurXe6yhpJScYYF1dqUbcIpJ+vr+7nVDQeNNNS7yOknZuWlpGa8cg01lwgFLFLeITU9NhfrS3gzvXLqKiKLvuah6Pm5aWMToxzYksuCPOvMVtZl8xsy4za0lFIBGJ7cljPfQMj/PWLZomiWVjfRkA+9sHPE6SfPGMuL8G3JPkHCIyjx/sbqe8MMRr1mb3tUlms6KmmMLcIC+2qbhxzj0B9KYgi4jMondkgodaOrlvUx15OUGv46SlYMBYX1eqEbeIpIfvNZ9lYjrM79za6HWUtLa5oZz97QOMT2X2zYMTVtxmtsPMms2subu7O1EPK5L1wmHHt547w/bllaxeXOJ1nLR2c2MlE1NhWtozez13worbObfTObfNObetpkZzcCKJ8sTRbs70jvIejbbndXNjBQDNpzJ7dldTJSJp7uu/PkV1cS73rNcp7vOpKcmjqaqQ5tN9XkdJqniWA/4L8AywxszazOz+5McSEYjc1eXRw92877YmcnM0zorHzY2V7Drdh3OZeyJOPKtK3u2cW+KcCznn6p1zX05FMBGBnU+coCAU5L23aZokXtuaKugdmcjoKwXqV7hImjo3cJEf7W3nnbc0UF6oMyXjdUtTZJ77+ZOZO8+t4hZJU198/AQOuP+O5V5H8ZWVNcUsLs3j6WM9XkdJGhW3SBpq77/It587wzu21dNQWeh1HF8xM26/oZpfH7+QsXfEUXGLpKF/+tVRAP74zlUeJ/GnO26opndkggPnMnM9t4pbJM2c6B7me7va+O2XLWNpuW6WcC3uuKEagKcydLpExS2SZv76JwcoCAX58Gtu8DqKby0qzWf14mKeOqriFpEke+TQeR493M1HXruKmpI8r+P42qtW1/D8yV6Gxia9jpJwKm6RNDE2Oc0nfnyAFTVFvO/lTV7H8b271tcyMR3m0cOZd+0kFbdImvj0L45w6sIof/Xm9TpLMgG2LqugujiPh1s6vY6ScPrpEEkDu073sfPJE7x7ewOvWKWLtCVCMGC8ft1iHj3cxdhkZl3mVcUt4rGhsUn+0/f2UVdWwMffuM7rOBnlng21jE5M82SGHaRUcYt4yDnHf/m3FznTO8qn3rGJ4rwcryNllNtWVFFZlMsP9rR5HSWhVNwiHvryUyf56f5OPnr3Gm5dUeV1nIyTmxPgvs11/PJAF30jE17HSRgVt4hHHmrp5G9+epC71y/mD165wus4GevtNzcwMR3mR3vbvY6SMCpuEQ88f7KX//CdPWyqL+cz79yCmXkdKWOtqytlY30Z33j2dMZcu0TFLZJizxy/wPu/+jz15QV85f23UJCru7Yn2/13LOdE9wiPHOryOkpCqLhFUugXB87z/q8+z9LyAr6z41Yqi3Sd7VS496Yl1JXl84XHj2fEnXFU3CIp4Jzjfz9ylB3fbGZNbQnf2XEri0rzvY6VNULBAB969UqaT/dlxKhbxS2SZF1DY/z+N5r5h58f4c2b6vjuH9xGVbGuQ5Jq796+jOXVRfzdzw4xOR32Os51UXGLJEk47HhgVxt3ffoJnjzaw3970zo+887N5Ic0p+2FUDDAx96wlqNdw/zzo8e9jnNdtNpfJMGcczx1rIe/f+gQLe2DbG4o5x/evokbFhV7HS3r3bW+ljdvquOfHjnKK1ZXs3VZhdeRromKWyRBRiem+On+Tr781EkOnhtkaXkBn33XZn5rYx2BgJb7pYtP3LeePWf72PGNZn7wR7f78tZwFs8RVjO7B/gsEAS+5Jz7u7m237Ztm2tubk5MQpE0NnBxkmeOX+Cn+8/xy4PnGZ2YZs3iEu5/xXLu21xHXo6mRdLRsa4h/v3nnyE/FOBrH9jOjUtKvY6Eme1yzm2La9v5itvMgsAR4PVAG/AC8G7n3IHZPkfFLZlofGqa410jHOoc5EDHIM+d7KWlYwDnoKIwxL03LeG3NtXxsuWVWX1CzQ/3tPPJhw/T0X+RuvICPnr3Gt6yZanXsa5yqHOQ933lefpGJ/nT163i/S9vojDXu0mIRBf3bcBfOufujr79MQDn3N/O9jkqbklnzjkmpsNMTIUZj75cnJhi4OIk/aOTl/7tHZmgo/8i7dGXcwNjTEfPvMvNCbC5oZyXr6zithVVbG2sIBTUsf4f7mnnY9/fz8XLLqNaEAryt//uprQs7+6hcf7rD/bziwPnqSgM8aaNdbxydQ1ra0tYWl6Q0imuhRR3PL9elgJnL3u7DXjZtQSbzzu+8AxjUy+9bm6s3yuO2L9sYm47y++lWO9eyML86861gOeP+d7r3K/Z9jRZX+/Yn5/4XLM9f9i5SyU9MRXfUrBgwKgtzaeuPJ9tjRXUVxSypraEG5eU0FRVRI6K+iqffPjwS0ob4OLkNJ98+HBaFndNSR7/53e3set0L1968iQP7Grjm8+eBiBgUFYQorQgRE7ACAaMgEX+DQaMWJVeVpjLNz64Pem54ynuWPmu+u9hZjuAHQDLli27pjAVRSEmpq6eE4z1Z+dsvwdj/4Uae+tY28bacra/emN962bdNuZzxRlg1lyz7Ffczz/Lcy3g6x17FxbwdYn5/LNtG9/jxn6fkRsMkBcKkJcTJC8ncNlLkPzcIGUFIcoLQpQXhigrCFGSHyKog4oL0tF/cUHvTxc3N1Zyc2MlY5PTtHYMcLhzmI7+iwxcjPwFNh12kRfnCEf/jaWsIJSSvPEUdxvQcNnb9UDHlRs553YCOyEyVXItYb743rj+ShCRNFVXXkB7jJKuKy/wIM3C5YeCl0o8ncXzt94LwCozW25mucC7gAeTG0tE/Oijd6+h4IoTjApCQT569xqPEmWmeUfczrkpM/tj4GEiywG/4pxrTXoyEfGdmXlsP6wq8bO41nEvlFaViIgszEJWleiwuIiIz6i4RUR8RsUtIuIzKm4REZ9RcYuI+IyKW0TEZ1TcIiI+o+IWEfGZpJyAY2bdwOlr/PRqoCeBcfxA+5z5sm1/Qfu8UI3OuZp4NkxKcV8PM2uO9+yhTKF9znzZtr+gfU4mTZWIiPiMiltExGfSsbh3eh3AA9rnzJdt+wva56RJuzluERGZWzqOuEVEZA6eFbeZ3WNmh83smJn9eYyP55nZv0Y//pyZNaU+ZeLEsb9/ZmYHzOxFM/uVmTV6kTOR5tvny7Z7m5k5M/P9CoR49tnM3hH9Xrea2bdTnTHR4vjZXmZmj5rZnujP971e5EwUM/uKmXWZWcssHzcz+1/Rr8eLZrY14SGccyl/IXInnePACiAX2Aesu2KbPwK+EH39XcC/epE1hfv7GqAw+vof+nl/493n6HYlwBPAs8A2r3On4Pu8CtgDVETfXuR17hTs807gD6OvrwNOeZ37Ovf5lcBWoGWWj98L/IzIvbBvBZ5LdAavRtzbgWPOuRPOuQngO8B9V2xzH/D16OsPAK+12W5rnv7m3V/n3KPOudHom88SuSmzn8XzPQb4a+B/AmOpDJck8ezz7wOfc871ATjnulKcMdHi2WcHlEZfLyPGzcb9xDn3BNA7xyb3Ad9wEc8C5Wa2JJEZvCrupcDZy95ui74v5jbOuSlgAKhKSbrEi2d/L3c/kd/YfjbvPpvZFqDBOfeTVAZLoni+z6uB1Wb2tJk9a2b3pCxdcsSzz38JvMfM2oCfAn+SmmieWej/9wWb92bBSRJr5Hzl8pZ4tvGLuPfFzN4DbANeldREyTfnPptZAPg08P5UBUqBeL7POUSmS15N5K+qJ81sg3OuP8nZkiWefX438DXn3KfM7Dbgm9F9Dic/nieS3l1ejbjbgIbL3q7n6j+fLm1jZjlE/sSa68+TdBbP/mJmrwM+DrzZOTeeomzJMt8+lwAbgMfM7BSRucAHfX6AMt6f6x855yadcyeBw0SK3K/i2ef7ge8COOeeAfKJXNMjU8X1//16eFXcLwCrzGy5meUSOfj44BXbPAi8L/r624BHXHTm34fm3d/otMEXiZS23+c9YZ59ds4NOOeqnXNNzrkmIvP6b3bONXsTNyHi+bn+IZED0ZhZNZGpkxMpTZlY8ezzGeC1AGZ2I5Hi7k5pytR6EPjd6OpZdFeaAAAAs0lEQVSSW4EB59y5hD6Dh0dm7wWOEDki/fHo+z5B5D8vRL653wOOAc8DK7w+mpzk/f0lcB7YG3150OvMyd7nK7Z9DJ+vKonz+2zAPwIHgP3Au7zOnIJ9Xgc8TWTFyV7gLq8zX+f+/gtwDpgkMrq+H/gQ8KHLvsefi3499ifj51pnToqI+IzOnBQR8RkVt4iIz6i4RUR8RsUtIuIzKm4REZ9RcYuI+IyKW0TEZ1TcIiI+8/8BmIjPGcgXmQgAAAAASUVORK5CYII=\n", "text/plain": [ "
" ] }, "metadata": { "needs_background": "light" }, "output_type": "display_data" }, { "name": "stdout", "output_type": "stream", "text": [ "0.7804878048780487\n" ] }, { "data": { "image/png": "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\n", "text/plain": [ "
" ] }, "metadata": { "needs_background": "light" }, "output_type": "display_data" }, { "name": "stdout", "output_type": "stream", "text": [ "0.7857142857142857\n" ] }, { "data": { "image/png": "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\n", "text/plain": [ "
" ] }, "metadata": { "needs_background": "light" }, "output_type": "display_data" }, { "name": "stdout", "output_type": "stream", "text": [ "0.7906976744186047\n" ] }, { "data": { "image/png": "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\n", "text/plain": [ "
" ] }, "metadata": { "needs_background": "light" }, "output_type": "display_data" }, { "name": "stdout", "output_type": "stream", "text": [ "0.7954545454545455\n" ] }, { "data": { "image/png": "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\n", "text/plain": [ "
" ] }, "metadata": { "needs_background": "light" }, "output_type": "display_data" }, { "name": "stdout", "output_type": "stream", "text": [ "0.7777777777777777\n" ] }, { "data": { "image/png": "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\n", "text/plain": [ "
" ] }, "metadata": { "needs_background": "light" }, "output_type": "display_data" }, { "name": "stdout", "output_type": "stream", "text": [ "0.7826086956521741\n" ] }, { "data": { "image/png": "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\n", "text/plain": [ "
" ] }, "metadata": { "needs_background": "light" }, "output_type": "display_data" }, { "name": "stdout", "output_type": "stream", "text": [ "0.7872340425531914\n" ] }, { "data": { "image/png": "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\n", "text/plain": [ "
" ] }, "metadata": { "needs_background": "light" }, "output_type": "display_data" }, { "name": "stdout", "output_type": "stream", "text": [ "0.7916666666666666\n" ] }, { "data": { "image/png": "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\n", "text/plain": [ "
" ] }, "metadata": { "needs_background": "light" }, "output_type": "display_data" }, { "name": "stdout", "output_type": "stream", "text": [ "0.7959183673469389\n" ] }, { "data": { "image/png": "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\n", "text/plain": [ "
" ] }, "metadata": { "needs_background": "light" }, "output_type": "display_data" }, { "name": "stdout", "output_type": "stream", "text": [ "0.7999999999999999\n" ] }, { "data": { "image/png": "iVBORw0KGgoAAAANSUhEUgAAAW4AAAD8CAYAAABXe05zAAAABHNCSVQICAgIfAhkiAAAAAlwSFlzAAALEgAACxIB0t1+/AAAADl0RVh0U29mdHdhcmUAbWF0cGxvdGxpYiB2ZXJzaW9uIDMuMC4zLCBodHRwOi8vbWF0cGxvdGxpYi5vcmcvnQurowAAH6JJREFUeJzt3Xl0XOd53/Hvg32wAwTABSAIkeIqSqJIWKtj2ZJtKXIqObHqWrESbyc8dhvXTh23dnOak64np26duscrE3mtvMe2VMeVLMdSZFkiJZCSuIrgChIgCYDY92Xm7R+DAUESy4CYO3fuzO9zDg4BzMXguRzwxxfvfd77mnMOEREJjiy/CxARkcVRcIuIBIyCW0QkYBTcIiIBo+AWEQkYBbeISMAouEVEAkbBLSISMApuEZGAyfHiSauqqlxDQ4MXTy0ikpb27t170TlXHc+xngR3Q0MDTU1NXjy1iEhaMrOWeI/VVImISMAouEVEAkbBLSISMApuEZGAUXCLiASMgltEJGAU3CIiAeNJH7eIyEyT4Qj/cOA85/tGuf+GFTRUFfldUqApuEXEU+OTET78zVd44fhFAD7/TDNfe3QHb9tU43NlwaWpEhHx1H/5h8O8cPwi//X3t/LiZ+5hw/Ji/vX3XuVc74jfpQWWgltEPHOwrY/v7G7hQ3c18P7b1rCqPMSX/3AHY5MRvvCrY36XF1gKbhHxzOefaaY8lMsn375h+nP1ywp5/+31/HhfK2e7h32sLrgWDG4z22hmr8146zezTyajOBEJrrPdwzx7tIM/vqOBslDuZY/tfMtanHP84JWzPlUXbAsGt3PuqHNum3NuG7ADGAZ+6nllIhJoj+85Q5YZj9xaf9VjK8tC3L2hmh/vbWUyHPGhumBb7FTJvcAJ51zctx8UkcwzGY7wo6az3LuphhVlBbMe8/CO1VzoH6WppSfJ1QXfYoP7fcD3vChERNLHy6e76Roa59231M55zFs3VpOXk8Uzh9uTWFl6iDu4zSwPeBD40RyP7zSzJjNr6uzsTFR9IhJATx28QEFuFm/dOPeGLkX5Ody1bhm/PHwB51wSqwu+xYy4fxfY55yb9b9H59wu51yjc66xujqu3XdEJA1FIo6nD13g7g3VFObNv8bvns3LOds9wukudZcsxmKC+xE0TSIiCzh0rp/2/jHu37piwWPvXLcMgN0nu7wuK63EFdxmVgi8A/iJt+WISNDFlra/+fqFf/NeW1VEdUk+L51QcC9GXPcqcc4NA8s8rkVE0sBvj19k04oSqkvyFzzWzLhj7TJ2n+zCOYeZJaHC4NPKSRFJmNGJMK+c7ubOdVVxf80d65bRMTDGyYtDHlaWXhTcIpIw+1p6GJuM8Ob18f+C/qaGiumvlfgouEUkYV44fpGcLOPW6+IP7rVVxRTn57C/tc/DytKLgltEEublU93cWFdGcX78t/rPyjJurC3j9dZeDytLLwpuEUmIsckw+9v6aFxTseivvXl1OUfO9zM2GfagsvSj4BaRhDh0rp/xyQg7riW468qYCDuOnB/woLL0o+AWkYSIXVzcXn9tI26A/ZouiYuCW0QSYt+ZHuoqQtSUzn43wPmsLCugojCXI+f7Pags/Si4RWTJnHPsbem5pmkSiC7E2biihDcuaKokHgpuEVmytt4R2vvHrjm4ATatKOXohQEiEd0pcCEKbhFZslgP9s115df8HJtWlDA8Hqa1R7u/L0TBLSJLdqCtj5ys6HTHtYp97ZELmudeiIJbRJbsYFsfG5aXUJCbfc3PsWF5CWZwVPPcC1Jwi8iSOOc42NbHjbVlS3qeovwc6isLeUMj7gUpuEVkSdp6R+gZnmBr3dKCG6Lz3OosWZiCW0SW5GBb9MLk1lWlS36ujctLOH1xiNEJLX2fj4JbRJbkQFsf2VnG5pVLD+51NcVEHLRoD8p5KbhFZEkOtPWzvqZ4SRcmY9ZVFwNwsnNwyc+VzuLdc7LczH5sZm+Y2REzu8PrwkQk9TnnOJSAC5Mx11UVAXBCwT2veG+a+wXgKefcw2aWBxR6WJOIBETnwBhdQ+NsScD8NkQ7S1aVFXCiU9uYzWfB4DazUuAtwAcBnHPjwLi3ZYlIEDS3R0fGG5df+8KbK62tLtZUyQLimSpZC3QC3zCzV83s78ysyOO6RCQAmtujrXvrExjc66qLONE5hHO6Z8lc4gnuHGA78BXn3C3AEPCZKw8ys51m1mRmTZ2dnQkuU0RS0bGOASoKc6kqzkvYc66tLmZwbJLOgbGEPWe6iSe4W4FW59yeqY9/TDTIL+Oc2+Wca3TONVZXVyeyRhFJUc3tg6xfXoKZJew5Y50lxzVdMqcFg9s5dwE4a2Ybpz51L3DY06pEJOU552huH2B9TXFCn3dtdXQm9qQuUM4p3q6SjwOPT3WUnAQ+5F1JIhIE7f1jDIxOsiGB89sQ3Q0nlJut4J5HXMHtnHsNaPS4FhEJkEsXJhM74jYz6isLOdOt4J6LVk6KyDWJBXeiR9wA9csKtex9HgpuEbkmx9oHqSzKo6o4P+HP3bCskDPdw9rGbA4KbhG5Js0dib8wGVO/rIixyQgdagmclYJbRBbNOcfx9kFPpkkA1lRG76rR0qV57tkouEVk0c73jTIwNsmGBF+YjFmzLBbcmueejYJbRBbNi6XuM60qD5GdZbSos2RWCm4RWbRjUzeX8mqqJDc7i9rykEbcc1Bwi8iiNbcPUFWcR2VR4u5RcqU1U50lcjUFt4gsWnPHIOtrvBltx6xRL/ecFNwisijRjpIBzy5MxqypLKJvZILeYd3+/0oKbhFZlLbeEYbGw55dmIypV2fJnBTcIrIoXl+YjJluCdQ891UU3CKyKJfuUeLtVEldRTS423pGPP0+QaTgFpFFaW4fpLokn/JC7zpKAIrzc6gozKW1RyPuKym4RWRRjnV4f2Eypq6ikFaNuK+i4BaRuEUijmPt3rcCxtRVhDTinoWCW0Ti1tY7wshE2PMLkzHR4B7Rju9XUHCLSNySdWEypq6ikLHJCBcH1cs9U1xbl5nZaWAACAOTzjltYyaSgZqnWgG97uGOqasIAdDaM0x1SeI3bAiqxYy43+ac26bQFslcx9oHWF6aT1koNynfL9YSqAuUl9NUiYjErbljIGnz2wC10yNuBfdM8Qa3A35pZnvNbKeXBYlIaopEHMeTcHOpmdTLPbu45riBu5xz58ysBnjGzN5wzj0/84CpQN8JUF9fn+AyRcRvZ3uGGZ2IJO3CZIx6ua8W14jbOXdu6s8O4KfArbMcs8s51+ica6yurk5slSLiu2RfmIxRL/fVFgxuMysys5LY+8A7gYNeFyYiqeXSdmXJHnGrl/tK8UyVLAd+amax47/rnHvK06pEJOUcax9gZVkBpQXJ6SiJmdnLrZbAqAWD2zl3Erg5CbWISAprbh9M+jQJqJd7NmoHFJEFhSOOE52DbKhJ7jQJwOpK9XJfScEtIgs60z3M2GQkqT3cMbXl6uW+koJbRBbk14VJgKL8HCqL8tRZMoOCW0QWdLzDn1bAmLqKEGc14p6m4BaRBTW3D1BbHqI4P941e4lVWx6iTSPuaQpuEVlQtKMk+dMkMbXlIdp61csdo+AWkXlNd5T4NE0C0ZtNjU5E6BrSfblBwS0iC2jpGmJ8MsJ6H1oBY2KdJdrxPUrBLSLzit2jxM8Rd+y+3G29Cm5QcIvIAo5NtQJe7+eIu0Ij7pkU3CIyr+aOQeoqQhT51FECUBbKpSQ/R73cUxTcIjKvY+3J3fVmLrUVIU2VTFFwi8icJsMRTnYO+doKGFNbHtKy9ykKbhGZ0+muYcbDkaRuVzYXjbgvUXCLyJxiFyaTvV3ZbOoqQgyMTtI/OuF3Kb5TcIvInGKtgH52lMTUlk+1BGq6RMEtInNr7hhgdWWIwjz/Okpiait0e9cYBbeIzOlY+wAbUmB+G2aunlRLYNzBbWbZZvaqmf3cy4JEJDVMhCOcujjk261cr1RVnEd+TpYuULK4EfcngCNeFSIiqeX0xSEmwi4lLkwCmJk6S6bEFdxmVge8C/g7b8sRkVSRCvcouVL0vtwK7nhH3P8L+LdAZK4DzGynmTWZWVNnZ2dCihMR/zS3D2AG66pTY8QN0ZZAXZyMI7jN7PeADufc3vmOc87tcs41Oucaq6urE1agiPjjWMcA9ZWFhPKy/S5lWm15iK6hcUbGw36X4qt4Rtx3AQ+a2Wng+8A9ZvZ/PK1KRHzX3D6YEismZ5q+S2CGz3MvGNzOuc865+qccw3A+4BfO+ce9bwyEfHN2GSYUxeH2LQixYK7XPflBvVxi8gsTnQMEY44NqRYcNfpvtwALGo5lHPuOeA5TyoRkZTRPHWPko0p1FECsLy0gJwso603sxfhaMQtIlc52j5AbrZxXVWR36VcJjvLWFFWkPGdJQpuEblK84UB1lYVk5eTehGhXm4Ft4jM4mj7QMrNb8do9aSCW0SuMDg2SWvPCBtTZKn7lerKQ7T3jzIRnnM9YNpTcIvIZS5tnpCaI+66ikIiDi70jfpdim8U3CJymaMXosG9aUWpz5XMLrYI52wG395VwS0ilznaPkAoN3u6ZzrVXLovd+bOcyu4ReQyze0DbFheTFaW+V3KrFaWFwCZvXpSwS0ilzl6YTBl57cB8nOyqSnJ14hbRASga3CMi4NjbEzRVsCYugxvCVRwi8i0VNw8YTa1FYUZvXpSwS0i045e6AdIubsCXqm2PMT5vhEiEed3Kb5QcIvItKPtg5QX5lJdku93KfOqrQgxEXZ0DIz5XYovFNwiMq25fYANNSWYpWZHSUxdrCUwQ+8SqOAWEQAiEccb5/vZvDK1p0ng0iKcTJ3nVnCLCABnuocZGg+zZVVqrpicaXoRToZ2lii4RQSAI+ejFyY3r0z94C7Kz6GiMFcj7rmYWYGZvWxmr5vZITP7j8koTESS6/D5frKzLOVbAWNqKzL3vtzxjLjHgHucczcD24D7zex2b8sSkWQ7fK6fddVFFORm+11KXGrLM3cRTjy7vDvn3ODUh7lTb5nZPCmSxo6c7w/ENElMbXkhbT0jOJd5cRTXHLeZZZvZa0AH8Ixzbo+3ZYlIMvUMjXOub5QtQQruihAjE2F6hif8LiXp4gpu51zYObcNqANuNbOtVx5jZjvNrMnMmjo7OxNdp4h4KHZhMggdJTF10y2BmdfLvaiuEudcL/AccP8sj+1yzjU65xqrq6sTVJ6IJMPhAHWUxGTyfbnj6SqpNrPyqfdDwNuBN7wuTESS5/D5fmpK8qkqTu2l7jPFRtyZeIEyJ45jVgLfMrNsokH/Q+fcz70tS0SS6fC5/kBNkwCUhXIpysvOyF7uBYPbObcfuCUJtYiID8YnI5zoHORtm2r8LmVRzCzay52BI26tnBTJcMc6BpgIu0B1lMTUVRRqjltEMs/Btj4AbgjYVAlEL1Cqq0REMs7+1j5KCnJoWFbkdymLVlsRon90koHRzOrlVnCLZLgDbX3cWFuWsru6zydT7xKo4BbJYGOTYY6c7+fGujK/S7kmsftyZ9o8t4JbJIM1XxhkIuy4qbbc71KuSZ1G3CKSafa39QJwU0BH3FXF+eTlZGVcL7eCWySDHWjto7wwd3oVYtBkZVn09q4KbhHJFPtboxcmU31z4PnUlodo1VSJiGSC0Ykwze0DgZ0midGIW0QyxpHz/UxGHDcG9MJkTG1FiIuDY4xOhP0uJWkU3CIZ6vWzwb4wGRPr5T6XQdMlCm6RDLXvTC8rSgtYVR7MC5MxlzZUUHCLSJrbd6aH7WuCPU0ClxbhKLhFJK11DIzS2jPC9voKv0tZspVlIfKys2jpGvK7lKRRcItkoH0t0fnt7WuCH9zZWcbqyhCnFdwiks72nekhLzsrkLdync11VUWcvpg5t3dVcItkoH0tPWytLSU/J9vvUhKiYVkRLd1DRCLO71KSIp7Ngleb2bNmdsTMDpnZJ5JRmIh4Y3wywv62PnakwTRJzJqqIkYnIrQPjPpdSlLEM+KeBD7lnNsM3A78KzPb4m1ZIuKVw+f7GZ+MpMWFyZjrpjaBOHUxM+a5Fwxu59x559y+qfcHgCNArdeFiYg39rb0AOlxYTKmoaoQgJauzJjnXtQct5k1EN3xfY8XxYiI914+1cXqyhDLSwv8LiVhYi2BpzXivpyZFQN/D3zSOdc/y+M7zazJzJo6OzsTWaOIJEgk4thzqps71i7zu5SEys4y6pcVZkxLYFzBbWa5REP7cefcT2Y7xjm3yznX6JxrrK6uTmSNIpIgR9sH6B2e4PY0C26IdpZkSktgPF0lBjwGHHHOfd77kkTEK7tPdgFwW1oGd3TEnQktgfGMuO8C/gi4x8xem3p7wOO6RMQDu092UV9ZOH1HvXTSUFXE2GRmtATmLHSAc+4FILjbY4gIcGl++51blvtdiieuq7rUEriyLP3+Y5pJKydFMkQ6z28DrFmWOS2BCm6RDJHO89sAq8pC5OdkcbJz0O9SPKfgFskQvz2evvPbEN3xfW11Mcc7FNwikgbGJyO8dOIiv7O+yu9SPLW+pphjCm4RSQd7W3oYGg9z94b0XmOxvqaY1p4Rhscn/S7FUwpukQzw/LFOcrKMO69P8xH38mIATnSk9wpKBbdIBvino53sWFNBcf6CHcCBtn55CQDHOgZ8rsRbCm6RNNfRP8rh8/3cvTG9p0kA1lQWkpttNLen9zy3glskzT1/7CJA2s9vA+RkZ7G2qpjjGnGLSJD96nA7y0vz2bwiPfaXXMj1y9O/s0TBLZLGRifC/FNzJ+/csoKsrMy4c8X6mmLOdA8zOhH2uxTPKLhF0tjzzZ2MTIS574YVfpeSNOtrSnAOTqTxCkoFt0ga++XhdkoLcrhtbaXfpSRNrCWwuT1957kV3CJpajIc4R+PtHPv5uXkZmfOP/XrqorIy8ni8LmrNupKG5nzaopkmJdPddMzPJG2t3GdS252FptWlHBIwS0iQfPEa+coysvmrRtr/C4l6W5YVcqhc/04l5674Si4RdLQ6ESYXxw8z31bVxDKy/a7nKTbsqqMvpEJ2npH/C7FEwpukTT07BsdDIxO8u5ttX6X4osbVkV71tN1uiSezYK/bmYdZnYwGQWJyNL97LU2qorzuXNdem6asJDNK0rJsgwObuCbwP0e1yEiCdI7PM6zb3Ty4M2ryMmgbpKZQnnZrK0u5vC5Pr9L8cSCr6pz7nmgOwm1iEgC/P2+NsbDEd6zIzOnSWJuWFXKgbYMDe54mdlOM2sys6bOzs5EPa2ILIJzjsf3tHBLfTk3rCrzuxxf3bK6nPb+Mc6l4QXKhAW3c26Xc67ROddYXZ3+dyETSUW7T3ZzsnOI99+2xu9SfLdjTXS16N6WHp8rSbzMnAATSVOP72mhLJTL79200u9SfLdpZQkFuVnsO6PgFpEUda53hKcOXuDhHXUU5GZe7/aVcrOzuLmunH2ZOOI2s+8BLwEbzazVzD7ifVkislh/+5uTAHzorgZ/C0kh29dUcOhcf9rd4jWerpJHnHMrnXO5zrk659xjyShMROLXPTTO918+y4PbVlFXUeh3OSljR30FkxHH/tb06i7RVIlIGvjmi6cZmQjzsbvX+V1KStm+pgKAPSe7fK4ksRTcIgHXPTTON144xTu3LJ/e5VyiKovy2LKylBeOX/S7lIRScIsE3Bd/fZyh8Un+/L6NfpeSkn5nfRX7zvQwPD7pdykJo+AWCbAzXcN8Z/dp3tu4mg0abc/qzeurmAg79pxKnwXgCm6RAPtvvzhCdpbxZ+/Y4HcpKetNDZXk5WTxm+b0mS5RcIsE1NOHLvDUoQt8/J71LC8t8LuclFWQm82d65bxzJELabOxgoJbJID6Ryf4yycOsmlFCTvfstbvclLefTes4Gz3CEfOp8cGwgpukYBxzvFXTxyic2CMv37PTRm1EfC1eseW5ZjBU4cu+F1KQugVFwmYHzW18pNX2/jEvRvYtrrc73ICoao4nzc1VPKLA+fTYrpEwS0SIAfb+vgPTxzkzddX8af3XO93OYHy+7fUcrxjkFfP9vpdypIpuEUC4kzXMB/8xitUFefzN/9iG9lZ5ndJgfLPbl5FYV42P3j5rN+lLJmCWyQALvSN8oFvvMxkJMK3Pnwr1SX5fpcUOMX5ObzrxpX83/3n6BuZ8LucJVFwi6S4lq4hHv7qi3QOjPHYBxq5vqbY75IC6wN3NjA8HuY7L532u5QlUXCLpLCXTnTxnq+8yNDYJN/9k9umd3WRa7O1tox7NtXw2AunGBoL7hJ4BbdICpoMR/jSs8d59LE9lIZy+dFH7+SmOnWQJMLH77menuEJvvjscb9LuWY5fhcgIpfb29LDXz5xkEPn+nnXTSv56z+4kZKCXL/LShu31Ffw8I46/vb5k7x7Wy0bVwTvHi8acYukAOcc+8708OFvvsJ7vvIiHQNjfPXR7XzpD7crtD3w7x/YTFkol489vpf+0eBdqIwruM3sfjM7ambHzewzXhclkiku9I3yrRdP87tf+A1/8OUXaTrdzafv28hzf/5W7t+qDX+9UlmUx5ffv50zXcPs/HYTAwELb1toFZGZZQPNwDuAVuAV4BHn3OG5vqaxsdE1NTUlsk6RwHPO0dY7wutn+3jtbA8vnuji0Ll+AG5YVcojt9bz0LZVgR9h/+zVNj739FHO9Y6wqjzEp+/byLtvqfW7rFk98Vobn/rh66ytLuJ//PObfb2OYGZ7nXON8Rwbzxz3rcBx59zJqSf/PvAQMGdwi2QC5xxjk5Ho20SYkYkwvcMTdA+P0zM0TvfQOBcHxznTPURL1zAtXcMMTnUy5OVkcXNdGf/u/k3cu7kmbe6l/bNX2/jsTw4wMrU5b1vvCJ/9yQGAlAzvh7bVUlmUx6d++DoPfvG3vG1jNQ/cuJLGhkrqKkIpex+YeIK7Fpi51KgVuM2LYt771ZcYnbx6N+a5filwzP7AnMfP+TxzHb+4exokrM5F1jNnlV4/P96/NnN/38XVmsjXZmwiPB3YC8nNNuoqCqmvLKRxTQXraorZtrqcTStKyctJzVBYis89fXQ6tGNGJsJ87umjKRncAL+zvppffepuHvvNKb738hmePdoJQJZFp1RCedkU5uZMr1S1qQWrZmBc/rnywjy+/eFbPa85nuCebV3tVT/WZrYT2AlQX19/TcVUFOUyPpk9exE2+/LeuRb9znH4nF8x1/GLfX5b7PMv8nkW+emE/b3Nt7h6sd9j7nMIxmuTn5NNfk4W+bnZFORmkZ9z6c+KwlwqivKoLMyjoiiPkvwcsjJoafq53pFFfT5VlBbk8mfv2MAn376eNy4McKCtj7Pdw3QNjTMyHmZobJKIg1j0OXcpBGODCAeUhZIzzRVPcLcCq2d8XAecu/Ig59wuYBdE57ivpZiv/VFc0zsikqJWlYdomyWkV5WHfKhm8cyMzStL2byy1O9S5hXP72qvAOvN7DozywPeBzzpbVkiEkSfvm8jodzLf2sO5WbzaW1knFALjridc5Nm9qfA00A28HXn3CHPKxORwInNYwelqySoFmwHvBZqBxQRWZzFtAOm32VtEZE0p+AWEQkYBbeISMAouEVEAkbBLSISMApuEZGAUXCLiASMgltEJGA8WYBjZp1AyzV+eRVwMYHlBIHOOf1l2vmCznmx1jjnquM50JPgXgoza4p39VC60Dmnv0w7X9A5e0lTJSIiAaPgFhEJmFQM7l1+F+ADnXP6y7TzBZ2zZ1JujltEROaXiiNuERGZh2/BbWb3m9lRMztuZp+Z5fF8M/vB1ON7zKwh+VUmThzn+2/M7LCZ7TezfzSzNX7UmUgLnfOM4x42M2dmge9AiOeczey9U6/1ITP7brJrTLQ4frbrzexZM3t16uf7AT/qTBQz+7qZdZjZwTkeNzP731N/H/vNbHvCi3DOJf2N6E46J4C1QB7wOrDlimP+JfDVqfffB/zAj1qTeL5vAwqn3v9YkM833nOeOq4EeB7YDTT6XXcSXuf1wKtAxdTHNX7XnYRz3gV8bOr9LcBpv+te4jm/BdgOHJzj8QeA/0d0P+vbgT2JrsGvEfetwHHn3Enn3DjwfeChK455CPjW1Ps/Bu61ubYTT30Lnq9z7lnn3PDUh7uJbsocZPG8xgD/GfjvwGgyi/NIPOf8J8CXnHM9AM65jiTXmGjxnLMDYrvvljHLZuNB4px7Huie55CHgG+7qN1AuZmtTGQNfgV3LXB2xsetU5+b9Rjn3CTQByxLSnWJF8/5zvQRov9jB9mC52xmtwCrnXM/T2ZhHorndd4AbDCz35rZbjO7P2nVeSOec/4r4FEzawV+AXw8OaX5ZrH/3hdtwc2CPTLbyPnK9pZ4jgmKuM/FzB4FGoG7Pa3Ie/Oes5llAX8DfDBZBSVBPK9zDtHpkrcS/a3qN2a21TnX63FtXonnnB8Bvumc+59mdgfwnalzjnhfni88zy6/RtytwOoZH9dx9a9P08eYWQ7RX7Hm+/UklcVzvpjZ24G/AB50zo0lqTavLHTOJcBW4DkzO010LvDJgF+gjPfn+gnn3IRz7hRwlGiQB1U85/wR4IcAzrmXgAKi9/RIV3H9e18Kv4L7FWC9mV1nZnlELz4+ecUxTwIfmHr/YeDXbmrmP4AWPN+paYOvEQ3toM97wgLn7Jzrc85VOecanHMNROf1H3TONflTbkLE83P9M6IXojGzKqJTJyeTWmVixXPOZ4B7AcxsM9Hg7kxqlcn1JPDHU90ltwN9zrnzCf0OPl6ZfQBoJnpF+i+mPvefiP7jheiL+yPgOPAysNbvq8ken++vgHbgtam3J/2u2etzvuLY5wh4V0mcr7MBnwcOAweA9/ldcxLOeQvwW6IdJ68B7/S75iWe7/eA88AE0dH1R4CPAh+d8Rp/aerv44AXP9daOSkiEjBaOSkiEjAKbhGRgFFwi4gEjIJbRCRgFNwiIgGj4BYRCRgFt4hIwCi4RUQC5v8D0V+h0qlb7gcAAAAASUVORK5CYII=\n", "text/plain": [ "
" ] }, "metadata": { "needs_background": "light" }, "output_type": "display_data" }, { "name": "stdout", "output_type": "stream", "text": [ "0.803921568627451\n" ] }, { "data": { "image/png": "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\n", "text/plain": [ "
" ] }, "metadata": { "needs_background": "light" }, "output_type": "display_data" }, { "name": "stdout", "output_type": "stream", "text": [ "0.8076923076923077\n" ] }, { "data": { "image/png": "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\n", "text/plain": [ "
" ] }, "metadata": { "needs_background": "light" }, "output_type": "display_data" }, { "name": "stdout", "output_type": "stream", "text": [ "0.8113207547169813\n" ] }, { "data": { "image/png": "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\n", "text/plain": [ "
" ] }, "metadata": { "needs_background": "light" }, "output_type": "display_data" }, { "name": "stdout", "output_type": "stream", "text": [ "0.8148148148148149\n" ] }, { "data": { "image/png": "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\n", "text/plain": [ "
" ] }, "metadata": { "needs_background": "light" }, "output_type": "display_data" }, { "name": "stdout", "output_type": "stream", "text": [ "0.8181818181818181\n" ] }, { "data": { "image/png": "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\n", "text/plain": [ "
" ] }, "metadata": { "needs_background": "light" }, "output_type": "display_data" }, { "name": "stdout", "output_type": "stream", "text": [ "0.8214285714285715\n" ] }, { "data": { "image/png": "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\n", "text/plain": [ "
" ] }, "metadata": { "needs_background": "light" }, "output_type": "display_data" }, { "name": "stdout", "output_type": "stream", "text": [ "0.824561403508772\n" ] }, { "data": { "image/png": "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\n", "text/plain": [ "
" ] }, "metadata": { "needs_background": "light" }, "output_type": "display_data" }, { "name": "stdout", "output_type": "stream", "text": [ "0.8275862068965518\n" ] }, { "data": { "image/png": "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\n", "text/plain": [ "
" ] }, "metadata": { "needs_background": "light" }, "output_type": "display_data" }, { "name": "stdout", "output_type": "stream", "text": [ "0.8305084745762713\n" ] }, { "data": { "image/png": "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\n", "text/plain": [ "
" ] }, "metadata": { "needs_background": "light" }, "output_type": "display_data" }, { "name": "stdout", "output_type": "stream", "text": [ "0.8333333333333333\n" ] }, { "data": { "image/png": "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\n", "text/plain": [ "
" ] }, "metadata": { "needs_background": "light" }, "output_type": "display_data" }, { "name": "stdout", "output_type": "stream", "text": [ "0.8360655737704917\n" ] }, { "data": { "image/png": "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\n", "text/plain": [ "
" ] }, "metadata": { "needs_background": "light" }, "output_type": "display_data" }, { "name": "stdout", "output_type": "stream", "text": [ "0.8387096774193548\n" ] }, { "data": { "image/png": "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\n", "text/plain": [ "
" ] }, "metadata": { "needs_background": "light" }, "output_type": "display_data" }, { "name": "stdout", "output_type": "stream", "text": [ "0.8412698412698412\n" ] }, { "data": { "image/png": "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\n", "text/plain": [ "
" ] }, "metadata": { "needs_background": "light" }, "output_type": "display_data" }, { "name": "stdout", "output_type": "stream", "text": [ "0.8437499999999999\n" ] }, { "data": { "image/png": "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\n", "text/plain": [ "
" ] }, "metadata": { "needs_background": "light" }, "output_type": "display_data" }, { "name": "stdout", "output_type": "stream", "text": [ "0.8307692307692307\n" ] }, { "data": { "image/png": "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\n", "text/plain": [ "
" ] }, "metadata": { "needs_background": "light" }, "output_type": "display_data" }, { "name": "stdout", "output_type": "stream", "text": [ "0.8333333333333334\n" ] }, { "data": { "image/png": "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\n", "text/plain": [ "
" ] }, "metadata": { "needs_background": "light" }, "output_type": "display_data" }, { "name": "stdout", "output_type": "stream", "text": [ "0.835820895522388\n" ] }, { "data": { "image/png": "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\n", "text/plain": [ "
" ] }, "metadata": { "needs_background": "light" }, "output_type": "display_data" }, { "name": "stdout", "output_type": "stream", "text": [ "0.8235294117647058\n" ] }, { "data": { "image/png": "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\n", "text/plain": [ "
" ] }, "metadata": { "needs_background": "light" }, "output_type": "display_data" }, { "name": "stdout", "output_type": "stream", "text": [ "0.8115942028985509\n" ] }, { "data": { "image/png": "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\n", "text/plain": [ "
" ] }, "metadata": { "needs_background": "light" }, "output_type": "display_data" }, { "name": "stdout", "output_type": "stream", "text": [ "0.8142857142857145\n" ] }, { "data": { "image/png": "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\n", "text/plain": [ "
" ] }, "metadata": { "needs_background": "light" }, "output_type": "display_data" }, { "name": "stdout", "output_type": "stream", "text": [ "0.8169014084507041\n" ] }, { "data": { "image/png": "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\n", "text/plain": [ "
" ] }, "metadata": { "needs_background": "light" }, "output_type": "display_data" }, { "name": "stdout", "output_type": "stream", "text": [ "0.8194444444444444\n" ] }, { "data": { "image/png": "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\n", "text/plain": [ "
" ] }, "metadata": { "needs_background": "light" }, "output_type": "display_data" }, { "name": "stdout", "output_type": "stream", "text": [ "0.8219178082191781\n" ] }, { "data": { "image/png": "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\n", "text/plain": [ "
" ] }, "metadata": { "needs_background": "light" }, "output_type": "display_data" }, { "name": "stdout", "output_type": "stream", "text": [ "0.8243243243243243\n" ] }, { "data": { "image/png": "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\n", "text/plain": [ "
" ] }, "metadata": { "needs_background": "light" }, "output_type": "display_data" }, { "name": "stdout", "output_type": "stream", "text": [ "0.8266666666666667\n" ] }, { "data": { "image/png": "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\n", "text/plain": [ "
" ] }, "metadata": { "needs_background": "light" }, "output_type": "display_data" }, { "name": "stdout", "output_type": "stream", "text": [ "0.8157894736842106\n" ] }, { "data": { "image/png": "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\n", "text/plain": [ "
" ] }, "metadata": { "needs_background": "light" }, "output_type": "display_data" }, { "name": "stdout", "output_type": "stream", "text": [ "0.8181818181818182\n" ] }, { "data": { "image/png": "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\n", "text/plain": [ "
" ] }, "metadata": { "needs_background": "light" }, "output_type": "display_data" }, { "name": "stdout", "output_type": "stream", "text": [ "0.8076923076923075\n" ] }, { "data": { "image/png": "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\n", "text/plain": [ "
" ] }, "metadata": { "needs_background": "light" }, "output_type": "display_data" }, { "name": "stdout", "output_type": "stream", "text": [ "0.810126582278481\n" ] }, { "data": { "image/png": "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\n", "text/plain": [ "
" ] }, "metadata": { "needs_background": "light" }, "output_type": "display_data" }, { "name": "stdout", "output_type": "stream", "text": [ "0.8000000000000003\n" ] }, { "data": { "image/png": "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\n", "text/plain": [ "
" ] }, "metadata": { "needs_background": "light" }, "output_type": "display_data" }, { "name": "stdout", "output_type": "stream", "text": [ "0.8024691358024693\n" ] }, { "data": { "image/png": "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\n", "text/plain": [ "
" ] }, "metadata": { "needs_background": "light" }, "output_type": "display_data" }, { "name": "stdout", "output_type": "stream", "text": [ "0.8048780487804879\n" ] }, { "data": { "image/png": "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\n", "text/plain": [ "
" ] }, "metadata": { "needs_background": "light" }, "output_type": "display_data" }, { "name": "stdout", "output_type": "stream", "text": [ "0.8072289156626503\n" ] }, { "data": { "image/png": "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\n", "text/plain": [ "
" ] }, "metadata": { "needs_background": "light" }, "output_type": "display_data" }, { "name": "stdout", "output_type": "stream", "text": [ "0.8095238095238095\n" ] }, { "data": { "image/png": "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\n", "text/plain": [ "
" ] }, "metadata": { "needs_background": "light" }, "output_type": "display_data" }, { "name": "stdout", "output_type": "stream", "text": [ "0.8117647058823532\n" ] }, { "data": { "image/png": "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\n", "text/plain": [ "
" ] }, "metadata": { "needs_background": "light" }, "output_type": "display_data" }, { "name": "stdout", "output_type": "stream", "text": [ "0.813953488372093\n" ] }, { "data": { "image/png": "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\n", "text/plain": [ "
" ] }, "metadata": { "needs_background": "light" }, "output_type": "display_data" }, { "name": "stdout", "output_type": "stream", "text": [ "0.8045977011494253\n" ] }, { "data": { "image/png": "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\n", "text/plain": [ "
" ] }, "metadata": { "needs_background": "light" }, "output_type": "display_data" }, { "name": "stdout", "output_type": "stream", "text": [ "0.806818181818182\n" ] }, { "data": { "image/png": "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\n", "text/plain": [ "
" ] }, "metadata": { "needs_background": "light" }, "output_type": "display_data" }, { "name": "stdout", "output_type": "stream", "text": [ "0.8089887640449437\n" ] }, { "data": { "image/png": "iVBORw0KGgoAAAANSUhEUgAAAXQAAAD8CAYAAABn919SAAAABHNCSVQICAgIfAhkiAAAAAlwSFlzAAALEgAACxIB0t1+/AAAADl0RVh0U29mdHdhcmUAbWF0cGxvdGxpYiB2ZXJzaW9uIDMuMC4zLCBodHRwOi8vbWF0cGxvdGxpYi5vcmcvnQurowAAHZhJREFUeJzt3Xt0nHd95/H3dzTSjO6yrJEs2VJsx46TOE1IIkKgXFLSAIVCOAtbQg8s7XKahd3etrv0wOk5y7Y9Z5cDlG7b5XRxKZTd7QIFciClWwKFpFwWnDiJc7Pj2PFF1sW6XyxpdJ3f/jEziiNrRqOZZ55nZvR5neNjXZ6Z5/to7I9//s3393vMOYeIiJS/UNAFiIiINxToIiIVQoEuIlIhFOgiIhVCgS4iUiEU6CIiFUKBLiJSIRToIiIVQoEuIlIhwn6erK2tze3du9fPU4qIlL3HH398zDkX2+w4XwN97969HDt2zM9TioiUPTO7kMtxm065mNkXzGzEzJ694mutZvY9Mzud+n1HIcWKiEjhcplD/xvgLeu+9lHg+865g8D3U5+LiEiANg1059wPgYl1X74X+FLq4y8B7/S4LhER2aJ8u1w6nHNDAKnf270rSURE8lH0tkUzu9/MjpnZsdHR0WKfTkRk28o30IfNrBMg9ftIpgOdc0ecc73Oud5YbNOuGxERyVO+gf4g8IHUxx8AvuVNOSIikq9c2ha/DPwUOGRm/Wb2QeATwD1mdhq4J/W5iMiWOef4+6cG+bvHLrK0kgi6nLK26cIi59x7M3zrbo9rEZFt6I++fYIv/uQ8AN87OcyR99+OmQVbVJnSXi4iEpinLk7xxZ+c53139vCRNx/ieyeG+d6J4aDLKlsKdBEJzJEfnaUpGuZjv3QD/+b1+9mzo5a/+tHZoMsqWwp0EQnEzMIy333uEu++vZv6SJhwVYj33tHDY+cn6Z+cD7q8sqRAF5FAPPz8CMurjrfd3Ln2tbff3AXAd569FFRZZU2BLiKBeOi5S7Q3Rri1u2Xtaz0769jfVs//e3E8wMrKlwJdRHy3mnD86PQYb7y+nVDo5R0tr752J4+em2BlVS2MW6VAFxHfPX9phssLK9y5f+dV33vNtW3MLq7w9MB0AJWVNwW6iPju0XPJDVzv2Nd61ffSX3viwqSvNVUCBbqI+O7RcxPs2VFLV0vtVd+LNUbobI7yjEboW6ZAFxHfPXVxitt6Mt/o7KbdzQr0PCjQRcRX47OLDE4v8HO7mzMe83O7mzk7OsflhWUfKyt/CnQR8dVzgzMAHN7dlPGYdNifSB0ruVGgi4ivnh1MTqUc7so8Qj+0qxGAF0ZmfampUijQRcRXzw5M09NaR3NtdcZjOpuj1NdU8aICfUsU6CLiq2cHZrgpy3QLgJlxbXsDZxToW6JAFxHfzC2u0Dcxz42d2QMd4EBMgb5VCnQR8c2Lo8mAPtDesOmx17Y3cGlmQZ0uW6BAFxHfpEfcB9obNz02Hfovjs4VtaZKokAXEd+cHpklHDKu2Vm36bEHU4GuaZfcKdBFxDdnRmbZ21ZPddXm0dPTWkc4ZGvTNLI5BbqI+ObFkVkOxDafPwcIV4XYs6OWixO6e1GuFOgi4oullQQXJuZzekM0rbu1ToG+BQp0EfHF+fE5VhOOgx25B3pPax19CvScKdBFxBenh5Nz4dfmOOUCyUCfnF9mRq2LOVGgi4gv0m9ubjXQAU275EiBLiK+OD8+R2dzlNqaqpwf061A3xIFuoj4om98fm3EnaueVL+65tFzo0AXEV+cH5/PaUHRlZqi1bTUVSvQc6RAF5Gim1tcYWx2kWt21m/5sd076uibiBehqsqjQBeRokuPsLc65QLQ1RJlaEqBngsFuogU3YXxZKDvzWOE3tlcy+BUHOec12VVnIIC3cz+vZk9Z2bPmtmXzSzqVWEiUjn6JpI7JvZscQ4dYHdLLXNLq8wsrHhdVsXJO9DNbDfw20Cvc+4moAq4z6vCRKRynB+fp6WuOutt5zLpbEmOE4emNe2ymUKnXMJArZmFgTpgsPCSRKTS9I3Pc00e8+eQnHIBGNQ8+qbyDnTn3ADwaaAPGAKmnXPf9aowEakcFybm6Mlj/hySUy4Ag1MLXpZUkQqZctkB3AvsA7qAejN73wbH3W9mx8zs2OjoaP6VikhZWl5NMDi1wN485s8BYo0RwiHTlEsOCply+UXgnHNu1Dm3DDwAvGb9Qc65I865XudcbywWK+B0IlKOBibjrCZcXi2LAFUho6MpqhF6DgoJ9D7gTjOrMzMD7gZOelOWiFSK8+PJDpd8FhWldbVENYeeg0Lm0I8CXweeAJ5JPdcRj+oSkQrRP5kM4u7W2ryfo7O5lqFpjdA3U1CXi3Pu4865651zNznn3u+cW/SqMBGpDINTccIho70x/2UqnS1RhqbjJBJaXJSNVoqKSFENTMXpbIlSFbK8n2N3Sy3Lq46xOY0Zs1Ggi0hRDUzG6WrOf7oFXupFH9Ibo1kp0EWkqAam4uzeUVig72pKTtdcmlGgZ6NAF5GiWV5NMDyzwJ6WwgK9oykCwMhlTblko0AXkaK5NL1AwlHwCH1nQ4SQwYhG6Fkp0EWkaAZSveO7W/JbVJRWFTJijRGGFehZKdBFpGgGUj3oXS2F76zd0RRleEZTLtko0EWkaNIj9K4C59AB2hujGqFvQoEuIkUzMBmnrSFCtLqq4OfqaIroTdFNKNBFpGgGpwtvWUzraIoyMbfE4sqqJ89XiRToIlI0A5PxglsW09Kti6MapWekQBeRonDOMTAV9+QNUYD21OIivTGamQJdRIpibHaJxZXE2h2HCtWR2txLveiZKdBFpCjWetB3FNaDntau1aKbUqCLSFEMri0q8maE3lpXQzhkal3MQoEuIkWRXlTkVZdLKGS0N0Y0h56FAl1EimJgKk5DJExTNOzZc7Y3RRm5rBF6Jgp0ESmK/sk4u1tqSd5y2BsdTdrPJRsFuogUhRf7oK+n/VyyU6CLSFEMTsU9e0M0raMpynR8mYVlrRbdiAJdRDw3u7jCdHzZ8xF6rFGrRbNRoIuI517aNrdIgT6rQN+IAl1EPDcwNQ9414OeFmvQCD0bBbqIeG5gKtmJssfjKZd2TblkpUAXEc8NTMapqQqtjai90lpfg5kCPRMFuoh4bmAqTmdLlFDIux50gHBViJ31NZpDz0CBLiKeG5icp6vZ2+mWtLaGiEboGSjQRcRzxVhUlBZrVKBnokAXEU8trSQYubzoeYdLWkwj9IwU6CLiqUvTCzjn3S6L68UaI4zOLuKcK8rzlzMFuoh4qj/Vg+7VvUTXizVGWFpJMLOwUpTnL2cFBbqZtZjZ183seTM7aWav9qowESlPxVolmqbl/5kVOkL/M+A7zrnrgVuAk4WXJCLlLH3ruU6Pbg69nlaLZpb3zvNm1gS8Hvg1AOfcErDkTVkiUq4Gp+K0N0aIhKuK8vzazyWzQkbo+4FR4Itm9qSZfd7M6tcfZGb3m9kxMzs2OjpawOlEpBwUs2URNOWSTSGBHgZuA/7SOXcrMAd8dP1Bzrkjzrle51xvLBYr4HQiUg4GJr3fB/1KzbXVVFeZAn0DhQR6P9DvnDua+vzrJANeRLapRMIxOLVQ1EA3M2INEcY05XKVvAPdOXcJuGhmh1Jfuhs44UlVIlKWxmYXWVpNFHXKBbRaNJNCb8f9W8DfmlkNcBb49cJLEpFyle5wKeYIHZKBPjilm0WvV1CgO+eOA70e1SIiZW4t0H0YoT/VP13Uc5QjrRQVEc+kFxUVfYTeEGF8dpHVhJb/X0mBLiKeGZiK0xgN0xitLup5Yo0REg4m5rT05UoKdBHxzMBknD076op+HvWib0yBLiKeGZgqbg96mlaLbkyBLiKecM7RPxn3/MbQG4k1JPeJ0Qj95RToIuKJmfgKs4srvozQ2xprAAX6egp0EfFEeh/0YrcsAtTVhKmvqVKgr6NAFxFPpFsW/ZhygZfuXCQvUaCLiCf8WiWallz+r9WiV1Kgi4gnBibjRKtDtNbX+HI+7edyNQW6iHiiP7Vtrpn5cr5YgwJ9PQW6iHgieWOL4i8qSos1RphZWGFhedW3c5Y6BbqIeMKvRUVp6cVF2hf9JQp0ESnY/NIKE3NLvnW4gJb/b0SBLiIFG5zyt2URtFp0Iwp0ESnYRZ+2zb3SS1Mu2nExTYEuIgVb2wfdxxH6zgYt/19PgS4iBRuYihMOGe2NUd/OWV2V7HkfndXiojQFuogUbGAyTmdLlKqQPz3oaepFfzkFuogUbGAqzp4W/3rQ07Ra9OUU6CJSsIHJuK/z52naoOvlFOgiUpCllQTDlxd87XBJS4/QndPNokGBLiIFGpqO45y/HS5psYYIC8sJLi+u+H7uUqRAF5GCrO2DHsAIvb0p2Ys+MqNpF1Cgi0iB+iaSdyrqbg3mTVGAEe2LDijQRaRAfRPzhENGZ7N/Pehp6b53jdCTFOgiUpC+iXl276glXOV/nHQ0aYR+JQW6iBTk4sQ8PQFMtwA0RMLUVldphJ6iQBeRgvRNzAcyfw5gZrQ3RRjW4iJAgS4iBbi8sMzk/HJgI3SA9sYIIzOacgEFuogU4OJEsmUx0EBvimr5f0rBgW5mVWb2pJl924uCRKR8pFsWAx+hK9ABb0bovwOc9OB5RKTMXAywBz2tvTHK7OIKc1otWligm9ke4G3A570pR0TKSd/EPE3RMM211YHV0L62uEij9EJH6P8N+H0gkekAM7vfzI6Z2bHR0dECTycipaRvYp6encGNzgE6mtKLi/TGaN6Bbma/DIw45x7Pdpxz7ohzrtc51xuLxfI9nYiUoCB70NPW9nPRCL2gEfrPA+8ws/PAV4A3mtn/9qQqESl5qwlH/2Q80PlzeGnKZVgj9PwD3Tn3MefcHufcXuA+4AfOufd5VpmIlLThmQWWVhOBj9Cba6upCYfUuoj60EUkT6XQsgip1aJqXQQg7MWTOOceAR7x4rlEpDz0jadaFncEG+iQ7kXXlItG6CKSl3Pjc4RDxp4A7lS0XntjlGFt0KVAF5H8nB+bo6e1LpBtc9drb9J+LqBAF5E8nRubY19bfdBlAMle9JmFFRaWV4MuJVAKdBHZskTCcX58jr0lEujpW9Ft904XBbqIbNmlmQUWlhMlM0JXL3qSAl1Etuz82BxACQV6avm/RugiIltztsQCPX1vUY3QRUS26PzYHNHqELtSG2MFbUddDTVVoW3fuqhAF5EtOzc2x96d9YRCFnQpAIRCRkdzhKHpeNClBEqBLiJbdm68dFoW0zqbaxma1pSLiEjOVlYT9I3Pl0zLYlpnc1Qj9KALEJHyMjAVZyXhSnKEPjy9SCLhgi4lMAp0EdmSF0dnAdhfcoEeZWk1wfjcUtClBEaBLiJbcno4GegHOxoDruTlOpuTHTeXtvE8ugJdRLbkheFZOpoigd4YeiOdzcldHwe38Ty6Al1EtuTMyGUOtpfW6Bygs0UjdAW6iOTMOcfpkVkOtDcEXcpVWlOLizRCFxHJweD0AvNLqxzsKL1AD4WMXc1RjdBFRHLxwvBlgJKccgHY1RxlaEqBLiKyqTPpDpcSnHIB6GqOMjSjKRcRkU2dHrlMW0MNO+prgi5lQ7uaa7k0vbBtFxcp0EUkZ6dHZkt2ugWgqyXK8qrbtouLFOgikhPnHGeGZ0vyDdG09Ha+23VPFwW6iORkYCrO5cWVklsheqWultTioikFuohIRieHkh0uN3Y2BVxJZt076gC4OKFAFxHJ6MTgDGZw/a7SHaE311XTGA1zcXI+6FICoUAXkZycGJpm38566iPhoEvJqntHHRcnFOgiIhmdGJrhhq7SnW5J626t5eKkplxERDY0HV/m4kS8pOfP07p31NE/OY9z268XXYEuIpt6fmgGKO03RNO6W+tYWE4wOrsYdCm+yzvQzazbzB42s5Nm9pyZ/Y6XhYlI6TiZDvQymXKB7dnpUsgIfQX4D865G4A7gX9nZjd6U5aIlJJnBmZoa6ihvTESdCmbSrcu9m/DTpe8A905N+SceyL18WXgJLDbq8JEpHQ81T/FLXtaMLOgS9nUnrVA1wg9L2a2F7gVOOrF84lI6ZhZWObF0Vlu6W4JupSc1NZU0dYQ2ZatiwUHupk1AN8Aftc5N7PB9+83s2Nmdmx0dLTQ04mIz57pn8Y5yibQId26qEDfEjOrJhnmf+uce2CjY5xzR5xzvc653lgsVsjpRCQAxy9OAXDLnuaAK8ndNa11nB9ToOfMkpNpfw2cdM59xruSRKSUHL84xb62elrqSnMP9I3sa2tgcDrOwvJq0KX4qpAR+s8D7wfeaGbHU7/e6lFdIlICnHMcvzhVVqNzgP2xepyDc2NzQZfiq7w3ZXDO/Rgo/be8RSRvg9MLjF5e5BVlNH8OyUAHODs6xw1lsBjKK1opKiIZHT07DsAr97UGXMnW7GtLB/pswJX4S4EuIhk9em6CpmiY63eV1yi3riZMZ3OUs9tsykWBLiIZHT03wR37WqkKld/s6v5YvUboIiIAIzMLnBub444ym25J29/WwNnRuW2166ICXUQ2dPTcBACv2rcz4Erysz9Wz+XFlW2166ICXUQ2dPTcOPU1VRwugx0WN7I/1gAkO122CwW6iFzFOcc/vzDKq6/dSbiqPGPiQHsy0E+PbJ959PJ8pUSkqM6OzXFxIs4bDrUHXUreupqjNEXDa3u5bwcKdBG5yiOnkhvp3XVd+e6/ZGZc39m0drel7UCBLiJXeeTUCNfG6ulurQu6lILc2NnE85cuk0hsj04XBbqIvEx8aZWj5ya4q4ynW9Ku39XI/NIqfdtkb3QFuoi8zCOnRlhaSXD39eUf6Ol9XLbLPLoCXURe5tvPDNHWUFO2C4qudGhXIyFToIvINjS/tMIPTo7w5sO7yrZd8UrR6iqu62jkeP900KX4ovxfMRHxzMPPjxJfXuVtN3cGXYpnbu1p4Xjf5LZ4Y1SBLiJrHnxqgLaGmrJd7r+RW3t2MLOwwtmxyl9gpEAXESC5Gdf3T47wrtv2lOXuipnc1pO8OccTfVMBV1J8CnQRAeBrj/ezknC855XdQZfiqf1tDTRFwzzZNxl0KUWnQBcREgnHlx/t49X7d65talUpQiHjtmt28Ghq98hKpkAXEf7p5DD9k3F+9VU9QZdSFK890MaLo3MMTsWDLqWoFOgi25xzjs8+fIae1jp+6aZdQZdTFK87mNyT5senxwKupLgU6CLb3I/PjPFU/zQfesO1FdF7vpHrOhpob4zww9OjQZdSVJX56olIThIJx2e+9wIdTRHedfvuoMspGjPjdQdj/PjMGCuriaDLKRoFusg29s3jAzzZN8V/fNMhIuGqoMspqntubGdqfpmfnh0PupSiUaCLbFOXF5b5r//4PLd0t/Cu2/YEXU7R3XWoncZImG8dHwy6lKJRoItsUx//1nOMzy7yh+84TKiCFhJlEq2u4k2Hd/HQs5dYWF4NupyiUKCLbEPfOj7AA08O8JtvPMgruluCLsc377y1i8uLK/zD00NBl1IUCnSRbebxC5P8/tefpveaHfz2Gw8EXY6vXnugjes6GvirH53FucrbrEuBLrKNnBic4YNfeozO5iife//tFdummImZ8Ruv28/zly7zyAuV18K4vV5NkW3sJ2fGeM/nfkptdRVf+td3sLMhEnRJgbj3Fbvpbq3lv/zDSZYrrIVRgS5S4RZXVvn0Q6d4/18fZVdzlG98+DVcs7M+6LICUxMO8Z9++TCnR2b5i++fDrocT4ULebCZvQX4M6AK+Lxz7hOeVCUiBVteTfDtpwf5k+++QP9knH95+x4+/o7DNEQK+mtfEe65sYN3376Hv3j4DAc7Gnn7LV1Bl+SJvF9ZM6sCPgvcA/QDj5nZg865E14VJyJbs7ya4IkLk/zg+RG+8cQAY7OLHO5q4hP/4mZee7At6PL45pMDfOqhUwxOxelqqeUjbz7EO28NZoXqH997E33j8/zuV49zYXyuIrY+KOSf6juAM865swBm9hXgXkCBLlIEzjkWVxLMLa4wv7TK6Owil6YXGJpe4ML4HCcGZzgxNMP80irhkHHXoRjvvaOHXzjUXhJ95t98coCPPfAM8VQP+MBUnI898AxAIKFeW1PFF3/9lXz0gWf49Hdf4GuP9/OeV3bzugMx9sfqqS/D/8kUUvFu4OIVn/cDryqsnI394d8/x+MXNt6cPlvnkWPjb2Z9TNbny/a4/FqgPK8/67kyfzdr9T6eL5+fx6aPy7M7Ld+fV6aH5V0/sLC0ytzSCplui9kQCXNDZyO/0tvNnftbec2BNpqi1Vmq9N+nHjq1FuZp8eVVPvXQqcBG6fWRMH9+3yt4+82d/OU/v8gnv3OKT3IKgObaauprqqiLhKlOjdwNsNS/jWZg2Eufb3Ku//6rt9HdWlecC0kpJNA3qv+qP25mdj9wP0BPT357LTdGq9lZX5O5EMv8o8z0nSwPyfKo7I/L9pTZH5fn+TJ8L9vz5fmtvH7Gycdt/Xz5niv7tZXGa5rP6wnJVY71NWHqIqnfa6poa4iwqzlKZ3OU5trqrD+3UpBpL/Kg9yg3M950eBdvOryLkZkFHjs/yfnxOYZnFphfWmV+aYXlVZf6RzcZcc4lP0r/o5/LmMGP2/oVEuj9wJX3qtoDXLVJgnPuCHAEoLe3N6+x0u/dc10+DxOREtLVUsvABuHd1VIbQDUba2+K8rabO4MuI2+FvAPwGHDQzPaZWQ1wH/CgN2WJSKX5yJsPUVv98h0da6ur+MibDwVUUeXJe4TunFsxs98EHiLZtvgF59xznlUmIhUlPU9eKl0ulcj83M+gt7fXHTt2zLfziYhUAjN73DnXu9lx5d10KSIiaxToIiIVQoEuIlIhFOgiIhVCgS4iUiEU6CIiFUKBLiJSIRToIiIVwteFRWY2ClzI8+FtwJiH5ZQDXfP2oGuufIVe7zXOudhmB/ka6IUws2O5rJSqJLrm7UHXXPn8ul5NuYiIVAgFuohIhSinQD8SdAEB0DVvD7rmyufL9ZbNHLqIiGRXTiN0ERHJouQC3czeYmanzOyMmX10g+9HzOyrqe8fNbO9/lfprRyu+ffM7ISZPW1m3zeza4Ko00ubXfMVx73bzJyZlXVHRC7Xa2a/knqdnzOz/+N3jV7L4c91j5k9bGZPpv5svzWIOr1kZl8wsxEzezbD983M/jz1M3nazG7ztADnXMn8InnnoxeB/UAN8BRw47pj/i3wP1If3wd8Nei6fbjmXwDqUh9/eDtcc+q4RuCHwM+A3qDrLvJrfBB4EtiR+rw96Lp9uOYjwIdTH98InA+6bg+u+/XAbcCzGb7/VuAfSd5//E7gqJfnL7UR+h3AGefcWefcEvAV4N51x9wLfCn18deBu63Ub3ee3abX7Jx72Dk3n/r0ZyRvyF3OcnmdAf4Y+CSw4GdxRZDL9f4G8Fnn3CSAc27E5xq9lss1O6Ap9XEzG9xkvtw4534ITGQ55F7gf7qknwEtZubZXalLLdB3Axev+Lw/9bUNj3HOrQDTwE5fqiuOXK75Sh8k+S98Odv0ms3sVqDbOfdtPwsrklxe4+uA68zsJ2b2MzN7i2/VFUcu1/yfgfeZWT/wf4Hf8qe0QG317/uW5H2T6CLZaKS9vg0nl2PKSc7XY2bvA3qBNxS1ouLLes1mFgL+FPg1vwoqslxe4zDJaZe7SP4P7EdmdpNzbqrItRVLLtf8XuBvnHN/YmavBv5X6poTxS8vMEXNr1IbofcD3Vd8voer/xu2doyZhUn+Vy3bf3FKXS7XjJn9IvAHwDucc4s+1VYsm11zI3AT8IiZnSc51/hgGb8xmuuf628555adc+eAUyQDvlzlcs0fBP4OwDn3UyBKcs+TSpbT3/d8lVqgPwYcNLN9ZlZD8k3PB9cd8yDwgdTH7wZ+4FLvNpSpTa85Nf3wOZJhXu5zq7DJNTvnpp1zbc65vc65vSTfN3iHc+5YMOUWLJc/198k+eY3ZtZGcgrmrK9VeiuXa+4D7gYwsxtIBvqor1X670HgX6W6Xe4Epp1zQ549e9DvCmd4F/gFku+Q/0Hqa39E8i80JF/0rwFngEeB/UHX7MM1/xMwDBxP/Xow6JqLfc3rjn2EMu5yyfE1NuAzwAngGeC+oGv24ZpvBH5CsgPmOPCmoGv24Jq/DAwByyRH4x8EPgR86IrX+bOpn8kzXv+51kpREZEKUWpTLiIikicFuohIhVCgi4hUCAW6iEiFUKCLiFQIBbqISIVQoIuIVAgFuohIhfj/BQLNUvKnt04AAAAASUVORK5CYII=\n", "text/plain": [ "
" ] }, "metadata": { "needs_background": "light" }, "output_type": "display_data" }, { "name": "stdout", "output_type": "stream", "text": [ "0.8111111111111111\n" ] }, { "data": { "image/png": "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\n", "text/plain": [ "
" ] }, "metadata": { "needs_background": "light" }, "output_type": "display_data" }, { "name": "stdout", "output_type": "stream", "text": [ "0.8131868131868134\n" ] }, { "data": { "image/png": "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\n", "text/plain": [ "
" ] }, "metadata": { "needs_background": "light" }, "output_type": "display_data" }, { "name": "stdout", "output_type": "stream", "text": [ "0.815217391304348\n" ] }, { "data": { "image/png": "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\n", "text/plain": [ "
" ] }, "metadata": { "needs_background": "light" }, "output_type": "display_data" }, { "name": "stdout", "output_type": "stream", "text": [ "0.8172043010752688\n" ] }, { "data": { "image/png": "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\n", "text/plain": [ "
" ] }, "metadata": { "needs_background": "light" }, "output_type": "display_data" }, { "name": "stdout", "output_type": "stream", "text": [ "0.8191489361702126\n" ] }, { "data": { "image/png": "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\n", "text/plain": [ "
" ] }, "metadata": { "needs_background": "light" }, "output_type": "display_data" }, { "name": "stdout", "output_type": "stream", "text": [ "0.8210526315789475\n" ] }, { "data": { "image/png": "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\n", "text/plain": [ "
" ] }, "metadata": { "needs_background": "light" }, "output_type": "display_data" }, { "name": "stdout", "output_type": "stream", "text": [ "0.8125\n" ] }, { "data": { "image/png": "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\n", "text/plain": [ "
" ] }, "metadata": { "needs_background": "light" }, "output_type": "display_data" }, { "name": "stdout", "output_type": "stream", "text": [ "0.8144329896907216\n" ] }, { "data": { "image/png": "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\n", "text/plain": [ "
" ] }, "metadata": { "needs_background": "light" }, "output_type": "display_data" }, { "name": "stdout", "output_type": "stream", "text": [ "0.8061224489795918\n" ] }, { "data": { "image/png": "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\n", "text/plain": [ "
" ] }, "metadata": { "needs_background": "light" }, "output_type": "display_data" }, { "name": "stdout", "output_type": "stream", "text": [ "0.808080808080808\n" ] }, { "data": { "image/png": "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\n", "text/plain": [ "
" ] }, "metadata": { "needs_background": "light" }, "output_type": "display_data" }, { "name": "stdout", "output_type": "stream", "text": [ "0.8099999999999998\n" ] }, { "data": { "image/png": "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\n", "text/plain": [ "
" ] }, "metadata": { "needs_background": "light" }, "output_type": "display_data" }, { "name": "stdout", "output_type": "stream", "text": [ "0.811881188118812\n" ] }, { "data": { "image/png": "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\n", "text/plain": [ "
" ] }, "metadata": { "needs_background": "light" }, "output_type": "display_data" }, { "name": "stdout", "output_type": "stream", "text": [ "0.8137254901960784\n" ] } ], "source": [ "import matplotlib.pyplot as plt\n", "\n", "for i in range(k):\n", " p = numpy.linspace(0,1,1000)\n", " l = p**y[i]*(1-p)**(i+1-y[i])*pi0(p)\n", " l = l/numpy.sum(l)*1000\n", " plt.plot(p,l)\n", " plt.scatter(numpy.array([p0]),numpy.array([0]))\n", " plt.show()\n", " print(numpy.mean(l*p))" ] }, { "cell_type": "markdown", "metadata": {}, "source": [ "# Maximum Likelihood Estimator" ] }, { "cell_type": "markdown", "metadata": {}, "source": [ "We are writing now the sequence of maximum likelihood estimators as red point denoting the argument where the maximum is taken of the likelihood function." ] }, { "cell_type": "code", "execution_count": 124, "metadata": {}, "outputs": [ { "data": { "image/png": "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\n", "text/plain": [ "
" ] }, "metadata": { "needs_background": "light" }, "output_type": "display_data" }, { "data": { "image/png": "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\n", "text/plain": [ "
" ] }, "metadata": { "needs_background": "light" }, "output_type": "display_data" }, { "data": { "image/png": "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\n", "text/plain": [ "
" ] }, "metadata": { "needs_background": "light" }, "output_type": "display_data" }, { "data": { "image/png": "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\n", "text/plain": [ "
" ] }, "metadata": { "needs_background": "light" }, "output_type": "display_data" }, { "data": { "image/png": "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\n", "text/plain": [ "
" ] }, "metadata": { "needs_background": "light" }, "output_type": "display_data" }, { "data": { "image/png": "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\n", "text/plain": [ "
" ] }, "metadata": { "needs_background": "light" }, "output_type": "display_data" }, { "data": { "image/png": "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\n", "text/plain": [ "
" ] }, "metadata": { "needs_background": "light" }, "output_type": "display_data" }, { "data": { "image/png": "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\n", "text/plain": [ "
" ] }, "metadata": { "needs_background": "light" }, "output_type": "display_data" }, { "data": { "image/png": "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\n", "text/plain": [ "
" ] }, "metadata": { "needs_background": "light" }, "output_type": "display_data" }, { "data": { "image/png": "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\n", "text/plain": [ "
" ] }, "metadata": { "needs_background": "light" }, "output_type": "display_data" }, { "data": { "image/png": "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\n", "text/plain": [ "
" ] }, "metadata": { "needs_background": "light" }, "output_type": "display_data" }, { "data": { "image/png": "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\n", "text/plain": [ "
" ] }, "metadata": { "needs_background": "light" }, "output_type": "display_data" }, { "data": { "image/png": "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\n", "text/plain": [ "
" ] }, "metadata": { "needs_background": "light" }, "output_type": "display_data" }, { "data": { "image/png": "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\n", "text/plain": [ "
" ] }, "metadata": { "needs_background": "light" }, "output_type": "display_data" }, { "data": { "image/png": "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\n", "text/plain": [ "
" ] }, "metadata": { "needs_background": "light" }, "output_type": "display_data" }, { "data": { "image/png": "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\n", "text/plain": [ "
" ] }, "metadata": { "needs_background": "light" }, "output_type": "display_data" }, { "data": { "image/png": "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\n", "text/plain": [ "
" ] }, "metadata": { "needs_background": "light" }, "output_type": "display_data" }, { "data": { "image/png": "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\n", "text/plain": [ "
" ] }, "metadata": { "needs_background": "light" }, "output_type": "display_data" }, { "data": { "image/png": "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\n", "text/plain": [ "
" ] }, "metadata": { "needs_background": "light" }, "output_type": "display_data" }, { "data": { "image/png": "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\n", "text/plain": [ "
" ] }, "metadata": { "needs_background": "light" }, "output_type": "display_data" }, { "data": { "image/png": "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\n", "text/plain": [ "
" ] }, "metadata": { "needs_background": "light" }, "output_type": "display_data" }, { "data": { "image/png": "iVBORw0KGgoAAAANSUhEUgAAAZYAAAD8CAYAAABU4IIeAAAABHNCSVQICAgIfAhkiAAAAAlwSFlzAAALEgAACxIB0t1+/AAAADl0RVh0U29mdHdhcmUAbWF0cGxvdGxpYiB2ZXJzaW9uIDMuMC4zLCBodHRwOi8vbWF0cGxvdGxpYi5vcmcvnQurowAAIABJREFUeJzt3Xt8XNV16PHf0ujlhx62JNuyZCM/ZGzJ+EGEnQAGAjfBwC12UlNMkoa2tCQNtPeGtgGaT1803E/dpHEfgaY0JCGE1ia0BSchcaAYsHkYy9jGlm1Z8lt+SbIlWbKs97p/zBYV4xnNSB7pzGN9Px9/mNmzz9rrjIWXztnn7COqijHGGBMtKV4nYIwxJrFYYTHGGBNVVliMMcZElRUWY4wxUWWFxRhjTFRZYTHGGBNVVliMMcZElRUWY4wxUWWFxRhjTFSlep2AF/Lz87WkpMTrNIwxJq5s3769UVULwvVLysJSUlJCZWWl12kYY0xcEZGjkfSzU2HGGGOiygqLMcaYqLLCYowxJqqssBhjjIkqKyzGGGOiygqLMcaYqLLCYowxJqqS8j4WY0zi6uju5fXqBg42tJGVmcoNpQWU5I/zOq2kYoXFGJMwXt59ir/cUEV9a+dH2u/6WDF/cWc54zPsn7zRYN+yMSbuqSr/+N+1rH31AOVTs/nWXQu5pmQijW2d/Pjdo3xvy2H2njrPc7+7lNyx6V6nm/BsjsUYE/eeevMQa189wK9fXcyLD1zHDXMKGJPuY9rEsTx6+zy+d28FNWfauP/Z7XT29HqdbsKzwmKMiWubquv5m1/u544FhXxz1QLSfJf+s/bJKyfxzbsW8N7hc3z7Vwc8yDK5WGExxsSt5vYuvvbCB1w5OYtvrVpISoqE7LtiURH3LJnOv24+xPvHmkYxy+RjhcUYE7ce+9lezl3o4lt3LWRMui9s/z+9fS6TszP5i5eq6OvTUcgwOVlhMcbEpe1Hz/Gf75/g92+cxfyinIi2ycpM448+fSW7T7Tw892nRjjD5GWFxRgTd1SVx3++j0lZGXzlk7OGtO1nFhcxd0oW337lAL121DIirLAYY+LOxqrTvH+smYc+NYex6UO7a8KXIjx482wON17glb1nRijD5GaFxRgTV/rvWZmZP45VHyseVozl5VOYNnEM//LmwShnZ8AKizEmzmyuaWTvqfN86caZpAa5tDgSqb4Ufue6Gew41syeEy1RztBE9LciIstFpFpEakXkkSCfZ4jIevf5VhEpGfDZo669WkRuDRdTRGa4GDUuZvpgY4jI50Vk54A/fSKyaLhfiDEmtn33jYNMzs5g5eKiy4rz2cXFZKSmsG7bsShlZvqFLSwi4gOeAG4DyoB7RKQsoNt9QJOqzgbWAmvctmXAaqAcWA48KSK+MDHXAGtVtRRocrFDjqGqz6nqIlVdBPwmcERVdw79qzDGxLoP6pp5++BZfue6GWSkhr+8eDA5Y9O4/apCXtpxkotddjd+NEVyxLIEqFXVQ6raBawDVgT0WQE8416/ANwiIuLa16lqp6oeBmpdvKAx3TY3uxi4mCvDjDHQPcC/R7BPxpg49KN3jjIu3cfnlk6PSry7r5lGa2ePXXocZZEUliLg+ID3da4taB9V7QFagLxBtg3Vngc0uxiBY4UaY6C7scJiTEJqae/mp7tOcueiIrIy06ISc+mMiVyRN5aXdp6ISjzjF0lhCbZGQuDF36H6RKs9bB4ishRoV9U9QfohIveLSKWIVDY0NATrYoyJYf+5o47Onj4+H6WjFQAR4Y6rCnn74FnOXeiKWtxkF0lhqQOmDXhfDJwM1UdEUoEc4Nwg24ZqbwRyXYzAsUKN0W81gxytqOpTqlqhqhUFBQWD7K4xJtaoKs9tPcbCabkR32UfqTsWFNLbp/xyz+moxk1mkRSWbUCpu1orHf8/4BsC+mwA7nWvVwGvqaq69tXuiq4ZQCnwXqiYbptNLgYu5kthxkBEUoC78M/VGGMSzPvHmqitb+PzS6J3tNKvrDCbmfnj+PnuwN+XzXCFLSxuPuNBYCOwD3heVatE5DERudN1exrIE5Fa4CHgEbdtFfA8sBf4JfCAqvaGiuliPQw85GLludghx3BuAOpU9dBwvgRjTGz7z/dPkJmWwu0LCqMeW0S4Y0Eh7xw8y9m2zvAbmLDE/dKfVCoqKrSystLrNIwxEejq6eOax1/lxjkF/OM9i0dkjA/qmrnzO2/x7d9YyGevHt7d/MlARLarakW4fnbnvTEmpr1eXU/LxW4+c5k3RA5m/tQc8sdn8Nr++hEbI5lYYTHGxLQXd54gb1w6y0rzR2yMlBTh5rkFvHmgge7evhEbJ1lYYTHGxKyWi928uq+eX1s4ddjrgkXq5rmTON/Rw/aj9nTJy2WFxRgTszZWnaarp++y1wWLxPWlBaT5hE12OuyyWWExxsSsX+45TfGEMSwsju69K8GMz0hl6Yw8m2eJAissxpiYdL6jmy01jSwvn8KlywKOjBvm5FNT38aZ8x2jMl6issJijIlJr+2rp6u3j9uumjJqY147y3+BwNsHG0dtzERkhcUYE5N+secUk7MzWDxtwqiNWVaYTe7YNN6qPTtqYyYiKyzGmJjT3tXDGwcauLV8Cikpo3MaDPyXHX9iZh7vHDxLMt48Hi1WWIwxMef16gY6uvtYPn/0ToP1u3Z2PieaL3L0bPuoj50orLAYY2LOL/acJm9cOktKJo762NfN8j/m6e2DdjpsuKywGGNiSndvH6/vr+eWeZNG/KbIYGbkj2NKdiZv2QT+sFlhMcbElG1HztHa2cMt8yZ7Mr6I8PGZE9l2+JzNswyTFRZjTEx5bV896b4Urp89cmuDhVNRMpH61k6On7voWQ7xzAqLMSamvLa/no/PymNcRmr4ziPkGje3s+3IuTA9TTBWWIwxMeNQQxuHGi9wy9xJnuZROmk82ZmpVNqClMNihcUYEzP61+m62ePCkpIifOyKCVTaEcuwRFRYRGS5iFSLSK2IPBLk8wwRWe8+3yoiJQM+e9S1V4vIreFiisgMF6PGxUyPYIwFIvKOiFSJyG4RyRzOl2GM8dZr++u5cnIW0yaO9ToVKkomUlPfRtOFLq9TiTthC4uI+IAngNuAMuAeESkL6HYf0KSqs4G1wBq3bRmwGigHlgNPiogvTMw1wFpVLQWaXOzBxkgFfgx8WVXLgZuA7iF+D8YYj53v6Oa9w+e4eZ63Ryv9+udZ7PksQxfJEcsSoFZVD6lqF7AOWBHQZwXwjHv9AnCL+JcjXQGsU9VOVT0M1Lp4QWO6bW52MXAxV4YZ49PAB6q6C0BVz6pqb+RfgTEmFmw+0EhPn3o+v9JvQXEO6b4Uth2102FDFUlhKQKOD3hf59qC9lHVHqAFyBtk21DteUCzixE4Vqgx5gAqIhtF5H0R+VoE+2SMiTH/vf8MuWPTWDx99BadHExmmo+rinOoPGJHLEMVSWEJtgJc4F1DofpEq32wMVKB64HPu/9+RkRuCewoIveLSKWIVDY0NAQJZYzxSl+f8uaBBm6cU4BvFBedDOfq6bnsPtFCd2+f16nElUgKSx0wbcD7YuBkqD5uziMHODfItqHaG4FcFyNwrMHGeENVG1W1HXgZuDpwJ1T1KVWtUNWKgoKCCHbbGDNa9p9upbGti2WlsfX/5qJpE+jq6WP/qVavU4krkRSWbUCpu1orHf9k/IaAPhuAe93rVcBr6l8LYQOw2l3RNQMoBd4LFdNts8nFwMV8KcwYG4EFIjLWFZwbgb2RfwXGGK9tqfWfRfDybvtgFk7zPxJ5Z12zx5nEl7CFxc1nPIj/H/B9wPOqWiUij4nIna7b00CeiNQCDwGPuG2rgOfx/0P/S+ABVe0NFdPFehh4yMXKc7EHG6MJ+Db+YrUTeF9Vfz7cL8QYM/o21zRSOmk8U3Ji606Botwx5I9PZ9dxKyxDEdGaCar6Mv5TTAPb/nzA6w7grhDbPg48HklM134I/1Vjge2DjfFj/JccG2PiTEd3L+8dPsfnlk73OpVLiAgLi3OtsAyR3XlvjPHU9qNNdPb0saw0tk6D9Vs4LZfahjZaO+z2uEhZYTHGeOrNmgbSfMLSGXlepxLUwmm5qMLuuhavU4kbVliMMZ7aUtPI4ukTPF3NeDALi20Cf6issBhjPHO2rZOqk+dZFmNXgw2UOzadGfnjbJ5lCKywGGM885Z7rvz1MTq/0m9hcQ67jtupsEhZYTHGeGZLTQPZmaksKM71OpVBLZyWy+nzHZxu6fA6lbhghcUY4wlVZUtNI9fOyo+pZVyCWTjNX/h22TxLRKywGGM8cajxAidbOlg2J7ZPgwGUFWbjSxGqTtjpsEhYYTHGeGJLTSMAy2bH1vpgwWSm+ZhdMJ49J897nUpcsMJijPHE5ppGpk8cy/Q8758WGYnyqdnssSOWiFhhMcaMuu7ePt49dDbmrwYbqLwoh/rWTurP2wR+OFZYjDGjbtfxZto6e2L6/pVA86dmA1Blp8PCssJijBl1m2saSRG4dlb8FJYyV1jsdFh4VliMMaNuS20jVxXnkjM2zetUIpaVmcaM/HF2xBIBKyzGmFF1vqObnceb4+o0WL/yqdnsOWlHLOFYYTHGjKp3D56lt0/jauK+3/yiHOqaLtLc3uV1KjHNCosxZlRtqW1kbLqPq6dP8DqVISu3CfyIRFRYRGS5iFSLSK2IPBLk8wwRWe8+3yoiJQM+e9S1V4vIreFiisgMF6PGxUwfbAwRKRGRiyKy0/357nC/DGPMyNtS08jSGRNJT42/32vLp/qX0LcJ/MGF/ZsVER/wBHAbUAbcIyJlAd3uA5pUdTawFljjti0DVgPlwHLgSRHxhYm5BlirqqVAk4sdcgznoKoucn++PKRvwBgzak40X+RQ4wWuL439u+2DmTgunaLcMXYHfhiR/MqwBKhV1UOq2gWsA1YE9FkBPONevwDcIiLi2tepaqeqHgZqXbygMd02N7sYuJgrw4xhjIkTW2oaAGL2McSRKJ+abWuGhRFJYSkCjg94X+fagvZR1R6gBcgbZNtQ7XlAs4sROFaoMQBmiMgOEXlDRJYF2wkRuV9EKkWksqGhIYLdNsZE25s1jUzOzqB00nivUxm2+UU5HGq8QFtnT/jOSSqSwhLsqEAj7BOt9sHGOAVMV9XFwEPAv4lI9iUdVZ9S1QpVrSgoiM/DcGPiWV+f8nZtI9fNzieeTzbMK/T/81J92k6HhRJJYakDpg14XwycDNVHRFKBHODcINuGam8Ecl2MwLGCjuFOs50FUNXtwEFgTgT7ZYwZRVUnz9PU3h3Xp8EA5hVmAbD3VKvHmcSuSArLNqDUXa2Vjn8yfkNAnw3Ave71KuA1VVXXvtpd0TUDKAXeCxXTbbPJxcDFfGmwMUSkwF0MgIjMdGMcivwrMMaMhs21/lPQ18XhjZEDFeWOITszlX2n7IgllNRwHVS1R0QeBDYCPuD7qlolIo8Blaq6AXgaeFZEavEfqax221aJyPPAXqAHeEBVewGCxXRDPgysE5FvADtcbEKNAdwAPCYiPUAv8GVVPTf8r8QYMxK21DQyd0oWk7IyvU7lsogIcwuz2W+FJaSwhQVAVV8GXg5o+/MBrzuAu0Js+zjweCQxXfsh/FeNBbYHHUNV/wP4j7A7YYzxzMWuXiqPNPHFT1zhdSpRMW9KFj/ZXkdfn5IS449V9kL83aFkjIk77x05R1dvX1wu4xLMvMJs2rt6OXau3etUYpIVFmPMiNtS00C6L4WlM/LCd44D/VeG2TxLcFZYjDEjbnNNIx+7YgJj0n1epxIVV07JIkWssIRihcUYM6IaWjvZf7o1YU6DAWSm+ZiRP459p+2S42CssBhjRtRbtY1AfC/jEsy8wmw7YgnBCosxZkRtrmkkd2zahysDJ4p5hdnUNV3kfEe316nEHCssxpgRo6psqW3guln5+BLsstz+O/D32x34l7DCYowZMbX1bZw535lQ8yv97Mqw0KywGGNGzOYa//zK9XG+jEswU7IzyR2bxn5bjPISVliMMSNmS20jJXljmTZxrNepRJ2IMG9Kti1GGYQVFmPMiOjq6ePdQ2cT8jRYv3mF2VSfPk9vX+CTRJKbFRZjzIh4/1gT7V29XD87cZ9/NLcwi47uPo6cveB1KjHFCosxZkRsqWnElyJcOzsxlnEJpswm8IOywmKMGRGbaxpYPC2X7Mw0r1MZMbMnjceXIlZYAlhhMcZEXdOFLj440ZLQ8yvgX9plVsE4u5clgBUWY0zUvXWwEVVYVpq48yv9bGmXS0VUWERkuYhUi0itiDwS5PMMEVnvPt8qIiUDPnvUtVeLyK3hYrrHFW8VkRoXMz3cGO7z6SLSJiJ/PNQvwRgTXZsPNJKVmcrC4sRaxiWYeYXZnGzpoLm9y+tUYkbYwuKeJ/8EcBtQBtwjImUB3e4DmlR1NrAWWOO2LcP/COFyYDnwpIj4wsRcA6xV1VKgycUOOcYAa4FfRLrjxpiRoapsrvEv45LqS/yTInOnuKVdbKXjD0Xyt74EqFXVQ6raBawDVgT0WQE8416/ANwiIuLa16lqp6oeBmpdvKAx3TY3uxi4mCvDjIGIrAQOAVWR77oxZiQcarzAyZYOls1J7PmVfnZl2KUiKSxFwPEB7+tcW9A+qtoDtAB5g2wbqj0PaHYxAscKOoaIjAMeBv4qgn0xxoywzQcaALghCeZXAAqyMpg4Lt0m8AeIpLAEW5I08DbTUH2i1T7YGH+F/9RZW5DP/ydBkftFpFJEKhsaGgbraoy5DJtrEncZl2BEhHmFWeyzNcM+FElhqQOmDXhfDJwM1UdEUoEc4Nwg24ZqbwRyXYzAsUKNsRT4WxE5Avxf4E9F5MHAnVDVp1S1QlUrCgqS4zcpY0ZbV08f7yT4Mi7BzJ2STfXpVlvaxYmksGwDSt3VWun4J+M3BPTZANzrXq8CXlNVde2r3RVdM4BS4L1QMd02m1wMXMyXBhtDVZepaomqlgB/D/w/Vf3OEL4DY0yU9C/jkgyXGQ80rzCbzp4+Djfa0i4AqeE6qGqPOwLYCPiA76tqlYg8BlSq6gbgaeBZEanFfxSx2m1bJSLPA3uBHuABVe0FCBbTDfkwsE5EvgHscLEJNYYxJnZsrmnAlyJ8YlbiLuMSzP9cGXae2ZPGe5yN98R/kJBcKioqtLKy0us0jEk4d35nC+m+FF74/Wu9TmVUdfb0Uv7nG/nSjTP5k1vnep3OiBGR7apaEa5f4l9kbowZFU0Xuth9oiXpToMBZKT6mFUw3q4Mc6ywGGOi4sNlXJLk/pVAcwuz7F4WxwqLMSYq3jzQQFZmKguKEn8Zl2D6l3Zpae/2OhXPWWExxlw2VeWNAw3cUFqQFMu4BNM/gW/3s1hhMcZEwb5TrZw538mNVybf/Eq//qVd9tvpMCssxpjLt6m6HoCb5iRvYelf2mWfTeBbYTHGXL7Xq+uZX5TNpOxMr1PxTP/SLvvtVJgVFmPM5Wlp72b70SY+eeUkr1Px3Nwp2VSfsaVdrLAYYy7LmzUN9CncZIWFeYXZdHT3ceRsci/tYoXFGHNZNlXXkzs2jUXTcr1OxXMfXhmW5BP4VliMMcPW16e86S4z9qUEe7JFcimdPB5filhh8ToBY0z82nOyhca2Lj45N3mvBhvIv7TLuKRf2sUKizFm2Dbtb0AkeZ4WGYl5hdl2xOJ1AsaY+LWpup6Fxbnkjc/wOpWYMXeKLe1ihcUYMyxn2zrZVddslxkHmFdoS7tYYTHGDMvr1Q2oYvMrAebZ0i5WWIwxw/PqvjNMzs5g/tTkXM04lEm2tEtkhUVElotItYjUisgjQT7PEJH17vOtIlIy4LNHXXu1iNwaLqaIzHAxalzM9MHGEJElIrLT/dklIp8Z7pdhjIlMR3cvbxxo4JZ5k0mxy4w/QkSYOyW5l3YJW1hExAc8AdwGlAH3iEhZQLf7gCZVnQ2sBda4bcvwP5u+HFgOPCkivjAx1wBrVbUUaHKxQ44B7AEqVHWRG+NfRCR1aF+DMWYo3jl4lvauXj5VNtnrVGLSvMLkXtolkiOWJUCtqh5S1S5gHbAioM8K4Bn3+gXgFhER175OVTtV9TBQ6+IFjem2udnFwMVcOdgYqtquqj2uPRNIzr9JY0bRr/aeYVy6j2tn5XmdSkxK9qVdIiksRcDxAe/rXFvQPu4f+RYgb5BtQ7XnAc0DCsXAsUKNgYgsFZEqYDfw5QHbG2OirK9PeXXfGW68soCMVJ/X6cSkZF/aJZLCEuwEauBRQag+0WofNA9V3aqq5cA1wKMicsna3SJyv4hUikhlQ0NDkFDGmEjsqmumobXTToMNon9pl2S9Az+SwlIHTBvwvhg4GaqPm9/IAc4Nsm2o9kYgd8AcycCxQo3xIVXdB1wA5gfuhKo+paoVqlpRUGCXRxozXK/sPYMvRez+lUH0L+1iRyyhbQNK3dVa6fgn4zcE9NkA3OterwJeU1V17avdFV0zgFLgvVAx3TabXAxczJcGG8PFSAUQkSuAK4EjEX8DxpgheWXvGZaUTCR3bLrXqcS0eYXZ7D9tRyxBufmKB4GNwD7geVWtEpHHRORO1+1pIE9EaoGHgEfctlXA88Be4JfAA6raGyqmi/Uw8JCLledihxwDuB7YJSI7gf8CvqKqjcP7OowxgznSeIGa+jY7DRaBuVOyOdF8MSmXdonoslxVfRl4OaDtzwe87gDuCrHt48DjkcR07YfwXzUW2B50DFV9Fng27E4YYy7bK3vPAFhhiUD/0i77T59n6czkunrO7rw3xkTsl1WnmTsli2kTx3qdSszrX9olGedZrLAYYyJyquUi2482ccdVhV6nEhf6l3ZJxnkWKyzGmIj8YvdpAG5fYIUlEv1Lu9gRizHGhPDy7lPMnZLFrILxXqcSN5J1aRcrLMaYsE63dFBpp8GGrHyqf2mXgw1tXqcyqqywGGPC+sWeU4CdBhuqq4r8jxTYXdficSajywqLMSYsOw02PDMLxjMmzcfuE1ZYjDHmQ2fO22mw4fKlCGVTs6k6aYXFGGM+9Ivdp1C102DDdVVRDlUnzyfVBL4VFmPMoH76gZ0Guxzzi3Jo7+rlcGPyTOBbYTHGhHTsbDvbjzaxYlHgI5hMpD6cwE+ieRYrLMaYkF7ceQIRWLFoqtepxK1ZBePITEthd13y3ChphcUYE5Sq8uKOE3x8Rh5Tc8d4nU7cSvWlMK8wmz12xGKMSXa76lo41HiBzyy202CXyz+B30JfkkzgW2ExxgT14o4TpKemsPyqKV6nEvfmF+VwoauXw2cveJ3KqLDCYoy5RHdvHz/ddZJPzZtMdmaa1+nEvf4J/GQ5HWaFxRhzic01DZy90MVKOw0WFaWTxpORmpI0S7tEVFhEZLmIVItIrYg8EuTzDBFZ7z7fKiIlAz571LVXi8it4WK6Z9hvFZEaFzN9sDFE5FMisl1Edrv/3jzcL8MY4/f8tjomjkvnxjkFXqeSEPon8JPlkuOwhUVEfMATwG1AGXCPiJQFdLsPaFLV2cBaYI3btgxYDZQDy4EnRcQXJuYaYK2qlgJNLnbIMYBG4NdU9SrgXuwxxcZclobWTl7dd4Zfv7qI9FQ7qREt84uyqTp5Pikm8CP5qVkC1KrqIVXtAtYBKwL6rACeca9fAG4REXHt61S1U1UPA7UuXtCYbpubXQxczJWDjaGqO1T1pGuvAjJFJCPSL8AY81H/8X4dPX3K3ddM8zqVhLKgOJe2zh4ONSb+BH4khaUIOD7gfZ1rC9pHVXuAFiBvkG1DtecBzS5G4Fihxhjo14EdqtoZwX4ZYwKoKuu3HeeakgnMnpTldToJ5erpuQDsONbkcSYjL5LCIkHaAo/lQvWJVnvYPESkHP/psS8F6YeI3C8ilSJS2dDQEKyLMUlv6+FzHG68wN3XTPc6lYQzM388WZmp7Dje7HUqIy6SwlIHDDwmLgZOhuojIqlADnBukG1DtTcCuS5G4FihxkBEioH/Ar6oqgeD7YSqPqWqFapaUVBgE5LGBLN+23GyMlJtifwRkJIiLJqWy45jVlgAtgGl7mqtdPyT8RsC+mzAP3EOsAp4TVXVta92V3TNAEqB90LFdNtscjFwMV8abAwRyQV+Djyqqm8NZeeNMf+jpb2bl3efYsXiqYxJ93mdTkJaPH0C1afPc6GzJ3znOBa2sLj5jAeBjcA+4HlVrRKRx0TkTtftaSBPRGqBh4BH3LZVwPPAXuCXwAOq2hsqpov1MPCQi5XnYoccw8WZDfyZiOx0fyYN8/swJmk9X3mczp4+7llip8FGyuLpufQpfJDg97OI/yAhuVRUVGhlZaXXaRgTM3r7lJu+tYnC7DE8/+VPeJ1Owmq60MXiv36Fry2/kq/cNNvrdIZMRLarakW4fnaRujGG1/bXc/zcRX7ruhKvU0loE8alMzN/XMLPs1hhMcbwzNtHKMzJ5NNlk71OJeEtmu6fwE/ks0VWWIxJcjVnWtlS28gXPn4FqT77J2GkLZ4+gca2TuqaLnqdyoixnyJjktwP3j5CemqKTdqPksXT3I2SCXw/ixUWY5JYfWsHL2yv49evLmLiuHSv00kKc6dkMTbdx/Yj57xOZcRYYTEmif3grSP09PbxpRtmeZ1K0kj1pfCxKyaw9bAVFmNMgjnf0c2P3znK7VcVUpI/zut0ksqSkolUn2mlub3L61RGhBUWY5LUc+8eo7Wzhy/faEcro23pzDxUYduRxFyQ0gqLMUmoo7uXp7cc5oY5Bcx3j801o2dBcQ7pqSlsPXTW61RGhBUWY5LQj989SmNbJw/cZEcrXshM87F4Wi7vJegEvhUWY5LMhc4ennz9IMtK81k6M/CRRma0LJ0xkT0nWmjt6PY6laizwmJMkvnh20c4d6GLhz41x+tUktrSmXn0KWw/mnjzLFZYjEkiLRe7+Zc3DvK/5k1i8fQJXqeT1BZPzyU1RRLysmMrLMYkke++cZDzHT181Y5WPDc2PZXF03N5q7bR61SizgqLMUni+Ll2nt5ymJWLplI+1a4EiwXLSgvYfaKFcxcS634WKyzGJIm/+cV+fCI8fNtcr1MxzrLSfFRhS4IdtVhhMSYJvHf4HD/ffYov3zhrk+uOAAAOyElEQVSLwpwxXqdjnAXFueSMSWPzgQavU4mqiAqLiCwXkWoRqRWRR4J8niEi693nW0WkZMBnj7r2ahG5NVxMEZnhYtS4mOmDjSEieSKySUTaROQ7w/0ijElUvX3KYz+rojAnk/tvmOl1OmYAX4pw/ex83qxpSKjns4QtLCLiA54AbgPKgHtEpCyg231Ak6rOBtYCa9y2ZcBqoBxYDjwpIr4wMdcAa1W1FGhysUOOAXQAfwb88RD33Zik8IO3DrPnxHm+fsc8xqT7vE7HBFhWms+Z853U1Ld5nUrURHLEsgSoVdVDqtoFrANWBPRZATzjXr8A3CIi4trXqWqnqh4Gal28oDHdNje7GLiYKwcbQ1UvqOoW/AXGGDPA8XPt/N2vDnDL3EnccVWh1+mYIJbNKQDg9ep6jzOJnkgKSxFwfMD7OtcWtI+q9gAtQN4g24ZqzwOaXYzAsUKNERERuV9EKkWksqEhsc5nGhOMqvL1F/eQIvDXK+fj/73NxJqi3DHMnZLFK3vPeJ1K1ERSWIL9NAaeDAzVJ1rtkeYRkqo+paoVqlpRUFAQ6WbGxK0Xttfx5oEG/uTWK5maaxP2sezW8ilUHm2isa3T61SiIpLCUgdMG/C+GDgZqo+IpAI5wLlBtg3V3gjkuhiBY4UawxgT4EjjBf5yQxVLZkzkNz9R4nU6JoxPl09GFf57X2IctURSWLYBpe5qrXT8k/EbAvpsAO51r1cBr6n/EocNwGp3RdcMoBR4L1RMt80mFwMX86UwYxhjBuju7eP/rN+JL0X4+7sX4UuxU2Cxrqwwm+IJY9hYlRiFJTVcB1XtEZEHgY2AD/i+qlaJyGNApapuAJ4GnhWRWvxHEavdtlUi8jywF+gBHlDVXoBgMd2QDwPrROQbwA4Xm1BjuFhHgGwgXURWAp9W1b3D/VKMiWd//+oBdh1v5onPXW2nwOKEiPDpsin8eOtR2jp7GJ8R9p/mmCbJ+Et/RUWFVlZWep2GMVH3q6rT3P/sdu6umMaaVQu8TscMwdZDZ7n7qXf5h9WLWLEo8Pqo2CAi21W1Ilw/u/PemARRc6aVr67fycLiHP5qRbnX6ZghuqZkIoU5mby0M3AKO/5YYTEmATS3d/F7P6pkTHoq3/3Nj5GZZjdCxpuUFGHFoiLeONDA2Ti/OswKizFxrr2rh9/54TZONnfwz1+42tYCi2OfvbqI3j7lZx+c8jqVy2KFxZg41t3bx1eee5+dx5v5h9WLuKZkotcpmcswZ3IWZYXZvLC9zutULosVFmPiVHdvH19dv5PXqxv4xsqruM2WbEkIq5dMY/eJFnYdb/Y6lWGzwmJMHOrs6eUrz73Pzz44xaO3zeVzS6d7nZKJks8sLmJcuo8fvXPU61SGzQqLMXGmrbOH332mklf2nuGxFeV86cZZXqdkoigrM43PXl3MTz84GbdPlrTCYkwcOX6unVX//DZvHzzL365awBdtuZaE9MVPXEFXTx8/fPuI16kMixUWY+LEu4fOsvKJtzjZfJEf/vY1/EbFtPAbmbhUOjmLW8sn84O3DtNysdvrdIbMCosxMa67t4+/+1U19/zru+SMSePFB65jWamt0J3o/vCWUlo7evjhW0e8TmXIrLAYE8P2nTrPqu++wz+9Vsuqq4v56R9cz8yC8V6nZUZB+dQcPlU2me9tPkR9a3w9x9AKizEx6EJnD4//fC//+5+2cPxcO/90z2K+eddCxsX54oRmaP709nl09vTxNy/v9zqVIbGfUmNiSEd3L/+29RhPvn6QxrZO7lkyja/dOpcJ49K9Ts14YEb+OH7vhhk8sekgqyqKuXZWvtcpRcQKizExoKW9m59sP873Nh/m9PkOPjEzj6e++DGunj7B69SMxx745Gxe3n2ar67fyct/uIy88RlepxSWFRZjPKKq7KprYf2247y44wQXu3tZUjKRb//GQq6dHR+/mZqRNzY9lX+6ZzGfffJt/u/6nXz/t64hzRfbsxhWWIwZRT29fXxwooVX9p7hZx+c5Pi5i2SkprByURFfvPYKyqfmeJ2iiUHzi3L465XlPPwfu/njn+zi7+5aSGoMF5eICouILAf+Af/THr+nqn8T8HkG8CPgY8BZ4G5VPeI+exS4D+gF/lBVNw4W0z3CeB0wEXgf+E1V7RrOGMZ4raO7l32nzvNBXQtvH2zk7YNnae3owZciXDc7nz+8uZRPl08hZ0ya16maGHf3NdNpbOvimxurOX+xm79fvThmf27CFhYR8QFPAJ8C6oBtIrIh4NG/9wFNqjpbRFYDa4C7RaQM/yOEy4GpwKsiMsdtEyrmGmCtqq4Tke+62P881DH6H4FszGi42NXLkbMXONJ4gUON/v/uPXWe6tOt9PT5n9JalDuG2+cXcn1pPtfPzrcJ+ZH03HPw9a/DsWMwfTo8/jh8/vNeZ3XZHvjkbHLGpPEXG6r41Lff4M/+dxm3X1WIL0W8Tu0jIjliWQLUquohABFZB6zA/xz7fiuAv3SvXwC+IyLi2tepaidw2D2vfonrd0lMEdkH3Ax8zvV5xsX952GM8U6E34FJYqpKd6/S3dtHd28fXT19dLn/XujspbWzm9aOHto6emjt6Kats4eWi93Ut3ZSf76T+tYO6ls7ae3o+Ujc/PEZzCvM4v4bZrKgOIerinOZmpOJ/0fWjKjnnoP774f2dv/7o0f97yEhissXPn4FC4pz+NoLH/AH/76Db/2qml9bMJXrZudz5ZQsJsbALyyRFJYi4PiA93XA0lB9VLVHRFqAPNf+bsC2/Q9zDhYzD2hW1Z4g/YczRlTVnGnlj36y68P3qh/9XPlowyWfX9I/8PPAlksNecwwY1wy4mVuf7nfySXpRHs8/PMc3b36YREZqsy0FAqyMpiUlUnppCyum53P5OxMpk8cy4z8cVyRN5aszNg8RZEUvv71/ykq/drb/e0JUFgAFhTn8rM/uJ6NVWf40TtHePL1Wr6zqRaAMWk+csakkZWZii9FEBFSBEQgRYRlpfn8ya1zRzS/SApLsF+xAv//DtUnVHuwWafB+g9njI8mKHI/cD/A9OnDW2LclyLkBfw2EPgbaGAyl/6CKoN+HmxnLu0TJkaY/mHeDnmfhrr9peMP7Tu53O8jzZfy4Z/01BTSffLh6zRfCunu9dh0H1mZ/v9BszJTGZ+RyvjMVDJS7bG/Me3YsaG1x6lUXwp3LCjkjgWFNLd3sfN4M7X1bZxu6aDlov/ourfP/6uWqtKn0Kc6KjfZRjJCHTBwtbti4GSIPnUikgrkAOfCbBusvRHIFZFUd9QysP9wxviQqj4FPAVQUVER/tAgiJkF4/nBby8J39EY453p0/2nv4K1J6jcsencdOUkbrpyktepAJEt6bINKBWRGSKSjn+ifENAnw3Ave71KuA19Z/D2ACsFpEMd7VXKfBeqJhum00uBi7mS8McwxiTjB5/HMaO/Wjb2LH+djMqwh6xuPmMB4GN+C8N/r6qVonIY0Clqm4AngaedRPn5/AXCly/5/FP9PcAD/RfrRUsphvyYWCdiHwD2OFiM5wxjDFJqH8eJQGvCosXEsmEcaKpqKjQyspKr9Mwxpi4IiLbVbUiXL/YvXXTGGNMXLLCYowxJqqssBhjjIkqKyzGGGOiygqLMcaYqLLCYowxJqqssBhjjIkqKyzGGGOiKilvkBSRBiDIYkIRy8e/rlmySLb9BdvnZGH7PDRXqGpBuE5JWVgul4hURnL3aaJItv0F2+dkYfs8MuxUmDHGmKiywmKMMSaqrLAMz1NeJzDKkm1/wfY5Wdg+jwCbYzHGGBNVdsRijDEmqqywhCAiy0WkWkRqReSRIJ9niMh69/lWESkZ/SyjK4J9fkhE9orIByLy3yJyhRd5RlO4fR7Qb5WIqIjE/RVEkeyziPyG+7uuEpF/G+0coy2Cn+3pIrJJRHa4n+/bvcgzWkTk+yJSLyJ7QnwuIvKP7vv4QESujmoCqmp/Av7gf6rlQWAmkA7sAsoC+nwF+K57vRpY73Xeo7DPnwTGute/nwz77PplAW8C7wIVXuc9Cn/Ppfif3jrBvZ/kdd6jsM9PAb/vXpcBR7zO+zL3+QbgamBPiM9vB34BCPBxYGs0x7cjluCWALWqekhVu4B1wIqAPiuAZ9zrF4BbRERGMcdoC7vPqrpJVdvd23eB4lHOMdoi+XsG+Gvgb4GO0UxuhESyz78HPKGqTQCqWj/KOUZbJPusQLZ7nQOcHMX8ok5V38T/CPdQVgA/Ur93gVwRKYzW+FZYgisCjg94X+fagvZR1R6gBcgblexGRiT7PNB9+H/jiWdh91lEFgPTVPVno5nYCIrk73kOMEdE3hKRd0Vk+ahlNzIi2ee/BL4gInXAy8AfjE5qnhnq/+9DkhqtQAkm2JFH4OVzkfSJJxHvj4h8AagAbhzRjEbeoPssIinAWuC3RiuhURDJ33Mq/tNhN+E/Kt0sIvNVtXmEcxspkezzPcAPVfXvROQTwLNun/tGPj1PjOi/X3bEElwdMG3A+2IuPTT+sI+IpOI/fB7s0DPWRbLPiMj/Ar4O3KmqnaOU20gJt89ZwHzgdRE5gv9c9IY4n8CP9Gf7JVXtVtXDQDX+QhOvItnn+4DnAVT1HSAT/5paiSqi/9+HywpLcNuAUhGZISLp+CfnNwT02QDc616vAl5TNysWp8Luszst9C/4i0q8n3eHMPusqi2qmq+qJapagn9e6U5VrfQm3aiI5Gf7RfwXaiAi+fhPjR0a1SyjK5J9PgbcAiAi8/AXloZRzXJ0bQC+6K4O+zjQoqqnohXcToUFoao9IvIgsBH/FSXfV9UqEXkMqFTVDcDT+A+Xa/Efqaz2LuPLF+E+fxMYD/zEXadwTFXv9CzpyxThPieUCPd5I/BpEdkL9AJ/oqpnvcv68kS4z38E/KuIfBX/KaHfiudfFEXk3/Gfysx380Z/AaQBqOp38c8j3Q7UAu3Ab0d1/Dj+7owxxsQgOxVmjDEmqqywGGOMiSorLMYYY6LKCosxxpiossJijDEmqqywGGOMiSorLMYYY6LKCosxxpio+v8lT/kaBgfKQwAAAABJRU5ErkJggg==\n", "text/plain": [ "
" ] }, "metadata": { "needs_background": "light" }, "output_type": "display_data" }, { "data": { "image/png": "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\n", "text/plain": [ "
" ] }, "metadata": { "needs_background": "light" }, "output_type": "display_data" }, { "data": { "image/png": "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\n", "text/plain": [ "
" ] }, "metadata": { "needs_background": "light" }, "output_type": "display_data" }, { "data": { "image/png": "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\n", "text/plain": [ "
" ] }, "metadata": { "needs_background": "light" }, "output_type": "display_data" }, { "data": { "image/png": "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\n", "text/plain": [ "
" ] }, "metadata": { "needs_background": "light" }, "output_type": "display_data" }, { "data": { "image/png": "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\n", "text/plain": [ "
" ] }, "metadata": { "needs_background": "light" }, "output_type": "display_data" }, { "data": { "image/png": "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\n", "text/plain": [ "
" ] }, "metadata": { "needs_background": "light" }, "output_type": "display_data" }, { "data": { "image/png": "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\n", "text/plain": [ "
" ] }, "metadata": { "needs_background": "light" }, "output_type": "display_data" }, { "data": { "image/png": "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\n", "text/plain": [ "
" ] }, "metadata": { "needs_background": "light" }, "output_type": "display_data" }, { "data": { "image/png": "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\n", "text/plain": [ "
" ] }, "metadata": { "needs_background": "light" }, "output_type": "display_data" }, { "data": { "image/png": "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\n", "text/plain": [ "
" ] }, "metadata": { "needs_background": "light" }, "output_type": "display_data" }, { "data": { "image/png": "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\n", "text/plain": [ "
" ] }, "metadata": { "needs_background": "light" }, "output_type": "display_data" }, { "data": { "image/png": "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\n", "text/plain": [ "
" ] }, "metadata": { "needs_background": "light" }, "output_type": "display_data" }, { "data": { "image/png": "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\n", "text/plain": [ "
" ] }, "metadata": { "needs_background": "light" }, "output_type": "display_data" }, { "data": { "image/png": "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\n", "text/plain": [ "
" ] }, "metadata": { "needs_background": "light" }, "output_type": "display_data" }, { "data": { "image/png": "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\n", "text/plain": [ "
" ] }, "metadata": { "needs_background": "light" }, "output_type": "display_data" }, { "data": { "image/png": "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\n", "text/plain": [ "
" ] }, "metadata": { "needs_background": "light" }, "output_type": "display_data" }, { "data": { "image/png": "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\n", "text/plain": [ "
" ] }, "metadata": { "needs_background": "light" }, "output_type": "display_data" }, { "data": { "image/png": "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\n", "text/plain": [ "
" ] }, "metadata": { "needs_background": "light" }, "output_type": "display_data" }, { "data": { "image/png": "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\n", "text/plain": [ "
" ] }, "metadata": { "needs_background": "light" }, "output_type": "display_data" }, { "data": { "image/png": "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\n", "text/plain": [ "
" ] }, "metadata": { "needs_background": "light" }, "output_type": "display_data" }, { "data": { "image/png": "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\n", "text/plain": [ "
" ] }, "metadata": { "needs_background": "light" }, "output_type": "display_data" }, { "data": { "image/png": "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\n", "text/plain": [ "
" ] }, "metadata": { "needs_background": "light" }, "output_type": "display_data" }, { "data": { "image/png": "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\n", "text/plain": [ "
" ] }, "metadata": { "needs_background": "light" }, "output_type": "display_data" }, { "data": { "image/png": "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\n", "text/plain": [ "
" ] }, "metadata": { "needs_background": "light" }, "output_type": "display_data" }, { "data": { "image/png": "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\n", "text/plain": [ "
" ] }, "metadata": { "needs_background": "light" }, "output_type": "display_data" }, { "data": { "image/png": "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\n", "text/plain": [ "
" ] }, "metadata": { "needs_background": "light" }, "output_type": "display_data" }, { "data": { "image/png": "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\n", "text/plain": [ "
" ] }, "metadata": { "needs_background": "light" }, "output_type": "display_data" }, { "data": { "image/png": "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\n", "text/plain": [ "
" ] }, "metadata": { "needs_background": "light" }, "output_type": "display_data" }, { "data": { "image/png": "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\n", "text/plain": [ "
" ] }, "metadata": { "needs_background": "light" }, "output_type": "display_data" }, { "data": { "image/png": "iVBORw0KGgoAAAANSUhEUgAAAXcAAAEDCAYAAADOc0QpAAAABHNCSVQICAgIfAhkiAAAAAlwSFlzAAALEgAACxIB0t1+/AAAADl0RVh0U29mdHdhcmUAbWF0cGxvdGxpYiB2ZXJzaW9uIDMuMC4zLCBodHRwOi8vbWF0cGxvdGxpYi5vcmcvnQurowAAIABJREFUeJzt3Xt0XOV57/Hvo/vFsiRLsi3JlgXG3C8GFAKkSSikDaEU2oY0pCSFlpaVnKZJV9vTFZpT2tKT05N2tT0nJU3qJixImiYEmpO4hJQ2FwKk3GQwvmBjS2Bh2ZY1lq27dZ3n/DEzjhAjz0iamT2X32etWZ6Z/c7ez5bkR6/e/ez3NXdHRETyS1HQAYiISOopuYuI5CEldxGRPKTkLiKSh5TcRUTykJK7iEgeCjS5m9n9ZtZvZrtStL9/N7NBM3t03vsfN7MuM3Mza0zFsUREslnQPfcHgOtTuL+/Bj4S5/2fAO8BelJ4LBGRrBVocnf3J4Hjc98zs43RHvg2M3vKzM5dxP5+AIzEef8ldz+w7IBFRHJESdABxLEF+Ki77zeztwP/AFwbcEwiIjklq5K7ma0ArgYeNrPY2+XRbb8C3BvnY4fc/b2ZiVBEJDdkVXInMkw06O6b529w928B38p8SCIiuSfoC6pv4u7DwOtm9gEAi7gk4LBERHJO0KWQXweeAc4xs14zuxO4DbjTzF4GdgM3L2J/TwEPA9dF9/fe6PufMLNeYB2ww8y+lOpzERHJJqYpf0VE8k9WDcuIiEhqBHZBtbGx0dvb24M6vIhITtq2bdsxd29K1C6w5N7e3k5nZ2dQhxcRyUlmltSd9hqWERHJQ0ruIiJ5KOnkbmbFZvbS/BkXo9vKzeyh6MyLz5lZeyqDFBGRxVlMz/2TwJ4Ftt0JnHD3s4C/Az673MBERGTpkkruZrYO+AVgoZt/bgYejD5/hMhNRLZAWxERSbNke+7/B/gjILzA9lbgIIC7zwBDQMP8RmZ2l5l1mllnKBRaQrgiIpKMhMndzG4E+t192+maxXnvLbe+uvsWd+9w946mpoRlmiIiskTJ9NzfAdxkZgeAbwDXmtk/z2vTC6wHMLMSoJZ5i3CIiCzG5Mws3+w8yAM/eZ0TY1NBh5NzEt7E5O53A3cDmNk1wB+6+4fnNdsK3E5kErBbgB+6Jq0RkSUam5zh1770HC8fHATgCz/u5pGPXs36VVUBR5Y7llznbmb3mtlN0ZdfBhrMrAv4feBTqQhORArTXzz6CrsODXHfr13Kd37nHZycmuUT33iJcFh9xmQtavoBd38CeCL6/J45708AH0hlYCJSmHb0DvJQ50F++51ncuPFLQDc84sX8IcPv8y/7+7jhouaA44wN+gOVRHJKl94opuVFaX87rVnnXrvly9tZUNDFV966rUAI8stSu4ikjV6T4zz+O4+PnRFGzUVpafeLy4yfuPqdl58Y5Bdh4YCjDB3KLmLSNb49kuHCDt8+Mq2t2y7eXMrJUXGozuOBBBZ7lFyF5Gs8eiOI3RsqGdd/VurYuqry3jHWY18d+dhVIyXmJK7iGSFrv5R9vaN8AsXL3zB9IaL1nLw+En29o1kMLLcpOQuIlnhezuPYAbvu3Dh5P6usyN3tj+9/1imwspZSu4ikhWe2BfiotZa1tZWLNimubaSjU3VPNWl5J6IkruIBG54YprtBwd516bEc069c1MTz78+wNTMQvMYCii5i0gWeKZ7gNmw885NjQnbvq19FRPTYV45MpyByHKXkruIBO7p/ceoKivm0rb6hG0v21AHwIs9J9IdVk5TcheRwD3z2gBvP2MVZSWJU1JzbSXNtRW8FJ1UTOJTcheRQA2OT9HVP0pH+6qkP3NZW7167gkouYtIoLZFk3THhsRDMjGXttVxaPAkR4cn0hVWzlNyF5FAbes5QUmRcfG6uqQ/c2lbpO3OXs0zsxAldxEJVGfPCS5oWUllWXHSnzln7UrMUMXMaSi5i0hgpmfDvHxwkMs3JD/eDrCivIQNq6rYo+S+oGQWyK4ws+fN7GUz221mfx6nzR1mFjKz7dHHb6UnXBHJJ68cHmZyJszlixhvjzm/ZaV67qeRTM99ErjW3S8BNgPXm9mVcdo95O6bo48vpTRKEclLO3oj5Yyb25Ifb485b+1KegbGGZ2cSXVYeSFhcveI0ejL0uhD822KyLLtPDTEquoyWk4zn8xCzm9ZCcBe9d7jSmrM3cyKzWw70A/8p7s/F6fZ+81sh5k9YmbrF9jPXWbWaWadoVBoGWGLSD7YdWiYC1pWYmaL/ux5zZHkrnH3+JJK7u4+6+6bgXXAFWZ24bwm/wa0u/vFwPeBBxfYzxZ373D3jqamxBMEiUj+mpyZZd/RES5srV3S55trK1hRXkJX/2jixgVoUdUy7j4IPAFcP+/9AXefjL78J+DylEQnInnr1b4RZsLORUtM7mbGxtUr6AopuceTTLVMk5nVRZ9XAu8B9s5rM3d2/ZuAPakMUkTyz65DkeGUC1uWltwBNjZV090/lqqQ8koyPfdm4EdmtgN4gciY+6Nmdq+Z3RRt84lomeTLwCeAO9ITrojki12Hh1hZUcL6VZVL3sdZq1fQNzzByMR0CiPLDyWJGrj7DuDSOO/fM+f53cDdqQ1NRPLZrkNDXNhau6SLqTEbm1YA0B0aY/P6xZdT5jPdoSoiGTc9G2bvkaVfTI05a3U0ueui6lsouYtIxnWHRpmaDXNBtFZ9qdpWVVFabLqoGoeSu4hk3Kt9IwCcs7ZmWfspLS5iQ0O1eu5xKLmLSMbt7RuhpMg4s3HFsvd1VpPKIeNRcheRjNvXN8KZTdVJLauXyJlN1fQMjDMzG05BZPlDyV1EMm5v3wjnrF3eeHtMe0M1s2Hn8KBWZZpLyV1EMmpkYppDgyc5d5nj7TFtDVUA9BzXzUxzKbmLSEbtOxoZHz97TWqS+4Zocj8wMJ6S/eULJXcRyahYpUyqeu5raiooKynijQH13OdScheRjNp3dISqsmJa65Y+7cBcRUVG26oqetRzfxMldxHJqL19w5y9poaioqVPOzDfhlVVvHFcyX0uJXcRyRh359W+kZQNycS0NUSSu7sWiYtRcheRjAmNTnJifDplF1Nj2huqGZ+aJTQ6mbhxgVByF5GM2dcXqZRZ7rQD88XKId/QuPspSu4ikjFd/ZFKmU2rlz/twFwbVkVr3ZXcT0lmJaYKM3vezF6OLsjx53HalJvZQ2bWZWbPmVl7OoIVkdzWHRqjpqKEpprylO53XX0VRQY9uqh6SjI990ngWne/BNgMXG9mV85rcydwwt3PAv4O+GxqwxSRfNDVP8rGphXLWqAjnrKSIpprK+lRrfspCZO7R8SmXCuNPuZfkr4ZeDD6/BHgOkv1d09Ecl53aPTUAhuptqFBte5zJTXmbmbFZrYd6Ceyhupz85q0AgcB3H0GGAIaUhmoiOS24Ylp+kcmTy2Nl2rr6is5NHgyLfvORUkld3efdffNwDrgCjO7cF6TeL30txScmtldZtZpZp2hUGjx0YpIzootqJGunvu6+ipCI5NMTM+mZf+5ZlHVMu4+CDwBXD9vUy+wHsDMSoBa4Hicz29x9w5372hqalpSwCKSm7qiyX1jU3Va9h+bzuCweu9ActUyTWZWF31eCbwH2Duv2Vbg9ujzW4Afum4VE5E5ukNjlBZH5oFJh9b6SHLX0ExESRJtmoEHzayYyC+Db7r7o2Z2L9Dp7luBLwNfNbMuIj32W9MWsYjkpK7+UdobqikpTs/tNetiyf2EkjskkdzdfQdwaZz375nzfAL4QGpDE5F88lpoNOV3ps61dmUFxUWmnnuU7lAVkbSbmgnTc3w8bZUyACXFRaxdWUGveu6AkruIZEDPwBizYU9bpUxMa32lhmWilNxFJO260lwGGbOuTrXuMUruIpJ23aFIcj8zTWWQMa31lRwZOsn0bDitx8kFSu4iknZd/aO01lVSVZZMgd7StdZVEnboG5pI63FygZK7iKRdd2gs7b12iNylCqp1ByV3EUmzcNjTOmHYXK2qdT9FyV1E0urI8ATjU7NpLYOMaa6tAFA5JEruIpJm6Z4wbK6K0mJW15RzaFBT/yq5i0ha/XTCsPQnd4jWumvMXcldRNKrOzRKbWUpjSvKMnK81rpKDcug5C4iadbVH7mYmqnF2VrrKzkyOEE4XNgT0yq5i0hadYfG0jaHezwttZVMzYYZGJvK2DGzkZK7iKTN0Pg0x0YnM3IxNSZWMVPoNzIpuYtI2nSFMnsxFaAltiLTUGGPuyu5i0jaZLIMMmZttOd+pMArZpJZZm+9mf3IzPaY2W4z+2ScNteY2ZCZbY8+7om3LxEpLN2hUcpKik5NC5AJDdVllJUUcaTAh2WSmcVnBvgDd3/RzGqAbWb2n+7+yrx2T7n7jakPUURyVVf/KGc2VlNclJlKGQAzo7m2gsMFntwT9tzd/Yi7vxh9PgLsAVrTHZiI5L7u0GhGx9tjmmsrNCyzmMZm1k5kPdXn4my+ysxeNrPvmdkFC3z+LjPrNLPOUCi06GBFJHdMzszyxvFxNmZwvD2mpbay4Idlkk7uZrYC+Ffg99x9eN7mF4EN7n4J8PfAt+Ptw923uHuHu3c0NTUtNWYRyQEHjo0TdjJa4x7TXFdB3/AEswV8I1NSyd3MSokk9q+5+7fmb3f3YXcfjT5/DCg1s8aURioiOaU7gDLImObaSmbDTmhkMuPHzhbJVMsY8GVgj7v/7QJt1kbbYWZXRPc7kMpARSS3xMogM7FIx3wtdZFyyEKudU+mWuYdwEeAnWa2PfreHwNtAO7+ReAW4GNmNgOcBG5198L9e0hE6A5lZmm9eJprIzcyHRmciGaqwpPwq+7uTwOnrWNy9/uA+1IVlIjkvq7QaCAXUyFyQRXgSAH33HWHqoikXDjsdPdndsKwuVZWllBVVszhwcKtmFFyF5GU6xue4OR0ZpbWiyd2I1PfsHruIiIpE6uUyeScMvO11FWq5y4ikkqZXlovnrUrKzTmLiKSSt2hUVZWlGRsab14musq6R+ZZHo2HFgMQVJyF5GU6+4fY2MGl9aLp6W2Anc4OlyYQzNK7iKSct2hUc4KcEgGIj13oGDnmFFyF5GUGp6Ypn9kMrAa95iW6KIdhwt0dkgldxFJqe4suJgK6rkruYtISnWHxoBgZoOca0V5CTUVJQU7r7uSu4ikVHdolNJio21V5pbWW0hLbWXBrsik5C4iKdXVP0p7QzUlxcGnl+a6wq11D/6rLyJ5Jail9eJprq2MzAxZgJTcRSRlpmfDvDEwzsbVwY63x7TUVjAwNsXE9GzQoWSckruIpEzPwDgzYc+ennu0YqYQb2RScheRlInNKRPkhGFz/bTWXcn9LcxsvZn9yMz2mNluM/tknDZmZp8zsy4z22Fml6UnXBHJZrHZIM/Msp57IV5UTWb9qxngD9z9RTOrAbaZ2X+6+ytz2rwP2BR9vB34QvRfESkg3aFR1q6sYEV55pfWi6e5gO9STdhzd/cj7v5i9PkIsAdondfsZuArHvEsUGdmzSmPVkSyWnf/aNZcTAWoKC2mobqsIGvdFzXmbmbtwKXAc/M2tQIH57zu5a2/ADCzu8ys08w6Q6HQ4iIVkawWDjv7+0fZtLom6FDepLmuoiDvUk06uZvZCuBfgd9z9+H5m+N8xN/yhvsWd+9w946mpqbFRSoiWe3Q4EnGp2bZtCY7xttjmmsLc0WmpJK7mZUSSexfc/dvxWnSC6yf83odcHj54YlIrtjfPwLA2Wuyq+feUlvB4QK8oJpMtYwBXwb2uPvfLtBsK/Dr0aqZK4Ehdz+SwjhFJMvtOxqplDk7y4ZlWuoqGZmYYWRiOuhQMiqZS9rvAD4C7DSz7dH3/hhoA3D3LwKPATcAXcA48BupD1VEstm+oyOsrimntqo06FDeZO7UvzUV2RVbOiVM7u7+NPHH1Oe2ceB3UhWUiOSe/UdHs25IBt68aEc2xpcuukNVRJYtHHa6+kez7mIqRIZloPAW7VByF5Fl6z1xkpPTs1nZM15dU06RFd6NTEruIrJs+47GKmWyr+deUlzEmpUVBVcOqeQuIsu2L1oGeVaWVcrEtNRVqucuIrJY+49G5pSprczOapTm2sJbkUnJXUSWbd/Rkay8mBrTUhdZSzVS2FcYlNxFZFlmo5Uy2XgxNaaltoKpmTADY1NBh5IxSu4isiwHj48zORPOyoupMaduZCqgi6pK7iKyLLFKmU1Z3XOPJPdCmmNGyV1ElmV/dGm9TVmytF48LXWFt2iHkruILMvevhFa6yqzet6WVdVllJcUFdRdqkruIrIse44Mc17zyqDDOC0zo7m2Qj13EZFkTEzP8lpolPObs3e8PabQbmRScheRJdt3dISwk/U9d4isyKRhGRGRJOw5EllxMxeSe0tdBUeHJ5iZDQcdSkYouYvIku05MkJVWTFtq6qCDiWhlrpKwg5HRyaDDiUjkllm734z6zezXQtsv8bMhsxse/RxT+rDFJFstOfIMOesraGo6LTr+WSF5uiiHUcKZNw9mZ77A8D1Cdo85e6bo497lx+WiGQ7d8+JSpmY2KIdhwtk3D1hcnf3J4HjGYhFRHLI4aEJhidmcia5N9cW1o1MqRpzv8rMXjaz75nZBQs1MrO7zKzTzDpDoVCKDi0iQdhzOHIxNRfKIAFqKkqpqSjRsMwivAhscPdLgL8Hvr1QQ3ff4u4d7t7R1NSUgkOLSFD29kWS+zlrc6PnDpE5ZjQskyR3H3b30ejzx4BSM2tcdmQiktX2HBmhbVUVK8pLgg4laS11hXOX6rKTu5mtNTOLPr8ius+B5e5XRLJb5GJqbgzJxDQX0F2qCX/lmtnXgWuARjPrBf4UKAVw9y8CtwAfM7MZ4CRwqxfSciciBWhscobXB8a4eXNr0KEsyrr6Sk6MTzM2OUN1Dv3FsRQJz87dP5Rg+33AfSmLSESy3u7Dw7jDxetqgw5lUdbVR2626j1xknPW5tZfHYulO1RFZNF29A4CcGFrbiX39fWRWvfeE+MBR5J+Su4ismg7Dw3RXFtBU0150KEsSqznfvC4kruIyFvsPDTERTnWawdoXFFGRWkRvSfy/6KqkruILMrIxDSvhcZyMrmbGevqqzioYRkRkTfbdShy89JFOXYxNWZ9faV67iIi8+06NASQkz13iIy7a8xdRGSeHYeGaK2rpGFFbl1MjVm/qpLhiRmGTk4HHUpaKbmLyKLs7B3M2V47zK11z+/eu5K7iCRt6OQ0BwbGc3a8HWD9qXLI/B53V3IXkaRtPxi5eenS9XUBR7J06wrkRiYldxFJ2raeExQZXJLDyb2uqpQV5SV5XzGj5C4iSXux5wTnrl2Z05NuRWrdK9VzFxEBmA072w8OcvmG+qBDWbZIOaR67iIi7Ds6wujkDJdtyN0hmZhYzz2fZydXcheRpLz4xgkALm9bFXAky7d+VRVjU7McH5sKOpS0SZjczex+M+s3s10LbDcz+5yZdZnZDjO7LPVhikjQtvWcoHFFOetXVQYdyrK1N0TKIQ8MjAUcSfok03N/ALj+NNvfB2yKPu4CvrD8sEQk27zYc4LL2uqIrqqZ09obqwE4cCx/L6omTO7u/iRw/DRNbga+4hHPAnVm1pyqAEUkeAOjkxwYGM+Li6kQuZGpyNRzT6QVODjndW/0PRHJEy8ciI6350lyLyspYl19Fa8fU3I/nXh/o8W9BG1md5lZp5l1hkKhFBxaRDLh2dcGqCwt5uJ1uV8pE9PeWK2eewK9wPo5r9cBh+M1dPct7t7h7h1NTU0pOLSIZMKzrw3Q0V5PWUn+FNid0VDFgWP5Ww6Ziu/UVuDXo1UzVwJD7n4kBfsVkSwwMDrJ3r4RrjyzIehQUqq9sZrRyRmOjeZnOWTCe4jN7OvANUCjmfUCfwqUArj7F4HHgBuALmAc+I10BSsimff865F6inxM7gA9A2M5t9B3MhImd3f/UILtDvxOyiISkazyzGsDVJUVc3EOT/MbzxkNkeT++rExOtpz/8as+fJnAE1E0uKZ7gE62ldRWpxf6aK1vpLiIsvbi6r59d0SkZTqH5lgf/8oV+XZkAxAaXER6+sr8/ZGJiV3EVnQk/uOAfCusxsDjiQ92hur87bWXcldRBb0xKv9NNWUc37zyqBDSYszG1fw+rExwuH8K4dUcheRuGbDzlP7j/Hus5vyYj6ZeDatWcHJ6VkODebf3O5K7iIS1/aDgwydnObdZ+fvDYdnr1kBwP7+kYAjST0ldxGJ68f7QhQZvHNTfo63A5y1ugaAfUdHA44k9ZTcRSSuH7/az6Vt9dRVlQUdStrUVpayZmU5+5XcRaQQHBk6ycu9Q1x77uqgQ0m7TatrNCwjIoXhP3YfBeD6C9cGHEn6bVqzgq7+0byrmFFyF5G3+PddfWxavYKNTSuCDiXtNq2uYXwq/ypmlNxF5E0GRid57vWBgui1Q6TnDtDVn1/j7kruIvIm399zlLAXxpAMwNnRipm9ffk17q7kLiJv8t2dfaxfVZm3d6XOV1tVyrr6SnYfHgo6lJRScheRU/qHJ3h6f4ibL2nN27tS47mwpZbdh4eDDiOllNxF5JTvbD9M2OGXLyusNe4vaFnJ68fGGJmYDjqUlEkquZvZ9Wb2qpl1mdmn4my/w8xCZrY9+vit1IcqIun2ry/2snl9XUFUycx1YWtkIZI9R/Jn3D1hcjezYuDzwPuA84EPmdn5cZo+5O6bo48vpThOEUmz3YeH2Ns3wvsLrNcOcEFr5PrCrkP5M+6eTM/9CqDL3V9z9yngG8DN6Q1LRDLt4c5eSouNGy9uCTqUjFtdU8HqmnJ25dFF1WSSeytwcM7r3uh7873fzHaY2SNmtj7ejszsLjPrNLPOUCi0hHBFJB1GJ2d4ZFsvv3BRM/XV+TuXzOlc0LKSV/LoomoyyT3eJfP59+n+G9Du7hcD3wcejLcjd9/i7h3u3tHUlL/TiIrkmv/3Yi+jkzPcfnV70KEE5qJ1dew7OsLo5EzQoaREMsm9F5jbE18HHJ7bwN0H3H0y+vKfgMtTE56IpJu78+AzPVy8rpbN6+uCDicwl2+oJ+yw/Y3BoENJiWSS+wvAJjM7w8zKgFuBrXMbmFnznJc3AXtSF6KIpNPTXcfo6h/l9qvaC6q2fb5L2+owg86e40GHkhIliRq4+4yZfRx4HCgG7nf33WZ2L9Dp7luBT5jZTcAMcBy4I40xi0gK3ffDLtasLOfGS5oTN85jKytKOWdNDdt6TgQdSkokTO4A7v4Y8Ni89+6Z8/xu4O7UhiYi6fb868d57vXj3HPj+ZSXFAcdTuA62uv59kuHmQ07xUW5/VeM7lAVKWCf+8F+GleU8aEr2oIOJSu8rX0Vo5MzeVE1o+QuUqB+vC/E013H+Oi7N1JZpl47wFUbGwB4qiv3S7WV3EUK0GzY+V/f3UPbqio+ctWGoMPJGqtrKjiveSVP7TsWdCjLpuQuUoD+5fk3ePXoCHe/71yNtc/zrk2NdPYcZ3wqt+vdldxFCsyRoZN89nt7ecdZDQWzIMdivHNTE9OzzjPdA0GHsixK7iIFxN35k2/vYiYc5i9/+eKCrmtfyNvOqKemvITHd/cFHcqyKLmLFJCvPtvD9/f084c/fw5tDVVBh5OVykuK+bnz1/D47qNMz4aDDmfJlNxFCsTO3iH+56N7+NlzmvjNd5wRdDhZ7YaLmhk6Oc1PunL3wqqSu0gBODR4kjsffIHGFWX8za9upijHb9BJt3ee3UhNRQnf2X44ceMspeQukucGx6e44/7nOTk9ywO/eQWrCnRK38UoLynmVy5t5bs7jjAwOpn4A1lIyV0kj4VGJrl1y7P0DIzzjx++nLPX1AQdUs748JUbmJoN81DnwcSNs5CSu0ie6uof5Vf/8Rl6Bsa5/463cfVZjUGHlFM2ranh6o0NPPhfBzg5NRt0OIum5C6Shx7dcZib73ua4ZPT/PNvXcHPbFJiX4pPXreJo8OTPPBfB4IOZdGU3EXyyNHhCT72z9v4+L+8xKY1NTz6iZ/h8g2rgg4rZ739zAauPXc1//BEF31DE0GHsyhK7iJ54NjoJJ/57iu8+69/xA/29vPf33sOD3/0KpprK4MOLef9yY3nMzPr/MHD2wmH568wmr2Sms9dRLLP9GyYZ18b4JudvTy+q4+ZcJhf2tzKJ9+ziQ0N1UGHlzfOaKzmT3/xfD71rZ38j+/s4jO/dGFO3NmbVHI3s+uB/0tkJaYvufv/nre9HPgKkbVTB4APuvuB1IYqUtimZsLs7Rtm+8FBnnvtOE/uCzEyOcPKihJ+7e1tfOSqDWxsWhF0mKnzta/Bpz8Nb7wBbW3wmc/AbbcFEsoH37aenuPjfOGJbk6MTfGXv3IRdVXZXVKaMLmbWTHweeDniCyW/YKZbXX3V+Y0uxM44e5nmdmtwGeBD6YjYJF84e5MzoSZmJ5lcibMyalZToxPcXxsioGxKU6MTdE3PEHPwDgHBsY4eHyc6dnIsMDqmnJuuKiZnz13Ndec00RFaZ7N7Pi1r8Fdd8H4eOR1T0/kNQSS4M2MP3rvOdRVlvLXj7/K0/uP8cG3refa81ZzQXMttVWlGY8pEXM//RiSmV0F/Jm7vzf6+m4Ad//LOW0ej7Z5xsxKgD6gyU+z846ODu/s7Fx0wE/vP8ZfPb437raFjuYsfI4LfuY0X5aFNiX6Wi7+OPE3pjq2Bbdk6Dgp/xosYVg0k3HPTeiJVJUVs6GhmvaGKtobq7motZZL1tfRUluRE0MDS9beHkno823YAAcOZDqaN9nbN8zf/6CL/3il79Qv25qKElZWlFJRWkR5STGxb82pf/np98oM3n/ZOm6/un1Jxzezbe7ekahdMsMyrcDcKv5e4O0LtYkuqD0ENABvmpjBzO4C7gJoa1vasl5lJUU0nOYOu4V+4E/332Dh/yMLf2qhzyzlOLaU45zmQAvu77SfWeg4p4ltwc9k5jin27ikr+npDpPC45SXFFFRWkx5aTEVpUVUlBRTEX1eX1VGfXUZDdVlrKouo6qsOL+T+ELeeGNx72fQuWtX8vnbLmN0cobnXhugOzTKoRMnGZ2cjf7SjtTEx34loarCAAAFBUlEQVT5z/01H+tEZGLlq2SSe7yfrPndkmTa4O5bgC0Q6bkncey3uOKMVVxxxhVL+aiI5Iq2tvg99yV2CtNhRXkJ1523huvOWxN0KHElUwrZC6yf83odMH82nVNtosMytcDxVAQoIgXoM5+BqnlTEldVRd6XpCST3F8ANpnZGWZWBtwKbJ3XZitwe/T5LcAPTzfeLiJyWrfdBlu2RMbYzSL/btkSWLVMLko4LBMdQ/848DiRUsj73X23md0LdLr7VuDLwFfNrItIj/3WdAYtIgXgttuUzJchqTp3d38MeGzee/fMeT4BfCC1oYmIyFJp+gERkTyk5C4ikoeU3EVE8pCSu4hIHlJyFxHJQ0ruIiJ5SMldRCQPJZwVMm0HNgsBcSaPSEoj8yYlKwA658Kgcy4MyznnDe7elKhRYMl9OcysM5kpL/OJzrkw6JwLQybOWcMyIiJ5SMldRCQP5Wpy3xJ0AAHQORcGnXNhSPs55+SYu4iInF6u9txFROQ0lNxFRPJQVid3M7vezF41sy4z+1Sc7eVm9lB0+3Nm1p75KFMriXP+fTN7xcx2mNkPzGxDEHGmUqJzntPuFjNzM8v5srlkztnMfjX6vd5tZv+S6RhTLYmf7TYz+5GZvRT9+b4hiDhTxczuN7N+M9u1wHYzs89Fvx47zOyylAbg7ln5ILLqUzdwJlAGvAycP6/NfwO+GH1+K/BQ0HFn4Jx/FqiKPv9YIZxztF0N8CTwLNARdNwZ+D5vAl4C6qOvVwcddwbOeQvwsejz84EDQce9zHN+F3AZsGuB7TcA3wMMuBJ4LpXHz+ae+xVAl7u/5u5TwDeAm+e1uRl4MPr8EeA6M7MMxphqCc/Z3X/k7uPRl88SWbA8lyXzfQb4C+CvgIlMBpcmyZzzbwOfd/cTAO7en+EYUy2Zc3ZgZfR5LXA4g/GlnLs/SWTZ0YXcDHzFI54F6sysOVXHz+bk3gocnPO6N/pe3DbuPgMMAQ0ZiS49kjnnue4k8ps/lyU8ZzO7FFjv7o9mMrA0Sub7fDZwtpn9xMyeNbPrMxZdeiRzzn8GfNjMeoks6/m7mQktMIv9/74oSa2hGpB4PfD5dZvJtMklSZ+PmX0Y6ADendaI0u+052xmRcDfAXdkKqAMSOb7XEJkaOYaIn+dPWVmF7r7YJpjS5dkzvlDwAPu/jdmdhXw1eg5h9MfXiDSmr+yuefeC6yf83odb/0z7VQbMysh8qfc6f4MynbJnDNm9h7g08BN7j6ZodjSJdE51wAXAk+Y2QEiY5Nbc/yiarI/299x92l3fx14lUiyz1XJnPOdwDcB3P0ZoILIBFv5Kqn/70uVzcn9BWCTmZ1hZmVELphunddmK3B79PktwA89eqUiRyU85+gQxT8SSey5Pg4LCc7Z3YfcvdHd2929nch1hpvcvTOYcFMimZ/tbxO5eI6ZNRIZpnkto1GmVjLn/AZwHYCZnUckuYcyGmVmbQV+PVo1cyUw5O5HUrb3oK8oJ7jafAOwj8hV9k9H37uXyH9uiHzzHwa6gOeBM4OOOQPn/H3gKLA9+tgadMzpPud5bZ8gx6tlkvw+G/C3wCvATuDWoGPOwDmfD/yESCXNduDng455mef7deAIME2kl34n8FHgo3O+x5+Pfj12pvrnWtMPiIjkoWwelhERkSVSchcRyUNK7iIieUjJXUQkDym5i4jkISV3EZE8pOQuIpKH/j8cSnMYzCwLbQAAAABJRU5ErkJggg==\n", "text/plain": [ "
" ] }, "metadata": { "needs_background": "light" }, "output_type": "display_data" }, { "data": { "image/png": "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\n", "text/plain": [ "
" ] }, "metadata": { "needs_background": "light" }, "output_type": "display_data" }, { "data": { "image/png": "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\n", "text/plain": [ "
" ] }, "metadata": { "needs_background": "light" }, "output_type": "display_data" }, { "data": { "image/png": "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\n", "text/plain": [ "
" ] }, "metadata": { "needs_background": "light" }, "output_type": "display_data" }, { "data": { "image/png": "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\n", "text/plain": [ "
" ] }, "metadata": { "needs_background": "light" }, "output_type": "display_data" }, { "data": { "image/png": "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\n", "text/plain": [ "
" ] }, "metadata": { "needs_background": "light" }, "output_type": "display_data" }, { "data": { "image/png": "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\n", "text/plain": [ "
" ] }, "metadata": { "needs_background": "light" }, "output_type": "display_data" }, { "data": { "image/png": "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\n", "text/plain": [ "
" ] }, "metadata": { "needs_background": "light" }, "output_type": "display_data" }, { "data": { "image/png": "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\n", "text/plain": [ "
" ] }, "metadata": { "needs_background": "light" }, "output_type": "display_data" }, { "data": { "image/png": "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\n", "text/plain": [ "
" ] }, "metadata": { "needs_background": "light" }, "output_type": "display_data" }, { "data": { "image/png": "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\n", "text/plain": [ "
" ] }, "metadata": { "needs_background": "light" }, "output_type": "display_data" }, { "data": { "image/png": "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\n", "text/plain": [ "
" ] }, "metadata": { "needs_background": "light" }, "output_type": "display_data" }, { "data": { "image/png": "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\n", "text/plain": [ "
" ] }, "metadata": { "needs_background": "light" }, "output_type": "display_data" }, { "data": { "image/png": "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\n", "text/plain": [ "
" ] }, "metadata": { "needs_background": "light" }, "output_type": "display_data" }, { "data": { "image/png": "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\n", "text/plain": [ "
" ] }, "metadata": { "needs_background": "light" }, "output_type": "display_data" }, { "data": { "image/png": "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\n", "text/plain": [ "
" ] }, "metadata": { "needs_background": "light" }, "output_type": "display_data" }, { "data": { "image/png": "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\n", "text/plain": [ "
" ] }, "metadata": { "needs_background": "light" }, "output_type": "display_data" }, { "data": { "image/png": "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\n", "text/plain": [ "
" ] }, "metadata": { "needs_background": "light" }, "output_type": "display_data" }, { "data": { "image/png": "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\n", "text/plain": [ "
" ] }, "metadata": { "needs_background": "light" }, "output_type": "display_data" }, { "data": { "image/png": "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\n", "text/plain": [ "
" ] }, "metadata": { "needs_background": "light" }, "output_type": "display_data" }, { "data": { "image/png": "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\n", "text/plain": [ "
" ] }, "metadata": { "needs_background": "light" }, "output_type": "display_data" }, { "data": { "image/png": "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\n", "text/plain": [ "
" ] }, "metadata": { "needs_background": "light" }, "output_type": "display_data" }, { "data": { "image/png": "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\n", "text/plain": [ "
" ] }, "metadata": { "needs_background": "light" }, "output_type": "display_data" }, { "data": { "image/png": "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\n", "text/plain": [ "
" ] }, "metadata": { "needs_background": "light" }, "output_type": "display_data" }, { "data": { "image/png": "iVBORw0KGgoAAAANSUhEUgAAAX4AAAEDCAYAAAAyZm/jAAAABHNCSVQICAgIfAhkiAAAAAlwSFlzAAALEgAACxIB0t1+/AAAADl0RVh0U29mdHdhcmUAbWF0cGxvdGxpYiB2ZXJzaW9uIDMuMC4zLCBodHRwOi8vbWF0cGxvdGxpYi5vcmcvnQurowAAIABJREFUeJzt3XlwnfV97/H3V7tkrbZkyUiyZTs2EHaiONsNy01CHHoLbbNBSQttGt+moZ1JlylpZ0KHTOdmkttl2qYh7o2btkMgG0ndBEIphIQmECx227HBtmxLNrZWS7L25Xv/OOeYg6zlSHrOec7yec1oOOfZzveRzEc//Z7f83vM3RERkdyRF3YBIiKSWgp+EZEco+AXEckxCn4RkRyj4BcRyTEKfhGRHJO2wW9mu8ysy8z2BnS8H5rZGTP7/qzlZmZ/aWavmNkvzOwPgvg8EZF0lbbBD3wN2B7g8b4I/MYcy+8AmoGL3P1i4IEAP1NEJO2kbfC7+0+AvvhlZrY52nJ/1syeNLOLlnC8x4ChOVZ9ErjH3Wei23WtpG4RkXSXtsE/j53A77v7W4A/Bv4xgGNuBj5qZm1m9rCZbQngmCIiaasg7AISZWblwDuBb5lZbHFxdN2vAffMsdsJd3//IocuBsbcvTV6nF3Au4OpWkQk/WRM8BP56+SMu185e4W7Pwg8uMzjdgLfib7+LvDPyzyOiEhGyJiuHncfBNrN7MNwbjTOFQEc+nvA/4y+vhZ4JYBjioikLUvX2TnN7H7gOqAWOA3cDTwOfBlYBxQCD7j7XF08cx3vSeAioBzoBT7u7o+YWTVwH7AeOAv8rru/GOzZiIikj7QNfhERSY6M6eoREZFgpOXF3draWm9paQm7DBGRjPHss8/2uHtdItumZfC3tLTQ1tYWdhkiIhnDzI4luu2iwW9mu4D/BXS5+6VzrP8T4La4410M1Ll7n5kdJXK37DQw5e6tiRYmIiLJkUgf/9dYYM4cd/+iu18ZHV//GeDH7h4/1cL10fUKfRGRNLBo8M81Z84CbgXuX1FFIiKSVIGN6jGzMiJ/GXwnbrED/xmdVG3HIvvviM6X09bd3R1UWSIiMkuQwzl/GfjprG6ed7n71cAHgE+Z2TXz7ezuO9291d1b6+oSujAtIiLLEGTw38Ksbh53Pxn9bxeReXC2Bfh5IiKyDIEEv5lVEZnn5t/jlq0ys4rYa+AGIJCnaYmIyPIlMpzz3Jw5ZtZJZM6cQgB3vze62a8C/+nuw3G71gPfjU6hXAB83d1/GFzpIpIrpmecH7z8GmfHpviVqy6grCgtb0HKGGk5V09ra6vrBi4RAXB3/vhbL/Gd5zoBeMuGGu7/xNspKtCMM/HM7NlEh83rOyciae3R/af5znOdfOr6zfzVh6/g2WP9/PNP28MuK6Mp+EUkrf3jE4fZWLuKT793Kx98SxPXbq3jn55sZ3xqOuzSMpaCX0TS1gsdZ3ih4wx3vLOFgvxIXH38f2yk5+w4j/2iK+TqMpeCX0TS1veeP0FxQR6/dnXjuWXvelMta1YV8cO9p0KsLLMp+EUkLbk7j+4/zbu31FFRUnhueX6e8d6L63n8QJe6e5ZJwS8iaWnfyUFOnBnlhkvqz1t3wyX1nB2fou1ofwiVZT4Fv4ikpSdf7QHg+gvXnrfurRtXk2fw8/ZE54+UeAp+EUlLz7T3srluFXUVxeetqywp5M0XVPJMe28IlWU+Bb+IpJ3pGaftaD9v27Rm3m22tazh+eNn1M+/DAp+EUk7+08OMjQ+xds2rp53m9aWGsanZjjw2lAKK8sOCn4RSTs/j3bhvG3j/C3+yxqrAHj5xEBKasomCn4RSTsvdJyhsbqUhqqSebdpqimlqrSQfScV/Eul4BeRtLP3xMC5Fv18zIzLGqvU4l8GBb+IpJXBsUmO9o5wWdPCwQ9wSWMlB08NMTE1k4LKsoeCX0TSyt5oC/7SRVr8ABc3VDI57RzrHV50W3mdgl9E0srLnZHgX6yrB+BNa8sBeLXrbFJryjYKfhFJKy+fGKCxupTVq4oW3XZzXTlm8OppBf9SKPhFJK0cODXExesqE9q2tCif5poyXu3SWP6lUPCLSNoYn5qmvWeYCxvKE95ny9pyDqmrZ0kU/CKSNtp7hpmecbbWVyS8z5vqyznSPczUtEb2JGrR4DezXWbWZWZ751l/nZkNmNkL0a/Pxq3bbmYHzeyQmd0VZOEikn1eifbVLyX4t6ytYGJ6huN9I8kqK+sk0uL/GrB9kW2edPcro1/3AJhZPvAl4APAm4FbzezNKylWRLLbq6eHyM8zNtWtSnifzdFtD3drSGeiFg1+d/8JsJxJr7cBh9z9iLtPAA8ANy/jOCKSIw6eGqJlTRnFBfkJ79OyJhL8GsufuKD6+N9hZi+a2cNmdkl0WSPQEbdNZ3TZnMxsh5m1mVlbd3d3QGWJSCZ5tevskrp5AKrLCqksKeBYr7p6EhVE8D8HbHD3K4C/B74XXW5zbOvzHcTdd7p7q7u31tXVBVCWiGSSsclpjvUOs2WJwW9mtNSu4pj6+BO24uB390F3Pxt9/RBQaGa1RFr4zXGbNgEnV/p5IpKdDnWdZcbhwiUGP8D61WXq6lmCFQe/mTWYmUVfb4sesxfYA2wxs41mVgTcAuxe6eeJSHY60hMJ7tg0DEvRsmYVnf2jTGpIZ0IKFtvAzO4HrgNqzawTuBsoBHD3e4EPAZ80sylgFLjF3R2YMrM7gUeAfGCXu+9LylmISMZr7x7GDDasKVvyvhvWlDE945w8M8qGNYmPCMpViwa/u9+6yPp/AP5hnnUPAQ8trzQRySXtPWe5oKqUksLER/TEtNRGwv5o74iCPwG6c1dE0kJ77wgba5cX2htWR/5KUD9/YhT8IhI6d6e9++yyg7+uopjigjw6+0cDriw7KfhFJHT9I5MMjk2d67JZKjOjsbqUzn4N6UyEgl9EQtfeE5mjZ9Mygx+gsaaUE2rxJ0TBLyKha++JtNSX2+IHaKop5cQZBX8iFPwiErr2nrMU5BlNNaXLPkZjdSk9ZycYm5wOsLLspOAXkdC19wzTvLqMwvzlR1Jj9JeGWv2LU/CLSOjae5Y/lDOmsToypFMjexan4BeRULk7R3uGVx78sRa/gn9RCn4RCdXpwXFGJ6dXdGEXoL6imPw848QZDelcjIJfREJ1JIChnAAF+Xk0VJaoxZ8ABb+IhOpoAEM5Yxo1pDMhCn4RCdXxvhEK842GypIVH6upulQXdxOg4BeRUHX0j9BUU0Z+3lwP7VuaxppSTg+OMaV5+Rek4BeRUHX0jazoxq14DVUlzDh0nx0P5HjZSsEvIqHq6Bth/eqlP3xlLuuqIt1Frw2MBXK8bKXgF5HQDI1N0j8ySXNAwd9QGfnL4ZSCf0EKfhEJTUdf5EJsUC3+hmiLX8G/MAW/iITmeF9kKGdzTTDBX1NWSFFBHqcGFfwLWTT4zWyXmXWZ2d551t9mZi9Fv35mZlfErTtqZi+b2Qtm1hZk4SKS+WIPTmleHczFXbPIsFC1+BeWSIv/a8D2Bda3A9e6++XA54Cds9Zf7+5Xunvr8koUkWzV0TdCRUkBVaWFgR2zoUrBv5hFg9/dfwL0LbD+Z+7eH337NNAUUG0ikuWO943QXFOG2crH8MesqyrhtUHdxLWQoPv4Pw48HPfegf80s2fNbMdCO5rZDjNrM7O27u7ugMsSkXTU0T8a2IXdmIbKEk4PjOPugR43mwQW/GZ2PZHg/9O4xe9y96uBDwCfMrNr5tvf3Xe6e6u7t9bV1QVVloikKXeno28ksP79mIaqEiamZ+gbngj0uNkkkOA3s8uB/wfc7O69seXufjL63y7gu8C2ID5PRDJf99A441MzgY3hj4ndxKWRPfNbcfCb2XrgQeA33P2VuOWrzKwi9hq4AZhzZJCI5J5zQzkDDv76So3lX0zBYhuY2f3AdUCtmXUCdwOFAO5+L/BZYA3wj9ELNFPRETz1wHejywqAr7v7D5NwDiKSgTr6gx3DH7OuKtJ1pGkb5rdo8Lv7rYus/x3gd+ZYfgS44vw9RERev2s3qAnaYmrLi8gzOK2unnnpzl0RCcXxvhHqK4spKcwP9LgF+XmsrShRi38BCn4RCUWQs3LOppu4FqbgF5FQdERv3kqGhsoSjepZgIJfRFJuYmqG1wbHaFKLPxQKfhFJuRNnRnEPbjrm2dZVlXB2fIqhscmkHD/TKfhFJOU6zk3HHOyInpjYvPwa2TM3Bb+IpFzs5q31a5LT4l9bEQt+PXt3Lgp+EUm5jv4RivLzqI8GdND0JK6FKfhFJOU6+0ZprCklLy+46Zjj1VcWA3B6SME/FwW/iKTc8b6RwOfoiVdWVEBFSQGn1eKfk4JfRFKuo38kaRd2Y+orS9THPw8Fv4ik1ODYJGdGJpM2lDNGN3HNT8EvIinVkaTpmGdbW1lMl4J/Tgp+EUmpWPCnosXfNTTOzIwewTibgl9EUio2HXOy5umJqa8sYWrG6dUjGM+j4BeRlOroH6GipICqssKkfk7sSVy6e/d8Cn4RSanjSZyVM965sfwK/vMo+EUkpZI5D3+8Bj10fV4KfhFJmZkZp6N/NGlz9MSrLS/GTPP1zEXBLyIp0312nImpmaTfvAVQmJ9HbXmx7t6dQ0LBb2a7zKzLzPbOs97M7O/M7JCZvWRmV8etu93MXo1+3R5U4SKSeWKzcibrASyzNVSWaL6eOSTa4v8asH2B9R8AtkS/dgBfBjCz1cDdwNuAbcDdZlaz3GJFJLOlagx/TH1lsWbonENCwe/uPwH6FtjkZuBfPeJpoNrM1gHvBx519z537wceZeFfICKSxWJj+Burk9/VA5EhnV1D6uOfLag+/kagI+59Z3TZfMvPY2Y7zKzNzNq6u7sDKktE0snxvhEaKksoKcxPyefVV5bQNzzB+NR0Sj4vUwQV/HNNqu0LLD9/oftOd29199a6urqAyhKRdNLRP0Lz6tS09iHSxw/QpZE9bxBU8HcCzXHvm4CTCywXkRzUkaKbt2LW6iauOQUV/LuB34yO7nk7MODurwGPADeYWU30ou4N0WUikmPGp6Y5NTiW9Fk5473+0HW1+OMVJLKRmd0PXAfUmlknkZE6hQDufi/wEHAjcAgYAX4ruq7PzD4H7Ike6h53X+gisYhkqZNnxnBP/nTM8WLP9NXdu2+UUPC7+62LrHfgU/Os2wXsWnppIpJNjqd4KCdAdVkhRQV5mpd/Ft25KyIp8foDWFJ3cdfMImP5FfxvoOAXkZTo6BuhKD/vXPdLqjRUluji7iwKfhFJiY7+EZpqSsnLm2uUd/Ks1UPXz6PgF5GU6OgbTdkcPfFiLf7IpUgBBb+IpMjxvhHWp7B/P6a+spiRiWmGxqdS/tnpSsEvIkk3MDrJwOhkSm/eiqk/d/eu+vljFPwiknSvj+gJL/hPDaifP0bBLyJJ19mf+jH8MQ166Pp5FPwiknSx6ZjD7OrRWP7XKfhFJOmO941QWVJAVVlhyj+7tCifypIC9fHHUfCLSNJFpmNOfWs/pr6yRC3+OAp+EUm64ymejnm2hirdxBVPwS8iSTUz43T2j7J+TXjBv7ZC0zbEU/CLSFKdHhpjYmom1K6ehqpiuobGmZnR3bug4BeRJDvaExnKuXHNqtBqqK8sYXrG6RlWdw8o+EUkyY71DgOwIcSunno9e/cNFPwiklTtvcMU5hsXVKd+np6Y1+/eVT8/KPhFJMmO9USGcuaneDrmeOfu3h1S8IOCX0SS7GjvcKj9+wC15UXkGZxWix9IMPjNbLuZHTSzQ2Z21xzr/8bMXoh+vWJmZ+LWTcet2x1k8SKS3tydY70jbAg5+Avy86gtL9ZY/qhFH7ZuZvnAl4D3AZ3AHjPb7e77Y9u4+6fjtv994Kq4Q4y6+5XBlSwimaJ7aJzRyWlaasO7sBuju3dfl0iLfxtwyN2PuPsE8ABw8wLb3wrcH0RxIpLZjvZGhnKG3eKHSPDrJq6IRIK/EeiIe98ZXXYeM9sAbAQej1tcYmZtZva0mf3KfB9iZjui27V1d3cnUJaIpLuj0aGcLSEO5YypryxW8EclEvxzXYqf7/a3W4Bvu/t03LL17t4K/Drwt2a2ea4d3X2nu7e6e2tdXV0CZYlIujvWO0xBntEY4lDOmIbKEvpHJhmfml584yyXSPB3As1x75uAk/Nsewuzunnc/WT0v0eAJ3hj/7+IZLGjPSM01ZRSkB/+AELdxPW6RH4ae4AtZrbRzIqIhPt5o3PM7EKgBngqblmNmRVHX9cC7wL2z95XRLLT0d5hWmrD798HqK/Sk7hiFg1+d58C7gQeAX4BfNPd95nZPWZ2U9ymtwIPuHt8N9DFQJuZvQj8CPh8/GggEclesaGcLWlwYRciffygJ3FBAsM5Adz9IeChWcs+O+v9X8yx38+Ay1ZQn4hkqN7hCc6OT4U6R0+815+9q66e8DveRCQrHTs3oic9WvxVpYUUFeSpqwcFv4gkSWw65nRp8ZsZDRrLDyj4RSRJ2nuGyc8zmkJ85OJs9ZXFmqETBb+IJMnh7rNsWF1GUUH6xEx9ZQldQ+rjT5+fiIhklcPdZ9lUVx52GW9QX1nCqYEx3jj4MPco+EUkcFPTMxztGWHz2vS4sBvTUFnC6OQ0Q+NTYZcSKgW/iASus3+UiekZNqdZi39tdCx/rs/Lr+AXkcAd7j4LkHbBr7H8EQp+EQnc68GfXl095569m+NDOhX8IhK4I93D1JYXUV1WFHYpb1Bfqfl6QMEvIkmQjiN6AEqL8qksKVDwh12AiGSfw93DadfNE9NQpbt3FfwiEqi+4Qn6hifS7sJuTOTZu7q4KyISmCNpOqInpr6yhC61+EVEgpOuQzlj6iuL6RoaZ3omd+/eVfCLSKAOdw9TVJBHY034z9mdS0NlCdMzTu9w7nb3KPhFJFAHTw2xua6c/DwLu5Q5nRvSOaDgFxEJxMFTQ1zUUBF2GfPSWH4Fv4gEaGBkklODY1yYxsHfUKW7dxX8IhKYg6eHALiwPn2Df82qIvKMnB7Zk1Dwm9l2MztoZofM7K451t9hZt1m9kL063fi1t1uZq9Gv24PsngRSS/ngj+NW/wF+XnUVRTndIu/YLENzCwf+BLwPqAT2GNmu919/6xNv+Hud87adzVwN9AKOPBsdN/+QKoXkbRy8NQgFSUFrIt2p6SrXL+JK5EW/zbgkLsfcfcJ4AHg5gSP/37gUXfvi4b9o8D25ZUqIunu4KkhLqyvwCw9R/TENFaXcqJ/JOwyQpNI8DcCHXHvO6PLZvugmb1kZt82s+Yl7ouZ7TCzNjNr6+7uTqAsEUkn7h4J/jTu5olpqimls380Zx/BmEjwz/Wre/Z36z+AFne/HPgv4F+WsG9koftOd29199a6uroEyhKRdHJqcIzBsam0HsoZ07y6jPGpGbrP5mZ3TyLB3wk0x71vAk7Gb+Duve4e+w7+E/CWRPcVkexw4FTswm5lyJUsrrmmDICOvtGQKwlHIsG/B9hiZhvNrAi4Bdgdv4GZrYt7exPwi+jrR4AbzKzGzGqAG6LLRCTLHHgt/YdyxjRFp5PozNF+/kVH9bj7lJndSSSw84Fd7r7PzO4B2tx9N/AHZnYTMAX0AXdE9+0zs88R+eUBcI+79yXhPEQkZHtPDNC8upSqssKwS1lUU7TF39mfmy3+RYMfwN0fAh6ateyzca8/A3xmnn13AbtWUKOIZIC9Jwe4rLEq7DISUlqUT215ER19udni1527IrJiA6OTHOsd4ZILMiP4IdLqz9UWv4JfRFZs38kBgIxp8UNkZE9HjvbxK/hFZMX2nRgE4NIMCv6mmlJOnhnNyQeyKPhFZMVePjFAY3Upq1cVhV1Kwpprypic9pycnlnBLyIrtvfkAJdckP7j9+PFhnTm4gVeBb+IrMjZ8Snae4Yzqn8fIn38kJtDOhX8IrIi+04M4J5Z/fsAF1SXYEZOXuBV8IvIijx3/AwAVzRXh1zJ0hQX5LOusoTjvQp+EZElee54P5tqV2XUhd2YjXWrONIzHHYZKafgF5Flc3eeP97PVetrwi5lWTbWruJI99mcm55ZwS8iy3a8b4SesxNcvSGzunliNtaWMzg2Rf/IZNilpJSCX0SW7bnjkaeoXp2hLf5NtasAaO85G3IlqaXgF5Fle+7YGcqLC9iaAVMxz2VjNPiPdOdWP7+CX0SWbc/RPq5sriY/L72fsTufpppSCvKM9hy7wKvgF5Fl6Rue4MCpId6+aXXYpSxbQX4e61eXKfhFRBLx8yO9ALxj85qQK1mZjbWrFPwiIol46kgvpYX5XN6UmSN6YjbWruJo7zAzOTRLp4JfRJblqcO9tLbUUJif2TGyqa6csckZTpzJnTl7MvsnJiKh6B4a59WusxnfzQNwYUM5AAdPDYVcSeokFPxmtt3MDprZITO7a471f2hm+83sJTN7zMw2xK2bNrMXol+7gyxeRMLxs8M9ALxzc23IlaxcbCjqwdO5E/yLPmzdzPKBLwHvAzqBPWa22933x232PNDq7iNm9kngC8BHo+tG3f3KgOsWkRA9cbCb1auKMm4q5rlUlBTSWF3KAbX432AbcMjdj7j7BPAAcHP8Bu7+I3ePTXH3NNAUbJkiki6mZ5wnDnZx7da6jB2/P9tFDRUcPDUYdhkpk0jwNwIdce87o8vm83Hg4bj3JWbWZmZPm9mvzLeTme2IbtfW3d2dQFkiEoYXO8/QPzLJdRfWhV1KYC5aV8GR7mEmpmbCLiUlEgn+uX6lzznuycw+BrQCX4xbvN7dW4FfB/7WzDbPta+773T3VndvravLnn9QItnmiQNd5BlcuzV7/j+9sKGSqRnncHduzNmTSPB3As1x75uAk7M3MrP3An8O3OTu47Hl7n4y+t8jwBPAVSuoV0RC9tiBLq5eX0N1WebNvz+fixoiF3gP5Eh3TyLBvwfYYmYbzawIuAV4w+gcM7sK+AqR0O+KW15jZsXR17XAu4D4i8IikkGO9gyz7+Qg2y9tCLuUQG2sXUVhvuXMBd5FR/W4+5SZ3Qk8AuQDu9x9n5ndA7S5+24iXTvlwLfMDOC4u98EXAx8xcxmiPyS+fys0UAikkF+8PJrAHzgsnUhVxKswvw8LmyoYO+JgbBLSYlFgx/A3R8CHpq17LNxr987z34/Ay5bSYEikj5+8NJrXL2+msbq0rBLCdwVTdXsfuEkMzNOXpaMVpqP7twVkYS09wyz/7VBfunyC8IuJSmuaK5maHyKIznwUBYFv4gk5MHnOskzuPGy7Orfj7mqOTLZ3Asd2d/do+AXkUVNTc/wrbZOrt1ax7qq7OvmgchkbeXFBbzYcSbsUpJOwS8ii/rxK92cGhzjo29dH3YpSZOfZ1zeVMXzHf1hl5J0Cn4RWdT9z3RQW17Mey5eG3YpSfXWltXsPznIwOhk2KUklYJfRBZ0tGeYxw6c5qNvbcr4ufcX847Na5hxeKa9L+xSkiq7f4oismI7nzxCYX4et7+zJexSku6q9dUUF+Sdm3Y6Wyn4RWRe3UPjfPvZTj54dRNrK0rCLifpigvyaW2p4anDvWGXklQKfhGZ15efOMzU9AyfePfGsEtJmXduruXAqSG6BsfCLiVpFPwiMqdjvcP829NH+UhrM5vqysMuJ2Xee3E9AI/+4nTIlSSPgl9E5vSFRw5SkJfHp9+3NexSUmprfTkta8p4ZJ+CX0RyyOMHTvODl17jf1+7ifrK7O/bj2dmvP+SBp463JO1wzoV/CLyBoNjk/zZg3vZWl/OJ6+b87lJWW/7pQ1MTjsPR2cjzTYKfhE5x93502+/RNfQGF/40BUUF+SHXVIormyuZmt9OV9/5njYpSSFgl9Ezvnyjw/z8N5T3PWBi7gyOmlZLjIzfn3bel7qHMjKOfoV/CICwDf3dPCFHx7kl6+4gE+8e1PY5YTuV69uoqwon3t/fDjsUgKn4BcR7n/mOH/64Etcs7WO//vhy4k+SS+nVZUW8tvv2sj3X3qNfSezq9Wv4BfJYeNT09zzH/v5zIMvc82WOr7ysbfkbL/+XD5xzSaqSgv53Pf3MzPjYZcTGAW/SI56/ng/v/z3/82un7Zzxztb+OrtrZQWKfTjVZUW8mc3XsTTR/r46n+3h11OYBJ65q6IZAd357nj/Xz5iSP81y9O01BZwj/f8Vauvyi7p1teiY+0NvP4gS4+/8MDXFBdyi9dnvkPmk+oxW9m283soJkdMrO75lhfbGbfiK7/uZm1xK37THT5QTN7f3Cli0giJqZmeKa9j7959BXe89c/5oNffopn2nv5o/dt5dE/vCYzQv+++6ClBfLyIv+9776UfbSZ8dcfuZKrmqv5gwee5+8fe5XJ6ZmUfX4ymPvC/VZmlg+8ArwP6AT2ALe6+/64bX4PuNzdf9fMbgF+1d0/amZvBu4HtgEXAP8FbHX36YU+s7W11dva2lZwWiK5YWJqhtGJaUYmpxgYnaRrcJyuoXG6hsY40j3MK6eHeOX0EGOTM5jBWzes5kNvaeLGy9dRXpwhf/Dfdx/s2AEjI68vKyuDnTvhtttSVsbZ8Sn+7MGX2f3iSRqrS/lwaxPv3lLLm9ZWUFVamLI65mNmz7p7a0LbJhD87wD+wt3fH33/GQB3/z9x2zwS3eYpMysATgF1wF3x28Zvt9BnLjf4P3LvU4xNnf87ZaFTdOZfOd9+Cx9vgXWLfK+X/llzrwy6vgWrnu97tIzPWWi/VP4MF7Kc71EyfoZjk9OMTkwztcAFx7UVxWytr2BrfQXbNq7mHZvWUFUWfkAtWUsLHDt2/vING+Do0VRXw48OdnHvE4d55mjfuZ9TRXEBq4oLWFWcT2F+3rlRUQaYRb+w6H/nV1VWxL/+9rZl1bWU4E/kV34j0BH3vhN423zbuPuUmQ0Aa6LLn561b+M8Re8AdgCsX7+853rWrCpkYmrui1MLDU9b6Acx/24LHG+BA863auF9lv5Zyz3efKsW/h4t/Xu7nO/RQp+14ODDZXwv0v1nWFyQT1lR5Ku0qICyonwqSgpYW1HC2opi1lYWU1aUIS36xRyf5+7Z+ZYn2fUXruX6C9fSNzxB29E+2nuGOTU4xvD4FMPj00xMz8TKSk+0AAAFJElEQVT94nbcI7+s3RdqokSk6i+HRP5lzPXPb3b9822TyL6Rhe47gZ0QafEnUNd5vvIbCf2yE5FMsn793C3+ZTYQg7J6VRE3XNIQag3LlcjF3U6gOe59E3Byvm2iXT1VQF+C+4qIzO8v/zLSpx+vrCyyXJYlkeDfA2wxs41mVgTcAuyetc1u4Pbo6w8Bj3ukM3Q3cEt01M9GYAvwTDCli0hOuO22yIXcDRsi/V8bNqT8wm62WbSrJ9pnfyfwCJAP7HL3fWZ2D9Dm7ruBrwL/ZmaHiLT0b4nuu8/MvgnsB6aATy02okdE5Dy33aagD9Cio3rCoOGcIiJLs5RRPZqyQUQkxyj4RURyjIJfRCTHKPhFRHKMgl9EJMco+EVEcoyCX0Qkx6TlOH4z6wbmmJwjIbVAT4DlZAKdc/bLtfMFnfNSbXD3ukQ2TMvgXwkza0v0JoZsoXPOfrl2vqBzTiZ19YiI5BgFv4hIjsnG4N8ZdgEh0Dlnv1w7X9A5J03W9fGLiMjCsrHFLyIiC1Dwi4jkmIwNfjPbbmYHzeyQmd01x/piM/tGdP3Pzawl9VUGJ4Hz/UMz229mL5nZY2a2IYw6g7TYOcdt9yEzczPL+KF/iZyzmX0k+rPeZ2ZfT3WNQUvg3/Z6M/uRmT0f/fd9Yxh1BsXMdplZl5ntnWe9mdnfRb8fL5nZ1YEX4e4Z90XkSWCHgU1AEfAi8OZZ2/wecG/09S3AN8KuO8nnez1QFn39yUw+30TPObpdBfAT4GmgNey6U/Bz3gI8D9RE368Nu+4UnPNO4JPR128GjoZd9wrP+RrgamDvPOtvBB4GDHg78POga8jUFv824JC7H3H3CeAB4OZZ29wM/Ev09beB95iZpbDGIC16vu7+I3cfib59msiD7TNZIj9jgM8BXwDGUllckiRyzp8AvuTu/QDu3pXiGoOWyDk7UBl9XQWcTGF9gXP3nxB5RO18bgb+1SOeBqrNbF2QNWRq8DcCHXHvO6PL5tzG3aeAAWBNSqoLXiLnG+/jRFoMmWzRczazq4Bmd/9+KgtLokR+zluBrWb2UzN72sy2p6y65EjknP8C+JiZdQIPAb+fmtJCs9T/35ds0Yetp6m5Wu6zx6Umsk2mSPhczOxjQCtwbVIrSr4Fz9nM8oC/Ae5IVUEpkMjPuYBId891RP6qe9LMLnX3M0muLVkSOedbga+5+1+Z2TuAf4ue80zyywtF0rMrU1v8nUBz3Psmzv/z79w2ZlZA5E/Ehf68SmeJnC9m9l7gz4Gb3H08RbUly2LnXAFcCjxhZkeJ9IXuzvALvIn+u/53d59093bgIJFfBJkqkXP+OPBNAHd/CighMplZtkro//eVyNTg3wNsMbONZlZE5OLt7lnb7AZuj77+EPC4R6+cZKBFzzfa7fEVIqGf6f2+sMg5u/uAu9e6e4u7txC5rnGTu7eFU24gEvl3/T0iF/Ixs1oiXT9HUlplsBI55+PAewDM7GIiwd+d0ipTazfwm9HRPW8HBtz9tSA/ICO7etx9yszuBB4hMipgl7vvM7N7gDZ33w18lcifhIeItPRvCa/ilUnwfL8IlAPfil7DPu7uN4VW9AoleM5ZJcFzfgS4wcz2A9PAn7h7b3hVr0yC5/xHwD+Z2aeJdHnckcGNOMzsfiJddbXR6xZ3A4UA7n4vkesYNwKHgBHgtwKvIYO/fyIisgyZ2tUjIiLLpOAXEckxCn4RkRyj4BcRyTEKfhGRHKPgFxHJMQp+EZEc8/8B+ux3bAdkyWcAAAAASUVORK5CYII=\n", "text/plain": [ "
" ] }, "metadata": { "needs_background": "light" }, "output_type": "display_data" }, { "data": { "image/png": "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\n", "text/plain": [ "
" ] }, "metadata": { "needs_background": "light" }, "output_type": "display_data" }, { "data": { "image/png": "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\n", "text/plain": [ "
" ] }, "metadata": { "needs_background": "light" }, "output_type": "display_data" }, { "data": { "image/png": "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\n", "text/plain": [ "
" ] }, "metadata": { "needs_background": "light" }, "output_type": "display_data" }, { "data": { "image/png": "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\n", "text/plain": [ "
" ] }, "metadata": { "needs_background": "light" }, "output_type": "display_data" }, { "data": { "image/png": "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\n", "text/plain": [ "
" ] }, "metadata": { "needs_background": "light" }, "output_type": "display_data" }, { "data": { "image/png": "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\n", "text/plain": [ "
" ] }, "metadata": { "needs_background": "light" }, "output_type": "display_data" }, { "data": { "image/png": "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\n", "text/plain": [ "
" ] }, "metadata": { "needs_background": "light" }, "output_type": "display_data" }, { "data": { "image/png": "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\n", "text/plain": [ "
" ] }, "metadata": { "needs_background": "light" }, "output_type": "display_data" }, { "data": { "image/png": "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\n", "text/plain": [ "
" ] }, "metadata": { "needs_background": "light" }, "output_type": "display_data" }, { "data": { "image/png": "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\n", "text/plain": [ "
" ] }, "metadata": { "needs_background": "light" }, "output_type": "display_data" }, { "data": { "image/png": "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\n", "text/plain": [ "
" ] }, "metadata": { "needs_background": "light" }, "output_type": "display_data" }, { "data": { "image/png": "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\n", "text/plain": [ "
" ] }, "metadata": { "needs_background": "light" }, "output_type": "display_data" }, { "data": { "image/png": "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\n", "text/plain": [ "
" ] }, "metadata": { "needs_background": "light" }, "output_type": "display_data" }, { "data": { "image/png": "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\n", "text/plain": [ "
" ] }, "metadata": { "needs_background": "light" }, "output_type": "display_data" }, { "data": { "image/png": "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\n", "text/plain": [ "
" ] }, "metadata": { "needs_background": "light" }, "output_type": "display_data" }, { "data": { "image/png": "iVBORw0KGgoAAAANSUhEUgAAAXcAAAEDCAYAAADOc0QpAAAABHNCSVQICAgIfAhkiAAAAAlwSFlzAAALEgAACxIB0t1+/AAAADl0RVh0U29mdHdhcmUAbWF0cGxvdGxpYiB2ZXJzaW9uIDMuMC4zLCBodHRwOi8vbWF0cGxvdGxpYi5vcmcvnQurowAAIABJREFUeJzt3XtwnHd97/H3V/e7ZFlrW5Yvcu5XcsGEJE1PU6AlhE5C29CGCRA66fGBllKm7ZmSdoaWdHrOtGcOzHA5pOZACW1KUyjlGE6AEyAMuRMlcRw7ThwnvkmyZUnWzbpfvuePfdberFfSSnp2n93V5zWjyV5++zzfx1I++un3/J7fY+6OiIgUl5KoCxARkfAp3EVEipDCXUSkCCncRUSKkMJdRKQIKdxFRIpQpOFuZl8zs5Nmtjek7f3QzAbN7Pspr7/DzJ43s71m9oCZlYWxPxGRfBV1z/3rwC0hbu9/AB9KfsHMSoAHgDvd/QrgCHB3iPsUEck7kYa7u/8cOJX8mpmdH/TAnzOzx8zskiVs7yfASMrLa4FJdz8QPH8E+O2V1C0iku+i7rmnsxP4I3d/K/BnwP9a4fb6gHIz2x48vwPYvMJtiojktbwaezazOuBG4Ftmlni5Mnjvt4D70nysy93fPd823d3N7E7gc2ZWCfw/YCbUwkVE8kxehTvxvyQG3f3q1Dfc/TvAd5azUXd/CvhlADP7deCilRQpIpLv8mpYxt2HgUNm9n4Ai7tqpds1s3XBfyuBPwfuX+k2RUTyWdRTIb8JPAVcbGadZnYPcBdwj5m9COwDbl/C9h4DvgW8M9heYrjmv5rZfmAP8D13/2moByIikmdMS/6KiBSfvBqWERGRcER2QrWlpcXb29uj2r2ISEF67rnn+tw9tli7yMK9vb2djo6OqHYvIlKQzOxIJu00LCMiUoQU7iIiRUjhLiJShBYNdzOrMrNfmNmLZrbPzD6Tps1HzKzXzHYHX7+fnXJFRCQTmZxQnQTe4e6nzawceNzMfuDuT6e0e8jdPx5+iSIislSLhrvHr3I6HTwtD7505ZOISB7LaMzdzErNbDdwEnjE3Z9J0+y3zWyPmX3bzNIuqWtmO8ysw8w6ent7V1C2iIgsJKNwd/fZYKXGTcB1ZnZFSpPvAe3u/hbgx8TvfJRuOzvdfbu7b4/FFp2DLyKr2MT0LP/4xCF+uPd41KUUpCVdxOTug2b2M+K3xtub9Hp/UrOvAH8XSnUisirNzTn3PPAsTxyMR8tnbrucu29sj7aoApPJbJmYmTUFj6uBdwGvpLRpTXp6G7A/zCJFZHXZ9WI3Txzs5zO3Xc7NF8f4+x++wsDoVNRlFZRMhmVagUfNbA/wLPEx9++b2X1mdlvQ5hPBNMkXgU8AH8lOuSKyGnzjqcNcsK6OD12/lXvfcymjU7P8W8exqMsqKJnMltkDXJPm9U8nPb4XuDfc0kRkNTp48jTPHx3k3vdcQkmJcfGGeq7a3MR3d3fzX37l/KjLKxi6QlVE8soPXjqOGfzmNW1nXnvvlRvYf3yYE0MTEVZWWBTuIpJXHjvYx+UbG1jXUHXmtRvPbwHgydf7oiqr4CjcRSRvjE7O8MLRAW664M1TpS9rbWBNTTlPvt4/zycllcJdRPLGLw6dYnrWuemClje9XlJiXH/eWp5+Q+GeKYW7iOSNF44NUmJw7damc967enMTnQPjmhKZIYW7iOSNPZ2DXLiunpqKcyfyXdHWCMDe7qFcl1WQFO4ikhfcnT2dQ7xlU2Pa96/YGH/9pS6FeyYU7iKSFzoHxjk1OjVvuDfWlLO5uZp9XcM5rqwwKdxFJC8keuRv2XTueHvC5a2N7D+ucM+Ewl1E8sKeziHKS41LWuvnbXPBujqOnBpjamYuh5UVJoW7iOSFAz0jnB+ro7KsdN42F6yrY3bOOdw/msPKCpPCXUTywoGeES5YV7dgm8T7B0+eXrCdKNxFJA+MTc3QOTDORevnH5IBOD9Whxm81qNwX4zCXUQil+iJX7R+4Z57dUUpbU3VHOxVuC9G4S4ikTsQ9MQvWLdwzx1gW0stRzXmviiFu4hE7rWTI1SUltC+tmbRtluaazhyaiwHVRU2hbuIRO61ntOcF6ulrHTxSNrSXMPg2DRD49M5qKxwKdxFJHJv9MbDPRNbmuO9+2PqvS9I4S4ikZqenaNzYJz2tRmG+1qFeyYWDXczqzKzX5jZi8FNsD+Tpk2lmT1kZgfN7Bkza89GsSJSfLoHx5mZc9pbMgv3zUHPXePuC8uk5z4JvMPdrwKuBm4xs+tT2twDDLj7BcDngL8Lt0wRKVaH++MhnWnPvaGqnDU15RxVuC9o0XD3uMSk0vLgy1Oa3Q48EDz+NvBOM7PQqhSRonW4Lz6tMZOZMglbmms0LLOIjMbczazUzHYDJ4FH3P2ZlCZtwDEAd58BhoC1abazw8w6zKyjt7d3ZZWLSFE43D9KTUUpsfrKjD+zZW0tR/oV7gvJKNzdfdbdrwY2AdeZ2RUpTdL10lN797j7Tnff7u7bY7FYmo+IyGpzpH+MrWtrWcof+21N1RwfGmdu7pyYkcCSZsu4+yDwM+CWlLc6gc0AZlYGNAKnQqhPRIrc4f7RJQ3JAGxsqmJ61ukbncxSVYUvk9kyMTNrCh5XA+8CXklptgu4O3h8B/BTd9evVBFZ0MzsHMdOxXvuS9HaWA3A8cGJbJRVFDLpubcCj5rZHuBZ4mPu3zez+8zstqDNV4G1ZnYQ+BPgU9kpV0SKyfGhCaZnnW0tS+u5tzZWBZ8fz0ZZReHcW4yncPc9wDVpXv900uMJ4P3hliYixS5x040tzUvruW9sivfcu9Vzn5euUBWRyHQNxHvem5url/S5NTXlVJaVqOe+AIW7iESme3CcEoP1DVVL+pyZsbGpmu4h9dzno3AXkch0DU6wvqGK8gxWg0zV2ljF8UH13OejcBeRyHQPjp8ZP1+q1sZqjqvnPi+Fu4hEpntoJeFeRc/wBDOzcyFXVRwU7iISibk55/jgBBubljbentDaVMWcw8kRXciUjsJdRCLRNzrJ1OwcbcvsuW9MXMikGTNpKdxFJBKJOeqJkF6q1qDHr7nu6SncRSQS3cFMl5WcUAU4oZOqaSncRSQSiXBf7rBMQ1UZlWUlnBxRuKejcBeRSHQNjlNbUUpD9aKroKRlZqxvqNIJ1Xko3EUkEok57iu5adu6+kp6htVzT0fhLiKR6B6cWPZ4e8K6hkr13OehcBeRSKzk6tSEdfVV9A4r3NNRuItIzk1Mz9I/OkXbMi9gSojVVzIyOcP41GxIlRUPhbuI5NxKp0EmJFaT1IyZcyncRSTnzlzAtOJhmUpASxCko3AXkZxb6Rz3hHUN8XDXjJlzKdxFJOe6BsexZdykI9W6+mBYRidVz7FouJvZZjN71Mz2m9k+M/vjNG1uNrMhM9sdfH063bZERCDec19XX0lF2cr6l2tqyikvNQ3LpJHJpWEzwJ+6+/NmVg88Z2aPuPvLKe0ec/ffCL9EESk2K1nHPZmZsa6+SidU01j016a7H3f354PHI8B+oC3bhYlI8QrjAqaEWH0lveq5n2NJfxOZWTtwDfBMmrdvMLMXzewHZnb5PJ/fYWYdZtbR29u75GJFpPC5O12D4ys+mZqwrr5SY+5pZBzuZlYH/DvwSXcfTnn7eWCru18FfAH4brptuPtOd9/u7ttjsdhyaxaRAtY/OsXUzBwbG1d2MjVhXUMlPRqWOUdG4W5m5cSD/UF3/07q++4+7O6ng8cPA+Vm1hJqpSJSFM5Mg1xTE8r21tVXMTg2zeSMrlJNlslsGQO+Cux398/O02ZD0A4zuy7Ybn+YhYpIcTh7dWpIPffgQiaNu79ZJrNlfgn4EPCSme0OXvsLYAuAu98P3AF8zMxmgHHgTnf3LNQrIgWuK7g6Nawx97NLEEyyKaS/BorBouHu7o8DCy647O5fBL4YVlEiUry6B8epqSilsbo8lO3F1HNPS1eoikhOhXGTjmQtdfFw7zutcE+mcBeRnApjHfdka+sqAOgbmQptm8VA4S4iOdU1OLHiddyTlZeW0FRTrp57CoW7iOTMxPQsfacn2dgYXs8d4kMzCvc3U7iLSM6cGApnHfdULXUVCvcUCncRyZmw7sCUKt5z15h7MoW7iORMV0g36UjVUldJn6ZCvonCXURypntwIn6TjsbKULebuFH2xLSWIEhQuItIznQPjhOrq6SyrDTU7bYkpkNq3P0MhbuI5ExYN+lIdfZCJo27JyjcRSRnwlzHPdmZcNe4+xkKdxHJCXcPrk4N7wKmhJZ6LUGQSuEuIjkxMDbNxPRcVoZl1tZqzD2Vwl1EciJbc9wBqspLqa8q05h7EoW7iOREtua4J8TqKulVz/0MhbuI5EQ2e+6gC5lSKdxFJCe6B8epKi9hTU04N+lI1VKv9WWSKdxFJCe6BydCvUlHKq0v82YKdxHJiWzNcU9oqatkaHyaqZm5rO2jkCwa7ma22cweNbP9ZrbPzP44TRszs8+b2UEz22Nm12anXBEpVN2D46Gv454scSFT/6iGZiCznvsM8KfufilwPfCHZnZZSpv3ABcGXzuAL4dapYgUtMmZWU6OTGbtZCokrS+j2+0BGYS7ux939+eDxyPAfqAtpdntwDc87mmgycxaQ69WRArS2Zt0hH91aoKuUn2zJY25m1k7cA3wTMpbbcCxpOednPsLADPbYWYdZtbR29u7tEpFpGBle447xOe5A5rrHsg43M2sDvh34JPuPpz6dpqP+DkvuO909+3uvj0Wiy2tUhEpWN2D2bm9XrKzK0Mq3CHDcDezcuLB/qC7fydNk05gc9LzTUD3yssTkWKQuIBpQ2P2hmWqK0qprSjVmHsgk9kyBnwV2O/un52n2S7gw8GsmeuBIXc/HmKdIlLAugfHaamrpKo83Jt0pGqpr1TPPVCWQZtfAj4EvGRmu4PX/gLYAuDu9wMPA7cCB4Ex4PfCL1VEClV8jnv2eu0J8QuZFO6QQbi7++OkH1NPbuPAH4ZVlIgUl+7BcS5aX5/1/bTUVXCobzTr+ykEukJVRLIqfpOOiayeTE3QEgRnKdxFJKsGx6YZn57NWbgPjE0xM6slCBTuIpJVZ+e452DMvb4Sdzg1qt67wl1Esirb67gniwVLEOhCJoW7iGRZLq5OTTh7IZN67gp3EcmqroH4TTqag5tYZ9OZcNcdmRTuIpJdiXXcs3WTjmRaPOwshbuIZFXX4Dhta2pysq/ailKqyksU7ijcRSTLugayewemZGZGrL6SXg3LKNxFJHvGp2bpH53KyTTIBF3IFKdwF5GsOTNTZk1ueu4QX9ddPXeFu4hk0dlpkLkZcweIaWVIQOEuIlnUNRBBz72+klNjU0yv8iUIFO4ikjVdg2OUlhjrgymKuRDTEgSAwl1EsqhrYJwNDVWUleYuahIXMq32cXeFu4hkTXyOe+6GZCDecweFu8JdRLKma2CcTTma454QU88dULiLSJZMz85xYngiup77Kp8xo3AXkaw4MTTBnOdmNchkVeWl1FeVqee+WAMz+5qZnTSzvfO8f7OZDZnZ7uDr0+GXKSKFJooLmBJi9ZWrvue+6A2yga8DXwS+sUCbx9z9N0KpSESKwpk57jnuuUN8xox67otw958Dp3JQi4gUka4c3oEpVay+ctWv6R7WmPsNZvaimf3AzC4PaZsiUsC6BsZpqaukqrw05/vW+jLhhPvzwFZ3vwr4AvDd+Rqa2Q4z6zCzjt7e3hB2LSL5Koo57gmx+kpGJmeYmJ6NZP/5YMXh7u7D7n46ePwwUG5mLfO03enu2919eywWW+muRSSPHT01xpbm3C0YlkwXMoUQ7ma2wYL7Z5nZdcE2+1e6XREpXDOzc3QNjrOlObqeO6zuue6LzpYxs28CNwMtZtYJ/BVQDuDu9wN3AB8zsxlgHLjT3T1rFYtI3js+NMHsnEfXc9dVqouHu7t/YJH3v0h8qqSICBAfkgHY0lwbyf41LKMrVEUkC470B+G+Npqee3NtBWYKdxGRUB09NUZ5qbGhIXf3Tk1WXlpCc03Fqr4jk8JdREJ37NQYm9fUUFpikdUQq1/dc90V7iISuiOnRtkc0cnUhJa61b2+jMJdREJ3tD+6Oe4J6rmLiIRoaGya4YmZvAn31TozW+EuIqE6Mw0yopkyCbG6SiZn5hiZnIm0jqgo3EUkVEdOjQLkRc8dWLWrQyrcRSRUiZ571CdUV/uFTAp3EQnVsVNjtNRVUFeZyb2AsqclWILgpMJdRGTljp4ai7zXDrC+IR7uPcMTEVcSDYW7iITqSB5MgwRorC6nsqxE4S4islIT07N0DY6zrSWaBcOSmRkbGqs4MaxhGRGRFTnSP4Y7eRHuAOsbqugZUs9dRGRF3ug9DcD5sbqIK4nb0FBFz4jCXURkRd7oi89xz5ee+4bGKk4MTazKq1QV7iISmjd6R1nfUEltxNMgE9Y3VDE5M8fQ+HTUpeScwl1EQnOo7zTnteTHkAxwZj35E6twxozCXURC80bfKNti+TEkA7ChMT7X/cQqPKmqcBeRUJwanWJwbJrz8mS8HeLDMrA6L2RaNNzN7GtmdtLM9s7zvpnZ583soJntMbNrwy9TRPLdob74TJnz8qjnvq4+GJYZWn1z3TPpuX8duGWB998DXBh87QC+vPKyRKTQvN4bnymTT2PuFWUlrK2t0Jh7Ou7+c+DUAk1uB77hcU8DTWbWGlaBIlIYDvWNUl5qbFpTHXUpb7K+oUrDMsvUBhxLet4ZvHYOM9thZh1m1tHb2xvCrkUkX7zRe5otzTWUlebXqbzEXPfVJozvQrrbm6e9YsDdd7r7dnffHovFQti1iOSL13pOc+G6+qjLOId67svXCWxOer4J6A5huyJSICamZzncP8pFG/Iv3Dc0VNE/OsXkzGzUpeRUGOG+C/hwMGvmemDI3Y+HsF0RKRCv955mzuHi9XkY7sFc95OrbHXIRa8RNrNvAjcDLWbWCfwVUA7g7vcDDwO3AgeBMeD3slWsiOSnAz0jAFy0Pn9myiS0NsZP8B4fmsiLm4jkyqLh7u4fWOR9B/4wtIpEpOC8euI05aVGex5dwJTQFsze6RocA5qjLSaH8uu0togUpAM9I5wfq6M8z2bKALQ1BeE+MB5xJbmVf98JESk4B3pGuDAPx9sBqspLaamroGtQ4S4ikrHTkzN0DoxzcR6Otye0NVXTqZ67iEjmXgtOpuZrzx3i4+4alhERWYJ93cMAXNbaEHEl89u0poauwfFVdUcmhbuIrMi+7iEaq8vzbk2ZZG1N1UzOzNF3eirqUnJG4S4iK7Kve5jLNzZglm4lkvyQmDHTOTAWcSW5o3AXkWWbnp3jleMjXNHWGHUpCzo71331jLsr3EVk2Q6ePM3U7ByXb8zf8XZICvdVdFJV4S4iy7a3awiAyzfmd8+9oaqchqoy9dxFRDKxr3uYmopStuXhsgOp2tbUcOyUxtxFRBa1t2uIS1sbKC3J35OpCe1razjSr3AXEVnQ1Mwce7qGuGZzU9SlZKS9pZajp8aYmZ2LupScULiLyLK8fHyYqZk5rt26JupSMrJtbS0zc75qxt0V7iKyLM8fGQDg2i2FEe6J5YgP9Y1GXEluKNxFZFleODZIa2MVGxqroi4lI+0t8Rt1HFa4i4jM7/kjAwXTaweI1VVSW1HK4VVyUlXhLiJLdnJ4gq7Bca7ZUhgnUwHM4neK0rCMiMg8nj0cH29/a4GcTE3Y1lLL4X6F+xlmdouZvWpmB83sU2ne/4iZ9ZrZ7uDr98MvVUTyxZOv91FXWcaVeb6mTKptLbUcOzXG1EzxT4dcNNzNrBT4EvAe4DLgA2Z2WZqmD7n71cHX/w65ThHJI0++3s/btzVTlof3TF1I+9pa5hyOrYLVITP5zlwHHHT3N9x9CvhX4PbsliUi+ap7cJxDfaPccP7aqEtZsvNi8emQB0+ejriS7Msk3NuAY0nPO4PXUv22me0xs2+b2eZ0GzKzHWbWYWYdvb29yyhXRKL21Ov9APzSBS0RV7J0FwW3AjxwYiTiSrIvk3BPt2hE6r2qvge0u/tbgB8DD6TbkLvvdPft7r49FostrVIRyQtPvN5Hc20FF+fxPVPnU1tZxpbmGl7pUbhDvKee3BPfBHQnN3D3fnefDJ5+BXhrOOWJSD6Zm3Mee62PG89fS0kBLBaWzsUb6nlVPXcAngUuNLNtZlYB3AnsSm5gZq1JT28D9odXoojki92dg/SOTPJrl62PupRlu3h9PYf6RpmcmY26lKxaNNzdfQb4OPAj4qH9b+6+z8zuM7PbgmafMLN9ZvYi8AngI9kqWESi8+OXeygtMW6+aF3UpSzbxRvqmZ3zoj+pWpZJI3d/GHg45bVPJz2+F7g33NJEJN888nIP17U301hTHnUpy3bJhvi5gldPjOT9HaRWorAmqYpIZA73jfLaydO8q4CHZCB+IVNVeQl7u4ajLiWrFO4ikpH/+9JxAH69wMO9rLSEK9sa2X1sIOpSskrhLiKLcne+83wnb2tfw+bmmqjLWbGrNjWxt3u4qJchULiLyKL2dA7xeu8ov3XtpqhLCcXVW5qYmpkr6imRCncRWdR/vNBFRVkJt17ZunjjAnB1cN/XYh6aUbiLyILGp2b57u4ufu2y9TRWF+4smWRtTdW01FXywtHBqEvJGoW7iCzoP17oYnBsmrtvaI+6lNCYGW/f1syTr/fjnrqaSnFQuIvIvNydrz95iMs3NvC29sK6McdibrqwhRPDE7zeW5w371C4i8i8Hnm5hwM9p7nnpm2YFeZaMvO5KVjV8vHXinOFWoW7iKQ1N+d89pEDbGup5barNkZdTug2N9ewdW0Njx/si7qUrFC4i0hau17s5pUTI3zinRcU3B2XMvXLF7bwxMF+xqZmoi4ldMX5HRORFRmZmOZvH97PVZsaue2qdPfmKQ7vvXIj49Oz/GT/yahLCZ3CXUTO8Xc/fIW+05Pcd/sVlBbouu2ZuG5bM+sbKvnei92LNy4wCncReZOf7O/hn58+yu/ftI2rgot9ilVpifHeKzfys1d7GRybirqcUCncReSMAz0jfPKh3VzW2sCfvfviqMvJid952yamZuf4l18cjbqUUCncRQSAzoExfu8fn6W6vJSv3L2dyrLSqEvKiUs2NHDTBS088ORhJqaL5+5MCncR4dUTI9zx5acYnpjmax95G21N1VGXlFN/8Kvn0zM8yT8+cTjqUkKjcBdZxdydh549yvu+9ARz7nzrozdwRVvx3p1oPjee38K7Ll3PF376Gof6iuOKVYW7yCrk7jx7+BS/8w9P8ef//hLXbGli18dv4pINDVGXFpm/ed/lVJSV8LF/fo6hsemoy1mxjMLdzG4xs1fN7KCZfSrN+5Vm9lDw/jNm1h52oSKycof6Rvnq44d47+cf5/33P8WhvjH+229eyT/f83Y2NFZFXd6bPfggtLdDSUn8vw8+mNXdtTZW84UPXMMbvaP87s6nCv4G2rbYimhmVgocAH4N6ASeBT7g7i8ntfkD4C3u/lEzuxP4TXf/3YW2u337du/o6Fhp/SISmJtzxqZnGZucYXB8muNDE/QMTdA5MMbLx0d4uXuI7qEJAC5tbeCD12/hfVe3UVtZFnHlaTz4IOzYAWNjZ1+rqYGdO+Guu7K6658f6OWTD+1mZGKa913dxi1XbODqzU0011bkxfo6Zvacu29ftF0G4X4D8Nfu/u7g+b0A7v7fk9r8KGjzlJmVASeAmC+w8eWG++Ov9fH3P3ol7XsLHYoz/5sLfm7BbS70ueUtI5qVY1hwfwtsc4HPLfxmdvaZ6+/vQvLpGGbnnLGpWcbnmelhBue11HL5xkbeunUN77hkXf7fKq+9HY4cOff1rVvh8OGs775neIIvPXqQb3V0nvl3rakopbG6nKryUirLSigrfXPQGynP0/weSLz0W9du4u4b25dVW6bhnsmv7DbgWNLzTuDt87Vx9xkzGwLWAm9akcfMdgA7ALZs2ZLBrs9VUVbC2tqKed9f6DfrQr9zF/6FvMA2F/jccveX+kOS8eeWuc1lvhXsM/x/74U/t7z9LXyMhf39NTNqK0qpqSijtjL+34bqclobq9jQUMX6hioqygrs9NrReeacz/d6yNY3VHHf7VfwF7deyvNHBni1Z4TOgXGGx6eZmJljYnqWubmzv3FTf/em++Wf/Ep1RfanmWYS7ul+pFIrz6QN7r4T2AnxnnsG+z7HdduauW7bdcv5qIgUii1b0vfcl9kpXK6q8lJuvKCFG4PlgQtJJr/OO4HNSc83AakLMZxpEwzLNAKnwihQRFahv/3b+Bh7spqa+OuSkUzC/VngQjPbZmYVwJ3ArpQ2u4C7g8d3AD9daLxdRGRBd90VP3m6dWt8PGrr1pycTC0miw7LBGPoHwd+BJQCX3P3fWZ2H9Dh7ruArwL/ZGYHiffY78xm0SKyCtx1l8J8BTKaA+XuDwMPp7z26aTHE8D7wy1NRESWq8BOoYuISCYU7iIiRUjhLiJShBTuIiJFSOEuIlKEFO4iIkVI4S4iUoQWXRUyazs26wXSLB6RkRZSFiVbBXTMq4OOeXVYyTFvdffYYo0iC/eVMLOOTJa8LCY65tVBx7w65OKYNSwjIlKEFO4iIkWoUMN9Z9QFREDHvDromFeHrB9zQY65i4jIwgq15y4iIgtQuIuIFKG8Dnczu8XMXjWzg2b2qTTvV5rZQ8H7z5hZe+6rDFcGx/wnZvayme0xs5+Y2dYo6gzTYsec1O4OM3MzK/hpc5kcs5n9TvC93mdm/5LrGsOWwc/2FjN71MxeCH6+b42izrCY2dfM7KSZ7Z3nfTOzzwf/HnvM7NpQC3D3vPwiften14HzgArgReCylDZ/ANwfPL4TeCjqunNwzL8K1ASPP7YajjloVw/8HHga2B513Tn4Pl8IvACsCZ6vi7ruHBzzTuBjwePLgMNR173CY/5PwLXA3nnevxX4AWDA9cAzYe4/n3vu1wEH3f0Nd58C/hW4PaXN7cADweNvA+80M8thjWFb9Jjd/VF3HwuePk38huWFLJPvM8DfAH8PTOSyuCzJ5Jj/M/Aldx8AcPeTOa4xbJmCYFaEAAACWElEQVQcswMNweNGoDuH9YXO3X9O/Laj87kd+IbHPQ00mVlrWPvP53BvA44lPe8MXkvbxt1ngCFgbU6qy45MjjnZPcR/8xeyRY/ZzK4BNrv793NZWBZl8n2+CLjIzJ4ws6fN7JacVZcdmRzzXwMfNLNO4rf1/KPclBaZpf7/viQZ3UM1Iul64KnzNjNpU0gyPh4z+yCwHfiVrFaUfQses5mVAJ8DPpKrgnIgk+9zGfGhmZuJ/3X2mJld4e6DWa4tWzI55g8AX3f3/2lmNwD/FBzzXPbLi0RW8yufe+6dwOak55s498+0M23MrIz4n3IL/RmU7zI5ZszsXcBfAre5+2SOasuWxY65HrgC+JmZHSY+NrmrwE+qZvqz/X/cfdrdDwGvEg/7QpXJMd8D/BuAuz8FVBFfYKtYZfT/+3Llc7g/C1xoZtvMrIL4CdNdKW12AXcHj+8AfurBmYoCtegxB0MU/0A82At9HBYWOWZ3H3L3Fndvd/d24ucZbnP3jmjKDUUmP9vfJX7yHDNrIT5M80ZOqwxXJsd8FHgngJldSjzce3NaZW7tAj4czJq5Hhhy9+OhbT3qM8qLnG2+FThA/Cz7Xwav3Uf8f26If/O/BRwEfgGcF3XNOTjmHwM9wO7ga1fUNWf7mFPa/owCny2T4ffZgM8CLwMvAXdGXXMOjvky4AniM2l2A78edc0rPN5vAseBaeK99HuAjwIfTfoefyn493gp7J9rLT8gIlKE8nlYRkRElknhLiJShBTuIiJFSOEuIlKEFO4iIkVI4S4iUoQU7iIiRej/Az/IouJxofqUAAAAAElFTkSuQmCC\n", "text/plain": [ "
" ] }, "metadata": { "needs_background": "light" }, "output_type": "display_data" }, { "data": { "image/png": "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\n", "text/plain": [ "
" ] }, "metadata": { "needs_background": "light" }, "output_type": "display_data" }, { "data": { "image/png": "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\n", "text/plain": [ "
" ] }, "metadata": { "needs_background": "light" }, "output_type": "display_data" }, { "data": { "image/png": "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\n", "text/plain": [ "
" ] }, "metadata": { "needs_background": "light" }, "output_type": "display_data" }, { "data": { "image/png": "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\n", "text/plain": [ "
" ] }, "metadata": { "needs_background": "light" }, "output_type": "display_data" }, { "data": { "image/png": "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\n", "text/plain": [ "
" ] }, "metadata": { "needs_background": "light" }, "output_type": "display_data" }, { "data": { "image/png": "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\n", "text/plain": [ "
" ] }, "metadata": { "needs_background": "light" }, "output_type": "display_data" }, { "data": { "image/png": "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\n", "text/plain": [ "
" ] }, "metadata": { "needs_background": "light" }, "output_type": "display_data" }, { "data": { "image/png": "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\n", "text/plain": [ "
" ] }, "metadata": { "needs_background": "light" }, "output_type": "display_data" } ], "source": [ "for i in range(k):\n", " p = numpy.linspace(0,1,1000)\n", " l = p**y[i]*(1-p)**(i+1-y[i])\n", " plt.plot(p,l)\n", " plt.scatter(numpy.array([y[i]/(i+1)]),numpy.array([0]),color='r')\n", " plt.show()" ] }, { "cell_type": "markdown", "metadata": {}, "source": [ "# Tests\n", "\n", "We are testing in the sequel whether rather the null hypothesis $p_0$ holds true or the alternative $p_1$: we first plot the likelihood for different values of $p$ given the data (Bayesian approach!), and we also plot the likelihood ratio function (actually its logarithm) depending on data given null and alternative hypothesis." ] }, { "cell_type": "code", "execution_count": 125, "metadata": {}, "outputs": [ { "name": "stdout", "output_type": "stream", "text": [ "Likelihood-Ratio: -0.3364722366212129\n" ] }, { "data": { "image/png": "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\n", "text/plain": [ "
" ] }, "metadata": { "needs_background": "light" }, "output_type": "display_data" }, { "name": "stdout", "output_type": "stream", "text": [ "Likelihood-Ratio: -0.6729444732424256\n" ] }, { "data": { "image/png": "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\n", "text/plain": [ "
" ] }, "metadata": { "needs_background": "light" }, "output_type": "display_data" }, { "name": "stdout", "output_type": "stream", "text": [ "Likelihood-Ratio: -1.0094167098636386\n" ] }, { "data": { "image/png": "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\n", "text/plain": [ "
" ] }, "metadata": { "needs_background": "light" }, "output_type": "display_data" }, { "name": "stdout", "output_type": "stream", "text": [ "Likelihood-Ratio: -1.3458889464848514\n" ] }, { "data": { "image/png": "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\n", "text/plain": [ "
" ] }, "metadata": { "needs_background": "light" }, "output_type": "display_data" }, { "name": "stdout", "output_type": "stream", "text": [ "Likelihood-Ratio: -1.6823611831060643\n" ] }, { "data": { "image/png": "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\n", "text/plain": [ "
" ] }, "metadata": { "needs_background": "light" }, "output_type": "display_data" }, { "name": "stdout", "output_type": "stream", "text": [ "Likelihood-Ratio: -2.0188334197272773\n" ] }, { "data": { "image/png": "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\n", "text/plain": [ "
" ] }, "metadata": { "needs_background": "light" }, "output_type": "display_data" }, { "name": "stdout", "output_type": "stream", "text": [ "Likelihood-Ratio: -2.35530565634849\n" ] }, { "data": { "image/png": "iVBORw0KGgoAAAANSUhEUgAAAW4AAAD8CAYAAABXe05zAAAABHNCSVQICAgIfAhkiAAAAAlwSFlzAAALEgAACxIB0t1+/AAAADl0RVh0U29mdHdhcmUAbWF0cGxvdGxpYiB2ZXJzaW9uIDMuMC4zLCBodHRwOi8vbWF0cGxvdGxpYi5vcmcvnQurowAAHTBJREFUeJzt3Xl03XWd//HnOzdNQtJ0SZsutE3TUmjZ2xJ2ZRcRsDjCKAoCc5CO6DBuZ+aMg/NzZvzhcRzHH/7Un05UHFRWERRBkX2o2JamC3Sne5puSZq0zULW+/79cW+ZkqbNN+2993u/N6/HOTm5yze5r0+WV24+9/v9fszdERGR6MgLO4CIiAyOiltEJGJU3CIiEaPiFhGJGBW3iEjEqLhFRCJGxS0iEjEqbhGRiFFxi4hETH46PunYsWO9srIyHZ9aRCQnLV26tNHdy4Nsm5birqyspKamJh2fWkQkJ5nZtqDbaqpERCRiVNwiIhGj4hYRiRgVt4hIxAQqbjP7opmtNrNVZvaImRWlO5iIiPRvwOI2s0nA3wJV7n4GEANuTncwERHpX9CpknzgBDPLB4qBnemLJCIiRzNgcbv7DuDbQC2wC9jv7s+nO5iISJS8uGYPP35tM/F4+peDDDJVMhq4AZgGnAiUmNmt/Ww338xqzKymoaEh9UlFRLLYr5fV8fNFW8nLs7Q/VpCpkquALe7e4O7dwJPARX03cvdqd69y96ry8kBHbYqI5AR3p2ZbM+dUjM7I4wUp7lrgAjMrNjMDrgTWpjeWiEh01DW/Q0NLJ+dMzZLidvfFwBPAMmBl8mOq05xLRCQyltU2AzA3Q8Ud6CRT7v414GtpziIiEklLtzVTUhBj5vjSjDyejpwUETlOS7c1M6diNPmxzFSqiltE5Di0dfawdteBjE2TgIpbROS4LKttJu5k7IVJUHGLiByXJVuayDOYWzEqY4+p4hYROQ5vbG3itBNHUFo0LGOPqeIWETlGXT1xltfu49zKsow+ropbROQYrdyxn86eOOepuEVEoqFmaxMAVSpuEZFoWLK1ieljSygvLczo46q4RUSOQTzuLNnanPH5bVBxi4gckw31rex/p5uqysztv32QiltE5Bi8kZzfPm+annGLiETCki1NjCstpKKsOOOPreIWERkkd2fJ1ibOnVZGYpmCzFJxi4gMUl3zO+za35Hx/bcPCrLm5EwzW3HI2wEz+0ImwomIZKMlyfntMPYogQALKbj7emA2gJnFgB3AU2nOJSKStRZvbmJEUT4zJ2Rm4YS+BjtVciWwyd23pSOMiEgULNy8l/OnjyGWgRXd+zPY4r4ZeCQdQUREoqCuuZ3apnYunD4mtAyBi9vMCoB5wK+OcP98M6sxs5qGhoZU5RMRySoLN+0F4MKTIlDcwIeAZe6+p7873b3a3avcvaq8vDw16UREsszCzXspKynI2MLA/RlMcX8CTZOIyBDm7izatJcLppeRF9L8NgQsbjMrBj4APJneOCIi2au2qZ2d+ztCnd+GALsDArh7OxBuUhGRkGXD/DboyEkRkcAWbt5LeWkhJ5UPDzWHiltEJAB3Z+GmvVwwfUwo5yc5lIpbRCSAzY1t1Ld0hj6/DSpuEZFADs5vXxTy/DaouEVEAlm4aS8TRxYxdUzmz7/dl4pbRGQA8bizcPNeLjwp/PltUHGLiAxoza4DNLV18f6Tx4YdBVBxi4gM6LUNifMvXTxDxS0iEgkL3m7k1IkjGFdaFHYUQMUtInJU7V09LN3WzCVZMk0CKm4RkaNavKWJrt4471Nxi4hEw4K3GynMzwttfcn+qLhFRI5iwYYGzptWRtGwWNhR3qXiFhE5gt37O9hQ38olJ2fX4jAqbhGRI1iQ3A0wm+a3QcUtInJECzY0MnZ4IbMmhLdMWX+CroAzysyeMLN1ZrbWzC5MdzARkTDF487rGxu55OSxWXGY+6ECrYADfBd4zt1vSq72Hv5ZVkRE0mjNrgPsbevKumkSCFDcZjYCuAS4A8Ddu4Cu9MYSEQnXy+vqMYNLT8muFyYh2FTJdKAB+JmZLTezn5hZSd+NzGy+mdWYWU1DQ0PKg4qIZNLL6+o5e/IoxgwvDDvKYYIUdz4wF/ihu88B2oB/6LuRu1e7e5W7V5WXZ99fKBGRoPa2dvJm3T4unzku7Cj9ClLcdUCduy9OXn+CRJGLiOSk/367AXe4YlZEi9vddwPbzWxm8qYrgTVpTSUiEqKX19VTXlrI6SeOCDtKv4LuVXIP8FByj5LNwF+lL5KISHh6euO89nYDHzx9Anl52bUb4EGBitvdVwBVac4iIhK6ZbX7ONDRk7XTJKAjJ0VE3uPldfXk51lW7r99kIpbROQQr66v59zKMkqLhoUd5YhU3CIiSTv2vcO63S1ZPU0CKm4RkXe9sq4egMtV3CIi0fDi2j1UlBVzUvlhB4dnFRW3iAjQ0tHNnzfu5YOnj8+6swH2peIWESFxtGRXb5yrT58QdpQBqbhFRIDnV+9hTEkBcytGhx1lQCpuERnyunrivLKunqtOHU8sS4+WPJSKW0SGvEWb99LS2cPVp48PO0ogKm4RGfKeX7Ob4oIYF8/I3qMlD6XiFpEhLR53Xlizh0tPKadoWCzsOIGouEVkSHtrx372HOiMzDQJqLhFZIh7fvVuYnnGFTNV3CIiWc/d+ePq3Zw/rYyRxdl7Uqm+Ap2P28y2Ai1AL9Dj7jo3t4hE3tt7WtnU0MYdF08LO8qgBF0BB+Byd29MWxIRkQx79q2d5BlcE4GjJQ+lqRIRGZLcnWdW7uKC6WMoLy0MO86gBC1uB543s6VmNj+dgUREMmHd7hY2N7Rx3VkTw44yaEGnSi52951mNg54wczWuftrh26QLPT5ABUVFSmOKSKSWs++tSuS0yQQ8Bm3u+9Mvq8HngLO62ebanevcveq8vLy1KYUEUkhd+fZlbu46KSxjBkerWkSCFDcZlZiZqUHLwNXA6vSHUxEJF3W7DrAlsZoTpNAsKmS8cBTyROL5wMPu/tzaU0lIpJGz761i1ie8cEITpNAgOJ2983A2RnIIiKSdv8zTTKGspKCsOMcE+0OKCJDyqodB9i2t51rz4zmNAmouEVkiPnNih0UxPL40BnRnCYBFbeIDCE9vXGefnMnl88qZ1RxNKdJQMUtIkPInzftpaGlk7+YMynsKMdFxS0iQ8Zvlu9gRFE+l80cF3aU46LiFpEhob2rh+dW7+a6syZGZqWbI1Fxi8iQ8PzqPbR39fIXcyaHHeW4qbhFZEh4avkOJo06gaqpo8OOctxU3CKS8xpaOlmwoYGPzDmRvDwLO85xU3GLSM57+s2dxJ3I701ykIpbRHKau/Ormu2cNXkkM8aVhh0nJVTcIpLTVu7Yz7rdLXysakrYUVJGxS0iOe2xJdspGpbHvNknhh0lZVTcIpKz3unq5ekVO7n2jImMKBoWdpyUUXGLSM76w6pdtHT28LFzc2eaBFTcIpLDHl2yncoxxZw/rSzsKCkVuLjNLGZmy83smXQGEhFJhS2NbbyxpYm/rJpCcgWvnDGYZ9yfB9amK4iISCo9XrOdPIObzon+Ie59BSpuM5sMXAf8JL1xRESOX3dvnCeW1nH5zHGMH1EUdpyUC/qM+37g74H4kTYws/lmVmNmNQ0NDSkJJyJyLJ5fvYeGlk4+eX5F2FHSYsDiNrPrgXp3X3q07dy92t2r3L2qvLw8ZQFFRAbr5wu3Mnn0CZE/7/aRBHnGfTEwz8y2Ao8CV5jZL9OaSkTkGL29p4XFW5q45fypxHLghFL9GbC43f0r7j7Z3SuBm4GX3f3WtCcTETkGv1i4jYL8PD6eY/tuH0r7cYtIzmjt7OHJZXVcf+ZEykqiuxjwQPIHs7G7vwq8mpYkIiLH6alldbR19fKpC6eGHSWt9IxbRHKCu/OLRds4c9JIZk8ZFXactFJxi0hOWLS5ibf3tPKpC6bm3JGSfam4RSQn/PRPmykrKcip07ceiYpbRCJvU0MrL66t59YLplI0LBZ2nLRTcYtI5P30T1soyM/jthx/UfIgFbeIRFpTWxe/XlrHR+dMYuzwwrDjZISKW0Qi7ZeLttHZE+fO900LO0rGqLhFJLI6unv5+cKtXDaznJPH58YK7kGouEUksp5esZPG1i7uev/0sKNklIpbRCKpN+786LVNnDZxBBedNCbsOBml4haRSPrDql1sbmjjc5fPyPkDbvpScYtI5Lg73395IyeVl3DNGRPCjpNxKm4RiZyX1tazbncLn7t8Rs6ec/toVNwiEinuzvde2ciUshOYd3buH97eHxW3iETK6xv38ub2fdx96QzyY0OzwoKsOVlkZm+Y2ZtmttrM/iUTwURE+vO9lzcwYUQRN54zKewooQny56oTuMLdzwZmA9eY2QXpjSUicrjXNzayeEsTf33pdArzc/9kUkcy4Ao47u5Aa/LqsOSbpzOUiEhf7s6//3E9E0cW8YnzKsKOE6pAE0RmFjOzFUA98IK7L05vLBGR93ppbT0rtu/j81eePCRO3Xo0gYrb3XvdfTYwGTjPzM7ou42ZzTezGjOraWhoSHVOERnC4nHn28+vp3JMMTeeMznsOKEb1Euy7r6PxGLB1/RzX7W7V7l7VXl5eYriiYjAMyt3sW53C1/8wCkMG6J7khwqyF4l5WY2Knn5BOAqYF26g4mIAPT0xrn/hbeZOb6UD581NPfb7mvAFyeBicCDZhYjUfSPu/sz6Y0lIpLwq6V1bG5s4z8/dQ55Q/Aoyf4E2avkLWBOBrKIiLxHa2cP//H8eqqmjubq08aHHSdraLJIRLLWD1/dSGNrF1+9/rQhdwbAo1Fxi0hWqmtu58cLtvCR2Scye8qosONkFRW3iGSlf//jegz4u2tmhR0l66i4RSTrLK9t5rcrdnLX+6czadQJYcfJOipuEckq8bjzr8+sYezwQj5z2Ulhx8lKKm4RySqP12xnee0+vvKhWQwvDLLH8tCj4haRrNHU1sU3n1vHeZVlfHTu0D1t60BU3CKSNb713DpaOnr4+kfO0O5/R6HiFpGssKy2mUeXbOfO901j5oTSsONkNRW3iISupzfOV59axYQRRXz+ypPDjpP1NPMvIqGrXrCZNbsO8KNb51KiFyQHpGfcIhKqjfUt3P/iBq49cwLXnDEx7DiRoOIWkdD0xp2/e+ItSgpi/Mu8w9ZnkSPQ/yQiEpqfvb6F5bX7+O7NsykvLQw7TmToGbeIhGJLYxvffn49V506jnlna4GEwQiyAs4UM3vFzNaa2Woz+3wmgolI7urujfPFx1ZQEMvjf3/kTO2zPUhBpkp6gC+7+zIzKwWWmtkL7r4mzdlEJEd976UNrNi+jx98ci4TRhaFHSdyBnzG7e673H1Z8nILsBbQsagickyWbG3i+69s5KZzJnPdWdqL5FgMao7bzCpJLGO2OB1hRCS3Hejo5guPrmBKWTH/PO/0sONEVuDiNrPhwK+BL7j7gX7un29mNWZW09DQkMqMIpID3J17n1rF7gMd3P/x2Trz33EIVNxmNoxEaT/k7k/2t427V7t7lbtXlZeXpzKjiOSAXyzaxu/e3MmXPnAKcypGhx0n0oLsVWLAT4G17v6d9EcSkVyzrLaZrz+zhitnjePuS7U4wvEK8oz7YuBTwBVmtiL5dm2ac4lIjtjb2snnHlrGhJFFfOdjs8nL065/x2vASSZ3/xOgr7SIDFpv3PnCYyvY29bFk3dfxMjiYWFHygk6clJE0ua+Z9eyYEMjX7/hdM6YNDLsODlDxS0iafHw4loeeH0Ld1xUycfPrQg7Tk5RcYtIyv15UyP/67eruPSUcr563alhx8k5Km4RSanNDa3c/ctlVI4t4XufnEN+TDWTavqKikjK1B/o4LYH3iCWZ/z09ipGFOnFyHRQcYtIShzo6Oa2B96gqa2Ln91xLlPHlIQdKWepuEXkuHV093LXgzVsrG/lR7eew9lTRoUdKafpZAEicly6e+Pc88hyFm9p4rs3z+aSU3TKi3TTM24ROWbdvXH+9pHlvLBmD/8y73RumK0zPmeCiltEjklPb5wvPLqCP6zazT9dfxq3X1QZdqQhQ8UtIoPW0xvni4+/ybMrd/HV607lzvdNCzvSkKI5bhEZlI7uXv7m4WW8uLaer3xoFp9+//SwIw05Km4RCexARzeffrCGJVub+PoNp/OpCyvDjjQkqbhFJJDG1k5uf+AN1u9u4f6Pz9YLkSFScYvIgDbWt3Lng0vYc6CDn9xexWUzx4UdaUhTcYvIUf1pQyN3P7SUwvw8Hr7rAuZq2bHQBVm67AEzqzezVZkIJCLZ4xeLtnH7z97gxJEn8JvPXazSzhJBdgf8L+CaNOcQkSzS0d3LPz61kn/6TeLUrL/+7EVMHl0cdixJCrJ02WtmVpn+KCKSDbY3tXP3Q0tZteMAf33pdP7+g7OIaZ3IrKI5bhF51wtr9vDlx1cA8OPbqvjAaeNDTiT9SVlxm9l8YD5ARYWWKRKJkvauHr7x+7X8clEtZ04ayf+7ZS5TyjQ1kq1SVtzuXg1UA1RVVXmqPq+IpNfy2ma+9PibbN3bxl3vn8aXr55J0bBY2LHkKDRVIjJEdXT38v2XN/LD/97EhBFFPPzpC7jwpDFhx5IABixuM3sEuAwYa2Z1wNfc/afpDiYi6fP6xkbufWolW/e2c+PcyXxt3mlaZixCguxV8olMBBGR9Gts7eQbz67lyeU7qBxTzC/vPJ/3nTw27FgySJoqERkCOrp7efDPW/n+yxvp6Onlnitm8LnLZ2guO6JU3CI5zN155q1d/Ntz66hrfocrZo3jH6+dxYxxpWFHk+Og4hbJQe7Oq+sbuP/Ft3mzbj+nThzBQ58+i4tnaFokF6i4RXJI38KePPoEvnXTWdw4d7KOfswhKm6RHNDdG+f3K3fxkwVbWLkjUdj/duOZfHTuZIbFtEJhrlFxi0TY/vZuHn6jlgf/vJXdBzqYPraEb370TG48R4Wdy1TcIhHj7izZ2sxjS7bz+5W7eKe7l4tnjOEbHz2Dy04ZR56mRHKeilskIuoPdPDk8h08vmQ7mxvbGF6Yz0fmnMhtF1Zy6sQRYceTDFJxi2SxxtZO/rBqN8++tZPFW5pwh6qpo7n7spO47qyJFBfoV3go0nddJMtsb2rn1fX1PLd6Nws37SXuML28hHuuOJl5Z5/IjHHDw44oIVNxi4SsqyfOkq1NvLq+nlfWN7CxvhWAaWNL+OxlM7jurInMmlCKmeauJUHFLZJhHd29LK/dxxtbmli8ZS/Lapvp6I5TEMvj/OllfOK8Ci6fWc60sSUqa+mXilskjdydbXvbeWvHflbW7WPF9n28uX0/Xb1xzODUCSO4+dwKLjppDBfPGEtJoX4lZWD6KRFJkfauHjbVt7GhvoX1e1pYtWM/K+v2c6CjB4CC/DxOmziCOy6u5PxpZVRNLWNksU6lKoOn4hYZhO7eOLv2dbC9uZ3apna2Nraxob6VDfUt1DW/gyfXfhoWM2ZNGMH1Z5/IWZNGcubkkZwyvlQHxUhKBCpuM7sG+C4QA37i7t9MayqREPTGnb1tndQf6GTPgQ72HOikvqXjPUW9a38HvfH/WZmvIJbH9PISZk8ZzV+eM4VTxg9nxrhSKscUk6+SljQJsgJODPgB8AGgDlhiZk+7+5p0hxM5Fu5OZ0+c9q5eWjq62dfeTfOzf2T/I7+iua2LfeMns++yq9g3ZRrN7d00tXVR39JBY2vXe0r5oLHDC5lSdgLnTB3NlNHFVJQVM7nsBCrKipkwoihrC/qhlQ9x70v3Uru/loqRFdx35X3ccuYtYceSFAjyjPs8YKO7bwYws0eBGwAVt+DuxB164nHiceh1p7fXE+/jyTd34nGnJ3k97k5Pb+J9V2+czu44Xb1xunridPb00tVz8PJ7b+tMbtvZE+edrh5aO3tp7+qhrbOHtq5e2jt7aO3sob2rl57DCng4XPhX714r3dnG6JbtjBpXRllJAbMmlDJ+RBHjRxRSXpp4P35EEWOHF1KQn53FfDQPrXyI+b+bT3t3OwDb9m9j/u/mA6i8c0CQ4p4EbD/keh1wfjrCXP+9BXR0x4FEIRzqsOdBfuSrA32sH/axfuT7Bliv/tDHGszj9L1/oI/tu8XRP3aQ4z/KGPreEPeDRZws6wG+PqlUkJ9HYfKtuCCfksJ8SgpijCouYNLoGCUHbyuMJe4viDG8aBij7/kMo7ZtYlRHC6PeaWFkRyv5HoepU2Hr1swNIIPufened0v7oPbudu596V4Vdw4IUtz97Uh62K+rmc0H5gNUVFQcU5gZ5cPp7j3kU/d55L5B+u7jau+579g/9vDH7bPtUT734fcN8LFHeeCBx3BsGRP3H3n/4KONwQxieUaeGfl5Rl6eETMjP5a4LZYHsbw8YsntYnl5xPJIbP/uNsmPNWPYIWVcEItROCyPgljeuyWdeB9jWMyOfZ/mRb/v/y9wbe2xfb4IqN3f/9iOdLtES5DirgOmHHJ9MrCz70buXg1UA1RVVR3T87D7b55zLB8mcnQVFbBtW/+356iKkRVs23/4mCtG5u6Yh5Igk3dLgJPNbJqZFQA3A0+nN5ZICt13HxQXv/e24uLE7Tnqvivvo3jYe8dcPKyY+67M3TEPJQMWt7v3AH8D/BFYCzzu7qvTHUwkZW65BaqrE3PaZon31dWJ23PULWfeQvWHq5k6ciqGMXXkVKo/XK357RxhfV/ISoWqqiqvqalJ+ecVEclVZrbU3auCbBu9/ZxERIY4FbeISMSouEVEIkbFLSISMSpuEZGIUXGLiESMiltEJGJU3CIiEZOWA3DMrAHo5+QQgYwFGlMYJwo05tw31MYLGvNgTXX38iAbpqW4j4eZ1QQ9eihXaMy5b6iNFzTmdNJUiYhIxKi4RUQiJhuLuzrsACHQmHPfUBsvaMxpk3Vz3CIicnTZ+IxbRESOIrTiNrNrzGy9mW00s3/o5/5CM3ssef9iM6vMfMrUCTDeL5nZGjN7y8xeMrOpYeRMpYHGfMh2N5mZm1nk90AIMmYz+1jye73azB7OdMZUC/CzXWFmr5jZ8uTP97Vh5EwVM3vAzOrNbNUR7jcz+7/Jr8dbZjY35SHcPeNvQAzYBEwHCoA3gdP6bPNZ4EfJyzcDj4WRNYPjvRwoTl6+O8rjDTrm5HalwGvAIqAq7NwZ+D6fDCwHRievjws7dwbGXA3cnbx8GrA17NzHOeZLgLnAqiPcfy3wBxJrdF8ALE51hrCecZ8HbHT3ze7eBTwK3NBnmxuAB5OXnwCutGNe5jt0A47X3V9x9/bk1UUkFmWOsiDfY4CvA98COjIZLk2CjPku4Afu3gzg7vUZzphqQcbswIjk5ZH0s9h4lLj7a0DTUTa5Afi5JywCRpnZxFRmCKu4JwHbD7lel7yt3208se7lfmBMRtKlXpDxHupOEn+xo2zAMZvZHGCKuz+TyWBpFOT7fApwipm9bmaLzOyajKVLjyBj/mfgVjOrA34P3JOZaKEZ7O/7oOWn8pMNQn/PnPvu3hJkm6gIPBYzuxWoAi5Na6L0O+qYzSwP+D/AHZkKlAFBvs/5JKZLLiPxX9UCMzvD3felOVu6BBnzJ4D/cvf/MLMLgV8kxxxPf7xQpL27wnrGXQdMOeT6ZA7/9+ndbcwsn8S/WEf79ySbBRkvZnYVcC8wz907M5QtXQYacylwBvCqmW0lMRf4dMRfoAz6c/1bd+929y3AehJFHlVBxnwn8DiAuy8Eikic0yNXBfp9Px5hFfcS4GQzm2ZmBSRefHy6zzZPA7cnL98EvOzJmf8IGnC8yWmD/yRR2lGf94QBxuzu+919rLtXunsliXn9ee5eE07clAjyc/0bEi9EY2ZjSUydbM5oytQKMuZa4EoAMzuVRHE3ZDRlZj0N3Jbcu+QCYL+770rpI4T4yuy1wNskXpG+N3nbv5L45YXEN/dXwEbgDWB62K8mp3m8LwJ7gBXJt6fDzpzuMffZ9lUivldJwO+zAd8B1gArgZvDzpyBMZ8GvE5ij5MVwNVhZz7O8T4C7AK6STy7vhP4DPCZQ77HP0h+PVam4+daR06KiESMjpwUEYkYFbeISMSouEVEIkbFLSISMSpuEZGIUXGLiESMiltEJGJU3CIiEfP/AYWCa5SsJwWXAAAAAElFTkSuQmCC\n", "text/plain": [ "
" ] }, "metadata": { "needs_background": "light" }, "output_type": "display_data" }, { "name": "stdout", "output_type": "stream", "text": [ "Likelihood-Ratio: -1.8444800325824995\n" ] }, { "data": { "image/png": "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\n", "text/plain": [ "
" ] }, "metadata": { "needs_background": "light" }, "output_type": "display_data" }, { "name": "stdout", "output_type": "stream", "text": [ "Likelihood-Ratio: -2.180952269203712\n" ] }, { "data": { "image/png": "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\n", "text/plain": [ "
" ] }, "metadata": { "needs_background": "light" }, "output_type": "display_data" }, { "name": "stdout", "output_type": "stream", "text": [ "Likelihood-Ratio: -1.6701266454377217\n" ] }, { "data": { "image/png": "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\n", "text/plain": [ "
" ] }, "metadata": { "needs_background": "light" }, "output_type": "display_data" }, { "name": "stdout", "output_type": "stream", "text": [ "Likelihood-Ratio: -2.0065988820589347\n" ] }, { "data": { "image/png": "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\n", "text/plain": [ "
" ] }, "metadata": { "needs_background": "light" }, "output_type": "display_data" }, { "name": "stdout", "output_type": "stream", "text": [ "Likelihood-Ratio: -2.3430711186801476\n" ] }, { "data": { "image/png": "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\n", "text/plain": [ "
" ] }, "metadata": { "needs_background": "light" }, "output_type": "display_data" }, { "name": "stdout", "output_type": "stream", "text": [ "Likelihood-Ratio: -1.8322454949141571\n" ] }, { "data": { "image/png": "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\n", "text/plain": [ "
" ] }, "metadata": { "needs_background": "light" }, "output_type": "display_data" }, { "name": "stdout", "output_type": "stream", "text": [ "Likelihood-Ratio: -2.16871773153537\n" ] }, { "data": { "image/png": "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\n", "text/plain": [ "
" ] }, "metadata": { "needs_background": "light" }, "output_type": "display_data" }, { "name": "stdout", "output_type": "stream", "text": [ "Likelihood-Ratio: -2.505189968156583\n" ] }, { "data": { "image/png": "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\n", "text/plain": [ "
" ] }, "metadata": { "needs_background": "light" }, "output_type": "display_data" }, { "name": "stdout", "output_type": "stream", "text": [ "Likelihood-Ratio: -2.8416622047777955\n" ] }, { "data": { "image/png": "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\n", "text/plain": [ "
" ] }, "metadata": { "needs_background": "light" }, "output_type": "display_data" }, { "name": "stdout", "output_type": "stream", "text": [ "Likelihood-Ratio: -2.330836581011805\n" ] }, { "data": { "image/png": "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\n", "text/plain": [ "
" ] }, "metadata": { "needs_background": "light" }, "output_type": "display_data" }, { "name": "stdout", "output_type": "stream", "text": [ "Likelihood-Ratio: -2.667308817633018\n" ] }, { "data": { "image/png": "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\n", "text/plain": [ "
" ] }, "metadata": { "needs_background": "light" }, "output_type": "display_data" }, { "name": "stdout", "output_type": "stream", "text": [ "Likelihood-Ratio: -3.003781054254231\n" ] }, { "data": { "image/png": "iVBORw0KGgoAAAANSUhEUgAAAW4AAAD8CAYAAABXe05zAAAABHNCSVQICAgIfAhkiAAAAAlwSFlzAAALEgAACxIB0t1+/AAAADl0RVh0U29mdHdhcmUAbWF0cGxvdGxpYiB2ZXJzaW9uIDMuMC4zLCBodHRwOi8vbWF0cGxvdGxpYi5vcmcvnQurowAAIABJREFUeJzt3XlYXOd9L/Dvb4ZhZ1gHGLEKtICEdmxJkW1tXhSvieMkTmUnNzetbuIsbpo2y3Xvvc5t1bS5T1L3ZqmtJk7SRHEcx04sO16q2mBbthYjC0kIgVY2sQw7Yh9m3v4x4GCExAAzc+ac8/08D8/DMNv3CPjq8J73PUeUUiAiIv2waB2AiIhmh8VNRKQzLG4iIp1hcRMR6QyLm4hIZ1jcREQ6w+ImItIZFjcRkc6wuImIdCYiGC+alpam8vPzg/HSRESGdPTo0Q6llMOfxwaluPPz81FRURGMlyYiMiQRqff3sRwqISLSGRY3EZHOsLiJiHSGxU1EpDMsbiIinWFxExHpDIubiEhngjKPm4jMQymFo/XdONbQgzGvwrIFdmwqTEWElfuFwcLiJqI5O97Yg0f+cBJVl/o+8PX81Fj8w0dX4EOL0jRKZmwsbiKak98fa8LXf3cCKXGR+O7HVuLW5RmIsFpw4Gw7vvtqLXb+9DC+89EVuP/6XK2jGg6Lm4hm7fnKS/jq08exsSAVjz+4Dokxtvfv21HixE1LHPjCr97Dt35/EnFREbhr1QIN0xoPB6GIaFaO1nfhr585jg0FKfj5f7/uA6U9ITYyAk88uA6lecn4xrMncM7Vr0FS42JxE5Hf+obd+MpTlXAmxuCJB0oRFWG96mOjbVb84FNrEWOz4qtPV8LjVSFMamwsbiLy26PPn0Jr3zAeu381EmOv3NOeKjMxGo/evRwnL/Xi14f9PvkdzYDFTUR+eed8B547dgkPbSnE2txkv59350onbliUhv/3ai16h9xBTGgeLG4imtGYx4tv76tGdnIMvrh10ayeKyL41u1F6Bsew08PXAxSQnNhcRPRjH7zbiNq2y7jb+9Yhmjb1ce1r2b5gkR8uCQTTx64iN5B7nXPF4ubiK5pZMyDH5Wdw7q8ZNy2PGPOr/PlbYvRPzKGpysaApjOnFjcRHRNT7/biJbeYXz15iUQkTm/zrIFdqxfmIJfvFPPGSbzxOImoqsadnvw47LzuC4/GZsWpc779T67aSEu9Qxhf3VbANKZF4ubiK7qD8cuobVvGA9vn9/e9oRblmUgKykGezk1cF5Y3EQ0LaUUfvZ2HYqd9oDsbQOA1SK4b102DpzrQEvvUEBe04z8Lm4RsYrIMRF5MZiBiCg8HDzfidq2y/jspvyA7G1PuHdtFpQC/nCsOWCvaTaz2eN+GMDpYAUhovDy5NsXkRoXibsDfIKovNQ4XJefjGffa4JSPEg5F34Vt4hkA7gDwE+CG4eIwkF95wBeq3Hhz9bnzmne9kzuXZuNc67+K87jTf7xd4/7MQBfB+ANYhYiChO/PtIAiwge2JAXlNffsTwTVovg5aqWoLy+0c1Y3CJyJwCXUuroDI/bJSIVIlLR3t4esIBEFFpujxfPHr2ErUvTkWGPDsp7JMdFYmNBKl6pauVwyRz4s8e9CcDdIlIH4DcAtonIr6Y+SCm1RylVqpQqdTgcAY5JRKFSXtuOjv4RfPK6nKC+z20lmbjQMYAzbTxX92zNWNxKqW8ppbKVUvkA7gfwulLqgaAnIyJNPP1uIxwJUdi6NLg7YLctz4AI8EpVa1Dfx4g4j5uI3ufqG0ZZrQv3rs0K+lXa0xOisS43Gf9RzeKerVl9Z5RS5UqpO4MVhoi09ftjl+DxKnyiNLjDJBO2FqXjVHMfXJeHQ/J+RsE9biJ63x8qm7E6JwmFjviQvN/mJb7hmDdqOaFhNljcRAQAONt2Gadb+nDP6tBdkX35AjvSE6JQfobFPRssbiICAOw73gyLAHesdIbsPUUEm5c48NaZdox5uEzEXyxuIoJSCvuON2NjYSrSE4Izd/tqtixNR9/wGCobe0L6vnrG4iYiHG/qRX3nIO5ZlRXy975hcRqsFsEbHC7xG4ubiLCvshmRVgtuK8kM+XsnxtiwMjsR75zvDPl76xWLm8jkvF6FF080Y/NSBxJjbJpk2FiQiuONPRgYGdPk/fWGxU1kcscau+G6PII7Q3hQcqqNhakY8yq8W9elWQY9YXETmdwrVa2ItFqwrShdswyleSmwWQUHOVziFxY3kYkppfDKqVZsWpSKhGhthkkAICbSijU5yTh4gcXtDxY3kYlVt/ShsWsIOzQ4KDnVhsJUVF3qRe+QW+soYY/FTWRir1S1wiLAzcUZWkfBxoJUeBVw5CLHuWfC4iYysVeqWnH9whSkxkdpHQVrcpNgswoqeIByRixuIpM65+rHWVc/dizXfpgEAKJtVpRkJaKivlvrKGGPxU1kUq+e8p0H+9YwKW4AKM1LxsmmXgy7PVpHCWssbiKTevVUK1blJGFBUozWUd63Li8Fox4vqi71ah0lrLG4iUyorW8YJ5p6cdty7Q9KTrYuLxkAOFwyAxY3kQmV17oAQNNFN9NxJEQhPzUWFXUs7mthcROZUFlNO5yJ0ViakaB1lCusy0vBew3dUEppHSVssbiJTGZ0zIsD5zqwZWk6RETrOFcozU9G18AoLnYMaB0lbLG4iUymor4L/SNj2LrUoXWUaZVynHtGLG4ikymvbUek1YJNi9K0jjKtQkc87NERONbAK+JcDYubyGTKalxYX5CCuKgIraNMy2IRrMpJwnFeyuyqWNxEJtLYNYizrn5sWRpes0mmWpWdhNq2yxga5UKc6bC4iUykfPy6juE6vj1hVU4SPF6F6hYuxJkOi5vIRMprXMhLjcXCtDito1zTquxEAEBlI4t7OixuIpMYdnvw9vkObA3TaYCTpduj4UyM5jj3VbC4iUzi8MUuDLu92BLmwyQTVmUn4UQTi3s6LG4ikyircSHaZsGGglSto/hlVU4S6joH0TM4qnWUsMPiJjKJ8loXPlSYhmibVesofpkY5z7exHHuqVjcRCZwsWMAdZ2DYT+bZLKS7ESIgOPc02BxE5nA6zW+swGG+/ztyezRNhQ64jnOPQ0WN5EJlNe6sCg9HjkpsVpHmZVV2UmobOzlmQKnYHETGdzAyBgOX+jS1TDJhBVZdnT0j8B1eUTrKGGFxU1kcO+c78Sox4utOhommbA8y3eAkpcy+6AZi1tEokXkiIgcF5FTIvLtUAQjosAoq3UhLtKK0vwUraPMWrHTDhHgVHOf1lHCij+nBxsBsE0p1S8iNgAHRORlpdShIGcjonlSSqG8xoUbFqchMkJ/f2DHR0VgYVoc97inmPE7qXz6x2/axj94pIBIB8609aO5d1iXwyQTli9I5B73FH79FywiVhGpBOACsF8pdTi4sYgoEMpq9TcNcKqSBXZc6hlC9wBXUE7wq7iVUh6l1GoA2QCuF5GSqY8RkV0iUiEiFe3t7YHOSURzUFbjQrHTjszEaK2jzNnyBb4DlNUt3OueMKtBL6VUD4ByADumuW+PUqpUKVXqcOhv2hGR0fQNu1FR363LaYCTLV9gB8CZJZP5M6vEISJJ45/HALgZQE2wgxHR/Bw42wGPV2FrkX6HSQAgOS4SWUkxHOeexJ9ZJU4AvxARK3xF/1ul1IvBjUVE81VW44I9OgJrcpK0jjJvyxfYUdXMPe4JMxa3UuoEgDUhyEJEAeL1KpSfacdNSxyIsOpvGuBUyxckYv/pNgyMjIXtRY5DSf/fUSK6QnVLH9ovj+h6GuBkJVl2KAWc5gFKACxuIkMqq3FBBNis8wOTEyZmlvAApQ+Lm8iAympdWJmdhLT4KK2jBESGPQpp8ZGo4gFKACxuIsPpGhjFscYe3U8DnExEUOy0c6hkHIubyGDeOtsOpWCY8e0Jy5x2nG3rx5jHq3UUzbG4iQzm9RoXUuMisWL8lKhGUeRMwKjHiwsdA1pH0RyLm8hAPF6FN860Y/NSBywW0TpOQBU7fSsoOVzC4iYylMrGHvQMug03TAIAhY542KyC0y2XtY6iORY3kYGU17pgEeCmxcY5MDnBZrVgUXoC97jB4iYylLJaF9blJSMx1qZ1lKAodiagppXFzeImMghX3zCqLvXp+tzbMynOtKOtbwRdJj83N4ubyCDKz/jOg2/E8e0JEwcoa0w+XMLiJjKI8loXMuxRKHYmaB0laIrGt83sF1VgcRMZgNvjxVtnOrB1aTpEjDUNcLK0+Cg4EqJMP7OExU1kAEfru3F5ZMzQ49sTip120x+gZHETGUBZrQs2q2DTolStowRdcWYCzrb1w23ipe8sbiIDKKtx4br8FCREG3Ma4GTFTjtGPV5cNPHSdxY3kc41dQ/iTFs/tun82pL+mjhAaeaFOCxuIp0rq3EBgCnGt4E/LX0388wSFjeRzr1e40JuSiwKHXFaRwmJiaXvNSaeWcLiJtKxoVEP3jnfiW1Fxp4GOFWx09znLGFxE+nYwQsdGBnzYqtJxrcnFGfa4bo8gs7+Ea2jaILFTaRjr9e4EGOzYv3CFK2jhNT7S99bzTlcwuIm0imlFMpq2rFpURqibVat44RUsclnlrC4iXTqTFs/LvUMmWYa4GSp40vfzTqzhMVNpFOvj08D3FpkvIsm+KPYaUcth0qISE/KalwodtrhTIzROoomzLz0ncVNpEO9g24cbejGNpPubQN/uuq7GZe+s7iJdOiNs+3weJUpx7cnmPmq7yxuIh0qq3EhOdaG1TnJWkfRTEGab+m7GacEsriJdMbjVSivdWHzEgesFvOslpwqMsKCQkc897iJKPxVNvage9BtutWS0yl22k15zhIWN5HOvHa6DVaLYPMS8x6YnFDsTEBr3zC6TXbVdxY3kc7sr27D9fkpSIqN1DqK5ooyzbn0ncVNpCN1HQM46+rHLcsytI4SFsx6UQUWN5GO7K9uAwAW97j0hGikxUea7uLBMxa3iOSISJmInBaRUyLycCiCEdGV9le3oSgzATkpsVpHCRtFmXYOlUxjDMDXlFLFADYA+KKILAtuLCKaqmtgFBX1XbiVe9sfUJSZgNrWyxgz0dL3GYtbKdWilHpv/PPLAE4DyAp2MCL6oNdOt8GrgFuWZWodJawUOe0YGfOirnNQ6yghM6sxbhHJB7AGwOFp7tslIhUiUtHe3h6YdET0vv3VbXAmRqMky651lLAycW5uM41z+13cIhIP4FkAf6mUuuJfSCm1RylVqpQqdTg4v5QokIbdHrx1tgM3F2eY6tqS/liUHg+rRUy1EMev4hYRG3ylvVcp9VxwIxHRVAfOdmDI7eFskmlERVhR6Igz1ZRAf2aVCICfAjitlPp+8CMR0VT7q9uQEBWBDQWpWkcJS2abWeLPHvcmAA8C2CYileMftwc5FxGN83gVXqtpw+alDkRGcOnFdIqddlzqGULvkFvrKCERMdMDlFIHAHBQjUgjR+u70dE/ituWczbJ1UysoKxtvYzrTXDFe/73TRTmXjrZgqgIC88GeA3Fmea6qAKLmyiMeb0KL1e1YPMSB+KjZvwD2bQy7FFIjrWZZkogi5sojB1r7EZb3whuX+HUOkpYExEUZdpx2iRTAlncRGHspZOtiLRasK2YwyQzKXL6lr57vUrrKEHH4iYKU0opvHyyBTcuToM92qZ1nLBXnGnHkNuD+i7jL31ncROFqeNNvWjuHcaHOUzil4mrvteY4AAli5soTL18sgURFsEtxVwt6Y/FGfGwCHDaBAtxWNxEYUgphZeqWrBpURoSYzlM4o9omxUL08yx9J3FTRSGTjX3obFrCLev4KKb2Sh22k0xJZDFTRSGXjjR7Bsm4bm3Z6XYaUdj1xAuDxt76TuLmyjMeL0KL1Q246YlDqTE8Urus1GU6Vv6fqbN2OPcLG6iMFNR343m3mHcs3qB1lF0p2h8Zkm1wRfisLiJwszzlZcQY7PiZs4mmbUFidGwR0cYfkogi5sojIyOefHHky24ZVkG4nhuklkTERQ5jX9ubhY3URg5cK4dPYNuDpPMQ3Gm8Ze+s7iJwsjzlc1IirXhxsW8butcFTvt6B8ZQ1P3kNZRgobFTRQmBkfH8B+n2nD7CievdDMPEwcoTxt4Pjd/OojCxP7qNgy5PbhnFYdJ5mNJRjxEYOirvrO4icLE7442ISspBtflG//SW8EUGxmB/FRjL31ncROFgUs9QzhwrgP3rcuGxcJLvM5XsTPB0EvfWdxEYeC5o01QCrhvXbbWUQyhONOOus5Bwy59Z3ETaczrVXjmaBM2FqQiJyVW6ziGUJKVCACobjbmXjeLm0hjR+q60NA1iI+Xcm87UCaK++SlXo2TBAeLm0hjz1Q0IT4qAh8u4ZVuAsWREIVMezROcY+biAKtf2QML51swZ0rnYiJtGodx1BKsuzc4yaiwPvjiWYMuT0cJgmCkqxEnG/vx8DImNZRAo7FTaShvYcbsCg9Hmtzk7WOYjgrshKhFFBtwPncLG4ijZxo6sGJpl48sD4XIpy7HWgTByirDDhcwuIm0sivDtUjxmbFvZy7HRQZ9mg4EqIMOc7N4ibSQO+gG/uON+MjaxbAHs2ruAfLiqxE7nETUWA8+14Tht1e7Fyfp3UUQytZYMc5Vz+GRj1aRwkoFjdRiCmlsPdwPVbnJL0/DkvBUZKVCK8BD1CyuIlC7O1znTjfPoAHNnBvO9hWZBvzACWLmyjE/u2tC0iLj8Jdq7hSMtgy7dFIjYs03AFKFjdRCJ1pu4w3zrTjMxvzEBXBlZLBJiJYmZ2IE009WkcJqBmLW0SeFBGXiFSFIhCRkf30rYuItlmwk8MkIbMmNxlnXf3oM9ApXv3Z4/45gB1BzkFkeO2XR/D7Y5fwsbXZSImL1DqOaazOSYJSwIlG4wyXzFjcSqk3AXSFIAuRof3yUD1GPV587oaFWkcxlVU5SQCAysZujZMEDse4iUJgcHQMvzpUj5uL01HgiNc6jqkkxtiwKD0exxqMM84dsOIWkV0iUiEiFe3t7YF6WSJD2HuoAV0Do/jClkVaRzGl1TlJONbYA6WU1lECImDFrZTao5QqVUqVOhyOQL0ske4Nuz144s0L2LQoFevyeBZALazJTULXwCgauga1jhIQHCohCrKnjjSgo38EX9m2WOsoprUmx/cfplGGS/yZDvgUgIMAlopIk4h8LvixiIxh2O3B42+cx/ULU7C+IFXrOKa1JCMesZFWVDYao7gjZnqAUupToQhCZETPHG1CW98Ivvfx1VpHMbUIqwUrshJxrMEYM0s4VEIUJEOjHvzw9bNYl5eMTYu4t621NbnJONXch2G3/s8UyOImCpKfvXMRbX0j+PptS3mFmzCwNjcJY16F4wYYLmFxEwVBz+Ao/rX8PLYXpXNsO0xcl58CADhyUf/rCVncREHw4/Lz6B8Zw9/sWKp1FBqXHBeJoswEHKljcRPRFE3dg/j5O3W4d002ijLtWsehSdYvTMHR+m64PV6to8wLi5sowP7+xdOwiuBrty7ROgpNsb4gFYOjHt2fn5vFTRRAB8524JVTrfjStkVYkBSjdRyawijj3CxuogBxe7x49IVTyEuNxZ/fyDMAhiNHQhQKHXE4fKFT6yjzwuImCpCfvX0R51z9+N93LuPVbcLY+oJUVNR1w+PV7wmnWNxEAVDfOYDv7z+Dm4szsL04Q+s4dA3rF6bg8sgYqpv1e+V3FjfRPCml8M1nT8JmseDvP1KidRyawcZC37z6t87p9/TTLG6ieXrqSCMOXujE/7yjGJmJ0VrHoRmkJ0Sj2GnHm2dY3ESm1Ng1iH946TQ2FqTi/utytI5Dftq8xIGj9d0YGBnTOsqcsLiJ5mjM48XDvzkGEeC7963k+Uh05KYlaXB7FA6e1+fsEhY30Rz9y2tn8V5DD3Z/dAVyUmK1jkOzUJqXgthIK97Q6XAJi5toDg6e78QPy87hvnXZuHvVAq3j0CxFRliwsSAVb55lcROZwqWeIXzx1++hIC0O3757udZxaI42L3WgvnMQ59v7tY4yayxuolkYGvVg179XwD3mxZ5PlyIuasaLSFGYumWZb779K1WtGieZPRY3kZ+8XoVvPHsC1S19eOz+1Sh0xGsdiebBmRiD1TlJePUUi5vIsP7p1RrsO96Mv751KVdHGsSOkkycaOpFU/eg1lFmhcVN5IefHriIJ964gAc35OGhLYVax6EA2bE8EwDw6qk2jZPMDoubaAbPVDTi716sxodLMvHo3cs5X9tA8tPiUJSZgFeqWrSOMissbqJr+M2RBnz92RO4cXEa/vmTq2G1sLSN5o4VTrxb143GLv0Ml7C4ia7ilwfr8M3nTmLzEgf+7dOliLbxVK1GdO+6bIgAz77XpHUUv7G4iabwehX+8eUa/K/nT+Hm4nQ88eA6lraBZSXF4EOFqfjd0SZ4dXKObhY30SRDox48/HQlHn/jPP5sfS4ef2AdL4pgAh9fl4Om7iEcuqiPc5ewuInGXWjvx0d//DZeON6Mb+wowu6PlCDCyl8RM9hRkonEGBt+ebBe6yh+4U8lEYAXjjfj7h++jba+Yfz8s9fhC1sKOXvERKJtVuxcn4tXT7WioTP8D1KyuMnU2i+P4KG9R/Hlp45hSUY8/viVG7FlabrWsUgDn/lQPqwWwZNvX9Q6yox4ogUyJY9X4XdHG/Gdl2swOOLB39y2FLtuKoCNQyOmlWGPxl2rFuDpdxvx0JZCpNvD92pG/Ckl0zlwtgN3/uAAvvHsSRQ64vHSwzfgi1sXsbQJX9m2GG6PF4+9dlbrKNfEPW4yBaUU3jzbgX8tP4dDF7qQnRyDH3xqDe5c6eRYNr0vPy0OD2zIwy8P1eOzH8rH4owErSNNi8VNhjYwMoY/nmzBL96pw6nmPmTao/G3dxTjgQ15nJtN0/rK9sV47r0mPPL7Kjy1a0NYrpZlcZPhjHm8eLeuG/uOX8K+ymYMjHpQ6IjDdz+2Eh9Zk4XICA6J0NWlxEXi/9y1HF975jh+8tYF/I/N4XdSMRY3GUL3wCgOX+zEf5524bXTbegedCPGZsUdK5345HU5KM1L5pAI+e3etVnYX92Gf3qlBkszE8JuppFfxS0iOwD8CwArgJ8opf4xqKmIrmHM48WFjgFUN/ehsrEHhy50oqb1MgAgIToC24vScdvyTNy0xMEr1NCciAi+94lVuO/xQTy09z3sebAUNyxO0zrW+0Spa6/NFxErgDMAbgHQBOBdAJ9SSlVf7TmlpaWqoqIikDnJZLxehc6BUTR0DaKhawANnUOo7xrAOVc/alovY3TMCwCItllQmpeCDQUp2FCQipXZSdMPhezdCzzyCNDQAOTmArt3Azt3hnirQmvvyb145LVH0NDbgNzEXOzevhs7Vxh7mwPN1TeMTz95BOfb+/FXtyzFX9y4MGiraUXkqFKq1K/H+lHcGwE8qpS6bfz2twBAKfWdqz2HxW0+Xq+C2+uF26Mw5vFi1OP73D3mhdvjxciYF4OjHvSPuNE/4sHAyBj6h8fQP+L76B4cRfvlEXT0j6KjfwRdA6PwTDnhjzMxGgWOOCxz2rFsgR3LnIkocMTNPI1v715g1y5gcNKKuNhYYM8ew5b33pN7seuFXRh0/2mbY22x2HPXHpb3LPUOuvHN507g5apW5KbE4v7rc7B5iQOFjviAHuAOdHHfB2CHUurPx28/CGC9UupLV3vOXIv7zh+8hWG3b09qaq4rUqpr3pzx+VM3W015xOT7Z/gnCuh7zfz82T53ltlm8e+u4FvI4vZ4MTaPs6rFRlqRHBuJtPhIpMVH+T4SIuGIj0JuaixyU+KQnRwz91+S/HygfppzUOTlAXV1c84dzvIfy0d975XbnJeYh7q/rAt9IJ1TSuH1Ghd+XH4eR+u7AQAiQEpsJKJtVkRFWGC1CJJjI/Hbz2+c03vMprj9GQCc7ojOFb+lIrILwC4AyM3N9ee9r7DIEQ+3Z9JLT3nnqUGmHmy68v75PX/yF2TKvTO/9tT7Z3j+td58yv3zfq8rnn/tg3bXev0Iq8BmFURYLIiMsMBmFdisFtisFkRaLbBF+G5HWCyIj4pAXJQVCdERiIuK8N2OjIAl2NOtGhpm93UDaOidftuu9nW6NhHB9uIMbC/OQGvvMA5d6ERd5wDa+kYwOubFyJgHXqVgj7aFJI8/xd0EIGfS7WwAzVMfpJTaA2AP4NvjnkuYx+5fM5enEV1bbu70e9xz3MHQg9zE3Gn3uHMTjbvNoZKZGI2PrMnSNIM/o+zvAlgsIgtFJBLA/QD2BTcWUQDt3u0b054sNtb3dYPavX03Ym0f3OZYWyx2bzfuNpvJjMWtlBoD8CUArwI4DeC3SqlTwQ5GFDA7d/oORObl+cZ98vIMfWASAHau2Ik9d+1BXmIeBIK8xDwemDSQGQ9OzgVnlRARzc5sDk5y7S8Rkc6wuImIdIbFTUSkMyxuIiKdYXETEekMi5uISGdY3EREOsPiJiLSmaAswBGRdgDTnBzCL2kAOgIYRw+4zcZntu0FuM2zlaeUcvjzwKAU93yISIW/q4eMgttsfGbbXoDbHEwcKiEi0hkWNxGRzoRjce/ROoAGuM3GZ7btBbjNQRN2Y9xERHRt4bjHTURE16BZcYvIDhGpFZFzIvLNae6PEpGnx+8/LCL5oU8ZOH5s71+JSLWInBCR10QkT4ucgTTTNk963H0iokRE9zMQ/NlmEfnE+Pf6lIj8OtQZA82Pn+1cESkTkWPjP9+3a5EzUETkSRFxiUjVVe4XEfn/4/8eJ0RkbcBDKKVC/gHACuA8gAIAkQCOA1g25TEPAXh8/PP7ATytRdYQbu9WALHjn39Bz9vr7zaPPy4BwJsADgEo1Tp3CL7PiwEcA5A8fjtd69wh2OY9AL4w/vkyAHVa557nNt8EYC2AqqvcfzuAl+G7LvcGAIcDnUGrPe7rAZxTSl1QSo0C+A2Ae6Y85h4Avxj//HcAtstMlyMPXzNur1KqTCk1OH7zEHwXZdYzf77HAPB3AL4LYDiU4YLEn23+CwA/Ukp1A4BSyhXijIHmzzYrAPbxzxMxzcXG9UQp9SaArms85B4A/658DgFIEhFnIDNoVdxo95C2AAACR0lEQVRZABon3W4a/9q0j1G+6172AkgNSbrA82d7J/scfP9j69mM2ywiawDkKKVeDGWwIPLn+7wEwBIReVtEDonIjpClCw5/tvlRAA+ISBOAlwB8OTTRNDPb3/dZiwjki83CdHvOU6e3+PMYvfB7W0TkAQClADYHNVHwXXObRcQC4J8B/LdQBQoBf77PEfANl2yB76+qt0SkRCnVE+RsweLPNn8KwM+VUt8TkY0Afjm+zd7gx9NE0LtLqz3uJgA5k25n48o/n95/jIhEwPcn1rX+PAln/mwvRORmAI8AuFspNRKibMEy0zYnACgBUC4idfCNBe7T+QFKf3+un1dKuZVSFwHUwlfkeuXPNn8OwG8BQCl1EEA0fOf0MCq/ft/nQ6vifhfAYhFZKCKR8B183DflMfsAfGb88/sAvK7GR/51aMbtHR82eAK+0tb7uCcwwzYrpXqVUmlKqXylVD584/p3K6UqtIkbEP78XP8BvgPREJE0+IZOLoQ0ZWD5s80NALYDgIgUw1fc7SFNGVr7AHx6fHbJBgC9SqmWgL6DhkdmbwdwBr4j0o+Mf+3/wvfLC/i+uc8AOAfgCIACrY8mB3l7/xNAG4DK8Y99WmcO9jZPeWw5dD6rxM/vswD4PoBqACcB3K915hBs8zIAb8M346QSwK1aZ57n9j4FoAWAG769688B+DyAz0/6Hv9o/N/jZDB+rrlykohIZ7hykohIZ1jcREQ6w+ImItIZFjcRkc6wuImIdIbFTUSkMyxuIiKdYXETEenMfwErCzSHhAnX+wAAAABJRU5ErkJggg==\n", "text/plain": [ "
" ] }, "metadata": { "needs_background": "light" }, "output_type": "display_data" }, { "name": "stdout", "output_type": "stream", "text": [ "Likelihood-Ratio: -3.3402532908754434\n" ] }, { "data": { "image/png": "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\n", "text/plain": [ "
" ] }, "metadata": { "needs_background": "light" }, "output_type": "display_data" }, { "name": "stdout", "output_type": "stream", "text": [ "Likelihood-Ratio: -3.6767255274966564\n" ] }, { "data": { "image/png": "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\n", "text/plain": [ "
" ] }, "metadata": { "needs_background": "light" }, "output_type": "display_data" }, { "name": "stdout", "output_type": "stream", "text": [ "Likelihood-Ratio: -3.165899903730666\n" ] }, { "data": { "image/png": "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\n", "text/plain": [ "
" ] }, "metadata": { "needs_background": "light" }, "output_type": "display_data" }, { "name": "stdout", "output_type": "stream", "text": [ "Likelihood-Ratio: -3.502372140351879\n" ] }, { "data": { "image/png": "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\n", "text/plain": [ "
" ] }, "metadata": { "needs_background": "light" }, "output_type": "display_data" }, { "name": "stdout", "output_type": "stream", "text": [ "Likelihood-Ratio: -3.8388443769730918\n" ] }, { "data": { "image/png": "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\n", "text/plain": [ "
" ] }, "metadata": { "needs_background": "light" }, "output_type": "display_data" }, { "name": "stdout", "output_type": "stream", "text": [ "Likelihood-Ratio: -4.175316613594305\n" ] }, { "data": { "image/png": "iVBORw0KGgoAAAANSUhEUgAAAW4AAAD8CAYAAABXe05zAAAABHNCSVQICAgIfAhkiAAAAAlwSFlzAAALEgAACxIB0t1+/AAAADl0RVh0U29mdHdhcmUAbWF0cGxvdGxpYiB2ZXJzaW9uIDMuMC4zLCBodHRwOi8vbWF0cGxvdGxpYi5vcmcvnQurowAAIABJREFUeJzt3Xl0XGeZJvDnLZX2Ku1V2jdbsmx5dxTbSchCEshCEgLNknQIcKBxw0Af6OHMDAxMQ89MhqGbATo0fWhDZ4CQhGQggUCShmxOHGPLlvdFtixrs2QtpX1fquqbP6oqKFZZupKr6ta99/mdo6PFV3XfT1V6fPXe735XlFIgIiLjsOldABERLQ+Dm4jIYBjcREQGw+AmIjIYBjcRkcEwuImIDIbBTURkMAxuIiKDYXATERmMPRoPmpeXpyoqKqLx0EREpnT48OF+pZRLy7ZRCe6Kigo0NDRE46GJiExJRNq1bstWCRGRwTC4iYgMhsFNRGQwDG4iIoNhcBMRGQyDm4jIYBjcREQGE5V53ERkXZ6xGbzS2IuhyVnUFmbgpmoXbDbRuyxT0RTcItIGYAyAD4BXKVUXzaKIyHiUUvjx3hZ8549NmPX63/76+qIM/PAvt6EiL13H6sxlOa2SdyultjC0iehySik88kIj/teLZ3HLGhf++Lc34fTf34HvfXQzuoan8KEf7Udb/4TeZZoGe9xEdNUe29eGn7zVik9eX4F/ffgarMl3Ij3Zjg9sLcGvPns9/Eph1+MNmJr16V2qKWgNbgXgjyJyWER2RbMgIjKWM5dG8e2XzuI9tfn4u3tqIfLOfnaV24Hvf3QLmnrH8d2Xz+lUpbloDe4blFLbANwF4PMictPlG4jILhFpEJEGj8cT0SKJKD75/Qr/5dcnkJmWiG//xaYrnoS8aY0LH60rxU//1IZWtkyumqbgVkpdCr7vA/AcgO1httmtlKpTStW5XJpWJiQig3vuaBdOdo3ga3evQ0560qLbfvmONUi2J+DbL52NUXXmtWRwi0i6iDhDHwN4L4BT0S6MiOLb1KwP//iHc9hckon7Nhctub3bmYJP3VCBP5zpQXPfeAwqNC8tR9z5AN4SkeMADgJ4QSn179Eti4ji3dOHOtAzOo3/evc6zfO0P3F9BZISbPjJ3pYoV2duSwa3UqpFKbU5+LZeKfVILAojovjl9fnx472tuLYiGztW5Wr+vlxHMj50TQmePdqF4cnZKFZobpwOSETL9sLJbnQNT+Gvb1q97O99aEc5Zr1+/PbYpShUZg0MbiJatsfeasVqVzpuXete9vfWFmVgQ3EGnmm4GIXKrIHBTUTLcvrSCI53juDhneUrXoPkI3WlOH1pFI3doxGuzhoY3ES0LE8fuogkuw33by1e8WO8b2MhbAK8dLI7gpVZB4ObiDSbnvPhuaNduGtDAbLSFp+3vZhcRzK2V+bgpVM9EazOOhjcRKTZiye7MTbtxQPXll31Y921oRDn+8bR3DcWgcqshcFNRJo9d7QLpTmp2Lkq56of6471BQCAl07yqHu5GNxEpEn/+Az2Nffjvs1FCxaSWomCzBRsLcvCy429EajOWhjcRKTJS6d64FfAvRoub9fqljVunOwaweAEL8ZZDgY3EWnyu+OXUO12oCbfGbHHvLnGBaWAvee5ouhyMLiJaEk9I9M41DaIeyPUJgnZWJyJrLREvNHE4F4OBjcRLemlU91QCrhnU2FEHzfBJnhXVR72nu+HUiqij21mDG4iWtIrjb2odjuwyuWI+GPfvMYFz9gMGrs5LVArBjcRLWpkag71LYO4vTY/Ko9/Q1UeAKC+dSAqj29GDG4iWtQbTR54/Qq3r4tOcBdlpaIkOxUHWwej8vhmxOAmokW9cqYXeY4kbCnNito+tlfm4GDrIPvcGjG4ieiK5nx+vH6uD++ucSNhhSsBarGjMgcDE7O44OEtzbRgcBPRFR1qHcTYtDdq/e2Q7ZWBu+jUs12iCYObiK7o5cZeJNltuLE6L6r7qchNg9uZzD63RgxuIrqi18724YbVuUhLskd1PyKC7ZU5qG9hn1sLBjcRhdU+MIH2gUncUrP825OtxPbKHPSMTqNreCom+zMyBjcRhfXm+X4AwE1rXDHZ39bSbADAsYvDMdmfkTG4iSisN5s8KMlORUVuWkz2t7bQiWS7Dcc6GNxLYXAT0QJzPj/2XxjAjdWuiC4qtZjEBBs2FGfyiFsDBjcRLXDs4jDGZ7y4eU10Z5NcbktpFk52jWDO54/pfo2GwU1EC+xt8iDBJrhudWyDe3NpFma8fpzr4YJTi2FwE9ECb5zvx5bSLGSmJsZ0v1uDl9UfZbtkUQxuInqH4clZnOgcjvpFN+GUZKciNz2JJyiXwOAmonfY1zwApYAbq2MzDXA+EcGW0iwcuzgU830bCYObiN7hrWYPnCl2bC7J1GX/m0qy0NI/gYkZry77NwLNwS0iCSJyVER+H82CiEhfB1oGsaMyB/YEfY7r1hdlQCmgsXtUl/0bwXKemS8CaIxWIUSkv77RabT2T2BHcLU+PWwoDhzpn77E4L4STcEtIiUA3gfgJ9Eth4j0dCC4Ot+OVTm61ZCfkYzc9CSc6hrRrYZ4p/WI+/sA/jOAK86KF5FdItIgIg0ejycixRFRbNW3DMCRbEdtYYZuNYgIaosyeMS9iCWDW0TuAdCnlDq82HZKqd1KqTqlVJ3LFfuz0UR09epbB1FXka1bfztkQ3EmmnrHMOP16VpHvNLy7NwA4D4RaQPwSwC3isgvoloVEcVc//gMmvvGde1vh6wvyoDXr3C+l7cyC2fJ4FZKfVUpVaKUqgDwAIDXlFIfi3plRBRTB+Ogvx2yoSh0gpJ97nA4j5uIAAT622lJCdhYrM/87fnKctLgSLbjVBf73OEs635ESqk9APZEpRIi0lV96yCuKc9Gos79bQCw2UInKHnEHY7+zxAR6W5oYhZne8awo1L/NknI+qIMnOkehc/Pe1BejsFNRDjYFupv639iMmRdYQam5/xoH5jQu5S4w+AmItS3DCLZbsMmndYnCWdtgRMAuDZ3GAxuIkJ96wC2lWUj2Z6gdylvq3Y7YROgkcG9AIObyOJGpuZwpns0LqYBzpealICK3HSc6+HMkssxuIksrqFtEEohLi68uVxNgZOtkjAY3EQWV986iKQEG7aWZeldygI1BU60D05icpZrc8/H4CayuPqWAWwpzUJKYvz0t0PWFjihFHjp+2UY3EQWNj7jxalL8dffDqkpCKxSyHbJOzG4iSysoW0QPr+Ky/42ELj0PSXRhrMM7ndgcBNZWH3rIOw2wbby+OtvA0CCTVCT78S5Xs4smY/BTWRh9S0D2FSSibSkZS1bFFOcWbIQg5vIoiZnvTjRORJXl7mHU1OQgf7xWfSPz+hdStxgcBNZ1JH2YXj9Kq4WlgqHl74vxOAmsqj61gEk2AR1FfEd3DXB4OYJyj9jcBNZVH3LIDYUZcCRHL/9bQDIcwTu+t7cx+AOYXATWdD0nA/HLg7HfX87ZLXbwYtw5mFwE1nQ0Y5hzPr8cd/fDql2O3C+bxxK8aYKAIObyJIOtAxABHHf3w6pdjswMjUHD2eWAGBwE1lSfesAagszkJmaqHcpmlTnB05QNrNdAoDBTWQ5M14fjnYMx+1l7uFUux0AgPN9DG6AwU1kOccvjmDG64/bhaXCcTmTkZFix3nOLAHA4CaynPqWAQDAdoP0twFARFCd7+TMkiAGN5HF1LcOYm2BE9npSXqXsixVLgcueBjcAIObyFLmfH4cbh8yzDTA+arzHegfn8XgxKzepeiOwU1kISc6RzA15zPMhTfzVQVPUDbzBCWDm8hK6luD/W1DHnEHpgTyBCWDm8hS6lsGUeV2IM+RrHcpy1aUmYL0pASeoASDm8gyvD4/GtoGDdnfBgIzS6rcDrZKoCG4RSRFRA6KyHEROS0ifx+Lwogosk5fGsXErDH72yFVbidbJdB2xD0D4Fal1GYAWwDcKSI7o1sWEUVaqL+906BH3EDgBGXv6AxGp+f0LkVXSwa3Cgj9bZIYfOMSXUQGU98yiMq8dLgzUvQuZcWqObMEgMYet4gkiMgxAH0AXlZK1YfZZpeINIhIg8fjiXSdRHQVfH6Fgwbub4dU5weD2+InKDUFt1LKp5TaAqAEwHYR2RBmm91KqTqlVJ3L5Yp0nUR0FRq7RzE27TXU+iThlGSnIdlus3yfe1mzSpRSwwD2ALgzKtUQUVTUtw4CgKFWBAwnwSZY7XJYfpVALbNKXCKSFfw4FcDtAM5GuzAiipz6lgGU5qSiKCtV71KuWhVvY6bpiLsQwOsicgLAIQR63L+PbllEFCl+v0J96yCuM/A0wPmq3Q50DU9hctardym6WfL2zkqpEwC2xqAWIoqCsz1jGJmaM3ybJCS0ZsmFvglsLMnUuRp98MpJIpM7EFx/2+gnJkPeXmzKY90TlAxuIpOrbw30t0uy0/QuJSLKc9ORYBNLz+VmcBOZWKi/vdMkbRIASLLbUJGbxuAmInM61zuG4ck57DTJicmQKre1pwQyuIlMzGz97ZAqtwPtA5OY9fr1LkUXDG4iEzvQYq7+dkiV2wGfX6F9YELvUnTB4CYyKTP2t0Oq3YG74Vi1z83gJjKpUH/byOtvX8kqVzoAWLbPzeAmMqn6UH/b4CsChpOWZEdxViqPuInIXA60DKIkOxWlOebqb4dU51v3NmYMbiITCvS3B0w3DXC+KpcDFzzj8Pmtd18XBjeRCTX1jWHIhPO356tyOzDj9aNraErvUmKOwU1kQgcumLe/HWLlNUsY3EQmZPb+NjAvuC3Y52ZwE5mM369wwOT9bQDISktCniPZkjdVYHATmcyZ7lEMT87hXVV5epcSdVXudDR7GNxEZHBvNfcDAK6vMvcRNxBolzT3jUMpa80sYXATmcy+5n6syXfA7UzRu5Soq3Y7MTbtRd/YjN6lxBSDm8hEpud8ONg6iBss0CYBrHuCksFNZCJHOoYw4/Vbor8NMLiJyAT2NfcjwSamXFgqHLczGc4UO4ObiIzrreYBbC3NgiPZrncpMSEiwbvhWOsiHAY3kUmMTM3hZOcwrrdImySkyuVAc5+1bqjA4CYyiQMtA/ArWKa/HVLldqB/fAbDk7N6lxIzDG4ik9jX3I+0pARsKc3Su5SYqs633glKBjeRSbzV3I8dlTlIslvr17rKZb3bmFnrGSYyqUvDU2jxTFhm/vZ8xdmpSLbbGNxEZCxvNnkAADdWu3SuJPYSbIJVLoel1ixhcBOZwJ5zHhRmpmBNsN9rNdVuh6VWCVwyuEWkVEReF5FGETktIl+MRWFEpM2cz499zf24pcYFEdG7HF1UuR3oGp7C5KxX71JiQssRtxfAl5VS6wDsBPB5EamNbllEpNWR9iGMzXhx8xrrtUlCQpe+t3isMZ97yeBWSnUrpY4EPx4D0AigONqFEZE2e5o8sNvEkicmQ6ottmbJsnrcIlIBYCuA+mgUQ0TL98Y5D64pz4YzJVHvUnRTnpuOBJtY5tJ3zcEtIg4AvwbwJaXUaJh/3yUiDSLS4PF4IlkjEV1B3+g0znSP4uYa67ZJACDJbkN5bhqPuOcTkUQEQvsJpdSz4bZRSu1WStUppepcLmu/iIhiZU9wGuAta9w6V6K/wJolDG4AgAROU/8bgEal1HejXxIRafVGkwduZzLWFTr1LkV31fkOtA1MYtbr17uUqNNyxH0DgIcB3Coix4Jvd0e5LiJagtfnx94mD25eY91pgPNVuR3w+RXaB8w/s2TJRXuVUm8B4KuCKM4cvTiM0WkvbqlhmwR455ol1fnm/guEV04SGdQrZ3qRmCC4aY11pwHOt9qdDsAaUwIZ3EQG9fKZXuxclWvpaYDzpSXZUZyVivMMbiKKRxc842jpn8B7avP1LiWuVLmtMbOEwU1kQC+f6QUA3L6OwT1flduBC55x+PxK71KiisFNZECvnOnF+qIMFGWl6l1KXKl2OzDj9aNraErvUqKKwU1kMP3jMzjcMcSj7TBCi001e8x96TuDm8hgXjvbB6XA/nYYVRZZbIrBTWQwL5/pRVFmCtYXZehdStzJSktCniOJwU1E8WNy1ou95z24vTafV0teQZXbgXMmvxsOg5vIQF4/68H0nB93bSjUu5S4tbYgA009Y/CbeGYJg5vIQF44eQl5jmRsr8zRu5S4ta7Qiak5HzoGJ/UuJWoY3EQGMTnrxWtn+3DXhgIk2NgmuZK1BYHe/9meBbcNMA0GN5FBhNokd29km2Qxa/KdEAEau807JZDBTWQQbJNok5qUgMrcdB5xE5G+2CZZnrWFTpzt4RE3EenotbN9mJ7z432b2CbRYm1BBtoHJjEx49W7lKhgcBMZwAsnupHnSMa1FWyTaLG2IHAjhXO95jzqZnATxbmRyTm82tiHezcXsk2i0brC4MwSk56gZHATxbkXTnZj1ufHB7eW6F2KYRRnpcKRbDftCUoGN1Gce+5oJ6rdDmwo5tokWtlsgpoCJ4+4iSj2OgYmcahtCB/YVsy1SZZpXaETjT2jUMp8l74zuIni2HNHuyAC3L+lWO9SDGdtQQbGpr3oGjbfTRUY3ERxSimFZ4924rpVubzTzQqETlCevmS+PjeDmyhONbQPoX1gEh/YyqPtlagtzIBNgFNdI3qXEnEMbqI49WR9B5zJdl50s0KpSQmodjtxksFNRLEwPDmLF0524wPbipGWZNe7HMPaUJyJU10jpjtByeAmikO/PtKFWa8fD1xbpncphraxOAP947PoHZ3Ru5SIYnATxRmlFJ462IEtpVmo5X0lr8rGkkwAMF27hMFNFGcOtQ2huW8cf7mDR9tXq7YwEzaxYHCLyGMi0icip2JREJHVPX6gHc5kO+7hScmrlpqUgCq3w3QzS7Qccf8UwJ1RroOIAFwansKLJ7vx0WtLeVIyQjYUZ1rviFsp9SaAwRjUQmR5P9vfBqUUPnlDhd6lmMbG4kx4xmbQOzqtdykRwx43UZyYmPHiqfoO3LWhECXZaXqXYxqbgicoT3Sa56g7YsEtIrtEpEFEGjweT6QelsgyfnW4E6PTXnzqXZV6l2Iq64syYbcJjnQM6V1KxEQsuJVSu5VSdUqpOpfLFamHJbIEn1/h/+5rxZbSLFxTnq13OaaSkpiA9UUZONLO4CaiCHrhZDfaBiax66ZVepdiStvKs3GicwRzPr/epUSElumATwHYD6BGRDpF5NPRL4vIOvx+hR+8eh7VbgfuXF+gdzmmtK0sG1NzPtPcWGHJ+UZKqQdjUQiRVf376R6c7xvHow9uhY33lIyKbcH205GOobevpjQytkqIdOT3Kzz66nmscqXjfRt5wU20FGWmoCAjxTQnKBncRDr645lenO0Zw9/cWsU7uEeRiGBbeRYOm+QEJYObSCdenx//+IezWOVKx72bivQux/S2lWWjc2gKfSa4EIfBTaSTpxsu4oJnAl+5cy3sCfxVjLbtlTkAgAOtxr8QnK8WIh2Mz3jxvZfPY3tFDt5Tm693OZawvigTzhQ79l/o17uUq8bgJtLB7jcuoH98Bl+9ey1E2NuOhQSbYEdlLv50YUDvUq4ag5soxlr7J/CjN1tw3+YibC3jVZKxdP3qXLQPTKJreErvUq4Kg5sohpRS+Mbzp5GUYMPX37dO73Is57rVuQCA/QY/6mZwE8XQS6d68GaTB19+7xq4M1L0LsdyavKdyElPwp8M3udmcBPFyNDELL7x/GnUFmbg4Z3lepdjSTabYOeqHOy/MGDoO78zuIli5L/99hSGJ2fxnQ9v5vQ/Hd1Y7UL3yDSaesf1LmXF+OohioHfHb+E35/oxhdvq+ad23X27ho3AODVs706V7JyDG6iKLs4OImv/+YUNpdk4rM3r9a7HMsryEzBhuIMvNbYp3cpK8bgJoqiGa8Pn3/yCPxK4dEHt7JFEiduXZuPIx1DGJyY1buUFeGriCiKHnmhESc6R/CdD29GeW663uVQ0O3r3PArYM85Yx51M7iJouTxA+34+f52fObGStzBGyTElQ1FmSjISMGLJ3v0LmVFGNxEUfD6uT5847encNtaN75yFy+0iTc2m+CeTYV4o6kPI5NzepezbAxuogg72jGELzxxBOsKM/Dog1u5znacum9LEeZ8Ci+d6ta7lGVjcBNF0InOYXz8sYPIdSTjsU9ei/TkJe8OSDrZWJyJyrx0PH/8kt6lLBuDmyhCjl0cxsP/dhCZqYl4atdO5POS9rgmIrhvcxH2twzg4uCk3uUsC4ObKAJeO9uLB3cfQEaqHU99ZieKs1L1Lok0+Mi1pRAAT9R36F3KsjC4ia6CUgq/ONCOz/z8MKrcDjz7uRtQmpOmd1mkUXFWKt5Tm4+nD3Vges6ndzmaMbiJVmhy1osvP3McX//NKdxUnYdf7toJlzNZ77JomT5+XQWGJufw+xPGOUnJ4CZagROdw3j/P+/Dc8e68Le3r8FPPsETkUZ1/epc1OQ78S97muHzG2PFQAY30TJMzfrwrRcbcf8P92F0eg4//9R2fPH2ak75MzARwZdur0aLZwLPH+/SuxxNeIhApIHPr/DskU589+UmdI9M48HtpfjKXeuQmZqod2kUAXesL0BtYQa+84cm3LG+AGlJ8R2NPOImWsSs149nj3Ti7n/ai//0qxNwO5PxzF9fh299cBND20RsNsE371uPruEpPPpqs97lLCm+/1sh0sml4Sk8d7QLj+9vR8/oNKrdDvzgwa24Z1Mh78puUtsrc/Dha0rw470tuHWtG9src/Qu6YoY3ERBPSPT2HOuD787cQl/ujAApYDrVuXiWx/ciFtqXAxsC/i7e2vR0D6ELzx5BM99/oa4nY8vWu67JiJ3AvgnAAkAfqKU+t+LbV9XV6caGhoiUyFRlPSNTeNYxzAOdwzhzaZ+NHaPAgDKctLwwW3F+MDWYi7FakFne0bx4R/tR3ZaEp74qx0xm5cvIoeVUnWatl0quEUkAUATgPcA6ARwCMCDSqkzV/oeBjfFC6UUhifn0Prks7jw5G/QolLQUlyNU6s2oWsucIrHbhPUVWTjlho3bqlxoSbfaYqj6ydOPoGvvfo1dIx0oCyzDI/c9gge2viQ3mUZQmD5gnrYRPA/798QkxZZpIP7OgDfVErdEfz8qwCglPrWlb6HwU1XSymFWZ8fs14/5nwq+N7/9tem5nwYm/ZibHruHe9Hp+bQNzaDvrEZ9I5Oo29sBrNe/9uPm+ibQ/lQN9YOXsSWm7dh6/23Yn1RJlISE3QcbeQ9cfIJ7PrdLkzO/XkNjrTENOy+dzfDW6O2/gl84akjONU1inWFGfiLbcXYuSoXVW5HVF4vkQ7uDwG4Uyn1V8HPHwawQyn1hSt9z0qD+54f7MX03J9/yS6vLWylatFPNT/O5T8GFWarBdtomKsf7ud7+VdWtO+w+1pqT1ofZwU/rwg9V36lMOcLhPVyiQDOZDtczmTkZ6QgPyMFbmcy3N//B1S0nMaqwS6UDvfAroKvsfJyoK1t2fsxgorvV6B9pH3B18szy9H2pbbYF2RQPr/Cr4904hcH2nGic+TtrzuT7chITUSCTWCTwKwUmwhy0pLwzGevW9G+lhPcWk5Ohvv7YMFvlYjsArALAMrKyrTse4Eql2PhL6ws+mlo3xq2iczjLKxn4Vba9rX444T7q2zh16Kzb+2Ps/ifjmHHsMQ4BUCS3YbEBBuS7DYkBd8nvv1ekGy3ITkxARkpdjhTEuEMvk9LTIAt3IUw9/ws/P+yHcZaWGg5OkbCj+1KX6fwEmyCj9SV4iN1pbg4OInjncNo9UxgcHIWo1Ne+JWCz6/gV4G3jJTYTBHVEtydAErnfV4CYMECtkqp3QB2A4Ej7pUU8/0Htq7k24gWV1YGtC88+sQKDzCMoCyzLOwRd1mmecccbaU5aXGzgJiWC3AOAagWkUoRSQLwAIDno1sWUQQ98giQdtkvXFpa4Osm9chtjyAt8Z1jTktMwyO3mXfMVrJkcCulvAC+AOAPABoBPKOUOh3twogi5qGHgN27Az1tkcD73bsDXzephzY+hN337kZ5ZjkEgvLMcp6YNBFN87iXi7NKiIiWZzknJ7lWCRGRwTC4iYgMhsFNRGQwDG4iIoNhcBMRGQyDm4jIYBjcREQGw+AmIjKYqFyAIyIeAGEWh9AkD0B/BMsxAo7Z/Kw2XoBjXq5ypZRLy4ZRCe6rISINWq8eMguO2fysNl6AY44mtkqIiAyGwU1EZDDxGNy79S5ABxyz+VltvADHHDVx1+MmIqLFxeMRNxERLUK34BaRO0XknIg0i8hXwvx7sog8Hfz3ehGpiH2VkaNhvP9RRM6IyAkReVVEyvWoM5KWGvO87T4kIkpEDD8DQcuYReQjwef6tIg8GesaI03Da7tMRF4XkaPB1/fdetQZKSLymIj0icipK/y7iMijwZ/HCRHZFvEilFIxfwOQAOACgFUAkgAcB1B72Tb/AcCPgh8/AOBpPWqN4XjfDSAt+PHnjDxerWMObucE8CaAAwDq9K47Bs9zNYCjALKDn7v1rjsGY94N4HPBj2sBtOld91WO+SYA2wCcusK/3w3gJQTufb0TQH2ka9DriHs7gGalVItSahbALwG8/7Jt3g/gZ8GPfwXgNlnqtuLxa8nxKqVeV0pNBj89gMBNmY1My3MMAP8DwD8AmI5lcVGiZcyfAfBDpdQQACil+mJcY6RpGbMCkBH8OBNhbjZuJEqpNwEMLrLJ+wH8XAUcAJAlIoWRrEGv4C4GcHHe553Br4XdRgXuezkCIDcm1UWelvHO92kE/sc2siXHLCJbAZQqpX4fy8KiSMvzvAbAGhHZJyIHROTOmFUXHVrG/E0AHxORTgAvAvib2JSmm+X+vi+bPZIPtgzhjpwvn96iZRuj0DwWEfkYgDoAN0e1ouhbdMwiYgPwPQCfjFVBMaDlebYj0C65BYG/qvaKyAal1HCUa4sWLWN+EMBPlVL/R0SuA/B4cMz+6Jeni6hnl15H3J0ASud9XoKFfz69vY2I2BH4E2uxP0/imZbxQkRuB/A1APcppWZiVFu0LDVmJ4ANAPaISBsCvcDnDX6CUuvr+rdKqTmlVCuAcwjkLP0kAAABRElEQVQEuVFpGfOnATwDAEqp/QBSEFjTw6w0/b5fDb2C+xCAahGpFJEkBE4+Pn/ZNs8D+ETw4w8BeE0FO/8GtOR4g22Df0UgtI3e9wSWGLNSakQplaeUqlBKVSDQ179PKdWgT7kRoeV1/RsETkRDRPIQaJ20xLTKyNIy5g4AtwGAiKxDILg9Ma0ytp4H8PHg7JKdAEaUUt0R3YOOZ2bvBtCEwBnprwW/9t8R+OUFAk/u/wPQDOAggFV6n02O8nhfAdAL4Fjw7Xm9a472mC/bdg8MPqtE4/MsAL4L4AyAkwAe0LvmGIy5FsA+BGacHAPwXr1rvsrxPgWgG8AcAkfXnwbwWQCfnfcc/zD48zgZjdc1r5wkIjIYXjlJRGQwDG4iIoNhcBMRGQyDm4jIYBjcREQGw+AmIjIYBjcRkcEwuImIDOb/A3ACJviw1wT+AAAAAElFTkSuQmCC\n", "text/plain": [ "
" ] }, "metadata": { "needs_background": "light" }, "output_type": "display_data" }, { "name": "stdout", "output_type": "stream", "text": [ "Likelihood-Ratio: -4.511788850215518\n" ] }, { "data": { "image/png": "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\n", "text/plain": [ "
" ] }, "metadata": { "needs_background": "light" }, "output_type": "display_data" }, { "name": "stdout", "output_type": "stream", "text": [ "Likelihood-Ratio: -4.000963226449527\n" ] }, { "data": { "image/png": "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\n", "text/plain": [ "
" ] }, "metadata": { "needs_background": "light" }, "output_type": "display_data" }, { "name": "stdout", "output_type": "stream", "text": [ "Likelihood-Ratio: -4.33743546307074\n" ] }, { "data": { "image/png": "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\n", "text/plain": [ "
" ] }, "metadata": { "needs_background": "light" }, "output_type": "display_data" }, { "name": "stdout", "output_type": "stream", "text": [ "Likelihood-Ratio: -4.673907699691953\n" ] }, { "data": { "image/png": "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\n", "text/plain": [ "
" ] }, "metadata": { "needs_background": "light" }, "output_type": "display_data" }, { "name": "stdout", "output_type": "stream", "text": [ "Likelihood-Ratio: -5.010379936313166\n" ] }, { "data": { "image/png": "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\n", "text/plain": [ "
" ] }, "metadata": { "needs_background": "light" }, "output_type": "display_data" }, { "name": "stdout", "output_type": "stream", "text": [ "Likelihood-Ratio: -5.346852172934379\n" ] }, { "data": { "image/png": "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\n", "text/plain": [ "
" ] }, "metadata": { "needs_background": "light" }, "output_type": "display_data" }, { "name": "stdout", "output_type": "stream", "text": [ "Likelihood-Ratio: -4.836026549168388\n" ] }, { "data": { "image/png": "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\n", "text/plain": [ "
" ] }, "metadata": { "needs_background": "light" }, "output_type": "display_data" }, { "name": "stdout", "output_type": "stream", "text": [ "Likelihood-Ratio: -5.1724987857896005\n" ] }, { "data": { "image/png": "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\n", "text/plain": [ "
" ] }, "metadata": { "needs_background": "light" }, "output_type": "display_data" }, { "name": "stdout", "output_type": "stream", "text": [ "Likelihood-Ratio: -5.5089710224108135\n" ] }, { "data": { "image/png": "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\n", "text/plain": [ "
" ] }, "metadata": { "needs_background": "light" }, "output_type": "display_data" }, { "name": "stdout", "output_type": "stream", "text": [ "Likelihood-Ratio: -5.845443259032026\n" ] }, { "data": { "image/png": "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\n", "text/plain": [ "
" ] }, "metadata": { "needs_background": "light" }, "output_type": "display_data" }, { "name": "stdout", "output_type": "stream", "text": [ "Likelihood-Ratio: -6.181915495653239\n" ] }, { "data": { "image/png": "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\n", "text/plain": [ "
" ] }, "metadata": { "needs_background": "light" }, "output_type": "display_data" }, { "name": "stdout", "output_type": "stream", "text": [ "Likelihood-Ratio: -6.518387732274452\n" ] }, { "data": { "image/png": "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\n", "text/plain": [ "
" ] }, "metadata": { "needs_background": "light" }, "output_type": "display_data" }, { "name": "stdout", "output_type": "stream", "text": [ "Likelihood-Ratio: -6.007562108508462\n" ] }, { "data": { "image/png": "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\n", "text/plain": [ "
" ] }, "metadata": { "needs_background": "light" }, "output_type": "display_data" }, { "name": "stdout", "output_type": "stream", "text": [ "Likelihood-Ratio: -6.344034345129675\n" ] }, { "data": { "image/png": "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\n", "text/plain": [ "
" ] }, "metadata": { "needs_background": "light" }, "output_type": "display_data" }, { "name": "stdout", "output_type": "stream", "text": [ "Likelihood-Ratio: -6.680506581750888\n" ] }, { "data": { "image/png": "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\n", "text/plain": [ "
" ] }, "metadata": { "needs_background": "light" }, "output_type": "display_data" }, { "name": "stdout", "output_type": "stream", "text": [ "Likelihood-Ratio: -7.016978818372101\n" ] }, { "data": { "image/png": "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\n", "text/plain": [ "
" ] }, "metadata": { "needs_background": "light" }, "output_type": "display_data" }, { "name": "stdout", "output_type": "stream", "text": [ "Likelihood-Ratio: -7.353451054993313\n" ] }, { "data": { "image/png": "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\n", "text/plain": [ "
" ] }, "metadata": { "needs_background": "light" }, "output_type": "display_data" }, { "name": "stdout", "output_type": "stream", "text": [ "Likelihood-Ratio: -6.842625431227322\n" ] }, { "data": { "image/png": "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\n", "text/plain": [ "
" ] }, "metadata": { "needs_background": "light" }, "output_type": "display_data" }, { "name": "stdout", "output_type": "stream", "text": [ "Likelihood-Ratio: -7.179097667848535\n" ] }, { "data": { "image/png": "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\n", "text/plain": [ "
" ] }, "metadata": { "needs_background": "light" }, "output_type": "display_data" }, { "name": "stdout", "output_type": "stream", "text": [ "Likelihood-Ratio: -7.515569904469748\n" ] }, { "data": { "image/png": "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\n", "text/plain": [ "
" ] }, "metadata": { "needs_background": "light" }, "output_type": "display_data" }, { "name": "stdout", "output_type": "stream", "text": [ "Likelihood-Ratio: -7.852042141090961\n" ] }, { "data": { "image/png": "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\n", "text/plain": [ "
" ] }, "metadata": { "needs_background": "light" }, "output_type": "display_data" }, { "name": "stdout", "output_type": "stream", "text": [ "Likelihood-Ratio: -8.188514377712174\n" ] }, { "data": { "image/png": "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\n", "text/plain": [ "
" ] }, "metadata": { "needs_background": "light" }, "output_type": "display_data" }, { "name": "stdout", "output_type": "stream", "text": [ "Likelihood-Ratio: -8.524986614333386\n" ] }, { "data": { "image/png": "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\n", "text/plain": [ "
" ] }, "metadata": { "needs_background": "light" }, "output_type": "display_data" }, { "name": "stdout", "output_type": "stream", "text": [ "Likelihood-Ratio: -8.8614588509546\n" ] }, { "data": { "image/png": "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\n", "text/plain": [ "
" ] }, "metadata": { "needs_background": "light" }, "output_type": "display_data" }, { "name": "stdout", "output_type": "stream", "text": [ "Likelihood-Ratio: -9.197931087575812\n" ] }, { "data": { "image/png": "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\n", "text/plain": [ "
" ] }, "metadata": { "needs_background": "light" }, "output_type": "display_data" }, { "name": "stdout", "output_type": "stream", "text": [ "Likelihood-Ratio: -9.534403324197026\n" ] }, { "data": { "image/png": "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\n", "text/plain": [ "
" ] }, "metadata": { "needs_background": "light" }, "output_type": "display_data" }, { "name": "stdout", "output_type": "stream", "text": [ "Likelihood-Ratio: -9.870875560818238\n" ] }, { "data": { "image/png": "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\n", "text/plain": [ "
" ] }, "metadata": { "needs_background": "light" }, "output_type": "display_data" }, { "name": "stdout", "output_type": "stream", "text": [ "Likelihood-Ratio: -10.207347797439452\n" ] }, { "data": { "image/png": "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\n", "text/plain": [ "
" ] }, "metadata": { "needs_background": "light" }, "output_type": "display_data" }, { "name": "stdout", "output_type": "stream", "text": [ "Likelihood-Ratio: -10.543820034060664\n" ] }, { "data": { "image/png": "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\n", "text/plain": [ "
" ] }, "metadata": { "needs_background": "light" }, "output_type": "display_data" }, { "name": "stdout", "output_type": "stream", "text": [ "Likelihood-Ratio: -10.880292270681878\n" ] }, { "data": { "image/png": "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\n", "text/plain": [ "
" ] }, "metadata": { "needs_background": "light" }, "output_type": "display_data" }, { "name": "stdout", "output_type": "stream", "text": [ "Likelihood-Ratio: -11.21676450730309\n" ] }, { "data": { "image/png": "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\n", "text/plain": [ "
" ] }, "metadata": { "needs_background": "light" }, "output_type": "display_data" }, { "name": "stdout", "output_type": "stream", "text": [ "Likelihood-Ratio: -11.553236743924304\n" ] }, { "data": { "image/png": "iVBORw0KGgoAAAANSUhEUgAAAW4AAAD8CAYAAABXe05zAAAABHNCSVQICAgIfAhkiAAAAAlwSFlzAAALEgAACxIB0t1+/AAAADl0RVh0U29mdHdhcmUAbWF0cGxvdGxpYiB2ZXJzaW9uIDMuMC4zLCBodHRwOi8vbWF0cGxvdGxpYi5vcmcvnQurowAAHdxJREFUeJzt3Xl0nFed5vHvr0pLaZe1W7Il2bEd24kTYtx0EiAQQrMNIT0Mhw4nQDfQ7abZG+ZwYHrmdPdw0nPOnB5IM0N3j6eXYfGwdDpAWIeBkA4NSYiXJHZsx4tkeZEslSRbq7XWnT+qyrFlLSWp3nrrrXo+5+hEVfVW1e+q5CdX9733veacQ0REgiPkdwEiIrI8Cm4RkYBRcIuIBIyCW0QkYBTcIiIBo+AWEQkYBbeISMAouEVEAkbBLSISMAVevGhdXZ1rb2/34qVFRHLS/v37+51z9akc60lwt7e3s2/fPi9eWkQkJ5lZV6rHaqhERCRgFNwiIgGj4BYRCRgFt4hIwCi4RUQCRsEtIhIwCm4RkYDxZB63iMhyzMzGePS5bkYnZ/jt21qojBT6XVJWU3CLiK+mZ2P8wVf28fiLUQC+8mQXD3/wDqpLi3yuLHtpqEREfPWVJ7t4/MUof/62m/jK+1/B6f4x/uKHR/0uK6spuEXEN0OXp/niz07w6s11vPeONu7aUs/v3dnOw/vP0TUw5nd5WUvBLSK+eeTAOYYuT/PpN27FzADYfddGwiHjq0+mfOmOvJNScJvZH5vZC2Z22My+bmYRrwsTkdzmnOPrvz7Dreuq2LGu6sr9DZURXrOlnu8/30Ms5nysMHstGdxm1gJ8DNjlnLsZCAP3e12YiOS2F7qHOd47yu/8Rut1j917azMXhid45vSgD5Vlv1SHSgqAEjMrAEqBbu9KEpF88JMjvYQM3nhT43WPvX5bI0XhED871udDZdlvyeB2zp0H/hI4A/QAQ865n3hdmIjktp+8cIFdbTXUlhdf91hZcQE726r51xP9PlSW/VIZKlkD3AdsAJqBMjN79zzH7TazfWa2LxqNpr9SEckZF4YmOHZhhHu2NSx4zKs21XGkZ5j+0ckMVhYMqQyVvB7odM5FnXPTwCPAnXMPcs7tcc7tcs7tqq9PafcdEclTT3bEe9Kv3FS34DHJx57qGMhITUGSSnCfAW43s1KLz9e5B9DseBFZsSdPDVBVUsj2tZULHnNzSxXFBSEOnrmUwcqCIZUx7qeBh4EDwKHEc/Z4XJeI5LBfnRrg9o01hEK24DGF4RC3rKvi4JmLGawsGFKaVeKc+1Pn3Fbn3M3Oufc45zToJCIrcnZwnHMXL3PnDQsPkyTd1rqGw93DTM3EMlBZcGjlpIhk1IFED3pX+5olj71tfTVTMzGO9gx7XVagKLhFJKOePXuJSGGIGxsrljz25pb4isojCu5rKLhFJKOeO3uJHS1VFISXjp+W6hLKiws4puC+hoJbRDJmejbG4e5hXra+OqXjQyHjxqYKjl4Y8biyYFFwi0jGvHhhhKmZGLemGNwA29ZWcLRnGOd0wakkBbeIZMyzZ+Nzsm9dl3pwb22qZGRihu6hCa/KChwFt4hkzHNnL1FbVsS6NSUpP2fb2vhJTI1zv0TBLSIZc7h7mB3rqq5smpCKLYnZJ8c0zn2FgltEMmJ6NsbJvhG2LbLMfT4VkUKaKiOcio56VFnwKLhFJCM6omNMzzq2Ni09f3uujfVlnIpqD8okBbeIZMSxC/Ex6uX2uAFuqC+nIzqqmSUJCm4RyYgjPcMUhUNsqCtb9nM31pcxMjFD/+iUB5UFj4JbRDLiWM8ImxrKKUxhxeRcG+vLAejQODeg4BaRDDl2YZita5c/vg2wMdFL1zh3nIJbRDw3ODZF7/Ak25qWP74N8WuWFBeE1ONOUHCLiOeSJyZX2uMOhYwNdWV09qvHDQpuEcmAYz3xxTNbV9jjBmirLeXM4Hi6Sgo0BbeIeO5E3wg1ZUXUVxSv+DVaa+LBrSmBCm4RyYCO6NiVE4wrtb6mlMmZGNER7Zyo4BYRz3X0j61o/vbV1teUAmi4BAW3iHhsZGKa6MgkG+pXF9ytCu4rFNwi4qnT/fGgXe1QSUt1CWYKblBwi4jHOvrjc6831JWv6nUihWGaKiOcHbycjrICTcEtIp7q7B/DLD6db7XW15RyVj1uBbeIeKuzf4yW6hIiheFVv1ZySmC+U3CLiKc60zCjJKm1ppQLwxNMTM+m5fWCSsEtIp5xztGZhjncScmZJecu5nevW8EtIp7pH51iZHImbT3u5CbD5y/l947vCm4R8UzyolAb6lc3oySpuToe3N2X8ntmiYJbRDzTmZgKmK6hkoaKYsIhU3D7XYCI5K6O/jGKwqErPeXVKgiHaKqMcF7BLSLijc7oGG21pYRDlrbXbK6OqMftdwEikrvSORUwaW1VCd06OSkikn6zMUfXwPiqLy41V3N1CT1Dl4nF8ve63ApuEfFE96XLTM3G0nZiMqmlOsL0rKN/NH+vy63gFhFPdCSnAq7y4lJzJU905vMJSgW3iHgiuSN7use4X5rLnb/j3ApuEfFEZ/8YFcUF1JUXpfV1tQgnxeA2s2oze9jMjpnZUTO7w+vCRCTYOvvH2FBfhln6pgICVEYKKC8u0FBJCv4K+LFzbitwK3DUu5JEJBd0RNM/FRDAzPJ+LveSwW1mlcBdwN8DOOemnHOXvC5MRIJrYnqW7qHLngQ3xIdLuocU3IvZCESBfzSzg2b2d2bmzachIjmha2Ac59J/YjKpuTq/F+GkEtwFwE7gb5xztwFjwGfmHmRmu81sn5nti0ajaS5TRIIkeXGpG9J0VcC5WqpLGByb4vJUfm6okEpwnwPOOeeeTtx+mHiQX8M5t8c5t8s5t6u+vj6dNYpIwCTncLd71ONurIwAcGE4P3vdSwa3c+4CcNbMbkzcdQ9wxNOqRCTQOqNjNFQUU15c4MnrNyWDeyg/gzvVn+pHgb1mVgR0AO/zriQRCTovLi51taaqYgB687THnVJwO+eeBXZ5XIuI5IjO/jHecFOjZ6/fVBVfhKOhEhGRNBgan2ZgbMrTHnd5cXwRTr4OlSi4RSStOge8ubjUXI2VxXk7VKLgFpG0Sk4F9LLHDdBUFdFQiYhIOnRGxwgZtNaUevo+jZURDZWIiKRDR/8Y62tKKSrwNl6aKiP0jUwym4c74Si4RSStvJ4KmLS2KsJszDGQhzvhKLhFJG2ccxkL7nxePangFpG06R2eZHxqNu37TM6nqSp/V08quEUkbTquzCjxdiogvLTsPR+nBCq4RSRtOpMbBNd73+OuLS8mHDINlYiIrEZndIzighBrE71hL4VDRkNFMT0aKhERWbnkiclQKL37TC6kqSqioRIRkdXI1IySpKY8XYSj4BaRtJiejXFmcDyjwd1YGaF3WPO4RURW5NzFy8zEHBs92q5sPk1VEUYnZxidnMnYe2YDBbeIpEWmLi51tXzdCUfBLSJp0RGNTwXMxOKbpEYFt4jIynX2j1FdWsiasqKMvefaqvxc9q7gFpG0yPSMEnhp2Xu+TQlUcItIWvgR3JHCMFUlhRoqERFZrvGpGXqGJjI6vp2Uj1uYKbhFZNVO948Dmbm41FzxudwKbhGRZblycSkfetxNlfm396SCW0RWrSMan8PdXuftPpPzaaqKEM2zLcwU3CKyap39Y6ytilBaVJDx926ojBBz0J9HW5gpuEVk1Tp8mFGSlI+rJxXcIrIqzjk6oqP+B3cejXMruEVkVS6OTzM8MeNbcDdWFQPQp+AWEUlN8uJSGzOwXdl8asvybwszBbeIrEry4lJ+zOGGl7YwuzCkk5MiIinp6B+jMGysX1PiWw35tghHwS0iq9IRHaW1ppSCsH9x0qTgFhFJXUd0LKO73synsbJYY9wiIqmYjTm6BsZ9OzGZ1FgVYWRihvGp/NjCTMEtIit27uI4U7MxbvDpxGRSvi3CUXCLyIpd2a7M5x53MrjzZcd3BbeIrNipaHIOt7897obK/NoJR8EtIivWkdhnsiaD+0zOpynP9p5MObjNLGxmB83s+14WJCLB0REd9WXXm7nKiwsoLy7QGPc8Pg4c9aoQEQmebJgKmNRYWUzfiIL7CjNbB/wb4O+8LUdEgmJkYpq+kUnfT0wmNVZG1OOe4yHg00BsoQPMbLeZ7TOzfdFoNC3FiUj2Sm5XttHnqYBJ8dWTmlUCgJm9Fehzzu1f7Djn3B7n3C7n3K76+vq0FSgi2Sk5FfCGbOlxV8WXvcfyYAuzVHrcrwTeZmangW8ArzOzr3lalYhkvY7oKCGD1trM7zM5n6bKCDMxx+D4lN+leG7J4HbOfdY5t8451w7cDzzmnHu355WJSFY71T/G+ppSigvCfpcCxE9OQn6sntQ8bhFZkY7oWFZMBUxqzKNFOMsKbufc4865t3pVjIgEQyzm6OwfzZqpgPDSIpx8OEGpHreILFvP8AQT07GsmQoIUF9ejFl+rJ5UcIvIsp3oHQFgUxb1uAvCIerKi+nVGLeIyPVO9MYvLrWlscLnSq7VVBlRj1tEZD4n+kaoKy9mjc8Xl5orX/aeVHCLyLId7x1lc0P2DJMkNVUVK7hFROZyznGyb5QtjdkX3I0VES6OTzMxPet3KZ5ScIvIsvQMTTA6OcOmLBvfhviyd4C+HJ8SqOAWkWU50Zc4MZmNQyXJRTg5fnlXBbeILEtyKuDmLOxxX9kJJ8enBCq4RWRZjveOUFde5Pt2ZfNprMiPZe8KbhFZlhN9o2xuyL7eNkBlSQGRwpB63CIiSc45TvaOsjkLZ5QAmFl8Q4URnZwUEQHi1wEZmZzJyvHtpMbKSM4ve1dwi0jKjieWumfj4pukpqrcX/au4BaRlCVnlGTbNUqu1pi4XolzubuFmYJbRFJ2vHeE2rLsnFGS1FgZYWomxtDlab9L8YyCW0RSdrRnhG1rK/0uY1HJRTi5PFyi4BaRlMzMxnixd4Rta7N3mATiF5qC3F6Eo+AWkZR09o8xNRPL+h53Qx4swlFwi0hKjvQMA2R9cL+0aXDuzuVWcItISo70DFMYNm7Iou3K5lNUEKK2rEhj3CIiR3tG2NRQQVFB9sdGU1WEnkuX/S7DM9n/CYhIVjjaM5z1JyaTmqtL6L6kHreI5LH+0UmiI5Nsz/Lx7aSW6hK61eMWkXx2NCAnJpNaqksYmZxheCI3F+EouEVkSUEL7ubqEoCc7XUruEVkSUd7RmiqjGT1UverNVfHpwSev6jgFpE8dfj8ENubg9HbhvhQCajHLSJ5amxyhpPRUW5ZV+V3KSmrKy+mKBzifI7OLFFwi8iiDp8fwjkCFdyhkLG2OsJ59bhFJB8dOj8EwI6Wap8rWZ7mqtydEqjgFpFFPX9uiOaqCPUVxX6XsiwtaxTcIpKnnj93iVvWBau3DfEpgb3DE0zPxvwuJe0U3CKyoKHxaU4PjLMjQOPbSS3VEWIuN6/LreAWkQUd7o6PbwfpxGRSLi/CUXCLyIKeO3cJgFsCdmISXprLnYszSxTcIrKgg2cusaGujKrSQr9LWbZkjzsXV08quEVkXs45DnRd5OVta/wuZUUihWEaKoo5Mzjudylpt2Rwm9l6M/u5mR01sxfM7OOZKExE/HV6YJyBsanABjdAa00pXfkY3MAM8Cnn3DbgduDDZrbd27JExG/7Tg8CsCvIwV1bytl8DG7nXI9z7kDi+xHgKNDidWEi4q/9XRepjBRk/R6Ti2mrKePC8AQT07N+l5JWyxrjNrN24Dbg6Xke221m+8xsXzQaTU91IuKb/Ynx7VDI/C5lxdpqS3EOzl3MrV53ysFtZuXAPwOfcM4Nz33cObfHObfLObervr4+nTWKSIZdGp/iRN9ooMe3AdbXlALQNZCHwW1mhcRDe69z7hFvSxIRvx04cxGAl7fV+FzJ6rTVxoM712aWpDKrxIC/B4465z7vfUki4renOwYpCod42frgLby5Wm1ZEWVF4bzscb8SeA/wOjN7NvH1Fo/rEhEf/erUALe1VlNSFPa7lFUxM9bXlOZcj7tgqQOcc/8KBPfshIgsy6XxKQ53D/GJe7b4XUpatNWWcio65ncZaaWVkyJyjac6BnEO7txU63cpadFWW8aZwXFiMed3KWmj4BaRazx5qp+SwjC3BvAa3PNZX1PK1EyM3pHcubyrgltErvGrUwO8YkMNRQW5EQ9tOTglMDc+GRFJi77hCU70jXLnDbkxTAKwoa4MgNP9uTPOreAWkSv+5Xh81fOrNtf5XEn6NFeXUFwQ4mTfqN+lpI2CW0SueOxYH02VEbavrfS7lLQJh4yN9eWcjCq4RSTHTM3E+MWJfu7e2kB83V3u2NRQrh63iOSeZ04PMjo5w+u2NvhdStptqi/n/KXLXJ7KjasEKrhFBICfHe2jqCDEK3Nk/vbVNjWU4xycypHhEgW3iOCc47FjvdyxsZbSoiUXVAfODQ3xmSUKbhHJGcd7Rzk9MM7rt+XeMAnEpwSGDE7lyDi3gltE+P7z3YQM3nTzWr9L8URxQZjWmtKcmVmi4BbJc845fvB8D7dvrKW+otjvcjyzqaGcE70KbhHJAUd7RujoH+OttzT7XYqntjZV0tE/lhP7Tyq4RfLc957vJhwy3nhTo9+leOqm5kpmY47jvSN+l7JqCm6RPDYzG+Of95/jtVvqqS3P3WESgO3N8dWgL3Rft2Vu4Ci4RfLYEyei9I1M8s7fWO93KZ5bv6aU8uICjii4RSTIvvXMOerKi3JyteRcoZCxbW0FR3oU3CISUH0jE/z0aC+//bIWCsP5EQU3NVdxtGeY2YDvhpMfn5aIXOdrT3Yx6xwP3N7mdykZc1NzJeNTs4FfQangFslDE9OzfO3pM9yzteHKRgP54OVtawDY33XR50pWR8Etkoe+++x5BsemeP+rNvhdSkZtqCtjTWkhBxTcIhIkM7Mx/ubxU9zUXMkdG3PvSoCLMTN2tq7hwBkFt4gEyLcPnuf0wDifeP2WnNswIRU729ZwKjrGpfEpv0tZMQW3SB6ZmonxxcdOsKOlKmevBLiUna3xce59p4Pb61Zwi+SRL//qNGcHL/PJN+RnbxtgZ1s1kcIQvzgR9buUFVNwi+SJvpEJ/upnJ7j7xnruvjE/e9sQv8Tr7Rtr+cWJfr9LWTEFt0ieePAHR5mcmeU/vXW736X47tWb6+noH+Ps4LjfpayIglskD/zoUA/ffbabD712Exvry/0ux3ev2VIHxK/VEkQKbpEcd2Fogv/w7UPsaKniI6/b5Hc5WeGG+nLaakv58eELfpeyIgpukRw2MT3LH35tP5MzMb7wO7fmzTVJlmJm3HtLM7882U90ZNLvcpZNn6JIjorFHJ995BDPnb3E5995K5saKvwuKavce2szMQc/OtzjdynLpuAWyUHOOf700Rf49sHz/Ps3bMnZTYBX48amCrY2VfCtfWdxLlhXC1Rwi+SYqZkYn/qn5/jqU1384V0b+fDdGtdeyHvuaOPw+WGe7hz0u5RlUXCL5JC+4Ql+9x9+zSMHzvPJ39rCZ968NW8X2qTi3+1cR01ZEXue6PC7lGVRcIvkAOccPzzUwxsfeoIDZy7y+Xfeysfu2azQXkKkMMz77mznsWN9PN0x4Hc5KVNwiwTcwTMXedf/eooP7T1Ay5oSfvCxV/H2nev8Liswfv/VG2mpLuE/fucwl6dm/S4nJSkFt5m9ycxeNLOTZvYZr4sSkcWNTEzz7YPnePtf/5J/+9e/4njvKJ+77ya+86FXavbIMpUUhfkvb9/Byegon33keWIB2NZsyeA2szDwJeDNwHbgXWamNbMSLHv3Qns7hELx/+7d63dFyzIxPcv+rkH2PHGK9/3jr3n5537KH3/zOQbGpvize7fzxKfv5j13tFNw1TztvYf20v5QO6E/D9H+UDt7DwWrzZl015Z6PvVbW/jOs9189BsHGZ6Y9rukRRWkcMwrgJPOuQ4AM/sGcB9wxMvCRNJm717YvRvGE9el6OqK3wZ44AFfSpqZjTExE2NievbK18jEDINjUwyMTjEwNkXv8ASnB8boTFxTI9kR3FBXxnvvaOPNO5q4bf0aQqHrx7H3HtrL7u/tZnw63uauoS52fy/e5gd2+NPmbPfhuzcRDoX4y5+8yNMdgzzwm6289sZ6tjZVUlIU9ru8a9hS8xfN7B3Am5xzv5+4/R7gN51zH1noObt27XL79u1bdjFv/e+/YGI6dt39C9W4aOULPLjYc5b7Pov96NwCz1r0Ocv8C22xzy4TNS/+s1z4XZb7nIXbkmL7L16EWCxx/1UhFwpBdfWSr7eCpixa89RsjOnZpT/s0qIw7bVlbKgvY0NtGTvWVbGzdQ31FcVLPrf9oXa6hrquu7+tqo3Tnzi95PPz2bNnL/GF/3ecfzn+0nVMKiMFlBYVUBA2CsMhQsa8J35rSov41gfvWNH7mtl+59yuVI5Npcc932np637rzGw3sBugtbU1lfe+zqb68oV/oRc4Ob7YOfOFzqgv/pzlvc9iZ+0XfGSRAmyBB5db1+LPWaTmhZ6zgsZkoubFJk1ceeh/PHLt/cmANoOPfnSRalZe12LPKSoIUVIYJlIYIlIYvvJVXhymtqyYmrIiasuLKC1K5Z/n/M4MnVnW/fKSl62v5svvfwXRkUmeOT1IZ/8YfcMTTEzHmJ6NMR1zC46DV0RW/pktRyrvcg5Yf9XtdUD33IOcc3uAPRDvca+kmIfuv20lTxNZ3Md/HB8emautDd72t5mvJwNaq1rn7XG3Vq2sU5WP6iuKecuO7FxxmsqskmeAzWa2wcyKgPuBR70tSySNHnwQSkuvva+0NH5/jnrwngcpLby2zaWFpTx4T+62OZ8sGdzOuRngI8D/BY4C33LOveB1YSJp88ADsGdPvIdtFv/vnj2+nZjMhAd2PMCee/fQVtWGYbRVtbHn3j06MZkjljw5uRIrPTkpIpKvlnNyUisnRUQCRsEtIhIwCm4RkYBRcIuIBIyCW0QkYBTcIiIBo+AWEQkYBbeISMB4sgDHzKLAPBeHSEkd0J/GcoJAbc59+dZeUJuXq805V5/KgZ4E92qY2b5UVw/lCrU59+Vbe0Ft9pKGSkREAkbBLSISMNkY3Hv8LsAHanPuy7f2gtrsmawb4xYRkcVlY49bREQW4Vtwm9mbzOxFMztpZp+Z5/FiM/tm4vGnzaw981WmTwrt/aSZHTGz583sZ2bW5ked6bRUm6867h1m5sws8DMQUmmzmb0z8Vm/YGb/J9M1plsKv9utZvZzMzuY+P1+ix91pouZ/YOZ9ZnZ4QUeNzP7YuLn8byZ7Ux7Ec65jH8BYeAUsBEoAp4Dts855kPA3ya+vx/4ph+1ZrC9dwOlie//KMjtTbXNieMqgCeAp4Bdftedgc95M3AQWJO43eB33Rlo8x7gjxLfbwdO+133Ktt8F7ATOLzA428BfkR8v+rbgafTXYNfPe5XACedcx3OuSngG8B9c465D/hy4vuHgXtssS3Vs9uS7XXO/dw5N564+RTxTZmDLJXPGOBzwH8FJjJZnEdSafMfAF9yzl0EcM71ZbjGdEulzQ6oTHxfxTybjQeJc+4JYHCRQ+4DvuLingKqzSytuw77FdwtwNmrbp9L3DfvMS6+7+UQUJuR6tIvlfZe7QPE/48dZEu22cxuA9Y7576fycI8lMrnvAXYYma/NLOnzOxNGavOG6m0+c+Ad5vZOeCHwEczU5pvlvvvfdkK0vliyzBfz3nu9JZUjgmKlNtiZu8GdgGv8bQi7y3aZjMLAV8Afi9TBWVAKp9zAfHhktcS/6vqF2Z2s3Pukse1eSWVNr8L+N/Ouf9mZncAX020OeZ9eb7wPLv86nGfA9ZfdXsd1//5dOUYMysg/ifWYn+eZLNU2ouZvR74E+BtzrnJDNXmlaXaXAHcDDxuZqeJjwU+GvATlKn+Xn/XOTftnOsEXiQe5EGVSps/AHwLwDn3JBAhfk2PXJXSv/fV8Cu4nwE2m9kGMysifvLx0TnHPAr8buL7dwCPucTIfwAt2d7EsMH/JB7aQR/3hCXa7Jwbcs7VOefanXPtxMf13+ac2+dPuWmRyu/1d4ifiMbM6ogPnXRktMr0SqXNZ4B7AMxsG/Hgjma0ysx6FHhvYnbJ7cCQc64nre/g45nZtwDHiZ+R/pPEff+Z+D9eiH+4/wScBH4NbPT7bLLH7f0p0As8m/h61O+avW7znGMfJ+CzSlL8nA34PHAEOATc73fNGWjzduCXxGecPAu8we+aV9nerwM9wDTx3vUHgA8CH7zqM/5S4udxyIvfa62cFBEJGK2cFBEJGAW3iEjAKLhFRAJGwS0iEjAKbhGRgFFwi4gEjIJbRCRgFNwiIgHz/wFgL+Ngu1lmGAAAAABJRU5ErkJggg==\n", "text/plain": [ "
" ] }, "metadata": { "needs_background": "light" }, "output_type": "display_data" }, { "name": "stdout", "output_type": "stream", "text": [ "Likelihood-Ratio: -11.889708980545516\n" ] }, { "data": { "image/png": "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\n", "text/plain": [ "
" ] }, "metadata": { "needs_background": "light" }, "output_type": "display_data" }, { "name": "stdout", "output_type": "stream", "text": [ "Likelihood-Ratio: -12.226181217166728\n" ] }, { "data": { "image/png": "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\n", "text/plain": [ "
" ] }, "metadata": { "needs_background": "light" }, "output_type": "display_data" }, { "name": "stdout", "output_type": "stream", "text": [ "Likelihood-Ratio: -12.562653453787942\n" ] }, { "data": { "image/png": "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\n", "text/plain": [ "
" ] }, "metadata": { "needs_background": "light" }, "output_type": "display_data" }, { "name": "stdout", "output_type": "stream", "text": [ "Likelihood-Ratio: -12.899125690409154\n" ] }, { "data": { "image/png": "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\n", "text/plain": [ "
" ] }, "metadata": { "needs_background": "light" }, "output_type": "display_data" }, { "name": "stdout", "output_type": "stream", "text": [ "Likelihood-Ratio: -13.235597927030367\n" ] }, { "data": { "image/png": "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\n", "text/plain": [ "
" ] }, "metadata": { "needs_background": "light" }, "output_type": "display_data" }, { "name": "stdout", "output_type": "stream", "text": [ "Likelihood-Ratio: -12.724772303264377\n" ] }, { "data": { "image/png": "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\n", "text/plain": [ "
" ] }, "metadata": { "needs_background": "light" }, "output_type": "display_data" }, { "name": "stdout", "output_type": "stream", "text": [ "Likelihood-Ratio: -13.061244539885589\n" ] }, { "data": { "image/png": "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\n", "text/plain": [ "
" ] }, "metadata": { "needs_background": "light" }, "output_type": "display_data" }, { "name": "stdout", "output_type": "stream", "text": [ "Likelihood-Ratio: -13.397716776506803\n" ] }, { "data": { "image/png": "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\n", "text/plain": [ "
" ] }, "metadata": { "needs_background": "light" }, "output_type": "display_data" }, { "name": "stdout", "output_type": "stream", "text": [ "Likelihood-Ratio: -12.886891152740812\n" ] }, { "data": { "image/png": "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\n", "text/plain": [ "
" ] }, "metadata": { "needs_background": "light" }, "output_type": "display_data" }, { "name": "stdout", "output_type": "stream", "text": [ "Likelihood-Ratio: -12.376065528974822\n" ] }, { "data": { "image/png": "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\n", "text/plain": [ "
" ] }, "metadata": { "needs_background": "light" }, "output_type": "display_data" }, { "name": "stdout", "output_type": "stream", "text": [ "Likelihood-Ratio: -12.712537765596034\n" ] }, { "data": { "image/png": "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\n", "text/plain": [ "
" ] }, "metadata": { "needs_background": "light" }, "output_type": "display_data" }, { "name": "stdout", "output_type": "stream", "text": [ "Likelihood-Ratio: -13.049010002217248\n" ] }, { "data": { "image/png": "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\n", "text/plain": [ "
" ] }, "metadata": { "needs_background": "light" }, "output_type": "display_data" }, { "name": "stdout", "output_type": "stream", "text": [ "Likelihood-Ratio: -13.38548223883846\n" ] }, { "data": { "image/png": "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\n", "text/plain": [ "
" ] }, "metadata": { "needs_background": "light" }, "output_type": "display_data" }, { "name": "stdout", "output_type": "stream", "text": [ "Likelihood-Ratio: -13.721954475459674\n" ] }, { "data": { "image/png": "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\n", "text/plain": [ "
" ] }, "metadata": { "needs_background": "light" }, "output_type": "display_data" }, { "name": "stdout", "output_type": "stream", "text": [ "Likelihood-Ratio: -14.058426712080886\n" ] }, { "data": { "image/png": "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\n", "text/plain": [ "
" ] }, "metadata": { "needs_background": "light" }, "output_type": "display_data" }, { "name": "stdout", "output_type": "stream", "text": [ "Likelihood-Ratio: -14.394898948702098\n" ] }, { "data": { "image/png": "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\n", "text/plain": [ "
" ] }, "metadata": { "needs_background": "light" }, "output_type": "display_data" }, { "name": "stdout", "output_type": "stream", "text": [ "Likelihood-Ratio: -13.884073324936107\n" ] }, { "data": { "image/png": "iVBORw0KGgoAAAANSUhEUgAAAW4AAAD8CAYAAABXe05zAAAABHNCSVQICAgIfAhkiAAAAAlwSFlzAAALEgAACxIB0t1+/AAAADl0RVh0U29mdHdhcmUAbWF0cGxvdGxpYiB2ZXJzaW9uIDMuMC4zLCBodHRwOi8vbWF0cGxvdGxpYi5vcmcvnQurowAAHYRJREFUeJzt3Xt0nHed3/H3VzOSRnfZ1sWyLVu+xHGMAwlxQi6QLCTdQmBJ4dBuckK2y6G40LJd2J7TwuHs2d1y6JYeuoW2ObtrQqF0vbAsbCFwtqGUTZY0d5tcnNhxsGxZsiVb9+voPr/+MTOOLOvySHqeefTMfF7n+ETSPDPz/VnKxz/9nt/FnHOIiEh0FIVdgIiIrIyCW0QkYhTcIiIRo+AWEYkYBbeISMQouEVEIkbBLSISMQpuEZGIUXCLiERMPIgXraurcy0tLUG8tIhIXjp27Fivc67ey7WBBHdLSwtHjx4N4qVFRPKSmZ3zeq2GSkREIkbBLSISMQpuEZGIUXCLiESMgltEJGIU3CIiEaPgFhGJGAW3iORU19A4X//FGZ4+3Rt2KZEVyAIcEZGFnB9Ict9/e4q+sSkA/vjD1/PALdtDrip61OMWkZz5/R++yuRMikc/fQfvuqaOL/7kBN0jE2GXFTkKbhHJiROdwzx+qodP3rWLt26r5Yv3HWBiepZvPtUWdmmRo+AWkZz41tNnKS+J8dCtLQC01FXwnn2N/PXRDmZmU+EWFzEKbhEJ3NRMisdevch7D2ymprz48tc/dONWekenOHZuIMTqokfBLSKBe7q1l+GJGe490HTF1+/cW0dxzPj5690hVRZNCm4RCdzPTlyioiTGO6+pu+LrVYli3rFzEz8/eSmkyqJJwS0igXvmTB+37NxIojh21WN37q2jtWeMnpHJECqLJgW3iASqe3iCMz1j3LZ704KP37RjIwDHzvXnsqxIU3CLSKCePZsO5Ft3LRzcB7ZWUxIv4oU23aD0SsEtIoF69kwfVaVx9jdVL/h4aTzGDdtqOaqZJZ4puEUkUC+c7edgywbiscXj5m3NNZzsGmZa87k9UXCLSGDGJmc43TPK25prl7zuLVtqmJpJcaZnLEeVRZuCW0QC81rnMM7BW7fVLHnd/i3VmeuHclFW5Cm4RSQwr5wfBODA1qWDe1ddBaXxIk50DueirMhTcItIYI5fGKKpJkFDVWLJ6+KxIvY1VXOiS8HthYJbRAJz/PzQsr3trL0NlfyqezTgivKDgltEAjE2OcOZ3jEObPEW3LsbKukZmWRofDrgyqJPwS0igcj2nvc1VXm6fk99JQCtPep1L0fBLSKBeOPiCADXNnoL7t0N6eA+reGSZXkKbjP7rJm9Zmavmtl3zGzpOw0iUvBOXRohUVxE88ZyT9c3byijJFakHrcHywa3mW0F/hVw0Dl3AIgB9wddmIhE2xuXRtjTUEmsyDxdH48V0VJXTqt63MvyOlQSB8rMLA6UA53BlSQi+eCNSyPs9ThMkrWnoZJWrZ5c1rLB7Zy7AHwFaAe6gCHn3P8JujARia7B5BSXhic9j29n7a6v5FzfmPYsWYaXoZINwH3ATmALUGFmH13gukNmdtTMjvb09PhfqYhExhuX0sMdezevLLi3bywn5aBzcDyIsvKGl6GSe4Czzrke59w08DfA7fMvcs4dds4ddM4drK+v97tOEYmQNy6tbEZJ1vbMjcz2/qTvNeUTL8HdDtxqZuVmZsDdwMlgyxKRKDvdPUpFSYymmpVNQNu+ScHthZcx7ueA7wO/BI5nnnM44LpEJMLO9I6xq76SdF/Pu8aqBCWxIgX3MuJeLnLO/QHwBwHXIiJ54mzvKDc2b1jx84qKjG0by+hQcC9JKydFxFeTM7NcGBinpa5iVc/fvrFcPe5lKLhFxFcd/UlSLr3H9mps31hOe5+CeykKbhHxVfb4sZ1rCO7hiRmGktolcDEKbhHx1dnedHCvdqikWVMCl6XgFhFfne0do66yhJqy4lU9X3O5l6fgFhFfnekdW/UwCbzZ4z7Xrz1LFqPgFhFftfWO0bJp9cFdWRqntrxYy96XoOAWEd+MTs7QPTLJzvrVBzdAU00ZXYMTPlWVfxTcIuKbtsyNydVOBczaUpOgc0jBvRgFt4j45kxvdipg5ZpeZ0ttmYZKlqDgFhHfnO0Zwwx2bPJ2XNlimmoTDI1Pk5ya8amy/KLgFhHfnO0dZUtNGYni2JpeZ0tNGQCdGudekIJbRHzT3p+8PA97LbLbwWq4ZGEKbhHxTcfAuC/BvaU23ePuGlJwL0TBLSK+GJ+apWdkkuaNZWt+rc01Ccw0VLIYBbeI+OL8QHqJerMPPe7iWBH1laUaKlmEgltEfNHhY3BDerikS3O5F6TgFhFfZPfQbt7gV3An6NQY94IU3CLii46BccqKY9RVlvjyek016UU4zjlfXi+fKLhFxBft/UmaN5at+IDgxWypLWNiOsWgDlS4ioJbRHzR4dMc7qwt2bncGi65ioJbRNbMOUdHf5JtPo1vAzTVavXkYhTcIrJmA8lpxqZmfZtRAm+unrw0rOCeT8EtImvWkTlmzM+hkk0VJRQZdCu4r6LgFpE1y54P6ceqyax4rIi6ylIuKrivouAWkTW7vPjGxzFugMbqBJeGJ319zXyg4BaRNevoT7KpooSK0rivr5sObvW451Nwi8iadfSPs83H8e2sxupSukfU455PwS0ia+bXPtzzNVYn6B+bYnJm1vfXjjIFt4isycxsis7BcZo3+HdjMquxuhSAHvW6r6DgFpE16RqaYCblAulxN1RrLvdCFNwisiZ+b+c61+bLwa0e91wKbhFZk/P96b1EghrjBvW451Nwi8iatPcniRXZ5SXqftpQXkxxzNTjnsdTcJtZrZl938xeN7OTZnZb0IWJSDR0DCRpqkkQj/nfDzQzGqoSWvY+j9fZ8l8DHnPOfcTMSgD/fycSkUgKaipgVmN1KZdGFNxzLftPpJlVA3cC3wBwzk055waDLkxEoqGjf9z3pe5zNVYnuKizJ6/g5XebXUAP8E0ze9HMHjGzivkXmdkhMztqZkd7enp8L1RE1p/k1Ay9o5Ns3xRscHdrjPsKXoI7Drwd+FPn3I3AGPC5+Rc55w475w465w7W19f7XKaIrEfnB9IzSrYFsPgmq7E6wcjkDGOTM4G9R9R4Ce7zwHnn3HOZz79POshFpMAFsQ/3fNnVk9qz5E3LBrdz7iLQYWbXZr50N3Ai0KpEJBLe3Ic72KES0FzuubzOKvkd4EhmRskZ4GPBlSQiUdHRP05ZcYxNFSWBvUe2x63gfpOn4HbOvQQcDLgWEYmY7FRAMwvsPbRfydW0clJEVu38QNLX48oWUlUap7wkptWTcyi4RWRVnHO09ycDHd+G9OpJnYRzJQW3iKxK/9gUyanZQBffZDVUlSq451Bwi8iq5GJGSVZjdULTAedQcIvIqnQMBLed63zZHrdzLvD3igIFt4isSnbxTZCrJrMaqxNMTKcY0epJQMEtIqvU0Z9kU0UJFaVel4OsXkN29aTGuQEFt4isUnt/MtDNpeZqqNIRZnMpuEVkVToGkjmZUQJz9ytRjxsU3CKyCtOzKToHJ3JyYxLmrp5UjxsU3CKyCl2DE8ymXM6Cu7I0TmVpXHO5MxTcIrJiHQOZGSUBL3efq6GqVHO5MxTcIrJi7TnYh3u+hupSzSrJUHCLyIq19yeJFxlNNbnrcaf3K1GPGxTcIrIKHf1Jtm4oI1YU3Hau82n15JsU3CKyYh2ZfbhzqbE6weRMiuEJrZ5UcIvIiuViO9f5slMCNc6t4BaRFRqZmGYgOZ2zxTdZDVXZI8w0zq3gFpEV6ejP3a6Ac2UPDdbqSQW3iKxQGFMBQT3uuRTcIrIi5weyByjkbiogQIVWT16m4BaRFWnvT1KViFNTVpzz926oLqVHqycV3CKyMu2ZqYBmuZvDndVYpUODQcEtIivU0Z+77Vzna6gu5ZJuTiq4RcS7VMrRMTCeswMU5musTtA9PFnwqycV3CLiWffIJFMzqZwvvslqqCpNr54cL+zVkwpuEfEsu51rcw4OCF7I5QMVCny4RMEtIp6194UzhzursSp7aHBhzyxRcIuIZ+39Scxga0g97sbLR5ipxy0i4kl7f5LN1QlK47FQ3r8hc2iwhkpERDxq6xujZVNFaO9fXhKnqjSuoZKwCxCR6GjrHaOlLrzghswRZupxi4gsbyiZ3s61JaQ53Fk6wkzBLSIetfWNAYTf484cYVbIPAe3mcXM7EUz+0mQBYnI+nQ5uEMc44bM6smRwl49uZIe9+8CJ4MqRETWt3OZOdw7Qh4qaahOMDWTYmh8OtQ6wuQpuM1sG/B+4JFgyxGR9aqtd4ymmgSJ4nCmAmbpQAXvPe6vAv8GSC12gZkdMrOjZna0p6fHl+JEZP0Ieypglo4w8xDcZvYBoNs5d2yp65xzh51zB51zB+vr630rUETWh7a+JC114Q6TADRWq8ftpcd9B/BBM2sDvgu8x8z+ItCqRGRdGRqfpn9sal30uBuqtOx92eB2zn3eObfNOdcC3A/8nXPuo4FXJiLrxrnMjJId6yC4y0piVCXiBX2EmeZxi8iy2jIzSnaGPIc7K70Ip3B73PGVXOycewJ4IpBKRGTdautN97jD2s51vkJfhKMet4gsq60vPRWwrCTcqYBZ2UU4hUrBLSLLausdC33hzVwN1aUFffakgltElnWuL7luxrchPbNkajbFYLIwV08quEVkSUPj0/SNTa2LGSVZjQV+oIKCW0SW1NozCsCe+sqQK3nT5dWTBboIR8EtIks63Z0O7t0N6yi4C3wRjoJbRJbU2jNKSayI5pAOCF5I9uzJQp1ZouAWkSW1do/SUldOPLZ+4iJRHKM6EadbPW4Rkau19oyxZx0Nk2QV8hFmCm4RWdTkzCzn+sbYvY5uTGY1VJdqVomIyHzn+pKkHOuzx12V0KwSEZH5WrMzStZljztB98gEqVThrZ5UcIvIorJTAXfVr5/FN1lbahNMzzp6xwqv163gFpFFtfaMsrW2jPKSFW0kmhNbatLTEzsHC2+cW8EtIos63TO6LnvbAE216UU4XYPjIVeSewpuEVlQKuVo7V6fUwFhTo97SD1uEREAuoYnGJ+eXZc3JgFqy4spK47RqR63iEjaGxdHANjbWBVyJQszM5pqE3QNKbhFRAA4eXEYgGs3r8/ghvRwiW5OiohknLo4wpaaBDVlxWGXsqgttQkNlYiIZJ26OMK+puqwy1hSU00ZPaOTTM2kwi4lpxTcInKVqZkUrT2j63qYBNI9bucKb19uBbeIXOVM7yjTs4596z64s4twCmu4RMEtIlc5lZlRsm/z+h8qAegqsLncCm4RucrrF0cojtm6XTWZtSWzerKzwKYEKrhF5Cqvdw2zu76S4nV06s1Cykvi1JYXa6hEROTUxZF1P76d1VRTRleBzeVWcIvIFQbGpugcmuC6dT4VMGtrbYIL6nGLSCE7fmEIgOu31oRciTfbNpTT0Z/EucI5UEHBLSJXyAb3WyIS3Ns3ljM2NUv/2FTYpeSMgltErvDqhSF2bCpf10vd52reWA5Ax0DhDJcouEXkCq+cH4rMMAmke9wAHf3JkCvJHQW3iFw2MDbFhcHxSAX3tg3pRTjtCu43mVmzmT1uZifN7DUz+91cFCYiuRe1G5MAFaVx6ipLOD9QOMHt5QTQGeBfO+d+aWZVwDEz+5lz7kTAtYlIjkXtxmTWtg3l6nHP5Zzrcs79MvPxCHAS2Bp0YSKSe8fPD9ESoRuTWds3ltPRr5uTCzKzFuBG4LkgihGRcL18fpDrt9WGXcaKNW8s48LgODOzhbEvt+fgNrNK4AfAZ5xzwws8fsjMjprZ0Z6eHj9rFJEc6Bwcp2togpu2RzC4N5Qzm3IFs0ugp+A2s2LSoX3EOfc3C13jnDvsnDvonDtYX1/vZ40ikgPHzg0AcLBlY8iVrFyhTQn0MqvEgG8AJ51zfxJ8SSIShmPnBigrjkVmc6m5dtSlt5892zcWciW54aXHfQfwEPAeM3sp8+fegOsSkRw7dm6AG5pria/zrVwX0lSdIFFcxNmewgjuZacDOuf+H2A5qEVEQpKcmuFE1zCfumt32KWsSlGR0bKpgjO9hRHc0funVUR891LHILMpx007NoRdyqrtrq/kTM9o2GXkhIJbRHjuTD9FBm+PcHDvqq+gY2CcqZn8nxKo4BYRnmnt4/qtNZFbeDPXzroKZlOO9v78Hy5RcIsUuOTUDC92DHDb7rqwS1mTXfWVAJwpgBuUCm6RAvdC2wDTs47bd28Ku5Q1yZ5IXwg3KBXcIgXu6dZeimPGwZbojm8DVCeKqassLYgblApukQL39Ok+bmiupbzEy2ah69uuugoNlYhIfusZmeT4hSHedU1+bFOxp7GSNy6N5P3BwQpukQL2+KluAO6+riHkSvxx3eYqhidm8n6zKQW3SAF7/PVuNlcn2N9UHXYpvtiXacepiyMhVxIsBbdIgZqaSfHkr3p5974G0nvJRd+1mQ2yTl68aufpvKLgFilQz5/tZ3Ryhrv35ccwCaRnlmytLeP1LvW4RSQP/e2rXZSXxLhjT7QX3sy3b3MVr6vHLSL5Zno2xWOvXuSe6xopK4mFXY6v9jVV0dozxsT0bNilBEbBLVKAnm7to39sig+8tSnsUnx3/dYaZlOOk1352+tWcIsUoB+/3ElVaZy7rs2P+dtzva05fWbmyx2DIVcSHAW3SIFJTs3w01cv8utv2UxpPL+GSQCaasporC7lJQW3iOSLn7zcxcjkDL95c3PYpQTmbdtqefn8UNhlBEbBLVJg/vL5dvY0VHJzxDeVWsoN22s52zvGYHIq7FICoeAWKSAnOod5qWOQB27ZnjeLbhZyQ2ac+8X2/BwuUXCLFJBvPnWW0ngRH75xa9ilBOrG5g2UxIp45kxf2KUEQsEtUiA6B8f54UsXuP/mZjZUlIRdTqDKSmLcuL2Wp1t7wy4lEApukQLxyJNncQ4+ceeusEvJidt31/Fa53BejnMruEUKwKXhCb7zfDsfvGEL2zaUh11OTty+ZxPOwbNn+sMuxXcKbpEC8JWfnmI25fjsPXvDLiVn3ratlsrSOI+/3h12Kb5TcIvkudc6h/j+L8/z23e00LyxMHrbACXxIt69r4GfnbzEzGwq7HJ8peAWyWPTsyk+94PjbCwv4V++e0/Y5eTc+w5spn9siufb8mu4RMEtksf+/O9bOX5hiC/+owPUlBWHXU7O/dq19ZTGi3js1Ythl+IrBbdInnqhrZ+v/fxXvP+tTdx7ff7tAuhFeUmce/Y38qOXOhmfyp9tXhXcInmoc3CcT/3FMbZtKOfff+j6sMsJ1UO37mBofJofv9IZdim+UXCL5Jnu4Qk++shzTEyn+Ppv3VSQQyRzvWPnRq5pqOTbz7ThnAu7HF8ouEXySEd/kge+/iwXhyf45sduZk9DVdglhc7M+MS7dvHqheG8GetWcIvkiSd/1cN9Dz9Fz8gk3/rYLdzcsjHsktaND799K3sbK/nyY68zORP9sW4Ft0jE9Y9N8bkfvMJD33iejRUl/OjT7+SWnQrtueKxIr7w/v209SX5yk9PhV3OmsW9XGRm7wW+BsSAR5xz/yHQqkRkWecHkhx5rp1vP93G+PQs//yuXXz2nr0kivPvVBs/3LW3nodu3cHXnzzLnoZKfvPm7WGXtGrL9rjNLAY8DLwP2A88YGb7gy5MxFdHjkBLCxQVpf975EjYFa1YKuU40TnMI0+e4YHDz/LOLz/On/19K+/e18Bjn7mTz7/vuitC+8jxI7R8tYWiPyqi5astHDkevTb77fc/sJ8799bzb39wnC8/9npkT4L30uO+BTjtnDsDYGbfBe4DTgRZmIhvjhyBQ4cgmUx/fu5c+nOABx8Mr66M6dkU49OzjE/NkpyaZWxyhp7RSXqGJ+kemaCjf5w3ukf41aVRRidnANhVX8Hv/YO9fOjGrQsuYz9y/AiHfnyI5HS6zeeGznHox+k2P3h9+G0OS0m8iMMP3cQf/fgEf/pEKz988QL/+KZt3LGnjms3V1FTVhyJAyZsuekxZvYR4L3OuX+W+fwh4B3OuU8v9pyDBw+6o0ePrriYD/zXJ5mYvnpPgaVqXLL6JR5c7KHVvtdiT3NLPGupv/rVzFryu/b08xZ+cOnnrO69Fnvm6t8r8+jAAMxe2bNyZune94YN857j4fVWWMdiDzpgcmaW6dmlv9mbKkq4prGSaxuruH5bLbfv3sSW2rIln9Py1RbODZ276us7anbQ9pm2JZ9bKJ4+3cvDT5zmmdY+UplvQXHMqCkrpiRWRFGRES8yioqMIo9hvrG8hO998rZV1WNmx5xzB71c66XHvVDFV/2kmdkh4BDA9u2rGzvaU1+5+A/xEn9vS/2VLvWv52KPLPU9Ws17LfktX/K9lqh9kYeWrs/n91plw1bz97um2h/+XwuGp+Hg01f3P1bb41pNjaXFRZQXxygrSf8pL4lRXhKnrrKEhqoE9VWlqxqzbh9qX9HXC9Hte+q4fU8dfaOTvHJhiNbuUfrGphhMTjMzm2I25Zh1jpmUW+Zf5jdVJTzdNlwzLz3u24A/dM79w8znnwdwzv3xYs9ZbY9bJBAtLenhkfl27IC2tlxXkxPqcUfPSnrcXqYDvgBcY2Y7zawEuB94dC0FiuTUl74E5fPGgcvL01/PU1+6+0uUF1/Z5vLicr50d/62uZAsG9zOuRng08BPgZPA95xzrwVdmIhvHnwQDh9O97DN0v89fHhd3JgMyoPXP8jh3zjMjpodGMaOmh0c/o3DBX1jMp8sO1SyGhoqERFZGb+HSkREZB1RcIuIRIyCW0QkYhTcIiIRo+AWEYkYBbeISMQouEVEIkbBLSISMYEswDGzHmCBzSE8qQN6fSwnCtTm/Fdo7QW1eaV2OOfqvVwYSHCvhZkd9bp6KF+ozfmv0NoLanOQNFQiIhIxCm4RkYhZj8F9OOwCQqA2579Cay+ozYFZd2PcIiKytPXY4xYRkSWEFtxm9l4zO2Vmp83scws8Xmpmf5V5/Dkza8l9lf7x0N7fM7MTZvaKmf3czHaEUaeflmvznOs+YmbOzCI/A8FLm83sn2S+16+Z2V/muka/efjZ3m5mj5vZi5mf73vDqNMvZvbfzazbzF5d5HEzs/+S+ft4xcze7nsRzrmc/wFiQCuwCygBXgb2z7vmXwB/lvn4fuCvwqg1h+19N1Ce+fhTUW6v1zZnrqsCfgE8CxwMu+4cfJ+vAV4ENmQ+bwi77hy0+TDwqczH+4G2sOteY5vvBN4OvLrI4/cC/5v02de3As/5XUNYPe5bgNPOuTPOuSngu8B98665D/gfmY+/D9xtqz1+O3zLttc597hzLpn59FlgW45r9JuX7zHAF4H/CEzksriAeGnzJ4CHnXMDAM657hzX6DcvbXZAdebjGqAzh/X5zjn3C6B/iUvuA77t0p4Fas2syc8awgrurUDHnM/PZ7624DUufe7lELApJ9X5z0t75/o46X+xo2zZNpvZjUCzc+4nuSwsQF6+z3uBvWb2lJk9a2bvzVl1wfDS5j8EPmpm54G/BX4nN6WFZqX/v69Y3M8XW4GFes7zp7d4uSYqPLfFzD4KHATuCrSi4C3ZZjMrAv4z8Nu5KigHvHyf46SHS36N9G9VT5rZAefcYMC1BcVLmx8AvuWc+09mdhvwPzNtTgVfXigCz66wetzngeY5n2/j6l+fLl9jZnHSv2It9evJeualvZjZPcAXgA865yZzVFtQlmtzFXAAeMLM2kiPBT4a8RuUXn+uf+Scm3bOnQVOkQ7yqPLS5o8D3wNwzj0DJEjv6ZGvPP3/vhZhBfcLwDVmttPMSkjffHx03jWPAv808/FHgL9zmZH/CFq2vZlhgz8nHdpRH/eEZdrsnBtyztU551qccy2kx/U/6Jw7Gk65vvDyc/1D0jeiMbM60kMnZ3Japb+8tLkduBvAzK4jHdw9Oa0ytx4Ffiszu+RWYMg51+XrO4R4Z/Ze4A3Sd6S/kPnavyP9Py+kv7l/DZwGngd2hX03OeD2/l/gEvBS5s+jYdccdJvnXfsEEZ9V4vH7bMCfACeA48D9YdecgzbvB54iPePkJeDXw655je39DtAFTJPuXX8c+CTwyTnf44czfx/Hg/i51spJEZGI0cpJEZGIUXCLiESMgltEJGIU3CIiEaPgFhGJGAW3iEjEKLhFRCJGwS0iEjH/H/N1rqz54oNoAAAAAElFTkSuQmCC\n", "text/plain": [ "
" ] }, "metadata": { "needs_background": "light" }, "output_type": "display_data" }, { "name": "stdout", "output_type": "stream", "text": [ "Likelihood-Ratio: -14.220545561557321\n" ] }, { "data": { "image/png": "iVBORw0KGgoAAAANSUhEUgAAAW4AAAD8CAYAAABXe05zAAAABHNCSVQICAgIfAhkiAAAAAlwSFlzAAALEgAACxIB0t1+/AAAADl0RVh0U29mdHdhcmUAbWF0cGxvdGxpYiB2ZXJzaW9uIDMuMC4zLCBodHRwOi8vbWF0cGxvdGxpYi5vcmcvnQurowAAHX9JREFUeJzt3XlwnPd93/H3F4vFsbhvCQBJkBR1ULJlybAsW/UlOR7HtSV3RnUUy2nsOub4iGun7aRuM5O48eiPpnUq1+MkZR0ncUJfld1adtL4lCaWLUuiToqkKV64eAAgLgJY3PvrH7tLgiCOxeJ59tln9/Oa4RDHs7vfHwB++MPv+R3mnENERMKjJOgCRERkcxTcIiIho+AWEQkZBbeISMgouEVEQkbBLSISMgpuEZGQUXCLiISMgltEJGRK/XjS5uZm19XV5cdTi4gUpGefffaCc64lk2t9Ce6uri4OHjzox1OLiBQkM+vN9FoNlYiIhIyCW0QkZBTcIiIho+AWEQkZBbeISMgouEVEQkbBLSISMgpuEcm5F/rH+fLPTtE/Gg+6lFDyZQGOiMhafnD4PB8/8BxLCccXf3qCb3/sjVzXWh10WaGiHreI5MzF2QX+03cOsffaWr77ibswgz/87svo0PLNUXCLSM48cnCAkel5PvfeW7h1Wz2fvHsPvzg5wksDE0GXFioKbhHJiUTC8dUne7h9ez2v2VYPwPu6O4mVRfjGM33BFhcyCm4RyYlDZyboGYnz/tfvuPSxmoood9/Yyo+ODJFIaLgkUwpuEcmJHx45T6TEePtNrVd8/O03tXFhao4XB8YDqix8FNwikhM/OjLIHV2N1MfKrvj4W29oIVJi/PRXQwFVFj4KbhHx3dDFWV4ZnOJtN159TkB9rIyb22t56vRoAJWFk4JbRHz3dE8ylO/Y2bTq57t3NPJi/zjzi4lclhVaCm4R8d0zp0eJlUW4ub121c93dzUwt5jg8FlNC8yEgltEfPfU6VFu395ANLJ65HTvaADg2d6xXJYVWgpuEfHVRHyBY4OT3LGzcc1rWmsraKst5/DZizmsLLwU3CLiqxcHxnEOXpvqVa/l5vY6jii4M6LgFhFfvZwat76lvW7d6/ZeW8uJ4SlmF5ZyUVaoKbhFxFeHz1xkW2MldbHoutfd3F7LUsLxyuBkjioLLwW3iPjq5bMTG/a2AfamZpwcPafhko0ouEXENxMzC/SOxLmlY+Pg7myIUV5awsnh6RxUFm4KbhHxTfpm41rzt5eLlBg7m6s4OTTld1mhp+AWEd8cSQ177M0guAF2t1ZzYljBvREFt4j45sTQJA2xKC3V5Rldv7ulmv7RuGaWbEDBLSK+eWVwij1tNZhZRtfvbqki4aB3RIcIr0fBLSK+cM5xfHCSPZs4CDh9aPBJDZesK6PgNrPfM7PDZvaymX3dzCr8LkxEwm1oco6Ls4tc31aT8WN2NSeD+4RuUK5rw+A2sw7g3wDdzrlbgAjwgN+FiUi4pRfSbKbHXVkWoaO+ktMXNCVwPZkOlZQClWZWCsSAs/6VJCKF4Phgste8ZxM9boBtjZX0jWqMez0bBrdz7gzw34A+4Bww4Zz7od+FiUi4HR+apD4Wpbm6bOOLl9nRWKXg3kAmQyUNwH3ATqAdqDKzD6xy3T4zO2hmB4eHh72vVERC5fjgFNe3Zj6jJG17U4zhyTlm5jUlcC2ZDJW8HTjtnBt2zi0A3wHeuPIi59x+51y3c667peXqc+VEpLgcH5pi9ybGt9O2NcYA6B9Tr3stmQR3H3CnmcUs+V/nPcBRf8sSkTAbm55nYmaB3S1Vm37s9lRway732jIZ434KeAR4DjiUesx+n+sSkRA7lZoV0tWUfXBrnHttpZlc5Jz7I+CPfK5FRApETyq4d2bR426IRakuL6Vfwb0mrZwUEc/1jExTYrCtIbbpx5oZ2xpj6nGvQ8EtIp47fWGazoYYZaXZRcx2zeVel4JbRDzXMzLNzubND5OktddXcm58Buech1UVDgW3iHjKOcfp4a0Fd0d9JdPzS1ycWfSwssKh4BYRTw1PzTE9v0RX0+bHt9Pa6ysBODM+41VZBUXBLSKe6rmQHJvu2uJQCcBZBfeqFNwi4qn0VMD0Fq3ZaK9P7hx9dkLBvRoFt4h46vTINNGIXQrfbDRXlVMWKdFQyRoU3CLiqZ4L02xrjFEayT5eSkqMa+srODs+62FlhUPBLSKe6hmJZ7XUfaX2ukqNca9BwS0innHOMTAaZ1tD5Zafq71ewb0WBbeIeGZiZoHJucVLW7NuRUd9BYMXZ1lYSnhQWWFRcIuIZ/pHkz3kziz2KFmpvb6ShIPBixrnXknBLSKeSR9+sK3Rm6ESQDcoV6HgFhHPpLdi9WKo5NJcbo1zX0XBLSKe6R+LU1cZpbYiuuXnaqtNBreGSq6m4BYRz/SPzlw6wWaraiqiVJVFOK/gvoqCW0Q80z8W92R8O62troKhi3OePV+hUHCLiCcSCcfA6ExWp96spa2mQj3uVSi4RcQTQ5NzzC8l6PRoqATgmroKzk8ouFdScIuIJy5NBfRg1WRaa205Q5OzOglnBQW3iHjCy6mAadfUVrCw5BidnvfsOQuBgltEPJFeNdlR712P+5rUlECNc19JwS0inugfi9NWW05FNOLZc7amglszS66k4BYRT/SPxj2dUQLJm5OgHvdKCm4R8cTA2Iyn49sArTXlgFZPrqTgFpEtW1hKcG5ixtMZJQDRSAnN1WUK7hUU3CKyZWfHZ0g4PJ3DndZWq7ncKym4RWTL+lJTAb3ap2S5ttoKBnVz8goKbhHZsvRUQK/HuCEd3OpxL6fgFpEt6x+LE43YpXnXXrqmtoKR6XnmF3WEWZqCW0S2rH80Tnt9JZES8/y522qTM0uGJtXrTlNwi8iW9Y95uyvgcm11OlBhpYyC28zqzewRM/uVmR01szf4XZiIhMfAqLf7cC93zaWTcHSDMq00w+u+APyjc+5+MysD/PmvVURCZ3pukZHpeU9Odl9N+ggzTQm8bMPgNrNa4M3ABwGcc/OAtuoSESC5YhL8mVEC0BCLUhYpYVBj3JdkMlSyCxgG/srMnjezL5tZlc91iUhIXNrO1eNVk2lmRktNuTaaWiaT4C4Fbgf+3Dl3GzANfGblRWa2z8wOmtnB4eFhj8sUkXzV58M+3Cu1pQ5UkKRMgnsAGHDOPZV6/xGSQX4F59x+51y3c667paXFyxpFJI/1j8WpjEZoqirz7TW0evJKGwa3c+480G9mN6Q+dA9wxNeqRCQ0+kdn2N4Yw8z7OdxpWj15pUxnlXwSOJCaUXIK+JB/JYlImAyM+TcVMK2lppzJ2UVm5peoLPPuoIawyii4nXMvAN0+1yIiIeOco380zp27mnx9nfSUwKHJWXY0aW6EVk6KSNbG4gtMzy/5emMSLi971zh3koJbRLLm91TAtNYaLXtfTsEtIlnrH/N/KiAs32hKPW5QcIvIFvi5D/dydZVRykpLGFKPG1Bwi8gW9I3GaYhFqS7PdIJadsyMttpyDZWkKLhFJGvJqYC52XOutUaLcNIU3CKStf7R3AW3lr1fpuAWkawsJRxnxv07QGGl1poKbTSVouAWkaycvzjLwpLzfdVkWmttOZNzi8TnF3PyevlMwS0iWUnP4d6eq6GS1Fxu9boV3CKSpcuLb3I1xq1FOGkKbhHJSv/YDGbQXp+boZJLy961CEfBLSLZ6R+Nc21tBWWluYmR1ktDJepxK7hFJCu5nAoIUFtZSnlpiZa9o+AWkSz153DxDaRXT+pABVBwi0gWZheWGLw4l7Mbk2la9p6k4BaRTRsYS24utb0pNzcm07QIJ0nBLSKbdmk71xz3uFtryzXGjYJbRLIwMJqbfbhXaqutYGpukam54l49qeAWkU3rG41TVlpCS3V5Tl+3tSZ1oEKRj3MruEVk0/pHZ9jWUElJieX0dS8fGlzcwyUKbhHZtFxPBUy7fGiwetwiIpvSNxrP+Y1JgNZabTQFCm4R2aSJ+AKTs4s52851uZryUiqiJepxB12AiIRLeipgrrZzXS69elJj3CIim9CXmgrYGcBQCST35VaPW0RkE/oDmsOd1qJFOApuEdmc/rE4dZVR6iqjgby+etwKbhHZpL7RmUBuTKa11ZYTn18q6tWTCm4R2ZTekWm6mqoCe30dYabgFpFNWFhKMDA2E2hwp5e9K7hFRDIwMDbDUsKxoymYG5NweRHOcBHfoFRwi0jGekamAehqDnKoRD1uBbeIZKz3QjK4g+xxV5eXEiuLMFjEy94zDm4zi5jZ82b2fT8LEpH81TMSp6oskvPtXJczM1privsIs830uD8FHPWrEBHJf70j0+xoqsIst9u5rtRa5MveMwpuM+sE/jnwZX/LEZF81jsSp6s5uGGStLbaiqI+TCHTHvfDwO8DibUuMLN9ZnbQzA4ODw97UpyI5I/FpQT9Y3F2BDgVMC05VDKHcy7oUgKxYXCb2buBIefcs+td55zb75zrds51t7S0eFagiOSHcxOzLCw5ugK8MZnWVlvOzMISk0W6ejKTHvddwL1m1gN8A7jbzP7O16pEJO+cvjSjJPged1uRH6iwYXA75/6jc67TOdcFPAD81Dn3Ad8rE5G80puew50Hwd1akw7u4hzn1jxuEclIz0icimjJpQUwQWpNL8KZLM7gLt3Mxc65x4HHfalERPJaenOpoKcCgoZK1OMWkYz0jMQDXTG5XHV5KVVFvHpSwS0iG1pKOPpG4nkxvp3WVltRtEMlCm4R2dC5iRnmlxJ5MaMkraWmXDcnRUTWcnI4OaNkd0v+BHcxn/au4BaRDZ0cmgJgd2t1wJVc1lab3GiqGFdPKrhFZEMnh6eoq4zSVFUWdCmXtNZUMLuQ4OJs8a2eVHCLyIZODk+xuyU/pgKmpedyF+M4t4JbRDZ0cnia3S35M0wCy+ZyF+E4t4JbRNY1MbPA8ORcXo1vQ3Gf9q7gFpF1nRpO3ZjMsx735dPe1eMWEblCPk4FBKgqL6W6vJShIlyEo+AWkXWdHJ4iGjG2N+bHcvflWmvLi3K/EgW3iKzrxNAUXU1VlEbyLy6K9dDg/PtOiEheSU4FzK/x7bRi3a9EwS0ia1pYStA3Emd3a36Nb6clDw0uvrMnFdwisqbekTiLCZe3Pe7WmnLmFhNcnCmu1ZMKbhFZ0/HBSQCuy7M53Gmt6bncRTZcouAWkTX96vwkZrCntSboUlbVdmkut4JbRASAY+cn6WqqorIsEnQpqyrWI8wU3CKypmODk9zQlp+9bbi80dR59bhFRGBmfomekWluuCZ/gztWVkp9LMrZ8ZmgS8kpBbeIrOr40CTOwY15HNwAnQ2VnFFwi4gkb0wCed3jBuior2RgTMEtIsKx85NUREvy6oDg1XTUxzgzNlNUi3AU3CKyqmPnJ9nTWkOkJH9OvVlNR0MlMwtLjMUXgi4lZxTcIrKqX52fzPvxbUiOcQOcKaLhEgW3iFzlwtQcF6bm8n58G5Jj3AADY/GAK8kdBbeIXOXQmQkAbumoC7iSjV3qcRfRzBIFt4hc5eWBZHDf3F4bcCUbq6uMUlUWKaqZJQpuEbnKoTMT7GquoqYiGnQpGzIzOhti6nGLSHF7+cxEKIZJ0joaimsut4JbRK4wMjXH2YlZXhWm4K6v5IxuTopIsQrTjcm0joZKLs4uMjlbHHO5NwxuM9tmZo+Z2VEzO2xmn8pFYSISjJdTwX1zR/7fmEwrtpklmfS4F4F/55y7CbgT+ISZ7fW3LBEJyqEzE+xsrqI2BDcm09JzuftHFdwAOOfOOeeeS709CRwFOvwuTESCcWggXDcmAbpS+6n0jkwHXElubGqM28y6gNuAp/woRkSCdW5ihrMTs9y+vT7oUjalPhaltqKUHgX3lcysGvg28Gnn3MVVPr/PzA6a2cHh4WEvaxSRHHmudxyA1+5oCLiSzTEzdjZX0TtSHDNLMgpuM4uSDO0DzrnvrHaNc26/c67bOdfd0tLiZY0ikiPP9o5RES3hpmvDc2MybUdTFacvqMcNgJkZ8JfAUefcn/pfkogE5dm+MW7trCcaCd9M4a7mKs6OzzC3uBR0Kb7L5LtzF/BbwN1m9kLqz7t8rktEcmx2YYnDZyZCN0yS1tUUI+GKY2ZJ6UYXOOeeAPJ7J3UR2bKXBiZYTLjwBnfz5Zkl17VWB1yNv8L3+5CI+OKZnlEAbtsezuDemZoSWAzj3ApuEQHgFycvcOM1NTRWlQVdSlbSUwKLYWaJgltEmF1Y4mDPGG/c3Rx0KVlLTwkshrncCm4R4fm+ceYWE7xxd1PQpWxJV3MVp4YV3CJSBJ48eYESgzt2NQZdypZc31bDmfGZgt8lUMEtIvzi5Aiv6qwP1cZSq7m+LXm48fGhqYAr8ZeCW6TITcws8EL/OHeFfJgE4IZ0cA9OBlyJvxTcIkXuZ8eHWUw47r6xNehStqyzoZLKaIRj59XjFpEC9tOjQ9THoqGdv71cSYmxp62aV9TjFpFCtZRwPHZsiLfd0EqkpDAWSF/fVqPgFpHC9UL/OGPxhYIYJkm7oa2Gock5xqbngy7FNwpukSL2wyPnKS0x3nx94WzFvKctuU9JIfe6FdwiRco5x/dfPMeb9jRTVxnuaYDLpfcSP3LuqvNeCoaCW6RIPd8/zpnxGd5za3vQpXiqrbaC1ppyDg1MBF2KbxTcIkXqey+epay0hF/b2xZ0KZ57dWcdLw6MB12GbxTcIkVocSnB3790jrtvaKUm5KslV/PqznpOXZgu2KXvCm6RIvTYsWGGJuf4F7d3BF2KL17dWYdzcOhMYQ6XKLhFitDXnuqlrbacewpoGuByr+6sB5Kn+hQiBbdIkRkYi/P4K8P8Rvc2SkN4KHAmGqvK6GqKcbBnLOhSfFGY3zURWdOBp/ow4Dfu2B50Kb66c1cTT58eYSnhgi7FcwpukSJycXaBv3uyl19/1bV01FcGXY6v7tzVxMXZRY4W4HxuBbdIEfnbJ3uZnFvkY2/ZHXQpvrtzV3Kb2l+eGgm4Eu8puEWKxNTcIl954jRvvaGFWzrqgi7Hd9fUVbCzuYonTyq4RSSk/uLxk4xMz/Ope/YEXUrOvGlPMz8/eYGZ+aWgS/GUglukCAyMxflfPzvFfa9pL4h9tzP1jr3XMLuQ4IkTF4IuxVMKbpEC55zjs48eAeD333ljwNXk1ut3NVJbUcoPDp8PuhRPKbhFCtx3njvDj48O8u/fcUPBzyRZKRop4Z6b2vjx0UHmFxNBl+MZBbdIATt9YZrPfu8wd3Q18q//2c6gywnEva9pZzy+wI+ODAZdimcU3CIF6uLsAr/zN89QWmJ8/n23FszRZJv15j0tdNRX8rWne4MuxTMKbpECFJ9fZN9XD9I7EufPHnwt2xpjQZcUmEiJ8f7Xb+fnJ0Y4MVQYp+IouEUKzER8gQ/+1TM8fXqUz7/vVt6wuynokgL3wOu2ESuL8IWfnAi6FE8ouEUKyLHzk9z3pSd4vm+Mhx+4jfteU5jbtm5WU3U5H7qri++9eJbn+sK/8ZSCW6QAzC4s8cWfHOc9X3yCqbklvvaRO7m3wI4k26qPvmU37XUV/IdHXiI+vxh0OVuSUXCb2TvN7JiZnTCzz/hdlIhkZiK+wFeeOM2b/+QxPv+jV/i1m9v4x0+/idd1NQZdWt6pqYjyX+5/NSeGp/i9b74Q6umBGwa3mUWALwG/DuwFftPM9vpdmIinDhyAri4oKUn+feBA0BVlbWAszree6efjB57ldQ/9mD/+/hG6mqr4+kfu5Evvv53m6nIADhw6QNfDXZT85xK6Hu7iwKHwttkrb9rTwh++ey8/ODzIb3/lac6OzwRdUlZKM7jmDuCEc+4UgJl9A7gPOOJnYSKeOXAA9u2DeDz5fm9v8n2ABx8Mrq5lEgnHzMIS8fklZuaXiC8sMjo1z+DkLEMX5zg3McvxoUmOnZ/iwtQcAM3V5bz/9du5/7Wd3Nxei9nl6X4HDh1g3/f2EV9Itrl3opd930u2+cFX5Uebg/Khu3ZSH4vymW8f4u7PP857X9PB229q45aOOlpqykMxbdKcW3+TcTO7H3inc+53Uu//FvB659zvrvWY7u5ud/DgwU0X8+4v/ozZhdV/fVmvznVbsM4n13tctq+33pfTrfPIdR+X5T7wedWGLF9vvUdm/HojI5BIXP25SAQarty3I+uvyxZ+PueXEsxt8Gt7VVmE61qrub6thr3ttbxxdzPXt1VfEdbLdT3cRe/E1fOWd9TtoOfTPeu+VrEYGIvzhR8f5+8PnSOe2oTKDBpiZZRFSiiNGKUlRqTE1vw6r9QYK+NbH31DVvWY2bPOue5Mrs2kx71axVf9LJrZPmAfwPbt2Z2scV1LNQtL6/yYr/O1W+/Lut4Xff3H5fb11m/fOs+ZdZ0+vF6WDfS1DX/2f1j5I2su9cSf+MQmXy+73th6D4tGSqiMRoiVJf9UlpVSGY3QWFVGa205rTXlVJeXbuq1+yb6NvXxYtTZEOO//stb+dx7b+HlMxMcPXeR4ck5RqbnWVhKsJhwLCUci+tl0go1FZlE6tZl8ioDwLZl73cCZ1de5JzbD+yHZI87m2IefuC2bB4msr5P/0NyeGSlHTvgvj/PfT05sL1u+6o97u11hX1cWTYqohG6uxrpDtEN3UxmlTwD7DGznWZWBjwAPOpvWSIeeughiK1YORiLJT9eoB665yFi0SvbHIvGeOiewm1zMdkwuJ1zi8DvAj8AjgLfcs4d9rswEc88+CDs35/sYZsl/96/P29uTPrhwVc9yP737GdH3Q4MY0fdDva/Z3/R35gsFBvenMxGtjcnRUSK1WZuTmrlpIhIyCi4RURCRsEtIhIyCm4RkZBRcIuIhIyCW0QkZBTcIiIho+AWEQkZXxbgmNkwkO2Rys3ABQ/LCQO1ufAVW3tBbd6sHc65lkwu9CW4t8LMDma6eqhQqM2Fr9jaC2qznzRUIiISMgpuEZGQycfg3h90AQFQmwtfsbUX1Gbf5N0Yt4iIrC8fe9wiIrKOwILbzN5pZsfM7ISZfWaVz5eb2TdTn3/KzLpyX6V3MmjvvzWzI2b2kpn9xMx2BFGnlzZq87Lr7jczZ2ahn4GQSZvN7H2p7/VhM/tarmv0WgY/29vN7DEzez718/2uIOr0ipl9xcyGzOzlNT5vZvY/Ul+Pl8zsds+LcM7l/A8QAU4Cu4Ay4EVg74prPg78RertB4BvBlFrDtv7NiCWevtjYW5vpm1OXVcD/BPwS6A76Lpz8H3eAzwPNKTebw267hy0eT/wsdTbe4GeoOveYpvfDNwOvLzG598F/D+S507fCTzldQ1B9bjvAE4450455+aBbwD3rbjmPuBvUm8/Atxj2R6xHbwN2+uce8w5F0+9+0uShzKHWSbfY4DPAX8CzOayOJ9k0uaPAF9yzo0BOOeGclyj1zJpswNqU2/Xscph42HinPsnYHSdS+4DvuqSfgnUm9m1XtYQVHB3AP3L3h9IfWzVa1zy3MsJoCkn1Xkvk/Yu92GS/2OH2YZtNrPbgG3Oue/nsjAfZfJ9vh643sx+bma/NLN35qw6f2TS5s8CHzCzAeAfgE/mprTAbPbf+6aVevlkm7Baz3nl9JZMrgmLjNtiZh8AuoG3+FqR/9Zts5mVAP8d+GCuCsqBTL7PpSSHS95K8reqn5nZLc65cZ9r80smbf5N4K+dc583szcAf5tqc8L/8gLhe3YF1eMeALYte7+Tq399unSNmZWS/BVrvV9P8lkm7cXM3g78AXCvc24uR7X5ZaM21wC3AI+bWQ/JscBHQ36DMtOf6+865xacc6eBYySDPKwyafOHgW8BOOeeBCpI7ulRqDL6974VQQX3M8AeM9tpZmUkbz4+uuKaR4HfTr19P/BTlxr5D6EN25saNvifJEM77OOesEGbnXMTzrlm51yXc66L5Lj+vc65g8GU64lMfq7/L8kb0ZhZM8mhk1M5rdJbmbS5D7gHwMxuIhncwzmtMrceBf5VanbJncCEc+6cp68Q4J3ZdwGvkLwj/Qepj/0xyX+8kPzm/m/gBPA0sCvou8k+t/fHwCDwQurPo0HX7HebV1z7OCGfVZLh99mAPwWOAIeAB4KuOQdt3gv8nOSMkxeAdwRd8xbb+3XgHLBAsnf9YeCjwEeXfY+/lPp6HPLj51orJ0VEQkYrJ0VEQkbBLSISMgpuEZGQUXCLiISMgltEJGQU3CIiIaPgFhEJGQW3iEjI/H9vtJLBzTJO/gAAAABJRU5ErkJggg==\n", "text/plain": [ "
" ] }, "metadata": { "needs_background": "light" }, "output_type": "display_data" }, { "name": "stdout", "output_type": "stream", "text": [ "Likelihood-Ratio: -13.70971993779133\n" ] }, { "data": { "image/png": "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\n", "text/plain": [ "
" ] }, "metadata": { "needs_background": "light" }, "output_type": "display_data" }, { "name": "stdout", "output_type": "stream", "text": [ "Likelihood-Ratio: -14.046192174412543\n" ] }, { "data": { "image/png": "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\n", "text/plain": [ "
" ] }, "metadata": { "needs_background": "light" }, "output_type": "display_data" }, { "name": "stdout", "output_type": "stream", "text": [ "Likelihood-Ratio: -13.535366550646552\n" ] }, { "data": { "image/png": "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\n", "text/plain": [ "
" ] }, "metadata": { "needs_background": "light" }, "output_type": "display_data" }, { "name": "stdout", "output_type": "stream", "text": [ "Likelihood-Ratio: -13.871838787267766\n" ] }, { "data": { "image/png": "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\n", "text/plain": [ "
" ] }, "metadata": { "needs_background": "light" }, "output_type": "display_data" }, { "name": "stdout", "output_type": "stream", "text": [ "Likelihood-Ratio: -14.208311023888978\n" ] }, { "data": { "image/png": "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\n", "text/plain": [ "
" ] }, "metadata": { "needs_background": "light" }, "output_type": "display_data" }, { "name": "stdout", "output_type": "stream", "text": [ "Likelihood-Ratio: -14.544783260510192\n" ] }, { "data": { "image/png": "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\n", "text/plain": [ "
" ] }, "metadata": { "needs_background": "light" }, "output_type": "display_data" }, { "name": "stdout", "output_type": "stream", "text": [ "Likelihood-Ratio: -14.881255497131404\n" ] }, { "data": { "image/png": "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\n", "text/plain": [ "
" ] }, "metadata": { "needs_background": "light" }, "output_type": "display_data" }, { "name": "stdout", "output_type": "stream", "text": [ "Likelihood-Ratio: -15.217727733752616\n" ] }, { "data": { "image/png": "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\n", "text/plain": [ "
" ] }, "metadata": { "needs_background": "light" }, "output_type": "display_data" }, { "name": "stdout", "output_type": "stream", "text": [ "Likelihood-Ratio: -15.55419997037383\n" ] }, { "data": { "image/png": "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\n", "text/plain": [ "
" ] }, "metadata": { "needs_background": "light" }, "output_type": "display_data" }, { "name": "stdout", "output_type": "stream", "text": [ "Likelihood-Ratio: -15.04337434660784\n" ] }, { "data": { "image/png": "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\n", "text/plain": [ "
" ] }, "metadata": { "needs_background": "light" }, "output_type": "display_data" }, { "name": "stdout", "output_type": "stream", "text": [ "Likelihood-Ratio: -15.379846583229051\n" ] }, { "data": { "image/png": "iVBORw0KGgoAAAANSUhEUgAAAXQAAAD8CAYAAABn919SAAAABHNCSVQICAgIfAhkiAAAAAlwSFlzAAALEgAACxIB0t1+/AAAADl0RVh0U29mdHdhcmUAbWF0cGxvdGxpYiB2ZXJzaW9uIDMuMC4zLCBodHRwOi8vbWF0cGxvdGxpYi5vcmcvnQurowAAHjdJREFUeJzt3Xt03Gd95/H3d2akGV1GsmxJlmzZkm1sJ44DJJiQC5eAC5tSIP2DpeEkFLpZvFzaLbvs6dLD4ZSlm7M9PVs23T0U8GlZSmtum3IJLNsCgZQCScBJnDi2Y8exZVmyJEvW/X579o+ZcRRZGs3lN/Oby+d1jo81M7+Z+T4a+eNHz+95np855xARkeIX8LsAERHxhgJdRKREKNBFREqEAl1EpEQo0EVESoQCXUSkRCjQRURKxLqBbmZfMrPLZvbcsvs2mtmPzOyF+N8NuS1TRETWk0oP/cvAXSvu+wTwiHNuN/BI/LaIiPjIUlkpamYdwPedc/vjt08Ddzrnes2sFXjUObd3vddpbGx0HR0dWRUsIlJunnzyyUHnXNN6x4UyfP3NzrlegHioN6fypI6ODo4ePZrhW4qIlCczu5DKcTk/KWpmh8zsqJkdHRgYyPXbiYiUrUwDvT8+1EL878trHeicO+ycO+CcO9DUtO5vDCIikqFMA/1h4P3xr98PfNebckREJFOpTFv8GvAYsNfMus3sfuDPgLea2QvAW+O3RUTER+ueFHXOvXeNhw56XIuIiGRBK0VFREqEAl1EpEQo0EXEN6NT83zlsU7++YymNHsh04VFIiJZmZhd4D1ffIzT/eMAfObuG/jd2zr8LarIqYcuIr74wqMvcrp/nC/c9xru3NvEf/vB8/SPzfhdVlFToItI3k3PLfK3v+zkt25s5a79LXz6nTcws7DIkcdTWuEua1Cgi0je/dOJPsZnF7j31u0AdDTW8IbdTTz0ZDepbBgoq1Ogi0jefevpHtoaqrh1x6ar973jla1cGp3hVO+4j5UVNwW6iOTV9Nwij5+7wr+6oYVAwK7ef+ee2F5PPz295tZQsg4Fuojk1ePnrzC3sMQb97x8s77mugj7t9bxqAI9Ywp0Ecmrn50ZIBwK8LodG6957I5djTxzcZTZhUUfKit+CnQRyaujncPcvL2BSEXwmsdu2r6BucUlTlwa86Gy4qdAF5G8mZ5b5FTvGDe3b1j18Zu2x643/3TXSD7LKhkKdBHJm+cujbKw5Hj1toZVH99cF2Hrhiqe6hrOc2WlQYEuInlzLN7zfvW21XvoAK9sq+ekhlwyokAXkbx5+uIw2zZW0RQNr3nMdS11dF6ZZGpuIY+VlQYFuojkzTMXR3lV29q9c4DrWqM4B2f6J/JUVelQoItIXoxOz9MzMs2+LXVJj7u+Jfb4870adkmXAl1E8uJMfJvc61qiSY9ra6iiujLIKQV62hToIpIXp/tigb63JXkPPRAwdm+OcnZAQy7pUqCLSF6c7hsnGg6xpT6y7rE7G2s4PzCZh6pKiwJdRPLidN84e1qimNm6x+5srOHS6AzTc9oCIB0KdBHJOeccp/vH2bvO+HnCjqYaADqvqJeeDgW6iORc/9gso9Pz654QTdjRGAv0cxp2SYsCXURy7vm+2IyVPZvTC/Tzgzoxmg4Fuojk3NnLsWDem2KgV1eGaK2PcG5QPfR0KNBFJOfODU6yobqChprKlJ+zo7FGQy5pUqCLSM51Dk5eHUZJVUdjDRd0UjQtCnQRybnzg5Ps2JReoG9rqGZ4ap6JWW3SlSoFuojk1PTcIr2jM2n30NsaqgDoGZ7ORVklSYEuIjmVmEvekWGgdw9PeV5TqVKgi0hOdcZnqqTfQ68GoFs99JRlFehm9h/M7ISZPWdmXzOz9TdpEJGykph6mG4PvbG2knAooB56GjIOdDPbCvx74IBzbj8QBO7xqjARKQ2dg5M0R8PUhkNpPc/MaGuoUg89DdkOuYSAKjMLAdXApexLEpFScn5wMu3eeUJbQ7UCPQ0ZB7pzrgf470AX0AuMOud+uPI4MztkZkfN7OjAwEDmlYpIUeq8MsnOjAO9SkMuachmyKUBuBvYAWwBaszsvpXHOecOO+cOOOcONDU1ZV6piBSdsZl5Bifmsuqhay566rIZcvkN4LxzbsA5Nw98C7jdm7JEpBQkZrh0pLmoKEFTF9OTTaB3AbeaWbXFdqw/CJzypiwRKQUXh2Lj3+2bqjN6/tVAH9I4eiqyGUN/AngIeAo4Hn+twx7VJSIloGck1rPeGg/mdG1VDz0t6c0jWsE59yfAn3hUi4iUmO7haeoiIeoiFRk9v7EmTEXQ6Bub9biy0qSVoiKSM93D01dXfGYiEDA210XoG9WQSyoU6CKSMz3D0xkPtyS01EXoG5vxqKLSpkAXkZxwztE9PHX1xGamWuoj9I0q0FOhQBeRnBidnmdybpGtG7IL9Nb6CL2jMzjnPKqsdCnQRSQnEkv2sxlDB9hcF2F2YYnR6XkvyippCnQRyYnEVMNsh1xa62PP79Wwy7oU6CKSEy/10LMfQwc0jp4CBbqI5ET38DS14RD1VZnNQU9oTQS6ZrqsS4EuIjnRMzLN1g1VxHYGyVxTNIyZhlxSoUAXkZyILSrKbrgFoCIYoKk2rMVFKVCgi0hO9HgwBz2htT6i5f8pUKCLiOdGp+cZm1nIepVogpb/p0aBLiKe6/FoDnpCYnGRJKdAFxHP9YzEAj3bVaIJLfVVjM8sMKkrFyWlQBcRz3m1qChBUxdTo0AXEc/1DE8TqQiwsabSk9fbXBcL9H4NuySlQBcRzyX2Qc92DnpCc10YgMvjmumSjAJdRDzXPeLdlEWA5mgi0NVDT0aBLiKe6xme9uyEKEBtOERVRZDLmouelAJdRDw1ObvA8NS8Z1MWAcyM5rqwhlzWoUAXEU9dnbLo4ZALxIZdNOSSnAJdRDzl9ZTFhOZoRD30dSjQRcRTXu2DvlJTNMyAxtCTUqCLiKd6hqepDAVorAl7+rrNdWHGZxeYnlv09HVLiQJdRDzVPTxN24YqAgFv5qAnNEdji4s0jr42BbqIeKp7ZNrzE6KwfC66hl3WokAXEU95uQ/6cldXi2ocfU0KdBHxzPTcIoMTc54uKkrQkMv6FOgi4pnEHHQvFxUlNFRXUBE0DbkkoUAXEc/kag46xFaLNtWGNeSShAJdRDyTq1WiCU11EQ25JJFVoJvZBjN7yMyeN7NTZnabV4WJSPHpHp6mImhXx7u91hwNM6AhlzVl20P/S+AfnXPXAa8CTmVfkogUq+7haVrrqwh6PAc9IbafiwJ9LaFMn2hmdcAbgQ8AOOfmgDlvyhKRYpSrKYsJzdEIQ5NzzC0sURnSiPFK2XxHdgIDwP82s6fN7K/NrMajukSkCMWuVJTDQI/PRR+cUC99NdkEegi4Gfi8c+4mYBL4xMqDzOyQmR01s6MDAwNZvJ2IFLLZhUUuj8+ydYP3UxYTtFo0uWwCvRvods49Eb/9ELGAfxnn3GHn3AHn3IGmpqYs3k5ECtmlkdjsk1wPuQBcHtNMl9VkHOjOuT7gopntjd91EDjpSVUiUnQSc9BzNWURdLHo9WR8UjTuD4AjZlYJnAN+L/uSRKQY9eRoH/TlNtVUYqZAX0tWge6cOwYc8KgWESli3cPTBANGS11u5qADhIIBNtWEGdDiolVp3o+IeKJnZJqWugihYG5jpTmq5f9rUaCLiCe6czwHPaG5TouL1qJAFxFPdA/n5sIWKzXVhrWfyxoU6CKStbmFJfrHZnKybe5KzXVhBifmWFxyOX+vYqNAF5Gs9Y3OsOSgLQcXtlipORphcckxNKmdRlZSoItI1rpHcrcP+kqJ1aLadfFaCnQRyVr3cO6uVLTSS4uLNI6+kgJdRLLWPTyNGbTU524OesJL1xZVD30lBbqIZK1neJrN0UhetrRt0pDLmhToIpK17uEptm3M/fg5QKQiSDQS0gZdq1Cgi0jWYvug5378PEFXLlqdAl1EsjK/uETvaG4vbLFSczSiIZdVKNBFJCuJOejb8tlD1/L/VSnQRSQrF4fzNwc9ITbkMoNzWi26nAJdRLKSzznoCU3RMDPzS4zPLuTtPYuBAl1EstI9NEXAoHVD7uegJ7x0KToNuyynQBeRrHQPT9NaX0VFjvdBX07L/1enQBeRrORr29zltPx/dQp0EclKvi5ssVxTfMhFPfSXU6CLSMbmFpbozdM+6MvVRUJUhgKauriCAl1EMtY7Oo1z+Z2yCGBm8WuLashlOQW6iGQsMWUxn4uKErT8/1oKdBHJWLcPi4oStPz/Wgp0EcnYxaFpggGjNQ/7oK+k5f/XUqCLSMa6h6doqYsQyuMc9ITmaJjR6Xlm5hfz/t6FSoEuIhnrHp7O2z7oK+lCF9dSoItIxvK9D/pyuhTdtRToIpKR2YVF+sdnfDkhCst76Jq6mKBAF5GMXBqZic9B96mHXqchl5UU6CKSkcSUxW0+9dA31YQJmIZcllOgi0hGLg7FFhXle2OuhGDAaKwNawvdZRToIpKRC0OTVAYDtNb7E+gQG0fXjosvyTrQzSxoZk+b2fe9KEhEisOFwSnaNlYRDJhvNWj5/8t50UP/Q+CUB68jIkXkwtAUHZtqfK2hORpRoC+TVaCbWRvwW8Bfe1OOiBQD5xxdVybZvtGfGS4JzXVhrkzMsriki0VD9j30B4E/ApY8qEVEisTgxByTc4u0b/I50KNhlhxcmVQvHbIIdDN7B3DZOffkOscdMrOjZnZ0YGAg07cTkQLSNTQJ4PuQS5MuFv0y2fTQ7wDeZWadwNeBt5jZ3688yDl32Dl3wDl3oKmpKYu3E5FC0TkYm4O+3eceuvZzebmMA90598fOuTbnXAdwD/AT59x9nlUmIgXrwtAUAfNnH/TlmqO6WPRymocuImnrujJJa30V4VDQ1zoSPXQNucSEvHgR59yjwKNevJaIFL7OK1O+nxAFiFQEqa+qYGBCgQ7qoYtIBrqGpmj3+YRoQuxi0Qp0UKCLSJrGZuYZmpwriB46JC5FpzF0UKCLSJq6rsRmuLT7vKgooalWy/8TFOgikpYLiUAvkCGXzXWx5f/OabWoAl1E0nIhvqjI7znoCS31EeYWlhiemve7FN8p0EUkLecGJmmOhqkNezJJLmut9bHVor2j0z5X4j8Fuoik5dzABDubCmO4BaAlvh9736hOjCrQRSQt5wYn2dlU63cZV73UQ1egK9BFJGVDk3OMTM2zs7FweuiNtWGCAVMPHQW6iKThxYEJAHYVUA89GDA2R8PqoaNAF5E0nCvAQIfYTJe+MZ0UVaCLSMrODUxSGQqw1eddFldqra9SDx0Fuoik4cWBSTo2Vft6YejVtNRH6BudKfvFRQp0EUnZuYEJdjYW1nALxGa6TM0tMjaz4HcpvlKgi0hK5heX6BqaYldz4cxwSWiJT10s95kuCnQRSUnX0BQLS65ge+ig1aIKdBFJybmB2B4uhbRKNEGrRWMU6CKSkjP94wC8ornweujN0TBmWi2qQBeRlJzuG2frhiqikQq/S7lGRTBAU21YPXS/CxCR4nCmf5y9LVG/y1hTa32E3jEFuohIUvOLS7w4MFHggV7FpRGdFBURSer84CTzi469mws30NsaqugenirrxUUKdBFZ1+m+2AnRPQUc6FsbqpiZX2Jocs7vUnyjQBeRdZ3pHycYsIJcVJTQ1hC7JF73cPkOuyjQRWRdp/vG2dFYQzgU9LuUNbXFNwxToIuIJHG6f7ygx8+BqztAdg9P+VyJfxToIpLU1NwCXUNTBT3DBaAuUkFdJERPGc90UaCLSFJn+idwrrBPiCa0NVRryEVEZC3P9YwCsH9rnc+VrC8xdbFcKdBFJKkTl0ZpqK5g64bCukrRarY2VNE9PF22c9EV6CKS1PGeUfZvrcessK5StJq2hmqm5hYZnpr3uxRfKNBFZE1zC0uc7hvnhi31fpeSksTUxZ4yHUfPONDNbJuZ/dTMTpnZCTP7Qy8LExH/nekfZ37RFcX4OcC2+OKirqHyHEcPZfHcBeDjzrmnzCwKPGlmP3LOnfSoNhHx2YlLsROiN24tjh56+6ZYoHdemfS5En9k3EN3zvU6556Kfz0OnAK2elWYiPjveM8o0UiI7Rur/S4lJTXhEM3RMOcHFegZM7MO4CbgiVUeO2RmR83s6MDAgBdvJyJ58szFUfZvKY4Togkdm2q4oB56ZsysFvgH4GPOubGVjzvnDjvnDjjnDjQ1NWX7diKSJ9Nzi5zqHeM17Q1+l5KWjsZqzg+W5xh6VoFuZhXEwvyIc+5b3pQkIoXg2e4RFpYcN7dv8LuUtHQ01jA4Mcv4TPlNXcxmlosBfwOccs591ruSRKQQPNk1DMBN24qrh75jU2yL3wtXyq+Xnk0P/Q7gfcBbzOxY/M/bPapLRHz21IURdjbW0FBT6XcpaelojAV6OZ4YzXjaonPu50DxnCkRkZQ553i6a5g79zb7XUraOuI99M4yDHStFBWRa3QNTXFlcq7oxs8BqiqDtNRF6NSQi4gIPHFuCIDXdmz0uZLMxGa6TPhdRt4p0EXkGr98cZDG2jC7m2v9LiUju5pqOXt5oux2XVSgi8jLOOf45YtXuG3XpqJaULTc3pYoYzMLXB6f9buUvFKgi8jLvDgwyeXxWW7ftcnvUjK2uzl2daXTfeM+V5JfCnQReZnHXhwEKOpA37M5NlR0pl+BLiJl7Bdnr7ClPlI0G3KtZlNtmMbaSgW6iJSvuYUlfn52kDfsbira8fOE3c1RTveX10wXBbqIXPWr80NMzC5w8PriW1C00t6WKGf7x1laKp+ZLgp0Ebnqkef7qQwFeP3uRr9Lydp1LVEm5xa5UEZXL1KgiwgQm674yKnL3LFrE9WV2VzMrDDsj19l6XjPqM+V5I8CXUQAOHt5gq6hKQ5ev9nvUjyxZ3OUylCA5xToIlJuvv9sL2bw1n2lEeiVoQDXt0R5tnvE71LyRoEuIjjnePiZS9y2cxOb6yJ+l+OZ/VvrOdEzVjYnRhXoIsKz3aOcH5zk7ldv8bsUT72yrZ7x2QU6y+Qaowp0EeG7xy5RGQxw1w2tfpfiqVdti23/+1RXeQy7KNBFytzM/CLfOdbDm69ror66wu9yPLWnOcqG6gp+df6K36XkhQJdpMz94HgvQ5Nz3Hdru9+leC4QMF7bsZEnzg/5XUpeKNBFytxXHrvAzqYa7thV/IuJVvO6HRu5cGWKvtEZv0vJOQW6SBl75uIIxy6O8L5b2wkEinvvlrW8bkds18gnymDYRYEuUsb+109eoC4S4t2vafO7lJy5vjVKfVUFPzsz6HcpOadAFylTz3aP8ONTl/ngG3YSjZTWydDlQsEAb7mumZ88389iic9HV6CLlKnP/ugMG6or+MAdHX6XknMHr29meGqep7qG/S4lpxToImXoRyf7efT0AB+5c1dJ984T3riniYqg8aOT/X6XklMKdJEyMzW3wKcfPsHezVF+744dfpeTF3WRCl7/ika+98ylkh52UaCLlJn/+n9P0TMyzZ/+9n4qguUTAe85sI3e0Rl+9sKA36XkTPl8miLCd4/18NUnuvjQm3Zxy46NfpeTVwev38ymmkq+8auLfpeSMwp0kTJxtHOI//wPz3KgvYGPv22P3+XkXWUowLsPtPHDk32cGyjNa40q0EXKwLPdI/ybL/+aLfVVfOF9rymroZblPviGnYRDQf7ykRf8LiUnyvNTFSkjPzzRx+988XGikQq+cv8tNNaG/S7JN421Yd5/ewcPP3OJYxdLbwdGBbpIiZqYXeBT33mOQ3/3JLs31/Ltj95OW0O132X57iNv3kVLXYSPf/MYk7MLfpfjqawC3czuMrPTZnbWzD7hVVEikrnJ2QW+9PPzvOnPf8rfP3GB+1+/g2/+u9tojpbOlYiyURep4C/+9avovDLFR7/6FDPzi36X5JmMA93MgsDngN8E9gHvNbN9XhUmkhdHjkBHBwQCsb+PHPG7ooyMzczzj8/18UcPPcMtD/yYz3z/JHtbonzrw7fzqXfsI1IRvHrskeNH6Hiwg8B/CdDxYAdHjhdnm7Nx+ysaeeC39/PPZwb4ncOPc/byuN8leSKUxXNvAc46584BmNnXgbuBk14UJpJzR47AoUMwNRW7feFC7DbAvff6V9cyzjmm5xeZmltkanaR8dl5Lo/N0j82Q9/YDC9cnuD53jHOD06y5KA2HOLtN7Zyzy3beE37tdMSjxw/wqHvHWJqPtbmC6MXOPS9WJvvvbEw2pwv99yynYaaSv7TN5/hrgf/hbfdsJm37WvhxrZ6ttRXUVUZXP9FCow5l9mqKTN7N3CXc+7fxm+/D3idc+7313rOgQMH3NGjR9N+r09++/iaG9Qnqz9py5I8mOx5mb5fsm+zS/LMpM/LcMFbQbUhw/dL9sz1vi9XH75yBZaWrn0sGISGhjTecb3PKbPv9+JSLMyTvfa2jVVc31LH9a113L5rEze3NySdwdLxYAcXRi9cc397fTudH+tMUk3pGpyY5fOPvsi3n+5haHLu6v3RcIhwRZDKoFERChAMGOttMGy29hFfev9r2b4ps3MYZvakc+7Aesdl00NfrfJrfvTM7BBwCGD79u0ZvdGWDVXs3RxNr5L1H0r6zU/+vPy+X/L2JXnNjOvMwftl2MBM27D+cw3+6tusFqkG8NGPpv2eyT7fZNZ6WtCM6nCI6sogNZVBqipD1IaDNEUjbK4L0xyNUBlKb9S0a7QrrfvLQWNtmE+9Yx9//JvXcaZ/gpO9Y/SPzTAwPsvswhLzi7E/C+ttGbDOw+l+VpnIJtC7gW3LbrcBl1Ye5Jw7DByGWA89kzf66JtfkcnTRJL72A9iwywrtbfD3Z/Pfz15sL1++6o99O31mXW2SkkoGGDfljr2banzu5SMZfNfxq+B3Wa2w8wqgXuAh70pSyQPHngAqlf8ClxdHbu/RD1w8AGqK17e5uqKah44WLptLicZB7pzbgH4feCfgFPAN51zJ7wqTCTn7r0XDh+O9cjNYn8fPlwwJ0Rz4d4b7+XwOw/TXt+OYbTXt3P4nYfL7oRoqcr4pGgmMj0pKiJSzlI9KaqVoiIiJUKBLiJSIhToIiIlQoEuIlIiFOgiIiVCgS4iUiIU6CIiJUKBLiJSIvK6sMjMBoBVNs9ISSMw6GE5xUBtLg9qc+nLtr3tzrmm9Q7Ka6Bnw8yOprJSqpSozeVBbS59+WqvhlxEREqEAl1EpEQUU6Af9rsAH6jN5UFtLn15aW/RjKGLiEhyxdRDFxGRJAou0M3sLjM7bWZnzewTqzweNrNvxB9/wsw68l+lt1Jo8380s5Nm9qyZPWJm7X7U6aX12rzsuHebmTOzop4RkUp7zew98c/5hJl9Nd81ei2Fn+vtZvZTM3s6/rP9dj/q9JKZfcnMLpvZc2s8bmb2P+Pfk2fN7GZPC3DOFcwfIAi8COwEKoFngH0rjvkI8IX41/cA3/C77jy0+c1AdfzrD5dDm+PHRYGfAY8DB/yuO8ef8W7gaaAhfrvZ77rz0ObDwIfjX+8DOv2u24N2vxG4GXhujcffDvw/YtcbvxV4wsv3L7Qe+i3AWefcOefcHPB14O4Vx9wN/G3864eAg5bp5dYLw7ptds791Dk3Fb/5OLELchezVD5ngD8F/hyYyWdxOZBKez8IfM45NwzgnLuc5xq9lkqbHZC4InM9q1xkvtg4534GDCU55G7gKy7mcWCDmbV69f6FFuhbgYvLbnfH71v1GBe7rukosCkv1eVGKm1e7n5i/8MXs3XbbGY3Aducc9/PZ2E5kspnvAfYY2a/MLPHzeyuvFWXG6m0+dPAfWbWDfwA+IP8lOardP+9pyXk1Qt5ZLWe9sppOKkcU0xSbo+Z3QccAN6U04pyL2mbzSwA/A/gA/kqKMdS+YxDxIZd7iT2G9i/mNl+59xIjmvLlVTa/F7gy865vzCz24C/i7d5Kffl+San+VVoPfRuYNuy221c+2vY1WPMLETsV7Vkv+IUulTajJn9BvBJ4F3Oudk81ZYr67U5CuwHHjWzTmJjjQ8X8YnRVH+uv+ucm3fOnQdOEwv4YpVKm+8HvgngnHsMiBDb86SUpfTvPVOFFui/Bnab2Q4zqyR20vPhFcc8DLw//vW7gZ+4+NmGIrVum+PDD18kFubFPrYK67TZOTfqnGt0znU45zqInTd4l3PuqD/lZi2Vn+vvEDv5jZk1EhuCOZfXKr2VSpu7gIMAZnY9sUAfyGuV+fcw8Lvx2S63AqPOuV7PXt3vs8JrnAU+Q+wM+Sfj932G2D9oiH3o/wc4C/wK2Ol3zXlo84+BfuBY/M/Dftec6zavOPZRiniWS4qfsQGfBU4Cx4F7/K45D23eB/yC2AyYY8Db/K7ZgzZ/DegF5on1xu8HPgR8aNnn/Ln49+S41z/XWikqIlIiCm3IRUREMqRAFxEpEQp0EZESoUAXESkRCnQRkRKhQBcRKREKdBGREqFAFxEpEf8fjJbeA7bm1igAAAAASUVORK5CYII=\n", "text/plain": [ "
" ] }, "metadata": { "needs_background": "light" }, "output_type": "display_data" }, { "name": "stdout", "output_type": "stream", "text": [ "Likelihood-Ratio: -15.716318819850265\n" ] }, { "data": { "image/png": "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\n", "text/plain": [ "
" ] }, "metadata": { "needs_background": "light" }, "output_type": "display_data" }, { "name": "stdout", "output_type": "stream", "text": [ "Likelihood-Ratio: -16.052791056471477\n" ] }, { "data": { "image/png": "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\n", "text/plain": [ "
" ] }, "metadata": { "needs_background": "light" }, "output_type": "display_data" }, { "name": "stdout", "output_type": "stream", "text": [ "Likelihood-Ratio: -16.38926329309269\n" ] }, { "data": { "image/png": "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\n", "text/plain": [ "
" ] }, "metadata": { "needs_background": "light" }, "output_type": "display_data" }, { "name": "stdout", "output_type": "stream", "text": [ "Likelihood-Ratio: -16.725735529713905\n" ] }, { "data": { "image/png": "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\n", "text/plain": [ "
" ] }, "metadata": { "needs_background": "light" }, "output_type": "display_data" }, { "name": "stdout", "output_type": "stream", "text": [ "Likelihood-Ratio: -17.062207766335117\n" ] }, { "data": { "image/png": "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\n", "text/plain": [ "
" ] }, "metadata": { "needs_background": "light" }, "output_type": "display_data" }, { "name": "stdout", "output_type": "stream", "text": [ "Likelihood-Ratio: -17.39868000295633\n" ] }, { "data": { "image/png": "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\n", "text/plain": [ "
" ] }, "metadata": { "needs_background": "light" }, "output_type": "display_data" }, { "name": "stdout", "output_type": "stream", "text": [ "Likelihood-Ratio: -17.73515223957754\n" ] }, { "data": { "image/png": "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\n", "text/plain": [ "
" ] }, "metadata": { "needs_background": "light" }, "output_type": "display_data" }, { "name": "stdout", "output_type": "stream", "text": [ "Likelihood-Ratio: -17.224326615811552\n" ] }, { "data": { "image/png": "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\n", "text/plain": [ "
" ] }, "metadata": { "needs_background": "light" }, "output_type": "display_data" }, { "name": "stdout", "output_type": "stream", "text": [ "Likelihood-Ratio: -17.560798852432765\n" ] }, { "data": { "image/png": "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\n", "text/plain": [ "
" ] }, "metadata": { "needs_background": "light" }, "output_type": "display_data" }, { "name": "stdout", "output_type": "stream", "text": [ "Likelihood-Ratio: -17.049973228666776\n" ] }, { "data": { "image/png": "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\n", "text/plain": [ "
" ] }, "metadata": { "needs_background": "light" }, "output_type": "display_data" }, { "name": "stdout", "output_type": "stream", "text": [ "Likelihood-Ratio: -17.386445465287988\n" ] }, { "data": { "image/png": "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\n", "text/plain": [ "
" ] }, "metadata": { "needs_background": "light" }, "output_type": "display_data" }, { "name": "stdout", "output_type": "stream", "text": [ "Likelihood-Ratio: -17.7229177019092\n" ] }, { "data": { "image/png": "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\n", "text/plain": [ "
" ] }, "metadata": { "needs_background": "light" }, "output_type": "display_data" }, { "name": "stdout", "output_type": "stream", "text": [ "Likelihood-Ratio: -18.059389938530412\n" ] }, { "data": { "image/png": "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\n", "text/plain": [ "
" ] }, "metadata": { "needs_background": "light" }, "output_type": "display_data" }, { "name": "stdout", "output_type": "stream", "text": [ "Likelihood-Ratio: -18.395862175151624\n" ] }, { "data": { "image/png": "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\n", "text/plain": [ "
" ] }, "metadata": { "needs_background": "light" }, "output_type": "display_data" } ], "source": [ "p0 = 0.5\n", "p1 = 0.7\n", "\n", "for i in range(k):\n", " p = numpy.linspace(0,1,1000)\n", " l = p**y[i]*(1-p)**(i+1-y[i])\n", " l = l / numpy.mean(l)\n", " plt.plot(p,l)\n", " plt.scatter(numpy.array([p0]),numpy.array([0]),color='r')\n", " plt.scatter(numpy.array([p1]),numpy.array([0]),color='g')\n", " print('Likelihood-Ratio: ',numpy.log(p0**y[i]*(1-p0)**(i+1-y[i])/(p1**y[i]*(1-p1)**(i+1-y[i]))))\n", " plt.show()" ] }, { "cell_type": "code", "execution_count": 126, "metadata": {}, "outputs": [ { "data": { "image/png": "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\n", "text/plain": [ "
" ] }, "metadata": { "needs_background": "light" }, "output_type": "display_data" }, { "data": { "image/png": "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\n", "text/plain": [ "
" ] }, "metadata": { "needs_background": "light" }, "output_type": "display_data" }, { "data": { "image/png": "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\n", "text/plain": [ "
" ] }, "metadata": { "needs_background": "light" }, "output_type": "display_data" }, { "data": { "image/png": "iVBORw0KGgoAAAANSUhEUgAAAYAAAAD8CAYAAAB+UHOxAAAABHNCSVQICAgIfAhkiAAAAAlwSFlzAAALEgAACxIB0t1+/AAAADl0RVh0U29mdHdhcmUAbWF0cGxvdGxpYiB2ZXJzaW9uIDMuMC4zLCBodHRwOi8vbWF0cGxvdGxpYi5vcmcvnQurowAAIABJREFUeJzt3XeYlNX5xvHvw9JEVIysBimi0RiXpQ8dEQXBFhBRA0FFgyKWKBIlxC5qInaxIyqoIFYiAVTERiwgs0qVqISfBUVdG1hR9Pn9cSbJZrMLs8zsvDM79+e65mLKYc7ji7P3nnnfc465OyIikn9qRV2AiIhEQwEgIpKnFAAiInlKASAikqcUACIieUoBICKSpxQAIiJ5SgEgIpKnFAAiInmqdtQFbE7jxo29ZcuWUZchIpIzSkpKPnH3wmTaZnUAtGzZkng8HnUZIiI5w8zeSbatvgISEclTCgARkTylABARyVMKABGRPKUAEBHJUwoAEZE8pQAQEclTKQeAmTU3s2fNbJWZrTSzMytoY2Y20cxWm9kyM+uQar+bc+mlsHhxdfYgIpL70jEC2AT8wd33AboCp5lZUbk2BwN7JW4jgVvT0G+FPv8cbr8dunaFcePg22+rqycRkdyWcgC4+zp3fzVx/0tgFdC0XLOBwD0eLAQamVmTVPuuyI47wooV8LvfwYQJ0K4dvPhidfQkIpLb0noOwMxaAu2BReVeagq8V+bxWv43JNKmUSO44w6YNw82boR994Uzz4Svv66uHkVEck/aAsDMGgKPAKPdfUP5lyv4K17J+4w0s7iZxUtLS1Oq6cADw2jgtNNg4kRo3RqeeSaltxQRqTHSEgBmVofww3+auz9aQZO1QPMyj5sBH1T0Xu4+yd1j7h4rLExqQbvNatgQbrwRFiyAggLo0wdOPhnWr0/5rUVEclo6rgIy4E5glbtfW0mzWcBxiauBugLr3X1dqn1Xxb77wrJlcM45MHkyFBfD3LmZrEBEJLukYwTQAzgWOMDMliRuh5jZKDMblWgzF1gDrAbuAE5NQ79Vts02cOWV8PLLsMMOcOihMHw4fPZZFNWIiETL3Cv8Kj4rxGIxr679ADZuhMsvh7/8BXbaCW65BY44olq6EhHJGDMrcfdYMm3zdiZwvXowfnyYMLbrrjB4MBx9NHz8cdSViYhkRt4GwL+0aweLFoXRwGOPQVERTJ8OWTwwEhFJi7wPAIA6deDcc+G112DPPWHYMBg4EN5/P+rKRESqjwKgjKKiMGv4mmtg/nxo1QruukujARGpmRQA5RQUwJgx4ZLRdu1gxAjo3x/eSXqbZRGR3KAAqMSee4ZZw7fcEi4bLS6Gm2+Gn36KujIRkfRQAGxGrVpwyilhOYnu3eH006F3b3jrragrExFJnQIgCbvtBk88Ec4HLFsGbdqE8wQ//hh1ZSIiW08BkCQzOOEEeP31sMjc2WdDjx7hsYhILlIAVNGuu4b5AtOnw+rV0L59mEPwww9RVyYiUjUKgK1gBkOHht/+Bw2C88+Hzp1hyZKoKxMRSZ4CIAU77wwzZsCjj8K6ddCpE1xwQVhnSEQk2ykA0mDQoDAaGDYMLrsMOnQIy0uIiGQzBUCa/OxnMGVK2GNgw4Zw2ejZZ8M330RdmYhIxRQAaXbwwbByJZx0UrhUtG3bsBuZiEi2UQBUg+23h9tuCzOJf/oJ9tsvTCL78suoKxMR+Q8FQDXaf/8wcWz06LCkROvW8NRTUVclIhIoAKrZttvCddfBCy9A/frQrx+ceCJ88UXUlYlIvlMAZEj37mGewLhx4WRxq1bwt79FXZWI5DMFQAbVrx/2IF64MOxDPGBAuHT0k0+irkxE8lFaAsDM7jKzj81sRSWv9zaz9Wa2JHG7MB395qpYDOJxuPhiePDBsBHNQw9FXZWI5Jt0jQCmAAdtoc3f3b1d4jY+Tf3mrLp14aKLoKQEWrQIG9IPHgwffhh1ZSKSL9ISAO6+APgsHe+Vb9q0CV8JTZgAc+aE0cC992obShGpfpk8B9DNzJaa2eNm1iqD/Wa92rVh7FhYuhT22QeOOw4OOwzWro26MhGpyTIVAK8Cu7l7W+BG4K+VNTSzkWYWN7N4aWlphsrLDnvvHWYN33ADPPdcuFJo0iSNBkSkemQkANx9g7t/lbg/F6hjZo0raTvJ3WPuHissLMxEeVmloADOOAOWL4eOHeHkk6FvX1izJurKRKSmyUgAmNnPzcwS9zsn+v00E33nqj32gKefhttvh8WLwyziG2/UpvQikj7pugz0fuBlYG8zW2tmI8xslJmNSjQ5ElhhZkuBicAQd32xsSVmMHJkWFxuv/3CyKBXL3jjjagrE5GawLL553AsFvN4PB51GVnBHe67D848MywxPX48jBkTTiCLiPyLmZW4eyyZtpoJnCPM4Nhjw8YzhxwCf/wjdOsWzhWIiGwNBUCO+fnP4ZFHwgzid94JJ4rHj4fvv4+6MhHJNQqAHGQGRx0VRgNHHRVmFHfqFGYVi4gkSwGQwxo3hmnT4LHHoLQUunSBc8+F776LujIRyQUKgBpgwIAwGjj++LDaaPv28NJLUVclItlOAVBDNGoEkyfDk0+Gq4R69oSzzoKvv466MhHJVgqAGqZfP1ixAk49Fa6/Piw29+yzUVclItlIAVADbbcd3HQTPP881KoFBxwAp5wCGzZEXZmIZBMFQA3Wq1dYYfQPfwiLyhUXwxNPRF2ViGQLBUAN16ABXH01vPgiNGwIBx8MJ5wAn38edWUiEjUFQJ7o2hVeew3OPz9sOFNUFC4fFZH8pQDII/XqwaWXhtVFd9kFDj8chgwJcwhEJP8oAPJQ+/YhBC67DGbODKOBGTO08YxIvlEA5Kk6deC88+DVV8PeA0OHwqBBsG5d1JWJSKYoAPJcq1Zh1vDVV4dJZEVFMGWKRgMi+UABIBQUhEtFly0LO4+dcEK4Wujdd6OuTESqkwJA/m2vvcJm9DfdBC+8EEYHt92mbShFaioFgPyXWrXgtNPCchJdu4YZxAccAKtXR12ZiKSbAkAq1LIlzJsXFphbsiSsKXTddfDjj1FXJiLpogCQSpnBiBFhU/o+fcIexD17wqpVUVcmIumQlgAws7vM7GMzW1HJ62ZmE81stZktM7MO6ehXMqNpU5g1K2w+8+ab0K5d2Hfghx+irkxEUpGuEcAU4KDNvH4wsFfiNhK4NU39SoaYwW9/GzaeGTgw7DzWtWtYbE5EclNaAsDdFwCfbabJQOAeDxYCjcysSTr6lszaZZewIf3DD8P770MsBhdeCBs3Rl2ZiFRVps4BNAXeK/N4beI5yVGDB4dzA0OHhvWFOnaEV16JuioRqYpMBYBV8FyFc03NbKSZxc0sXqpVyrLaTjvBPffAnDmwfj106wZjx8K330ZdmYgkI1MBsBZoXuZxM+CDihq6+yR3j7l7rLCwMCPFSWoOOSTMGzjxRLjqKmjbNkwkE5HslqkAmAUcl7gaqCuw3t217FgNssMOcPvtMH9+uDqoVy844wz46quoKxORyqTrMtD7gZeBvc1srZmNMLNRZjYq0WQusAZYDdwBnJqOfiX79OkDy5fD738flpRo3RqefjrqqkSkIuZZvOxjLBbzeDwedRmylV54IUwke/PN8PXQ1VeHkYKIVB8zK3H3WDJtNRNYqk3PnmEZibFj4a67wuJyc+ZEXZWI/IsCQKrVNtvAhAmwcCE0agSHHQbHHguffhp1ZSKiAJCM6NQJSkrCpLEZM8LGM488EnVVIvlNASAZU68eXHIJxOPQrBkceSQcdRR89FHUlYnkJwWAZFzbtrBoUVhQbtasMBqYNk3bUIpkmgJAIlG7NowbF04S7703HHMMDBgQ1hcSkcxQAEik9tkH/v73sNnM00+H0cDkyRoNiGSCAkAiV1AAo0eHCWQdOsBJJ0G/fvD221FXJlKzKQAka/ziF2EUcOut4RxBcXGYTaxN6UWqhwJAskqtWjBqVFhcrmfPsKTEfvuF2cQikl4KAMlKLVrA44/DlCkhDNq2DUtJaFN6kfRRAEjWMoPhw8M2lP37wznnQPfuYSMaEUmdAkCyXpMmMHNmmEG8Zg20bx92IdOm9CKpUQBITjCD3/wmjAYGDw5LSnTqBK+9FnVlIrlLASA5pbAQ7r8/jAg++iiEwHnnwXffRV2ZSO5RAEhOOvzwMBo47jj485/D/IGFC6OuSiS3KAAkZ+24Y9hn4IknwtaT3bvDH/4A33wTdWUiuUEBIDmvf/9wqeioUXDttdCmDTz/fNRViWQ/BYDUCNtvD7fcAs8+G9YR6t0bTj0Vvvwy6spEspcCQGqU3r1h2TI46yy47bawnMS8eVFXJZKdFABS42y7bfgq6MUXoUGD8BXR734Hn38edWUi2SUtAWBmB5nZG2a22szGVfD68WZWamZLErcT09GvyOZ06xbmCZx7LtxzT9iUftasqKsSyR4pB4CZFQA3AwcDRcBQMyuqoOkD7t4ucZucar8iyahfHy6/HF55JcwhGDgQfvtbKC2NujKR6KVjBNAZWO3ua9z9e2AGMDAN7yuSNh06wOLFMH48PPxw2HjmgQe08Yzkt3QEQFPgvTKP1yaeK2+wmS0zs4fNrHllb2ZmI80sbmbxUv2aJmlUty5ccAG8+irsvjsMGRKWlVi3LurKRKKRjgCwCp4r/3vV34CW7t4GmA9MrezN3H2Su8fcPVZYWJiG8kT+W3ExvPQSXHklzJ0bRgNTp2o0IPknHQGwFij7G30z4IOyDdz9U3ffmHh4B9AxDf2KbLXatcPy0kuXhpPDxx8PhxwC774bdWUimZOOAFgM7GVmu5tZXWAI8F/XWphZkzIPBwCr0tCvSMr23hsWLIAbbwyb0xcXw+23axtKyQ8pB4C7bwJOB54k/GB/0N1Xmtl4MxuQaHaGma00s6XAGcDxqfYrki61asHpp4dN6Tt1CktK9O0L//xn1JWJVC/zLP7iMxaLeTwej7oMySPuMHkynH02bNoUVho9/XQoKIi6MpHkmFmJu8eSaauZwCJlmMFJJ4VtJ3v3htGjoVcv+Mc/oq5MJP0UACIVaNYMZs+Ge++FVaugXTu44oowKhCpKWrkV0CX/G0lr3+woRoqknz0/ffw1mr4pBQabge/2jusNySSTkW7bs9Fv26V8vvoKyCRNKpbF1oVhfkCG7+DkhJ4+23NG5DcVzvqAqpDOlJUpCKffBLOC0y7Ar5tHXYkiyX1u5ZI9tEIQKQKGjeG++4Lq4p++il06QLjxsG330ZdmUjVKQBEtsKvfx2uFDrhBJgwAdq3D/sPiOQSBYDIVmrUKMwZmDcPvvsO9t0XzjwTvv466spEkqMAEEnRgQeGTelPOw0mToTWreGZZ6KuSmTLFAAiadCwYVhP6Pnnw6zhPn3g5JNh/fqoKxOpnAJAJI169QorjJ59dvh6qLg4LDktko0UACJp1qABXHVV2HNg++3h0ENh+HD47LOoKxP5bwoAkWrSpUvYfez882H69DCRbObMqKsS+Q8FgEg1qlcPLr007Ee8665wxBFw9NHw8cdRVyaiABDJiHbtYNEiuPxyeOyxMBqYPl3LSUi0FAAiGVKnDpx7Lrz2Guy5JwwbBgMHwvvvR12Z5CsFgEiGFRWFWcPXXANPPRX2JL7rLo0GJPMUACIRKCiAMWNg2TJo2xZGjID+/eGdd6KuTPKJAkAkQnvtBc8+CzffDC+/HOYN3HKLNqWXzFAAiESsVi049dSwnET37mFJif33h9Wro65Marq0BICZHWRmb5jZajMbV8Hr9czsgcTri8ysZTr6FalJdtsNnnginA9YuhTatAnnCX78MerKpFpNmwYtW4bfBFq2DI8zJOUAMLMC4GbgYKAIGGpmReWajQA+d/c9geuACan2K1ITmYUlpl9/Hfr2DUtK9OgRHksNNG0ajBwZTv64hz9HjsxYCKRjBNAZWO3ua9z9e2AGMLBcm4HA1MT9h4E+ZmZp6FukRtp11zBfYPr08FVQ+/ZhDsEPP0RdmaTVeefBN9/893PffBOez4B0BEBT4L0yj9cmnquwjbtvAtYDO1X0ZmY20sziZhYvLS1NQ3kiuckMhg4Nv/0ffnhYUqJzZ1iyJOrKJG3efbdqz6dZOgKgot/ky1/RnEyb8KT7JHePuXussLAw5eJEct3OO8MDD8Cjj8K6ddCpE1xwAWzcGHVlkrIWLar2fJqlIwDWAs3LPG4GfFBZGzOrDewAaG1EkSoYNCiMBoYNg8sugw4dwvISksMuvzwsH1tWgwbh+QxIRwAsBvYys93NrC4wBJhVrs0sYHji/pHAM+6a9yhSVT/7GUyZEvYY2LAhXDZ6zjn/+zWy5Ihhw2DSpHAJmFn4c9Kk8HwGWDp+DpvZIcD1QAFwl7tfbmbjgbi7zzKz+sC9QHvCb/5D3H3Nlt43Fot5PB5PuT6RmmjDBhg7Fm6/PawtdOedYUMayW9mVuLusaTaZvMv4goAkS175hk48UT4v/8Lk8iuuCJsUSn5qSoBoJnAIjnugANg+XI488ywjERxcVhkTmRLFAAiNcC228L118Pf/w7160O/fmFU8MUXUVcm2UwBIFKD9OgR5gmMGwd33x2Wmp49O+qqJFspAERqmPr14S9/CZeI7rQT/PrXcMwx8MknUVcm2UYBIFJDxWIQj8PFF4eJZEVF8NBDUVcl2UQBIFKD1a0LF10EJSVhcunRR8ORR8KHH0ZdmWQDBYBIHmjTBhYuDJeIzp4dRgP33qttKPOdAkAkT9SuDX/8YzhJvM8+cNxxcNhhsHZt1JVJVBQAInnmV7+CBQvghhvguefClUJ33KHRQD5SAIjkoYICOOOMMIGsY8ewB8mBB4bZxJI/FAAieWyPPWD+/LCe0CuvhFnEN96oTenzhQJAJM/VqhVGACtXwn77hZFBr17w5ptRVybVTQEgIgA0bw5z5sDUqSEM2rSBK6+ETZuirkyqiwJARP7NLFwd9PrrcPDB4aqhbt3CuQKpeRQAIvI/mjQJW1A+8AC88044UTx+PHz/fdSVSTopAESkQmZh5vDKlWH28EUXhf2IS0qirkzSRQEgIptVWAjTp8Nf/wqlpdClC5x7Lnz3XdSVSaoUACKSlIEDw2hg+PCw2mj79vDyy1FXJalQAIhI0nbcMew9/OSTYSP6Hj3grLPg66+jrky2hgJARKqsXz9YsQJOOSXsRNamDTz7bNRVSVWlFABm9jMze8rM3kr8uWMl7X40syWJ26xU+hSR7LDddnDzzWE9IbOwN/Epp8CGDVFXJslKdQQwDnja3fcCnk48rsi37t4ucRuQYp8ikkX22w+WLYMxY8KSEsXF8MQTUVclyUg1AAYCUxP3pwKHp/h+IpKDGjSAa66Bl16Chg3DJLITToDPP4+6MtmcVANgF3dfB5D4c+dK2tU3s7iZLTQzhYRIDdW1K7z6arhM9N57w8Yzjz0WdVVSmS0GgJnNN7MVFdwGVqGfFu4eA34LXG9mv9hMfyMTYREvLS2tQhcikg3q14fLL4fFi2GXXeDww2Ho0DCHQLLLFgPA3fu6e3EFt8eAj8ysCUDiz48reY8PEn+uAZ4D2m+mv0nuHnP3WGFh4Vb8J4lINmjfPoTApZfCI4+E0cCMGdp4Jpuk+hXQLGB44v5w4H8Ge2a2o5nVS9xvDPQAXk+xXxHJAXXqwPnnh6+Fdt89jAQGDYJ166KuTCD1ALgCONDM3gIOTDzGzGJmNjnRZh8gbmZLgWeBK9xdASCSR4qLwwniq64Kk8iKimDKFI0Gomaexf8CsVjM4/F41GWISBq9+SaMGAEvvAD9+8OkSdCiRdRV1RxmVpI457pFmgksIhn1y1/C88+HrSdfeCFsSn/bbdqGMgoKABHJuFq14PTTw3ISXbuGGcR9+sA//xl1ZflFASAikWnZEubNg8mTw4ni1q3huuvgxx+jriw/KABEJFJm4ZzAypVhFDBmDPTsCatWRV1ZzacAEJGs0KwZzJoF990XThS3axf2HdCm9NVHASAiWcMMhg0Lm9IPGBCWlOjSBZYujbqymkkBICJZZ5dd4KGH4OGHYe1aiMXCnsTalD69FAAikrUGDw6jgaFDYfx46NgxLC8h6aEAEJGsttNOcM89MHt2WF66a1cYOxa+/TbqynKfAkBEcsKhh4YrhUaMCEtKtG0bJpLJ1lMAiEjO2GGHsHTEU0/BDz9Ar15wxhnw1VdRV5abFAAiknP69oXly8Ns4ptuChPInn466qpyjwJARHJSw4YwcSIsWAB164ZQGDkS1q+PurLcoQAQkZzWsycsWRJODN95Z1hcbs6cqKvKDQoAEcl522wDEybAwoXQqBEcdhgcdxx8+mnUlWU3BYCI1BidOkFJCVx4Idx/f9h45pFHoq4qeykARKRGqVcPLrkE4nFo2hSOPBKOOgo++ijqyrKPAkBEaqS2bWHRIvjzn8Mic0VFMG2atqEsSwEgIjVWnTrwpz+Fk8S//CUcc0xYZO7996OuLDsoAESkxttnnzBr+LrrwnyBoqJwxVC+jwZSCgAzO8rMVprZT2ZW6SbEZnaQmb1hZqvNbFwqfYqIbI2CAhg9GpYtgw4d4MQToV8/ePvtqCuLTqojgBXAEcCCyhqYWQFwM3AwUAQMNbOiFPsVEdkqe+4ZRgG33houGy0uDrOJ83FT+pQCwN1XufsbW2jWGVjt7mvc/XtgBjAwlX5FRFJRqxaMGhUWl+vZE37/e+jdG956K+rKMisT5wCaAu+Vebw28ZyISKRatIDHH4e77w5rC7VpA9dckz+b0m8xAMxsvpmtqOCW7G/xVsFzlZ56MbORZhY3s3hpaWmSXYiIbB0zOP74sPFM//5w9tnQvXsYHdR0WwwAd+/r7sUV3B5Lso+1QPMyj5sBH2ymv0nuHnP3WGFhYZJdiIikpkkTmDkTZsyANWugfXu47LKw7HRNlYmvgBYDe5nZ7mZWFxgCzMpAvyIiVWIGv/lNGA0MHgwXXACdO8Nrr0VdWfVI9TLQQWa2FugGzDGzJxPP72pmcwHcfRNwOvAksAp40N3zYHAlIrmqsDCsJTRzJnz4YVhj6PzzYePGqCtLL/MsngkRi8U8Ho9HXYaI5LHPP4cxY2DKlDCh7O67oUuXqKuqnJmVuHul87LK0kxgEZHN2HHH8EP/8cfD1pPdu4cTxd98E3VlqVMAiIgk4aCDYMUKOPnkcKlo27bw/PNRV5UaBYCISJK23x5uuQWeeSbMHO7dG047Db78MurKto4CQESkivbfP6wpdNZZYUmJ4mKYNy/qqqpOASAishW23RauvRZefBEaNAiTyEaMgC++iLqy5CkARERS0K1bmCfwpz/B1KlhqelZOTLTSQEgIpKi+vXDzmOLFoU5BAMHwrBh8MknUVe2eQoAEZE06dgRFi8OexI/9FAYDTz4YPZuPKMAEBFJo7p14cILoaQEdtstLC0xeHCYUZxtFAAiItWgdWt4+WWYMAHmzg2jgXvuya7RgAJARKSa1K4NY8fC0qUhAIYPh0MPhffe2/LfzQQFgIhINdt7b1iwACZODLOHW7WCSZOiHw0oAEREMqBWrbD15PLlYXXRk0+Gvn3D3gOR1RRd1yIi+WePPWD+/DACWLw4nCu44YZotqFUAIiIZJgZnHRS2Hayd28YPRp69YINGzJbhwJARCQizZvD7Nnh6qA994Tttsts/7Uz252IiJRlBsceG26ZphGAiEieUgCIiOQpBYCISJ5KKQDM7CgzW2lmP5lZpZsQm9nbZrbczJaYmXZ5FxHJAqmeBF4BHAHcnkTb/d09yxdHFRHJHykFgLuvAjCz9FQjIiIZk6lzAA7MM7MSMxuZoT5FRGQztjgCMLP5wM8reOk8d38syX56uPsHZrYz8JSZ/cPdF1TS30hgJECLFi2SfHsREakq8zQsR2dmzwFnu/sWT/Ca2cXAV+5+dRJtS4F3trKsxkC2nXPIxppAdVVFNtYEqquqsrGudNW0m7sXJtOw2mcCm9m2QC13/zJxvx8wPpm/m+x/RCX9xt290iuTopCNNYHqqopsrAlUV1VlY11R1JTqZaCDzGwt0A2YY2ZPJp7f1czmJprtArxgZkuBV4A57v5EKv2KiEjqUr0KaCYws4LnPwAOSdxfA7RNpR8REUm/mjwTeFLUBVQgG2sC1VUV2VgTqK6qysa6Ml5TWk4Ci4hI7qnJIwAREdmMnA8AMzvIzN4ws9VmNq6C1+uZ2QOJ1xeZWcssqOl4MytNrI20xMxOzEBNd5nZx2a2opLXzcwmJmpeZmYdqrumJOvqbWbryxyrCzNQU3Mze9bMViXWujqzgjYZP15J1hXF8apvZq+Y2dJEXZdU0Cajn8Mka8r457BM3wVm9pqZza7gtcwdK3fP2RtQAPwT2AOoCywFisq1ORW4LXF/CPBAFtR0PHBTho9VL6ADsKKS1w8BHgcM6AosypK6egOzM3ysmgAdEve3A96s4N8w48crybqiOF4GNEzcrwMsArqWa5Ppz2EyNWX8c1im7zHA9Ir+rTJ5rHJ9BNAZWO3ua9z9e2AGMLBcm4HA1MT9h4E+Vr2LFyVTU8Z5mHn92WaaDATu8WAh0MjMmmRBXRnn7uvc/dXE/S+BVUDTcs0yfrySrCvjEsfgq8TDOolb+ZOLGf0cJllTJMysGXAoMLmSJhk7VrkeAE2B98o8Xsv/fiD+3cbdNwHrgZ0irglgcOKrg4fNrHk11pOsZOuOQrfEUP5xM2uVyY4Tw+/2hN8gy4r0eG2mLojgeCW+0lgCfAw85e6VHq8MfQ6TqQmi+RxeD4wFfqrk9Ywdq1wPgIpSsXzKJ9MmnZLp729AS3dvA8znP2kfpUwfp2S9Spja3ha4Efhrpjo2s4bAI8Bod99Q/uUK/kpGjtcW6orkeLn7j+7eDmgGdDaz4nJNMn68kqgp459DMzsM+NjdSzbXrILnquVY5XoArAXKpnYz4IPK2phZbWAHqvcrhy3W5O6fuvvGxMM7gI7VWE+ykjmWGefuG/41lHf3uUAdM2tc3f2aWR3CD9lp7v5oBU0iOV5bqiuq41Wm/y+A54CDyr2U6c/hFmuK6HPYAxhgZm8Tvh4+wMzuK9cmY8cq1wNgMbCXme1uZnWnC923AAABRklEQVQJJ0xmlWszCxieuH8k8Iwnzq5EVVO574oHEL7Ljdos4LjE1S1dgfXuvi7qoszs5//6/tPMOhP+n/20mvs04E5glbtfW0mzjB+vZOqK6HgVmlmjxP1tgL7AP8o1y+jnMJmaovgcuvuf3L2Zu7ck/Gx4xt2PKdcsY8eq2heDq07uvsnMTgeeJFx9c5e7rzSz8UDc3WcRPjD3mtlqQooOyYKazjCzAcCmRE3HV2dNAGZ2P+EKkcYW1m+6iHBiDHe/DZhLuLJlNfANcEJ115RkXUcCp5jZJuBbYEg1BziE39KOBZYnvkMGOBdoUaauKI5XMnVFcbyaAFPNrIAQOA+6++woP4dJ1pTxz2FlojpWmgksIpKncv0rIBER2UoKABGRPKUAEBHJUwoAEZE8pQAQEclTCgARkTylABARyVMKABGRPPX/NYdaemG7KgYAAAAASUVORK5CYII=\n", "text/plain": [ "
" ] }, "metadata": { "needs_background": "light" }, "output_type": "display_data" }, { "data": { "image/png": "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\n", "text/plain": [ "
" ] }, "metadata": { "needs_background": "light" }, "output_type": "display_data" }, { "data": { "image/png": "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\n", "text/plain": [ "
" ] }, "metadata": { "needs_background": "light" }, "output_type": "display_data" }, { "data": { "image/png": "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\n", "text/plain": [ "
" ] }, "metadata": { "needs_background": "light" }, "output_type": "display_data" }, { "data": { "image/png": "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\n", "text/plain": [ "
" ] }, "metadata": { "needs_background": "light" }, "output_type": "display_data" }, { "data": { "image/png": "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\n", "text/plain": [ "
" ] }, "metadata": { "needs_background": "light" }, "output_type": "display_data" }, { "data": { "image/png": "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\n", "text/plain": [ "
" ] }, "metadata": { "needs_background": "light" }, "output_type": "display_data" }, { "data": { "image/png": "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\n", "text/plain": [ "
" ] }, "metadata": { "needs_background": "light" }, "output_type": "display_data" }, { "data": { "image/png": "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\n", "text/plain": [ "
" ] }, "metadata": { "needs_background": "light" }, "output_type": "display_data" }, { "data": { "image/png": "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\n", "text/plain": [ "
" ] }, "metadata": { "needs_background": "light" }, "output_type": "display_data" }, { "data": { "image/png": "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\n", "text/plain": [ "
" ] }, "metadata": { "needs_background": "light" }, "output_type": "display_data" }, { "data": { "image/png": "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\n", "text/plain": [ "
" ] }, "metadata": { "needs_background": "light" }, "output_type": "display_data" }, { "data": { "image/png": "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\n", "text/plain": [ "
" ] }, "metadata": { "needs_background": "light" }, "output_type": "display_data" }, { "data": { "image/png": "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\n", "text/plain": [ "
" ] }, "metadata": { "needs_background": "light" }, "output_type": "display_data" }, { "data": { "image/png": "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\n", "text/plain": [ "
" ] }, "metadata": { "needs_background": "light" }, "output_type": "display_data" }, { "data": { "image/png": "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\n", "text/plain": [ "
" ] }, "metadata": { "needs_background": "light" }, "output_type": "display_data" }, { "data": { "image/png": "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\n", "text/plain": [ "
" ] }, "metadata": { "needs_background": "light" }, "output_type": "display_data" }, { "data": { "image/png": "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\n", "text/plain": [ "
" ] }, "metadata": { "needs_background": "light" }, "output_type": "display_data" }, { "data": { "image/png": "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\n", "text/plain": [ "
" ] }, "metadata": { "needs_background": "light" }, "output_type": "display_data" }, { "data": { "image/png": "iVBORw0KGgoAAAANSUhEUgAAAYAAAAD8CAYAAAB+UHOxAAAABHNCSVQICAgIfAhkiAAAAAlwSFlzAAALEgAACxIB0t1+/AAAADl0RVh0U29mdHdhcmUAbWF0cGxvdGxpYiB2ZXJzaW9uIDMuMC4zLCBodHRwOi8vbWF0cGxvdGxpYi5vcmcvnQurowAAIABJREFUeJzt3Xm81HP///HHq5UI0YmQsn+d094okRRJQiWh5OIinStXLluIy1KyXZHtki0kXFkuS4pCyRpK075ZktAiuaQQsrx+f7ynn+N0Tp3TzDmfOTPP++02tzPzmc+Zz6u5Tec5n897M3dHRESyT6WoCxARkWgoAEREspQCQEQkSykARESylAJARCRLKQBERLKUAkBEJEspAEREslSJA8DMRprZV2Y2v8C2W8zsAzOba2ZjzGynYn53qZnNM7PZZhZPReEiIpIcK+lIYDNrC3wPPOruDRPbOgKvufuvZjYUwN0HFvG7S4GYu39dmuJq167tDRo0KM2viIhktRkzZnzt7jkl2bdKSV/U3d8yswaFtk0s8HAq0KOkr1cSDRo0IB7XCYOISEmZ2Wcl3TeVbQBnAy8V85wDE81shpnlp/CYIiKylUp8BrA5ZnYl8CswuphdDnP3FWZWB5hkZh+4+1vFvFY+kA+w1157paI8EREpQtJnAGZ2JnA80NuLaVBw9xWJn18BY4CWxb2eu49w95i7x3JySnQZS0REtkJSAWBmnYCBQBd3X1/MPtuZWc2N94GOwPyi9hURkfJTmm6gTwDvAQea2TIz6wMMB2oSLuvMNrP7EvvubmYTEr+6KzDFzOYA7wPj3f3llP4rRESk1ErTC6hXEZsfKmbfFUDnxP0lQJOtqk5ERMqMRgKLiGSpjAyA666D6dOjrkJEJL1lXAB88w3cfz8ccghceimsL7JpWkREMi4Adt4ZFiyAc86BYcOgSRN4442oqxIRST8ZFwAAO+4YzgJeew3coX176NcP1q6NujIRkfSRkQGwUfv2MHcuDBgADzwAeXkwfnzUVYmIpIeMDgCAGjXCpaD33oNateD446F3b1i9OurKRESilfEBsFHLljBjBgweDE8/Dbm58OST4RKRiEg2ypoAAKhWDQYNgpkzYZ99oFcv6NoVli+PujIRkfKXVQGwUcOG8O67cOut8Oqr4WzggQd0NiAi2SUrAwCgcmW4+GKYNw9atID8fDjqKFi8OOrKRETKR9YGwEb77guTJ8OIEaGNoHHjcGbw229RVyYiUrayPgAAzKBvX1i4EDp0gEsugdatYb4mrRaRDKYAKGCPPWDs2NA7aOlSaN489BrasCHqykREUk8BUIgZnHpqOBs45RS49toQBNOmRV2ZiEhqKQCKUbs2/Oc/8OKLYQqJ1q1Do/EPP0RdmYhIapQqAMxspJl9ZWbzC2zb2cwmmdnHiZ+1ivndMxP7fJxYR7hCOO64MLlcv35w++2hkfi116KuSkQkeaU9AxgFdCq07XJgsrvvD0xOPP4TM9sZGAS0IiwIP6i4oEhHO+wA99wTZhWtVCl0F+3bF779NurKRES2XqkCwN3fAr4ptLkr8Eji/iNAtyJ+9Rhgkrt/4+5rgElsGiRp74gjwuRyl10GI0eGyeXGjYu6KhGRrZOKNoBd3X0lQOJnnSL22QP4osDjZYltFc6228LQoaFRuHbtMJVEz57w1VdRVyYiUjrl1QhsRWwrcuIFM8s3s7iZxVen8ZSdsRjE42H5yTFj4KCDQqOxppMQkYoiFQGwyszqAiR+FvVdeBlQr8DjPYEVRb2Yu49w95i7x3JyclJQXtmpWhWuugpmzYIDDoC//CVMN/3FF1v+XRGRqKUiAMYBG3v1nAmMLWKfV4COZlYr0fjbMbEtI+TmwpQpcOedoaE4Lw/uvRd+/z3qykREilfabqBPAO8BB5rZMjPrA/wLONrMPgaOTjzGzGJm9iCAu38DXAdMT9yGJLZljMqV4fzzw/QRrVrB3/8eViT76KOoKxMRKZp5Gl+0jsViHo/Hoy6j1Nxh1KgwcOynn8Jo4osvhipVoq5MRDKdmc1w91hJ9tVI4DJgBmedFaaTOPZYGDgwnBXMmRN1ZSIif1AAlKG6deHZZ8MSlMuWhZ5DV18NP/8cdWUiIgqAMmcGPXrAokVhMfrrr4dmzcKKZCIiUVIAlJOddw7tAi+/HCaUa9MGLrgAvv8+6spEJFspAMrZMceEnkL9+8O//w2NGsGkSVFXJSLZSAEQgZo14a674O23oXp16NgRzj4b1qyJujIRySYKgAi1aQOzZ8MVV8Cjj4YBZWPGRF2ViGQLBUDEttkGbrwR3n8fdtsNuneHk0+GL7+MujIRyXQKgDTRvHkIgRtvhBdeCGcDjz6qyeVEpOwoANJI1arhctDs2SEAzjwzDCT77LOoKxORTKQASEP/93/w1luhoXjKFGjYEIYP1+RyIpJaCoA0VakSnHdeWI/4sMPgH/+Atm3hww+jrkxEMoUCIM3Vrw8vvRQGkS1cCE2awE03wS+/RF2ZiFR0CoAKwCy0ByxcCCecAP/8Z5hcbtasqCsTkYpMAVCB7LZbmFju2Wdh5Uo4+OAQBj/9FHVlIlIRKQAqoO7dw9nAGWeEy0FNmoTGYhGR0lAAVFC1asHIkTBxImzYAIcfHhqNv/su6spEpKJIOgDM7EAzm13gts7MLiy0TzszW1tgn2uSPa4ERx8N8+aF5SjvuSd0GX0lY1ZbFpGylHQAuPuH7t7U3ZsCLYD1QFEz2ry9cT93H5LsceUP228fFqSfMgVq1IBOnUKj8f/+F3VlIpLOUn0J6CjgE3fX2NUIHHpoGEV81VXw+ONhNPHTT2s6CREpWqoDoCfwRDHPtTazOWb2kpnlFfcCZpZvZnEzi69evTrF5WW+6tXhuusgHod69eCUU+Ckk0KvIRGRglIWAGZWDegCPF3E0zOB+u7eBLgLeL6413H3Ee4ec/dYTk5OqsrLOk2awNSpMHRoGEiWmwsPP6yzARH5QyrPAI4FZrr7qsJPuPs6d/8+cX8CUNXMaqfw2FKEKlXgsstgzhxo3DgsOtOxI3z6adSViUg6SGUA9KKYyz9mtpuZWeJ+y8Rx1URZTg44AF5/He69F6ZNCz2F7rwTfvst6spEJEopCQAzqwEcDTxXYFs/M+uXeNgDmG9mc4B/Az3ddTGiPFWqBP36hcnljjgCLrwwjB1YuDDqykQkKpbOf4djsZjH4/Goy8g47qGX0AUXhIFjV18NAweG9QhEpGIzsxnuHivJvhoJnIXMoHfv8O2/e/cQALEYzJgRdWUiUp4UAFmsTh144gkYOxa+/hpatgxnAj/+GHVlIlIeFABCly6hbaBPH7j55tBj6M03o65KRMqaAkAA2GknGDECJk8OS0+2awfnngvr1kVdmYiUFQWA/MmRR8LcuXDxxSEQ8vJgwoSoqxKRsqAAkE1stx3ceiu8+y7ssAMcdxycfnpoJxCRzKEAkGK1agUzZ8KgQfDUU2E6iaee0nQSIplCASCbVb06DB4cgqB+fejZE7p1gxUroq5MRJKlAJASadQI3nsPhg0Lq5Dl5sKDD+psQKQiUwBIiVWpAgMGhBXImjaFvn2hQwdYsiTqykRkaygApNT22w9eew3uvx+mTw+Ty91+uyaXE6loFACyVSpVgvz8MJ3EkUeGbqOHHRYGlIlIxaAAkKTsuSe88AKMHg2ffALNmsGQIbBhQ9SViciWKAAkaWZw2mnhbKBHj9BttEWLcHlIRNKXAkBSJicnTDM9bhysWQOHHAKXXALr10ddmYgURQEgKXfCCaEtoG/fMKK4cWN4442oqxKRwlK5KPxSM5tnZrPNbJNVXCz4t5ktNrO5ZtY8VceW9LPjjnDffaG3EED79vC3v8HatdHWJSJ/SPUZQHt3b1rMajTHAvsnbvnAvSk+tqSh9u3D5HKXXBIGjuXlwYsvRl2ViED5XgLqCjzqwVRgJzOrW47Hl4jUqAG33BJGEteqFS4RnXYarF4ddWUi2S2VAeDARDObYWb5RTy/B/BFgcfLEtskS7RsGZadvPZaeOaZMJ3E449rOgmRqKQyAA5z9+aESz39zaxtoeetiN/Z5L++meWbWdzM4qv1FTHjVKsG11wDs2bBvvuGtYm7dIFly6KuTCT7pCwA3H1F4udXwBigZaFdlgH1CjzeE9hkTkl3H+HuMXeP5eTkpKo8STN5efDOO3DbbWEVstzcMLXE779HXZlI9khJAJjZdmZWc+N9oCMwv9Bu44AzEr2BDgHWuvvKVBxfKqbKleGii2D+fDj4YOjXD446ChYvjroykeyQqjOAXYEpZjYHeB8Y7+4vm1k/M+uX2GcCsARYDDwA/D1Fx5YKbp994NVXQy+hWbPC1NPDhsGvv0ZdmUhmM0/jFrhYLObx+CZDCiSDrVgBf/87jB0bzgoeeigEgoiUjJnNKKYr/iY0EljSyu67w5gxYenJpUuhefMwt9DPP0ddmUjmUQBI2jGDU06BRYvCEpRDhoQgmDo16spEMosCQNLWLrvAY4/B+PGwbh0cemhYd+CHH6KuTCQzKAAk7XXuHCaXO/fcsPJYo0ah66iIJEcBIBXCDjvA3XfDm2+GtYk7dIBzzoFvv426MpGKSwEgFUrbtjBnDgwcCKNGhQFkY8dGXZVIxaQAkApn223hX/+CadOgTh3o1g1OPRVWrYq6MpGKRQEgFdbGZSevvx6efz6cDTz2mCaXEykpBYBUaFWrwpVXwuzZcOCBcMYZcNxx8PnnUVcmkv4UAJIRDjoI3n4b7rwzNBTn5cE992hyOZHNUQBIxqhcGc4/P0wu17o19O8P7drBRx9FXZlIelIASMbZe2945RV4+GGYNy8sSj90qCaXEylMASAZyQz++ldYuDAMJLv8cmjVKrQViEigAJCMVrcuPPdcWIJy+XKIxUKj8U8/RV2ZSPQUAJIVTjopnA2cfjrceCM0awbvvht1VSLRUgBI1th55zB6+OWXYf16aNMmNBp//33UlYlEI+kAMLN6Zva6mS0yswVmdkER+7Qzs7VmNjtxuybZ44psrWOOCT2F+veH4cOhYUOYODHqqkTKXyrOAH4FBrj7QcAhQH8zyy1iv7fdvWniNiQFxxXZajVrwl13wVtvwTbbhFA46yxYsybqykTKT9IB4O4r3X1m4v53wCJgj2RfV6Q8tGkTegZdcUWYRiI3NzQai2SDlLYBmFkDoBkwrYinW5vZHDN7yczyUnlckWRss01oGJ4+PfQaOukk6NEDvvwy6spEylbKAsDMtgeeBS5093WFnp4J1Hf3JsBdwPObeZ18M4ubWXz16tWpKk9ki5o1CzOM3nQTvPhiOBsYNUqTy0nmSkkAmFlVwh//0e6+yQm0u69z9+8T9ycAVc2sdlGv5e4j3D3m7rGcnJxUlCdSYlWrhkFjc+aE+YTOOgs6dQoL1ItkmlT0AjLgIWCRu99WzD67JfbDzFomjvu/ZI8tUlYOPDBMKjd8eBgv0LBhaDTW5HKSSVJxBnAY8BfgyALdPDubWT8z65fYpwcw38zmAP8GerrrxFrSW6VKoavo/Pl/jBlo2xY++CDqykRSw9L573AsFvN4PB51GSK4h15CF14IP/wAgwbBpZeGS0Yi6cTMZrh7rCT7aiSwSAmYhcVmFi2CLl3CfEItW8KsWVFXJrL1FAAipbDrrvD00/Dss6Gb6MEHhzEEmlxOKiIFgMhW6N49TC53xhlhgfomTWDKlKirEimdKlEXUBaufWEBC1cUHoogUgZawdEHhFXHetwDu4+FffYOq5OJbK3c3Xdg0AllP15WZwAiSapVK6wzsMcesGJ5GFGsOYWkIsjIM4DySE6Rorz3HvTpA5NGhctDt90Gu+wSdVUiRdMZgEgKtW4degZddRU8/niYTuKZZzSdhKQnBYBIilWvDtddB/E41KsHJ58cJphbuTLqykT+TAEgUkaaNIGpU2HoUHjppXA28PDDOhuQ9KEAEClDVarAZZeFyeUaN4azz4aOHeHTT6OuTEQBIFIuDjgAXn8d7r03TDndsCHceSf89lvUlUk2UwCIlJNKlaBfP1iwAI44IswrdPjhYUCZSBQUACLlrF49GD8e/vOfMICsWTO4/nr45ZeoK5NsowAQiYAZ9O4dvv2feCJcfXUYTDZjRtSVSTZRAIhEqE4dePJJGDsWvv46zDA6cCD8+GPUlUk2UACIpIEuXULbQJ8+cPPNoQvpm29GXZVkOgWASJrYaScYMQImTw69g9q1g3PPhXWa11DKSKoWhe9kZh+a2WIzu7yI56ub2VOJ56eZWYNUHFckEx15JMydCxdfHAIhLw8mTIi6qjI2ejQ0aBC6SjVoEB5LmUvFovCVgbuBY4FcoJeZ5RbarQ+wxt33A24HhiZ7XJFMtt12cOutYUH6HXaA446D008P7QQZZ/RoyM+Hzz4Lw6Q/+yw8VgiUuVScAbQEFrv7EnffADwJdC20T1fgkcT9Z4CjzMxScGyRjNaqFcycGdYgfuqpMJ3EU09l2HQSV14J69f/edv69WG7lKlUBMAewBcFHi9LbCtyH3f/FVgLFDlJrpnlm1nczOKrV69OQXkiFVv16jB4cAiC+vWhZ0/o1g2WL4+6shT5/PPSbZeUSUUAFPVNvvD3k5LsEza6j3D3mLvHcnJyki5OJFM0ahTWGxg2DCZNCm0DDz6YAWcDe+1Vuu2SMqkIgGVAvQKP9wRWFLePmVUBdgS+ScGxRbJKlSowYEBoJG7WDPr2hQ4d4JNPoq4sCTfcADVq/HlbjRphu5SpVATAdGB/M9vbzKoBPYFxhfYZB5yZuN8DeM29wn9vEYnMfvuF7qL33x+WoGzUKKw+ViEnl+vdO3R3ql8/DJGuXz887t076soynqXi77CZdQbuACoDI939BjMbAsTdfZyZbQM8BjQjfPPv6e5LtvS6sVjM4/F40vWJZLJly8J4gRdfDCOJH3oozDYq2cnMZrh7rET7pvMXcQWASMm4hyklzj8f1q4NHWiuuAKqVYu6MilvpQkAjQQWyQBm0KtXmFzu5JNDr6EWLeD996OuTNKZAkAkg+TkhPFT48bBmjVhkfpLLtm0m70IKABEMtIJJ4TJ5fr2DSOKGzUKK5KJFKQAEMlQO+4I990X/vCbhTmG/va30EYgAgoAkYzXrl0YN3DJJWHgWG4uvPBC1FVJOlAAiGSBGjXglltg6lTYZZew/sBpp4FmW8luCgCRLHLwwRCPw7XXwjPPwEEHweOPZ8B0ErJVFAAiWaZaNbjmGpg1K4wo7t07nBEsWxZ1ZVLeFAAiWSovD955J0whMXlyaBu4/374/feoK5PyogAQyWKVK8NFF8H8+eHyUL9+obfQxx9HXZmUBwWAiLDPPvDqq6GX0OzZ0LhxmHb611+jrkzKkgJARIAwVqBPnzCdxDHHwKWXhpHEc+dGXZmUFQWAiPzJ7rvDmDFh6cnPPgtzCl1zDfz8c9SVSaopAERkE2ZwyimwaFFYgvK666B58zCOQDKHAkBEirXLLvDYYzB+PHz3HRx6aGg0/uGHqCuTVFAAiMgWde4cegqdey7ccUeYXG7y5KirkmQlFQBmdouZfWBmc81sjJntVMx+S81snpnNNjOt8CJSAe2wA9x9N7z5ZlibuEMHOOcc+PbbqCuTrZXsGcAkoKG7NwY+Aq7YzL7t3b1pSVeqEZH01LYtzJkDAwfCqFFhANnYsVFXJVsjqQBw94nuvrGn8FRgz+RLEpF0t+228K9/wbRpUKcOdOsGp54Kq1ZFXZmURirbAM4GXirmOQcmmtkMM8tP4TFFJEItWsD06XD99fD88+Fs4LHHNLlcRbHFADCzV81sfhG3rgX2uRL4FRhdzMsc5u7NgWOB/mbWdjPHyzezuJnFV2uuWpG0V7VqWIR+9mw48EA44ww47jj4/POoK5MtMU8yqs3sTKAfcJS7b3HlUTMbDHzv7sO2tG8sFvN4XG3GIhXFb7/BPffAFVeEsQRDh4b5hSqpv2G5MbMZJW1rTbYXUCdgINCluD/+ZradmdXceB/oCMxP5rgikp4qV4Z//CN0GW3dGvr3DyuSffRR1JVJUZLN5eFATWBSoovnfQBmtruZTUjssyswxczmAO8D49395SSPKyJprEEDeOUVePhhmDcvTC43dKgml0s3SV8CKku6BCRS8a1cCeedB889F6aTGDkSmjSJuqrMVW6XgEREtqRuXXj22bAE5fLlEIvBVVfBTz9FXZkoAESkXJx0UphqundvuOEGaNYM3n036qqymwJARMrNzjuH0cMvvwzr10ObNnD++fD991FXlp0UACJS7o45JvQU6t8fhg+Hhg1h4sSoq8o+CgARiUTNmnDXXfDWW7DNNiEUzjoL1qyJurLsoQAQkUi1aRNGEV9xRZhGIjc39BiSsqcAEJHIbbMN3HhjmFeobt3QYNyjB3z5ZdSVZTYFgIikjWbNwgyjN90EL74YzgYeeUSTy5UVBYCIpJWqVeHyy8OaA3l58Ne/QqdOsHRp1JVlHgWAiKSlAw8Mq48NHx7GCzRsGBqNf/896soyhwJARNJWpUqhq+j8+X+MGWjbFj74IOrKMoMCQETSXv368NJLoT1g4cIwl9CNN8Ivv0RdWcWmABCRCsEsLDazaBF07RoWoTn4YJg5M+rKKi4FgIhUKLvuCv/9bxgrsGoVtGwZxhD8+GPUlVU8CgARqZBOPDFcDjrzzLBAfdOmMGVK1FVVLAoAEamwatWChx6CSZNgwwY4/PCw9sB330VdWcWQ7JKQg81seWI1sNlm1rmY/TqZ2YdmttjMLk/mmCIihXXoEFYeu+CCsCZxXl5oNJbNS8UZwO3u3jRxm1D4STOrDNwNHAvkAr3MLDcFxxUR+f+23x7uuAPeeSfc79w5NBr/739RV5a+yuMSUEtgsbsvcfcNwJNA13I4rohkodatYdYsuPpqeOKJMJ3E009rOomipCIAzjOzuWY20sxqFfH8HsAXBR4vS2wTESkT1avDkCEQj0O9enDKKdC9O6xYEXVl6WWLAWBmr5rZ/CJuXYF7gX2BpsBK4NaiXqKIbcVmsZnlm1nczOKrV68u4T9DRGRTTZrA1Klw881hFbLc3NBorLOBYIsB4O4d3L1hEbex7r7K3X9z99+BBwiXewpbBtQr8HhPoNgcdvcR7h5z91hOTk5p/z0iIn9SpQpcemmYXK5JEzjnHDj6aFiyJOrKopdsL6C6BR6eCMwvYrfpwP5mtreZVQN6AuOSOa6ISGkdcAC8/jrcey+8/z40ahQajX/7LerKopNsG8DNZjbPzOYC7YGLAMxsdzObAODuvwLnAa8Ai4D/uvuCJI8rIlJqlSpBv36wYAG0awcXXRQmmVu4MOrKomGexhfDYrGYx+PxqMsQkQzkDo8/HsYOfPcdXHUVDBwI1apFXVlyzGyGu8dKsq9GAotIVjKD3r3Dt//u3eGaa8Lkctn0nVMBICJZrU6dMF5g7Fj4+mto1Qouuyw7JpdTAIiIAF26hLaBPn3gllugceOwIlkmUwCIiCTstBOMGAGTJ4elJ9u1g3PPhXXroq6sbCgAREQKOfJImDsXLr44BEJeHowfH3VVqacAEBEpwnbbwa23hgXpd9wRjj8eTj89tBNkCgWAiMhmtGoVlp0cNCisRHbQQfDkk5kxnYQCQERkC6pVg8GDYcYM2Htv6NULunWD5cujriw5CgARkRJq1Ajeew+GDQurkOXmwgMPVNyzAQWAiEgpVK4MAwaERuLmzSE/H446Cj75JOrKSk8BICKyFfbbL3QXvf/+cGmoUSO47baKNbmcAkBEZCtVqhTOABYsCGcBAwbAoYfC/KLmRU5DCgARkSTtuSeMGxemlFiyJFwauvZa2LAh6so2TwEgIpICZtCzJyxaBCefHHoNtWgR1h5IVwoAEZEUql0bRo+GF16ANWvCIvUDBsD69VFXtikFgIhIGTj++NA2kJ8fGocbNQorkqUTBYCISBnZccewBOUbb4QG4yOPDIGwdm3UlQXJrgn8lJnNTtyWmtnsYvZbmlg6craZZdFyCyIicMQRYVH6Sy+Fhx4KA8jGpcHK6EkFgLuf6u5N3b0p8Czw3GZ2b5/Yt0RLlYmIZJIaNeDmm2HaNNhlF+jaNTQaf/VVdDWl5BKQmRlwCvBEKl5PRCRTxWJh2ckhQ+C558LZwOjR0Uwnkao2gMOBVe7+cTHPOzDRzGaYWf7mXsjM8s0sbmbx1atXp6g8EZH0Ua0aXH01zJ4N++8fppk+4QT44ovyrWOLAWBmr5rZ/CJuXQvs1ovNf/s/zN2bA8cC/c2sbXE7uvsId4+5eywnJ6fE/xARkYomNxemTIE77gg9hPLy4L77wmpk5cE8yfMOM6sCLAdauPuyEuw/GPje3Ydtad9YLObxuNqMRSTzffpp6CH06qvQti1MmBAWpSktM5tR0rbWVFwC6gB8UNwffzPbzsxqbrwPdAQqyEwZIiLlY++9YeLE0Eto//237o9/aaUiAHpS6PKPme1uZhMSD3cFppjZHOB9YLy7v5yC44qIZBQzOPtsePDB8jlelWRfwN3/WsS2FUDnxP0lQJNkjyMiIqmlkcAiIllKASAikqUUACIiWUoBICKSpRQAIiJZSgEgIpKlFAAiIlkq6akgypKZrQY+28pfrw18ncJyKiq9D4Heh0DvQ5DJ70N9dy/RRGppHQDJMLO41h7Q+7CR3odA70Og9yHQJSARkSylABARyVKZHAAjoi4gTeh9CPQ+BHofAr0PZHAbgIiIbF4mnwGIiMhmZFwAmFknM/vQzBab2eVR1xMlM1tqZvPMbLaZZc3SamY20sy+MrP5BbbtbGaTzOzjxM9aUdZYHop5Hwab2fLEZ2K2mXWOssbyYGb1zOx1M1tkZgvM7ILE9qz7TBSWUQFgZpWBuwlrD+cCvcwsN9qqItfe3ZtmWZe3UUCnQtsuBya7+/7A5MTjTDeKTd8HgNsTn4mm7j6hiOczza/AAHc/CDiEsC55Ltn5mfiTjAoAoCWw2N2XuPsG4Emg6xZ+RzKMu78FfFNoc1fgkcT9R4Bu5VpUBIp5H7KOu69095mJ+98Bi4A9yMLPRGGZFgB7AF8UeLwssS1bOTDRzGaYWX49p0mAAAABeklEQVTUxURsV3dfCeEPAlAn4nqidJ6ZzU1cIsqqyx5m1gBoBkxDn4mMCwArYls2d3M6zN2bEy6J9TeztlEXJJG7F9gXaAqsBG6NtpzyY2bbA88CF7r7uqjrSQeZFgDLgHoFHu8JrIiolsgl1mbG3b8CxhAukWWrVWZWFyDx86uI64mEu69y99/c/XfgAbLkM2FmVQl//Ee7+3OJzVn/mci0AJgO7G9me5tZNaAnMC7imiJhZtuZWc2N94GOwPzN/1ZGGwecmbh/JjA2wlois/EPXsKJZMFnwswMeAhY5O63FXgq6z8TGTcQLNGt7Q6gMjDS3W+IuKRImNk+hG/9AFWAx7PlvTCzJ4B2hBkfVwGDgOeB/wJ7AZ8DJ7t7RjeQFvM+tCNc/nFgKfC3jdfBM5WZtQHeBuYBvyc2/5PQDpBVn4nCMi4ARESkZDLtEpCIiJSQAkBEJEspAEREspQCQEQkSykARESylAJARCRLKQBERLKUAkBEJEv9P5ZKIVYCZuZ5AAAAAElFTkSuQmCC\n", "text/plain": [ "
" ] }, "metadata": { "needs_background": "light" }, "output_type": "display_data" }, { "data": { "image/png": "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\n", "text/plain": [ "
" ] }, "metadata": { "needs_background": "light" }, "output_type": "display_data" }, { "data": { "image/png": "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\n", "text/plain": [ "
" ] }, "metadata": { "needs_background": "light" }, "output_type": "display_data" }, { "data": { "image/png": "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\n", "text/plain": [ "
" ] }, "metadata": { "needs_background": "light" }, "output_type": "display_data" }, { "data": { "image/png": "iVBORw0KGgoAAAANSUhEUgAAAXwAAAD8CAYAAAB0IB+mAAAABHNCSVQICAgIfAhkiAAAAAlwSFlzAAALEgAACxIB0t1+/AAAADl0RVh0U29mdHdhcmUAbWF0cGxvdGxpYiB2ZXJzaW9uIDMuMC4zLCBodHRwOi8vbWF0cGxvdGxpYi5vcmcvnQurowAAHB1JREFUeJzt3XuczmX+x/HXxyA6bg6prNJB/YxxnqisUkSoqFRKNlbJlg60qPhFBy062U42lVTEdlKK7UAnFTEoxw62VVGL1q+DVFLX74/PtNkyGea+55r7e7+fj8c87pl7hu/n+7gf3nO5ruv+XBZCQEREkq9c7AJERKR0KPBFRLKEAl9EJEso8EVEsoQCX0QkSyjwRUSyhAJfRCRLKPBFRLKEAl9EJEuUj13AlqpVqxZq164duwwRkYwyf/78T0MI1bf1c2Uq8GvXrk1BQUHsMkREMoqZfVCcn9OUjohIllDgi4hkCQW+iEiWUOCLiGQJBb6ISJZQ4IuIZAkFvohIlkhE4K9bB/36weefx65ERKTsSkTgz5wJt94Kubnw1FOxqxERKZsSEfhdu8KcOVC1Kpx0Epx1lo/6RUTkJ4kIfIDDDoOCArj6anj0UahbFx56CEKIXZmISNmQmMAHqFgRrroKFi6Egw+Gbt18xL9qVezKRETiS1Tg/6hePXjtNbj5Zp/fz82Fu+6CH36IXZmISDyJDHyAnBzfubNkiU/39OkDrVvDihWxKxMRiSOxgf+jAw+EGTPg7rthwQKoXx9uvBE2b45dmYhI6Up84AOYwbnnwrJl0LYtDBgARx4JixfHrkxEpPRkReD/qGZNeOIJmDwZVq6EJk1g6FD49tvYlYmIpF9WBT74aP+MM2D5ct+/f801Hvxz5sSuTEQkvbIu8H9UtSo8+CBMmwZffOFTPP37w1dfxa5MRCQ9sjbwf9ShAyxd6rt4brnFF3VnzoxdlYhI6mV94APsvjvceSe8/DKULw9t2sB558Fnn8WuTEQkdRT4WzjqKHjrLRg0CO67z9+w9eSTsasSEUkNBf7PVK4MI0bAG2/AXntB586+yLtmTezKRERKRoFfhKZNYd48uO4638qZmwsTJqgZm4hkLgX+r6hQAQYPhjffhEMPhe7doWNH+PDD2JWJiGw/BX4x1K0Ls2b5ISuvvOLN2e68U83YRCSzKPCLKScHLrrIm7EdcQRceCG0agXvvhu7MhGR4lHgb6fateHZZ30Xz+LF0KABjBypZmwiUvYp8HeAGfTo4c3YOnaEyy+H5s19rl9EpKxS4JfAPvvAY4/5kYqrV0N+vi/yfvNN7MpERH5JgZ8Cp57qo/2zz4brr4fGjeH112NXJSLy3xT4KVKlCowfD888Axs3wu9+BxdfDBs2xK5MRMQp8FOsXTvfydO3L9x+O+TlwXPPxa5KRESBnxa77eZ79mfNgkqV/JdAz56wfn3sykQkmynw06hFC9+5c+WV3ns/N9cXeUVEYlDgp1mlSjB8OBQUwL77Qpcu/vGvf8WuTESyjQK/lDRq5B04R4yAp5/20f748WrGJiKlJyWBb2bjzGytmS3Z4rkqZva8mb1X+LhnKq6VySpU8F77b73l/Xh69vT5/ZUrY1cmItkgVSP88cDxP3vucmBmCKEOMLPwa8E7b778su/imT3bd/LcdpuasYlIeqUk8EMIrwA/34PSCbi/8PP7gc6puFZSlCvnDdiWLIGWLX3PfsuWsHx57MpEJKnSOYdfI4TwCUDh415pvFbG2n9/mD4dHngA3n7b5/qvvx6++y52ZSKSNNEXbc2st5kVmFnBunXrYpcThZkfrrJsGXTq5P14mjWDBQtiVyYiSZLOwF9jZvsAFD6u3doPhRDGhhDyQwj51atXT2M5ZV+NGvDww/D4475ts1kzuOIK+Prr2JWJSBKkM/CnAucUfn4O8GQar5UoJ5/so/1zzvFtnI0awauvxq5KRDJdqrZlTgJmA4ea2Soz6wWMAI4zs/eA4wq/lmLac0+49154/nnYtMkXdPv2hS+/jF2ZiGQqC2XonT/5+fmhoKAgdhllzoYNMGSI9+epVQvuuguO//kmWBHJWmY2P4SQv62fi75oK9u2664wejS89hrssgu0b+/TPf/+d+zKRCSTKPAzyBFHwMKFPtp/6CFvz/DII2rPICLFo8DPMDvtBNde683YatWC00+HU06BTz6JXZmIlHUK/AzVsCHMmQOjRvkpW3XrwrhxGu2LSNEU+BmsfHkYMMCbsTVsCL16Qdu28M9/xq5MRMoiBX4CHHIIvPgijBnjLZjz8nyR9/vvY1cmImWJAj8hypWDPn1g6VJo1Qr69fOD1Jcti12ZiJQVCvyEqVXLD1iZMAHeew8aN/ZF3k2bYlcmIrEp8BPIDLp189H9KafAVVfBYYf5zh4RyV4K/ATbay+YNAmefBI+/RSaN4eBA9WMTSRbKfCzwEkn+dx+r15www3QoIGfuCUi2UWBnyV+8xsYOxZmzvSjFFu1gj/+Eb74InZlIlJaFPhZ5thjYdEi6N/ffwHUqwfTpsWuSkRKgwI/C+2yC9x0E7z+OuyxB5xwApx9ts/zi0hyKfCzWPPmfozi0KF+0lbdujB5stoziCSVAj/LVawIw4bB/PlwwAFw5pnQuTOsXh27MhFJNQW+AFC/PsyeDTfe6Kds5ebC3XdrtC+SJAp8+Y+cHLjsMl/UbdIEeveG1q3hH/+IXZmIpIICX37h4IPhhRd8F8/8+T76v/lmNWMTyXQKfNkqMzjvPG/P0KaNj/yPPBKWLIldmYjsKAW+/KqaNb01w6RJ8P77PtUzbJiasYlkIgW+bJMZdO0Ky5fDaafB1VdD06Ywd27sykRkeyjwpdiqVYOJE7398mef+aHql10GGzfGrkxEikOBL9utY0dvxta7ty/m1q/vJ26JSNmmwJcdsvvufqTiSy/5aVvHHuu/AD7/PHZlIlIUBb6UyNFH+yHqAwbAvff6G7amTo1dlYhsjQJfSmznnWHUKD9AvWpV6NTJF3nXro1dmYhsSYEvKZOf78coXnstTJnio/2JE9WeQaSsUOBLSlWsCEOGwMKFUKeOt10+8UT46KPYlYmIAl/SIjcXXn0VRo/2HTz16sFf/+qnbYlIHAp8SZucHLjkEli8GJo18yMVjz0W3nsvdmUi2UmBL2l34IHecvnee+HNN/0Q9RtugM2bY1cmkl0U+FIqzOAPf/BmbMcfDwMH+jt1Fy2KXZlI9lDgS6nad194/HE/UvHDD70nz1VXwbffxq5MJPkU+FLqzLwJ27JlcNZZvo2zcWOYMyd2ZSLJlvbAN7OVZrbYzN40s4J0X08yR9WqcP/9MH06bNjg/fb79YOvvopdmUgyldYI/5gQQqMQQn4pXU8ySPv23oztggt8G2deHsyYEbsqkeTRlI6UCbvtBrffDq+8AhUqwHHHQa9e3oZZRFKjNAI/AM+Z2Xwz6/3zb5pZbzMrMLOCdevWlUI5Upa1bOnN2C6/3Kd7cnPhiSdiVyWSDKUR+C1CCE2A9sCFZnbUlt8MIYwNIeSHEPKrV69eCuVIWVe5Mvz5z96Mba+94OST4fTTYc2a2JWJZLa0B34I4ePCx7XAFKBZuq8pydC0KcybB8OH+7m6ubnw4INqxiayo9Ia+Ga2i5nt9uPnQFtgSTqvKclSoQJceaW/Q/fQQ+H3v4cOHXwPv4hsn3SP8GsAr5rZW8BcYFoI4Zk0X1MSqG5dmDULbr3VH+vVgzvuUDM2ke2R1sAPIbwfQmhY+FEvhDA8ndeTZMvJgYsugiVLvC1D375+4tY778SuTCQzaFumZJzateHZZ+G++zz8GzaEESPUjE1kWxT4kpHMoEcPWL4cOnaEK66A5s19rl9Etk6BLxlt773hscfg0Udh9Wo/ZnHwYPjmm9iViZQ9CnxJhFNP9WZs3bvD9ddDo0bw2muxqxIpWxT4khhVqvi8/rPP+gi/ZUu4+GJvzCYiCnxJoLZtfTG3b1/vz5OXB889F7sqkfgU+JJIu+760579SpWgXTvo2RPWr49dmUg8CnxJtBYtfOfO4MHeliE31xd5RbKRAl8Sr1IluO46KCjwIxa7dPGPf/0rdmUipUuBL1mjUSOYO9ffpPX0096uYfx4NWOT7KHAl6xSvjwMGuQ99/PyfF6/XTtYuTJ2ZSLpp8CXrHToofDyy96AbfZsD//bblMzNkk2Bb5krXLl/BzdpUt/2rPfsqW3axBJIgW+ZL399oPp0+GBB+Dtt32uf/hw+O672JWJpJYCXwRvxta9u7dn6NwZhgyBww6DBQtiVyaSOgp8kS3UqAF/+xtMmeJn6DZr5geqf/117MpESk6BL7IVnTv7aL9HDxg50qd5Zs2KXZVIySjwRYqw555wzz3w/POwaRMcdRRceCF8+WXsykR2jAJfZBvatPFmbJdeCmPG+Hm6f/977KpEtp8CX6QYdtkFbrnFe+zvuit06AC//z38+9+xKxMpvvKxC0iFq59ayrKPv4hdhmSJvIthtw/g+Q+hUX+oUweqV49dlWS63H13Z+iJ9dJ6DY3wRbaTmR+k3rQp7FTJF3eXLvV5fpGyLBEj/HT/VhQpyubNPtVz1VWwaie46Sb4wx/8l4JIWaMRvkgJlC8PAwbAokXQsCGcey4cdxy8/37sykR+SYEvkgJ16sCLL/ounrlzoX59GD0avv8+dmUiP1Hgi6RIuXLQp4/P57dqBf36+YlbS5fGrkzEKfBFUqxWLT9gZcIEWLECGjeGa6/Voq7Ep8AXSQMz6NbNd/Cceqov6ubnw7x5sSuTbKbAF0mjvfaCSZPgySf9TVqHHw4DB8LGjbErk2ykwBcpBSed5KP9Xr3ghht8R8/LL8euSrKNAl+klOyxB4wdCzNn+lGKrVrBH/8IX+hN4lJKFPgipezYY2HxYujf338B1KsH06bFrkqygQJfJIKdd/Z35b7+uo/8TzjBF3nXrYtdmSSZAl8koubN/RjFoUPhkUcgNxcmT4YQYlcmSZT2wDez483sHTNbYWaXp/t6IpmmYkUYNgzmz4cDDoAzz4ROnWD16tiVpcnEid59rlw5f5w4MXZFWSOtgW9mOcAdQHsgFzjTzHLTeU2RTFW/Psye7VM9M2b4aH/sWF/gTYyJE6F3b/jgA/9vzAcf+NcK/VKR7hF+M2BFCOH9EMImYDLQKc3XFMlYOTm+mLt4sbdfPv98aN3a37GbCIMH//JNCBs3+vOSdukO/JrAR1t8varwuf8ws95mVmBmBeu0YiUCwEEH+fbNsWN9jr9BAx/5Z3wztg8/3L7nJaXSHfhb6wr+X8tRIYSxIYT8EEJ+dR0bJPIfZnDeef6GrTZt4E9/giOO8NF/xtpvv+17XlIq3YG/Cqi1xde/BT5O8zVFEqVmTW/NMGkSrFwJTZr4rp5vv41d2Q4YPtz3pG5p5539eUm7dAf+PKCOmR1gZhWBrsDUNF9TJHHMoGtXH+2fcQZcc43P8b/xRuzKtlO3bj5Ptf/+flP77+9fd+sWu7KskNbADyFsBvoCzwLLgYdDCOoOLrKDqlXztstPPw2ff+5TPP37w1dfxa5sO3Tr5v9V+eEHf1TYl5q078MPIUwPIRwSQjgohKD/t4mkQMeOfrDK+ef7mboNGsALL8SuSso6vdNWJEPtvrsfqfjSS/4eptatfZH3s89iVyZllQJfJMMdfbQfoj5wIIwb583YpmqlTLZCgS+SAJUrw8iRvohbrZq3ZujaFdaujV2ZlCUKfJEEyc+HggI/Q3fKFG/PMHGimrGJU+CLJEyFCjBkCCxcCHXqwNlnw4knwkcfbfvPSrIp8EUSKjcXXn0VRo+GF1/0uf2//jVhzdhkuyjwRRIsJwcuucTbMTRr5kcqHnMMvPde7MokBgW+SBY48EB4/nm45x546y3ftz9qFGzeHLsyKU0KfJEsYQa9enl7hnbtYNAgOPxw/wUg2UGBL5Jl9t3Xd/A8/LAv5Obnw//+b4Y2Y5PtosAXyUJmcNppPto/6yy47jpo3NhP3JLkUuCLZLGqVeH++2H6dNiwAVq0gEsvzbBmbFJsCnwRoX17b8Z2wQXwl79AXp6fqyvJosAXEQB22w1uvx1eecXfvHXccb7I+3//F7sySRUFvoj8l5YtfefO5Zf7dE+9evDEE7GrklRQ4IvIL1SuDH/+M8ydCzVqwMknw+mnw5o1sSuTklDgi0iRmjTx0B8+3M/VrVsXHnhAzdgylQJfRH5VhQpw5ZU+zVO3LpxzDnToAB98ELsy2V4KfBEplv/5H5g1C2691R/z8uCOO9SMLZMo8EWk2MqVg4sugiVL4MgjoW9fP3HrnXdiVybFocAXke1WuzY88wyMH+/79xs2hBEj4LvvYlcmv0aBLyI7xMzn85ctgxNOgCuugObN/eAVKZsU+CJSInvvDY8+6h8ffwyHHeaLvN98E7sy+TkFvoikxKmn+mi/e3ffw9+oEbz2WuyqZEsKfBFJmSpV4L774NlnfYTfsqUv8n75ZezKBBT4IpIGbdv6Tp6+fX3rZl6e/xKQuBT4IpIWu+760579nXeG44+HHj1g/frYlWUvBb6IpFWLFr5zZ/BgmDABcnPhscdiV5WdFPgiknaVKvmpWgUFfsRily6+yPvJJ7Eryy4KfBEpNY0aeTO2ESNg2jQf7Y8fr2ZspUWBLyKlqnx5GDQIFi2C+vWhZ09o1w5WroxdWfIp8EUkikMOgZde8l08s2f7Tp7bblMztnRS4ItINOXK+Tm6S5f6nv2LL/bH5ctjV5ZMCnwRiW6//WD6dD9c5e23fa5/+HA1Y0u1tAW+mQ0zs9Vm9mbhR4d0XUtEMp+Zt2VYtgw6d4YhQ7wvz4IFsStLjnSP8G8JITQq/Jie5muJSALUqAF/+xtMmQJr10KzZn6g+tdfx64s82lKR0TKpM6dfbTfoweMHOnTPLNmxa4qs6U78Pua2SIzG2dme6b5WiKSML/5DdxzD8yY4fP5Rx0FF14IX3wRu7LMVKLAN7MZZrZkKx+dgDHAQUAj4BPgpiL+jt5mVmBmBevWrStJOSKSUK1bw+LF0K8fjBnjWzj//vfYVWUeC6XwFjczqw08HULI+7Wfy8/PDwUFBWmvR0Qy15w50KvXT733b7kFqlaNXVVcZjY/hJC/rZ9L5y6dfbb48mRgSbquJSLZ4/DDfefOVVfBpElQty48/LDaMxRHOufwR5nZYjNbBBwD9EvjtUQki+y0E1x9NcyfD/vvD2ecAaec4kcsStHSFvghhO4hhPohhAYhhJNCCOqLJyIp1aCBt2W44QZ45hlvxnbvvRrtF0XbMkUko5UvD3/6ky/qNmoE554LbdrA++/HrqzsUeCLSCIcfDC88ALcdRfMm+edOEePhu+/j11Z2aHAF5HEKFcOevf2HTzHHOPbOFu08OZsosAXkQT67W/hqafgoYfgH/+Axo3h2mth06bYlcWlwBeRRDKDM8/00X6XLr6NMz/fp3uylQJfRBKtenUf6U+dCuvX+z7+AQNg48bYlZU+Bb6IZIUTT/S5/F694MYboWFDP3ErmyjwRSRr7LEHjB3ru3lC8IXdPn3g889jV1Y6FPgiknWOOcYPUb/sMrj7bqhXD6ZNi11V+inwRSQr7byzT+3Mng177gknnADdukGSm/Yq8EUkqzVr5j15hg2DRx7x9gyTJiWzPYMCX0SyXsWKMHSod+E88EA46yw46SRYtSp2ZamlwBcRKZSXB6+/DjffDDNn+tz+2LHwww+xK0sNBb6IyBZycrwlw+LF0LQpnH++n7i1YkXsykpOgS8ishUHHeSj/Lvv9qme+vV9kXfz5tiV7TgFvohIEcy83fKyZdC2rb9D98gjffSfiRT4IiLbULMmPPEETJ4MK1dCkya+yPvtt7Er2z4KfBGRYjDzoxSXLYOuXeGaa3yO/403YldWfAp8EZHtUK0aPPggPP20t2Q44gjo3x+++ip2ZdumwBcR2QEdO3oztj594JZb/HzdF16IXdWvU+CLiOyg3XeHO+/0rpvlyvn2zfPOg88+i13Z1inwRURK6OijvRnbwIEwbpy3Z3jyydhV/ZICX0QkBSpXhpEjfRG3enXo3NkXd9eujV3ZTxT4IiIplJ8PBQV+hu6UKVC3LkyYUDaasSnwRURSrEIFGDIEFi6EQw6B7t29/fJHH8WtS4EvIpImubnw6qvwl7/4wm5uLowZE68ZmwJfRCSNcnLg4othyRI/QP2CC6BVK3j33dKvRYEvIlIKDjgAnnvOd/EsXuyHqI8aVbrN2BT4IiKlxAx69vT2DO3bw6BB0Lw5vPVW6VxfgS8iUsr22Qcee8yPVFy1ynf2jB6d/usq8EVEIjCDLl1g+XI/PP2gg9J/zfLpv4SIiBSlShUYP750rqURvohIllDgi4hkiRIFvpmdZmZLzewHM8v/2feuMLMVZvaOmbUrWZkiIlJSJZ3DXwKcAty15ZNmlgt0BeoB+wIzzOyQEML3JbyeiIjsoBKN8EMIy0MI72zlW52AySGEb0MI/wRWAM1Kci0RESmZdM3h1wS2bBO0qvA5ERGJZJtTOmY2A9h7K98aHEIoqsW/beW5rTYHNbPeQG+A/fbbb1vliIjIDtpm4IcQ2uzA37sKqLXF178FPi7i7x8LjAXIz88vAx2jRUSSKV1vvJoKPGRmN+OLtnWAudv6Q/Pnz//UzD7YwWtWAz7dwT+bKZJ+j0m/P0j+Per+4ti/OD9UosA3s5OB24DqwDQzezOE0C6EsNTMHgaWAZuBC4uzQyeEUL0EtRSEEPK3/ZOZK+n3mPT7g+Tfo+6vbCtR4IcQpgBTivjecGB4Sf5+ERFJHb3TVkQkSyQp8MfGLqAUJP0ek35/kPx71P2VYRbKwlHqIiKSdkka4YuIyK9IROCb2fGFTdpWmNnlsetJNTNbaWaLzexNMyuIXU8qmNk4M1trZku2eK6KmT1vZu8VPu4Zs8aSKOL+hpnZ6sLX8U0z6xCzxpIws1pm9qKZLS9soHhJ4fNJeg2LuseMfR0zfkrHzHKAd4Hj8Dd8zQPODCEsi1pYCpnZSiA/hFAW9//uEDM7CtgAPBBCyCt8bhSwPoQwovAX954hhEEx69xRRdzfMGBDCOHGmLWlgpntA+wTQlhgZrsB84HOQA+S8xoWdY+nk6GvYxJG+M2AFSGE90MIm4DJePM2KcNCCK8A63/2dCfg/sLP78f/cWWkIu4vMUIIn4QQFhR+/iWwHO+XlaTXsKh7zFhJCPxsaNQWgOfMbH5h76GkqhFC+AT8HxuwV+R60qGvmS0qnPLJ2OmOLZlZbaAx8AYJfQ1/do+Qoa9jEgK/2I3aMliLEEIToD1wYeF0gWSeMcBBQCPgE+CmuOWUnJntCjwGXBpC+CJ2PemwlXvM2NcxCYFf7EZtmSqE8HHh41r8nc1JPVtgTeG86Y/zp2sj15NSIYQ1IYTvQwg/AHeT4a+jmVXAg3BiCOHxwqcT9Rpu7R4z+XVMQuDPA+qY2QFmVhE/aWtq5JpSxsx2KVwwwsx2AdriJ40l0VTgnMLPzwGKar+dkX4MwkInk8Gvo5kZcC+wPIRw8xbfSsxrWNQ9ZvLrmPG7dAAKt0WNBnKAcYV9fBLBzA7kp35F5YGHknB/ZjYJaIV3H1wDDAWeAB4G9gM+BE4LIWTkwmcR99cKnwYIwErg/B/nuzONmf0OmAUsBn4ofPpKfI47Ka9hUfd4Jhn6OiYi8EVEZNuSMKUjIiLFoMAXEckSCnwRkSyhwBcRyRIKfBGRLKHAFxHJEgp8EZEsocAXEckS/w+14zFxGRcNkgAAAABJRU5ErkJggg==\n", "text/plain": [ "
" ] }, "metadata": { "needs_background": "light" }, "output_type": "display_data" }, { "data": { "image/png": "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\n", "text/plain": [ "
" ] }, "metadata": { "needs_background": "light" }, "output_type": "display_data" }, { "data": { "image/png": "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\n", "text/plain": [ "
" ] }, "metadata": { "needs_background": "light" }, "output_type": "display_data" }, { "data": { "image/png": "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\n", "text/plain": [ "
" ] }, "metadata": { "needs_background": "light" }, "output_type": "display_data" }, { "data": { "image/png": "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\n", "text/plain": [ "
" ] }, "metadata": { "needs_background": "light" }, "output_type": "display_data" }, { "data": { "image/png": "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\n", "text/plain": [ "
" ] }, "metadata": { "needs_background": "light" }, "output_type": "display_data" }, { "data": { "image/png": "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\n", "text/plain": [ "
" ] }, "metadata": { "needs_background": "light" }, "output_type": "display_data" }, { "data": { "image/png": "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\n", "text/plain": [ "
" ] }, "metadata": { "needs_background": "light" }, "output_type": "display_data" }, { "data": { "image/png": "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\n", "text/plain": [ "
" ] }, "metadata": { "needs_background": "light" }, "output_type": "display_data" }, { "data": { "image/png": "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\n", "text/plain": [ "
" ] }, "metadata": { "needs_background": "light" }, "output_type": "display_data" }, { "data": { "image/png": "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\n", "text/plain": [ "
" ] }, "metadata": { "needs_background": "light" }, "output_type": "display_data" }, { "data": { "image/png": "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\n", "text/plain": [ "
" ] }, "metadata": { "needs_background": "light" }, "output_type": "display_data" }, { "data": { "image/png": "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\n", "text/plain": [ "
" ] }, "metadata": { "needs_background": "light" }, "output_type": "display_data" }, { "data": { "image/png": "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\n", "text/plain": [ "
" ] }, "metadata": { "needs_background": "light" }, "output_type": "display_data" }, { "data": { "image/png": "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\n", "text/plain": [ "
" ] }, "metadata": { "needs_background": "light" }, "output_type": "display_data" }, { "data": { "image/png": "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\n", "text/plain": [ "
" ] }, "metadata": { "needs_background": "light" }, "output_type": "display_data" }, { "data": { "image/png": "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\n", "text/plain": [ "
" ] }, "metadata": { "needs_background": "light" }, "output_type": "display_data" }, { "data": { "image/png": "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\n", "text/plain": [ "
" ] }, "metadata": { "needs_background": "light" }, "output_type": "display_data" }, { "data": { "image/png": "iVBORw0KGgoAAAANSUhEUgAAAXwAAAD8CAYAAAB0IB+mAAAABHNCSVQICAgIfAhkiAAAAAlwSFlzAAALEgAACxIB0t1+/AAAADl0RVh0U29mdHdhcmUAbWF0cGxvdGxpYiB2ZXJzaW9uIDMuMC4zLCBodHRwOi8vbWF0cGxvdGxpYi5vcmcvnQurowAAHtxJREFUeJzt3Xu8VXP+x/HX53RV7jlFKmEav+7FVmhKpUgZlXtyzyQjI1LCSMSgUD/jUsklJPWTJCLqFzWUnKTrGRPGJZKDoQipPr8/vtvPwel69t5rn73ez8djP87Z6+xan8d66H2Wtb7r8zF3R0REcl9e1AWIiEhmKPBFRGJCgS8iEhMKfBGRmFDgi4jEhAJfRCQmFPgiIjGhwBcRiQkFvohITJSPuoDi9tlnH69bt27UZYiIlCkLFy783N3zt/W5rAr8unXrUlBQEHUZIiJlipl9sD2f0yUdEZGYUOCLiMSEAl9EJCYU+CIiMaHAFxGJCQW+iEhMKPBFRGIiJwL/++/hsstg9eqoKxERyV45EfgLFsCYMdCgATz0EGhMr4jIb+VE4LdpA4sXQ+PGcMEFcOyx8O9/R12ViEh2yYnAB/j97+Hll+Hee2H+fGjUCO66CzZtiroyEZHskDOBD5CXBxdfDMuXw9FHh+v6rVtDYWHUlYmIRC+nAv8nderAc8/Bo4/Cv/4FzZrBTTfBjz9GXZmISHRyMvABzOCss2DFCujeHa67DhIJWLgw6spERKKRs4H/k+rV4Ykn4OmnoagIWrSAq66C776LujIRkczK+cD/Sdeu4Wy/Vy8YNgyaNIFXXom6KhGRzIlN4APsuWdYrz9rFmzeDG3bhpu8a9dGXZmISPrFKvB/0r49LFkCl18efgE0bAjTp0ddlYhIesUy8AGqVoU774TXXoPdd4cuXcJN3s8/j7oyEZH0iG3g/6RlS3jzTbj+epg4MbRnmDhR7RlEJPeUOvDNrLaZzTazQjNbbmaXJbfvbWYvmdnK5Ne9Sl9uelSqBEOGhOA/4AA44wzo1g0+/jjqykREUicVZ/gbgf7uXh84ArjEzBoAg4BZ7l4PmJV8n9UaN4Z58+D22+HFF8O1/bFjdbYvIrmh1IHv7qvd/c3k9+uAQmB/oCswLvmxcUC30u4rE8qXh/79YelSaN4c/vQn6NAB3nsv6spEREonpdfwzawu0Bx4Hajh7qsh/FIAqqdyX+n2u9+F5ZujR8Mbb4RmbCNGqBmbiJRdKQt8M9sVmAz0c/ftXtluZr3NrMDMCoqKilJVTkrk5UHv3uGBrfbt4YoroFUrWLYs6spERHZcSgLfzCoQwn68uz+V3LzGzPZL/nw/4LOS/qy7j3H3hLsn8vPzU1FOytWqBdOmweOPw7vvwqGHwg03wIYNUVcmIrL9UrFKx4AHgEJ3v7PYj54Bzk1+fy4wtbT7ipIZ9OgRzvZPOSWs6jnssHC5R0SkLEjFGX4r4GygvZm9lXx1Bm4FOprZSqBj8n2Zl58fzvSfeQb+8x844gi48kpYvz7qykREts48i9YcJhIJLygoiLqM7fb116Hz5ujRcPDBcP/90K5d1FWJSNyY2UJ3T2zrc7F/0rY09tgDRo2C2bPD+/bt4aKLwi8CEZFso8BPgbZtQzO2K68MD2o1aBBu8oqIZBMFfopUqQLDh4cB6tWqwYknwplnhqErIiLZQIGfYocfDgUFYdnmk09C/frhJm8W3SoRkZhS4KdBxYoweDAsWhSe2O3ZM5zxr1oVdWUiEmcK/DRq2BBefTX03Z81K1zbHz06TNsSEck0BX6alSsXJmstXRou9/TpE1bzrFwZdWUiEjcK/Aw5+GCYOTOs4nnrrTBE/fbbYePGqCsTkbhQ4GeQGfTqFdozHHccDBgARx4ZlnSKiKSbAj8CNWvClClhlOIHH4SePIMHww8/RF2ZiOQyBX5EzOC006CwMIxUHDo0dOGcPz/qykQkVynwI1atGjz6KEyfDuvWwVFHhZu8334bdWUikmsU+Fni+OPDYJWLL4aRI8N83Vmzoq5KRHKJAj+L7L473HMPvPJKmK3boQNceCF89VXUlYlILlDgZ6E2bWDxYhg4EB56KDywNbVMj48RkWygwM9Su+wCt90GCxaEoSvdusHpp8OaNVFXJiJllQI/yx12WGjGdtNN8PTT4Wz/0UfVjE1Edlyqhpg/aGafmdmyYtuGmNnHvxp7KDuhQgW49trwhO4hh8A550CXLvDhh1FXJiJlSarO8B8GOpWwfYS7N0u+pqdoX7FVvz7MnQt33QVz5oTmbPfeq2ZsIrJ9UhL47j4H+DIVf5dsXblycOmlYQnnkUfCJZeEiVv/+lfUlYlItkv3Nfy+ZrYkeclnrzTvK1bq1oUZM8IqnqVLQzO2225TMzYR2bJ0Bv59wMFAM2A1cEdJHzKz3mZWYGYFRZoHuEPM4LzzQjO2zp1h0CBo2TJc6xcR+bW0Bb67r3H3Te6+GbgfaLGFz41x94S7J/Lz89NVTk7bbz946qkwUvHjjyGRCDd5v/8+6spEJJukLfDNbL9ib7sDy7b0WUmNk08OZ/s9e8Lf/gbNm8Nrr0VdlYhki1Qty5wAzAMOMbNVZtYLGGZmS81sCdAOuDwV+5Kt23tvGDcOXngB1q+HP/wB/vIX+OabqCsTkaiZZ9ETPIlEwgsKCqIuI2esWwfXXBP689SpA2PGwLHHRl2ViKSamS1098S2PqcnbXPYbrvB3/8e1u5XrhymbJ1/PnypBbQisaTAj4FWrcLKnWuuCW0ZGjSAyZOjrkpEMk2BHxOVK8PNN4e+PDVrwimnhNenn0ZdmYhkigI/Zpo1g9dfh1tugWefDWf7Dz+sZmwicaDAj6EKFcJDWosXh348558PnTrB++9HXZmIpJMCP8YOOSRM17r77rBev1GjcJNXzdhEcpMCP+by8kIDtmXLfl6z36YN/POfUVcmIqmmwBcADjgAnn8+PLS1YgU0bRqe1v3xx6grE5FUUeDL/zMLw1UKC6Fr19CPp0ULePPNqCsTkVRQ4Mtv1KgBkyaFhmyffhpC/+qr4bvvoq5MREpDgS9b1L17uLxz3nlw661hSec//hF1VSKysxT4slV77QVjx8JLL8GGDdC6NfTtG/r0iEjZosCX7dKhQ1jJ069fmKPbsGG4ySsiZYcCX7Zb1aowYgS8+irsumuYsnXOOfDFF1FXJiLbQ4EvO+zII2HRIvjrX2HChNCe4X/+R+0ZRLKdAl92SqVKMHRoaMZWuzacdhqcdBKsXh11ZSKyJQp8KZWmTWH+fBg2LEzZql8fHnxQZ/si2ShVIw4fNLPPzGxZsW17m9lLZrYy+XWvVOxLsk/58jBgACxZEn4B9OoFHTvCe+9FXZmIFJeqM/yHgU6/2jYImOXu9YBZyfeSw+rVg9mz4b77YMECaNwYRo6ETZuirkxEIEWB7+5zgF8PzusKjEt+Pw7olop9SXbLy4M+fWD5cmjbFi6/PDRlW7Ei6spEJJ3X8Gu4+2qA5NfqadyXZJnatcOAlcceg5UroXnzcJN3w4aoKxOJr8hv2ppZbzMrMLOCoqKiqMuRFDKDnj3D2f1JJ8HgwXD44WFlj4hkXjoDf42Z7QeQ/PpZSR9y9zHunnD3RH5+fhrLkahUrx7W60+dCp9/Di1bwsCBasYmkmnpDPxngHOT358LTE3jvqQMOPHEcG2/Vy8YPhyaNAkTt0QkM1K1LHMCMA84xMxWmVkv4Fago5mtBDom30vM7bknjBkDs2aFUYpt28LFF8PatVFXJpL7zLPoCZlEIuEFusAbG99+G67rjxwJNWvCqFHQpUvUVYmUPWa20N0T2/pc5DdtJb6qVoU77ggD1HffHU44Ac46K1znF5HUU+BL5Fq2DGMUr78eJk4M7RmeeELtGURSTYEvWaFSJRgyJAT/gQdCjx5hru7HH0ddmUjuUOBLVmncGObNg9tvh5kzQ+vl++/X2b5IKijwJeuUKwf9+4dmbIceCr17wzHHwLvvRl2ZSNmmwJes9bvfheWbo0fDwoXh7P/OO9WMTWRnKfAlq+XlhTP85cvDXN3+/eGoo8J8XRHZMQp8KRNq1QqtGSZMCH32Dz0UbrhBzdhEdoQCX8oMMzjjDCgshFNPDat6Djss9N4XkW1T4EuZs88+MH48TJsG//lPGKp+5ZWwfn3UlYlkNwW+lFknnBCu7f/pT+GJ3caNw8QtESmZAl/KtD32CD14Zs8Ol3zat4eLLoKvv466MpHso8CXnNC2bVi3P2AAjB0bHtiaNi3qqkSyiwJfckaVKjBsGLz+OlSrFvrvn3kmaJCaSKDAl5yTSIQxijfeCE8+GZqxPf642jOIKPAlJ1WsCNddB4sWhSd2e/aEP/4RPvoo6spEoqPAl5zWsCG8+iqMGBFu7DZsGFo1bN4cdWUimZf2wDez981sqZm9ZWYaZyUZV64c9OsHS5dCixbQp09YzbNyZdSViWRWps7w27l7s+0ZwSWSLgcdBC+9BA88AG+9FYaoDx8OGzdGXZlIZuiSjsSKGVxwAaxYAccdBwMHhid1lyyJujKR9MtE4DvwopktNLPev/6hmfU2swIzKyjS+jnJkJo1YcqUMFLxww9DT57Bg+GHH6KuTCR9MhH4rdz9UOB44BIza1P8h+4+xt0T7p7Iz8/PQDkigRmcdlo42+/RA4YODV0458+PujKR9Eh74Lv7J8mvnwFTgBbp3qfIjqhWDR55BKZPh3XrQr/9yy+Hb7+NujKR1Epr4JtZVTPb7afvgWMBja6QrHT88aEZ25//DCNHhmZsM2dGXZVI6qT7DL8G8A8zWwwsAJ5z9xfSvE+RnbbbbnD33TBnDpQvDx07Qq9e8NVXUVcmUnppDXx3f8/dmyZfDd395nTuTyRVWreGxYth0CAYNy40Y3v66airEikdLcsU2YJddoFbbgnN2KpXh+7dw03eNWuirkxk55SPuoBUuWHaclZ8sjbqMiRH/b4P7PIRzH0fDhsQ+vPUqBF1VZJLGtTcnev/2DCt+9AZvsh2MIM6dUInzipV4J//DK0atG5fypKcOcNP929GkZ9s2gT33gtXXw0fGdx2W+jPk6fTJ8ly+k9UZAeVKweXXgrLloW2DJdcAkcfDW+/HXVlIlunwBfZSXXrwowZ8NBDIfybNoVbb1UzNsleCnyRUjCD886DwkLo0iVc5mnRInTjFMk2CnyRFNh3X5g8OYxU/OSTcHP32mvh+++jrkzkZwp8kRQ6+eTQjO3ss+Fvf4NmzcLELZFsoMAXSbG99w7X9WfMCGf4rVvDX/4C33wTdWUSdwp8kTQ59thwM7dv39Cfp1EjePHFqKuSOFPgi6TRrrvCXXfB3LlQuXKYsnX++fDll1FXJnGkwBfJgFatwsqda66BRx8NzdgmT466KokbBb5IhlSuDDffDAUFYcTiKaeE16efRl2ZxIUCXyTDmjWDBQvCQ1rPPgv168PDD4N71JVJrlPgi0SgfHm46qrQc79Ro3Bd/7jj4P33o65McpkCXyRChxwCr7wC99wD8+aF8P/732Hz5qgrk1yU9sA3s05m9raZvWNmg9K9P5GyJi8vzNFdtuznNfutW4d2DbEzfnxoUpSXF76OHx91RTkl3UPMywH3AMcDDYAeZtYgnfsUKasOOACmT4dHHgn99ps1C0/r/vhj1JVlyPjx0Ls3fPBBuKHxwQfhvUI/ZdJ9ht8CeCc523YD8ATQNc37FCmzzEJbhhUroFu30I+nRQt4882oK8uAa6+F9et/uW39+rBdUiLdgb8/8FGx96uS2/6fmfU2swIzKygqKkpzOSJlQ40aMHEiTJkSZui2aBE6cX73XdSVpdGHH+7Ydtlh6Q58K2HbLxafufsYd0+4eyI/Pz/N5YiULd26wfLloQXzrbeGyzxz50ZdVZrUqbNj22WHpTvwVwG1i72vBXyS5n2K5JS99oKxY+Gll2DDBmjTJvTnWbcu6spS7Oabw8Dg4qpUCdslJdId+G8A9czsQDOrCJwBPJPmfYrkpA4dwkqefv3CTN2GDeH556OuKoV69oQxY8Lda7PwdcyYsF1SIq2B7+4bgb7ADKAQmOTuy9O5T5FcVrUqjBgReuzvuit07gznnANffBF1ZSnSs2d4+mzz5vBVYZ9SaV+H7+7T3f337n6wu+v/zURS4MgjYdEiuO46mDAhtGeYNEntGWTr9KStSBlVqRLceCMsXBjua55+OnTvHkYsipREgS9SxjVpAvPnw7BhYcpWgwbwwAM625ffUuCL5IDy5WHAAFiyBJo2hQsvhI4d4b33oq5MsokCXySH1KsHs2fDffeFFsyNG8PIkbBpU9SVSTZQ4IvkmLw86NMnPLDVti1cfjn84Q+hXYPEmwJfJEfVrh0GrDz2GKxcCc2bw9Ch4eEtiScFvkgOMwtL2VesgJNOgsGD4fDDw5hFiR8FvkgMVK8e1utPnQqffw4tW8LAgTnejE1+Q4EvEiMnnhjO9nv1guHDw5LOV16JuirJFAW+SMzssUdoUTNrVuhg0LYtXHwxrF0bdWWSbgp8kZhq3x6WLoUrrgi/ABo2hOeei7oqSScFvkiMVakCd9wBr70WzvxPOAHOOitc55fco8AXEVq2DGMUhwwJTdjq14cnnlB7hlyjwBcRACpWhOuvD8F/0EHQo0eYuPXxx1FXJqmiwBeRX2jUKFziueOOMGWrQQO4/36d7ecCBb6I/Ea5cuFm7tKlcNhh0Ls3HHMMvPtu1JVJaSjwRWSLDj44LN8cPTr03W/cOJz5qxlb2ZS2wDezIWb2sZm9lXx1Tte+RCR9zMIZ/ooVYa7ulVeGiVtLl0ZdmeyodJ/hj3D3ZsnX9DTvS0TSaP/9Q2uGCRPg3/8Ol3qGDFEztrJEl3REZLuZwRlnQGEhnHYa3HADHHpo6L0v2S/dgd/XzJaY2YNmtldJHzCz3mZWYGYFRUVFaS5HRFJhn31C2+Vnn4Wvvw6XePr3h/Xro65Mtsa8FGutzGwmsG8JP7oWmA98DjgwFNjP3S/Y2t+XSCS8QH1bRcqUtWvhqqtg1Kiwfn/sWGjXLuqq4sXMFrp7YlufK9UZvrt3cPdGJbymuvsad9/k7puB+4EWpdmXiGSn3XcPIxVffjlM22rfPtzk/frrqCuTX0vnKp39ir3tDixL175EJHpHHx2GqA8YAA88EB7YmjYt6qqkuHRewx9mZkvNbAnQDrg8jfsSkSywyy4wbBi8/jpUqxb67/foAbo9lx3SFvjufra7N3b3Ju5+oruvTte+RCS7JBJhjOKNN8JTT4VmbI8/rvYMUdOyTBFJi4oV4brrYNEiqFcvzNb94x/ho4+iriy+FPgiklYNGsA//gEjR8Ls2WHQyqhRYdqWZJYCX0TSrlw5uOwyWLYs9N6/+OKwmmflyqgrixcFvohkzIEHwosvhlU8ixeHIerDh8PGjVFXFg8KfBHJKDO44AJYvhw6dYKBA8OTukuWRF1Z7lPgi0gkatYMK3gmTYIPPwzN2AYPhh9+iLqy3KXAF5HImMGpp4bWy2eeCUOHQvPmMG9e1JXlJgW+iESuWjUYNw6mT4dvvoFWraBfP/j226gryy0KfBHJGscfH67t//nP8N//HebrzpwZdVW5Q4EvIlllt93g7rthzhyoUAE6doReveCrr6KurOxT4ItIVmrdOizdHDQoXO5p0ACefjrqqso2Bb6IZK1ddoFbbgkTtWrUgO7dw6StNWuirqxsUuCLSNb7aYzizTeHubr168Mjj6gZ245S4ItImVChAlxzTbjMU78+nHsudO4c1vDL9lHgi0iZ8l//BXPnwl13ha8NG8I996gZ2/ZQ4ItImZOXB5deGpqxHXUU9O0bJm69/XbUlWU3Bb6IlFl168ILL8DDD4f1+02bwq23wo8/Rl1ZdipV4JvZqWa23Mw2m1niVz+72szeMbO3zey40pUpIlIys3A9f8UK6NIFrr46tGBetCjqyrJPac/wlwEnAXOKbzSzBsAZQEOgE3CvmZUr5b5ERLZo331h8mR48kn45BM4/PBwk/f776OuLHuUKvDdvdDdS7pq1hV4wt1/cPd/A+8ALUqzLxGR7XHyyeFs/+yzwxr+Zs3g1Vejrio7pOsa/v5A8cmVq5LbfsPMeptZgZkVFGm0vYikwN57w0MPwYwZ4Qy/detwk3fduqgri9Y2A9/MZprZshJeXbf2x0rYVuIjEu4+xt0T7p7Iz8/f3rpFRLbp2GPDSp6+fcPSzUaNwi+BuNpm4Lt7B3dvVMJr6lb+2CqgdrH3tYBPSlusiMiO2nXXn9fsV6kSpmyddx58+WXUlWVeui7pPAOcYWaVzOxAoB6wIE37EhHZplatwsqda6+Fxx4LzdgmT466qswq7bLM7ma2CjgSeM7MZgC4+3JgErACeAG4xN03lbZYEZHSqFwZbroJCgrCiMVTTgk3eVevjrqyzDDPou5DiUTCCwoKoi5DRGJg40a4/XYYMiR05RwxIqznt5LuQGY5M1vo7oltfU5P2opILJUvH3rtL14MjRvD+efDccfB++9HXVn6KPBFJNYOOQRefjms4pk3L6zkuesu2JSDF6EV+CISe3l5YY7u8uXQpg1cdllYu19YGHVlqaXAFxFJqlMHnnsOHn00dN5s1iwMXcmVZmwKfBGRYszgrLPC2X23bvDXv0IiAQsXRl1Z6SnwRURKUL06TJwIU6ZAUVHowDloEHz3XdSV7TwFvojIVnTrFpqxnXce3HZb6Lk/d27UVe0cBb6IyDbsuSeMHQszZ4b1+23awCWXwNq1UVe2YxT4IiLb6ZhjYOlS6NcP7rsvLOF8/vmoq9p+CnwRkR1QtWp4Kve112C33aBzZzjnHPjii6gr2zYFvojITjjiCHjzTbjuOpgwAerXh0mTIIu61fyGAl9EZCdVqgQ33hiWbNapA6efDiedFEYsZiMFvohIKTVpAvPnw7Bh8MILofXyAw9k39m+Al9EJAXKl4cBA2DJkrB088ILoUMHeO+9qCv7mQJfRCSF6tWD2bNh1Ch4443QiXPEiOxoxqbAFxFJsbw8uOii8MBWu3ZwxRVh4tby5RHXFe3uRURyV61aMG0ajB8P77wDzZvDDTfAhg3R1FPaEYenmtlyM9tsZoli2+ua2Xdm9lbyNar0pYqIlD1mcOaZoRnbySeHCVuJRLjck2mlPcNfBpwEzCnhZ++6e7Pkq08p9yMiUqbl54f1+lOnhoe0jjgi3ORdvz5zNZQq8N290N3fTlUxIiK57sQTw7X9Cy8MM3WbNAkTtzIhndfwDzSzRWb2ipm13tKHzKy3mRWYWUFRUVEayxERyQ577AGjR8P//m94364d9O+f/v2W39YHzGwmsG8JP7rW3adu4Y+tBuq4+xdmdhjwtJk1dPff9JZz9zHAGIBEIpFljymIiKRPu3Zh3f7118OBB6Z/f9sMfHfvsKN/qbv/APyQ/H6hmb0L/B4o2OEKRURyWJUqMHx4ZvaVlks6ZpZvZuWS3x8E1AOy6HkzEZH4Ke2yzO5mtgo4EnjOzGYkf9QGWGJmi4EngT7u/mXpShURkdLY5iWdrXH3KcCUErZPBiaX5u8WEZHU0pO2IiIxocAXEYkJBb6ISEwo8EVEYkKBLyISE+ZZNIPLzIqAD0rxV+wDfJ6icso6HYtf0vH4mY7FL+XC8TjA3fO39aGsCvzSMrMCd09s+5O5T8fil3Q8fqZj8UtxOh66pCMiEhMKfBGRmMi1wB8TdQFZRMfil3Q8fqZj8UuxOR45dQ1fRES2LNfO8EVEZAtyIvDNrJOZvW1m75jZoKjryTQze9DMPjOzZcW27W1mL5nZyuTXvaKsMVPMrLaZzTazQjNbbmaXJbfH9XhUNrMFZrY4eTxuSG4/0MxeTx6PiWZWMepaM8XMyiWn8T2bfB+bY1HmAz/Zd/8e4HigAdDDzBpEW1XGPQx0+tW2QcAsd68HzEq+j4ONQH93rw8cAVyS/O8hrsfjB6C9uzcFmgGdzOwI4DZgRPJ4/AfoFWGNmXYZUFjsfWyORZkPfKAF8I67v+fuG4AngK4R15RR7j4H+PW8ga7AuOT344BuGS0qIu6+2t3fTH6/jvAPe3/iezzc3b9Jvq2QfDnQnjCrAmJ0PMysFtAFGJt8b8ToWORC4O8PfFTs/arktrir4e6rIYQgUD3iejLOzOoCzYHXifHxSF7CeAv4DHgJeBf4yt03Jj8Sp38zI4GBwObk+2rE6FjkQuBbCdu09CjmzGxXwhCefu6+Nup6ouTum9y9GVCL8H/E9Uv6WGaryjwzOwH4zN0XFt9cwkdz9liUauJVllgF1C72vhbwSUS1ZJM1Zrafu682s/0IZ3exYGYVCGE/3t2fSm6O7fH4ibt/ZWYvE+5t7Glm5ZNntnH5N9MKONHMOgOVgd0JZ/yxORa5cIb/BlAveae9InAG8EzENWWDZ4Bzk9+fC0yNsJaMSV6TfQAodPc7i/0orscj38z2TH6/C9CBcF9jNnBK8mOxOB7ufrW713L3uoSc+F9370mMjkVOPHiV/I09EigHPOjuN0dcUkaZ2QSgLaHr3xrgeuBpYBJQB/gQODUOg+TN7A/AXGApP1+nvYZwHT+Ox6MJ4UZkOcIJ3iR3v9HMDiIscNgbWASc5e4/RFdpZplZW+BKdz8hTsciJwJfRES2LRcu6YiIyHZQ4IuIxIQCX0QkJhT4IiIxocAXEYkJBb6ISEwo8EVEYkKBLyISE/8HjlY8D6VCOPUAAAAASUVORK5CYII=\n", "text/plain": [ "
" ] }, "metadata": { "needs_background": "light" }, "output_type": "display_data" }, { "data": { "image/png": "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\n", "text/plain": [ "
" ] }, "metadata": { "needs_background": "light" }, "output_type": "display_data" }, { "data": { "image/png": "iVBORw0KGgoAAAANSUhEUgAAAXwAAAD8CAYAAAB0IB+mAAAABHNCSVQICAgIfAhkiAAAAAlwSFlzAAALEgAACxIB0t1+/AAAADl0RVh0U29mdHdhcmUAbWF0cGxvdGxpYiB2ZXJzaW9uIDMuMC4zLCBodHRwOi8vbWF0cGxvdGxpYi5vcmcvnQurowAAH6tJREFUeJzt3Xu8VXP+x/HX51QkyiWnXIrcp3uxdUFRohKS+71o5FIjiaGfSbk3CY37HKKQTCOX0EQSYVROuqgOE8YtTY57JkR9fn98t5mjOenU3uusffZ6Px+P/Th7r7PPWZ8Wvfdqfb/r8zV3R0RE8l9B3AWIiEjlUOCLiCSEAl9EJCEU+CIiCaHAFxFJCAW+iEhCKPBFRBJCgS8ikhAKfBGRhKgedwFlbb/99t6oUaO4yxARqVLmzp37mbsXbuh9ORX4jRo1ori4OO4yRESqFDP7oCLvy/iSjpk1NLMZZlZiZovNbGB6+3AzW2Zm89OPIzLdl4iIbLpsnOH/BAx29zfMrDYw18ympb93i7uPysI+REQkQxkHvrsvB5ann680sxJg50x/r4iIZFdWZ+mYWSOgNTA7vWmAmS00s/vMbNv1/Ew/Mys2s+LS0tJsliMiImVkLfDNbCtgEnCRu38D3AXsAbQi/AvgpvJ+zt2L3D3l7qnCwg0OMouIyCbKSuCbWQ1C2I9398cA3H2Fu69x97XAPUCbbOxLREQ2TTZm6RgwBihx95vLbN+xzNt6AYsy3ZeIiGy6bJzhHwicAXReZwrmSDN708wWAp2AQVnYV7m+/x4GDoTly6Pag4hI1ZeNWTqvAFbOt6Zk+rsras4cKCqCBx6Am2+GPn3AyqtIRCTB8qKXTseOsGABNG8OZ58Nhx8O//xn3FWJiOSWvAh8gL33hhdfhDvvhNmzoVkzuPVWWLMm7spERHJD3gQ+QEEBnH8+LF4MBx8crut36AAlJXFXJiISv7wK/J81bAjPPAMPPgj/+Ae0agXXXgs//hh3ZSIi8cnLwIcwaHv66bBkCfTqBUOHQioFc+fGXZmISDzyNvB/Vq8ePPIIPPEElJZCmzZw2WXw3XdxVyYiUrnyPvB/1rNnONvv2xdGjoSWLWHmzLirEhGpPIkJfIBttgnz9adPD7N3Dj44DPJ+803clYmIRC9Rgf+zzp1h4UK4+OLwAdC0KUyptNvERETikcjAB9hyS7jpJvj736FOHejRIwzyfvZZ3JWJiEQjsYH/s7Zt4Y03YNgwmDgRmjSBv/wF3OOuTEQkuxIf+ACbbw7Dh4cpm40awcknwzHHwLJlcVcmIpI9CvwymjeH116DUaNg2rRwtn/PPTrbF5H8oMBfR7VqMHhwGNTdd1/o1w8OPRTefTfuykREMqPAX4899wzTN//853Cpp3nz0HpZzdhEpKrKxopXDc1shpmVmNliMxuY3r6dmU0zs6Xpr+UuYp7LCgrCGf7ixeEsf/BgOOAAWKS1u0SkCsrGGf5PwGB3bwy0A/qbWRPgcmC6u+8FTE+/rpIaNIDJk2HCBHjvvXCp56qrYPXquCsTEam4jAPf3Ze7+xvp5yuBEmBnoCcwLv22ccAxme4rTmZh9k5JCZxwQpjVs99+8PrrcVcmIlIxWb2Gb2aNgNbAbKC+uy+H8KEA1MvmvuKy/fYwfjw89RR8+SW0aweXXAKrVsVdmYjIr8ta4JvZVsAk4CJ3r3B3GjPrZ2bFZlZcWlqarXIid+SR4dr+OeeEO3ZbtIAZM+KuSkRk/bIS+GZWgxD24939sfTmFWa2Y/r7OwKflvez7l7k7il3TxUWFmajnEqz9dZw993/DfrOneHcc+Hrr+OtS0SkPNmYpWPAGKDE3W8u863JQO/0897Ak5nuK1cdckiYt3/JJXDvveGGraeeirsqEZFfysYZ/oHAGUBnM5uffhwBjAAOM7OlwGHp13mrVi248UaYNQvq1oWjj4ZTTw2LroiI5ALzHOobkEqlvLi4OO4yMrZ6NYwYEdbRrVMHbr0VTjklzPQREck2M5vr7qkNvU932kZgs83gyith3jzYYw847TQ46ij46KO4KxORJFPgR6hp09Bv/+ab4YUXwus//xnWro27MhFJIgV+xKpVg0GDQjuG/feH884Ls3mWLo27MhFJGgV+Jdl9d3j++TCLZ/78MG9/1Cj46ae4KxORpFDgVyIz6NsXliyBrl3h0kuhffswpVNEJGoK/BjstBM8/nhYSvGDD0JPnmHD4Icf4q5MRPKZAj8mZnDiiaEZ2ymnwNVXhy6cs2bFXZmI5CsFfszq1oUHHoApU2DlytBv/+KL4d//jrsyEck3Cvwc0b17mMlz/vlwyy1hha3p0+OuSkTyiQI/h9SpA3fcAS+9BNWrQ5cuoRvnV1/FXZmI5AMFfg7q2BEWLIDLLoP77w/N2J7M29ZzIlJZFPg5aostQj+e2bOhXj045hg46SRYsSLuykSkqlLg57ifl1G89lp44olwtv/QQ5BDPe9EpIpQ4FcBNWrAFVeEO3T32QfOOAN69IAPP4y7MhGpShT4VUjjxvDyy/CnP4WB3aZN4c471YxNRCpGgV/FVKsGF14Y1tNt3x769w8rbv3jH3FXJiK5Lltr2t5nZp+a2aIy24ab2bJ1VsGSLGnUCJ59NsziefPN0IxtxAg1YxOR9cvWGf5YoFs5229x91bpx5Qs7UvSzKBPn9CeoUcPGDIE2rYN1/pFRNaVlcB395nAF9n4XbLxdtgBJk2CRx+FZcsglQqDvN9/H3dlIpJLor6GP8DMFqYv+Wwb8b4S77jjQuvl00+H66+H1q3DilsiIhBt4N8F7AG0ApYDN5X3JjPrZ2bFZlZcWloaYTnJsN12MHYsTJ0Kq1bBQQeFQd5vv427MhGJW2SB7+4r3H2Nu68F7gHarOd9Re6ecvdUYWFhVOUkTteuoRlb//5w++3QrBk891zcVYlInCILfDPbsczLXsCi9b1XolG7Ntx2G8ycCTVrhg+Bs86CL7+MuzIRiUO2pmVOAF4D9jGzj82sLzDSzN40s4VAJ2BQNvYlG++gg8LMnSFD4MEHQ3uGxx6LuyoRqWzmOdSUJZVKeXFxcdxl5LV58+Dss8MHwHHHhcs9O+wQd1Uikgkzm+vuqQ29T3faJkzr1jBnTpjF8/TT4Wx/3Dg1YxNJAgV+AtWoES7vzJ8fAr9PH+jWDd5/P+7KRCRKCvwE+81vwoDu7beH+frNmoVBXjVjE8lPCvyEKygIUzcXLfrvnP2OHeGtt+KuTESyTYEvAOy6K/ztb+F6fkkJtGwZrvP/+GPclYlItijw5T/M4MwzQ3uGnj1DP542beCNN+KuTESyQYEv/6N+fZg4MczV/9e/QugPGQLffRd3ZSKSCQW+rFevXuFsv3fv0Gu/VSt45ZW4qxKRTaXAl1+17bYwZgxMmwarV0OHDjBgAKxcGXdlIrKxFPhSIV26hJW1Bg4M6+g2axY6copI1aHAlwrbaisYPRpefRW23BK6dw+Xez7/PO7KRKQiFPiy0dq3Dz15hg6Fhx8Od+s++qjaM4jkOgW+bJLNN4err4biYmjYEE44ITRjW7487spEZH0U+JKRli1h1iwYOTLcuNW4Mdx3n872RXKRAl8yVr06XHopLFgQPgD69oXDD4f33ou7MhEpS4EvWbP33jBjBtx1F8yeDc2bh0HeNWvirkxEIHsrXt1nZp+a2aIy27Yzs2lmtjT9ddts7EtyW0EBnHceLF4MhxwCgwaFpmxLlsRdmYhk6wx/LNBtnW2XA9PdfS9gevq1JETDhmGBlYcegqVLw8Ir11wTbt4SkXhkJfDdfSbwxTqbewLj0s/HAcdkY19SdZjBaaeFs/tjj4Urr4T99w8ze0Sk8kV5Db++uy8HSH+tF+G+JIfVqwcTJsCTT8Jnn0HbtvD736sZm0hli33Q1sz6mVmxmRWXlpbGXY5E6Oijw7X9vn3hxhuhRQt46aW4qxJJjigDf4WZ7QiQ/vppeW9y9yJ3T7l7qrCwMMJyJBdssw0UFcH06WEpxUMOgfPPh2++ibsykfwXZeBPBnqnn/cGnoxwX1LFdO4MCxeGWTxFRdC0KUyZEndVIvktW9MyJwCvAfuY2cdm1hcYARxmZkuBw9KvRf5jyy3h5pvDAup16kCPHnD66eE6v4hkn3kO3QOfSqW8WFM4EumHH+CGG8I6uttsA7fdBieeGGb6iMivM7O57p7a0PtiH7QVgdCMbfhwmDsXGjWCk0+GY46BZcvirkwkfyjwJac0bw6vvQajRoVVtpo0gXvuUTM2kWxQ4EvOqVYNBg8Og7r77gv9+sGhh8K778ZdmUjVpsCXnLXnnmH6ZlFRuNTTvHkY5FUzNpFNo8CXnFZQAOecE9ozdOkSzvwPOAAWLdrwz4rILynwpUrYeefQmmHChNBnf9994aqr1IxNZGMo8KXKMAuzd0pKwpTN4cNhv/1gzpy4KxOpGhT4UuVsv31ou/z00/DVV2FR9cGDYdWquCsTyW0KfKmyevQIzdj69QuDuc2bhxW3RKR8Cnyp0urUCUsqvvhiGODt3Dl8AHz9ddyVieQeBb7khYMPDouoX3opjBkTbth66qm4qxLJLQp8yRu1asHIkWEB9bp1Q//9U04BLbMgEijwJe+kUmEZxauvhkmToHFjePhhtWcQUeBLXtpsMxg6FObNC3fsnnYaHHUUfPRR3JWJxEeBL3mtaVN49dUwi2fGjPD67rvDalsiSaPAl7xXrVpYWevNN6FNm7CkYufOsHRp3JWJVK7IA9/M3jezN81svplpdROJze67h5bL994L8+eHRdRvvBF++inuykQqR2Wd4Xdy91YVWZFFJEpm0LdvaMbWtSv8/vfQrl1oxSyS73RJRxJpp53g8cfhL38JA7n77QdXXhmWWhTJV5UR+A48Z2ZzzaxfJexPpELMQhO2JUvg1FPhmmtCF85Zs+KuTCQalRH4B7r7vkB3oL+ZdSz7TTPrZ2bFZlZcqjtkJAZ168K4cTBlCqxcGfrtDxoE//533JWJZFfkge/un6S/fgo8DrRZ5/tF7p5y91RhYWHU5YisV/fuoRnbBRfA6NGhGdvzz8ddlUj2RBr4ZralmdX++TlwOKC1iiRn1a4Nt98OM2dC9epw2GFhkPerr+KuTCRzUZ/h1wdeMbMFwBzgGXefGvE+RTLWoUNoxnb55eFyT5Mm8MQTcVclkplIA9/d33P3lulHU3e/Lsr9iWTTFlvADTeEZmz16kGvXmGQd8WKuCsT2TTV4y4gW656ajFLPvkm7jIkT+19HmzxEbz8Pux3aejPU79+3FVJPmmyUx2GHdU00n1oHr5IBZjBLruETpy1asFbb4VWDZq3L1VJ3pzhR/3JKPKzNWvgzjthyBD4yGDEiNCfp0CnT5Lj9L+oyEaqVg1+9ztYtCgsoD5gQFhx6+23465M5Ncp8EU2UaNG8OyzcP/9Ifxbtgxn+z/+GHdlIuVT4ItkwAz69IGSEujRI1zmads2dOMUyTUKfJEs2GGHsJzio4/CJ5+Ewd0rroDvv4+7MpH/UuCLZNFxx4VmbKefDtdfD61bw9//HndVIoECXyTLttsOxo6FqVNh1So46CC48EL49tu4K5OkU+CLRKRr1zCY279/6M/TrBk891zcVUmSKfBFIlS7Ntx2W2jGVrNm+BA46yz44ou4K5MkUuCLVIKDDgozd4YMgQcfDM3YJk2KuypJGgW+SCWpWTMM5L7+elhi8fjjw+Nf/4q7MkkKBb5IJWvdOnTgvP56ePrpcLY/diy4x12Z5DsFvkgMatQIl3cWLICmTcN1/W7d4P33465M8pkCXyRG++wDL70Ed9wR5us3axYGedeujbsyyUeRB76ZdTOzt83sHTO7POr9iVQ1BQVhHd1Fi8JKWxdeCB07hhbMiTZ+fGhYVFAQvo4fH3dFVV7Ua9pWA+4AugNNgFPMrEmU+xSpqnbdFaZMCUsqlpSEZmzXX5/QZmzjx0O/fvDBB2Fw44MPwmuFfkaiPsNvA7yTXupwNfAI0DPifYpUWWZw5pmhPcPRR4d+PG3awBtvxF1ZJbviinCbclmrVoXtssmiDvydgY/KvP44ve0/zKyfmRWbWXFpaWnE5YhUDfXrw1//Co89FqZttmkTBnm/+y7uyirJhx9u3HapkKgD38rZ9ovJZ+5e5O4pd08VFhZGXI5I1dKrVzjb79Mn9Npv1QpefjnuqirBLrts3HapkKgD/2OgYZnXDYBPIt6nSF7Zdlu4916YNg1Wrw4Duv37w8qVcVcWoeuuC4sHl1WrVtgumyzqwH8d2MvMdjOzzYCTgckR71MkL3XpEmbyDBwId90V5u9PnRp3VRE57TQoKgoj2Wbha1FR2C6bLNLAd/efgAHAs0AJMNHdF0e5T5F8tuWWMHp0mLNfuzZ07w69e8Pnn8ddWQROOy3cibZ2bfiqsM9Y5PPw3X2Ku+/t7nu4u/49JpIF7dqFmTtDh8LDD4f2DH/9q9ozyK/TnbYiVdTmm8PVV8PcudCwIZx4Ihx7bFhiUaQ8CnyRKq5FC5g1C268MVzTb9IExozR2b78LwW+SB6oXh0uuQTefDNM3fztb+Gww+C99+KuTHKJAl8kj+y5J7zwAtx9N8yZA82bh0HeNWvirkxygQJfJM8UFMC554Ybtjp1gkGD4MADYbHmxyWeAl8kTzVoAE89FfqNvfNOWHjlmmvCzVuSTAp8kTxmBqeeGs72jzsOrrwSUqmwzKIkjwJfJAHq1YMJE+DJJ8NNWu3awaWX/m9DSslvCnyRBDn66HC237cvjBoVeu6/+GLcVUllUeCLJMzWW4e2NNOnh64FnTrBeefB11/HXZlETYEvklCdO4d5+xdfDPfcE5qxPfNM3FVJlBT4IglWqxbcdFNoxrbNNnDkkaFHmdYiyk8KfBGhbdvQjG348NCErUkTeOQRtWfINwp8EQFgs81g2LAQ/LvvDqecAj17wrJlcVcm2aLAF5FfaNYsXOIZNQqefz6c7RcV6Ww/HyjwReR/VKsGgweHQd399gutGg49FN59N+7KJBORBb6ZDTezZWY2P/04Iqp9iUg09tgjTN8sKgp995s3D4O8asZWNUV9hn+Lu7dKP6ZEvC8RiYAZnHNOuGGrS5fQhrl9+7C+rlQtuqQjIhWy886hNcMjj4QlZvfdN8zqUTO2qiPqwB9gZgvN7D4z27a8N5hZPzMrNrPiUk3+FclpZnDSSeFs/8QT4aqrQvDPmRN3ZVIRGQW+mT1vZovKefQE7gL2AFoBy4Gbyvsd7l7k7il3TxUWFmZSjohUku23h4cegqefDi0Z2rcPg7xqxpbbqmfyw+7epSLvM7N7gKcz2ZeI5J4ePcLCKpddBjffDE88Edo0dO4cd2VSnihn6exY5mUvQEM8InmoTh24667QdbOgIEzfPOcc+OqruCuTdUV5DX+kmb1pZguBTsCgCPclIjE7+GBYsCD02b/vvtCMbfLkuKuSsiILfHc/w92bu3sLdz/a3ZdHtS8RyQ21asHIkTB7NtStG1oznHwyfPpp3JUJaFqmiEQglYLi4rCG7uOPh/YM48erPUPcFPgiEonNNoM//AHmzYO99oLTTw/tlz/6KO7KkkuBLyKRatIEXnkFRo8OA7tNm4ZB3rVr464seRT4IhK5atVg4MDQjqFtW7jggrC04tKlcVeWLAp8Eak0u+0Gzz0HY8aEGT0tWoRB3p9+iruyZFDgi0ilMoOzzw7tGbp1CzdttWsXPgAkWgp8EYnFTjvBY4/BxIlhIDeVgqFD4Ycf4q4sfynwRSQ2ZnDCCeFs/9RT4dproXVreO21uCvLTwp8EYld3bowbhxMmQLffgsHHggXXRSeS/Yo8EUkZ3TvHpqxnX8+/OlPYYWtadPirip/KPBFJKfUrg133AEzZ4abtw4/HPr2hS+/jLuyqk+BLyI5qUOHMHPn8svD5Z4mTUKbBtl0CnwRyVk1a8INN4QVtXbYAY49Nqy0tWJF3JVVTQp8Ecl5Py+jeN11YV3dxo3hgQfUjG1jKfBFpEqoUQP+7//CZZ7GjaF3bzjiCPjww7grqzoyXdP2BDNbbGZrzSy1zveGmNk7Zva2mXXNrEwRkeA3v4GXX4bbbgtfmzYNg7xqxrZhmZ7hLwKOBWaW3WhmTYCTgaZAN+BOM6uW4b5ERICwlOKAAaEZ2wEHhOcHHwxvvx13Zbkto8B39xJ3L+8Q9wQecfcf3P2fwDtAm0z2JSKyrkaNYOpUGDs2zN9v2RJGjFAztvWJ6hr+zkDZZQ4+Tm8TEckqs3A9f8mSsMDKkCGhBfP8+XFXlns2GPhm9ryZLSrn0fPXfqycbeWOp5tZPzMrNrPi0tLSitYtIvILO+wAjz4aHsuWhWZsV1wB338fd2W5Y4OB7+5d3L1ZOY8nf+XHPgYalnndAPhkPb+/yN1T7p4qLCzcuOpFRNZx3HHhbP+MM+D666FVK3j11biryg1RXdKZDJxsZpub2W7AXsCciPYlIvIL220H998Pzz4bzvA7dIDf/Q5Wroy7snhlOi2zl5l9DLQHnjGzZwHcfTEwEVgCTAX6u/uaTIsVEdkYhx8eZvIMGBCmbjZrFj4EkirTWTqPu3sDd9/c3eu7e9cy37vO3fdw933c/W+ZlyoisvG22gpuvTXM2a9VK6yy1acPfPFF3JVVPt1pKyKJcOCBMG9eGMh96KHQjG3SpLirqlwKfBFJjJo1w6paxcVhicXjjw+DvMuXx11Z5VDgi0jitGoVmrGNGAHPPBPO9seOzf9mbAp8EUmk6tXhsstCM7ZmzeCss6BrV3j//bgri44CX0QSbZ994KWXwiye114L4X/bbfnZjE2BLyKJV1AAF1wQpnB26AAXXhi+lpTEXVl2KfBFRNJ23RWmTAmLq7z1VrjWf9118OOPcVeWHQp8EZEyzEJbhiVLoGdP+MMfYP/94Y034q4scwp8EZFy1K8PEyeGhdNXrIA2bcKC6t99F3dlm06BLyLyK445Jpzt9+kDf/xjuMzz8stxV7VpFPgiIhuw7bZw770wbRqsXg0dO0L//lWvGZsCX0Skgrp0CTN5LroI7rorrKf7tyrUKUyBLyKyEbbcEm65JfTY32orOOIIOPNM+PzzuCvbMAW+iMgmaN8+NGMbOhQmTIDGjcMgby63Z1Dgi4hsos03h6uvhrlzYZdd4KST4Nhj4ZNy1/eLnwJfRCRDLVrArFkwciRMnRqasY0Zk3tn+5mueHWCmS02s7VmliqzvZGZfWdm89OPuzMvVUQkd1WvDpdeCgsXQsuW8NvfwmGHwXvvxV3Zf2V6hr8IOBaYWc733nX3VunHeRnuR0SkSthrL5gxA+6+O7Rgbt4cRo+GNTmwyGumSxyWuPvb2SpGRCQfFBTAueeGG7Y6dYJBg8KKW4sXx1xXhL97NzObZ2YvmVmHCPcjIpKTGjSAp56C8ePhnXegdeswyLt6dTz1bDDwzex5M1tUzqPnr/zYcmAXd28NXAw8bGZ11vP7+5lZsZkVl5aWbtqfQkQkR5nBqaeGVsvHHw/DhkEqBa+/Xvm1bDDw3b2Luzcr5/Hkr/zMD+7+efr5XOBdYO/1vLfI3VPuniosLNzUP4eISE4rLISHH4bJk+GLL6BduzDIu2pV5dUQySUdMys0s2rp57sDewE5NFYtIhKPo44K1/LPOQdGjQozel58sXL2nem0zF5m9jHQHnjGzJ5Nf6sjsNDMFgCPAue5+xeZlSoikh+23jrM4nnhhTBXv1MnGDw4+v1Wz+SH3f1x4PFytk8CJmXyu0VE8l2nTmHe/rBhsNtu0e8vo8AXEZHM1KoFN95YOftSawURkYRQ4IuIJIQCX0QkIRT4IiIJocAXEUkIBb6ISEIo8EVEEkKBLyKSEOY5tAaXmZUCH2TwK7YHPstSOVVR0v/8oGMAOgaQvGOwq7tvsPtkTgV+psys2N1TG35nfkr6nx90DEDHAHQM1keXdEREEkKBLyKSEPkW+EVxFxCzpP/5QccAdAxAx6BceXUNX0RE1i/fzvBFRGQ98iLwzaybmb1tZu+Y2eVx11MZzOw+M/vUzBaV2badmU0zs6Xpr9vGWWOUzKyhmc0wsxIzW2xmA9Pbk3QMaprZHDNbkD4GV6W372Zms9PH4C9mtlnctUbNzKqZ2Twzezr9OnHHoCKqfOCn1869A+gONAFOMbMm8VZVKcYC3dbZdjkw3d33AqanX+ern4DB7t4YaAf0T/93T9Ix+AHo7O4tgVZANzNrB/wRuCV9DL4E+sZYY2UZCJSUeZ3EY7BBVT7wgTbAO+7+nruvBh4BesZcU+TcfSaw7jrBPYFx6efjgGMqtahK5O7L3f2N9POVhL/sO5OsY+Du/m36ZY30w4HOhLWkIc+PAYCZNQB6APemXxsJOwYVlQ+BvzPwUZnXH6e3JVF9d18OIRCBejHXUynMrBHQGphNwo5B+lLGfOBTYBrwLvCVu/+UfksS/j6MBn4PrE2/rkvyjkGF5EPgWznbNPUoIcxsK2AScJG7fxN3PZXN3de4eyugAeFfu43Le1vlVlV5zOxI4FN3n1t2czlvzdtjsDHyYRHzj4GGZV43AD6JqZa4rTCzHd19uZntSDjry1tmVoMQ9uPd/bH05kQdg5+5+1dm9iJhPGMbM6uePsPN978PBwJHm9kRQE2gDuGMP0nHoMLy4Qz/dWCv9Kj8ZsDJwOSYa4rLZKB3+nlv4MkYa4lU+jrtGKDE3W8u860kHYNCM9sm/XwLoAthLGMGcHz6bXl9DNx9iLs3cPdGhL/7L7j7aSToGGyMvLjxKv3pPhqoBtzn7tfFXFLkzGwCcAihK+AKYBjwBDAR2AX4EDjB3dcd2M0LZnYQ8DLwJv+9dvt/hOv4STkGLQgDktUIJ28T3f1qM9udMHlhO2AecLq7/xBfpZXDzA4BLnH3I5N6DDYkLwJfREQ2LB8u6YiISAUo8EVEEkKBLyKSEAp8EZGEUOCLiCSEAl9EJCEU+CIiCaHAFxFJiP8HS2ScJNz4Z8QAAAAASUVORK5CYII=\n", "text/plain": [ "
" ] }, "metadata": { "needs_background": "light" }, "output_type": "display_data" }, { "data": { "image/png": "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\n", "text/plain": [ "
" ] }, "metadata": { "needs_background": "light" }, "output_type": "display_data" }, { "data": { "image/png": "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\n", "text/plain": [ "
" ] }, "metadata": { "needs_background": "light" }, "output_type": "display_data" }, { "data": { "image/png": "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\n", "text/plain": [ "
" ] }, "metadata": { "needs_background": "light" }, "output_type": "display_data" }, { "data": { "image/png": "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\n", "text/plain": [ "
" ] }, "metadata": { "needs_background": "light" }, "output_type": "display_data" }, { "data": { "image/png": "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\n", "text/plain": [ "
" ] }, "metadata": { "needs_background": "light" }, "output_type": "display_data" }, { "data": { "image/png": "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\n", "text/plain": [ "
" ] }, "metadata": { "needs_background": "light" }, "output_type": "display_data" }, { "data": { "image/png": "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\n", "text/plain": [ "
" ] }, "metadata": { "needs_background": "light" }, "output_type": "display_data" }, { "data": { "image/png": "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\n", "text/plain": [ "
" ] }, "metadata": { "needs_background": "light" }, "output_type": "display_data" }, { "data": { "image/png": "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\n", "text/plain": [ "
" ] }, "metadata": { "needs_background": "light" }, "output_type": "display_data" }, { "data": { "image/png": "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\n", "text/plain": [ "
" ] }, "metadata": { "needs_background": "light" }, "output_type": "display_data" }, { "data": { "image/png": "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\n", "text/plain": [ "
" ] }, "metadata": { "needs_background": "light" }, "output_type": "display_data" }, { "data": { "image/png": "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\n", "text/plain": [ "
" ] }, "metadata": { "needs_background": "light" }, "output_type": "display_data" }, { "data": { "image/png": "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\n", "text/plain": [ "
" ] }, "metadata": { "needs_background": "light" }, "output_type": "display_data" }, { "data": { "image/png": "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\n", "text/plain": [ "
" ] }, "metadata": { "needs_background": "light" }, "output_type": "display_data" }, { "data": { "image/png": "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\n", "text/plain": [ "
" ] }, "metadata": { "needs_background": "light" }, "output_type": "display_data" }, { "data": { "image/png": "iVBORw0KGgoAAAANSUhEUgAAAXwAAAD8CAYAAAB0IB+mAAAABHNCSVQICAgIfAhkiAAAAAlwSFlzAAALEgAACxIB0t1+/AAAADl0RVh0U29mdHdhcmUAbWF0cGxvdGxpYiB2ZXJzaW9uIDMuMC4zLCBodHRwOi8vbWF0cGxvdGxpYi5vcmcvnQurowAAHrpJREFUeJzt3Xu8VXP+x/HXp1Op3Eqd6KZkQud0b+uiyZRKSaNym5o0MdEPuRspmUyS+yXGZUQITTTSRJpJNbkTJ9LtoBCSUS4xbpG+vz8+2zjDqXNq733Wvryfj8d+nL3X2af1WezeZ/Vd3/X5WggBERHJfpWiLkBERCqGAl9EJEco8EVEcoQCX0QkRyjwRURyhAJfRCRHKPBFRHKEAl9EJEco8EVEckTlqAsoqU6dOqFJkyZRlyEiklGWLFnyUQghv6z3pVXgN2nShKKioqjLEBHJKGb2TnnepyEdEZEcocAXEckRCnwRkRyhwBcRyREKfBGRHKHAFxHJEQp8EZEckRWBv3EjnHsufPZZ1JWIiKSvrAj8hQvhppugoAAefTTqakRE0lNWBP6gQfDCC1C7Nhx1FPz2t37WLyIiP8qKwAc4+GAoKoLx4+Ghh6B5c/jrXyGEqCsTEUkPWRP4AFWrwrhx8Mor8ItfwJAhfsa/bl3UlYmIRC+rAv8HhYXw7LNw/fU+vl9QALffDlu3Rl2ZiEh0Eg58M6tmZi+a2atmttLMxse372dmi81stZk9aGZVEy+3/PLyfObOihU+3HPqqdCjB6xZU5FViIikj2Sc4W8GDgshtAbaAH3MrBNwFXBDCKEZ8CkwPAn72mFNm8KCBXDnnT7U07IlXHstbNkSRTUiItFJOPCD+yL+skr8EYDDgIfi26cCAxLd184yg+HDYdUq6N0bLrgAOneGZcuiqkhEpOIlZQzfzPLMbCmwAZgPvAlsCiH8cB69DmiwjZ8dYWZFZla0McVzKevXh1mz4MEH4Z13oH17v8i7eXNKdysikhaSEvghhO9DCG2AhkAHoHlpb9vGz04OIcRCCLH8/DJX6EqYGRx/PBQXw+DBMGECtGvn8/hFRLJZUmfphBA2AU8AnYCaZvbDEooNgfXJ3FeiateGe++FuXPhP/+BQw7xi7xffhl1ZSIiqZGMWTr5ZlYz/rw60BMoBhYBx8bfNgyYnei+UuGII3wmz2mnwaRJflF3wYKoqxIRSb5knOHXAxaZ2TLgJWB+CGEOcCFwnpmtAWoDU5Kwr5TYYw+45RZ46imoXBl69fKLvJs2RV2ZiEjyWEij3gOxWCwUFRVFWsPXX3t7hmuvhbp14dZbYUBk84tERMpmZktCCLGy3peVd9omonp1uPJKWLzYA3/gQL/I++GHUVcmIpIYBf42tG8PL70El10Gs2d7e4b77lMzNhHJXAr87ahSBcaOhaVL4cAD4Xe/gyOPhHffjboyEZEdp8Avh+bN4emn4cYb4cknvTnbrbeqGZuIZBYFfjnl5cFZZ/kUzs6dYeRI6NYN3ngj6spERMpHgb+D9tsP5s2Du+6C5cuhVSu46io1YxOR9KfA3wlmcNJJ3oytb18YPRo6dvSxfhGRdKXAT0C9evDww76k4vvvQyzmF3m/+SbqykREfk6BnwTHHONn+yecAJdfDm3bwnPPRV2ViMj/UuAnyV57wT33wD//6Xfr/vKXfpH3iy/K/FERkQqhwE+y3r19Js8ZZ8DNN0OLFvD441FXJSKiwE+J3XaDm27yufvVq/svgZNOgk8+iboyEcllCvwU6tLF19G96CJvy1BQADNnRl2ViOQqBX6KVasGEyd6X5569eDYY/3x739HXZmI5BoFfgVp2xZefNFn8cyZ42f7U6eqGZuIVBwFfgWqUgXGjPEbtAoK4MQToU8fWLs26spEJBco8CNw0EG+utbNN/t8/RYt4M9/VjM2EUktBX5EKlXyBmwrVvw4Z//QQ+G116KuTESylQI/Yo0bwz/+4eP5q1ZB69Y+zv/dd1FXJiLZRoGfBsx8cZXiYujf3/vxdOgAL78cdWUikk0U+Glk771hxgxvyPbvf3vojxnjrRpERBKlwE9DAwf68M6wYb6geps28MwzUVclIplOgZ+matWCKVNg/nz49lvo2tX78/znP1FXJiKZSoGf5nr29Jk855zj6+gWFvpFXhGRHaXAzwC77go33ADPPuuN2fr29Yu8H38cdWUikkkU+Bmkc2dvxvbHP8L06X637t/+pvYMIlI+CvwMs8sucOmlUFQEjRrB8cfD0UfD+vVRVyYi6U6Bn6Fat4YXXoCrrvJVtgoK/CKvzvZFZFsSDnwza2Rmi8ys2MxWmtnZ8e17mdl8M1sd/1or8XKlpMqVYdQoePVV/wVw8snQqxe89VbUlYlIOkrGGf4W4PwQQnOgEzDSzAqA0cDCEEIzYGH8taTAAQfAokVw223egrllS5g0Cb7/PurKRCSdJBz4IYQPQggvx5//BygGGgD9ganxt00FBiS6L9m2SpXg1FNh5Uro1g3OPdebsq1aFXVlIpIukjqGb2ZNgLbAYmDvEMIH4L8UgLrb+JkRZlZkZkUbN25MZjk5qVEjX2Dl/vth9WpfeGXCBL95S0RyW9IC38x2A2YC54QQPi/vz4UQJocQYiGEWH5+frLKyWlmMGSIn90ffTSMGwcHH+wze0QkdyUl8M2sCh7200IID8c3f2hm9eLfrwdsSMa+pPzq1vX5+rNnw0cfQceOfpFXzdhEclMyZukYMAUoDiFcX+JbjwDD4s+HAbMT3ZfsnKOO8rH94cPhmmugVSt48smoqxKRipaMM/wuwFDgMDNbGn/0Ba4EepnZaqBX/LVEpGZNmDwZFi70pRS7dYPTToPPyz34JiKZzkIa3akTi8VCkQaaU+6rr7w9w6RJUL8+/OUvcOSRUVclIjvLzJaEEGJlvU932uagGjXguuvg+ef9zL9fP7/Iq0lSItlNgZ/DOnSAJUvgT3/yJmwFBfDAA2rPIJKtFPg5rmpVuOQSXz+3aVMYPNjX1X3//agrE5FkU+ALAC1awHPP+VDPggV+tn/HHTrbF8kmCnz5r7w8OO88WL4c2reHESOgRw94882oKxORZFDgy8/sv79P35w82cf4W7aE669XMzaRTKfAl1KZwSmneHuGnj3h/PPhkEN8fV0RyUwKfNmuBg28NcP06d5nv107GD9ezdhEMpECX8pkBoMGQXExHHecT+Ns395774tI5lDgS7nVqQPTpsGjj8Knn/qi6n/4g9+5KyLpT4EvO6xfP2/GNmKET+Ns2dJX3BKR9KbAl52y556+pOITT/hqW4cd5r8APvss6spEZFsU+JKQX/3KF1G/4AKYMsVv2Hr00airEpHSKPAlYTVqwNVXw+LFULu2998fPBg2aMkbkbSiwJekicV8GcUJE+Dhh/1sf9o0tWcQSRcKfEmqqlXh4ovhlVegWTM44QT49a/hvfeirkxEFPiSEgUF8MwzvsjKokVQWOgLrWzdGnVlIrlLgS8pk5cHZ5/tzdg6dPAlFQ87DFavjroykdykwJeUa9oU5s/3WTxLl/oi6tdcA1u2RF2ZSG5R4EuFMIPf/96bsfXuDaNG+Z26y5ZFXZlI7lDgS4WqXx9mzYIZM+Ddd70nz7hxsHlz1JWJZD8FvlQ4M2/CtmqVz9efMAHatvVF1UUkdRT4EpnateHee2HuXPjiC+jSBc45B778MurKRLKTAl8id8QRvrDKaafBjTf6+roLFkRdlUj2UeBLWthjD7jlFnjqKahSBXr1guHDYdOmqCsTyR4KfEkrXbt6M7bRo2HqVL+B6+9/j7oqkeygwJe0U706XHGFr6i1994wcCAcfzx8+GHUlYlkNgW+pK127Tz0J070dXWbN/eLvGrGJrJzkhL4ZnaXmW0wsxUltu1lZvPNbHX8a61k7EtyS5UqcNFFPszTvDkMGwZ9+/ocfhHZMck6w78H6POTbaOBhSGEZsDC+GuRnXLQQfD003DTTf61sNAv8qoZm0j5JSXwQwhPAZ/8ZHN/YGr8+VRgQDL2JbmrUiU480yfwtm5M5xxhq+49frrUVcmkhlSOYa/dwjhA4D417qlvcnMRphZkZkVbdy4MYXlSLZo0gTmzYO77/bwb90arrxSzdhEyhL5RdsQwuQQQiyEEMvPz4+6HMkQZnDiiVBcDP36wZgx0LGjd+MUkdKlMvA/NLN6APGvWuFUkm6ffeChh/zx/vu+zOLYsfDNN1FXJpJ+Uhn4jwDD4s+HAbNTuC/Jcccc483Yhg6Fyy+HNm3g2WejrkokvSRrWuZ04HngQDNbZ2bDgSuBXma2GugVfy2SMnvt5eP68+b5GX7XrnDWWd6YTUTAQhrdxRKLxUJRUVHUZUgW+OILn79/882w774weTIcfnjUVYmkhpktCSHEynpf5BdtRVJht91+nLNfrZqvsnXSSfDJTycPi+QQBb5ktS5dfObO2LFw333ejG3mzKirEomGAl+yXrVqcNllUFTkSywee6xf5P3gg6grE6lYCnzJGW3aeDO2K6+Exx7zs/177lEzNskdCnzJKZUrw4UXejO2Fi18XL93b1i7NurKRFJPgS856cAD4cknvQHb8897+P/5z2rGJtlNgS85q1IlOP1078fzw5z9rl29XYNINlLgS85r3BjmzvXFVV57zcf6L78cvvsu6spEkkuBL4I3Yxs61NszDBjg0zgPPhhefjnqykSSR4EvUsLee8ODD8KsWb6GbocOvqD6119HXZlI4hT4IqUYMMDP9k88Ea66yod5nn466qpEEqPAF9mGWrXgzjth/nz49ls49FAYORI+/zzqykR2TuWoC0iG8Y+uZNV6/S2U1ImNgtpvw8x1MOdsOOAA784pkiwF9ffgkl8XpnQfOsMXKYdKlWD//aFtW8jLg+XLfUaPllWUTJIVZ/ip/q0oUtLmzTBxIlxxBXxQy1swH3ecz/QRSWc6wxfZQbvsApdeCkuWeK/93/wGBg6E9eujrkxk+xT4IjupVSt44QW4+mpfZaugAKZMUTM2SV8KfJEEVK4MF1wAy5ZB69Zw8snQqxe89VbUlYn8nAJfJAmaNYNFi+C227wFc8uWMGkSfP991JWJ/EiBL5IklSrBqafCypXQrRucey788pd+A5dIOlDgiyRZo0YwZw7cfz+sXu1TOSdM8Ju3RKKkwBdJATMYMsTP7o8+GsaNg1gMXnop6soklynwRVKobl2YPh1mz4aPP4ZOnWDUKPjqq6grk1ykwBepAEcd5Wf7J58M11zjM3qeeCLqqiTXKPBFKsiee8Ltt8O//uVz9bt394u8n30WdWWSKxT4IhWse3eft3/eeXDHHb6e7mOPRV2V5AIFvkgEatSA667zBdRr1oR+/eCEE+Cjj6KuTLKZAl8kQh06eE+eP/0JZsyA5s3hgQfUnkFSI+WBb2Z9zOx1M1tjZqNTvT+RTFO1Klxyia+f27QpDB7sK269/37UlWWAadOgSRO/661JE38t25TSwDezPOAW4AigABhsZgWp3KdIpmrRAp57zod65s/3Zmx33KGz/W2aNg1GjIB33vH/SO+8468V+tuU6jP8DsCaEMJbIYRvgQeA/inep0jGysvzi7nLl0P79p5fPXrAm29GXVkaGjv25zc0fPWVb5dSpTrwGwDvlXi9Lr7tv8xshJkVmVnRxo0bU1yOSGbYf39YuNCncS5Z4s3Yrr9ezdj+x7vv7th2SXngl7YG0P/8AzWEMDmEEAshxPLz81NcjkjmMPMz/FWroGdPOP98OOQQWLEi6srSxL777th2SXngrwMalXjdENC6QCI7oEEDb80wfbr32W/Xzmf15HwztokTfX5rSTVq+HYpVaoD/yWgmZntZ2ZVgUHAIynep0jWMYNBg6C4GI4/HsaP9+B/8cWoK4vQkCEweTI0buz/gRo39tdDhkRdWdpKaeCHELYAZwDzgGJgRghhZSr3KZLN6tTxtstz5nhLhs6dfagnZ5uxDRkCa9fC1q3+VWG/XSmfhx9CmBtCOCCEsH8IQf/WEkmCI4/0hVZGjPCLuS1beo8eke3RnbYiGWqPPXxJxSee8PuOevSAU06BTZuirkzSlQJfJMP96lfejO2CC+Cuu6CwEB7RlTIphQJfJAtUrw5XXw2LF0Pt2tC/v1/k3bAh6soknSjwRbJILAZFRb6G7qxZ3p7h/vvVnkGcAl8ky1StChdfDK+8As2awdCh3n75vffK/lnJbgp8kSxVUADPPAOTJvmF3cJCv8i7dWvUlUlUFPgiWSwvD84+25uxdegAp5/uK26tXh11ZRIFBb5IDmja1Fsu33knvPoqtGrlF3m3bIm6MqlICnyRHGEGw4d7M7beveHCC6FTJ/8FILlBgS+SY+rX9xk8M2b4hdxYDP74R9i8OerKJNUU+CI5yAyOO87P9gcPhssug7ZtfVF1yV4KfJEcVrs23HsvzJ0LX3wBXbrAOefAl19GXZmkggJfRDjiCG/GdvrpcOONvr7uggVRVyXJpsAXEQB23x1uvhmeespv3urVyy/yfvpp1JVJsijwReR/dO3qM3dGj4apU/0Grlmzoq5KkkGBLyI/U60aXHGFr6i1zz5w9NG+0taHH0ZdmSRCgS8i2/TDMooTJ3rL5ebN/SKvmrFlJgW+iGxXlSpw0UWwdKkH/rBhfpH3nXeirkx2lAJfRMrloIPg6afhppu8KVuLFnDLLWrGlkkU+CJSbpUqwZlnwooVvoD6GWf4iluvvx51ZVIeCnwR2WFNmsC8eXD33R7+rVvDlVfCd99FXZlsjwJfRHaKGZx4IhQX+wIrY8ZAx46+8IqkJwW+iCRkn33goYf8sX49HHwwjB0L33wTdWXyUwp8EUmKY47xZmxDh8Lll0ObNvDss1FXJSUp8EUkafbay8f1583zM/yuXeGss7wxm0RPgS8iSXf44X4x98wzvT9PYaH/EpBoKfBFJCV22807bz7zDNSoAX36+EXeTz6JurLcpcAXkZQ65BCfuTN2LEyb5s3YZs6MuqrclFDgm9lxZrbSzLaaWewn3xtjZmvM7HUz651YmSKSyapV81W1XnoJGjSAY4/1hmwffBB1Zbkl0TP8FcDRwFMlN5pZATAIKAT6ALeaWV6C+xKRDNemDSxe7DdpzZ3rZ/t3361mbBUlocAPIRSHEEq7qbo/8EAIYXMI4W1gDdAhkX2JSHaoXBkuvBCWLYOWLeH3v4feveHtt6OuLPulagy/AfBeidfr4tt+xsxGmFmRmRVt3LgxReWISLo54AB44glvwPb8896M7cYb4fvvo64se5UZ+Ga2wMxWlPLov70fK2Vbqf9oCyFMDiHEQgix/Pz88tYtIlmgUiVfR3flSm/Cds45Pne/uDjqyrJTmYEfQugZQmhRymP2dn5sHdCoxOuGwPpEixWR7LTvvvDYY3Dffd55s00bX3RFzdiSK1VDOo8Ag8xsFzPbD2gGvJiifYlIFjCDE07ws/sBA+Dii70vz8svR11Z9kh0WuZAM1sHdAYeM7N5ACGElcAMYBXwT2BkCEEjcyJSprp14cEHfeH0DRugQwdfUP3rr6OuLPNZSKP5ULFYLBQVFUVdhoikiU2b4A9/gClToFkzuPNOOPTQqKtKP2a2JIQQK+t9utNWRNJWzZoe8gsWwJYtfmF35Ej4/POoK8tMCnwRSXs9esDy5T6L57bbfArnP/4RdVWZR4EvIhlh113hhhvguedg992hb1/43e/g44+jrixzKPBFJKN06uQzd8aNg+nToXlzmDFD7RnKQ4EvIhlnl11g/HhYsgQaN4bf/AYGDvQlFmXbFPgikrFatfK2DNdc4wusFBT4jB6d7ZdOgS8iGa1yZZ+6uXy536F78snQsye89VbUlaUfBb6IZIVf/AL+9S/4y1+8737Lln6RV83YfqTAF5GsUakS/N//wapV0L07nHcedOnizdlEgS8iWahhQ3j0UV9Scc0aaNsWJkyAb7+NurJoKfBFJCuZwW9/683YjjnGp3HGYj7ck6sU+CKS1fLzfb7+7Nl+k1anTjBqFHz1VdSVVTwFvojkhKOO8rH9k0/2aZytW/uKW7lEgS8iOWPPPeH22302Twh+YffUU+Gzz6KurGIo8EUk53Tv7ouon38+3HEHFBb6ilvZToEvIjmpRg249lq/U7dWLejXD4YMgY0bo64sdRT4IpLTOnTwnjzjx8Pf/ubtGR54IDvbMyjwRSTnVa3q0zZfeQWaNoXBg6F/f3j//agrSy4FvohIXGGh99u//npfZaugACZPhq1bo64sORT4IiIl5OXBued6M7ZYzFs19Ojhd+xmOgW+iEgp9t/fz/LvuMMXXGnVCq67LrObsSnwRUS2wcxv1CouhsMP9zbMnTv72X8mUuCLiJShfn2YNctn76xdC+3awSWXwObNUVe2YxT4IiLlYOZLKa5aBYMGwaWXQvv2sHhx1JWVnwJfRGQH1KkD993nd+Z+9pkP8Zx3Hnz5ZdSVlU2BLyKyE/r29YVVTj3VV9Zq1cp79KQzBb6IyE7aYw+49VZ48klfbatHDzjlFNi0KerKSqfAFxFJ0KGHejO2UaPgrrv8hq3Zs6Ou6ucSCnwzu8bMXjOzZWY2y8xqlvjeGDNbY2avm1nvxEsVEUlf1avDVVf5Rdz8fBgwwC/ubtgQdWU/SvQMfz7QIoTQCngDGANgZgXAIKAQ6APcamZ5Ce5LRCTtxWJQVORr6M6aBc2bw/33p0cztoQCP4TweAhhS/zlC0DD+PP+wAMhhM0hhLeBNUCHRPYlIpIpqlSBiy/2ZmwHHABDh3r75ffei7auZI7h/x74R/x5A6Dkoa2Lb/sZMxthZkVmVrQxmxtRi0jOKSiAZ56BSZN8OcXCQrjttuiasZUZ+Ga2wMxWlPLoX+I9Y4EtwLQfNpXyR5X6D5oQwuQQQiyEEMvPz9+ZYxARSVt5eXD22bBiBXTsCKefDt26wRtvVHwtZQZ+CKFnCKFFKY/ZAGY2DOgHDAnhv6NU64BGJf6YhsD6ZBcvIpIp9tsPHn/cZ/EsX+6LqF99NWzZUvbPJkuis3T6ABcCR4UQvirxrUeAQWa2i5ntBzQDXkxkXyIimc4MTjrJ2zMccQRceKGf9b/6asXsP9Ex/JuB3YH5ZrbUzP4CEEJYCcwAVgH/BEaGEDK4qaiISPLUqwczZ/qSiuvW+cyeSZNSv9/KifxwCOEX2/neRGBiIn++iEi2MoNjj4Xu3eH8873/fqolFPgiIpKY2rXhnnsqZl9qrSAikiMU+CIiOUKBLyKSIxT4IiI5QoEvIpIjFPgiIjlCgS8ikiMU+CIiOcJCOnTljzOzjcA7O/njdYCPklhOFDL9GFR/9DL9GFT/zmkcQiiz3XBaBX4izKwohBCLuo5EZPoxqP7oZfoxqP7U0pCOiEiOUOCLiOSIbAr8yVEXkASZfgyqP3qZfgyqP4WyZgxfRES2L5vO8EVEZDuyIvDNrI+ZvW5ma8xsdNT1lIeZ3WVmG8xsRYlte5nZfDNbHf9aK8oat8XMGpnZIjMrNrOVZnZ2fHtG1A9gZtXM7EUzezV+DOPj2/czs8XxY3jQzKpGXev2mFmemb1iZnPirzOmfjNba2bL46vlFcW3ZcxnCMDMaprZQ2b2WvzvQ+d0PoaMD3wzywNuAY4ACoDBZlYQbVXlcg/Q5yfbRgMLQwjNgIXx1+loC3B+CKE50AkYGf9vnin1A2wGDgshtAbaAH3MrBNwFXBD/Bg+BYZHWGN5nA0Ul3idafV3DyG0KTGVMZM+QwA3Av8MIRwEtMb/X6TvMYQQMvoBdAbmlXg9BhgTdV3lrL0JsKLE69eBevHn9YDXo66xnMcxG+iVwfXXAF4GOuI3zVSOb/+fz1a6PYCGeKAcBswBLMPqXwvU+cm2jPkMAXsAbxO/FpoJx5DxZ/hAA+C9Eq/Xxbdlor1DCB8AxL/WjbieMplZE6AtsJgMqz8+HLIU2ADMB94ENoUQtsTfku6fpUnAKGBr/HVtMqv+ADxuZkvMbER8WyZ9hpoCG4G748Nqd5rZrqTxMWRD4Fsp2zT1qAKY2W7ATOCcEMLnUdezo0II34cQ2uBnyh2A5qW9rWKrKh8z6wdsCCEsKbm5lLemZf1xXUII7fDh2JFmdmjUBe2gykA74LYQQlvgS9Jp+KYU2RD464BGJV43BNZHVEuiPjSzegDxrxsirmebzKwKHvbTQggPxzdnTP0lhRA2AU/g1yNqmlnl+LfS+bPUBTjKzNYCD+DDOpPInPoJIayPf90AzMJ/6WbSZ2gdsC6EsDj++iH8F0DaHkM2BP5LQLP47ISqwCDgkYhr2lmPAMPiz4fhY+Npx8wMmAIUhxCuL/GtjKgfwMzyzaxm/Hl1oCd+wW0RcGz8bWl7DCGEMSGEhiGEJvhn/l8hhCFkSP1mtquZ7f7Dc+BwYAUZ9BkKIfwbeM/MDoxv6gGsIp2PIeqLCEm6eNIXeAMfgx0bdT3lrHk68AHwHX6mMBwfg10IrI5/3SvqOrdR+y/xoYJlwNL4o2+m1B8/hlbAK/FjWAGMi29vCrwIrAH+BuwSda3lOJZuwJxMqj9e56vxx8of/t5m0mcoXm8boCj+Ofo7UCudj0F32oqI5IhsGNIREZFyUOCLiOQIBb6ISI5Q4IuI5AgFvohIjlDgi4jkCAW+iEiOUOCLiOSI/wd+IKKQnXA3fgAAAABJRU5ErkJggg==\n", "text/plain": [ "
" ] }, "metadata": { "needs_background": "light" }, "output_type": "display_data" }, { "data": { "image/png": "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\n", "text/plain": [ "
" ] }, "metadata": { "needs_background": "light" }, "output_type": "display_data" }, { "data": { "image/png": "iVBORw0KGgoAAAANSUhEUgAAAXwAAAD8CAYAAAB0IB+mAAAABHNCSVQICAgIfAhkiAAAAAlwSFlzAAALEgAACxIB0t1+/AAAADl0RVh0U29mdHdhcmUAbWF0cGxvdGxpYiB2ZXJzaW9uIDMuMC4zLCBodHRwOi8vbWF0cGxvdGxpYi5vcmcvnQurowAAHvZJREFUeJzt3Xmc1ePfx/HXZ6aSbCmTXyoliqa9jhaJFilrRSjdCf10R8jyk8IPcdvqRnZ3hJAlJSWRFmRJNa2mJgpRpLJkF+m6/7iOn6GpmTrnzHWW9/PxmMfM+c6M867HeM+37/c6n8ucc4iISPrLCh1ARERKhwpfRCRDqPBFRDKECl9EJEOo8EVEMoQKX0QkQ6jwRUQyhApfRCRDqPBFRDJEmdABCttvv/1crVq1QscQEUkpCxYs+NI5l1Pc1yVV4deqVYu8vLzQMUREUoqZfVKSr9MlHRGRDKHCFxHJECp8EZEMocIXEckQKnwRkQyhwhcRyRAqfBGRDJEWhb9xI1x6KXz7begkIiLJKy0Kf+ZMuPtuyM2FF18MnUZEJDmlReH37AnvvguVK8PJJ8OZZ/qzfhER+VNaFD7A4YdDXh4MGwbjx0O9evDUU+Bc6GQiIskhbQofoFw5uPZaWLQIDjkEevf2Z/xr14ZOJiISXloV/h/q14e334Y774RZs/y1/f/7P9i6NXQyEZFw0rLwAbKz4ZJL4L33oEULGDAAOnSAlStDJxMRCSNtC/8PtWvD9Onw8MOweDE0agQjRsCWLaGTiYiUrrQvfAAz6NcPli+Hzp1h8GBo3RqWLg2dTESk9MRc+GZW3szmmdkSM1tmZsOixw8ys7lmttLMnjWzcrHHjc0BB8DEiTBuHHz6KTRv7m/ybt4cOpmISOLF4wx/M9DBOdcYaAJ0MbNWwG3Anc65OsA3QL84PFfMzOC00/zZfq9ecOON0KyZX8cvIpLOYi585/0QfVg2+uaADsD46PExQLdYnyueKleGxx+HqVPh++/hiCP8eIYffwydTEQkMeJyDd/Mss1sMbABmA58CGxyzv1xa3QtUC0ezxVvxx0H+flw/vkwciQ0aAAzZoROJSISf3EpfOfc7865JkB1oAVQr6gvK+p7zay/meWZWd7GQPMQ9t4b7rsP3ngDypaFTp38Td5Nm4LEERFJiLiu0nHObQJeB1oBFc2sTPRT1YHPt/M9o5xzEedcJCcnJ55xdtpRR8GSJTBkCIwZ41+w9cILQSOJiMRNPFbp5JhZxejHuwPHAAXAa0CP6Jf1BSbF+lylYffd4ZZbYO5cqFIFuneH00+H9etDJxMRiU08zvCrAq+Z2VJgPjDdOTcFuBK4zMxWAZWB0XF4rlLTvDnMnw833QSTJvmz/Sef1DA2EUld5pKowSKRiMvLywsdYxsFBf6a/pw5/ibvgw/CgQeGTiUi4pnZAudcpLivy4hX2saqXj14802/ycrs2X442/33axibiKQWFX4JZWfDRRf5JZytW8PAgdCuHXzwQehkIiIlo8LfSbVqwbRp8MgjfhJno0Zw220axiYiyU+FvwvM4Jxz/HiGE07wyzhbtvTTOEVEkpUKPwZVq8KECX5Lxc8+g0gErr4afvkldDIRkW2p8OPg1FP92X6fPnDzzdC0KbzzTuhUIiJ/pcKPk0qV4NFH4ZVX4Kef4MgjYdAg+OGH4r9XRKQ0qPDjrHNnv5Jn4EC45x4/jO3VV0OnEhFR4SfEXnv5sp89G8qX978EzjkHvvkmdDIRyWQq/AQ68ki/cmfoUHjiCT+e4fnnQ6cSkUylwk+w8uX9jdz58/2qnlNPhR494IsvQicTkUyjwi8lTZv6CZy33AJTpviz/cce0zA2ESk9KvxSVLasf5HWkiV+Hs8550CXLrB6dehkIpIJVPgBHHqo313r3nv9ev0GDfxNXg1jE5FEUuEHkpXll27m5/ubuxdf7HfcWrEidDIRSVcq/MBq1oSXX/ZbKi5fDo0b+5u8v/0WOpmIpBsVfhIwg7PO8hutdO3q5/EcfjgsXBg6mYikExV+Etl/fxg3zq/VX78eWrTwa/h//jl0MhFJByr8JNS9u7+807cv3HorNGkCb70VOpWIpDoVfpLad18YPRqmT4dff4W2beHCC+H770MnE5FUpcJPcscc43fWGjTI76PboIGfyCkisrNU+Clgzz1h5Eh4+23YYw847jh/ueerr0InE5FUosJPIa1bw6JF8O9/w1NP+fEMzz2n8QwiUjIq/BSz225www2Qlwc1asDpp8Mpp8C6daGTiUiyU+GnqMaN4d13Yfhwf02/Xj145BGd7YvI9qnwU1iZMnDFFX4YW+PG0K8fdOoEH38cOpmIJCMVfhqoWxdeew0eeADmzfMree66C37/PXQyEUkmMRe+mdUws9fMrMDMlpnZoOjxSmY23cxWRt/vG3tc2Z6sLBgwAJYtg3bt4JJL/FC25ctDJxORZBGPM/wtwOXOuXpAK2CgmeUCQ4CZzrk6wMzoY0mwGjX8BitPPgkrV/qNV2680b94S0QyW8yF75xb55xbGP34e6AAqAZ0BcZEv2wM0C3W55KSMYPevf3ZfffucO21fhhbXl7oZCISUlyv4ZtZLaApMBfY3zm3DvwvBaBKPJ9LilelCjzzDEyaBF9+CS1bwuDBGsYmkqniVvhmticwAbjEOffdTnxffzPLM7O8jRs3xiuOFHLyyf7afr9+MGIENGrkd9wSkcwSl8I3s7L4sh/rnHs+eni9mVWNfr4qsKGo73XOjXLORZxzkZycnHjEkSJUrAijRsHMmX4rxXbt4Pzz4bsS/2oWkVQXj1U6BowGCpxzdxT61GSgb/TjvsCkWJ9LYtehAyxdCpdd5n8B1K8PL70UOpWIlIZ4nOG3AfoAHcxscfTteOBWoJOZrQQ6RR9LEthjD7j9dr+B+j77wIkn+pu8X34ZOpmIJJK5JHotfiQScXlaSlKqfv3V76F7882+/O+5B844w6/0EZHUYGYLnHOR4r5Or7TNcOXKwfXX+/1za9eGXr2gWzf47LPQyUQk3lT4AvhxDO+84y/1TJ/uRy8/9JCGsYmkExW+/Ed2tr+Z+9570Lw59O8PHTvChx+GTiYi8aDCl20cfLBfvjlqFCxYAA0bwh13aBibSKpT4UuRzOC88/x4hmOOgcsvhyOOgPz80MlEZFep8GWHqlXzoxmefho++giaNYNhwzSMTSQVqfClWGbQsycUFPgtFa+/3l/jnzcvdDIR2RkqfCmx/fbzY5enTIFNm/ym6pdfDj/9FDqZiJSECl922gkn+GFs553nb+Y2bOh33BKR5KbCl12y997w4IPw+ut+t60OHfwyzm+/DZ1MRLZHhS8xOfpov4n6FVfA6NH+BVuTJ4dOJSJFUeFLzCpUgOHDYe5cf52/a1d/k3dDkQOxRSQUFb7ETSQC8+f7PXQnTvRn+2PHajyDSLJQ4UtclSsH11wDixbBIYfAf/0XnHQSrFkTOpmIqPAlIXJz4e234c47/Qqe+vX9Td6tW0MnE8lcKnxJmOxsuOQSP4ytRQu/pWKHDrByZehkIplJhS8JV7u2H7k8ejQsXuw3UR8xArZsCZ1MJLOo8KVUmMG55/phbF26wODB/pW6S5eGTiaSOVT4UqoOOACefx7GjYNPP/Uzea69FjZvDp1MJP2p8KXUmcFpp/mz/TPP9Ms4mzaFd98NnUwkvanwJZjKlWHMGJg6FX74wc/bv/RS+PHH0MlE0pMKX4I77jg/jO2CC2DkSL+/7owZoVOJpB8VviSFvfaCe++F2bOhbFno1An69fNjmEUkPlT4klTatvXD2IYM8Zd7cnPhhRdCpxJJDyp8STq77w633OKHsVWpAt27+5221q8PnUwktanwJWk1b+6Hsd10k99XNzcXnnhCw9hEdpUKX5Ja2bJw1VX+FbqHHQZnneV33Pr009DJRFJPXArfzB4xsw1mll/oWCUzm25mK6Pv943Hc0lmqlcP3nwT7r7b39itXx/uv1/D2ER2RrzO8B8Duvzt2BBgpnOuDjAz+lhkl2VlwUUXQX6+H8swcCC0awcffBA6mUhqiEvhO+dmA1//7XBXYEz04zFAt3g8l0itWjBtGjz6qJ/E2agR3HabhrGJFCeR1/D3d86tA4i+r5LA55IMYwZnnw0FBf6a/pAh0LKlv9YvIkULftPWzPqbWZ6Z5W3cuDF0HEkx//gHTJgA48fDZ5/5bRavvhp++SV0MpHkk8jCX29mVQGi74vc0to5N8o5F3HORXJychIYR9LZqaf6YWx9+sDNN0OTJn7HLRH5UyILfzLQN/pxX2BSAp9LhEqV/HX9adP8GX7btnDxxX4wm4jEb1nm08Ac4FAzW2tm/YBbgU5mthLoFH0sknDHHutX8lx4oZ/P06ABvPpq6FQi4cVrlU4v51xV51xZ51x159xo59xXzrmOzrk60fd/X8UjkjB77unX7L/5JpQvD507wznnwNf6KZQMFvymrUgitWnjV+5cdZUfy5Cb62/yimQiFb6kvfLl/TyevDy/xWKPHv7tiy9CJxMpXSp8yRhNmvgJnLfeClOm+HENjz2mYWySOVT4klHKloUrr/Qz9xs08Nf1O3eG1atDJxNJPBW+ZKRDD4U33oD77oM5c3z533OPhrFJelPhS8bKyvL76Obn/7lmv21bP65BJB2p8CXj1awJU6f6LRVXrPDX+m++GX77LXQykfhS4Yvgh7GddZYfz9Ctm5/Hc/jhsGBB6GQi8aPCFylk//3h2Wdh4kTYsMFP4BwyBH7+OXQykdip8EWK0K2bP9s/+2w/a79JE/+qXZFUpsIX2Y6KFeHhh2H6dPj1VzjqKL/L1vffh04msmvKhA4QD8NeXMbyz78LHUPSWGQwVP4YJqyFKRdD3bp+OqdIvOQesDfXnVQ/oc+hM3yREsjKgoMPhqZNITvbb624YoW2VZTUkhZn+In+rShS2ObNfjbPLbfAun39CObTTvMrfUSSmc7wRXbSbrvBDTf4JZsHHghnnAHdu8Pnn4dOJrJjKnyRXdSoEbz7Lgwf7nfZys2F0aM1jE2SlwpfJAZlysAVV8DSpdC4Mfzzn9CpE3z0UehkIttS4YvEQZ068Npr8MADMG8eNGwII0fC77+HTibyJxW+SJxkZcGAAbBsGbRvD5deCkce6V/AJZIMVPgicVajBrz4IowdCytX+lfp3nCDf/GWSEgqfJEEMIMzz/Sjlnv0gOuug0gE5s8PnUwymQpfJIFycuCpp2DyZPj6a2jVCgYPhp9+Cp1MMpEKX6QUnHSSv7bfrx+MGOFX9LzxRuhUkmlU+CKlZJ99YNQomDnTb6XYrh2cfz58pzFQUkpU+CKlrEMHP4vnssv8L4D69eGll0KnkkygwhcJoEIFuP12eOcdf+Z/4onQuzds3Bg6maQzFb5IQC1bwsKFcP318NxzfjzD009rPIMkRsIL38y6mNn7ZrbKzIYk+vlEUk25cn7Z5sKFULu2X87ZtSt89lnoZClk7FioVcu/+q1WLf9YtpHQwjezbOA+4DggF+hlZrmJfE6RVNWggb/Ec8cdMGOGP9t/6CGd7Rdr7Fjo3x8++cT/ZX3yiX+s0t9Gos/wWwCrnHMfOed+BZ4Buib4OUVSVna2H8nw3nvQvLnvrY4d4cMPQydLYldfve0LG376yR+Xv0h04VcD1hR6vDZ67D/MrL+Z5ZlZ3kbdsRIB/O5aM2f6VTwLFvhhbLffrmFsRfr00507nsESXfhF7QH0l3+gOudGOecizrlITk5OguOIpA4zOO88P3ztmGPgX/+C1q0hPz90siRz4IE7dzyDJbrw1wI1Cj2uDmhfIJGdUK0aTJoEzzwDq1dDs2Z+VY+GsUXddJNf51pYhQr+uPxFogt/PlDHzA4ys3JAT2Bygp9TJO2Y+a0Uly+H00+HYcN88c+bFzpZEujd21/7qlnT/0XVrOkf9+4dOlnSSWjhO+e2ABcC04ACYJxzblkin1Mkne23Hzz5JEyZAt9+6y/xXH65hrHRu7f/58/Wrf69yr5ICV+H75yb6pyr65w72Dmnf2OJxMEJJ/hhbP37+2WcDRvCrFmhU0my0yttRVLU3nv7LRVff92/3qhjR/8LYNOm0MkkWanwRVLc0UfDkiV+M/XRo/0wtsm6UyZFUOGLpIEKFWD4cJg7FypX9qMZevaEDRtCJ5NkosIXSSORCOTl+T10J06EevX8TV6NZxBQ4YuknXLl4N//hkWLoG5d6NPHj19es6b475X0psIXSVO5ufDWWzBypL+xW7++v8m7dWvoZBKKCl8kjWVnw6BBfhxDy5ZwwQXQvj2sXBk6mYSgwhfJAAcdBK++6lfxLFkCjRr5m7xbtoROJqVJhS+SIczg3HP9eIYuXeDKK6FVK/8LQDKDCl8kwxxwADz/PIwb5ycIRyL+Ju/mzaGTSaKp8EUykBmcdhoUFECvXvA//wNNm8KcOaGTSSKp8EUyWOXK8PjjMHUq/PADtGkDl1wCP/4YOpkkggpfRDjuOD+M7fzz4a67/P66M2aETiXxpsIXEQD22gvuuw9mz/Yv3urUCfr10zC2dKLCF5G/aNvWr9wZMgTGjPEv4HrhhdCpJB5U+CKyjfLl4ZZb/I5aVapA9+5+p63160Mnk1io8EVku5o1g/nz/fawkyb5YWyPP65hbKlKhS8iO1S2LFx1lb/MU68e9O3rb/J+8knoZLKzVPgiUiKHHQZvvgn33OOHsjVo4G/yahhb6lDhi0iJZWXBhRf6YWxHHOE/PvpoeP/90MmkJFT4IrLTatWCV16Bxx7z6/cbN4Zbb4XffgudTHZEhS8iu8TMX89fvtxvsDJ0qB/BvHhx6GSyPSp8EYnJP/4B48f7t88/98PYrr4afvkldDL5OxW+iMTFqaf6s/0+feDmm6FJE3j77dCppDAVvojETaVK8OijMG2aP8Nv2xYuvtgPZpPwVPgiEnfHHutX8lx0Edx7r1/COW1a6FSiwheRhNhzTz958803Yffd/S5bZ58NX38dOlnmiqnwzew0M1tmZlvNLPK3zw01s1Vm9r6ZdY4tpoikqjZtYNEifyP3ySf9MLYJE0KnykyxnuHnA6cAswsfNLNcoCdQH+gC3G9m2TE+l4ikqPLl/a5aeXl+i8UePfxN3nXrQifLLDEVvnOuwDlX1GvsugLPOOc2O+c+BlYBLWJ5LhFJfU2a+Amct94KL73kz/YffVTD2EpLoq7hVwPWFHq8NnpsG2bW38zyzCxv48aNCYojIsmiTBm48kpYuhQaNoRzz4XOnWH16tDJ0l+xhW9mM8wsv4i3rjv6tiKOFfk73Dk3yjkXcc5FcnJySppbRFJc3brw+ut+ANucOX4lz913w++/h06WvootfOfcMc65BkW8TdrBt60FahR6XB34PNawIpJesrLgggv8PJ62bWHQIDjqKCgoCJ0sPSXqks5koKeZ7WZmBwF1gHkJei4RSXEHHghTp/rNVVas8Nf6b7pJw9jiLdZlmd3NbC3QGnjJzKYBOOeWAeOA5cArwEDnnP6hJiLbZebHMixfDt26wTXXwOGHw4IFoZOlj1hX6Ux0zlV3zu3mnNvfOde50Oducs4d7Jw71Dn3cuxRRSQT7L8/PPssTJwIGzb4CZxXXgk//xw6WerTK21FJCl16+bP9s8+G4YP9zP3Z88u9ttkB1T4IpK0KlaEhx+GGTNgyxa/u9bAgfDdd6GTpSYVvogkvY4d4b334NJL4YEH/BLOl3WheKep8EUkJeyxB9xxB7zzDuy1Fxx/PJx1Fnz1VehkqUOFLyIppVUrWLgQrr0Wnn4a6tWDceM0nqEkVPgiknJ22w2GDfNLNmvWhDPOgFNO8Vssyvap8EUkZTVq5McyjBgBr7zih7GNHq2z/e1R4YtISitTBv71L39Tt0kT+Oc/oVMn+Oij0MmSjwpfRNLCIYfArFnw4IN+BHPDhjBypIaxFabCF5G0kZUF//3f/gVb7dv7ZZxt2vjhbKLCF5E0VL06vPgiPPUUfPghNG0KN9wAv/4aOllYKnwRSUtm0KuXP9vv0QOuuw6aN4f580MnC0eFLyJpLSfHn+lPngzffOPX8V9xBfz0U+hkpU+FLyIZ4aST/LX8886D//1fP4zt9ddDpypdKnwRyRj77ONX8cya5dfqt28PAwbAt9+GTlY6VPgiknHat/ebqF9+OTz0ENSvD1OmhE6VeCp8EclIFSr4Sztz5sC++/pLPmeeCRs3hk6WOCp8EcloLVr4mTzXXw/jx/vxDE8/nZ7jGVT4IpLxypXzyzYXLoTatf2Z/sknw9q1oZPFlwpfRCSqQQM/b/+OO2DmTH9tf9Qo2Lo1dLL4UOGLiBSSne1HMuTnQyTiRzV07AirVoVOFjsVvohIEWrX9nvpPvSQv9TTsKG/ybtlS+hku06FLyKyHWZ+3PLy5XDssf4Vuq1b+1HMqUiFLyJSjGrV4IUX4Nln4ZNPoFkzf5N38+bQyXaOCl9EpATM4PTToaAAevb00zebN4e5c0MnKzkVvojITqhcGZ54Al56yY9kaN0aLrsMfvwxdLLiqfBFRHbB8cf7YWwDBsCdd/r9dWfNCp1qx2IqfDMbYWYrzGypmU00s4qFPjfUzFaZ2ftm1jn2qCIiyWXvveH++/3Uzawsv3zzvPNg06bQyYoW6xn+dKCBc64R8AEwFMDMcoGeQH2gC3C/mWXH+FwiIknp6KP9MLbBg+GRR/x4hkmTQqfaVkyF75x71Tn3x6rUd4Hq0Y+7As845zY75z4GVgEtYnkuEZFktvvucNtt/iZuTg506+Zv7m7YEDrZn+J5Df9c4OXox9WANYU+tzZ6bBtm1t/M8swsb2M6j6kTkYwQiUBeHtx4I0ycCPXqwZNPJscwtmIL38xmmFl+EW9dC33N1cAWYOwfh4r4TxX5x3XOjXLORZxzkZycnF35M4iIJJWyZeGaa2DRIqhbF/r0gRNPhDVriv/eRCpT3Bc4547Z0efNrC9wItDRuf/8DlsL1Cj0ZdWBz3c1pIhIKsrNhbfegvvug6FD/TC2227z83myAqyRjHWVThfgSuBk51zhLYEnAz3NbDczOwioA8yL5blERFJRdjZcfLEfxtayJVxwgd9x64MPSj9LrL9j7gX2Aqab2WIzexDAObcMGAcsB14BBjrnfo/xuUREUtZBB8Grr8Lo0X5FT+PGMHx46Q5jM5cMdxKiIpGIy8vLCx1DRCSh1q2DgQP9Td1mzfxSzsaNd/2/Z2YLnHOR4r5Or7QVESllVavChAnw3HN+V61IBEaOTPzzqvBFRAIwgx49/DC23r3h4IMT/5zFrtIREZHEqVQJHnusdJ5LZ/giIhlChS8ikiFU+CIiGUKFLyKSIVT4IiIZQoUvIpIhVPgiIhlChS8ikiGSapaOmW0EPtnFb98P+DKOcUqTsoeh7GGkavZkzl3TOVfshiJJVfixMLO8kgwPSkbKHoayh5Gq2VM1d2G6pCMikiFU+CIiGSKdCn9U6AAxUPYwlD2MVM2eqrn/I22u4YuIyI6l0xm+iIjsQFoUvpl1MbP3zWyVmQ0JnWdHzOwRM9tgZvmFjlUys+lmtjL6ft+QGbfHzGqY2WtmVmBmy8xsUPR4Uuc3s/JmNs/MlkRzD4seP8jM5kZzP2tm5UJn3R4zyzazRWY2Jfo4JbKb2Wozey+653Ve9FhS/7z8wcwqmtl4M1sR/ZlvnSrZtyflC9/MsoH7gOOAXKCXmeWGTbVDjwFd/nZsCDDTOVcHmBl9nIy2AJc75+oBrYCB0b/rZM+/GejgnGsMNAG6mFkr4Dbgzmjub4B+ATMWZxBQUOhxKmVv75xrUmhJY7L/vPzhLuAV59xhQGP833+qZC+acy6l34DWwLRCj4cCQ0PnKiZzLSC/0OP3garRj6sC74fOWMI/xySgUyrlByoAC4GW+BfRlCnq5yiZ3oDq+HLpAEwBLIWyrwb2+9uxpP95AfYGPiZ6nzOVsu/oLeXP8IFqwJpCj9dGj6WS/Z1z6wCi76sEzlMsM6sFNAXmkgL5o5dEFgMbgOnAh8Am59yW6Jck88/NSGAwsDX6uDKpk90Br5rZAjPrHz2W9D8vQG1gI/Bo9FLaw2a2B6mRfbvSofCtiGNaepRAZrYnMAG4xDn3Xeg8JeGc+9051wR/ttwCqFfUl5VuquKZ2YnABufcgsKHi/jSpMse1cY51wx/yXWgmR0VOlAJlQGaAQ8455oCP5Jql2+KkA6FvxaoUehxdeDzQFl21XozqwoQfb8hcJ7tMrOy+LIf65x7Pno4ZfI75zYBr+PvQVQ0szLRTyXrz00b4GQzWw08g7+sM5LUyI5z7vPo+w3ARPwv21T4eVkLrHXOzY0+Ho//BZAK2bcrHQp/PlAnumqhHNATmBw4086aDPSNftwXf2086ZiZAaOBAufcHYU+ldT5zSzHzCpGP94dOAZ/A+41oEf0y5IuN4Bzbqhzrrpzrhb+Z3uWc643KZDdzPYws73++Bg4FsgnyX9eAJxzXwBrzOzQ6KGOwHJSIPsOhb6JEKcbLMcDH+Cvy14dOk8xWZ8G1gG/4c8i+uGvyc4EVkbfVwqdczvZj8RfOlgKLI6+HZ/s+YFGwKJo7nzg2ujx2sA8YBXwHLBb6KzF/DnaAVNSJXs045Lo27I//t9M9p+XQvmbAHnRn5sXgH1TJfv23vRKWxGRDJEOl3RERKQEVPgiIhlChS8ikiFU+CIiGUKFLyKSIVT4IiIZQoUvIpIhVPgiIhni/wHixbk9WFC6lgAAAABJRU5ErkJggg==\n", "text/plain": [ "
" ] }, "metadata": { "needs_background": "light" }, "output_type": "display_data" }, { "data": { "image/png": "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\n", "text/plain": [ "
" ] }, "metadata": { "needs_background": "light" }, "output_type": "display_data" }, { "data": { "image/png": "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\n", "text/plain": [ "
" ] }, "metadata": { "needs_background": "light" }, "output_type": "display_data" }, { "data": { "image/png": "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\n", "text/plain": [ "
" ] }, "metadata": { "needs_background": "light" }, "output_type": "display_data" }, { "data": { "image/png": "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\n", "text/plain": [ "
" ] }, "metadata": { "needs_background": "light" }, "output_type": "display_data" }, { "data": { "image/png": "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\n", "text/plain": [ "
" ] }, "metadata": { "needs_background": "light" }, "output_type": "display_data" }, { "data": { "image/png": "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\n", "text/plain": [ "
" ] }, "metadata": { "needs_background": "light" }, "output_type": "display_data" }, { "data": { "image/png": "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\n", "text/plain": [ "
" ] }, "metadata": { "needs_background": "light" }, "output_type": "display_data" }, { "data": { "image/png": "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\n", "text/plain": [ "
" ] }, "metadata": { "needs_background": "light" }, "output_type": "display_data" }, { "data": { "image/png": "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\n", "text/plain": [ "
" ] }, "metadata": { "needs_background": "light" }, "output_type": "display_data" }, { "data": { "image/png": "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\n", "text/plain": [ "
" ] }, "metadata": { "needs_background": "light" }, "output_type": "display_data" }, { "data": { "image/png": "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\n", "text/plain": [ "
" ] }, "metadata": { "needs_background": "light" }, "output_type": "display_data" }, { "data": { "image/png": "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\n", "text/plain": [ "
" ] }, "metadata": { "needs_background": "light" }, "output_type": "display_data" }, { "data": { "image/png": "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\n", "text/plain": [ "
" ] }, "metadata": { "needs_background": "light" }, "output_type": "display_data" }, { "data": { "image/png": "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\n", "text/plain": [ "
" ] }, "metadata": { "needs_background": "light" }, "output_type": "display_data" }, { "data": { "image/png": "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\n", "text/plain": [ "
" ] }, "metadata": { "needs_background": "light" }, "output_type": "display_data" }, { "data": { "image/png": "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\n", "text/plain": [ "
" ] }, "metadata": { "needs_background": "light" }, "output_type": "display_data" }, { "data": { "image/png": "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\n", "text/plain": [ "
" ] }, "metadata": { "needs_background": "light" }, "output_type": "display_data" }, { "data": { "image/png": "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\n", "text/plain": [ "
" ] }, "metadata": { "needs_background": "light" }, "output_type": "display_data" }, { "data": { "image/png": "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\n", "text/plain": [ "
" ] }, "metadata": { "needs_background": "light" }, "output_type": "display_data" }, { "data": { "image/png": "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\n", "text/plain": [ "
" ] }, "metadata": { "needs_background": "light" }, "output_type": "display_data" }, { "data": { "image/png": "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\n", "text/plain": [ "
" ] }, "metadata": { "needs_background": "light" }, "output_type": "display_data" }, { "data": { "image/png": "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\n", "text/plain": [ "
" ] }, "metadata": { "needs_background": "light" }, "output_type": "display_data" }, { "data": { "image/png": "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\n", "text/plain": [ "
" ] }, "metadata": { "needs_background": "light" }, "output_type": "display_data" }, { "data": { "image/png": "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\n", "text/plain": [ "
" ] }, "metadata": { "needs_background": "light" }, "output_type": "display_data" }, { "data": { "image/png": "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\n", "text/plain": [ "
" ] }, "metadata": { "needs_background": "light" }, "output_type": "display_data" }, { "data": { "image/png": "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\n", "text/plain": [ "
" ] }, "metadata": { "needs_background": "light" }, "output_type": "display_data" }, { "data": { "image/png": "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\n", "text/plain": [ "
" ] }, "metadata": { "needs_background": "light" }, "output_type": "display_data" }, { "data": { "image/png": "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\n", "text/plain": [ "
" ] }, "metadata": { "needs_background": "light" }, "output_type": "display_data" }, { "data": { "image/png": "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\n", "text/plain": [ "
" ] }, "metadata": { "needs_background": "light" }, "output_type": "display_data" }, { "data": { "image/png": "iVBORw0KGgoAAAANSUhEUgAAAXwAAAD8CAYAAAB0IB+mAAAABHNCSVQICAgIfAhkiAAAAAlwSFlzAAALEgAACxIB0t1+/AAAADl0RVh0U29mdHdhcmUAbWF0cGxvdGxpYiB2ZXJzaW9uIDMuMC4zLCBodHRwOi8vbWF0cGxvdGxpYi5vcmcvnQurowAAIABJREFUeJzt3Xu8lXP6//HX1a4kZ7VD5xDanWvpIJESUdOBmNhDyISJcZwURoqc+iFnE1HjEDmWMqjkTOykdOBbk6jRqEHOp7h+f3xWYw+7015r7Xutdb+fj8d+7HXfa619X2uvutZnf+7PfV3m7oiISP6rFHUAIiJSMZTwRURiQglfRCQmlPBFRGJCCV9EJCaU8EVEYkIJX0QkJpTwRURiQglfRCQmKkcdQGk1a9b0hg0bRh2GiEhOmTt37n/cvXBzj8uqhN+wYUNKSkqiDkNEJKeY2Qdb8jhN6YiIxERaRvhmtgL4EvgJWO/uCTPbFXgIaAisAI5198/ScTwREdl66RzhH+Lurdw9kdweBsxy98bArOS2iIhEJJNTOn2AicnbE4G+GTyWiIhsRroSvgPPmtlcMxuc3Lebu68GSH6vlaZjiYhIOaRrlU4nd//IzGoBM8zs3S19YvIDYjBA/fr10xSOiIj8WlpG+O7+UfL7GuBxoB3wsZntAZD8vmYjzx3n7gl3TxQWbnYZqYiIlFPKCd/MtjOzHTbcBg4DFgJTgYHJhw0EpqR6rI1ZuxbOPRc+/zxTRxARyX3pGOHvBrxsZvOBN4Dp7v40cDXQ3cyWAt2T2xkxaxbcdBMUFcGTT2bqKCIiuS3lOXx3Xw60LGP/J0C3VH/+lhgwAPbaCwYNgt694bjj4MYbQTNEIiK/yJsrbfffH0pKYORIeOQRaNIEHngA3KOOTEQkO+RNwgeoWhUuvRTmzYO994bi4jDiX7Uq6shERKKXVwl/g6ZN4ZVX4Prr4bnnwtz+3/4GP/8cdWQiItHJy4QPUFAQVu688w60awennw7dusGyZVFHJiISjbxN+BvsuSfMmAF33RWmepo3hzFjYP36qCMTEalYeZ/wAczCCp7Fi+Hww2HoUOjYERYsiDoyEZGKE4uEv0Ht2vD44/DQQ/DBB9C2LYwYAd9/H3VkIiKZF6uED2G0f+yxsGRJWL8/ahS0aQOvvx51ZCIimRW7hL9BjRpw770wfTp88QUccEA4yfv111FHJiKSGbFN+BsceSQsWhRW8YwdG07qzpwZdVQiIukX+4QPsOOOcNtt8MILULkydO8eTvKuWxd1ZCIi6aOEX8pBB8H8+XDhhTBxYrhg64knoo5KRCQ9lPB/Zdtt4eqrYc4cqFUL+vULJ3k//jjqyEREUqOEvxFt28Kbb8IVV8CUKWG0f++9KsYmIrlLCX8TqlSBiy+Gt9+GffeFE0+Enj3hww+jjkxEZOsp4W+BJk3gpZdCjf0XXwzF2W67TcXYRCS3pC3hm1mBmc0zs2nJ7UZmNsfMlprZQ2ZWNV3HikJBAfz5z7BwYSjLMGQIdOkC770XdWQiIlsmnSP8s4ElpbavAW5w98bAZ8CgNB4rMg0bwjPPwD33hEqcLVvCNdeoGJuIZL+0JHwzqwv0BO5KbhvQFXgk+ZCJQN90HCsbmMFJJ4XyDD17wrBh0L59mOsXEclW6RrhjwWGAhtmtWsA69x9w7h3FVCnrCea2WAzKzGzkrVr16YpnIqx++7w6KOhpeK//hXaLF5yCXz3XdSRiYj8VsoJ38x6AWvcfW7p3WU8tMwFje4+zt0T7p4ozNGu40cfHUovFxfD6NHQujW8+mrUUYmI/K90jPA7Ab3NbAXwIGEqZyyws5lVTj6mLvBRGo6VtXbdFSZMgKefhm++gQMPDCd5v/oq6shERIKUE767D3f3uu7eEBgAPOfuxcBsoH/yYQOBKakeKxccfnhYyTNkCNxyCzRrBs8+G3VUIiKZXYd/IXCemS0jzOmPz+CxssoOO8DNN4c1+9WqhQ+Bk0+GTz+NOjIRibO0Jnx3f97deyVvL3f3du6+t7sf4+6x6yt14IFh5c5FF4WyDEVF4SSviEgUdKVthlWrFk7klpSEFov9+4evf/876shEJG6U8CtIq1ahAudVV8G0aWG0P2GCirGJSMVRwq9AVaqEi7Tmzw/1eE4+GXr0gBUroo5MROJACT8C++4bumvdcktYr9+sWTjJq2JsIpJJSvgRqVQpLN1cuPCXNfudO4dyDSIimaCEH7EGDeAf/wgtFZcsCXP9V14JP/4YdWQikm+U8LOAWWiusmQJ9OkTmq60awdvvRV1ZCKST5Tws8huu8HkyfDYY2HZZrt2MHw4fPtt1JGJSD5Qws9C/fqFYmwDB4aG6q1awcsvRx2ViOQ6JfwstcsuMH48zJgBP/wQTuieeSZ8+WXUkYlIrlLCz3KHHho6a519duij27RpqMgpIrK1lPBzwPbbw9ix8Mor4fYRR4Tpnk8+iToyEcklSvg5pGNHmDcP/vpXeOCBUJ7h4YdVnkFEtowSfo7ZZhsYNSoUY6tXD449Fo46ClavjjoyEcl2Svg5qmVLeP11uPbaMKdfVAR3363RvohsXDp62lYzszfMbL6ZLTKzkcn9jcxsjpktNbOHzKxq6uFKaZUrw1/+AgsWQIsWMGgQHHYYvP9+1JGJSDZKxwj/e6Cru7cEWgE9zKwDcA1wg7s3Bj4DBqXhWFKGxo1h9my4/fZQgrlZM7jxRvjpp6gjE5Fsko6etu7uG1p1V0l+OaGZ+SPJ/ROBvqkeSzauUiU4/XRYtAi6dIFzzglF2RYvjjoyEckWaZnDN7MCM3sbWAPMAP4JrHP39cmHrALqpONYsmn16oUGK/fdB0uXQuvWcPnl4eItEYm3tCR8d//J3VsBdYF2QJOyHlbWc81ssJmVmFnJ2rVr0xFO7JlBcXEY3ffrB5deColEWNkjIvGV7ibm64DngQ7AzmZWOXlXXeCjjTxnnLsn3D1RWFiYznBir1YtePBBmDIlXKTVvj0MHapibCJxlY5VOoVmtnPy9rbAocASYDbQP/mwgcCUVI8l5dO7d5jbHzQIxowJK3peeCHqqESkoqVjhL8HMNvMFgBvAjPcfRpwIXCemS0DagDj03AsKaedd4Zx42DWrNBKsUsXOOMM+OKLqCMTkYpinkVX6iQSCS/RRHPGffNNKM8wdizUrg133AE9e0YdlYiUl5nNdffE5h6nK21jqHp1uO660EB9p52gV69wkvc//4k6MhHJJCX8GGvfPrRRvOyyUIStSZNwkjeL/ugTkTRSwo+5qlVhxIiQ+Bs1guOOg7594V//ijoyEUk3JXwBQjmG114LUz0zZoRibHfeqdG+SD5Rwpf/KiiA884LxdjatIHBg6FbN/jnP6OOTETSQQlffmPvvcPyzb/9DebOhebN4frrVYxNJNcp4UuZKlUKI/xFi8Io//zz4YADQn9dEclNSviySXXrwtSpMGkSLF8ObduGVT0qxiaSe5TwZbPMYMAAWLIktFQcOTLM8b/xRtSRicjWUMKXLVazZii7PG0afP55aKp+wQXhyl0RyX5K+LLVevYMc/uDB4dlnM2bh45bIpLdlPClXHbcMbRUfP75cIK3a9fwAfD551FHJiIbo4QvKTn4YJg/P0ztjB8fLth68smooxKRsijhS8qqVw919ufMgRo1Qv39444DNTATyS5K+JI2G9oojhoFjz0WirE98IDKM4hkCyV8SauqVUOt/XnzoHHjUHb5d7+DlSujjkxE0tHisJ6ZzTazJWa2yMzOTu7f1cxmmNnS5PddUg9XckVREbz8MtxwQ1jB07RpaLTy889RRyYSX+kY4a8Hznf3JoTm5UPMrAgYBsxy98bArOS2xEhBAZxzTijH0K5daKnYtSssXRp1ZCLxlHLCd/fV7v5W8vaXhAbmdYA+wMTkwyYCfVM9luSmPfcMJZfvvBPefjs0UR8zBtavjzoykXhJ6xy+mTUEWgNzgN3cfTWEDwWg1kaeM9jMSsysZK2WdeQtMzj1VFi8GHr0gKFDoUOHsKRTRCpG2hK+mW0PPAqc4+5fbOnz3H2cuyfcPVFYWJiucCRL1a4dVvBMnhxO5CYS4STv999HHZlI/ktLwjezKoRkf7+7P5bc/bGZ7ZG8fw9gTTqOJbnPDI45Joz2jz8errgCWrcOHbdEJHPSsUrHgPHAEne/vtRdU4GBydsDgSmpHkvyS40aMHEiPPUUfPUVdOoUTvJ+/XXUkYnkp3SM8DsBJwBdzezt5NeRwNVAdzNbCnRPbov8xhFHhGJsf/oT3HhjKMY2c2bUUYnkn3Ss0nnZ3c3dW7h7q+TXU+7+ibt3c/fGye+fpiNgyU877AC33AIvvgiVK0P37jBoEKxbF3VkIvlDV9pKVuncOazcGTYsTPcUFcETT0QdlUh+UMKXrLPttnDVVaGj1m67Qb9+odPWxx9HHZlIblPCl6y1oY3i6NEwZUoY7d97r4qxiZSXEr5ktSpV4KKLwjTPfvvBiSfCkUfChx9GHZlI7lHCl5yw337hhO5NN8FLL4VibLfeqmJsIltDCV9yRkEBnHUWLFwYGqifeWbouPXee1FHJpIblPAl5zRsCM88A/fcE5J/y5Zw9dXw449RRyaS3ZTwJSeZwUknwZIl0KsXDB8O7duHxisiUjYlfMlpu+8OjzwCjz4KH30E++8PF18M330XdWQi2UcJX/LCUUeFYmwnnABXXgmtWsErr0QdlUh2UcKXvLHrrmFe/5lnwgi/c2f4859DYTYRUcKXPHTYYeFk7plnhvo8zZrBs89GHZVI9JTwJS9tv/0va/arVYPDD4eTT4ZPVcJPYkwJX/Jap06hj+7FF4eyDEVF4QSvSBwp4Uveq1YtdNUqKYE6daB/fzj6aFi9OurIRCpWuloc3m1ma8xsYal9u5rZDDNbmvy+SzqOJVJerVrBnDnhIq3p08No/557VIxN4iNdI/wJQI9f7RsGzHL3xsCs5LZIpCpXhgsvDMXYmjWDU04J8/srVkQdmUjmpSXhu/uLwK9Ph/UBJiZvTwT6puNYIumw777wwguhANtrr4Xkf/PNKsYm+S2Tc/i7uftqgOT3Whk8lshWq1Qp9NFdtOiXNfudO4dyDSL5KPKTtmY22MxKzKxk7dq1UYcjMVS/Pjz1FPz97/Duu2Guf/RoFWOT/JPJhP+xme0BkPy+pqwHufs4d0+4e6KwsDCD4YhsnFkoy7B4MfTtC5dcEuryvPVW1JGJpE8mE/5UYGDy9kBgSgaPJZIWu+0GDz0Ejz8Oa9ZAu3ahofq330YdmUjq0rUscxLwGrCvma0ys0HA1UB3M1sKdE9ui+SEvn3D3P7AgXDNNWGa56WXoo5KJDXmWbQIOZFIeElJyVY/b+STi1j80RcZiEgEPvsM/u//QkG22rVhzz1D9y2RdCqqvSMjfte0XM81s7nuntjc4yI/aSuS7XbZJczn16kbau6/+aZq8khuqhx1AOlQ3k9Fka312mswaBDMnBBO8t5wA9SoEXVUIltGI3yRrdCxY2ijeMklMGkSNGkCkyerPIPkBiV8ka20zTZw+eWhGFv9+vD730O/fmG6RySbKeGLlFPLlvD663DttaHLVlERjB+v0b5kLyV8kRRUrgx/+QssWBA+AE49Fbp3h+XLo45M5LeU8EXSoHFjmD0b7rgD3ngDmjeHsWPhp5+ijkzkF0r4ImlSqRKcdlq4YKtLFzj3XDjwwFCuQSQbKOGLpFm9ejBtGtx3HyxdCq1bw6hR8MMPUUcmcaeEL5IBZlBcHEb3Rx0FI0ZAIhEu2hKJihK+SAbVqhXW60+ZAp98Ah06wNCh8M03UUcmcaSEL1IBevcOo/1Bg2DMmLCi54UXoo5K4kYJX6SC7LQTjBsHs2aFVopdusAZZ8AXqvsnFUQJX6SCde0K77wD550XPgCaNoXp06OOSuJACV8kAtWrw3XXwauvhpF/r17hJK+6fEomKeGLRKh9+9BGccQIePjhUJ5h0iSVZ5DMyHjCN7MeZvaemS0zs2GZPp5IrqlaFS67DObOhUaN4Pjjw0neVauijiyL3H8/NGwYrm5r2DBsy1bLaMI3swLgVuAIoAg4zsyKMnlMkVzVvHmot3/ddeHEbtOmYY4/9qP9+++HwYPhgw/CL+ODD8K2kv5Wy/QIvx2wzN2Xu/sPwINAnwwfUyRnFRSEk7nvvANt24ZSDd26wT//GXVkEbr44t9euPDNN2G/bJVMJ/w6wMpS26uS+/7LzAabWYmZlazVGSsRAPbaK4zyx40LUz3Nm8P118e0GNuHH27dftmoTCd8K2Pf//yB6u7j3D3h7onCwsIMhyOSO8zgj38MF2wdeiicfz4ccAAsXBh1ZBWsfv2t2y8blemEvwqoV2q7LqC+QCJboU6dUJrhwQfh/fehTRsYOTJGxdhGjw7rWEurXj3sl62S6YT/JtDYzBqZWVVgADA1w8cUyTtmoZXi4sVw7LFhVU/btqH2ft4rLg5zWw0ahF9EgwZhu7g46shyTkYTvruvB84EngGWAJPdfVEmjymSz2rWDGWXp02DdetCU/Xzz49BMbbiYlixItSkWLFCyb6cMr4O392fcvd93H0vd9ffYCJp0LNnaLQyeHA4mdu8eei4JbIputJWJEftuCPcfjs8/3y4Hqlr1/ABsG5d1JFJtlLCF8lxBx8M8+eHZurjx4cLtqbqTJmUQQlfJA9Urw7XXgtz5kCNGtCnDwwYAGvWRB2ZZBMlfJE8kkhASUnoofvYY9CkSTjJG/vyDAIo4YvknapV4a9/hbffhn32gRNOCOWXV67c/HMlvynhi+SpoiJ4+WUYOzac2G3aNJzk/fnnqCOTqCjhi+SxggI4++xQjqF9e/jTn+CQQ2Dp0qgjkygo4YvEQKNG8OyzYRXP/PnQokU4ybt+fdSRSUVSwheJCTM45ZRQnqFHD7jwQujQIXwASDwo4YvETO3aYQXPww+HE7mJRDjJ+/33UUcmmaaELxJDZtC/fxjtH388XHEFtG4dOm5J/lLCF4mxGjVg4kR46in46ivo1AnOOSfclvyjhC8iHHFEKMZ2xhlw442hGNuMGVFHJemmhC8iAOywA9x6K7z4Yrh467DDYNAg+OyzqCOTdFHCF5H/0blzWLkzbFiY7ikqgscfjzoqSYeUEr6ZHWNmi8zsZzNL/Oq+4Wa2zMzeM7PDUwtTRCpStWpw1VWho9buu8NRR8Exx8C//x11ZJKKVEf4C4GjgBdL7zSzIkI7w6ZAD+A2MytI8VgiUsHatAlJ/8or4cknw2j/739XMbZclVLCd/cl7v5eGXf1AR509+/d/X1gGdAulWOJSDSqVIHhw0MxtiZNYODAcJL3gw+ijky2Vqbm8OsApWvzrUruE5Ectd9+8NJLcPPNoShbs2bhJK+KseWOzSZ8M5tpZgvL+OqzqaeVsa/MPwLNbLCZlZhZydq1a7c0bhGJQKVKcOaZYQnnAQeE2wcfDO+V9Xe+ZJ3NJnx3P9Tdm5XxNWUTT1sF1Cu1XRf4aCM/f5y7J9w9UVhYuHXRi0gkGjSAp5+GCRNC8m/ZEq6+Gn78MerIZFMyNaUzFRhgZtuYWSOgMfBGho4lIhEwC/P5ixeHBivDh4cSzPPmRR2ZbEyqyzL7mdkqoCMw3cyeAXD3RcBkYDHwNDDE3X9KNVgRyT677w6PPBK+PvoI9t8fLroIvvsu6sjk18yzaH1VIpHwkpKSqMMQkXL67DM477ww1bPvvqH+fqdOUUeV/8xsrrsnNvc4XWkrImmzyy5wzz3wzDNhhN+5M5x1Fnz5ZdSRCSjhi0gGHHZYaKt41llh6WazZuFDQKKlhC8iGbH99qHy5ksvQfXqocvWSSfBp59GHVl8KeGLSEZ16hRW7lx0Edx3XyjP8OijUUcVT0r4IpJx1arB6NFQUgJ16oRuW0cfDatXRx1ZvCjhi0iFadUK5syBa66B6dPDaH/CBBVjqyhK+CJSoSpXhqFDYcGC0Fnr5JPh8MNhxYqoI8t/SvgiEol99oHnn4fbbgvN05s1g5tugp90iWbGKOGLSGQqVQp9dBctCmv2zz4bDjoIliyJOrL8pIQvIpGrXx+eegruvRfefTfM9Y8erWJs6aaELyJZwQz+8Icwuu/bFy65JNTlmTs36sjyhxK+iGSVWrXgoYdC4/Q1a0IFzmHD4Ntvo44s9ynhi0hW6ts3lF4+6aSwjLNlS3jxxc0+TTZBCV9EstbOO8Ndd8HMmbB+feiuNWQIfPFF1JHlJiV8Ecl63brBO+/AuefC7beHJZz/+EfUUeUeJXwRyQnbbQfXXw+vvgo77ABHHgknngiffBJ1ZLkj1Y5XY8zsXTNbYGaPm9nOpe4bbmbLzOw9Mzs89VBFRKBDB3jrLbj0Upg0CZo0gcmTVZ5hS6Q6wp8BNHP3FsD/AcMBzKwIGAA0BXoAt5lZQYrHEhEBYJttYOTIsGSzQQP4/e+hX7/QYlE2LqWE7+7Puvv65ObrQN3k7T7Ag+7+vbu/DywD2qVyLBGRX2vRIpRlGDMmNFgpKgptFTXaL1s65/BPATacRqkDrCx136rkPhGRtKpcGS64IJzUbdUKTj0VDj0Uli+POrLss9mEb2YzzWxhGV99Sj3mYmA9cP+GXWX8qDI/c81ssJmVmFnJ2rVry/MaRETYe2947jm44w54881QifOGG1SMrbTNJnx3P9Tdm5XxNQXAzAYCvYBi9//+IbUKqFfqx9QFypxdc/dx7p5w90RhYWFqr0ZEYq1SJTjttHDB1iGHwHnnhY5bixZFHVl2SHWVTg/gQqC3u39T6q6pwAAz28bMGgGNgTdSOZaIyJaqWxeefDK0VFy2DFq3hlGj4Icfoo4sWqnO4d8C7ADMMLO3zewOAHdfBEwGFgNPA0PcXX9YiUiFMYPi4lCMrX9/GDEC2rYN0z1xZZ5Fp7MTiYSXlJREHYaI5KEnnwy191evDlfsjhoF1atHHVV6mNlcd09s7nG60lZEYuF3vwtz+X/8I1x3XVjS+fzzUUdVsZTwRSQ2dtoprOKZPTtsH3IInH46fP55tHFVFCV8EYmdLl1CE/ULLoA774SmTWHatKijyjwlfBGJperVwxW6r70Gu+wSpnyKiyGfLwdSwheRWGvXLtTkuewyePjhUJ5h0qT8LM+ghC8isVe1ali2+dZbsOeecPzx0Ls3rFoVdWTppYQvIpLUrFmot3/ddTBrVpjbHzcOfv456sjSQwlfRKSUgoJQkuGdd8KFWqedFjpuLVsWdWSpU8IXESnDXnuFUf6dd4apnhYtwsg/l4uxKeGLiGyEWSi3vHgxdO8elnF27AgLF0YdWfko4YuIbEadOvDEE/Dgg7BiBbRpE1b15FoxNiV8EZEtYBZaKS5eHL6PHBkS/5w5UUe25ZTwRUS2Qs2acO+9MH16KMnQsWM4yfv111FHtnlK+CIi5XDkkaEY2+mnh85aLVqEjlvZTAlfRKScdtwRbrsNXnghLOfs1i1U41y3LurIypZqx6vLzWxBsvnJs2ZWO7nfzOwmM1uWvL9NesIVEck+Bx0E8+fD0KFw993hgq2pU6OO6rdSHeGPcfcW7t4KmAZcmtx/BKGtYWNgMHB7iscREclq224L11wTTuLWrAl9+sCAAbBmTdSR/SKlhO/uX5Ta3A7YUG6oD/B3D14HdjazPVI5lohILkgkoKQELr8cHn8cmjQJvXWzoRhbynP4ZjbazFYCxfwywq8DrCz1sFXJfSIiea9KFbjkEpg3D/bZB044AXr1gpUrN//cTNpswjezmWa2sIyvPgDufrG71wPuB87c8LQyflSZn29mNtjMSsysZG0+F6IWkdgpKoKXX4axY0M7xaZN4fbboyvGlrYm5mbWAJju7s3M7G/A8+4+KXnfe0AXd1+9qZ+hJuYikq/efx8GD4aZM8NJ3jvvDKP/dKiQJuZm1rjUZm/g3eTtqcCJydU6HYDPN5fsRUTyWaNG8OyzMH58WNHTsiVcey2sX19xMaQ6h391cnpnAXAYcHZy/1PAcmAZcCfwpxSPIyKS88zglFNCeYYePeDCC6F9+/ABUCHHT9eUTjpoSkdE4sIdHn0UhgyBTz8N/XXPOad8P6tCpnRERKR8zKB//zDaLy4O9fczrXLmDyEiIhtTowZMmFAxx9IIX0QkJpTwRURiQglfRCQmlPBFRGJCCV9EJCaU8EVEYkIJX0QkJpTwRURiIqtKK5jZWuCDcj69JvCfNIaTa+L++kG/A73++L7+Bu5euLkHZVXCT4WZlWxJLYl8FffXD/od6PXH+/VvCU3piIjEhBK+iEhM5FPCHxd1ABGL++sH/Q70+mWT8mYOX0RENi2fRvgiIrIJeZHwzayHmb1nZsvMbFjU8WSamdUzs9lmtsTMFpnZ2cn9u5rZDDNbmvy+S9SxZpKZFZjZPDObltxuZGZzkq//ITOrGnWMmWJmO5vZI2b2bvLfQcc4vf9mdm7y3/5CM5tkZtXi9P6XV84nfDMrAG4FjgCKgOPMrCjaqDJuPXC+uzcBOgBDkq95GDDL3RsDs5Lb+exsYEmp7WuAG5Kv/zNgUCRRVYwbgafdfT+gJeH3EIv338zqAH8GEu7eDCgABhCv979ccj7hA+2AZe6+3N1/AB4E+kQcU0a5+2p3fyt5+0vCf/Y6hNc9MfmwiUDfaCLMPDOrC/QE7kpuG9AVeCT5kLx9/Wa2I3AQMB7A3X9w93XE6P0ndOvb1swqA9WB1cTk/U9FPiT8OsDKUturkvtiwcwaAq2BOcBu7r4awocCUCu6yDJuLDAU+Dm5XQNY5+7rk9v5/O9gT2AtcE9ySusuM9uOmLz/7v4v4P8BHxIS/efAXOLz/pdbPiR8K2NfLJYemdn2wKPAOe7+RdTxVBQz6wWscfe5pXeX8dB8/XdQGWgD3O7urYGvydPpm7Ikz030ARoBtYHtCFO6v5av73+55UPCXwXUK7VdF/goolgqjJlVIST7+939seTuj81sj+T9ewBrooovwzoBvc1sBWEKrythxL9z8k98yO9/B6uAVe4+J7n9COEDIC7v/6HA++6+1t1/BB4DDiA+73+55UPCfxNonDxDX5Vw8mZqxDFlVHK+ejywxN2vL3XXVGDS0hhuAAAA50lEQVRg8vZAYEpFx1YR3H24u9d194aE9/s5dy8GZgP9kw/L59f/b2Clme2b3NUNWExM3n/CVE4HM6ue/L+w4fXH4v1PRV5ceGVmRxJGeAXA3e4+OuKQMsrMDgReAt7hlznsiwjz+JOB+oT/FMe4+6eRBFlBzKwLcIG79zKzPQkj/l2BecAf3P37KOPLFDNrRThhXRVYDpxMGMDF4v03s5HA7wkr1uYBpxLm7GPx/pdXXiR8ERHZvHyY0hERkS2ghC8iEhNK+CIiMaGELyISE0r4IiIxoYQvIhITSvgiIjGhhC8iEhP/H+wuBSeSTkTPAAAAAElFTkSuQmCC\n", "text/plain": [ "
" ] }, "metadata": { "needs_background": "light" }, "output_type": "display_data" }, { "data": { "image/png": "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\n", "text/plain": [ "
" ] }, "metadata": { "needs_background": "light" }, "output_type": "display_data" }, { "data": { "image/png": "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\n", "text/plain": [ "
" ] }, "metadata": { "needs_background": "light" }, "output_type": "display_data" }, { "data": { "image/png": "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\n", "text/plain": [ "
" ] }, "metadata": { "needs_background": "light" }, "output_type": "display_data" }, { "data": { "image/png": "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\n", "text/plain": [ "
" ] }, "metadata": { "needs_background": "light" }, "output_type": "display_data" } ], "source": [ "from scipy import special\n", "\n", "p0 = 0.5\n", "p1 = 0.7\n", "k=100\n", "\n", "for i in range(1,k):\n", " cs = numpy.linspace(0,i,i+1)\n", " test = numpy.ones(i+1)\n", " cdf = numpy.ones(i+1)\n", " for l in range(i+1):\n", " test[l] = numpy.log(p0**cs[l]*(1-p0)**(i-cs[l])/(p1**cs[l]*(1-p1)**(i-cs[l])))\n", " helper[l] = scipy.special.binom(i+1,l)*p0**cs[l]*(1-p0)**(i-cs[l])\n", " cdf = numpy.cumsum(helper)\n", " treshhold = test[numpy.argmax(cdf>0.95)]\n", " plt.plot(cs,test,color='blue')\n", " plt.plot(cs,numpy.ones(i+1)*treshhold)\n", " plt.scatter(numpy.array([y[i-1]]),numpy.array([0]),color='r')\n", " plt.show()" ] }, { "cell_type": "markdown", "metadata": {}, "source": [ "# $p$-value\n", "\n", "We shall also calculate the $p$-value, i.e. the probability with which values larger or equal than the actual observation would appear under null hypothesis. It measures how 'extreme' an observation is, given the null hypthesis is true." ] }, { "cell_type": "code", "execution_count": 127, "metadata": {}, "outputs": [ { "name": "stdout", "output_type": "stream", "text": [ "0.5\n" ] }, { "data": { "image/png": "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\n", "text/plain": [ "
" ] }, "metadata": { "needs_background": "light" }, "output_type": "display_data" }, { "name": "stdout", "output_type": "stream", "text": [ "0.25\n" ] }, { "data": { "image/png": "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\n", "text/plain": [ "
" ] }, "metadata": { "needs_background": "light" }, "output_type": "display_data" }, { "name": "stdout", "output_type": "stream", "text": [ "0.125\n" ] }, { "data": { "image/png": "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\n", "text/plain": [ "
" ] }, "metadata": { "needs_background": "light" }, "output_type": "display_data" }, { "name": "stdout", "output_type": "stream", "text": [ "0.0625\n" ] }, { "data": { "image/png": "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\n", "text/plain": [ "
" ] }, "metadata": { "needs_background": "light" }, "output_type": "display_data" }, { "name": "stdout", "output_type": "stream", "text": [ "0.03125\n" ] }, { "data": { "image/png": "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\n", "text/plain": [ "
" ] }, "metadata": { "needs_background": "light" }, "output_type": "display_data" }, { "name": "stdout", "output_type": "stream", "text": [ "0.015625\n" ] }, { "data": { "image/png": "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\n", "text/plain": [ "
" ] }, "metadata": { "needs_background": "light" }, "output_type": "display_data" }, { "name": "stdout", "output_type": "stream", "text": [ "0.0078125\n" ] }, { "data": { "image/png": "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\n", "text/plain": [ "
" ] }, "metadata": { "needs_background": "light" }, "output_type": "display_data" }, { "name": "stdout", "output_type": "stream", "text": [ "0.00390625\n" ] }, { "data": { "image/png": "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\n", "text/plain": [ "
" ] }, "metadata": { "needs_background": "light" }, "output_type": "display_data" }, { "name": "stdout", "output_type": "stream", "text": [ "0.001953125\n" ] }, { "data": { "image/png": "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\n", "text/plain": [ "
" ] }, "metadata": { "needs_background": "light" }, "output_type": "display_data" }, { "name": "stdout", "output_type": "stream", "text": [ "0.0107421875\n" ] }, { "data": { "image/png": "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\n", "text/plain": [ "
" ] }, "metadata": { "needs_background": "light" }, "output_type": "display_data" }, { "name": "stdout", "output_type": "stream", "text": [ "0.005859375\n" ] }, { "data": { "image/png": "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\n", "text/plain": [ "
" ] }, "metadata": { "needs_background": "light" }, "output_type": "display_data" }, { "name": "stdout", "output_type": "stream", "text": [ "0.003173828125\n" ] }, { "data": { "image/png": "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\n", "text/plain": [ "
" ] }, "metadata": { "needs_background": "light" }, "output_type": "display_data" }, { "name": "stdout", "output_type": "stream", "text": [ "0.01123046875\n" ] }, { "data": { "image/png": "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\n", "text/plain": [ "
" ] }, "metadata": { "needs_background": "light" }, "output_type": "display_data" }, { "name": "stdout", "output_type": "stream", "text": [ "0.0064697265625\n" ] }, { "data": { "image/png": "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\n", "text/plain": [ "
" ] }, "metadata": { "needs_background": "light" }, "output_type": "display_data" }, { "name": "stdout", "output_type": "stream", "text": [ "0.003692626953125\n" ] }, { "data": { "image/png": "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\n", "text/plain": [ "
" ] }, "metadata": { "needs_background": "light" }, "output_type": "display_data" }, { "name": "stdout", "output_type": "stream", "text": [ "0.0020904541015625\n" ] }, { "data": { "image/png": "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\n", "text/plain": [ "
" ] }, "metadata": { "needs_background": "light" }, "output_type": "display_data" }, { "name": "stdout", "output_type": "stream", "text": [ "0.0063629150390625\n" ] }, { "data": { "image/png": "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\n", "text/plain": [ "
" ] }, "metadata": { "needs_background": "light" }, "output_type": "display_data" }, { "name": "stdout", "output_type": "stream", "text": [ "0.0037689208984375\n" ] }, { "data": { "image/png": "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\n", "text/plain": [ "
" ] }, "metadata": { "needs_background": "light" }, "output_type": "display_data" }, { "name": "stdout", "output_type": "stream", "text": [ "0.0022125244140625\n" ] }, { "data": { "image/png": "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\n", "text/plain": [ "
" ] }, "metadata": { "needs_background": "light" }, "output_type": "display_data" }, { "name": "stdout", "output_type": "stream", "text": [ "0.0012884140014648438\n" ] }, { "data": { "image/png": "iVBORw0KGgoAAAANSUhEUgAAAYQAAAD8CAYAAAB3u9PLAAAABHNCSVQICAgIfAhkiAAAAAlwSFlzAAALEgAACxIB0t1+/AAAADl0RVh0U29mdHdhcmUAbWF0cGxvdGxpYiB2ZXJzaW9uIDMuMC4zLCBodHRwOi8vbWF0cGxvdGxpYi5vcmcvnQurowAAIABJREFUeJzt3XuclHXd//HXh7NnUdYwDgJKKZWirmipKHgC5KCJsKQCdqBU7kqrn3Yuyzu9D9ldmWla4HFBlGQRQwO0W0VlQeQgISugrJBgKCKEuO7n98f3mttxmGVnd2f2msP7+XjMY2au6zvXfGZ2dj5zfY/m7oiIiLSJOwAREckPSggiIgIoIYiISEQJQUREACUEERGJKCGIiAighCAiIhElBBERAZQQREQk0i7uAJqiS5cu3qtXr7jDEBEpKIsXL37T3csaK1dQCaFXr15UV1fHHYaISEExs1czKacqIxERAZQQREQkooQgIiKAEoKIiESUEEREBFBCEBGRSEYJwcyGmNlqM6sxs+vS7B9oZkvMrM7MRidtH2RmS5Muu8zsgmjfFDNbl7Svf/ZeloiINFWjCcHM2gK3AEOBfsA4M+uXUuw1YCJwX/JGd1/g7v3dvT8wGNgJPJZU5DuJ/e6+tPkvQyQ+NTVQWQlajVYKXSYD0wYANe6+FsDMKoFRwEuJAu6+PtpXv5fjjAYedfedzY5WJM+8/jqceWa4fuMN+MY34o5IpPkyqTLqBmxIul8bbWuqCuD+lG03mNkyM7vZzDo245gisXn3XRgxArZtg8GD4eqr4eGH445KpPkySQiWZluTTo7N7HDgM8DcpM3fBY4GTgIOAa5t4LGTzKzazKq3bNnSlKcVyZkPPoBx4+DFF2HaNKiqgvJy+MIXYPHiuKMTaZ5MEkIt0CPpfndgYxOfZwww093fT2xw900evAf8iVA1tQd3v93dy929vKys0bmZRFrFNdfA7Nnwm9/AsGGw774waxaUlcHw4fDaa3FHKNJ0mSSERUBfM+ttZh0IVT+zmvg840ipLorOGjAzAy4AVjTxmCKx+PWvw+Xqq+HKKz/c3rUrPPII7NwZksI778QXo0hzNJoQ3L0OmEyo7lkFTHf3lWZ2vZmNBDCzk8ysFrgYuM3MViYeb2a9CGcYT6Yc+l4zWw4sB7oAP2/5yxHJraqqkAhGjYL//M8993/qU/Dgg7BqFYwZA3V1rR+jSHOZF1BfufLyctf01xKXJUvg9NOhXz944gnYb7+Gy95xB3zlK/DVr8Ktt4Kla4kTaSVmttjdyxsrV1DrIYjEZcOGUA3UpUs4S9hbMgD48pfhlVfgxhuhb1/41rdaJ06RllBCEGnE9u0hGezYAU8/HdoKMnHDDSEpfOc70Ls3fP7zuY1TpKU0l5HIXtTVwdixsHIlPPAAfPrTmT+2TRuYOhVOPhkuvRSefz53cYpkgxKCSAPc4etfh0cfDe0A557b9GPss08YrNa1axjEtn591sMUyRolBJEG3HxzSAT/7/+FBuLmOuwwmDMHdu+G88+Ht9/OXowi2aSEIJLGzJnw7W/DRRfBL37R8uMdfXQ45po1MHo0vP9+448RaW1KCCIpFi2CSy6BAQPg7rtDW0A2nHkm/OEPMG8eXHGFZkeV/KNeRiJJXn011PV/7GNhKop99snu8SdMCD2PfvYzOOoouG6P1UVE4qOEIBLZti3U8e/aBQsWhLr/XPjpT0NS+O53oU+fMKJZJB8oIYgQ6vQvvhhWr4a5c+GYY3L3XGZw551hArzx46FHD/jsZ3P3fCKZUhuCCKHq5vHH4fbbw9oGudapU2hk7tEDRo4Ezewu+UAJQUrev/4VEsGll8Lll7fe83bpEga7vflmWIJTJG5KCFLy5swJq59NnNj6z92/Pxx7rBKC5AclBCl5lZWhV9GZZ8bz/BUV8MwzoYeTSJyUEKSkbd8eVj67+GJo2zaeGMaODdfTp8fz/CIJSghS0mbNCt1MKyrii6FPnzAITtVGEjclBClplZX50e2zoiIswPPyy/HGIaVNCUFK1tatYczB2LHZm56iucaMCeMTpk2LNw4pbUoIUrJmzgwD0uKsLkro1i0sz3n//ZrjSOKTUUIwsyFmttrMasxsj9lXzGygmS0xszozG52y7wMzWxpdZiVt721mz5nZGjObZmYdWv5yRDJXWRnmEzrhhLgjCSoqYNUqWLEi7kikVDWaEMysLXALMBToB4wzs34pxV4DJgL3pTnEv9y9f3QZmbT9JuBmd+8LvAV8qRnxizTLG2/A/PnhS9gs7miCiy4KPZ3UuCxxyeQMYQBQ4+5r3X03UAmMSi7g7uvdfRlQn8mTmpkBg4EZ0aapwAUZRy3SQjNmQH19flQXJRx2GJx1VkgIqjaSOGSSELoBG5Lu10bbMtXJzKrN7FkzS3zpHwq87e51jR3TzCZFj6/eoglfJEsqK8P6yJ/6VNyRfFRFBaxdC9XVcUcipSiThJDuhLopv196uns58AXgV2Z2ZFOO6e63u3u5u5eXlZU14WlF0tuwAZ56Kr/ODhIuvBDat1e1kcQjk4RQC/RIut8d2JjpE7j7xuh6LfAEcDzwJnCwmSWm327SMUVaIjEiODFCOJ8cfDAMHRq6n9ZnVAErkj2ZJIRFQN+oV1AHoAKY1chjADCzzmbWMbrdBTgVeMndHVgAJHokTQAebmrwIs1RWQnl5aGHUT6qqIDXX4enn447Eik1jSaEqJ5/MjAXWAVMd/eVZna9mY0EMLOTzKwWuBi4zcxWRg8/Bqg2sxcJCeBGd38p2nctcI2Z1RDaFO7M5gsTSaemJtTP52N1UcKIEWHpTlUbSWszL6DuDOXl5V6t1jZpgRtugB/8IKxW1qNH4+XjMnZsWMZz40Zop3UNpYXMbHHUlrtXGqksJaWyEk47Lb+TAYQzmC1bQlIQaS1KCFIyVqwIl3yuLkoYOhQOOEDVRtK6lBCkZEybFiaxGz268bJx69QpdEF96CF47724o5FSoYQgJcE9/NoePDisjlYIKirg7bfhscfijkRKhRKClIQlS0IPo0KoLko4+2w45BBVG0nrUUKQklBZGUYAX3hh3JFkrn37UL318MOwc2fc0UgpUEKQoldfHxLCeeeFX9yFpKICduwI6z6L5JoSghS9Z56B2trCqi5KGDgQunZVtZG0DiUEKXqVlaHXzsiRjZfNN23bhuU158yBbdvijkaKnRKCFLW6OnjgARg+PPTrL0QVFaHr6cOa7UtyTAlBitoTT8DmzYVZXZRwyilwxBGqNpLcU0KQolZZCfvvD8OGxR1J85mFuY0efxzefDPuaKSYKSFI0dq9Gx58EC64IMweWsgqKkL110MPxR2JFDMlBClajz0WRvoWcnVRQv/+8IlPqNpIcksJQYpWZSV07gznnBN3JC1nFhLbE0/Apk1xRyPFSglBitLOnaFXzkUXQYcOcUeTHRUVYU6mBx6IOxIpVkoIUpTmzIF334Vx4+KOJHuOOQaOO07VRpI7SghSlCorw6ymZ5wRdyTZVVEBCxfC+vVxRyLFKKOEYGZDzGy1mdWY2XVp9g80syVmVmdmo5O29zezhWa20syWmdnYpH1TzGydmS2NLv2z85Kk1L3zDjzySBjh27Zt3NFk19joP2j69HjjkOLUaEIws7bALcBQoB8wzsz6pRR7DZgI3JeyfScw3t0/BQwBfmVmByft/467948uS5v5GkQ+YtYs2LWrOHoXperdG04+WdVGkhuZnCEMAGrcfa277wYqgVHJBdx9vbsvA+pTtr/s7mui2xuBzUBZViIXaUBlJfTsGUb4FqOKCnjhBVi9Ou5IpNhkkhC6ARuS7tdG25rEzAYAHYBXkjbfEFUl3WxmHZt6TJFUW7fC3LmhaqVNkbaQXXxx6IY6bVrckUixyeRfxtJs86Y8iZkdDtwNXO7uibOI7wJHAycBhwDXNvDYSWZWbWbVW7ZsacrTSgl66KEworcYq4sSunUL02Lff3/ohiqSLZkkhFqgR9L97sDGTJ/AzA4EHgF+4O7PJra7+yYP3gP+RKia2oO73+7u5e5eXlam2ibZu8pK6NsXjj8+7khyq6IC/v53WL487kikmGSSEBYBfc2st5l1ACqAWZkcPCo/E7jL3R9I2Xd4dG3ABcCKpgQukuof/4AFC8KXpaU7ry0iF10UelCpcVmyqdGE4O51wGRgLrAKmO7uK83sejMbCWBmJ5lZLXAxcJuZrYwePgYYCExM0730XjNbDiwHugA/z+ork5IzY0ZYLrOYq4sSysrg7LNDQlC1kWSLeQF9msrLy726ujruMCRPnXZaGIOwbFnckbSOKVPg8svh2WdDV1SRhpjZYncvb6xckfbDkFLzj3/A00+HHjil4oILoF07mDkz7kikWCghSFGYMydcF+K6yc118MGht9Hs2XFHIsVCCUGKQlUV9OgBxx4bdySta/hwWLkS1q2LOxIpBkoIUvB27QrLSw4fXvy9i1KNGBGudZYg2aCEIAXvySdhx46QEErNUUfBJz+phCDZoYQgBa+qKqyZPGhQ3JHEY/jwsJLa9u1xRyKFTglBCpp7+HV8zjkhKZSiESNg9+5QbSbSEkoIUtBWroRXXy3N6qKEz30u9DhStZG0lBKCFLSqqnA9bFi8ccSpfXsYMiQsClRf33h5kYYoIUhBmz0bTjghzABayoYPh82bYdGiuCORQqaEIAXrzTfD+sKJrpelbOjQsP6Dqo2kJZQQpGA9+mhoVC7l9oOEQw6BU09VQpCWUUKQglVVBV27hiojCYlx6VLYsKHxsiLpKCFIQdq9OyyVOXx48S6V2VSJqrNHHok3Dilc+leSgvTUU2Gqa1UXfejoo6FPH1UbSfMpIUhBqqqCjh3DIjESmIUEOW8e7NwZdzRSiJQQpOC4h4QweDDst1/c0eSXESPCZH/z5sUdiRQiJQQpOC+/DK+8ouqidAYOhP33V7WRNI8SghScxJfd+efHG0c+6tABzjsvvEcFtDqu5ImMEoKZDTGz1WZWY2bXpdk/0MyWmFmdmY1O2TfBzNZElwlJ2080s+XRMX9tVmoz2UtzVVWFhXCOOCLuSPLTiBGwcSO88ELckUihaTQhmFlb4BZgKNAPGGdm/VKKvQZMBO5LeewhwI+Bk4EBwI/NrHO0+1ZgEtA3ugxp9quQkvHWW6GHkaqLGjZ0aGhgVrWRNFUmZwgDgBp3X+vuu4FKYFRyAXdf7+7LgNSptc4DHnf3re7+FvA4MMTMDgcOdPeF7u7AXcAFLX0xUvzmzoUPPlBC2JvDDoOTT1ZCkKbLJCF0A5LHPtZG2zLR0GO7RbcbPaaZTTKzajOr3rJlS4ZPK8Wqqgq6dIEBA+KOJL8NHx4mutu0Ke5IpJBkkhDS1e1n2lzV0GMzPqa73+7u5e5eXlZWluHTSjGqqwvzF51/PrRtG3c0+S0xannOnHjjkMKSSUKoBXok3e8ObMzw+A09tja63ZxjSolauDC0Iai6qHGf+Qz06KFqI2maTBLCIqCvmfU2sw5ABTArw+PPBc41s85RY/K5wFx33wRsN7NTot5F44GHmxG/lJDZs8NiMOeeG3ck+S8xavnxx8NANZFMNJoQ3L0OmEz4cl8FTHf3lWZ2vZmNBDCzk8ysFrgYuM3MVkaP3Qr8jJBUFgHXR9sArgDuAGqAV4BHs/rKpOhUVcEZZ8CBB8YdSWEYMQJ27IAnnog7EikU7TIp5O5zgDkp236UdHsRH60CSi73R+CPabZXA59uSrBSul55BVatgq9+Ne5ICsegQbDvvuHMaog6dUsGNFJZCkJiSme1H2SuU6cw+Z9GLUumlBCkIFRVwTHHwJFHxh1JYRkxAl59FVasiDsSKQRKCJL33nkHnnxSZwfNMWxYuFZvI8mEEoLkvccfh/ffV0Jojo9/HE48UQlBMqOEIHlv9mzo3Bk+97m4IylMI0aEMRxvvhl3JJLvlBAkr33wQWhQHjoU2mXUJ05SDR8eGpU1alkao4QgeW3RItiyRdVFLXH88XD44ao2ksYpIUhemz07zFukfvTN16ZNmP9p7lzYvTvuaCSfKSFIXquqgtNOC20I0nwjRoTeWv/7v3FHIvlMCUHy1muvwbJlqi7KhrPOgo4dVW0ke6eEIHlLo5OzZ7/9YPDgcMalUcvSECUEyVtVVXDUUfDJT8YdSXEYMSLMCbV6ddyRSL5SQpC8tGMHzJ8fzg4s3XJK0mTnnx+uVW0kDVFCkLw0bx68956qi7KpZ0849lglBGmYEoLkpdmzw7oHp58edyTFZcQIeOqpsPKcSColBMk79fUhIZx3HnToEHc0xWX48DD6+y9/iTsSyUdKCJJ3XngBNm1SdVEunHQSlJWp2kjSU0KQvDN7dmhITkzdLNnTtm1oXH70UairizsayTcZJQQzG2Jmq82sxsyuS7O/o5lNi/Y/Z2a9ou2XmNnSpEu9mfWP9j0RHTOx77BsvjApXFVV8NnPQpcucUdSnIYPD20IzzwTdySSbxpNCGbWFrgFGAr0A8aZWb+UYl8C3nL3o4CbgZsA3P1ed+/v7v2By4D17r406XGXJPa7++YsvB4pcBs3wuLFqi7KpXPOgfbtVW0ke8rkDGEAUOPua919N1AJjEopMwqYGt2eAZxltkfv8XHA/S0JVopfYopmJYTcOfBAOOMMJQTZUyYJoRuwIel+bbQtbRl3rwO2AYemlBnLngnhT1F10Q/TJBApQbNnwxFHwKc/HXckxW3ECFi1KoxcFknIJCGk+6JOnQ1lr2XM7GRgp7snL/V9ibt/Bjg9ulyW9snNJplZtZlVb9myJYNwpVDt2hWWy9To5NzTqGVJJ5OEUAv0SLrfHdjYUBkzawccBGxN2l9BytmBu78eXW8H7iNUTe3B3W9393J3Ly8rK8sgXClUc+bAzp0wcmTckRS/I48MZ2HTp8cdieSTTBLCIqCvmfU2sw6EL/dZKWVmAROi26OB+e5hTkUzawNcTGh7INrWzsy6RLfbA8OBFUhJmzIlLAp/1llxR1IaLrss9DRasybuSCRfNJoQojaBycBcYBUw3d1Xmtn1Zpb4LXcncKiZ1QDXAMldUwcCte6+NmlbR2CumS0DlgKvA39o8auRgrV5c+gbf+mloa+85N6ll4bV1O66K+5IJF+YF9Dk6OXl5V5dXR13GJIDv/oVXH01rFwJ/VI7NUvODBkSGpfXrQvJQYqTmS129/LGyukjIHlhypQwrYKSQeuaODGsTPfEE3FHIvlACUFi9+KL4TJhQuNlJbtGjYKDDoKpUxsvK8VPCUFiN3VqGDlbURF3JKVnn31gzBh48EF49924o5G4KSFIrN5/H+65J3Q1PTR1KKO0iokTwwp1M2bEHYnETQlBYvWXv8CWLaouitNnPwt9+6raSJQQJGZTpoT5+YcMiTuS0mUG48eHhuV16+KORuKkhCCx+ec/w1TXl14a2hAkPuPHh8Rw991xRyJxUkKQ2FRWhjYEVRfFr2dPGDQoVBsV0NAkyTIlBInNlCnQvz8cd1zckQiExuW1a+Gpp+KOROKihCCxeOklqK7W2UE++fznYf/91bhcypQQJBZTp0K7dvCFL8QdiSTstx+MHh1mQN25M+5oJA5KCNLq6upC4+WwYXCYVtLOKxMnwvbtMHNm3JFIHJQQpNX99a+waZOqi/LR6adDr16qNipVSgjS6qZMgUMO+XDVLskfbdqELqh//Sts2NB4eSkuSgjSqt5+G/7859B20LFj3NFIOuPHh66n99wTdyTS2pQQpFVNnw7vvafqonx25JGh6khjEkqPEoK0qilTwpoHJ54YdySyNxMmwOrV8NxzcUcirUkJQVrNyy/DwoWhJ4tZ3NHI3lx8cZgaW43LpUUJQVrNXXeFRstLL407EmnMgQeGgWqVlbBrV9zRSGvJKCGY2RAzW21mNWZ2XZr9Hc1sWrT/OTPrFW3vZWb/MrOl0eX3SY850cyWR4/5tZl+Mxaz+vqQEM47Dw4/PO5oJBMTJ4ZOALNmxR2JtJZGE4KZtQVuAYYC/YBxZpa68u2XgLfc/SjgZuCmpH2vuHv/6PK1pO23ApOAvtFFEyAXsQULQjdGNSYXjkGDoHt3VRuVkkzOEAYANe6+1t13A5XAqJQyo4DEx2YGcNbefvGb2eHAge6+0N0duAu4oMnRS8GYOjWs3Tsq9ZMjeattW7jsMpg7NwwklOKXSULoBiQPUamNtqUt4+51wDYgsSBibzN7wcyeNLPTk8rXNnJMKRLbt4c1eysqoFOnuKORppgwAT74AO69N+5IpDVkkhDS/dJP7Z3cUJlNQE93Px64BrjPzA7M8JjhwGaTzKzazKq3bNmSQbiSb2bMCJOlqbqo8Hzyk3DKKRqTUCoySQi1QI+k+92BjQ2VMbN2wEHAVnd/z93/CeDui4FXgE9E5bs3ckyix93u7uXuXl5WVpZBuJJvpkwJa/aeckrckUhzTJgAK1bAkiVxRyK5lklCWAT0NbPeZtYBqABS+x3MAhK//0YD893dzawsapTGzPoQGo/XuvsmYLuZnRK1NYwHHs7C65E8s24d/O1vGntQyMaODdOMqHG5+DWaEKI2gcnAXGAVMN3dV5rZ9WY2Mip2J3ComdUQqoYSXVMHAsvM7EVCY/PX3H1rtO8K4A6ghnDm8GiWXpPkkbvuCongssvijkSaq3Pn0Bngvvtg9+64o5FcMi+gisHy8nKvrq6OOwzJUH09HHUU9OkTZs+UwjVnTpid9qGH4MIL445GmsrMFrt7eWPlNFJZcuapp0KV0cSJcUciLXXuudC1q6qNip0SguTM1KlhjV79oix87dqFKUceeQTU2a94KSFITuzYEaa6HjMmrNUrhW/ChLD86X33xR2J5IoSguTEzJnw7rsae1BMPv3pMG25qo2KlxKC5MSUKdC7N5x2WtyRSDZNmAAvvADLlsUdieSCEoJk3YYNMH9++PJoo09YURk3Dtq311lCsdK/q2Td3XeHaQ7Gj487Esm2Ll1g+PAwt9H778cdjWSbEoJklXuoLho4MFQZSfGZMAHeeCPMgirFRQlBsmrhQlizRmMPitmwYVBWBn/6U9yRSLYpIUjW1NfDd74Dhx4Ko0fHHY3kSvv28OUvh1HLzzwTdzSSTUoIkjV33hm+IP77v+GAA+KORnLpe9+DHj3gq19VW0IxUUKQrNi8Ga69Fs48U43JpWD//eG3vw3TYt98c9zRSLYoIUhWfOtbYSDarbdqmutSMXIkXHAB/OQnsH593NFINighSIvNmwf33APXXQdHHx13NNKafv3rsPby5MlaUa0YKCFIi+zaBVdcEaa5/t734o5GWluPHnD99WHSu4ceijsaaSklBGmRG28M3UxvvRU6dYo7GonDv/0b9O8PX/86vPNO3NFISyghSLOtXg2/+AV84Qtw9tlxRyNxadcObrsNNm2CH/4w7mikJZQQpFnc4corYd994Ze/jDsaiduAAeHz8NvfwuLFcUcjzaWEIM1yzz1hArsbb4SPfSzuaCQf3HADHHZYGJvwwQdxRyPNkVFCMLMhZrbazGrM7Lo0+zua2bRo/3Nm1ivafo6ZLTaz5dH14KTHPBEdc2l0OSxbL0pya+vW0M30lFPgK1+JOxrJFwcdBL/6VThDuOWWuKOR5mg0IZhZW+AWYCjQDxhnZv1Sin0JeMvdjwJuBm6Ktr8JjHD3zwATgLtTHneJu/ePLptb8DqkFV17bUgKt92m6a3lo8aMgfPOgx/8AF5/Pe5opKky+XceANS4+1p33w1UAqNSyowCEjOkzwDOMjNz9xfcfWO0fSXQycw6ZiNwicdTT8Edd8DVV8Oxx8YdjeQbM/jd78J0Ft/4RtzRSFNlkhC6ARuS7tdG29KWcfc6YBtwaEqZi4AX3P29pG1/iqqLfmiWfnyrmU0ys2ozq96i1b1jtXs3fO1r0LNnGJ0qkk6fPqG30YMPhvEJUjgySQjpvqhTxyTutYyZfYpQjfTVpP2XRFVJp0eXy9I9ubvf7u7l7l5eVlaWQbiSK7/8JaxcGXqS7Ldf3NFIPvv2t6FfP7jqKtixI+5oJFOZJIRaoEfS/e7AxobKmFk74CBga3S/OzATGO/uryQe4O6vR9fbgfsIVVOSp9atCyNSL7wQRoyIOxrJdx06wO9/D6++Gj43UhgySQiLgL5m1tvMOgAVwKyUMrMIjcYAo4H57u5mdjDwCPBdd386UdjM2plZl+h2e2A4sKJlL0VyxT380mvbNsxdI5KJ00+HL34xnFkuXx53NJKJRhNC1CYwGZgLrAKmu/tKM7vezEZGxe4EDjWzGuAaINE1dTJwFPDDlO6lHYG5ZrYMWAq8Dvwhmy9MsmfGDHj0UfjZz6B797ijkULyH/8BBx8cxibU18cdjTTGvICmKCwvL/fq6uq4wygp27bBMcdA167w/PNhmgKRppg6NSypetttMGlS3NGUJjNb7O7ljZVTL3LZqx/8AP7xj/DPrGQgzTF+fFg46dpr4Y034o5G9kYJQRpUXR1GnF51FZx0UtzRSKEyC7Ph7tgRRrhL/lJCkLTq6kK9b9eu8POfxx2NFLqjjw4LKN17L/z1r3FHIw1RQpC0brkFliyB//mfMEeNSEt973thIaUrrwwLK0n+UUKQPdTWhraDoUNh9Oi4o5Fi0alTqDpasyasoyH5RwlBPuLvf4dhw8L0xbfcEup/RbLl7LPDgkr//u9w881ahznfKCHI/5k6FU48Max8NXMm9O4dd0RSjH73u/Cj45prYORIePPNuCOSBCUEYfv20DVw4sSw8tXSpWEKY5FcOOgg+POfQ/vUY4+F9Zj/9re4oxJQQih5S5dCeXno/fGTn4QeIN1S57IVyTIz+PrXYeFC2GcfGDQojITXSmvxUkIoUe6hjeCUU+Ddd2HePPjxj8N8RSKt5YQTQm+2igr40Y/g3HNDlaXEQwmhBL31Flx0EUyeDGedFc4Szjwz7qikVB1wQFij+49/DGcMxx0Hf/lL3FGVJiWEErNwYaizraqC//qvcK1lJiRuZnD55WF0/Mc+Fro8X3ttWHlNWo8SQomor4ebbgpTErdtC08/HaYR0JrIkk/69QuTKE6aFGZKHTgQ1q+PO6rSoa+DErAOQ/bfAAAJrElEQVR5c+jmd9118PnPwwsvhN5EIvlon33CZIrTpsFLL8Hxx8NDD8UdVWlQQihy8+aFOtknnwwrWE2bpqkopDCMGRN+vPTtG9q8rrpKU17kmhJCkaqrC9NPnHNOWKDk+efDZHUaeSyFpE8feOqpUL35u9+FXnGrV8cdVfHSDPdF5O23wwCfBQvCCmerV4eGut/8BvbbL+7oRJqnQ4fQAWLQIJgwIVQhDR0KgweHy9FH64dOtighFLAdO8KvpwULYP58WLw4NB536gSnngo//SmMHRt3lCLZcf758OKLYQDlY4992K7QtWtIFoMHh+s+fZQgmiujJTTNbAjwP0Bb4A53vzFlf0fgLuBE4J/AWHdfH+37LvAl4APg6+4+N5NjplPqS2ju2gXPPhu+/BcsgOeeC93y2reHk0/+8BfTySeHpCBSrNxh3boP/xfmzw8r+wH07Pnh/8KgQVoHHDJfQrPRhGBmbYGXgXOAWmARMM7dX0oqcyVwrLt/zcwqgAvdfayZ9QPuBwYAHwf+Cnwiethej5lOqSWE998P/bLnzw+XZ54JSaFNmzDdROJX0amnqkpISsS998L3vw+vvRa++W+4AS65BPcwU28iOSxYAFu3hof07fthchg0CA47LN6XEIdME0ImVUYDgBp3XxsduBIYBSR/eY8CfhLdngH81sws2l7p7u8B68ysJjoeGRyzKNTXwzvvhNHBb78drlMve9teVxeOc9xxcMUV4YN9+unqKSQl6N57wwCFnTvD/VdfDfcBu+QSjjkGjjkmLMBTXw/Ll3+YHO6/P3RlhTAyunPn0Nmic+f0l4b2degQ02tvJZkkhG7AhqT7tcDJDZVx9zoz2wYcGm1/NuWxianTGjtm1nzzm2EgVojvw+0N3U69X18fJt1qziXxhd6Qtm33/BD27h2uDzkkzPVyxhnQpUvL3gORgvf973+YDBJ27gzbL7nkI5vbtAk/oo47Dq6+OvwfLlkSul9v3PjRH141NR/eTj18qrZtm39p0+ajbRtNvf3AA9CrV2ZvVXNlkhDSNc+k1jM1VKah7em6u6atuzKzScAkgJ49ezYc5V4cdNBHTxMzefOT75s1/0PQrt1Hf22k/vLYf381gIlk5LXXmrY9Sbt2YTBmYwMyd+9u+Iz9rbfgX/9q/o/D+voPnyeTH6apP1JbY+LJTBJCLdAj6X53YGMDZWrNrB1wELC1kcc2dkwA3P124HYIbQgZxLuHn/60OY8SkbzSs2eoJkq3PUs6dAg/HkuxnQEyG5i2COhrZr3NrANQAcxKKTMLmBDdHg3M99BaPQuoMLOOZtYb6As8n+ExRUQ+dMMNsO++H922775hu2RFo2cIUZvAZGAuoYvoH919pZldD1S7+yzgTuDuqNF4K+ELnqjcdEJjcR1wlbt/AJDumNl/eSJSNBLtBGl6GUl2ZDQOIV+UWrdTEZFsyLTbqeYyEhERQAlBREQiSggiIgIoIYiISEQJQUREACUEERGJKCGIiAighCAiIpGCGphmZluANJOZZKQL8GYWw8kWxdU0iqtpFFfTFGtcR7h7WWOFCiohtISZVWcyUq+1Ka6mUVxNo7iaptTjUpWRiIgASggiIhIppYRwe9wBNEBxNY3iahrF1TQlHVfJtCGIiMjeldIZgoiI7EXRJQQzG2Jmq82sxsyuS7O/o5lNi/Y/Z2a9WiGmHma2wMxWmdlKM/tGmjJnmtk2M1saXX6U67ii511vZsuj59xjsQkLfh29X8vM7IRWiOmTSe/DUjN7x8y+mVKmVd4vM/ujmW02sxVJ2w4xs8fNbE103bmBx06IyqwxswnpymQ5rv80s79Hf6eZZnZwA4/d6988B3H9xMxeT/pbDWvgsXv9381BXNOSYlpvZksbeGwu36+03w2xfcbcvWguhNXXXgH6AB2AF4F+KWWuBH4f3a4AprVCXIcDJ0S3DwBeThPXmcDsGN6z9UCXvewfBjwKGHAK8FwMf9N/EPpRt/r7BQwETgBWJG37D+C66PZ1wE1pHncIsDa67hzd7pzjuM4F2kW3b0oXVyZ/8xzE9RPg2xn8nff6v5vtuFL2/zfwoxjer7TfDXF9xortDGEAUOPua919N1AJjEopMwqYGt2eAZxlZpbLoNx9k7sviW5vB1YB3XL5nFk0CrjLg2eBg83s8FZ8/rOAV9y9uQMSW8Td/0ZYFjZZ8mdoKnBBmoeeBzzu7lvd/S3gcWBILuNy98fcvS66+yzQPVvP15K4MpTJ/25O4or+/8cA92fr+TK1l++GWD5jxZYQugEbku7XsucX7/+Vif55tgGHtkp0QFRFdTzwXJrdnzWzF83sUTP7VCuF5MBjZrbYzCal2Z/Je5pLFTT8jxrH+wXwMXffBOEfGjgsTZm437cvEs7s0mnsb54Lk6OqrD82UP0R5/t1OvCGu69pYH+rvF8p3w2xfMaKLSGk+6Wf2o0qkzI5YWb7Aw8C33T3d1J2LyFUixwH/Ab4c2vEBJzq7icAQ4GrzGxgyv44368OwEjggTS743q/MhXn+/Z9oA64t4Eijf3Ns+1W4EigP7CJUD2TKrb3CxjH3s8Ocv5+NfLd0ODD0mxr0XtWbAmhFuiRdL87sLGhMmbWDjiI5p3iNomZtSf8we9194dS97v7O+7+bnR7DtDezLrkOi533xhdbwZmEk7dk2XynubKUGCJu7+RuiOu9yvyRqLaLLrenKZMLO9b1LA4HLjEo4rmVBn8zbPK3d9w9w/cvR74QwPPF9f71Q74PDCtoTK5fr8a+G6I5TNWbAlhEdDXzHpHvy4rgFkpZWYBidb40cD8hv5xsiWqo7wTWOXuv2ygTNdEW4aZDSD8bf6Z47j2M7MDErcJjZIrUorNAsZbcAqwLXEq2woa/OUWx/uVJPkzNAF4OE2ZucC5ZtY5qiI5N9qWM2Y2BLgWGOnuOxsok8nfPNtxJbc5XdjA82Xyv5sLZwN/d/fadDtz/X7t5bshns9YLlrO47wQesW8TOix8P1o2/WEfxKAToQqiBrgeaBPK8R0GuFUbhmwNLoMA74GfC0qMxlYSehd8SzwuVaIq0/0fC9Gz514v5LjMuCW6P1cDpS30t9xX8IX/EFJ21r9/SIkpE3A+4RfZF8itDnNA9ZE14dEZcuBO5Ie+8Xoc1YDXN4KcdUQ6pQTn7FEb7qPA3P29jfPcVx3R5+dZYQvusNT44ru7/G/m8u4ou1TEp+ppLKt+X419N0Qy2dMI5VFRAQoviojERFpJiUEEREBlBBERCSihCAiIoASgoiIRJQQREQEUEIQEZGIEoKIiADw/wHqDjvfYdHaBQAAAABJRU5ErkJggg==\n", "text/plain": [ "
" ] }, "metadata": { "needs_background": "light" }, "output_type": "display_data" }, { "name": "stdout", "output_type": "stream", "text": [ "0.0007448196411132812\n" ] }, { "data": { "image/png": "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\n", "text/plain": [ "
" ] }, "metadata": { "needs_background": "light" }, "output_type": "display_data" }, { "name": "stdout", "output_type": "stream", "text": [ "0.002171754837036133\n" ] }, { "data": { "image/png": "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\n", "text/plain": [ "
" ] }, "metadata": { "needs_background": "light" }, "output_type": "display_data" }, { "name": "stdout", "output_type": "stream", "text": [ "0.001299738883972168\n" ] }, { "data": { "image/png": "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\n", "text/plain": [ "
" ] }, "metadata": { "needs_background": "light" }, "output_type": "display_data" }, { "name": "stdout", "output_type": "stream", "text": [ "0.000771939754486084\n" ] }, { "data": { "image/png": "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\n", "text/plain": [ "
" ] }, "metadata": { "needs_background": "light" }, "output_type": "display_data" }, { "name": "stdout", "output_type": "stream", "text": [ "0.0004552602767944336\n" ] }, { "data": { "image/png": "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\n", "text/plain": [ "
" ] }, "metadata": { "needs_background": "light" }, "output_type": "display_data" }, { "name": "stdout", "output_type": "stream", "text": [ "0.00026676058769226074\n" ] }, { "data": { "image/png": "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\n", "text/plain": [ "
" ] }, "metadata": { "needs_background": "light" }, "output_type": "display_data" }, { "name": "stdout", "output_type": "stream", "text": [ "0.0007568597793579102\n" ] }, { "data": { "image/png": "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\n", "text/plain": [ "
" ] }, "metadata": { "needs_background": "light" }, "output_type": "display_data" }, { "name": "stdout", "output_type": "stream", "text": [ "0.00045611709356307983\n" ] }, { "data": { "image/png": "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\n", "text/plain": [ "
" ] }, "metadata": { "needs_background": "light" }, "output_type": "display_data" }, { "name": "stdout", "output_type": "stream", "text": [ "0.0002730563282966614\n" ] }, { "data": { "image/png": "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\n", "text/plain": [ "
" ] }, "metadata": { "needs_background": "light" }, "output_type": "display_data" }, { "name": "stdout", "output_type": "stream", "text": [ "0.00016245711594820023\n" ] }, { "data": { "image/png": "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\n", "text/plain": [ "
" ] }, "metadata": { "needs_background": "light" }, "output_type": "display_data" }, { "name": "stdout", "output_type": "stream", "text": [ "9.609758853912354e-05\n" ] }, { "data": { "image/png": "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\n", "text/plain": [ "
" ] }, "metadata": { "needs_background": "light" }, "output_type": "display_data" }, { "name": "stdout", "output_type": "stream", "text": [ "0.0002675263676792383\n" ] }, { "data": { "image/png": "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\n", "text/plain": [ "
" ] }, "metadata": { "needs_background": "light" }, "output_type": "display_data" }, { "name": "stdout", "output_type": "stream", "text": [ "0.0001620317343622446\n" ] }, { "data": { "image/png": "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\n", "text/plain": [ "
" ] }, "metadata": { "needs_background": "light" }, "output_type": "display_data" }, { "name": "stdout", "output_type": "stream", "text": [ "9.756279177963734e-05\n" ] }, { "data": { "image/png": "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\n", "text/plain": [ "
" ] }, "metadata": { "needs_background": "light" }, "output_type": "display_data" }, { "name": "stdout", "output_type": "stream", "text": [ "5.842093378305435e-05\n" ] }, { "data": { "image/png": "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\n", "text/plain": [ "
" ] }, "metadata": { "needs_background": "light" }, "output_type": "display_data" }, { "name": "stdout", "output_type": "stream", "text": [ "3.480084706097841e-05\n" ] }, { "data": { "image/png": "iVBORw0KGgoAAAANSUhEUgAAAX0AAAD8CAYAAACb4nSYAAAABHNCSVQICAgIfAhkiAAAAAlwSFlzAAALEgAACxIB0t1+/AAAADl0RVh0U29mdHdhcmUAbWF0cGxvdGxpYiB2ZXJzaW9uIDMuMC4zLCBodHRwOi8vbWF0cGxvdGxpYi5vcmcvnQurowAAIABJREFUeJzt3X+YlHW9//Hnm11YFBVR1lR+CBpo4PEHrgjrb90M7WpJv1BoxWol+S3PVVlXBztmZsfT0WPp1cm+ZWmRlWhmuf5Ia1VMXUAWRHFFlBAF4RDKDxPlx8L7+8dnJtZhlp1lZ+dzz8zrcV1z7cw998y894Z93fd87vvz+Zi7IyIi5aFX7AJERKRwFPoiImVEoS8iUkYU+iIiZUShLyJSRhT6IiJlRKEvIlJGFPoiImVEoS8iUkYqYxeQaeDAgT5s2LDYZYiIFJX58+e/6e7Vna2XuNAfNmwYLS0tscsQESkqZvZaLuupeUdEpIwo9EVEyohCX0SkjCj0RUTKiEJfRKSMKPRFRMqIQl9EpIwo9EVy9Mc/Qmtr7CpEukehL5KDmTPh/PPhzDPhtZy6wIgkk0JfpBMtLXDJJXDiibB1K9TXwzvvxK5KZM8o9EV2Y/Vq+PjH4aCD4IEH4O674YUXYOpU2LEjdnUiXafQF+nA5s0h8DdsgMbGEPznnAPf/z784Q9wzTWxKxTpusQNuCaSBO5w6aXwzDNw771w7LE7n/vyl2HRIvjud2H0aPjkJ+PVKdJVOtIXyeKGG+DXv4Zrrw0ncNszgx//GE4+GS6+GObPj1KiyB5R6ItkuP9+uPJK+MQn4Kqrsq9TVRW+ARx0EEycGNr+RYqBQl+kndZWuOgiOP54+MUvwlF9Rw46KLT1r18fvg1s3ly4OkX2lEJfJOWtt8LlmPvsA/fdB3vv3flrjj02NAPNnQvTpoVzASJJptAXAbZtg0mT4I03wpU5gwfn/trzzw9t/3fcATfe2HM1iuSDQl+EENizZsFPfwrjxnX99VddBZMnw/Tpat+XZFPoixCO7ocNC52u9oQZXH116LB1//15LU0krxT6UvY2bYKmptCev7sTt50ZPRqGDw8nd0WSSqEvZa+pKVx5U1/fvfcxC+/R1BR2JCJJlFPom9kEM1tiZkvNbHqW508zswVm1mZmk9otP87MZptZq5k9b2bquyiJ09gI/fvDaad1/73q62HLFvjLX7r/XiI9odPQN7MK4BbgXGAUcKGZjcpY7XXgYuC3GcvfBaa6+2hgAnCzme3f3aJF8mX79tAGf9550Lt399/v1FNh//3VxCPJlcvYO2OBpe6+DMDMZgITgRfTK7j78tRz7xt30N1fbnd/lZn9HagGNnS7cpE8mDsX1q7tftNOWu/eYQfywANhh1JRkZ/3FcmXXJp3BgEr2j1emVrWJWY2FugD/K2rrxXpKY2NUFkJEybk7z3r68OOZM6c/L2nSL7kEvrZrmfoUr9DMzsEuAO4xN13GYXczKaZWYuZtaxdu7Yrby3SLY2NcPrpoUkmXyZMCEf8auKRJMol9FcCQ9o9HgysyvUDzGw/4EHgKnfPeuzj7re6e42711RXV+f61iLd8sorsHhxGDAtn/r3hzPOUOhLMuUS+vOAEWY23Mz6AFOAnP47p9b/A/Ard//dnpcpkn/pUP7Yx/L/3vX18NJL8PLLna8rUkidhr67twGXA48Ai4G73b3VzK41s3oAMzvRzFYCk4Gfmllr6uWfAE4DLjazhanbcT3ym4h0UWMjHHNM6Imbb+kdiY72JWnMEzYsYE1Njbe0tMQuQ0rcW2+FoZG/+c0wA1ZPOO442G8/+Otfe+b9Rdozs/nuXtPZeuqRK2XpwQfDODn5ulQzm/p6ePppePPNnvsMka5S6EtZamyEQw+FE07ouc+YODHsWB56qOc+Q6SrFPpSdjZvhocfDu3uvXrwL2DMmLBjUbu+JIlCX8rOrFlhQLSebNqBnQOwPfywplKU5FDoS9m57z7o1w/OOqvnP6u+PuxgHn+85z9LJBcKfSkr7qG55SMfgb59e/7zzjorzLmrJh5JCoW+lJUFC2DVqp5v2kmrqgo7mMZGTZouyaDQl7LS2BhO3p53XuE+s74+7GgWLCjcZ4p0RKEvZaWxEWproZBDPJ13XtjR3Hdf4T5TpCMKfSkbr70GCxcWrmknbeBAOPlktetLMij0pWzcf3/4me9RNXMxcSI891zY8YjEpNCXstHYCEceCSNHFv6z098u0jsekVgU+lIWNm4MnbIK3bSTNmIEHHWU2vUlPoW+lIWHH4Zt2+KFPoTPnjUr7IBEYlHoS1lobAwnVMePj1dDfT20tcGf/hSvBhGFvpQ8d3j00dBJqqIiXh3jxsGAAfDYY/FqEFHoS8l79VVYswZOOSVuHRUV4ZtGc3PcOqS8KfSl5KVDtrY2bh3pGlpbYcOG2JVIuVLoS8lrboZ994XRo2NXsnPHM2dO3DqkfCn0peQ1N4f29Jjt+WknnhjqUBOPxKLQl5L29tuwaFEymnYgDLN87LEKfYknp9A3swlmtsTMlprZ9CzPn2ZmC8yszcwmZTzXYGavpG4N+SpcJBfPPBPmqU1K6EOoZe7ccPmmSKF1GvpmVgHcApwLjAIuNLNRGau9DlwM/DbjtQcA3wZOAsYC3zazAd0vWyQ3zc1h2sKTTopdyU61tfDOO/DCC7ErkXKUy5H+WGCpuy9z963ATOB9Q1a5+3J3fx7YkfHajwB/cfd17r4e+AswIQ91i+SkuRmOPhr6949dyU7pDmJq4pEYcgn9QcCKdo9XppblojuvFemWHTtg9uxkNe0AHHYYHHKIQl/iyCX0LcuyXCd+y+m1ZjbNzFrMrGXt2rU5vrXI7r34YjiRm7TQNws1KfQlhlxCfyUwpN3jwcCqHN8/p9e6+63uXuPuNdWFnNJISlqSOmVlqq0NPYVXr45diZSbXEJ/HjDCzIabWR9gCpDrHECPAOeY2YDUCdxzUstEelxzc5gW8YgjYleyq/SOaPbsuHVI+ek09N29DbicENaLgbvdvdXMrjWzegAzO9HMVgKTgZ+aWWvqteuA7xJ2HPOAa1PLRHpcc3MIV8vWyBjZ8cdDVZWaeKTwKnNZyd0fAh7KWHZ1u/vzCE032V57O3B7N2oU6bK1a+GVV+Dzn49dSXZVVVBTo9CXwlOPXClJ6WaTJLbnp9XWwvz5sHlz7EqknCj0pSQ1N0Pv3nDCCbEr6VhtLWzdCgsWxK5EyolCX0pSczOMGQN77RW7ko6pk5bEoNCXkrN1K8ybl+ymHYAPfCBcWaTQl0JS6EvJWbgwtJMnPfRhZyctz7W7o0g3KfSl5CS5U1am2towleOrr8auRMqFQl9KTnMzDB0Khx4au5LOpXdMauKRQlHoS0lxh6efLo6jfAhTOO67r0JfCkehLyVlxQpYtap4Qr+iIkzlqNCXQlHoS0kppvb8tNraMKXj22/HrkTKgUJfSkpzM+y9NxxzTOxKcldbG8b+f+aZ2JVIOVDoS0lpboaxY0Nv3GJx0klhUDg18UghKPSlZGzaFK7RL6amHQhTOR59tEJfCkOhLyVj3jzYvr34Qh9CzXPmhGYekZ6k0JeSkR5Zc9y4uHXsidpa2LgRFi+OXYmUOoW+lIzmZjjqKDjwwNiVdJ06aUmhKPSlJLjvnCmrGB1xBAwcqNCXnqfQl5Lw8suwbl3xhr7ZzsHXRHqSQl9KQjF2yspUWxt2Xm++GbsSKWUKfSkJzc0wYAAceWTsSvZceoeVPiEt0hNyCn0zm2BmS8xsqZlNz/J8lZndlXp+rpkNSy3vbWYzzGyRmS02syvzW75I0NwcZqLqVcSHMTU1UFmpJh7pWZ3+iZhZBXALcC4wCrjQzEZlrPY5YL27fxC4Cbg+tXwyUOXu/wKcAHwhvUMQyZf16+HFF4u7aQfC1I5jxij0pWflclw0Fljq7svcfSswE5iYsc5EYEbq/j3A2WZmgAP9zKwS2AvYCmhYKcmrOXPCz/Scs8WstjaMwbN1a+xKpFTlEvqDgBXtHq9MLcu6jru3ARuBAwk7gE3AauB14EZ3X9fNmkXeZ/bs0KwzdmzsSrpv/Pgw1ePzz8euREpVLqFvWZZlzujZ0Tpjge3AocBw4GtmdvguH2A2zcxazKxl7dq1OZQkstPs2WFUzX32iV1J96W/rehkrvSUXEJ/JTCk3ePBwKqO1kk15fQH1gEXAQ+7+zZ3/zvwNFCT+QHufqu717h7TXV1ddd/Cylb27fD3Lml0bQDMGQIDBqk0Jeek0vozwNGmNlwM+sDTAEaM9ZpBBpS9ycBj7m7E5p0zrKgHzAOeCk/pYuEE7j/+EfphD6E30WhLz2l09BPtdFfDjwCLAbudvdWM7vWzOpTq90GHGhmS4ErgPRlnbcA+wAvEHYev3B3tVZK3qTDsdRCf/ly+N//jV2JlKLKXFZy94eAhzKWXd3u/mbC5ZmZr3sn23KRfJk9O4xZc8QRsSvJn/bt+uefH7cWKT1F3JVFJATjuHFh7JpScfzxYeYvNfFIT1DoS9Fatw6WLCmtph2Avn1DJy2FvvQEhb4UrVLqlJVp/HhoaVEnLck/hb4UrXSnrBNPjF1J/qU7aT33XOxKpNQo9KVolVKnrEzqpCU9RaEvRanUOmVlUict6SkKfSlKra3wzjulG/qgTlrSMxT6UpRKsVNWpvHj4bXXYPXq2JVIKVHoS1GaM6f0OmVlUru+9ASFvhSl2bNDKJZSp6xMY8ZAnz4Kfckvhb4UnVLtlJWpqkqdtCT/FPpSdEq5U1YmddKSfFPoS9GZPRsqKkqzU1am8eNhyxZYuDB2JVIqFPpSdNKdsvr1i11Jz9PJXMk3hb4UlVLvlJVp8OBwU+hLvij0paiUQ6esTOqkJfmk0Jeikg6/cePi1lFI48bB66/DqsyZqUX2gEJfikopzpTVmfS3mvRVSyLdodCXolIOnbIyqZOW5JNCX4rGW2/Byy+XV3s+qJOW5JdCX4pGOXXKyqROWpIvOYW+mU0wsyVmttTMpmd5vsrM7ko9P9fMhrV77hgzm21mrWa2yMz65q98KSdz5pRPp6xM6qQl+dJp6JtZBXALcC4wCrjQzEZlrPY5YL27fxC4Cbg+9dpK4NfAZe4+GjgD2Ja36qWslFOnrEzqpCX5ksuR/lhgqbsvc/etwExgYsY6E4EZqfv3AGebmQHnAM+7+3MA7v6Wu2/PT+lSTsqtU1YmddKSfMkl9AcBK9o9XplalnUdd28DNgIHAiMBN7NHzGyBmX0j2weY2TQzazGzlrVr13b1d5AyUI6dsjKpk5bkQy6hn+3iOM9xnUrgFOBTqZ/nm9nZu6zofqu717h7TXV1dQ4lSbkph5myOjN+vDppSfflEvorgSHtHg8GMv/b/XOdVDt+f2BdavkT7v6mu78LPASM6W7RUn5mz4bqajj88NiVxKN2fcmHXEJ/HjDCzIabWR9gCtCYsU4j0JC6Pwl4zN0deAQ4xsz2Tu0MTgdezE/pUk7KsVNWpuOPVyct6b5OQz/VRn85IcAXA3e7e6uZXWtm9anVbgMONLOlwBXA9NRr1wM/IOw4FgIL3P3B/P8aUsrKtVNWpqoqOOEEhb50T2UuK7n7Q4SmmfbLrm53fzMwuYPX/ppw2abIHinnTlmZxo+HW24JnbT69IldjRQj9ciVxEvPlFVTE7uS+NKdtJ59NnYlUqwU+pJ4Tz8Nxx5bnp2yMqW/7Tz9dNw6pHgp9CXR3n0XmpvhrLNiV5IMgwbByJHw6KOxK5FipdCXRHvqqdB+XVcXu5LkqKuDJ57Q4GuyZxT6kmhNTeGE5SmnxK4kOerqYNOmMCyFSFcp9CXRmpqgtlbt+e2dcQb06hW2jUhXKfQlsd58M1yloqad9xswIFzJpNCXPaHQl8R67LHwU6G/q7q60Lzz9tuxK5Fio9CXxGpqgv32C71Q5f3OPjsMN/3EE7ErkWKj0JfEevRROPNMqMyp33h5qa2Fvn116aZ0nUJfEmnZsnBT0052ffvCqaeqXV+6TqEviZQ+glXod6yuLkwus3p17EqkmCj0JZGamkLv0yOPjF1JcqV3iGrika5Q6Evi7NgRgqyurrzHz+/MccfBAQeoiUe6RqEvifPcc2EMfTXt7F6vXuEqnqYm8MwJTEU6oNCXxEkfuZ69y2zKkqmuDt54A5YsiV2JFAuFviROUxOMHg2HHBK7kuRLfxtSE4/kSqEvibJ5Mzz5pJp2cnX44TB8uEJfcqfQl0SZPRvee0+h3xV1dfD449DWFrsSKQYKfUmURx8NUyOefnrsSopHXV0Yg2f+/NiVSDHIKfTNbIKZLTGzpWY2PcvzVWZ2V+r5uWY2LOP5oWb2jpl9PT9lS6lqaoKTToJ9941dSfE488zwU008kotOQ9/MKoBbgHOBUcCFZjYqY7XPAevd/YPATcD1Gc/fBPyp++VKKduwAebNU9NOV1VXh2v2FfqSi1yO9McCS919mbtvBWYCEzPWmQjMSN2/BzjbLHSrMbOPA8uA1vyULKVq1qzQMUuh33V1dWEu4U2bYlciSZdL6A8CVrR7vDK1LOs67t4GbAQONLN+wL8B3+l+qVLqmprCDFknnRS7kuJTVxfmzH3qqdiVSNLlEvrZOsJn9v/raJ3vADe5+zu7/QCzaWbWYmYta9euzaEkKUVNTeEEbp8+sSspPqecErabmnikM7mE/kpgSLvHg4FVHa1jZpVAf2AdcBJwg5ktB74CfNPMLs/8AHe/1d1r3L2murq6y7+EFL8VK0KvUjXt7Jl+/cIY+wp96UwuoT8PGGFmw82sDzAFaMxYpxFoSN2fBDzmwanuPszdhwE3A//p7j/KU+1SQjSUcvfV1cHChaAvy7I7nYZ+qo3+cuARYDFwt7u3mtm1ZlafWu02Qhv+UuAKYJfLOkV2p6kJDjoIjj46diXFK73DTM8tLJKNecKG56upqfGWlpbYZUgBuYdxds4+G37zm9jVFK+2Nhg4ECZPhp/9LHY1UmhmNt/dazpbTz1yJbrWVlizRk073VVZGTpqqV1fdkehL9Gl2/M1lHL31dXB8uVhfmGRbBT6El1TE4wcCUOHxq6k+GmoZemMQl+i2rYt9MTVUX5+jBwZ5hZW6EtHFPoS1axZ8M47cM45sSspDWZhWz7ySBiiWiSTQl+imjED9t8fJkyIXUnpuOiiMNRyY2ZvGhEU+hLR22/DvffClCnQt2/sakrHmWfC4MFhhyqSSaEv0fzud6EJ4uKLY1dSWioqYOrU0MSzKnPAFCl7Cn2JZsYMOPJIGDs2diWlZ+rUMEy1OrtJJoW+RLFsWZgAvaEhnHyU/DrySBg3LuxYE9bpXiJT6EsUv/pVCPvPfCZ2JaWroSH0dl6wIHYlkiQKfSm4HTvCEWhdXTjhKD3jk5+Eqiqd0JX3U+hLwT35ZBgqoKGh01WlGwYMgIkT4be/DbNqiYBCXyKYMQP23RfOPz92JaWvoQHeegsefDB2JZIUCn0pqE2bwqWakyfD3nvHrqb0nXMOHHywmnhkJ4W+FNS994ZhF3RtfmFUVsKnPx2O9DWjloBCXwpsxgw4/PAwkbcURkNDmGDlzjtjVyJJoNCXglmxIkzlN3Wqrs0vpKOPhjFj4Je/jF2JJIFCXwrmjjtCR6GpU2NXUn4aGuDZZ2HRotiVSGwKfSkI93CkefrpMHx47GrKz0UXQe/eOqErCn0pkDlz4JVXdG1+LAMHwkc/Cr/+dWjfl/KVU+ib2QQzW2JmS81sepbnq8zsrtTzc81sWGr5h81svpktSv08K7/lS7GYMSNcojlpUuxKyldDQ5iA/s9/jl2JxNRp6JtZBXALcC4wCrjQzEZlrPY5YL27fxC4Cbg+tfxN4GPu/i9AA3BHvgqX4rF5M8ycCRdcEDplSRznnQcHHqgTuuUulyP9scBSd1/m7luBmcDEjHUmAunWwnuAs83M3P1Zd0+P6N0K9DWzqnwULsXjvvtg40Zdmx9bnz7wqU+Ff4/162NXI7HkEvqDgBXtHq9MLcu6jru3ARuBAzPW+T/As+6+JfMDzGyambWYWcta9SApOTNmwJAhYUYniauhIYzDc9ddsSuRWHIJ/WxXVGeO0L3bdcxsNKHJ5wvZPsDdb3X3Gnevqa6uzqEkKRarV4cZnD7zGeilywaiO/74cN2+ruIpX7n8Ga4EhrR7PBjInITtn+uYWSXQH1iXejwY+AMw1d3/1t2CpbjccUcYSllX7SSDWWhmmzMHFi+OXY3EkEvozwNGmNlwM+sDTAEaM9ZpJJyoBZgEPObubmb7Aw8CV7r70/kqWorDW2/BDTfAWWfByJGxq5G0z3wG+vWD6btchyfloNPQT7XRXw48AiwG7nb3VjO71szqU6vdBhxoZkuBK4D0f6fLgQ8C3zKzhanbQXn/LSSRrrwSNmyAm2+OXYm0d9BB8O1vQ2MjPPBA7Gqk0MwTNoFmTU2Nt7S0xC5DumnuXBg/Hr76Vfj+92NXI5m2bYPjjoP33gtTKu61V+yKpLvMbL6713S2nk6tSd5t3w5f/CIccghcc03saiSb3r3hllvg1Vfhe9+LXY0UkkJf8u4nPwmTcf/gB+qMlWRnnBGu27/++jBEhpQHhb7k1Zo18O//HiY9/8QnYlcjnbnxRujbF/71X8OgeFL6FPqSV9/4Brz7LvzoRxozvxgcfDB897uhL8W998auRgpBoS958+ST8Ktfwde/DkceGbsaydUXvxhO6n7lK2EqSyltCn3Ji23bQngMHRqad6R4VFbCj38MK1eGo34pbQp9yYv/+R944QX44Q9Dxx8pLuPHw2c/G06+v/hi7GqkJyn0pdveeCN09vnoR6G+vvP1JZn+67/C1VZf+pJO6pYyhb5029e+FmZj+uEPdfK2mFVXh2v2Z82CO++MXY30FIW+dMujj4Zheq+8Eg4/PHY10l2f/zyceGLYkW/cGLsa6QkKfdlj69aFk7dHHBEu1ZTiV1ERTuquWROCX808pUehL3tk+XI4+eTw89ZbQwcfKQ01NWEnftttMHVqmHRFSkdl7AKk+CxYEE7abt4cJtk+/fTYFUm+fe97sM8+8K1vhYlwfv976N8/dlWSDzrSly7505/gtNPCfKtPP63AL1VmcNVVYYatJ56AU08N1/FL8VPoS85+/nP42MdgxAiYPRtGjYpdkfS0qVPDjn75chg3Dp5/PnZF0l0KfemUO1x9NVx6aRhI7a9/hUMPjV2VFEpdHTz1VLh/6qnhii0pXgp92a2tW+GSS0L3/M9+Fu6/X8Mll6Njjgnz6g4dChMmhLmPpTgp9KVD69aFE7YzZoTJUH7+8zD5hpSnwYPDEf9pp4Vmn//4jzBhjhQXhb7soqUldNIZMiT0zrz99jDMgnrbSv/+oY3/058OV/YMHx7Cf/Xq2JVJrhT6AsCmTeFIvqYm9Mi880646KKwA7jkktjVSZL06ROG0P797+Goo0L4DxkCkyZBUxPs2BG7QtmdnELfzCaY2RIzW2pm07M8X2Vmd6Wen2tmw9o9d2Vq+RIz+0j+Spd8WLQILr88nJi99FLYsiVMgLJqFfzsZ3DssbErlCQygwsuCP00XnkFrrgifCv88IfDXAo33ghvvhm7SsnGvJN+1mZWAbwMfBhYCcwDLnT3F9ut80XgGHe/zMymAOe7+yfNbBRwJzAWOBRoAka6e4ctgTU1Nd7S0tLNX0sy7dgBr70Whs1dvDj8XLgQnn0Wqqpg8mS47DKorVUzjuyZzZvD7Fs/+UmYUKdPn3C1z+jR8KEPhUt8P/ShMLBbTn7zmzA5w+uvhzPI110XJvWVrMxsvrvXdLZeLj1yxwJL3X1Z6o1nAhOB9qNuTwSuSd2/B/iRmVlq+Ux33wK8amZLU+83O9dfRDq2fTv84x9hYKy33971tmZNCPjFi+Gll+C993a+9uCDwx/gf/83XHwxDBwY7deQEtG3b2gSvOgiaG0N3xSbm8M5ofYzcg0cuHMnMHIkDBgA++2389a/P+zXdC/7feOr9HtvLQbhiGXatPAGCv5uySX0BwEr2j1eCZzU0Tru3mZmG4EDU8vnZLx20B5Xuxvr18NHEtJ4lPnlKdvjXG5tbTtv27bt+njLls5rOeyw8Ad25pnvP9oaMCB/v69IptGj4eabw3330Ju3/bfMxYvhd78LV4hldwFwAb3YThVbqKSNynfbqJy6g97fCLN9pW8VFeHbaftbr167Lmsv27fZJHzDPfrosJPsSbmEfrZNkdkm1NE6ubwWM5sGTAMYOnRoDiXtqlevZB2tdvafLPM/ZLZb797v/89dWblzWUVFmKHqn0dG++16O+AA2Hvvwv3OItmYhRO9Q4a8/8DMHTZsyP5NdeOFl/E2+7KR/mylD21Uso3etO3oTdt5l73vAKitbdcDph073v+4vWwt2kkZTbQQ4xvlEvorgSHtHg8GVnWwzkozqwT6A+tyfC3ufitwK4Q2/VyLb69/f3jooT15pYjEYBa+cWb91jn94dCkk+mww+Bnl/V4baUsl6t35gEjzGy4mfUBpgCNGes0Ag2p+5OAxzycIW4EpqSu7hkOjACeyU/pIlKyrrtu16+pe+8dlku3dHqkn2qjvxx4BKgAbnf3VjO7Fmhx90bgNuCO1InadYQdA6n17iac9G0DvrS7K3dERICdJ2t19U7edXrJZqHpkk0Rka7L9ZJN9cgVESkjCn0RkTKi0BcRKSMKfRGRMqLQFxEpIwp9EZEyotAXESkjCn0RkTKSuM5ZZrYWyDLoRs4GAsUwfYPqzK9iqROKp1bVmX89Weth7t7pbAWJC/3uMrOWXHqlxaY686tY6oTiqVV15l8SalXzjohIGVHoi4iUkVIM/VtjF5Aj1ZlfxVInFE+tqjP/otdacm36IiLSsVI80hcRkQ6UTOib2QQzW2JmS81seux6dsfMlpvZIjNbaGaJmTzAzG43s7+b2Qvtlh1gZn8xs1dSP6NPqd5BndeY2RupbbrQzM6LWWOqpiFm9riZLTazVjP7cmp5orbpbupM4jbta2bPmNlzqVq/k1o+3MzmprbpXalZ/pJY5y/N7NV22/S4ghfn7kV/I8zo9TfgcKAP8BwwKnZdu6l3OTAwdh0D0nSxAAAC+0lEQVRZ6joNGAO80G7ZDcD01P3pwPUJrfMa4Ouxa8uo8xBgTOr+vsDLwKikbdPd1JnEbWrAPqn7vYG5wDjgbmBKavlPgP+b0Dp/CUyKWVupHOmPBZa6+zJ33wrMBCZGrqnouPtfCdNdtjcRmJG6PwP4eEGLyqKDOhPH3Ve7+4LU/X8Ai4FBJGyb7qbOxPHgndTD3qmbA2cB96SWJ2GbdlRndKUS+oOAFe0erySh/2lTHPizmc03s2mxi+nEB9x9NYRwAA6KXM/uXG5mz6eaf6I3Q7VnZsOA4wlHfIndphl1QgK3qZlVmNlC4O/AXwjf8je4e1tqlUT8/WfW6e7pbXpdapveZGZVha6rVELfsixLxF61Aye7+xjgXOBLZnZa7IJKwP8DjgCOA1YD349bzk5mtg/we+Ar7v527Ho6kqXORG5Td9/u7scBgwnf8j+UbbXCVpWlgIw6zexo4ErgKOBE4ADg3wpdV6mE/kpgSLvHg4FVkWrplLuvSv38O/AHwn/cpFpjZocApH7+PXI9Wbn7mtQf2Q7gZyRkm5pZb0KQ/sbd700tTtw2zVZnUrdpmrtvAGYR2sr3N7PK1FOJ+vtvV+eEVFOau/sW4BdE2KalEvrzgBGpM/h9gClAY+SasjKzfma2b/o+cA7wwu5fFVUj0JC63wDcF7GWDqVDNOV8ErBNzcyA24DF7v6Ddk8lapt2VGdCt2m1me2fur8XUEc4B/E4MCm1WhK2abY6X2q3szfCeYeCb9OS6ZyVupzsZsKVPLe7+3WRS8rKzA4nHN0DVAK/TUqtZnYncAZhJMA1wLeBPxKujBgKvA5MdveoJ1E7qPMMQjOEE66O+kK63TwWMzsFeBJYBOxILf4mob08Mdt0N3VeSPK26TGEE7UVhIPWu9392tTf1UxCk8mzwKdTR9NJq/MxoJrQJL0QuKzdCd/C1FYqoS8iIp0rleYdERHJgUJfRKSMKPRFRMqIQl9EpIwo9EVEyohCX0SkjCj0RUTKiEJfRKSM/H+jYbQFiWtwXAAAAABJRU5ErkJggg==\n", "text/plain": [ "
" ] }, "metadata": { "needs_background": "light" }, "output_type": "display_data" }, { "name": "stdout", "output_type": "stream", "text": [ "2.062879502773285e-05\n" ] }, { "data": { "image/png": "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\n", "text/plain": [ "
" ] }, "metadata": { "needs_background": "light" }, "output_type": "display_data" }, { "name": "stdout", "output_type": "stream", "text": [ "5.808350397273898e-05\n" ] }, { "data": { "image/png": "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\n", "text/plain": [ "
" ] }, "metadata": { "needs_background": "light" }, "output_type": "display_data" }, { "name": "stdout", "output_type": "stream", "text": [ "3.5127392038702965e-05\n" ] }, { "data": { "image/png": "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\n", "text/plain": [ "
" ] }, "metadata": { "needs_background": "light" }, "output_type": "display_data" }, { "name": "stdout", "output_type": "stream", "text": [ "2.1138511328899767e-05\n" ] }, { "data": { "image/png": "iVBORw0KGgoAAAANSUhEUgAAAX0AAAD8CAYAAACb4nSYAAAABHNCSVQICAgIfAhkiAAAAAlwSFlzAAALEgAACxIB0t1+/AAAADl0RVh0U29mdHdhcmUAbWF0cGxvdGxpYiB2ZXJzaW9uIDMuMC4zLCBodHRwOi8vbWF0cGxvdGxpYi5vcmcvnQurowAAIABJREFUeJzt3XuYVNWZ7/HvS3NRRFGxNQbQxohGMI5id4MTUYOR4AUBjyZyMolmNJrMkCd5TCbBmGhixiTmZnwmnkRONDHmokaDgvfMMdH20tCNIkgQbPHWEhUFb5Fb0+/5Y+0ayrKbru6uqlVV+/d5nnq6ateu2m9v6F/tWnvttczdERGRdBgQuwARESkdhb6ISIoo9EVEUkShLyKSIgp9EZEUUeiLiKSIQl9EJEUU+iIiKaLQFxFJkYGxC8i11157eV1dXewyREQqypIlS15199qe1iu70K+rq6O1tTV2GSIiFcXMnstnPTXviIikiEJfRCRFFPoiIimi0BcRSRGFvohIiij0RURSRKEvIpIiCn2RPN12G6xYEbsKkf5R6Ivk4cYbYeZM+MhH4PnnY1cj0ncKfZEePPoofOYzUF8PW7bAjBnwj3/ErkqkbxT6Ijvw8ssh5PfaC26/Hf7wB1i2DM4+G9xjVyfSewp9kW5s3gyzZsH69aE9f5994MQT4Qc/gJtvhu98J3aFIr1XdgOuiZQDd/jc5+CRR+Cmm+CII7Y/d8EF4Wj/kkvg0EPhtNPi1SnSW3kd6ZvZNDNbZWZtZja3i+ePMbNHzazDzE7PWn64mT1iZivMbJmZfaKQxYsUy5VXwq9/DRdfDGec8e7nzODqq2HiRPjUp+Dxx6OUKNInPYa+mdUAVwEnAuOA2WY2Lme154Gzgd/nLH8H+LS7jwemAT81s937W7RIMd17L3z5y6Fp55JLul5np51g/nzYY4/Q5r9uXWlrFOmrfI70G4E2d1/j7luAG4AZ2Su4+7PuvgzozFm+2t2fSu6vBV4BehzkXySW1avhE5+A8ePhN7+BATv4C9l3X7j11nCy9/TTQ88ekXKXT+iPBF7IetyeLOsVM2sEBgNP9/a1IqXwxhtw6qkwcCAsWADDhvX8mvp6uPZaeOAB+MIX1KNHyl8+oW9dLOvVf20z2xe4HviMu3d28fx5ZtZqZq3r9D1ZIvn+96GtDW65BXozY+fs2TB3LsybF078ipSzfEK/HRid9XgUsDbfDZjZbsAdwDfcvbmrddx9nrvXu3t9ba1afySOW2+FKVPgmGN6/9oLL4RBg0LXTpFylk/otwBjzWyMmQ0GzgQW5PPmyfrzgd+4+x/7XqZIcbW1wZNPwvTpfXv9brvBscfCwoWFrUuk0HoMfXfvAOYA9wArgZvcfYWZXWpmpwKYWYOZtQNnAFebWWZYqo8DxwBnm9nS5HZ4UX4TkX7IhHVfQz/z2pUr4WmdtZIyZl5mZ57q6+u9tbU1dhmSMlOmhG6Xy5f3/T2eeQYOOACuuAK+9KXC1SaSDzNb4u71Pa2nYRgk9V5/HZqa+neUDzBmTOjqqSYeKWcKfUm9u++Gjo7+hz6E93jggfBBIlKOFPqSegsWQG0tNDb2/72mTw8fIHff3f/3EikGhb6k2tatcNddcPLJUFPT//ebODEMw6wmHilXCn1JtYceCk0xhWjagfDBcfLJ4YOko6Mw7ylSSAp9SbWFC2HwYJg6tXDvOX06bNgQPlBEyo1CX1Jt4cIw720+4+zka+rU8EGiJh4pRwp9Sa1Vq+CppwrXtJOx665w3HEKfSlPCn1JrUwon3JK4d97+vQwTPPq1YV/b5H+UOhLai1cCIcdBvvvX/j3znx70NG+lBuFvqTS+vXhRGuhm3Yy9t8fPvQhhb6UH4W+pNJdd8G2bcULfQjv/eCDoSePSLlQ6EsqLVwI++wDDQ3F28b06eGD5a67ircNkd5S6EvqbN0ahkk4+eQdz4HbX42NsPfeauKR8qLQl9Rpagrz4RazaQfCB0rm6tytW4u7LZF8KfQldRYuhCFD4IQTir+t6dPDB8yDDxZ/WyL5UOhLqriH0J8yBXbZpfjbO+EEXZ0r5UWhL6ny5JNhOsNiN+1kDBsWPmAWLgwfOCKxKfQlVYp5FW53pk8PE6+vWlW6bYp0R6EvqbJwIRx+OIweXbptZj5g1MQj5UChL6mxfj08/HDpmnYy9tsP/umf4PbbS7tdka4o9CU1mpuhsxOOP7702z7+eFi8GLZsKf22RbLlFfpmNs3MVplZm5nN7eL5Y8zsUTPrMLPTc547y8yeSm5nFapwkd5qbg595+vrS7/tSZNg0yZYtqz02xbJ1mPom1kNcBVwIjAOmG1m43JWex44G/h9zmv3BC4BJgKNwCVmtkf/yxbpvebmMKpmKbpq5po0aXsNIjHlc6TfCLS5+xp33wLcAMzIXsHdn3X3ZUBnzms/BvzZ3de7+wbgz8C0AtQt0iudnbBo0fbwLbVRo+D971foS3z5hP5I4IWsx+3Jsnz057UiBfPkk/Dmm/FC3yxsW6EvseUT+tbFsnwvM8nrtWZ2npm1mlnrunXr8nxrkfxlwjZW6Ge2/fTToP/iElM+od8OZPdqHgWszfP983qtu89z93p3r6+trc3zrUXy19wMe+wBY8fGqyHzgbNoUbwaRPIJ/RZgrJmNMbPBwJnAgjzf/x5gqpntkZzAnZosEymp5uYw1HExh1LuyZFHQk2Nmngkrh7/BNy9A5hDCOuVwE3uvsLMLjWzUwHMrMHM2oEzgKvNbEXy2vXAdwgfHC3ApckykZJ56y144om4TTsAQ4eGi7QU+hLTwHxWcvc7gTtzll2cdb+F0HTT1WuvBa7tR40i/dLSEgY7ix36EGq4/vowo1ZNTexqJI10Ra5UvUwbemNj3DoghP5bb4XeRCIxKPSl6jU3w8EHw557xq5EF2lJfAp9qWruIWDLoWkH4MADw4ePQl9iUehLVXv2WXjllfIJfV2kJbEp9KWqlcNFWbkmTYIVK8IVwiKlptCXqtbcHLpKHnpo7Eq2mzgxNDu1tMSuRNJIoS9VrbkZGhpgYF6dk0sj04tITTwSg0JfqtamTfDYY+XVtAOw++5wyCEKfYlDoS9V67HHYOvW8gt92H4y1/MdulCkQBT6UrUyR9ITJ8atoyuTJsGrr8KaNbErkbRR6EvVam6G/feHffeNXcl76SItiUWhL1WrnC7KyjV+fJi2UaEvpabQl6q0di08/3z5hn5NTejFo9CXUlPoS1XKDLJWrqEPobalS2HjxtiVSJoo9KUqNTfD4MFwxBGxK+nepEnQ0QGPPhq7EkkThb5UpeZmOPxwGDIkdiXdy/QqUhOPlJJCX6pOR0cY4qCcm3YA9tkHxoxR6EtpKfSl6ixfHtrJyz30QSNuSukp9KXqlOPImt2ZNAna28NNpBQU+lJ1mpth772hri52JT3LfDBlehuJFJtCX6pO5qIss9iV9CxzsllNPFIqCn2pKq+9BqtXV0bTDoRupRMmKPSldPIKfTObZmarzKzNzOZ28fwQM7sxeX6RmdUlyweZ2XVmttzMVprZhYUtX+TdFi8OPysl9CHU2toaRgQVKbYeQ9/MaoCrgBOBccBsMxuXs9o5wAZ3PxC4Arg8WX4GMMTdPwQcCZyf+UAQKYbmZhgwAOrrY1eSv0mTwtj/y5bFrkTSIJ8j/Uagzd3XuPsW4AZgRs46M4Drkvs3A8ebmQEO7GJmA4GdgS2AZgaVolm0KAxmtuuusSvJn0bclFLKJ/RHAi9kPW5PlnW5jrt3AG8AIwgfAP8A/g48D/zI3df3s2aRLmXmnS3H8fN3ZPTocKGW5syVUsgn9LvqA5E730936zQC24D3A2OAL5vZAe/ZgNl5ZtZqZq3r1q3LoySR91qzBtavD3PiVhKzULNCX0ohn9BvB0ZnPR4FrO1unaQpZziwHvjfwN3uvtXdXwEeAt7T2uru89y93t3ra2tre/9biLD9JG6lhT6EmleuhLfeil2JVLt8Qr8FGGtmY8xsMHAmsCBnnQXAWcn904H73N0JTTpTLNgFmAQ8WZjSRd6tpQV22gkOPTR2Jb3X0BCap5YsiV2JVLseQz9po58D3AOsBG5y9xVmdqmZnZqsdg0wwszagAuATLfOq4BhwBOED49fubv6KEhRtLSEoZQHDYpdSe9lvp2oiUeKbWA+K7n7ncCdOcsuzrq/idA9M/d1b3e1XKTQOjrCUfJnPxu7kr7Za68w4mamiUqkWHRFrlSFv/0tjKzZ2Bi7kr7TyVwpBYW+VIVMWFbiSdyMxkZ47jlQBzYpJoW+VIWWFhg+HA48MHYlfad2fSkFhb5UhcWLw9ALAyr4f/SECaF+tetLMVXwn4hIsGlTmC2rktvzAYYNg0MO0ZG+FJdCXyre0qWh904lt+dnNDaG0Pfca95FCkShLxWvkq/EzdXQEE7kPvdc7EqkWin0peK1tMC++8LI3GEAK5BO5kqxKfSl4rW0hLCshOkRe3LYYWE2LYW+FItCXyraG2/AqlXV0bQDIfAPP1yhL8Wj0JeK1toaflZ6z51sDQ3h99q2LXYlUo0U+lLRMkfElTQ9Yk8aGuDtt8M3GJFCU+hLRWtpgQ98APbcM3YlhZP51qImHikGhb5UtMWLq6c9P+Pgg8Mcv7oyV4pBoS8V66WXoL29utrzIQzFcOSROtKX4lDoS8WqhpE1u9PQAI8/Dps3x65Eqo1CXyrW4sXhqPiII2JXUniNjbBlCyzTPHNSYAp9qVgtLWE+3F12iV1J4enKXCkWhb5UJPftV+JWo/32g9pahb4UnkJfKtIzz8D69dUb+mbbR9wUKSSFvlSkTHfGauu5k62hIcz9+9ZbsSuRaqLQl4rU0gI77RTa9KtVQ0Noxnr00diVSDXJK/TNbJqZrTKzNjOb28XzQ8zsxuT5RWZWl/XcYWb2iJmtMLPlZrZT4cqXtGppCQOTDRoUu5Li0clcKYYeQ9/MaoCrgBOBccBsMxuXs9o5wAZ3PxC4Arg8ee1A4LfA59x9PHAcsLVg1UsqdXTAkiXV3bQD4URuXZ2uzJXCyudIvxFoc/c17r4FuAGYkbPODOC65P7NwPFmZsBUYJm7Pw7g7q+5u8YOlH5ZuRLeead6T+Jma2jQkb4UVj6hPxJ4Ietxe7Ksy3XcvQN4AxgBHAS4md1jZo+a2Vf7X7KkXTVfiZuroQGefTZMoShSCPmEflfzEeVO29zdOgOBo4FPJj9nmdnx79mA2Xlm1mpmrev0v1t60NICu+0GY8fGrqT4Mk1YmXkDRPorn9BvB0ZnPR4FrO1unaQdfziwPll+v7u/6u7vAHcCE3I34O7z3L3e3etra2t7/1tIqmRG1hyQgr5nEyaEPvtq15dCyefPpgUYa2ZjzGwwcCawIGedBcBZyf3Tgfvc3YF7gMPMbGjyYXAs8LfClC5ptGlTGI8mDU07EIZYPuQQtetL4QzsaQV37zCzOYQArwGudfcVZnYp0OruC4BrgOvNrI1whH9m8toNZvYTwgeHA3e6+x1F+l0kBR5/PPTeSUvoQ/hd77or9NmvhsnfJa4eQx/A3e8kNM1kL7s46/4m4IxuXvtbQrdNkX575JHwc+LEuHWU0sSJcN11sGZNmCVMpD9S0Coq1aSpCcaMgZG5/ceq2OTJ4eeDD8atQ6qDQl8qhnsI/UwIpsW4cbDHHuF3F+kvhb5UjNWrQ3/1tIX+gAFw9NEKfSkMhb5UjEzopS30IfzOq1fDyy/HrkQqnUJfKkZTUxiP5qCDYldSekcfHX6qXV/6S6EvFaOpKYRfGrstHnkk7Lyzmnik/xT6UhFefDHMlpXGph2AwYND102FvvSXQl8qQprb8zMmT4alS+HNN2NXIpVMoS8VoakJhg0LE6ek1eTJ0Nm5/QI1kb5Q6EtFaGqCo46CgXldQ16djjoKamrUxCP9o9CXsrdhAzzxRLqbdiB80zniCIW+9I9CX8reQw+Fq3HTHvoQ9sGiRbB5c+xKpFIp9KXsPfhgmAA9TYOsdWfy5BD4S5bErkQqlUJfyl5TE9TXh37qaZe5SEtNPNJXCn0paxs3hglE1LQT1NbCBz+o0Je+U+hLWVu8GLZuVehnmzw5nOfo7IxdiVQihb6UtaamMOzChz8cu5LyMXkyvP566NEk0lsKfSlrTU1w6KFhPHkJMt961MQjfaHQl7LV0QEPP6ymnVz77w+jRin0pW8U+lK2Hn8c3n5boZ/LLOyTpqZw/YJIbyj0pWxljmQz3RRlu6OPhrVrw8ijIr2h0Jey1dQEdXWhKUPeTe360ld5hb6ZTTOzVWbWZmZzu3h+iJndmDy/yMzqcp7fz8zeNrOvFKZsqXZpnQQ9X+PHa7J06ZseQ9/MaoCrgBOBccBsMxuXs9o5wAZ3PxC4Arg85/krgLv6X66kRVonQc/XgAGhG6tCX3ornyP9RqDN3de4+xbgBmBGzjozgOuS+zcDx5uFSe3MbCawBlhRmJIlDTRpSs80Wbr0RT6hPxJ4Ietxe7Ksy3XcvQN4AxhhZrsAXwO+3f9SJU0efDAMOXDwwbErKV+ZD8SHHopbh1SWfEK/q2moczuKdbfOt4Er3P3tHW7A7DwzazWz1nXr1uVRklS7NE+Cni9Nli59kU/otwOjsx6PAtZ2t46ZDQSGA+uBicAPzOxZ4EvA181sTu4G3H2eu9e7e31tbW2vfwmpLmvXwpo1atrpiSZLl77IJ/RbgLFmNsbMBgNnAgty1lkAnJXcPx24z4PJ7l7n7nXAT4HvuvvPClS7VCm15+dv8mR47DF4663YlUil6DH0kzb6OcA9wErgJndfYWaXmtmpyWrXENrw24ALgPd06xTJlyZBz58mS5feymuaaXe/E7gzZ9nFWfc3AWf08B7f6kN9kkKaBD1/2ZOlT50auxqpBLoiV8rKiy/CsmXwkY/ErqQyDBsGDQ1w992xK5FKodCXsnLbbeHnzJlx66gkM2dCayu88ELP64oo9KWszJ8f+uYfckjsSirHrFnh5623xq1DKoNCX8rGhg3w179uDzHJz0EHwbhx4QNTpCcKfSkbt98eJk5R6PferFnwwAPw2muxK5Fyp9CXsjF/PowcCfX1sSupPLNmwbZtsHBh7Eqk3Cn0pSy8807ogTJzZhhBUnpnwgTYbz/4059iVyLlTn9eUhbuuQc2blTTTl+ZhQ/Me+8NU0yKdEehL2Vh/vwwKcgxx8SupHLNmgWbN6vPvuyYQl+i27o1tEVPnw6DBsWupnIdfTSMGKFePLJjCn2J7v774fXX1bTTXwMHwqmnwh13wJYtsauRcqXQl+jmzw/jwmvsmP6bNQveeAP+8pfYlUi5UuhLVJ2d4UrSadNg6NDY1VS+E06AXXZRE490T6EvUbW0hElT1LRTGDvtBCeeGMYw6uyMXY2UI4W+RDV/fmiLPuWU2JVUj1mz4KWXoLk5diVSjhT6Eo17CP3jjgvdNaUwTj459IJSE490RaEv0axcCatXq2mn0IYPhylTQui7x65Gyo1CX6LJHInOmBG3jmo0axY8/TQ88UTsSqTcKPQlmvnzYeLEMMiaFNaMGWFoBjXxSC6FvkTx/POwZImadorlfe8L8+cq9CWXQl+iyMzypNAvntNOg6VL4ZlnYlci5UShL1HMnw/jx4dZn6Q4NI2idCWv0DezaWa2yszazGxuF88PMbMbk+cXmVldsvwEM1tiZsuTn1MKW75UoldfDbM86Si/uA44AA47TE088m49hr6Z1QBXAScC44DZZjYuZ7VzgA3ufiBwBXB5svxVYLq7fwg4C7i+UIVL5Vq4MFwtqtAvvlmz4MEH4eWXY1ci5SKfI/1GoM3d17j7FuAGILeT3QzguuT+zcDxZmbu/pi7r02WrwB2MrMhhShcKpM7XHUVHHggHHFE7Gqq38c/Hvb51VfHrkTKRT6hPxJ4Ietxe7Ksy3XcvQN4AxiRs87/Ah5z9819K1Wqwb33hl47X/ta6FIoxTVuXJin4MorNaOWBPmEfld/mrnX+e1wHTMbT2jyOb/LDZidZ2atZta6bt26PEqSSnXZZTBqFHz607ErSY+LLoL16+EXv4hdiZSDfEK/HRid9XgUsLa7dcxsIDAcWJ88HgXMBz7t7k93tQF3n+fu9e5eX1tb27vfQCrGAw9AUxN89asweHDsatJj4kT46EfhRz8K8xBLuuUT+i3AWDMbY2aDgTOBBTnrLCCcqAU4HbjP3d3MdgfuAC5094cKVbRUpssug733hnPPjV1J+lx0UTiZe+21sSuR2HoM/aSNfg5wD7ASuMndV5jZpWZ2arLaNcAIM2sDLgAy3TrnAAcC3zSzpclt74L/FlL2WlpCe/6XvxxmyZLSOvZY+PCH4fLLNZVi2pmX2TB89fX13traGrsMKbCZM0PzznPPwa67xq4mne66C046Ca65Bv71X2NXI4VmZkvcvb6n9XRFrhTd8uVhJqcvflGBH9O0aTBhAnzve9DREbsaiUWhL0X3ve/BsGHwhS/EriTdzELbflsb/PGPsauRWBT6UlRPPQU33gj/9m+w556xq5GZM0Pf/e9+V3PoppVCX4rq+98P3TMvuCB2JQIwYAB8/ethcpUFuX3wJBUU+lI0zz0Hv/kNfPazsM8+sauRjE98IgzGdtllmk4xjRT6UjQ//GFoR/6P/4hdiWQbOBAuvBBaW0M3WkkXhb4UxUsvwS9/CWedBaNH97y+lNanPx2Gw7jsstiVSKkp9KUofvxj2Lo1DKwm5Wfw4PANrKkpXD8h6aHQl4Jrb4ef/xxmzw5DKEt5OvfcMCzGxRfDtm2xq5FSUehLQf3jHzBjRuglcsklsauRHRk6FP7zP+H++0Mbv6TDwNgFSPXo7Axt+EuXhtmxxo6NXZH05LOfDf9eP/whHHIIfOYzsSuSYtORvhTMJZfALbeEIXxPOil2NZKvn/40DL18/vmhjV+qm0JfCuL3vw9NBeeeC1/6UuxqpDcGDYKbboIxY+C00+CZZ2JXJMWk0Jd+a24OozYee2yY/1bTIFaePfaA228PJ3SnT4c334xdkRSLQl/65fnnw3guI0fCzTdrRqxKNnZs+DdctSr0vFKPnuqk0Jc+e/ttOPXUMAXfwoWw116xK5L+mjIFfvYzuPPOMK2lVB/13pE+6eyEf/mXMFb+HXeEkRulOpx/Pvztb/CTn4QePZresroo9KXX3nknjJp5221w5ZVhcg6pLj/+cWjm+fznQ3/+2bN1rqZaqHlHemXBgnBUf/XVYb5bTYxSnQYODPMgTJgAn/wkTJ0Kq1fHrkoKQaEveXnmmdCrY8aMMAvW/feH/vg6+qtew4fDww/Df/0XLF4MH/oQfPOb4ZueVC6FvuzQ5s1hJMZx4+AvfwlXbj72GBxzTOzKpBRqamDOnNDU8/GPh2sxxo8P3TulMin0pVv//d9w2GHwjW/AKafAk0/CV74SLuaRdHnf++D668MH/9Ch4VvfzJlhohypLHmFvplNM7NVZtZmZnO7eH6Imd2YPL/IzOqynrswWb7KzD5WuNKl0N58M7TZz5kDBx8MJ5wQeuncfXeYSHvUqNgVSmzHHRe+6V1+Ofz5z/CBD8DRR8O3vw2PPAIdHbErlJ70GPpmVgNcBZwIjANmm1luB71zgA3ufiBwBXB58tpxwJnAeGAa8H+S95My0NERrqb9zndg8mQYMSK02f/qV2FI5J/9LHTJ/Jg+qiXL4MGhD//KlTB3LmzZEkL/n/85XKtx2mnwi1/A00/3cQO/+x3U1YWhWuvqwmMpGPMeJsk0s6OAb7n7x5LHFwK4+/ey1rknWecRMxsIvATUAnOz181er7vt1dfXe2tra79+qbTati1cKJV927ABXnwxjHH/4ovvvb9pUzgZe+SR4ch+6lQ46igYMiT2byOV5LXX4L77wvSL994brtQG2H33cLX2qFHhZ/b9978/dAoYOhR23jncdrrld9j55737bPHQoTBvXuhGJN0ysyXuXt/Tevn00x8JvJD1uB2Y2N067t5hZm8AI5LlzTmvHZnHNnttw4byPSLtafJp9+233MeZ27Zt7751dm6/nwn4LVt2vJ0hQ7b/4TU2hp/19XD88bqaVvpnxAg444xwc4enngrNP08+uf0gY/nyMI1mZ+eO3umT7MwsdmYjQ9hMDduoeWcbA84eQM23wonlmprwJWBA0k5h9u5b9rLulGuvs0MPhWuvLe428gn9rnZPbox1t04+r8XMzgPOA9hvv/3yKOm9Bgwo7+Dq6T9Zd/9pzbb/B8/8h8/8p8/czxwl7bzzu4+ahg6F3XbbfmQ1YkT5/meX6mEGBx0Ubrk6OkLwt7fD3/8eJt3ZuDEc2G/cCBsvvJR32JmN7MxmhrCNGjoZwLaOGrY1fOpdBzxdHRzBu+93paeDsJiGDy/+NvIJ/XYge2rrUcDabtZpT5p3hgPr83wt7j4PmAeheSff4rMNHx7GCxGR8jVwYDgI6bZTwC+u7bpL0P77w+8/VdTa0iKf3jstwFgzG2NmgwknZhfkrLMAOCu5fzpwn4eTBQuAM5PePWOAscDiwpQuIlXnssvCV9RsQ4eG5VIQPR7pJ230c4B7gBrgWndfYWaXAq3uvgC4BrjezNoIR/hnJq9dYWY3AX8DOoB/d3cN2CoiXcucrL3oonA2eL/9QuDrJG7B9Nh7p9TUe0dEpPfy7b2jK3JFRFJEoS8ikiIKfRGRFFHoi4ikiEJfRCRFFPoiIimi0BcRSRGFvohIipTdxVlmtg7oz3w8ewGvFqicQlJdvaO6ekd19U411rW/u9f2tFLZhX5/mVlrPlellZrq6h3V1Tuqq3fSXJead0REUkShLyKSItUY+vNiF9AN1dU7qqt3VFfvpLauqmvTFxGR7lXjkb6IiHSjakLfzKaZ2SozazOzubHryTCzZ81suZktNbOoEwWY2bVm9oqZPZG1bE8z+7OZPZX83KNM6vqWmb2Y7LelZnZSiWsabWZ/MbOVZrbCzL6YLI+6v3ZQV+z9tZOZLTazx5O6vp0sH2Nmi5L9dWMy+1451PVrM3sma38dXsq6suqrMbPHzOz25HHx95e7V/yNMKPX08ABwGDgcWBc7LqS2p4F9opdR1LLMcAE4ImsZT8/SrRZAAADJklEQVQA5ib35wKXl0ld3wK+EnFf7QtMSO7vCqwGxsXeXzuoK/b+MmBYcn8QsAiYBNwEnJks/wXw+TKp69fA6bH2V1Z9FwC/B25PHhd9f1XLkX4j0Obua9x9C3ADMCNyTWXH3R8gTGeZbQZwXXL/OmBmSYui27qicve/u/ujyf23gJXASCLvrx3UFZUHbycPByU3B6YANyfLY+yv7uqKzsxGAScDv0weGyXYX9US+iOBF7Iet1MGfwgJB+41syVmdl7sYrqwj7v/HUKgAHtHrifbHDNbljT/lLzZKcPM6oAjCEeJZbO/cuqCyPsraapYCrwC/Jnw7ft1d+9IVonyd5lbl7tn9tdlyf66wsyGlLou4KfAV4HO5PEISrC/qiX0rYtlZfFpDnzY3ScAJwL/bmbHxC6oQvwc+ABwOPB34McxijCzYcAtwJfc/c0YNXSli7qi7y933+buhwOjCN++D+lqtdJW9d66zOxQ4ELgg0ADsCfwtVLWZGanAK+4+5LsxV2sWvD9VS2h3w6Mzno8ClgbqZZ3cfe1yc9XgPmEP4Zy8rKZ7QuQ/Hwlcj0AuPvLyR9rJ/B/ibDfzGwQIVh/5+5/ShZH319d1VUO+yvD3V8H/kpoO9/dzAYmT0X9u8yqa1rSTObuvhn4FaXfXx8GTjWzZwnN0VMIR/5F31/VEvotwNjkzPdg4ExgQeSaMLNdzGzXzH1gKvDEjl9VcguAs5L7ZwG3Razlf2SCNTGLEu+3pH31GmClu/8k66mo+6u7uspgf9Wa2e7J/Z2BjxLON/wFOD1ZLcb+6qquJ7M+uI3Qbl7S/eXuF7r7KHevI+TVfe7+SUqxv2KfvS7UDTiJ0JPhaeCi2PUkNR1A6En0OLAidl3AHwhf/bcSvh2dQ2hH/H/AU8nPPcukruuB5cAyQtDuW+KajiZ8tV4GLE1uJ8XeXzuoK/b+Ogx4LNn+E8DFyfIDgMVAG/BHYEiZ1HVfsr+eAH5L0sMnxg04ju29d4q+v3RFrohIilRL846IiORBoS8ikiIKfRGRFFHoi4ikiEJfRCRFFPoiIimi0BcRSRGFvohIivx/xuwv0aNkIhMAAAAASUVORK5CYII=\n", "text/plain": [ "
" ] }, "metadata": { "needs_background": "light" }, "output_type": "display_data" }, { "name": "stdout", "output_type": "stream", "text": [ "1.266040180780692e-05\n" ] }, { "data": { "image/png": "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\n", "text/plain": [ "
" ] }, "metadata": { "needs_background": "light" }, "output_type": "display_data" }, { "name": "stdout", "output_type": "stream", "text": [ "7.548600478912704e-06\n" ] }, { "data": { "image/png": "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\n", "text/plain": [ "
" ] }, "metadata": { "needs_background": "light" }, "output_type": "display_data" }, { "name": "stdout", "output_type": "stream", "text": [ "2.0967078967260022e-05\n" ] }, { "data": { "image/png": "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\n", "text/plain": [ "
" ] }, "metadata": { "needs_background": "light" }, "output_type": "display_data" }, { "name": "stdout", "output_type": "stream", "text": [ "1.2724299324418098e-05\n" ] }, { "data": { "image/png": "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\n", "text/plain": [ "
" ] }, "metadata": { "needs_background": "light" }, "output_type": "display_data" }, { "name": "stdout", "output_type": "stream", "text": [ "7.687045098236922e-06\n" ] }, { "data": { "image/png": "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\n", "text/plain": [ "
" ] }, "metadata": { "needs_background": "light" }, "output_type": "display_data" }, { "name": "stdout", "output_type": "stream", "text": [ "4.623849960694315e-06\n" ] }, { "data": { "image/png": "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\n", "text/plain": [ "
" ] }, "metadata": { "needs_background": "light" }, "output_type": "display_data" }, { "name": "stdout", "output_type": "stream", "text": [ "2.769810798497474e-06\n" ] }, { "data": { "image/png": "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\n", "text/plain": [ "
" ] }, "metadata": { "needs_background": "light" }, "output_type": "display_data" }, { "name": "stdout", "output_type": "stream", "text": [ "1.652633354609634e-06\n" ] }, { "data": { "image/png": "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\n", "text/plain": [ "
" ] }, "metadata": { "needs_background": "light" }, "output_type": "display_data" }, { "name": "stdout", "output_type": "stream", "text": [ "9.823268882769298e-07\n" ] }, { "data": { "image/png": "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\n", "text/plain": [ "
" ] }, "metadata": { "needs_background": "light" }, "output_type": "display_data" }, { "name": "stdout", "output_type": "stream", "text": [ "5.81777902297631e-07\n" ] }, { "data": { "image/png": "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\n", "text/plain": [ "
" ] }, "metadata": { "needs_background": "light" }, "output_type": "display_data" }, { "name": "stdout", "output_type": "stream", "text": [ "3.433558868337627e-07\n" ] }, { "data": { "image/png": "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\n", "text/plain": [ "
" ] }, "metadata": { "needs_background": "light" }, "output_type": "display_data" }, { "name": "stdout", "output_type": "stream", "text": [ "2.0196608696565477e-07\n" ] }, { "data": { "image/png": "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\n", "text/plain": [ "
" ] }, "metadata": { "needs_background": "light" }, "output_type": "display_data" }, { "name": "stdout", "output_type": "stream", "text": [ "1.184175689727951e-07\n" ] }, { "data": { "image/png": "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\n", "text/plain": [ "
" ] }, "metadata": { "needs_background": "light" }, "output_type": "display_data" }, { "name": "stdout", "output_type": "stream", "text": [ "6.921677486726452e-08\n" ] }, { "data": { "image/png": "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\n", "text/plain": [ "
" ] }, "metadata": { "needs_background": "light" }, "output_type": "display_data" }, { "name": "stdout", "output_type": "stream", "text": [ "4.0338048012955596e-08\n" ] }, { "data": { "image/png": "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\n", "text/plain": [ "
" ] }, "metadata": { "needs_background": "light" }, "output_type": "display_data" }, { "name": "stdout", "output_type": "stream", "text": [ "2.3440920648987174e-08\n" ] }, { "data": { "image/png": "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\n", "text/plain": [ "
" ] }, "metadata": { "needs_background": "light" }, "output_type": "display_data" }, { "name": "stdout", "output_type": "stream", "text": [ "1.3584262936738867e-08\n" ] }, { "data": { "image/png": "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\n", "text/plain": [ "
" ] }, "metadata": { "needs_background": "light" }, "output_type": "display_data" }, { "name": "stdout", "output_type": "stream", "text": [ "7.851308825301828e-09\n" ] }, { "data": { "image/png": "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\n", "text/plain": [ "
" ] }, "metadata": { "needs_background": "light" }, "output_type": "display_data" }, { "name": "stdout", "output_type": "stream", "text": [ "4.52619564050849e-09\n" ] }, { "data": { "image/png": "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\n", "text/plain": [ "
" ] }, "metadata": { "needs_background": "light" }, "output_type": "display_data" }, { "name": "stdout", "output_type": "stream", "text": [ "2.602845716737079e-09\n" ] }, { "data": { "image/png": "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\n", "text/plain": [ "
" ] }, "metadata": { "needs_background": "light" }, "output_type": "display_data" }, { "name": "stdout", "output_type": "stream", "text": [ "1.493220769255288e-09\n" ] }, { "data": { "image/png": "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\n", "text/plain": [ "
" ] }, "metadata": { "needs_background": "light" }, "output_type": "display_data" }, { "name": "stdout", "output_type": "stream", "text": [ "8.54663007032741e-10\n" ] }, { "data": { "image/png": "iVBORw0KGgoAAAANSUhEUgAAAX0AAAD8CAYAAACb4nSYAAAABHNCSVQICAgIfAhkiAAAAAlwSFlzAAALEgAACxIB0t1+/AAAADl0RVh0U29mdHdhcmUAbWF0cGxvdGxpYiB2ZXJzaW9uIDMuMC4zLCBodHRwOi8vbWF0cGxvdGxpYi5vcmcvnQurowAAH9xJREFUeJzt3X2QVPWd7/H3FwZQFAFhvCoPDgpEZ6IO4wQfIInrU4BNRDe6ap7Mxlo2lVi1W0nW4E1d15g1FbN3o6mNuRuzZpOKbtSY6GUNSFS8iWKCDDAKA5IMagCRMAqCoPL4vX/8ThdtO8M0M919nj6vqq7uPv3r7u+B7s+c/p3f+R1zd0REJB8GxF2AiIjUjkJfRCRHFPoiIjmi0BcRyRGFvohIjij0RURyRKEvIpIjCn0RkRxR6IuI5Ehd3AWUGj16tDc0NMRdhohIqixbtuw1d6/vrV3iQr+hoYG2tra4yxARSRUz+1M57dS9IyKSIwp9EZEcUeiLiOSIQl9EJEcU+iIiOaLQFxHJkbJC38xmmNlaM+s0s7ndPP4hM1tuZvvM7IqSx641sz9Gl2srVbiIiBy+XkPfzAYCdwIzgUbgGjNrLGm2Hvgs8F8lzz0W+CfgbGAq8E9mNrL/ZYvUljvccw+sXx93JSL9U86W/lSg091fdPc9wH3A7OIG7v6yuz8PHCh57keAx9x9q7tvAx4DZlSgbpGa+s534NOfho98BHbsiLsakb4rJ/THABuK7m+MlpWjP88VSYTHHoMbboBp06CzEz71KThQunkjkhLlhL51s8zLfP2ynmtmc8yszczaurq6ynxpkepbtw6uugoaG+HRR+GOO+C//xtuvjnuykT6ppzQ3wiMK7o/FthU5uuX9Vx3v8vdW929tb6+1/mCRGpi50647LJw++GH4eij4QtfgM99Dr7xDfjlL+OtT6Qvygn9pcAkM5tgZoOBq4F5Zb7+QuASMxsZ7cC9JFomkmju8NnPwurVcP/9cMopYbkZfP/7cPbZ8JnPwKpVsZYpcth6DX133wdcTwjrNcAD7t5hZreY2aUAZvYBM9sIXAn8wMw6ouduBb5B+MOxFLglWiaSaN/8JvziF/Dtb8PFF7/7sSFDwlb+sGEwezZs2xZPjSJ9Ye7lds/XRmtrq2tqZYnTzp0wahRceik88EDYuu/OM8+Enbvf+hZ89au1rVGklJktc/fW3trpiFyREosWwZ498PnP9xz4AOedB2ecAQsW1K42kf5S6IuUWLAg7LSdPr33trNmweLFGrsv6aHQFyniHkL/wgtD331vZs6Effvg8cerX5tIJSj0RYqsWQN/+lMI83Kcey4ccwzMn1/dukQqRaEvUqTQP19u6A8aFEb3PPpo+JUgknQKfZEiCxZAUxOMH1/+c2bNgldegZUrq1eXSKUo9EUib74Jv/1t+Vv5BTOiKQTVxSNpoNAXiSxaBHv3Hn7on3ginHmmhm5KOij0RSKHM1SzVGHo5vbtla9LpJIU+iIcHKp50UUwePDhP3/mTNi/X0M3JfkU+iKEidXWrz/8rp2Cc8+F4cPVry/Jp9AX4fCHapaqq4NLLtHQTUk+hb4IIfTf/34YN673tj2ZORM2bYLnn69cXSKVptCX3HvzTXjqqb5v5Rdo6KakgUJfcu+JJ/o2VLPUCSdAc7OGbkqyKfQl9xYsCCdEmTat/681a1aYZ/+NN/r/WiLVoNCX3HviCbjggr4N1Sw1Y0YYuvmb3/T/tUSqQaEvubZtG6xbB+ecU5nXO+ssGDAAli+vzOuJVJpCX3KtvT1ct7RU5vWGDoXTTlPoS3Ip9CXXCuE8ZUrlXnPKFIW+JJdCX3Jt+XIYOxbq6yv3mi0tYbz+5s2Ve02RSlHoS64tX165rp2CwuutWFHZ1xWpBIW+5NbOnbB2beVDv7k5XKuLR5JIoS+59fzzYZ6cSof+8OEwcaK29CWZFPqSW4Ut8UqHfuE1taUvSaTQl9xavjzswD3xxMq/9pQp8NJL4TgAkSRR6EtuFXbimlX+tbUzV5JKoS+59M470NFRna4dODjuX108kjQKfcmlVatg377qhX59fZibX6EvSaPQl1wqdLtUK/QLr63uHUmaskLfzGaY2Voz6zSzud08PsTM7o8eX2JmDdHyQWb2EzNbaWZrzOzGypYv0jfLl4ehlRMmVO89WlrCcQA7d1bvPUQOV6+hb2YDgTuBmUAjcI2ZNZY0uw7Y5u4TgduB26LlVwJD3P104Czg7wp/EETiVM2duAUtLeE4gOeeq957iByucrb0pwKd7v6iu+8B7gNml7SZDfwkuv0gcKGZGeDAUWZWBxwJ7AF2VKRykT7auzcEcSUnWeuOduZKEpUT+mOADUX3N0bLum3j7vuA7cAowh+AXcCrwHrgf7v71n7WLNIvL7wAu3dXtz8fwvj/445T6EuylBP63f0A9jLbTAX2AycCE4Avm9nJ73kDszlm1mZmbV1dXWWUJNJ31TwSt5iZjsyV5Ckn9DcC44rujwU29dQm6soZDmwFPgE86u573X0LsBhoLX0Dd7/L3VvdvbW+knPcinRjxYpwspPJk6v/Xi0tsHp1OC5AJAnKCf2lwCQzm2Bmg4GrgXklbeYB10a3rwAWubsTunQusOAo4BzghcqULtI3y5eHmTAHDqz+e7W0hOMBVq2q/nuJlKPX0I/66K8HFgJrgAfcvcPMbjGzS6NmdwOjzKwT+BJQGNZ5J3A0sIrwx+M/3f35Cq+DSNkOHAhb+tXu2ikovI+6eCQp6spp5O7zgfkly24quv0OYXhm6fN2drdcJC6dnWHcfK1Cv6EBRoxQ6Ety6IhcyZVqnBP3UMx0zlxJFoW+5Mry5TB4MDSWHl5YRS0t4YQte/fW7j1FeqLQl1xZsQLe//4Q/LXS0hKOC3hBQxgkART6kiurVsEZZ9T2PU8//eB7i8RNoS+5sXUrbN4MTU21fd/Jk8Pw0I6O2r6vSHcU+pIbhdCtdegPGQKTJin0JRkU+pIbcYV+4T0V+pIECn3JjY4OGDYsnNGq1pqaYN06Tccg8VPoS250dIShmtWcQ78nTU3haGCN4JG4KfQlNzo64unagYPvqy4eiZtCX3Lhtddgy5b4Qn/SJKirU+hL/BT6kguFsK3lkbjFBg/WCB5JBoW+5EKcI3cKNIJHkkChL7nQ0QHHHANjx8ZXQ1MTvPgivPVWfDWIKPQlF1avjm/kTkFTE7hrBI/ES6EvuRDnyJ0CjeCRJFDoS+Z1dYVL3KE/aRIMGqTQl3gp9CXzkrATF0LgT56s0Jd4KfQl85IS+oUaFPoSJ4W+ZF5HBwwfDieeGHclIfRfegl27Yq7Eskrhb5kXmEnbpwjdwoKvzY0gkfiotCXTHNPxsidAo3gkbgp9CXTtmyB119PTuhPnBimZFDoS1wU+pJpSdqJC2HStfe9T6Ev8VHoS6YlLfRBI3gkXgp9ybSODhg5Eo4/Pu5KDmpqgpdfhp07465E8kihL5mWpJE7BYVfHWvWxFuH5JNCXzIraSN3CjSCR+Kk0JfM2rwZtm1LXuifcgoMGaLQl3go9CWzkrgTF2DgQDj1VIW+xKOs0DezGWa21sw6zWxuN48PMbP7o8eXmFlD0WNnmNnvzKzDzFaa2RGVK1+kZ0kNfdAIHolPr6FvZgOBO4GZQCNwjZmVnmn0OmCbu08Ebgdui55bB9wDfN7dm4Dzgb0Vq17kEFavhlGj4Ljj4q7kvRobYf16ePPNuCuRvClnS38q0OnuL7r7HuA+YHZJm9nAT6LbDwIXmpkBlwDPu/tzAO7+urvvr0zpIofW0RH/2bJ6Uvj1sXp1vHVI/pQT+mOADUX3N0bLum3j7vuA7cAoYDLgZrbQzJab2Q3dvYGZzTGzNjNr6+rqOtx1EHmPpI7cKdAIHolLOaHf3XaSl9mmDpgOfDK6vtzMLnxPQ/e73L3V3Vvr6+vLKEnk0F59Fd54I7mhf/LJcMQRCn2pvXJCfyMwruj+WGBTT22ifvzhwNZo+W/c/TV3fwuYD7T0t2iR3iR5Jy5oBI/Ep5zQXwpMMrMJZjYYuBqYV9JmHnBtdPsKYJG7O7AQOMPMhkZ/DD4MqBdTqi7poQ8awSPx6DX0oz766wkBvgZ4wN07zOwWM7s0anY3MMrMOoEvAXOj524DvkP4w9EOLHf3X1V+NUTeraMDRo9O5sidgqYm2LgRtm+PuxLJk7pyGrn7fELXTPGym4puvwNc2cNz7yEM2xSpmSTvxC0oHsFz7rnx1iL5oSNyJXOSPnKnQCN4JA4KfcmcV16BHTuSH/oTJsCRRyr0pbYU+pI5adiJCzBgAJx2mg7QktpS6EvmpCX0QSN4pPYU+pI5HR1h1M7o0XFX0rumptAd9cYbcVcieaHQl8xJw07cAs3BI7Wm0JdMcQ8BmrbQVxeP1IpCXzJlw4YwXXFaQv+kk2DoUIW+1I5CXzIlTTtxIYzgaWxU6EvtKPQlU9IW+qARPFJbCn3JlI4OOP54OPbYuCspX1NTmAp627a4K5E8UOhLpqRp5E6BduZKLSn0JTMOHEjXyJ0Chb7UkkJfMmP9eti1K32hP348HH20Ql9qQ6EvmZHGnbgQTtyuETxSKwp9yYxCaDY2xltHX2gEj9SKQl8yo6MDTjgBRo6Mu5LD19gIf/4zvP563JVI1in0JTPSOHKnQDtzpVYU+pIJBw7AmjUKfZHeKPQlE15+Gd56K72hP24cDBum0JfqU+hLJqR15E6BRvBIrSj0JRMK89GnceROQVOT5tWX6lPoSyasXAljx8KIEXFX0nennw5btoRRPCLVotCXTGhvhylT4q6if5qbw/Vzz8Vbh2SbQl9S7+234YUXDoZmWp15ZrhesSLeOiTbFPqSeqtWwf796Q/9kSPDmbTa2+OuRLJMoS+pVwjJtIc+hHVQ6Es1KfQl9drb4ZhjoKEh7kr6r7kZ1q4Ns4WKVINCX1KvvT2E5YAMfJqnTAH30GUlUg1lfU3MbIaZrTWzTjOb283jQ8zs/ujxJWbWUPL4eDPbaWZfqUzZIsH+/WG0Sxa6duDgemhnrlRLr6FvZgOBO4GZQCNwjZmVHgJzHbDN3ScCtwO3lTx+O7Cg/+WKvNu6daErJCuhP358ONZA/fpSLeVs6U8FOt39RXffA9wHzC5pMxv4SXT7QeBCMzMAM7sMeBHQAeZScVnaiQthOgbtzJVqKif0xwAbiu5vjJZ128bd9wHbgVFmdhTwVeDr/S9V5L3a26GuLt3TL5Rqbobnnw9dVyKVVk7oWzfLvMw2Xwdud/edh3wDszlm1mZmbV1dXWWUJBK0t4fAHzIk7koqp7k5HHD2xz/GXYlkUTmhvxEYV3R/LLCppzZmVgcMB7YCZwPfNrOXgX8A/qeZXV/6Bu5+l7u3untrfX39Ya+E5NeKFemffqFUYX20M1eqoZzQXwpMMrMJZjYYuBqYV9JmHnBtdPsKYJEHH3T3BndvAO4Avunu36tQ7ZJzmzeHS1b68wtOPRUGD1a/vlRHXW8N3H1ftHW+EBgI/MjdO8zsFqDN3ecBdwM/NbNOwhb+1dUsWgQOTkyWtdAfPDhMs6zQl2roNfQB3H0+ML9k2U1Ft98BruzlNW7uQ30iPSqEYmGisixpboZHHgkHall3e8xE+igDxzBKXrW3hwnKRo6Mu5LKa26Grq7QfSVSSQp9Sa3C9AtZVFgvdfFIpSn0JZV27QoTk2Vt5E6B5taXalHoSyqtXBn6u7O6pT98OJx8srb0pfIU+pJKWZt+oTuajkGqQaEvqdTeHiYmGz8+7kqqp7kZOjvhzTfjrkSyRKEvqVTYiZvl4YzNzaELa+XKuCuRLFHoS+rs3x8mJMvqTtwCTccg1aDQl9T5wx/ChGRZ7s8HGDMGRo1Sv75UlkJfUicPO3FBc+tLdSj0JXWWLg1TKZ96atyVVF9LS+jKeueduCuRrFDoS+osXgxTp4aJybLuvPNgzx5YtizuSiQrFPqSKm+9BcuXw/TpcVdSG9OmhevFi+OtQ7JDoS+p8uyzsG/fwTDMuvp6mDwZnn467kokKxT6kiqF8DvvvHjrqKXp08OW/oEDcVciWaDQl1RZvDicYCSL0yn3ZNo02Lo1TDAn0l8KfUmN/fvhmWfy059fUFhfdfFIJSj0JTU6OmDHjvz05xdMmhT69rUzVypBoS+pUdjSzduWvln4Q6ctfakEhb6kxuLFcOKJ0NAQdyW1N306rFun0ydK/yn0JTWefjps8WZ5Zs2eaLy+VIpCX1JhwwZYvz5/XTsFLS1wxBHq4pH+U+hLKhS2cPO2E7dg8OAw9YS29KW/FPqSCk8/DUcddfCE4Xk0fXqYgmLXrrgrkTRT6EsqLF4M55wDdXVxVxKfadPCsQrPPht3JZJmCn1JvB07wvTCee3PLzj33LATW/360h8KfUm83/8+zDuT1/78gpEjwxQU6teX/lDoS+I9/TQMGBC6d/Ju+vQwFcX+/XFXImml0JfEW7w47MAdNizuSuI3bRq8+SasWhV3JZJWCn1JtL17Q/dO3vvzCzT5mvRXWaFvZjPMbK2ZdZrZ3G4eH2Jm90ePLzGzhmj5xWa2zMxWRtcXVLZ8ybr29nC2LIV+cNJJMGaMQl/6rtfQN7OBwJ3ATKARuMbMGkuaXQdsc/eJwO3AbdHy14CPufvpwLXATytVuOTDokXhWqEfmMEHPwhPPqmTqkjflLOlPxXodPcX3X0PcB8wu6TNbOAn0e0HgQvNzNx9hbtvipZ3AEeY2ZBKFC758PDDcNZZYaI1CT72Mfjzn2HJkrgrkTQqJ/THABuK7m+MlnXbxt33AduBUSVtPg6scPfdpW9gZnPMrM3M2rq6usqtXTJu06bQn3/ZZXFXkiyzZoWD1B56KO5KJI3KCf3u5jT0w2ljZk2ELp+/6+4N3P0ud29199b6+voySpI8mDcvXF9+ebx1JM2IEXDBBSH0vfSbKNKLckJ/IzCu6P5YYFNPbcysDhgObI3ujwUeAj7j7uv6W7Dkx0MPwcSJ0Fi6B0m47DLo7ITVq+OuRNKmnNBfCkwyswlmNhi4GphX0mYeYUctwBXAInd3MxsB/Aq40d11HKGU7Y03wk7cyy/P5/z5vZkd7VV7+OF465D06TX0oz7664GFwBrgAXfvMLNbzOzSqNndwCgz6wS+BBSGdV4PTAT+l5m1R5fjKr4Wkjnz58O+fera6cmJJ4YjlNWvL4fLPGGdgq2trd7W1hZ3GRKzv/5reOopeOWVMAWDvNdtt8HcueHkMuPG9d5ess3Mlrl7a2/t9HWSxHnnHViwIHRhKPB7VvgVpC4eORz6SkniPP447NypoZq9mTwZTjtNXTxyeBT6kjgPPwzHHBOGJcqhXX45/Pa38PrrcVciaaHQl0TZvz+Mz581K5wXVg7tssvCv9kjj8RdiaSFQl8S5ZlnoKtLo3bK1doaJmBTv76US6EvifLQQ2ELf8aMuCtJB7Owtb9wYZiNVKQ3Cn1JDPewxXrRRaFPX8pz+eXw9tvw61/HXYmkgUJfEuO55+CllzRq53B96EPh/Lm/+EXclUgaKPQlMf7t3+CII9Sff7gGDQoHs/3857B5c9zVSNIp9CURNm2Cn/4UPvc5GD067mrS50tfgj17wh9OkUNR6EsifPe7Yejhl78cdyXpNHky/NVfwfe/H06cLtIThb7Ebvt2+Pd/hyuvhJNPjrua9LrhhjA76Q9/GHclkmQKfYndD34AO3aE0JK+mzoVzj8fbr89dPWIdEehL7HavRvuuCMM02xpibua9LvhBti4EX72s7grkaRS6Eus7rkHXn0VvvrVuCvJhhkz4PTT4V/+BQ4ciLsaSSKFvsTmwIEQTlOmwIUXxl1NNpiFrf2OjjA9tUgphb7EZt48WLs2hJROiVg5V10F48eHk6yIlFLoSyzcQyhNmABXXBF3NdkyaFAYt//UU/C738VdjSSNQl9i8dBD8Pvfh3H5dXVxV5M9110Hxx4b9pXs3x93NZIkCn2puT/9KYRSayv87d/GXU02HX10GLr51FPwz/8cdzWSJAp9qam9e+Gaa8LW53336UQp1fSZz4TLLbfAb34TdzWSFAp9qambbgr9zD/8IZxyStzVZN+dd8LEifCJT4ST04go9KVmfv1r+Na3QpfOVVfFXU0+HH003H9/OIfuZz+rsfui0Jca2bwZPv1paGoKR+BK7TQ3w7/+K8yfH/r5Jd8U+lJ1u3fDpz4VZn984AEYOjTuivLnC18I5ymYOxcWL467GomTQl+qasOGcGanJ54I/cuNjXFXlE9mcPfd4aCtiy6CH/847ookLgp9qZpFi+Css2DNmnAqv7/5m7gryreRI8OxEeedF/4vPv/58CtM8kWhLxXnHubUufjicBasZ58NJ/iQ+NXXw8KFYeqLH/wAPvzhMCun5IdCXypq+fJwYvMbbghBv2QJnHpq3FVJsbq6MAXGgw+GidlaWuBHP4K33467MqmFskLfzGaY2Voz6zSzud08PsTM7o8eX2JmDUWP3RgtX2tmH6lc6ZIUu3fDvfeGboOzzoLHHw9b+g88AMOGxV2d9OTjH4elS2Hs2HCE9Nix8I//CC++GHdlUk29hr6ZDQTuBGYCjcA1Zla6O+46YJu7TwRuB26LntsIXA00ATOA70evJynmHqZSePDBMHfO+PFhdM5rr4XhmK+8Al/5imbOTINTT4Vly+DJJ+GCC8KQzokT4aMfhe99L3TNpbLf/957oaEBBgwI1/feG3dFiVHOVFdTgU53fxHAzO4DZgOri9rMBm6Obj8IfM/MLFp+n7vvBl4ys87o9TT3X0IdOBB+5r/1FmzdGsbXFy6vvgqrVoUgKBzdOWhQOHHH9deHUSED1GGYOmbhNIvnnx/+YN91Vxjp86tfhccHDYIzzgi/4k46CY4//uDluOPCr7mhQ0O7RLj3XpgzJ3yIIWyhzJkTbn/yk/HVlRDlhP4YYEPR/Y3A2T21cfd9ZrYdGBUt/33Jc8f0udpDWL06HHGYF+49Lyt+zD0EufvB2wcOwL594bJ/f7jevTt8Rw7VrztoEEyaBH/5l/CBD4TLGWfAkCGVXTeJz5gx8PWvw803hx28S5eGy7PPhl92W7f2/Ny6uhD+Rx4ZPit1deEycGC4NguXAQMO3i78Giz+VVj6C/GwfzE+1wR7nnz3sreAzw2G7x7ma9XYxRfDrbdW9z3KCf3u/slLI6enNuU8FzObA8wBGD9+fBklvVddXRgpkifdfRm6+xIVvmTF14UvZOEyeDAcdVT40g4dGm6PGAEnnHBwq27kSHXZ5IUZjBsXLsUjr955592//rZsgV27wgZD4frttw9uVBRfChsexRsi8N6NlGLdbdz0as+mHpYDo5v78IK1U4t9YOWE/kZgXNH9sUDpv2qhzUYzqwOGA1vLfC7ufhdwF0Bra2tf/puZPDkcZi4i1XPEEaGLvKEh7koOoeELoUun1EknwfyXa15O0pTTA7sUmGRmE8xsMGHH7LySNvOAa6PbVwCL3N2j5VdHo3smAJOAZytTuohIN2699b1zfQwdWv1+k5TodUs/6qO/HlgIDAR+5O4dZnYL0Obu84C7gZ9GO2q3Ev4wELV7gLDTdx/wRXfXeXxEpHoKO2u/9jVYvz4ML7v1Vu3EjZj3qdOselpbW72trS3uMkREUsXMlrl7a2/tNMBORCRHFPoiIjmi0BcRyRGFvohIjij0RURyRKEvIpIjCn0RkRxR6IuI5EjiDs4ysy6gm4kzyjYaeK1C5cRF65AMWodk0DqU5yR3r++tUeJCv7/MrK2co9KSTOuQDFqHZNA6VJa6d0REckShLyKSI1kM/bviLqACtA7JoHVIBq1DBWWuT19ERHqWxS19ERHpQWZC38xmmNlaM+s0s7lx11MOM/uRmW0xs1VFy441s8fM7I/R9cg4a+yNmY0zsyfNbI2ZdZjZ30fLU7MeZnaEmT1rZs9F6/D1aPkEM1sSrcP90ZnjEs3MBprZCjN7JLqfqnUws5fNbKWZtZtZW7QsNZ+lAjMbYWYPmtkL0Xfj3KSsRyZC38wGAncCM4FG4Boza4y3qrL8GJhRsmwu8IS7TwKeiO4n2T7gy+5+GnAO8MXo3z5N67EbuMDdzwSagRlmdg5wG3B7tA7bgOtirLFcfw+sKbqfxnX4C3dvLhrimKbPUsF3gUfd/VTgTML/STLWw91TfwHOBRYW3b8RuDHuusqsvQFYVXR/LXBCdPsEYG3cNR7m+vxf4OK0rgcwFFgOnE04mKYuWv6uz1gSL8BYQphcADwCWArX4WVgdMmyVH2WgGOAl4j2mSZtPTKxpQ+MATYU3d8YLUuj/+HurwJE18fFXE/ZzKwBmAIsIWXrEXWLtANbgMeAdcAb7r4vapKGz9QdwA3Agej+KNK3Dg782syWmdmcaFmqPkvAyUAX8J9RV9t/mNlRJGQ9shL61s0yDUuqITM7GvgF8A/uviPueg6Xu+9392bC1vJU4LTumtW2qvKZ2UeBLe6+rHhxN00Tuw6Rae7eQuiq/aKZfSjugvqgDmgB/o+7TwF2kaAuqayE/kZgXNH9scCmmGrprz+b2QkA0fWWmOvplZkNIgT+ve7+y2hx6tYDwN3fAP4fYf/ECDOrix5K+mdqGnCpmb0M3Efo4rmDdK0D7r4put4CPET4A5y2z9JGYKO7L4nuP0j4I5CI9chK6C8FJkUjFQYDVwPzYq6pr+YB10a3ryX0kSeWmRlwN7DG3b9T9FBq1sPM6s1sRHT7SOAiwo63J4EromaJXgd3v9Hdx7p7A+Hzv8jdP0mK1sHMjjKzYYXbwCXAKlL0WQJw983ABjN7X7ToQmA1SVmPuHd6VHDnySzgD4S+2K/FXU+ZNf8MeBXYS9g6uI7QD/sE8Mfo+ti46+xlHaYTugyeB9qjy6w0rQdwBrAiWodVwE3R8pOBZ4FO4OfAkLhrLXN9zgceSds6RLU+F106Ct/jNH2WitalGWiLPlMPAyOTsh46IldEJEey0r0jIiJlUOiLiOSIQl9EJEcU+iIiOaLQFxHJEYW+iEiOKPRFRHJEoS8ikiP/H3TvOdOKCiKXAAAAAElFTkSuQmCC\n", "text/plain": [ "
" ] }, "metadata": { "needs_background": "light" }, "output_type": "display_data" }, { "name": "stdout", "output_type": "stream", "text": [ "3.054140051972354e-09\n" ] }, { "data": { "image/png": "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\n", "text/plain": [ "
" ] }, "metadata": { "needs_background": "light" }, "output_type": "display_data" }, { "name": "stdout", "output_type": "stream", "text": [ "1.771111590720409e-09\n" ] }, { "data": { "image/png": "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\n", "text/plain": [ "
" ] }, "metadata": { "needs_background": "light" }, "output_type": "display_data" }, { "name": "stdout", "output_type": "stream", "text": [ "1.0246228310251126e-09\n" ] }, { "data": { "image/png": "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\n", "text/plain": [ "
" ] }, "metadata": { "needs_background": "light" }, "output_type": "display_data" }, { "name": "stdout", "output_type": "stream", "text": [ "3.450712493524577e-09\n" ] }, { "data": { "image/png": "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\n", "text/plain": [ "
" ] }, "metadata": { "needs_background": "light" }, "output_type": "display_data" }, { "name": "stdout", "output_type": "stream", "text": [ "1.072898014875534e-08\n" ] }, { "data": { "image/png": "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\n", "text/plain": [ "
" ] }, "metadata": { "needs_background": "light" }, "output_type": "display_data" }, { "name": "stdout", "output_type": "stream", "text": [ "6.375016403659117e-09\n" ] }, { "data": { "image/png": "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\n", "text/plain": [ "
" ] }, "metadata": { "needs_background": "light" }, "output_type": "display_data" }, { "name": "stdout", "output_type": "stream", "text": [ "3.777915091873751e-09\n" ] }, { "data": { "image/png": "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\n", "text/plain": [ "
" ] }, "metadata": { "needs_background": "light" }, "output_type": "display_data" }, { "name": "stdout", "output_type": "stream", "text": [ "2.2330873816756025e-09\n" ] }, { "data": { "image/png": "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\n", "text/plain": [ "
" ] }, "metadata": { "needs_background": "light" }, "output_type": "display_data" }, { "name": "stdout", "output_type": "stream", "text": [ "1.316664333117501e-09\n" ] }, { "data": { "image/png": "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\n", "text/plain": [ "
" ] }, "metadata": { "needs_background": "light" }, "output_type": "display_data" }, { "name": "stdout", "output_type": "stream", "text": [ "7.744472840798267e-10\n" ] }, { "data": { "image/png": "iVBORw0KGgoAAAANSUhEUgAAAX0AAAD8CAYAAACb4nSYAAAABHNCSVQICAgIfAhkiAAAAAlwSFlzAAALEgAACxIB0t1+/AAAADl0RVh0U29mdHdhcmUAbWF0cGxvdGxpYiB2ZXJzaW9uIDMuMC4zLCBodHRwOi8vbWF0cGxvdGxpYi5vcmcvnQurowAAHxlJREFUeJzt3XuQVOWd//H3lxlmkIugiEq4OESugxcg42VNDF7Wddj9ReL+1GhupmKtSSVuaSW/GP2l4nrLbplNrdmqNal1YxIrcReIubGGYGIkY5JNjMiMCgMDIyIgKqOgKMr9+/vjOedH284wDdPd55w+n1dVV3effqb7O7dPP/2c5zzH3B0REcmHQUkXICIi1aPQFxHJEYW+iEiOKPRFRHJEoS8ikiMKfRGRHFHoi4jkiEJfRCRHFPoiIjlSn3QBxY477jhvampKugwRkUx58sknX3H3Mf21S13oNzU1sXz58qTLEBHJFDN7vpR2Gt4REckRhb6ISI4o9EVEckShLyKSIwp9EZEcUeiLiOSIQl9EJEcU+iIlcIcf/hA2bky6EpGBUeiLlODuu+ETn4CLL4YdO5KuRuTIKfRF+vHII/ClL8E558C6dSH8DxxIuiqRI6PQFzmE556Dj3wEZsyApUtDj3/xYrjjjqQrEzkyCn2RPuzcCR/+cOjV/+xnMGIEXHcdfOpTcOut8POfJ12hyOFT6Iv0wh0+/WlYuRIWLIDJk8N2M/j2t+GMM+DjH4fVq5OtU+RwKfRFevHII7BoEdx5Z9h5W2jIEPjJT+Coo+CGG5KpT+RIKfRFevHQQyHc+wr18ePhk5+E3/42DAOJZIVCX6QXS5fCeeeF3nxfWlthzx5YtqxqZYkMmEJfpMj69bB2Lcybd+h2554LQ4eGNwiRrFDoixSJQ7y19dDtGhvhggsU+pItCn2RIkuXwnvfC1Om9N+2tRWefTYctCWSBQp9kQK7d8Ojj4YwN+u/fTwEpN6+ZIVCX6TA738fZuP0N7QTiz8RKPQlKxT6IgWWLoWGBjj//NK/prU1zODZtatydYmUi0JfpMAvfxlm5QwfXvrXzJsHb78Njz1WubpEykWhLxLZtAlWrSp9aCc2d26YyaMhHskChb5IJA7t/ubnFxs6NAT/L39Z/ppEyk2hLxJZujQsr9DcfPhf29oKa9bAhg1lL0ukrBT6IsDevWGRtXnzSpuqWUxTNyUrFPoiwB//GE6DeLjj+bFp0+CkkxT6kn4KfRHClMtBg+DCC4/s683CEszLlulUipJuCn0RYMWK0FsfOfLIn+OMM8KnhfXry1eXSLkp9EWAjg6YNWtgzxF/fUfHwOsRqRSFvuTetm2wcSPMnj2w5znlFKirU+hLuin0JffikB5oT3/IEJgxA9rbB16TSKWUFPpm1mpmXWbWbWY39fJ4o5ktjB5/3Myaou2Dzex+M3vGzFab2c3lLV9k4MoV+hA+LainL2nWb+ibWR1wDzAPaAauMrPiw1euAba7+2TgbuCuaPvlQKO7nwq8D/hM/IYgkhbt7TBuHIwZM/DnmjULtmyBrVsH/lwilVBKT/9MoNvd17v7HmABML+ozXzg/uj2g8CFZmaAA8PMrB44CtgD7ChL5SJlUo6duDHtzJW0KyX0xwGbCu5vjrb12sbd9wGvA6MJbwA7gReBjcA33H3bAGsWKZtdu2D16oHvxI0p9CXtSgn93g5K9xLbnAnsB94DTAK+aGbvfdcLmF1rZsvNbHlPT08JJYmUx8qVsH9/+Xr6xx4LEycq9CW9Sgn9zcCEgvvjgS19tYmGckYC24CPAkvdfa+7bwX+ALQUv4C73+vuLe7eMqYcA6siJSrnTtzYrFmawSPpVUroPwFMMbNJZtYAXAksLmqzGLg6un0Z8Ki7O2FI5wILhgFnA2vKU7rIwHV0wNFHw6RJ5XvO2bOhqyucdlEkbfoN/WiM/jrgYWA1sMjdV5nZ7WZ2SdTsPmC0mXUDXwDiaZ33AMOBlYQ3j++5+9Nl/h5Ejlh7O5x+elh3p1xmzQL3MHQkkjb1pTRy9yXAkqJttxTc3kWYnln8dW/2tl0kDQ4cgKeegmuuKe/zxjuF29vhrLPK+9wiA6UjciW3urvDEEw5x/Mh7MgdNUo7cyWdFPqSW5XYiQthmeVZsxT6kk4Kfcmtjg4YPBhmziz/c8+eDU8/HaaDiqSJQl9yq709nA+3oaH8zz1rFrz9NqxdW/7nFhkIhb7kVjmXXyimI3MlrRT6kksvvRQu5Vp+odiMGeEThA7SkrRR6EsuVWonbmzw4HBSFfX0JW0U+pJLcRiffnrlXiOewePFK1WJJEihL7nU0QFNTWE+faXMng09PWF9fZG0UOhLLq1aBaeeWtnXOOWUcN3ZWdnXETkcCn3JnX37wlTKGTMq+zrx869eXdnXETkcCn3JnQ0bYM8emD69sq9z/PFh+GiN1pWVFFHoS+7EPe9K9/TNwmuopy9potCX3IlDuNI9fVDoS/oo9CV31qyBE0+s7Myd2PTp8PLLsH175V9LpBQKfcmd1asrP7QTi19H4/qSFgp9yRX3EPrVGNoBzeCR9FHoS668/DK8/nr1evpNTdDYqJ6+pIdCX3KlWjN3YnV1MHWqevqSHgp9yZVqztyJaQaPpIlCX3JlzRoYPhzGjavea06fDs89B7t2Ve81Rfqi0JdciXfimlXvNWfMgAMHYN266r2mSF8U+pIr1ZyuGdMMHkkThb7kxhtvwAsvVD/0p04Nnyw0g0fSQKEvuRGHbjV34gIcdVSYuqmevqSBQl9yo9rTNQtpBo+khUJfcmPNGqivh5NPrv5rT58OXV1hh65IkhT6khurV8PkyeGk5dU2Y0aYsvn889V/bZFCCn3JjSRm7sQ0g0fSQqEvubB3Lzz7bPV34sbi19UMHkmaQl9yobs7nBs3qZ7+6NEwZox6+pI8hb7kQpIzd2KawSNpoNCXXIiHVaZNS66G6dM1vCPJKyn0zazVzLrMrNvMburl8UYzWxg9/riZNRU8dpqZ/dHMVpnZM2Y2pHzli5Rm9WoYPx5GjEiuhhkz4NVXoacnuRpE+g19M6sD7gHmAc3AVWbWXNTsGmC7u08G7gbuir62Hvgh8Fl3nwmcB+wtW/UiJUpy5k4s3pmrIR5JUik9/TOBbndf7+57gAXA/KI284H7o9sPAheamQF/BTzt7k8BuPur7r6/PKWLlMY9HBiV1MydmM6XK2lQSuiPAzYV3N8cbeu1jbvvA14HRgNTATezh81shZnd2NsLmNm1ZrbczJb36LOvlNmWLfDmm8mH/oQJYR2erq5k65B8KyX0e1t53EtsUw98APhYdH2pmV34robu97p7i7u3jBkzpoSSREqXhp24AIMGhRU31dOXJJUS+puBCQX3xwNb+moTjeOPBLZF29vc/RV3fwtYAswZaNEihyPuWScd+nEN6ulLkkoJ/SeAKWY2ycwagCuBxUVtFgNXR7cvAx51dwceBk4zs6HRm8FcoLM8pYuUpqsLhg2r7ikS+zJtWjh14u7dSVciedVv6Edj9NcRAnw1sMjdV5nZ7WZ2SdTsPmC0mXUDXwBuir52O/AvhDeODmCFu/+i/N+GSN/WrAlhW81TJPZl+vSw0mZ3d9KVSF7Vl9LI3ZcQhmYKt91ScHsXcHkfX/tDwrRNkUR0dcE55yRdRRAPMXV1wcyZydYi+aQjcqWmvf02bNyYjvF8CDtyQeP6khyFvtS0devCPP2kp2vGRoyA97xHoS/JUehLTUvLdM1CWoNHkqTQl5oW96inTEm2jkLxtE0vPtpFpAoU+lLTurpg4sQwZTMtpk2D117TwmuSDIW+1LSurnQN7cA7Z/CIVJtCX2qW+8E5+mmiUydKkhT6UrNefDEdC60VmzgRhgxRT1+SodCXmpWmNXcKDRoUdiwr9CUJCn2pWWmcrhnTtE1JikJfalZXFwwdmo6F1orFC6/t2ZN0JZI3Cn2pWfHMnUEp/CufNg3274dnn026EsmbFP47iJRHGqdrxjRtU5Ki0Jea9PbbsGFD+kNf4/pSbQp9qUnd3elaaK3Y0UfD2LHq6Uv1KfSlJqV1umYhnTpRkqDQl5oUh2m8fn0aTZsWhne08JpUk0JfatKaNTBhQroWWis2fTps3w6vvJJ0JZInCn2pSWmeuRPTDB5JgkJfao67Ql+kLwp9qTkvvQQ7dqR35k7spJOgsVHTNqW6FPpSc9K85k6hurqwo1mhL9Wk0Jeas2pVuJ45M9k6StHcfLBekWpQ6EvN6eyEUaPCwU9pN3NmOHJ4586kK5G8UOhLzensDD1os6Qr6V9z88EdzyLVoNCXmhOHfhbEdXZ2JluH5IdCX2pKT0+4ZCX0J0+G+nqFvlSPQl9qShyeWQn9wYPDDB7tzJVqUehLTcla6EPYmauevlSLQl9qSmcnjBgB48cnXUnpmpth/fpwDgCRSlPoS03J0sydWHMzHDgAa9cmXYnkgUJfakqWZu7ENINHqqmk0DezVjPrMrNuM7upl8cbzWxh9PjjZtZU9PhEM3vTzP5PecoWebdt28K6O1kL/SlTwpIM2pkr1dBv6JtZHXAPMA9oBq4ys+J/q2uA7e4+GbgbuKvo8buBXw68XJG+ZXEnLoRF16ZMUU9fqqOUnv6ZQLe7r3f3PcACYH5Rm/nA/dHtB4ELzcKoqpl9GFgPqB8jFZXV0IdQs0JfqqGU0B8HbCq4vzna1msbd98HvA6MNrNhwJeB2wZeqsihdXbC0KEwcWLSlRy+5uZwMvfdu5OuRGpdKaHf2zyI4rN69tXmNuBud3/zkC9gdq2ZLTez5T09PSWUJPJu8U7cQRmcntDcDPv3w7p1SVcita6Uf4/NwISC++OBLX21MbN6YCSwDTgL+LqZbQBuAP6vmV1X/ALufq+7t7h7y5gxYw77mxCBsCM0i0M7cLBu7cyVSqsvoc0TwBQzmwS8AFwJfLSozWLgauCPwGXAo+7uwLlxAzO7FXjT3f+tDHWLvMNrr8GWLdkN/WnTwicUjetLpfUb+u6+L+qdPwzUAd9191Vmdjuw3N0XA/cBPzCzbkIP/8pKFi1SbPXqcJ3V0B8yBE4+WaEvlVdKTx93XwIsKdp2S8HtXcDl/TzHrUdQn0hJsjxzJ6YZPFINGdzlJfJunZ2ht9zUlHQlR665OSzFsHdv0pVILVPoS01YtQqmTw9HtmZVczPs26cZPFJZCn2pCZ2d2TgR+qFoDR6pBoW+ZN6OHbBpU7bH8yF8UjFT6EtlKfQl89asCddZD/2hQ2HSJIW+VJZCXzIvPqAp66EP4XtYuTLpKqSWKfQl89rbQy/55JOTrmTgTj89fHLRWbSkUhT6knnt7TBrVrZn7sTmzAlr8Ki3L5Wi0JdMO3AAOjpg9uykKymP+PtYsSLZOqR2KfQl07q74c03Qw+5FjQ1wahR4dOLSCUo9CXT4h5xrfT0zcL3op6+VIpCXzKtvR0GD87+gVmF5syBp5/WcgxSGQp9ybQVK+CUU6ChIelKymf27HAGrfj4A5FyUuhLZrmHnn6tjOfH4u9H4/pSCQp9yaxNm+DVV2sv9KdODccdaFxfKkGhL5kV94RrZSdurK4uHKSlnr5UgkJfMmvFinCKwdNOS7qS8ps9O4T+gQNJVyK1RqEvmdXeHs4tO2xY0pWU35w58MYbsH590pVIrVHoS2atWFF74/kxHZkrlaLQl0zq6YEXXqi98fzYzJlQX69xfSk/hb5kUhyGtdrTb2wMxx8o9KXcFPqSSfGwx6xZydZRSfFyDO5JVyK1RKEvmdTeHs4ydcwxSVdSOXPmhGGsLVuSrkRqiUJfMmnFitodz49pZ65UgkJfMmfHjrCkcq2O58dOPz2suqlxfSknhb5kTkdHuK71nv7w4WFJBvX0pZwU+pI5tT5zp9CcOerpS3kp9CVznngCxo6FE09MupLKmzMHNm6ErVuTrkRqhUJfMsUdHnsMPvCBpCupjvj7fOyxZOuQ2qHQl0zZsCEsqTx3btKVVMf73hfWFmprS7oSqRUKfcmUOPw++MFk66iWwYPhnHMU+lI+Cn3JlMceg2OPra1z4vZn7lx45plwwhiRgVLoS6a0tYVe/qAc/eXGQ1m/+12ydUhtKOlfx8xazazLzLrN7KZeHm80s4XR44+bWVO0/SIze9LMnomuLyhv+ZInmzeH9eXzMp4fO+MMGDJEQzxSHv2GvpnVAfcA84Bm4Cozay5qdg2w3d0nA3cDd0XbXwE+5O6nAlcDPyhX4ZI/cejlLfQbG+HssxX6Uh6l9PTPBLrdfb277wEWAPOL2swH7o9uPwhcaGbm7u3uHi8XtQoYYmaN5Shc8qetDUaOrM3TI/Zn7txwJPJrryVdiWRdKaE/DthUcH9ztK3XNu6+D3gdGF3U5n8D7e6+u/gFzOxaM1tuZst7enpKrV1ypq0Nzj03nDg8b+bODcco/P73SVciWVdK6Fsv24pX+D5kGzObSRjy+UxvL+Du97p7i7u3jBkzpoSSJG9efBHWrs3f0E7s7LOhoUFDPDJwpYT+ZmBCwf3xQPEK3/+/jZnVAyOBbdH98cBPgU+6+7MDLVjyKT4iNa+hf9RRcOaZCn0ZuFJC/wlgiplNMrMG4EpgcVGbxYQdtQCXAY+6u5vZKOAXwM3u/odyFS3509YGI0bU/sqahzJ3blhx8403kq5Esqzf0I/G6K8DHgZWA4vcfZWZ3W5ml0TN7gNGm1k38AUgntZ5HTAZ+KqZdUSX48v+XUjNa2uD978/nCw8r+bOhf374Q/qPskAlPQv5O5LgCVF224puL0LuLyXr7sTuHOANUrO9fRAZyd84hNJV5Ksc84Jb3ptbdDamnQ1klU5Oq5Rsioez8/Lejt9GTYMWlo0ri8Do9CX1GtrCzsyW1qSriR5c+eG8wns3Jl0JZJVCn1JvWXLwtBGQ0PSlSRv7lzYt0/z9eXIKfQl1bq6YOVK+NCHkq4kHc47Lwzz/OQnSVciWaXQl1T70Y/C9WWXJVtHWhx1FFxyCfz4x6HHL3K4FPqSagsXhlMGjite+CPHrrgirK2/bFnSlUgWKfQltTo7w9DOFVckXUm6tLbC8OGwaFHSlUgWKfQltX70IzDT0E6xIUNg/vwwrr93b9LVSNYo9CW1Fi0Kc/PHjk26kvS54grYtg1+85ukK5GsUehLKq1aFYZ3NLTTu4svhqOP1hCPHD6FvqTSwoXhPLh/+7dJV5JOjY3w4Q/DT38Ke/YkXY1kiUJfUsc99GDnzoUTT0y6mvS64opwJq1HHkm6EskShb6kzjPPhIOyPvKRpCtJt4suCqeP1BCPHA6FvqTOokUa2ilFQwNcein87Gew+10nIRXpnUJfUiUe2rngAtCZM/t3xRXw+uvwq18lXYlkhUJfUuXXv4Z16+Cqq5KuJBsuvDC8Od5zT9KVSFYo9CU13OG222D8ePjYx5KuJhsaGuCLX4SHH4bHH0+6GskChb6kxqOPwv/8D9x8c5iSKKX5/Odh9Gi4/fakK5EsUOhLKsS9/Pe8Bz796aSryZbhw+ELX4AlS8IJVkQORaEvqdDWBr/7Hdx0U1hbRg7PddfBMcfAHXckXYmknUJfUuG228IaO3/3d0lXkk1HHx16+//937BiRdLVSJop9CVxjz0Gv/0t3HijevkD8fd/D6NGaWxfDk2hL4m77TY44QT4zGeSriTbRo6EG26An/8cOjqSrkbSSqEviVq6NMzaufHGcCpAGZjrrw9DPV/+Mhw4kHQ1kkYKfUnMxo3w8Y/DzJnw2c8mXU1tGDUK/vEfwxG6//RPSVcjaaTQl0Ts3h3OiLVnTzgD1NChSVdUOz73uXBE81e/quUZ5N0U+pKI668Pc8rvvx+mTk26mtpiBv/xH+ET1Ec/Cs8/n3RFkiYKfam6+++Hf//3MI5/6aVJV1Obhg2DH/84nEP38su1CqccpNCXqvrzn8P4/fnnw9e+lnQ1tW3qVPj+98Mnqs99Tjt2JVDoS1W4w3e+E050fvzxsGAB1NcnXVXtu/RS+MpX4LvfhXnzoKcn6YokaQp9qbidO+FTnwpH2557buh5Hn980lXlxx13hOG0tjaYPTssaif5pdCXinrmGTjrLPjBD+DWW8O8fAV+dZnBtdeGsG9sDOce/sY3wni/5E9JoW9mrWbWZWbdZnZTL483mtnC6PHHzayp4LGbo+1dZnZx+UqXtNq/P6wBc/HFcNpp8PLLYb33f/gHqKtLurr8mjMnrMvzoQ/Bl74ETU3hU8BLLyVdmVRTv6FvZnXAPcA8oBm4ysyai5pdA2x398nA3cBd0dc2A1cCM4FW4FvR80mN2bkzrKFz550wZQpccgmsWhVCpbMznMRbkjdyZJjV89BDcOqpcMstMHFiOGnNwoWwYUPY/5IJDzwQ3rkGDQrXDzyQdEWZUMqutDOBbndfD2BmC4D5QGdBm/nArdHtB4F/MzOLti9w993Ac2bWHT3fH8tTvlSDO7z1Frz5JmzfDi+8EC6bN4eQeOKJMIyzf39o/8EPwte/DvPnw+DBiZYuvTCDv/mbcFm7Fr71Lfje9+A//zM8fsIJcPbZ0NwczmI2bly4nHBCWLt/+PAU/F4feCCMWb31Vrj//PPhPui0a/0oJfTHAZsK7m8GzuqrjbvvM7PXgdHR9j8Vfe24I672EDo7w85COai4x+be++XAgYOXffvCWO+ePeF6167Qi++r93fssWHY4OabQ1CcdRYcd1zlvzcpj6lT4ZvfhH/+5/DG/ac/Hbz84hfh76E3jY3hKOqGhvAGEF/q6kLHO76YvfMCB69jxfdL8tRM2LPsndveAj7dAP96BM+XEhddVPmpzKWEfm+/kuII6KtNKV+LmV0LXAswceLEEkp6t/p6hU1vevsHK7wU/4PW17/zH7mxEUaMCJfhw8NiXnHPb9w4LZ9QKwYPDm/ec+aEOf0QPrlt3Xrwk11PT/i0V3jZu/fgZc+ed3Yi9u9/Z+cCeu+IHJE9W/rYDhw36wifNHkjRlT+NUoJ/c3AhIL744Hin3jcZrOZ1QMjgW0lfi3ufi9wL0BLS8sR/RlMnRpOFyci5VFXF05sM3YstLQkXU2Rps/1vr7ESSfBkg1VLydLSpm98wQwxcwmmVkDYcfs4qI2i4Gro9uXAY+6u0fbr4xm90wCpgB/Lk/pIpJbX/vauz9mDh2qw7xL0G9PPxqjvw54GKgDvuvuq8zsdmC5uy8G7gN+EO2o3UZ4YyBqt4iw03cf8Hl331+h70VE8iLeWfuVr4Q1uidODIGvnbj9Mk/Z/KyWlhZfvnx50mWIiGSKmT3p7v0OxOmIXBGRHFHoi4jkiEJfRCRHFPoiIjmi0BcRyRGFvohIjij0RURyRKEvIpIjqTs4y8x6gF4W1SjZccArZSqnklRneWWlTshOraqzvCpd50nuPqa/RqkL/YEys+WlHJWWNNVZXlmpE7JTq+osr7TUqeEdEZEcUeiLiORILYb+vUkXUCLVWV5ZqROyU6vqLK9U1FlzY/oiItK3Wuzpi4hIH2om9M2s1cy6zKzbzG5Kup5CZvZdM9tqZisLth1rZr82s3XR9TFJ1hjVNMHMlpnZajNbZWbXp7FWMxtiZn82s6eiOm+Ltk8ys8ejOhdGZ3pLnJnVmVm7mT0U3U9dnWa2wcyeMbMOM1sebUvV7z1mZqPM7EEzWxP9rf5F2mo1s2nRzzK+7DCzG9JQZ02EvpnVAfcA84Bm4Coza062qnf4PtBatO0m4DfuPgX4TXQ/afuAL7r7DOBs4PPRzzFtte4GLnD304FZQKuZnQ3cBdwd1bkduCbBGgtdD6wuuJ/WOs9391kF0wrT9nuP/Suw1N2nA6cTfrapqtXdu6Kf5SzgfcBbwE9JQ53unvkL8BfAwwX3bwZuTrquohqbgJUF97uAsdHtsUBX0jX2UvPPgYvSXCswFFgBnEU48KW+t7+JBOsbT/jnvgB4CLCU1rkBOK5oW+p+78DRwHNE+yPTXGtBbX8F/CEtddZETx8YB2wquL852pZmJ7j7iwDR9fEJ1/MOZtYEzAYeJ4W1RkMmHcBW4NfAs8Br7r4vapKWv4FvAjcCB6L7o0lnnQ78ysyeNLNro22p+70D7wV6gO9FQ2bfMbNhpLPW2JXAf0W3E6+zVkLfetmmaUlHyMyGAz8GbnD3HUnX0xt33+/ho/N44ExgRm/NqlvVO5nZ/wK2uvuThZt7aZqGv9X3u/scwhDp583sg0kX1Id6YA7wbXefDewkPcNO7xLtr7kE+FHStcRqJfQ3AxMK7o8HtiRUS6leNrOxANH11oTrAcDMBhMC/wF3/0m0OZW1Arj7a8BvCfsgRplZffRQGv4G3g9cYmYbgAWEIZ5vkr46cfct0fVWwtjzmaTz974Z2Ozuj0f3HyS8CaSxVghvoivc/eXofuJ11kroPwFMiWZFNBA+Ti1OuKb+LAaujm5fTRg/T5SZGXAfsNrd/6XgoVTVamZjzGxUdPso4C8JO/OWAZdFzRKv091vdvfx7t5E+Jt81N0/RsrqNLNhZjYivk0Yg15Jyn7vAO7+ErDJzKZFmy4EOklhrZGrODi0A2moM+mdHGXcWfLXwFrC2O5Xkq6nqLb/Al4E9hJ6KtcQxnZ/A6yLro9NQZ0fIAw1PA10RJe/TlutwGlAe1TnSuCWaPt7gT8D3YSP041J/0wLaj4PeCiNdUb1PBVdVsX/P2n7vRfUOwtYHv3+fwYck8ZaCZMMXgVGFmxLvE4dkSsikiO1MrwjIiIlUOiLiOSIQl9EJEcU+iIiOaLQFxHJEYW+iEiOKPRFRHJEoS8ikiP/D0RCrzqe7e7oAAAAAElFTkSuQmCC\n", "text/plain": [ "
" ] }, "metadata": { "needs_background": "light" }, "output_type": "display_data" }, { "name": "stdout", "output_type": "stream", "text": [ "4.5445025520507443e-10\n" ] }, { "data": { "image/png": "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\n", "text/plain": [ "
" ] }, "metadata": { "needs_background": "light" }, "output_type": "display_data" }, { "name": "stdout", "output_type": "stream", "text": [ "1.427774343198962e-09\n" ] }, { "data": { "image/png": "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\n", "text/plain": [ "
" ] }, "metadata": { "needs_background": "light" }, "output_type": "display_data" }, { "name": "stdout", "output_type": "stream", "text": [ "8.469198675697953e-10\n" ] }, { "data": { "image/png": "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\n", "text/plain": [ "
" ] }, "metadata": { "needs_background": "light" }, "output_type": "display_data" }, { "name": "stdout", "output_type": "stream", "text": [ "2.522461794995934e-09\n" ] }, { "data": { "image/png": "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\n", "text/plain": [ "
" ] }, "metadata": { "needs_background": "light" }, "output_type": "display_data" }, { "name": "stdout", "output_type": "stream", "text": [ "1.5118173379846667e-09\n" ] }, { "data": { "image/png": "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\n", "text/plain": [ "
" ] }, "metadata": { "needs_background": "light" }, "output_type": "display_data" }, { "name": "stdout", "output_type": "stream", "text": [ "4.291089927832559e-09\n" ] }, { "data": { "image/png": "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\n", "text/plain": [ "
" ] }, "metadata": { "needs_background": "light" }, "output_type": "display_data" }, { "name": "stdout", "output_type": "stream", "text": [ "2.597470794007961e-09\n" ] }, { "data": { "image/png": "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\n", "text/plain": [ "
" ] }, "metadata": { "needs_background": "light" }, "output_type": "display_data" }, { "name": "stdout", "output_type": "stream", "text": [ "1.568271734697646e-09\n" ] }, { "data": { "image/png": "iVBORw0KGgoAAAANSUhEUgAAAX0AAAD8CAYAAACb4nSYAAAABHNCSVQICAgIfAhkiAAAAAlwSFlzAAALEgAACxIB0t1+/AAAADl0RVh0U29mdHdhcmUAbWF0cGxvdGxpYiB2ZXJzaW9uIDMuMC4zLCBodHRwOi8vbWF0cGxvdGxpYi5vcmcvnQurowAAHxhJREFUeJzt3X2QXHWd7/H3NxOSkAcek5hAHiaQycSQQIAhgKtcAWGSLS/oCrXBJ1yzhZaiq+LuhUtdVlndW9zaJW6VXLcoo7LiIoqsRsEMGnzYcpUwgQQmD5MJIU8kgfDghBBCMsn3/vE756ZtZpieme7+ne7zeVV1dffp093fmen59O/8zu/8jrk7IiKSD8NiFyAiItWj0BcRyRGFvohIjij0RURyRKEvIpIjCn0RkRxR6IuI5IhCX0QkRxT6IiI5Mjx2AcXGjx/vjY2NscsQEakpq1evftHdJ/S3XuZCv7Gxkfb29thliIjUFDPbVsp66t4REckRhb6ISI4o9EVEckShLyKSIwp9EZEcUeiLiOSIQl9EJEcU+iIlcId774Xt22NXIjI0Cn2REixdCh/5CLS2wr59sasRGTyFvkg/Vq6Ev/1beMc7oKsLPvpROHo0dlUig6PQF3kLW7fCX/4lzJ4NK1bAnXfCT34CX/lK7MpEBkehL9KHAwfg/e+Hnh748Y9h3Dj4zGdCS//v/x5++tPYFYoMnEJfpA+f/SysXQv33QdNTWGZGfzrv8L558OHP6wdu1J7FPoivThwIIzW+cQnYNGiP33s+OPDF8G+ffDDH8apT2SwFPoivfjtb+GNN+B97+v98aYmmDMH2tqqW5fIUCn0RXqxYgWMGgWXXNL3OgsXwm9+A6+9Vr26RIZKoS/SixUr4N3vDl05fVm4EA4dCsEvUisU+iJFnn0WOjtDqL+Vd70rfCmsWFGdukTKQaEvUiTtp+8v9EeNgksvVehLbVHoixRpa4Pp02HWrP7XbW0NR+lu2VL5ukTKQaEvUuDQoTDtwsKFYUx+f9KtAY3ikVqh0Bcp8Pvfw6uv9t+1k2pqghkz1MUjtUOhL1JgxQoYPhwuu6y09c3CF8Sjj4atBJGsU+iLFGhrC7NpnnBC6c9pbYX9++G//qtydYmUS0mhb2YLzazTzDab2c29PD7SzO5PHn/MzBqT5ceZ2T1m9rSZbTCzW8pbvkj57NkDTz5ZetdO6rLLwtaBunikFvQb+mbWANwFLALmANeZ2Zyi1ZYAr7j7TGApcEey/FpgpLvPA84HPpF+IYhkzSOPhOuBhv64cfDOdyr0pTaU0tJfAGx29y3ufgj4PnB10TpXA/cktx8ALjczAxwYY2bDgeOBQ4DOOySZtHIlTJwI55wz8Oe2toYZOffuLX9dIuVUSuifDuwouL8zWdbrOu7eA3QDpxK+AF4DdgPbgX9y95eHWLNIRTzxBFxwAQwbxJ6uBQvC9Zo15a1JpNxK+Xj3NlrZS1xnAXAEOA2YAdxkZme86Q3MbjCzdjNr36umkkRw8CBs3Di4Vj4ce97ateWrSaQSSgn9ncDUgvtTgF19rZN05ZwIvAx8EFjh7ofd/QXgd0BL8Ru4+93u3uLuLRMmTBj4TyEyROvXhzNkzZ8/uOefeipMmaKWvmRfKaH/ONBkZjPMbASwGFhetM5y4Prk9jXAo+7uhC6dyywYA1wEbCxP6SLlk7bQB9vST5+rlr5kXb+hn/TR3wi0ARuAH7j7OjO73cyuSlZbBpxqZpuBLwDpsM67gLFAB+HL49vu/lSZfwaRIVuzBsaMgTPPHPxrzJ8PGzaEriKRrBpeykru/jDwcNGy2wpuHyQMzyx+3v7elotkzdq1MG8eNDQM/jXOOQeOHAldReedV77aRMpJR+RK7rmHlv5g+/NT6fPVry9ZptCX3Nu+Hbq7h9afD6FraMwY9etLtin0JffSlvlQW/rDhsHZZ6ulL9mm0JfcW7s2zJY5b97QXysdwePFR7KIZIRCX3JvzZowL/6YMUN/rfnzQ1fRtm1Dfy2RSlDoS+6tXTv0/vyUjsyVrFPoS67t2xfObzvU/vzUvHmhq0j9+pJVCn3JtaeSQwXL1dIfMyZ0FamlL1ml0JdcK9fInULz56ulL9ml0JdcW7s2TJZ22mnle81zzoFnnw07dEWyRqEvuZYeiWu9TQ4+SOlWw1OaZUoySKEvudXTAx0d5evPT2kEj2SZQl9ya9OmMCNmOfvzIXQVjR+vfn3JJoW+5FY55tDvjVl4TYW+ZJFCX3Jr3bowlfLs2eV/7Xnzwtz6R4+W/7VFhkKhL7m1cSOccQaMGFH+125uhgMHYOfO8r+2yFAo9CW3OjtDOFdCuvXQ2VmZ1xcZLIW+5NKRI9DVVZmuHTj2ZaLQl6xR6EsubdsGb7xRuZb+pEkwblzoQhLJEoW+5FLaAq9U6JuF11ZLX7JGoS+5VOnQh9B1pNCXrFHoSy51dsLJJ8OECZV7j+Zm2LEDXnutcu8hMlAKfcmljRtDKJdzzp1i6VbEpk2Vew+RgVLoSy5VcrhmSsM2JYsU+pI7+/bB7t2VG66ZmjkzbEko9CVLFPqSO2l3S6Vb+scfD9OnK/QlWxT6kjvp2PlKhz6ErQmN1ZcsUehL7nR2wrBhcOaZlX+v5uawZeFe+fcSKYVCX3KnszNMtDZyZOXfq7k5DNl87rnKv5dIKRT6kjvVGLmT0hw8kjUKfcmVo0dDd0u1Qj8dIaR+fckKhb7kyvbt4RSJ1Qr9yZNh7Fi19CU7FPqSK2n4VnqMfkoTr0nWKPQlV6ox0VoxTbwmWaLQl1zZuBFOPBEmTqzeezY3h/n7Dxyo3nuK9KWk0DezhWbWaWabzezmXh4faWb3J48/ZmaNBY+dbWa/N7N1Zva0mY0qX/kiA5OO3KnkRGvF0q2Krq7qvadIX/oNfTNrAO4CFgFzgOvMbE7RakuAV9x9JrAUuCN57nDgXuCT7n4W8G7gcNmqFxmgzs7q9eenNPGaZEkpLf0FwGZ33+Luh4DvA1cXrXM1cE9y+wHgcjMz4ErgKXdfC+DuL7n7kfKULjIwr74aDpKqZn8+QFOTJl6T7Cgl9E8HdhTc35ks63Udd+8BuoFTgVmAm1mbmT1hZn839JJFBqdaE60VO/54mDZNY/UlG4aXsE5vvZ/FM4n0tc5w4J3ABcABYKWZrXb3lX/yZLMbgBsApk2bVkJJIgOX9qnPmlX99541S336kg2ltPR3AlML7k8BdvW1TtKPfyLwcrL8N+7+orsfAB4Gzit+A3e/291b3L1lQiXPXye5lrb0qzHRWrGmJk28JtlQSug/DjSZ2QwzGwEsBpYXrbMcuD65fQ3wqLs70AacbWajky+D/wasL0/pIgPT1QVTp8Lo0dV/76Ym6O6GF1+s/nuLFOq3e8fde8zsRkKANwDfcvd1ZnY70O7uy4FlwHfNbDOhhb84ee4rZnYn4YvDgYfd/aEK/Swib6mrK4RvDGmXUldXZU/GLtKfUvr0cfeHCV0zhctuK7h9ELi2j+feSxi2KRJVVxdc2+untPLSL5uuLnjHO+LUIAI6Ildy4uWXwyVWS7+xERoatDNX4lPoSy6kYRsr9I87DmbMUOhLfAp9yYXYoZ++t0JfYlPoSy5s2hTOi3vGGfFq0LBNyQKFvuRCVxdMn16d8+L2pakpnC93z554NYgo9CUXYg7XTBWO4BGJRaEvdc9doS+SUuhL3du7F/btix/606eHUTwKfYlJoS91LwsjdyCM0z/zTIW+xKXQl7qXldBPa1DoS0wKfal7mzaFVnZjY+xKjoX+0aOxK5G8UuhL3evqCuPzjzsudiUh9A8eDGfwEolBoS91Lwsjd1IawSOxKfSlrrnD5s0KfZGUQl/q2u7d4SjYrIT+lCkwapRCX+JR6Etdy9LIHQjz/2jYpsSk0Je6lrXQB50kXeJS6Etd27QJRoyAadNiV3JMUxM88wwcORK7Eskjhb7Uta6u0J3S0BC7kmOamuDQIdi+PXYlkkcKfalrWRqumdIIHolJoS916+jR0I2i0Bc5RqEvdWvHjnD0a9ZCf/JkGDMm7G8QqTaFvtStzs5w3dwct45iZmEET1qfSDUp9KVuZTX0IdSk0JcYFPpStzo7Ydw4mDQpdiVv1twM27bB66/HrkTyRqEvdauzM4SrWexK3qy5+di8QCLVpNCXurVpUza7duBYXdqZK9Wm0Je6dOBAOPgpq6E/a1a4Vr++VJtCX+pSOgY+q6E/diycfrpCX6pPoS91Kcsjd1IawSMxKPSlLqVhOnNm3DreSjpW3z12JZInCn2pS52dMHVqOPI1q5qb4Y9/hL17Y1cieaLQl7qU5ZE7KY3gkRgU+lJ33I+N0c+ytD7160s1KfSl7jz/POzbl/3Qnz4dRo5U6Et1lRT6ZrbQzDrNbLOZ3dzL4yPN7P7k8cfMrLHo8Wlmtt/MvlieskX6VgsjdyCc2GXmTIW+VFe/oW9mDcBdwCJgDnCdmc0pWm0J8Iq7zwSWAncUPb4U+PnQyxXpX62EPmjYplRfKS39BcBmd9/i7oeA7wNXF61zNXBPcvsB4HKzMOOJmb0P2AKsK0/JIm+tsxNGjQqjd7Ju1qxwopfDh2NXInlRSuifDuwouL8zWdbrOu7eA3QDp5rZGOB/AF9+qzcwsxvMrN3M2vdq/JoMUWdnOHHKsBrYY9XcDD098OyzsSuRvCjl36K3OQqLDyfpa50vA0vdff9bvYG73+3uLe7eMmHChBJKEulbLQzXTGnYplRbKaG/EyjcUJ4C7OprHTMbDpwIvAxcCPwfM9sKfA74n2Z24xBrFunToUOwZUvthb769aVahpewzuNAk5nNAJ4DFgMfLFpnOXA98HvgGuBRd3fgXekKZvYlYL+7f70MdYv0assWOHKkdkL/lFNg/HiFvlRPv6Hv7j1J67wNaAC+5e7rzOx2oN3dlwPLgO+a2WZCC39xJYsW6UstjdxJaQSPVFMpLX3c/WHg4aJltxXcPghc289rfGkQ9YkMSK2G/kMPxa5C8qIGxjeIlK6zE972NjjxxNiVlK65ORxF3N0duxLJA4W+1JXOzmNnpaoVOouWVJNCX+qGO2zcCLNnx65kYN7+9nC9YUPcOiQfFPpSN/bsgZdegnnzYlcyMGeeGSZe6+iIXYnkgUJf6kYamnPnxq1joIYPD619hb5Ug0Jf6kathj6EmhX6Ug0KfakbHR0wcSLU4kwec+fCzp3h9IkilaTQl7rR0VGbrXw4Vvc6zUUrFabQl7pw9GgIzFrbiZtK61YXj1SaQl/qwrZt8NprtdvSnzoVxo1T6EvlKfSlLtTyTlwAM+3MlepQ6EtdePrpcD2n+ESeNWTu3PBzePHZKkTKSKEvdaGjA6ZPhxNOiF3J4M2dGw4ue/752JVIPVPoS12o5ZE7qbR+dfFIJSn0peYdPhzm3FHoi/RPoS81r6srBH+tDtdMTZwYLgp9qSSFvtS8Wh+5U0gjeKTSFPpS8zo6oKGhts6W1Ze5c8NBZkePxq5E6pVCX2re009DUxOMGhW7kqGbOxf27w8Hm4lUgkJfal49jNxJaWeuVJpCX2ragQPwzDP1E/pnnRWuFfpSKQp9qWkbNoQjWOsl9E84AaZNU+hL5Sj0pabV08id1Lx5Cn2pHIW+1LSOjnB+2ZkzY1dSPnPnhoPNDh+OXYnUI4W+1LQ1a0I/eEND7ErK5+yz4dCh0HUlUm4KfalZR4/C44/DggWxKymvCy4I148/HrcOqU8KfalZXV3Q3V1/oT9zJpx0EqxaFbsSqUcKfalZaSjWW+ibhZ9JoS+VoNCXmrVqFYwdC7Nnx66k/BYsCEcaHzgQuxKpNwp9qVmrVkFLS33txE0tWABHjsCTT8auROqNQl9q0htvhJE79da1k0p35qqLR8pNoS816amnwrDGeg39SZPCkbkKfSk3hb7UpHrdiVtIO3OlEhT6UpNWrQqt4SlTYldSOQsWwJYt8OKLsSuReqLQl5q0alXo9zaLXUnlpFsxOkhLyqmk0DezhWbWaWabzezmXh4faWb3J48/ZmaNyfIrzGy1mT2dXF9W3vIlj7q7w9w09dy1A3D++TBsmLp4pLz6DX0zawDuAhYBc4DrzGxO0WpLgFfcfSawFLgjWf4i8N/dfR5wPfDdchUu+dXeHq7rPfTHjoU5cxT6Ul6ltPQXAJvdfYu7HwK+D1xdtM7VwD3J7QeAy83M3P1Jd9+VLF8HjDKzkeUoXPIrDcGWlrh1VEO6M9c9diVSL0oJ/dOBHQX3dybLel3H3XuAbuDUonU+ADzp7m8Uv4GZ3WBm7WbWvnfv3lJrl5xatSqcE/eUU2JXUnkLFoQduVu3xq5E6kUpod/brrLidsdbrmNmZxG6fD7R2xu4+93u3uLuLRMmTCihJMmzVavqv2snlf6c6uKRcikl9HcCUwvuTwF29bWOmQ0HTgReTu5PAf4D+Ki7PzPUgiXfnnsOdu3KT+jPnQujRin0pXxKCf3HgSYzm2FmI4DFwPKidZYTdtQCXAM86u5uZicBDwG3uPvvylW05FceDsoqdNxxcN55Cn0pn35DP+mjvxFoAzYAP3D3dWZ2u5ldlay2DDjVzDYDXwDSYZ03AjOB/2Vma5LLxLL/FJIb//mf4fSI8+fHrqR6LroojNV//fXYlUg9MM/YsICWlhZvT8fkiRQ56yw47TT4xS9iV1I9K1bAokXhurU1djWSVWa22t37HdOmI3KlZuzYAevX5y/4LrkkbN20tcWuROqBQl9qRtq6z1vojx4dgv+RR2JXIvVAoS81o60NJk8OI1ry5sorYd062LkzdiVS6xT6UhOOHAkt/SuvrO9J1vqSbt2otS9DpdCXmtDeDq+8kr+undTcuWErR/36MlQKfakJjzwSWvhXXBG7kjjMwlbOL38ZtnpEBkuhLzWhrS1MNTx+fOxK4mlthZdfhtWrY1citUyhL5nX3Q1/+ENo6ebZFVeEFr+6eGQoFPqSeStXhi6NvPbnp8aPD1MyKPRlKBT6knltbTBuHFx8cexK4mttDVs93d2xK5FapdCXTHMPoX/ZZWHysbxrbQ1bPY8+GrsSqVUKfcm0ri7Ytk39+amLLgqnUVyxInYlUqsU+pJp//7vYefle98bu5JsGDEiTL72ox/BG286B51I/xT6kllHjsC3vw3veQ9Mmxa7muz4+MfhpZdgefFZLURKoNCXzFq5ErZvhyVLYleSLVdcAVOnwrJlsSuRWqTQl8xatiyc/Px974tdSbY0NMDHPhaOUt6+PXY1UmsU+pJJL70EP/4xfPjDYS55+VN/9VdhZNN3vhO7Eqk1Cn3JpHvvhUOH1LXTlxkz4PLLwz6Po0djVyO1RKEvmeMeunZaWuDss2NXk11LlsDWrfCrX8WuRGqJQl8yZ/VqePpptfL78/73w0knaYeuDIxCXzJn2TIYNQoWL45dSbaNGgUf+hA8+GA414BIKRT6kin79oUDsq65JrRi5a0tWRIO0tIOXSmVQl8y5StfgVdfhc99LnYlteHcc+HSS+GrX1VrX0qj0JfM2LwZvva1MAb9/PNjV1M7li4NgX/77bErkVqg0JfM+OIXw5j8f/zH2JXUlnPOgb/+a/j612HjxtjVSNYp9CUTVq6En/wEbr0VJk2KXU3t+Yd/gNGj4aabYlciWafQl+h6ekIf/owZ6ssfrIkT4bbb4OGH4ec/j12NZJlCX6K7+27o6IB//ucwDFEG5zOfgaYm+MIX4PDh2NVIVin0Jaonn4Sbbw4jUDSx2tCMGAF33hn69T//+XBks0gxhb5E88wz4YQgJ50E//Zv4WQpMjTvfW/YIX7XXWH4q0ix4bELkHzasyecArGnB379a5gyJXZF9eOOO+CFF0If/4QJ8MlPxq5IskShL1XX3R1a+Hv2hBN8z54du6L6MmwYfPObYXrqT30qBP8HPhC7KskKde9IVa1eDe96V9hx++CDcOGFsSuqT8cdBz/4QTiR+nXXhWMftHNXQKEvVfL662GH7YUXwosvwk9/Cq2tsauqb6NHw0MPhR3kt94KCxaEHeeSbyWFvpktNLNOM9tsZjf38vhIM7s/efwxM2sseOyWZHmnmenfPGcOHID77oP580Nf88c+BuvXw8KFsSvLh5NPDi3+H/0Idu+GCy4IB3Bt2hS7Moml39A3swbgLmARMAe4zszmFK22BHjF3WcCS4E7kufOARYDZwELgf+bvJ7UsYMHw4k9liwJR9d+8INw5Eg4p+s3v6nZM2P4i78IX7Yf+UiY36i5GS6+GL7xDdi1K3Z1/fje96CxMeysaGwM92XQzPsZzGtmFwNfcvfW5P4tAO7+vwvWaUvW+b2ZDQf2ABOAmwvXLVyvr/draWnx9vb2If1QUjnu4TSGr70Ge/eGnbG7d8Nzz8FTT4Xug/XrQ8iPHQvXXgsf/Shcckn4n5X4du0K01ffc0/YtwLwtreFGTvPPRfOPBMmTw5f2JMmwQknhK6iKH+/730PbrghbDKmRo8OR/R96EMRCsouM1vt7i39rVfK6J3TgR0F93cCxbvf/v867t5jZt3AqcnyPxQ99/QS3nPA1q8PXQfSt/T7vfC6+PbRo8eujxwJAX/4cLgcPAj794flvZk0KYTGVVeFWTJbW8P/p2TLaaeFsfw33QRr18Jvfxu+rJ94An75yzCMtjejR4fLiBFhR3F6GTYsXMx6v8Cbr0u29iw4VHQ+yAPAx0fAvwzwtWrAFVeEabIrqZTQ7+3PVLx50Nc6pTwXM7sBuAFg2rRpJZT0ZsOHw/jxg3pqrvT2T1h4u6Hh2D/xsGF/+s89alRovY8ZEy7jxx9rDU6eDKecEudnksExC/ta5s8/tuyNN8KW2549xy6vvhq27PbvDw3uwoZAT8+xRkJ6SRsQvTUyBuxQH31Ph4Dx83t/rIaNG1f59ygl9HcCUwvuTwGK/xLpOjuT7p0TgZdLfC7ufjdwN4TunVKLLzRrVphsSkQGb+TI0G3e2Bi7kkTjp2Dbtjcvnz4dHt5a9XLqQSm9dI8DTWY2w8xGEHbMLi9aZzlwfXL7GuBRDzsLlgOLk9E9M4AmYFV5SheRuvfVr765j3D06Mr3gdSxflv6SR/9jUAb0AB8y93XmdntQLu7LweWAd81s82EFv7i5LnrzOwHwHqgB/i0u/fRIywiUiTdWXvrrbB9O0ybFgJfO3EHrd/RO9Wm0TsiIgNX6ugdDaITEckRhb6ISI4o9EVEckShLyKSIwp9EZEcUeiLiOSIQl9EJEcU+iIiOZK5g7PMbC/Qy2QbJRsPvFimcspJdQ2M6hoY1TUw9VjXdHef0N9KmQv9oTKz9lKOSqs21TUwqmtgVNfA5Lkude+IiOSIQl9EJEfqMfTvjl1AH1TXwKiugVFdA5PbuuquT19ERPpWjy19ERHpQ92EvpktNLNOM9tsZjdHrONbZvaCmXUULDvFzH5hZl3J9ckR6ppqZr8ysw1mts7M/iYLtZnZKDNbZWZrk7q+nCyfYWaPJXXdn5y1rerMrMHMnjSzn2WlLjPbamZPm9kaM2tPlmXhM3aSmT1gZhuTz9nFsesys+bk95Re9pnZ52LXldT2+eQz32Fm9yX/CxX/fNVF6JtZA3AXsAiYA1xnZnMilfMdYGHRspuBle7eBKxM7ldbD3CTu78duAj4dPI7il3bG8Bl7n4OMB9YaGYXAXcAS5O6XgGWVLmu1N8AGwruZ6WuS919fsHwvth/R4B/AVa4+2zgHMLvLWpd7t6Z/J7mA+cDB4D/iF2XmZ0OfBZocfe5hLMSLqYany93r/kLcDHQVnD/FuCWiPU0Ah0F9zuBycntyUBnBn5nPwGuyFJtwGjgCeBCwgEqw3v7+1axnimEQLgM+BlgGalrKzC+aFnUvyNwAvAsyX7CrNRVVMuVwO+yUBdwOrADOIVw2tqfAa3V+HzVRUufY7/A1M5kWVa8zd13AyTXE2MWY2aNwLnAY2SgtqQLZQ3wAvAL4Bngj+7ek6wS6+/5NeDvgKPJ/VMzUpcDj5jZajO7IVkW++94BrAX+HbSHfZNMxuTgboKLQbuS25HrcvdnwP+CdgO7Aa6gdVU4fNVL6FvvSzTsKRemNlY4EfA59x9X+x6ANz9iIfN7ynAAuDtva1WzZrM7L3AC+6+unBxL6vG+Jz9mbufR+jO/LSZXRKhhmLDgfOAb7j7ucBrxOli6lXSN34V8MPYtQAk+xCuBmYApwFjCH/PYmX/fNVL6O8EphbcnwLsilRLb543s8kAyfULMYows+MIgf89d38wS7UBuPsfgV8T9jmcZGbDk4di/D3/DLjKzLYC3yd08XwtA3Xh7ruS6xcI/dMLiP933AnsdPfHkvsPEL4EYteVWgQ84e7PJ/dj1/Ue4Fl33+vuh4EHgXdQhc9XvYT+40BTsud7BGEzbnnkmgotB65Pbl9P6E+vKjMzYBmwwd3vzEptZjbBzE5Kbh9P+GfYAPwKuCZWXe5+i7tPcfdGwufpUXf/UOy6zGyMmY1LbxP6qTuI/Hd09z3ADjNrThZdDqyPXVeB6zjWtQPx69oOXGRmo5P/zfT3VfnPV6ydKhXYMfLnwCZCf/CtEeu4j9BHd5jQ+llC6AteCXQl16dEqOudhE3Fp4A1yeXPY9cGnA08mdTVAdyWLD8DWAVsJmySj4z4N3038LMs1JW8/9rksi79rMf+OyY1zAfak7/lj4GTM1LXaOAl4MSCZVmo68vAxuRz/11gZDU+XzoiV0QkR+qle0dEREqg0BcRyRGFvohIjij0RURyRKEvIpIjCn0RkRxR6IuI5IhCX0QkR/4fGm64UeOP1rwAAAAASUVORK5CYII=\n", "text/plain": [ "
" ] }, "metadata": { "needs_background": "light" }, "output_type": "display_data" }, { "name": "stdout", "output_type": "stream", "text": [ "9.445142445940746e-10\n" ] }, { "data": { "image/png": "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\n", "text/plain": [ "
" ] }, "metadata": { "needs_background": "light" }, "output_type": "display_data" }, { "name": "stdout", "output_type": "stream", "text": [ "5.674669623090267e-10\n" ] }, { "data": { "image/png": "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\n", "text/plain": [ "
" ] }, "metadata": { "needs_background": "light" }, "output_type": "display_data" }, { "name": "stdout", "output_type": "stream", "text": [ "3.4013003524790975e-10\n" ] }, { "data": { "image/png": "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\n", "text/plain": [ "
" ] }, "metadata": { "needs_background": "light" }, "output_type": "display_data" }, { "name": "stdout", "output_type": "stream", "text": [ "2.033978541149395e-10\n" ] }, { "data": { "image/png": "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\n", "text/plain": [ "
" ] }, "metadata": { "needs_background": "light" }, "output_type": "display_data" }, { "name": "stdout", "output_type": "stream", "text": [ "5.862472729489809e-10\n" ] }, { "data": { "image/png": "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\n", "text/plain": [ "
" ] }, "metadata": { "needs_background": "light" }, "output_type": "display_data" }, { "name": "stdout", "output_type": "stream", "text": [ "3.5380276486307594e-10\n" ] }, { "data": { "image/png": "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\n", "text/plain": [ "
" ] }, "metadata": { "needs_background": "light" }, "output_type": "display_data" }, { "name": "stdout", "output_type": "stream", "text": [ "2.130270404521184e-10\n" ] }, { "data": { "image/png": "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\n", "text/plain": [ "
" ] }, "metadata": { "needs_background": "light" }, "output_type": "display_data" }, { "name": "stdout", "output_type": "stream", "text": [ "1.2797496395933194e-10\n" ] }, { "data": { "image/png": "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\n", "text/plain": [ "
" ] }, "metadata": { "needs_background": "light" }, "output_type": "display_data" }, { "name": "stdout", "output_type": "stream", "text": [ "7.671019375266042e-11\n" ] }, { "data": { "image/png": "iVBORw0KGgoAAAANSUhEUgAAAX0AAAD8CAYAAACb4nSYAAAABHNCSVQICAgIfAhkiAAAAAlwSFlzAAALEgAACxIB0t1+/AAAADl0RVh0U29mdHdhcmUAbWF0cGxvdGxpYiB2ZXJzaW9uIDMuMC4zLCBodHRwOi8vbWF0cGxvdGxpYi5vcmcvnQurowAAHYNJREFUeJzt3XtwXOWZ5/HvI8l3g02MTfBVvsjEMky4KDYJsCQx7HBJ4uwsTOwEwmRhnMyGCruZSQKbCUNImC1SU/FO1bJJXJCMA4TLkFDxZMxQ2DAMCUFYYC6RZGzJXKzYxja+gAFbFnr2j7dPSW5LqCV19zl9zu9T5eru0293P2q3fv3qPe95j7k7IiKSDVVxFyAiIuWj0BcRyRCFvohIhij0RUQyRKEvIpIhCn0RkQxR6IuIZIhCX0QkQxT6IiIZUhN3AflOPPFEr62tjbsMEZGK8swzz+xx98kDtUtc6NfW1tLU1BR3GSIiFcXMXi2knYZ3REQyRKEvIpIhCn0RkQxR6IuIZIhCX0QkQxT6IiIZotAXEcmQxM3TF0maJ56ARx7puX388XDddTBiRHw1iQyVQl/kfXR1wfLl8Mc/ghlEp5SeNi1sF6k0Gt4ReR9r14bAf/BB6O6G996DOXPgJz+JuzKRoVHoi7yPn/wEpk6FT30q3K6qghUr4PHHYdOmeGsTGQqFvkg/XnkFHnoIrr4aanoNhH7pS2E8X719qUQKfZF+3H57GMe/5pqjt0+ZAn/2Z7B6Nbz7bjy1iQyVQl+kD0eOwB13wCWXwMyZx97/5S/Dvn3wz/9c/tpEhqOg0Dezi8zsJTNrM7Pr+7h/lJndl7u/0cxqc9tHmNlqM3vRzFrN7Ibili9SGmvWwM6dIdz78vGPw/z5GuKRyjNg6JtZNXAbcDFQDyw3s/q8ZlcD+9x9HrASuDW3/XJglLufBpwFfDn6QhBJsh//GGbMgIsv7vt+s/CF8OST8OKL5a1NZDgK6ekvAtrcfau7dwL3Akvz2iwFVueuPwAsMTMDHBhnZjXAGKATeLMolYuUyLZtsG5dGMuvru6/3VVXwahR8LOfla82keEqJPSnAdt63e7Ibeuzjbt3AQeASYQvgLeBHcBrwD+4+978FzCzFWbWZGZNu3fvHvQPIVJMv/tduPz0p9+/3aRJ8LGPwW9/W/qaRIqlkNC3PrZ5gW0WAe8BU4HZwF+b2ZxjGrqvcvcGd2+YPHnAUzyKlNRTT8GYMXDaaQO3XbwYnnsODh0qfV0ixVBI6HcAM3rdng5s769NbihnArAX+Dzwb+5+xN13Ab8DGoZbtEgpNTbCWWcdPTe/P4sXh5k+GzeWvi6RYigk9DcAdWY228xGAsuANXlt1gBX5a5fBjzq7k4Y0vmkBeOAswEdxyiJ1dkZAnzx4sLaR+0aG0tXk0gxDRj6uTH6a4GHgVbgfndvNrObzewzuWZ3AJPMrA34OhBN67wNGA/8gfDl8TN3f6HIP4NI0Tz/PBw+DGefXVj7k08O8/gV+lIpClpl093XAmvztt3Y6/ohwvTM/Mcd7Gu7SFI99VS4LLSnH7WNHieSdDoiV6SXxsbQe58+vfDHLF4c1unZtatkZYkUjUJfpJfGxhDi1td8tH5oXF8qiUJfJOeNN6CtbXBDOwBnnhkO4lLoSyVQ6IvkRKFd6E7cyNix8OEPa1xfKoNCXySnsTGcJKVhCEeSLF4MGzaEs2uJJJlCXySnsREWLoTx4wf/2MWL4c03dTYtST6Fvgihh/7004Mfz49oZ65UCoW+CLBlSzgpymDH8yPz58PEiRrXl+RT6IvQ00Mfak+/qgoWLVJPX5JPoS9C2Ak7fjwsWDD051i8OJxQRefNlSRT6IsALS1QX//+J00ZyKmnhn0DmzcXry6RYlPoiwCtrcPr5UPP41tbh1+PSKko9CXzDhyAHTuGH/rz54ex/ZaW4tQlUgoKfcm8qGc+3NAfNQrmzFFPX5JNoS+ZF/XM6+uH/1wLFij0JdkU+pJ5ra2hlz579vCfa8GCsCO3q2v4zyVSCgp9ybzW1jAeP5yZO5EFC8I5c7duHf5ziZSCQl8yrxgzdyLREJGGeCSpFPqSae++Cy+/XLzQ/9CHwqVCX5JKoS+ZtnkzuBcv9I8/HqZNU+hLcin0JdOKNV2zN83gkSRT6EumtbaGA6rmzy/ec0ah71685xQpFoW+ZFprazigavTo4j3nggVw8CB0dBTvOUWKRaEvmVbMmTsRrcEjSabQl8zq6go7chX6kiUKfcmsl1+Gzs7ih/6UKXDCCQp9SSaFvmRWKWbuAJhpBo8kl0JfMisK5eiAqmJS6EtSKfQls1paYOpUmDCh+M+9YAHs3g179hT/uUWGQ6EvmVWKmTsRrcEjSaXQl0xyh02bShf6msEjSaXQl0zatQveegtOOaU0zz9zZlijv62tNM8vMlQKfcmkKIznzSvN81dVhSN9FfqSNAp9yaRSh3703Ap9SRqFvmRSW1s4U9asWaV7jSj0tfCaJIlCXzKprQ1qa2HEiNK9xrx54SQtO3aU7jVEBkuhL5nU1lbaoR3oeX4N8UiSFBT6ZnaRmb1kZm1mdn0f948ys/ty9zeaWW2v+/7EzH5vZs1m9qKZFXERW5HBc4ctWxT6kk0Dhr6ZVQO3ARcD9cByM6vPa3Y1sM/d5wErgVtzj60B7gK+4u4LgY8DR4pWvcgQ7N0LBw6UPvRnzoSaGoW+JEshPf1FQJu7b3X3TuBeYGlem6XA6tz1B4AlZmbAfwZecPfnAdz9DXd/rziliwxNOWbuQAj82bMV+pIshYT+NGBbr9sduW19tnH3LuAAMAmYD7iZPWxmz5rZN4dfssjwlCv0o9dQ6EuSFBL61se2/Elo/bWpAc4FvpC7/C9mtuSYFzBbYWZNZta0e/fuAkoSGbq2trD88ezZpX8tTduUpCkk9DuAGb1uTwe299cmN44/Adib2/64u+9x93eAtcCZ+S/g7qvcvcHdGyZPnjz4n0JkENraepZJKLV588JyD+rLSFIUEvobgDozm21mI4FlwJq8NmuAq3LXLwMedXcHHgb+xMzG5r4MzgdailO6yNCUY7pmRDN4JGkGDP3cGP21hABvBe5392Yzu9nMPpNrdgcwyczagK8D1+ceuw/4IeGL4zngWXf/1+L/GCKFU+hLltUU0sjd1xKGZnpvu7HX9UPA5f089i7CtE2R2O3fH05sUq7Qr60Ni68p9CUpdESuZEp7e7gsV+iPHBnW94leVyRuCn3JlHJO14xo2qYkiUJfMiUK3zlzyveaCn1JEoW+ZEpbG0ybBmPHlu81580LSz/s3Vu+1xTpj0JfMqWcM3ci0etpXF+SQKEvmRJn6GuIR5JAoS+ZcfAg7NwJc+eW93Wj5R4U+pIECn3JjHJP14yMGQPTpyv0JRkU+pIZcUzXjGgGjySFQl8yIwrdcg/vgEJfkkOhL5nR3g6TJ8Pxx5f/tefOhV27woqbInFS6EtmtLfH08uHntfdujWe1xeJKPQlM5IQ+pqrL3FT6EsmdHbCtm0KfRGFvmTCK69Ad3d8oT9hAkyapNCX+Cn0JRPinLkTmTtXM3gkfgp9yYSohx1n6M+bp56+xE+hL5nQ3g7jxsFJJ8VXw9y58NprYf+CSFwU+pIJ7e1hDX2z+GqYOzfsV3j11fhqEFHoSybEOV0zohk8kgQKfUm97u5wUJRCX0ShLxmwfTscPhzPQmu9ffCD4YxdCn2Jk0JfUi8JM3cg7E+YM0ehL/FS6EvqJSX0oxoU+hInhb6kXlsb1NTAzJlxV9IT+t3dcVciWaXQl9Rrb4dZs0Lwx23uXDh0CHbsiLsSySqFvqReEqZrRqKdyRrikbgo9CX1khT6mrYpcVPoS6rt3Qv79ycn9GfOhOpqhb7ER6EvqZakmTsAI0aE/QsKfYmLQl9SLWmhD5q2KfFS6EuqReE6Z068dfSm0Jc4KfQl1drbw/IH48bFXUmPuXPDvoZ9++KuRLJIoS+p1taWrKEd0AweiZdCX1KtrS3+hdbyRfXo1IkSB4W+pNbBg+HI17q6uCs5WtTT37Il3jokmxT6klpRT3r+/HjryDd2LEyfrtCXeBQU+mZ2kZm9ZGZtZnZ9H/ePMrP7cvc3mllt3v0zzeygmf1NccoWGVgUqknr6UP4IlLoSxwGDH0zqwZuAy4G6oHlZlaf1+xqYJ+7zwNWArfm3b8SeGj45YoUbvPmcJm0MX0IX0RRfSLlVEhPfxHQ5u5b3b0TuBdYmtdmKbA6d/0BYIlZOAW1mX0W2Ao0F6dkkcJs2QInnwzjx8ddybHq6sK0zb17465EsqaQ0J8GbOt1uyO3rc827t4FHAAmmdk44FvAd4dfqsjgbNmSvPH8SDTkpCEeKbdCQt/62OYFtvkusNLdD77vC5itMLMmM2vavXt3ASWJDGzz5mSO50PPl5GGeKTcCjmtRAcwo9ft6cD2ftp0mFkNMAHYCywGLjOzHwATgW4zO+Tu/7f3g919FbAKoKGhIf8LRWTQ9u+HPXuSG/pz5kBVlXr6Un6FhP4GoM7MZgN/BJYBn89rswa4Cvg9cBnwqLs7cF7UwMxuAg7mB75IKURhmtThnZEjw2qbCn0ptwFD3927zOxa4GGgGvipuzeb2c1Ak7uvAe4A7jSzNkIPf1kpixYZSJKna0bq6hT6Un4FnTXU3dcCa/O23djr+iHg8gGe46Yh1CcyJJs3g1ny1t3pbf58WL0a3EOtIuWgI3IllbZsgRkzYPTouCvpX10dvPUW7NoVdyWSJQp9SaUkT9eMaNqmxEGhL6njHoI0yeP5oNCXeCj0JXX27AlTNpMe+rW1UFOjufpSXgp9SZ2kT9eM1NSE+frq6Us5KfQldSphumZE0zal3BT6kjqbN0N1NcyeHXclA4tCv7s77kokKxT6kjpbtoTAHzEi7koGNn8+vPsubM9f2ESkRBT6kjqVMHMnohk8Um4KfUmVSpmuGVHoS7kp9CVVduyAt9+unNCfMQNGjdK0TSkfhb6kShSelRL6VVXhdI4KfSkXhb6kSmtruKzPP4tzgi1Y0FO3SKkp9CVVmpvhuONg+vS4KyncwoXQ3h5m8YiUmkJfUqW5OfTyK2mp4vr6sAN606a4K5EsUOhLqrS0hJ5zJYnqbWmJtw7JBoW+pMaePWFt+koL/bq6sA5Pc3PclUgWKPQlNaKeciXtxIVwvty6OvX0pTwU+pIaUU+50nr6EGpWT1/KQaEvqdHSUnkzdyL19bB1q2bwSOkp9CU1KnHmTmThwrDS5ksvxV2JpJ1CX1Kjubkyh3agp24N8UipKfQlFaKZO5W2EzcSzeDRzlwpNYW+pEIUlpXa049m8KinL6Wm0JdUqPTQh1C7evpSagp9SYVKXHMnX319WIPn0KG4K5E0U+hLKrS0VO7MnYhm8Eg5KPQlFSp55k5EM3ikHBT6UvHeeANef71yZ+5EtAaPlINCXypeGnbigtbgkfJQ6EvFi3rGld7Th/AzqKcvpaTQl4oXrbkzY0bclQxfdBYtzeCRUlHoS8V7/nk49dTKnrkTOe20MIPnD3+IuxJJK4W+VLT33oNnn4WGhrgrKY7o53jmmXjrkPRS6EtF27wZDh5MT+jPmgWTJkFTU9yVSFop9KWiReGYltA3Cz+LQl9KRaEvFa2pCcaNg1NOibuS4mloCGP6OqGKlIJCXypaUxOceSZUV8ddSfE0NEBXF7zwQtyVSBoVFPpmdpGZvWRmbWZ2fR/3jzKz+3L3N5pZbW77hWb2jJm9mLv8ZHHLlyzr6oKNG9MztBOJfh4N8UgpDBj6ZlYN3AZcDNQDy80s/zCYq4F97j4PWAncmtu+B/i0u58GXAXcWazCRVpbwxBI2kJ/2jQ46SSFvpRGIT39RUCbu291907gXmBpXpulwOrc9QeAJWZm7r7R3bfntjcDo81sVDEKF4lC8ayz4q2j2MzCz6TQl1IoJPSnAdt63e7Ibeuzjbt3AQeASXlt/iuw0d0PD61UkaM1NYUjcevq4q6k+BoawpHGb78ddyWSNoWEfl/HOfpg2pjZQsKQz5f7fAGzFWbWZGZNu3fvLqAkkRD6Z50FVSmcjtDQEI7Mfe65uCuRtCnk16UD6L2qyXRge39tzKwGmADszd2eDjwIfNHd2/t6AXdf5e4N7t4wefLkwf0EkkmdnWH5hbSN50eiISsN8UixFRL6G4A6M5ttZiOBZcCavDZrCDtqAS4DHnV3N7OJwL8CN7j774pVtEhzMxw+nN7Qnzo1/FPoS7ENGPq5MfprgYeBVuB+d282s5vN7DO5ZncAk8ysDfg6EE3rvBaYB3zHzJ7L/ZtS9J9CMidtR+L2RUfmSinUFNLI3dcCa/O23djr+iHg8j4e933g+8OsUeQYTU0wcSLMmRN3JaXT0AD/8i/w5ptw/PFxVyNpkcJdYJIFTU0hFNOwnHJ/GhrAPRyAJlIsCn2pOIcPw4svpntoB3p25m7YEG8dki4Kfak4Tz0FR47ARz8adyWlNWUKzJ0LTzwRdyWSJgp9qTjr1oW5+eefH3clpbdkCfz7v4d1hkSKQaEvFWf9eli0CCZMiLuS0rvggrAjV7N4pFgU+lJR3nwTnn469ICz4BOfCJfr1sVbh6SHQl8qyuOPh/PiXnBB3JWUx4knwhlnhL9uRIpBoS8VZd06GDMm/Ttxe1uyBJ58Et55J+5KJA0U+lJR1q+H886DURlaoPuCC8JaQ7/9bdyVSBoo9KVi7NwZ1tzJynh+5NxzYcQIDfFIcSj0pWJEoZe10B83LgxnaWeuFINCXyrG+vXwgQ/A6afHXUn5XXBBWI7hjTfirkQqnUJfKoJ76Ol+4hNQXR13NeW3ZEl4Dx57LO5KpNIp9KUitLXBtm3ZmaqZ7yMfCaeG1Li+DJdCXypCNJ6dtfH8yIgRYdmJRx4JPX6RoVLoS0W4//5wAvR58+KuJD6XXgrt7TpvrgyPQl8S77XXwqJjV1yR7vXzB3L55aHHf9ddcVcilUyhL4n3i1+EyyuuiLeOuE2aFHr7v/iFVt2UoVPoS6K5w513wjnnpPvUiIW68spwkJp26MpQKfQl0Z57Dlpa1MuPXHppODewhnhkqBT6kmh33gkjR8Kf/3nclSTDqFHhvfjVr+DgwbirkUqk0JfE6uoK49eXXhqOxJXgyivDipsPPhh3JVKJFPqSWOvWweuva2gn3znnwOzZ4a8gkcFS6Eti3XUXnHBC6OlLD7PwRbh+PWzfHnc1UmkU+pJI27fDL38Jn/tcttbOL9SVV0J3N/zoR3FXIpVGoS+J9L3vhTH9b3wj7kqSqa4uHKy1cmUYAhMplEJfEqe9HW6/HVas0Nz89/O978GhQ/D3fx93JVJJFPqSODfeGJYb+Nu/jbuSZDvlFPiLv4Af/xhefTXuaqRSKPQlUV54Ae65B667Dk4+Oe5qku/v/i7s2L3pprgrkUqh0JdE+fa3YcIE+OY3466kMsyYAV/9Kvz85+HIZZGBKPQlMdavh9/8JgT+CSfEXU3luOGGcB7db3xDa+3LwBT6kgivvQbLl4dx6q99Le5qKsuJJ8LNN8PatdqpKwNT6Evs3nkHPvtZOHwYfv3r0GuVwbnuOvj85+E73wl/LYn0R6EvsXKHa64Jq2nec0/o6cvgmYVprmecEcJ/06a4K5KkUuhLbNzDsMQ994RhiUsuibuiyjZmTFiEbcwYWLoUduyIuyJJIoW+xOLNN8MSCzfdFJYU+Na34q4oHWbODMtXdHTA6afDo4/GXZEkjUJfyu7556GhIawJ/4MfwOrV2T73bbGdey5s2BBOr3jhhfD974d1ekSgwNA3s4vM7CUzazOz6/u4f5SZ3Ze7v9HManvdd0Nu+0tm9qfFK10qzaZN8JWvwOLF4QQgjz0Wphkq8Iuvvh6efhqWLQs7dz/ykTCMduRI3JVJ3AYMfTOrBm4DLgbqgeVmVp/X7Gpgn7vPA1YCt+YeWw8sAxYCFwH/L/d8khE7d8J994XlkRcsgH/6J/jiF2HjRjjvvLirS7fx48Py1D//Obz9dtjBO2cO3HILNDVV4MnV774bamuhqipc3n133BVVJPMBjuYws48CN7n7n+Zu3wDg7v+7V5uHc21+b2Y1wE5gMnB977a92/X3eg0NDd7U1DSsH0rK58gROHAA9u2D3bvh5ZfDv/Z2ePJJ2Lw5tJsyJRw5+ld/BZMnx1tzFnV3w0MPwQ9/2DPOf9xxYSjo1FPDSVlqa2H69HBg3AknwNixCfor7O67wwp877zTs23sWFi1Cr7whfjqShAze8bdGwZqV1PAc00DtvW63QEs7q+Nu3eZ2QFgUm77U3mPnVbAaw5aS0tYfEoKk/9d796zLbre3d1z2dUF770XQv7w4fDv0CF4992+n/+DHwzj9n/5l3D++WEqYU0hnzYpiaqq8NfWpZeGcxX8x3/A44/DE0+EI6E7O499TE1NmAk0enQ4p8HIkWFbdXX4V1UVvhSiSzj2Mv/6kD2/EDofO3rbO8B/Gwn/WITnT4gLLwx/iZVSIb+Gff2X5f950F+bQh6Lma0AVgDMnDmzgJKOVVMTjkyUwuX/Mpr1bIt+kaNf6pqanl/40aN7guC442DixNAznDQp9BhnzQqdMEmmqVPDWP+yZeF2d3eY3vnyy+ELYf/+8Jfb/v3hiz36d+RI+OKPOgBRhyDaSdy70xAp2rIQnf2cIqwTOPH0Ir1I/I47rvSvUUjodwAzet2eDuT/D0RtOnLDOxOAvQU+FndfBayCMLxTaPG9zZ8fDkMXkcGpqoJp08K/xKr9732vHz1rFqx9pezlVLJCZu9sAOrMbLaZjSTsmF2T12YNcFXu+mXAox52FqwBluVm98wG6oCni1O6iGTGLbcc++fj2LGlHwtJoQF7+rkx+muBh4Fq4Kfu3mxmNwNN7r4GuAO408zaCD38ZbnHNpvZ/UAL0AV81d3fK9HPIiJpFe2s/fa3w+p8M2eGwNdO3EEbcPZOuWn2jojI4BU6e0dH5IqIZIhCX0QkQxT6IiIZotAXEckQhb6ISIYo9EVEMkShLyKSIQp9EZEMSdzBWWa2G+hjkY2CnQjsKVI5aaD342h6P46l9+Rolfp+zHL3ARcuT1zoD5eZNRVyVFpW6P04mt6PY+k9OVra3w8N74iIZIhCX0QkQ9IY+qviLiBh9H4cTe/HsfSeHC3V70fqxvRFRKR/aezpi4hIP1IT+mZ2kZm9ZGZtZnZ93PWUm5nNMLPHzKzVzJrN7Lrc9g+Y2SNmtiV3eULctZabmVWb2UYz+03u9mwza8y9J/flzgiXCWY20cweMLNNuc/KR7P+GTGz/5n7nfmDmd1jZqPT/BlJReibWTVwG3AxUA8sN7P6eKsquy7gr919AXA28NXce3A9sN7d64D1udtZcx3Q2uv2rcDK3HuyD7g6lqri8Y/Av7n7h4APE96XzH5GzGwa8DWgwd1PJZwdcBkp/oykIvSBRUCbu291907gXmBpzDWVlbvvcPdnc9ffIvwyTyO8D6tzzVYDn42nwniY2XTgUuD23G0DPgk8kGuSmffEzI4H/hPh9Ka4e6e77yfjnxHCaWPHmFkNMBbYQYo/I2kJ/WnAtl63O3LbMsnMaoEzgEbgJHffAeGLAZgSX2Wx+D/AN4Hu3O1JwH5378rdztJnZQ6wG/hZbrjrdjMbR4Y/I+7+R+AfgNcIYX8AeIYUf0bSEvrWx7ZMTksys/HAL4H/4e5vxl1PnMzsU8Aud3+m9+Y+mmbls1IDnAn8yN3PAN4mQ0M5fcntv1gKzAamAuMIw8T5UvMZSUvodwAzet2eDmyPqZbYmNkIQuDf7e6/ym1+3cxOzt1/MrArrvpicA7wGTN7hTDk90lCz39i7k95yNZnpQPocPfG3O0HCF8CWf6MXAC87O673f0I8CvgY6T4M5KW0N8A1OX2uI8k7IhZE3NNZZUbq74DaHX3H/a6aw1wVe76VcCvy11bXNz9Bnef7u61hM/Eo+7+BeAx4LJcs8y8J+6+E9hmZqfkNi0BWsjwZ4QwrHO2mY3N/Q5F70lqPyOpOTjLzC4h9OKqgZ+6+y0xl1RWZnYu8ATwIj3j1/+LMK5/PzCT8AG/3N33xlJkjMzs48DfuPunzGwOoef/AWAjcIW7H46zvnIxs9MJO7VHAluBLxE6f5n9jJjZd4HPEWbAbQSuIYzhp/IzkprQFxGRgaVleEdERAqg0BcRyRCFvohIhij0RUQyRKEvIpIhCn0RkQxR6IuIZIhCX0QkQ/4/2WmqxlY7NDYAAAAASUVORK5CYII=\n", "text/plain": [ "
" ] }, "metadata": { "needs_background": "light" }, "output_type": "display_data" }, { "name": "stdout", "output_type": "stream", "text": [ "4.588229796098631e-11\n" ] }, { "data": { "image/png": "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\n", "text/plain": [ "
" ] }, "metadata": { "needs_background": "light" }, "output_type": "display_data" }, { "name": "stdout", "output_type": "stream", "text": [ "2.738609339303366e-11\n" ] }, { "data": { "image/png": "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\n", "text/plain": [ "
" ] }, "metadata": { "needs_background": "light" }, "output_type": "display_data" }, { "name": "stdout", "output_type": "stream", "text": [ "1.6311951789305112e-11\n" ] }, { "data": { "image/png": "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\n", "text/plain": [ "
" ] }, "metadata": { "needs_background": "light" }, "output_type": "display_data" }, { "name": "stdout", "output_type": "stream", "text": [ "9.696576874773655e-12\n" ] }, { "data": { "image/png": "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\n", "text/plain": [ "
" ] }, "metadata": { "needs_background": "light" }, "output_type": "display_data" }, { "name": "stdout", "output_type": "stream", "text": [ "2.892275308141734e-11\n" ] }, { "data": { "image/png": "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\n", "text/plain": [ "
" ] }, "metadata": { "needs_background": "light" }, "output_type": "display_data" }, { "name": "stdout", "output_type": "stream", "text": [ "1.7338130930966145e-11\n" ] }, { "data": { "image/png": "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\n", "text/plain": [ "
" ] }, "metadata": { "needs_background": "light" }, "output_type": "display_data" }, { "name": "stdout", "output_type": "stream", "text": [ "4.970746037002982e-11\n" ] }, { "data": { "image/png": "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\n", "text/plain": [ "
" ] }, "metadata": { "needs_background": "light" }, "output_type": "display_data" }, { "name": "stdout", "output_type": "stream", "text": [ "3.0039970511097636e-11\n" ] }, { "data": { "image/png": "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\n", "text/plain": [ "
" ] }, "metadata": { "needs_background": "light" }, "output_type": "display_data" }, { "name": "stdout", "output_type": "stream", "text": [ "1.8115842159716067e-11\n" ] }, { "data": { "image/png": "iVBORw0KGgoAAAANSUhEUgAAAX0AAAD8CAYAAACb4nSYAAAABHNCSVQICAgIfAhkiAAAAAlwSFlzAAALEgAACxIB0t1+/AAAADl0RVh0U29mdHdhcmUAbWF0cGxvdGxpYiB2ZXJzaW9uIDMuMC4zLCBodHRwOi8vbWF0cGxvdGxpYi5vcmcvnQurowAAIABJREFUeJzt3X+UVPV9//HnG1YWQUXBxRp+uLuCP9YEjW6IpsYo+APT1k3z1QaStPaEU5qT0iRNmxS/aWxi6mnNaUuThuYbW22tMf740tZyUr7Bxk3S5hdxiYoC4i4rygYNqyBGCCw/3t8/PveWcZx17+7O7L1z7+txzpyZufOZue/LsK/5zOd+5l5zd0REpBjGpV2AiIiMHYW+iEiBKPRFRApEoS8iUiAKfRGRAlHoi4gUiEJfRKRAFPoiIgWi0BcRKZCGtAsod+qpp3pzc3PaZYiI1JUNGza86O5NQ7XLXOg3NzfT1dWVdhkiInXFzJ5N0k7DOyIiBaLQFxEpEIW+iEiBKPRFRApEoS8iUiAKfRGRAkkU+ma2yMy2mlmPma2o8Hijmd0fPb7ezJqj5ceZ2V1m9oSZbTGzm6pbvoiIDMeQoW9m44FVwLVAG7DEzNrKmi0F9rj7HGAlcFu0/Aag0d3fAlwE/G78gSBST9zhnntgx460KxEZnSQ9/flAj7v3uvsAcB/QUdamA7grur0aWGhmBjgw2cwagOOBAeCVqlQuMob+5m/ggx+Eq6+GV/Q/WOpYktCfAZT2b/qiZRXbuPthYC8wjfABsA94HngO+Et3312+AjNbZmZdZtbV398/7I0QqaXOTvjkJ+GSS6C7G268EY4eTbsqkZFJEvpWYZknbDMfOAK8CWgB/tDMWl/X0P12d2939/ampiEPHSEyZp59Ft73PjjrLFi3Dv7qr+DBB+HP/zztykRGJkno9wGzSu7PBHYO1iYaypkC7AbeD3zT3Q+5+y7g+0D7aIsWGQuHDsF73xuuH3wQTjwRPvrRMMzzmc+EDwGRepMk9B8B5ppZi5lNABYDa8rarAFujG5fD3S6uxOGdBZYMBm4GHiqOqWL1NZ3vws/+Ql8+cuhpw9gBl/9KsyeDV/8Yrr1iYzEkKEfjdEvB9YBW4AH3H2Tmd1iZtdFze4ApplZD/AJIJ7WuQo4AXiS8OHxj+6+scrbIFIT69bBccfBe97z2uWTJsGv/Rp85ztw4EAqpYmMmIUOeXa0t7e7Dq0sWXD++TBtWtiRW+4b3wjB/61vwcKFY1+bSDkz2+DuQw6f6xe5IhU8/zxs3AjXXFP58csvD98CNK4v9UahL1LBQw+F66uvrvz4CSfAL/+yQl/qj0JfpIJ162D69DDEM5hrrgnfBp5/fuzqEhkthb5ImaNH4T//M/Tyx73BX0g89BN/KxCpBwp9kTKPPgovvjj4eH7s/PPDtwGFvtQThb5ImXic/qqr3rjduHHh28BDD+mwDFI/FPoiZdatgwsugNNOG7rt1VeHbwWPPlr7ukSqQaEvUuKVV+AHPxh6aCcWz+7RLB6pFwp9kRI//CEcPgxXXpms/WmnwVveEg7ZIFIPFPoiJTZGBwl561uTP+fCC489TyTrFPoiJTZuhBkzwuEXkpo3D154AXbtql1dItWi0BcpsXFjCPHhiNs/8UT16xGpNoW+SGRgALZsGXnoa4hH6oFCXySydWs4YcpwQ3/69LBDV6Ev9UChLxKJQ3u4oR8/R6Ev9SBR6JvZIjPbamY9ZraiwuONZnZ/9Ph6M2uOln/AzB4ruRw1swuquwki1bFxYzhc8tlnD/+58+bBpk1huqdIlg0Z+mY2nnAGrGuBNmCJmbWVNVsK7HH3OcBK4DYAd7/H3S9w9wuA3wS2u/tj1dwAkWrZuBHa2kLwD9e8eXDwIHR3V78ukWpK0tOfD/S4e6+7DwD3AR1lbTqAu6Lbq4GFZmZlbZYA946mWJFaGsnMnZh25kq9SBL6M4AdJff7omUV20Tn1N0LlM90fh+DhL6ZLTOzLjPr6u/vT1K3SFW9+CLs3Dny0D/3XGhoUOhL9iUJ/fIeO0D5iXXfsI2ZvR3Y7+5PVlqBu9/u7u3u3t7U1JSgJJHqiufYv9FJU95IYyOcc45CX7IvSej3AbNK7s8Edg7WxswagCnA7pLHF6OhHcmw0czciWkGj9SDJKH/CDDXzFrMbAIhwNeUtVkD3Bjdvh7odHcHMLNxwA2EfQEimbRx47H59iM1bx489xy8/HL16hKptiFDPxqjXw6sA7YAD7j7JjO7xcyui5rdAUwzsx7gE0DptM7LgD53761u6SLVM5qduDEdjkHqQUOSRu6+FlhbtuzmktsHCL35Ss/9DnDxyEsUqa0jR+DJJ+EjHxnd65TO4HnnO0dfl0gt6Be5Ung9PXDgwOh7+m96E0ydqnF9yTaFvhRePBzzlreM7nXMwmso9CXLFPpSeFu3huuRHH6h3DnnwNNPj/51RGpFoS+F190dhmYmTx79a82dC7t3h4tIFin0pfC6u0NYV0P8OjoGj2SVQl8KT6EvRaLQl0Lbuxf6+6sX+q2tMG6cQl+yS6EvhRaHc7VCv7ERZs9W6Et2KfSl0Kod+vFrKfQlqxT6Umjd3WF+/ZlnVu81zzorvK6XH4tWJAMU+lJo3d0waxYcf3z1XnPu3LCv4MUXq/eaItWi0JdCq+bMnZhm8EiWKfSl0J5+unahr1/mShYp9KWwXnoJ9uypfug3N8P48erpSzYp9KWwajFzB+C446ClRaEv2ZQo9M1skZltNbMeM1tR4fFGM7s/eny9mTWXPDbPzH5oZpvM7Akzm1i98kVGrlahH7+mQl+yaMjQN7PxwCrgWqANWGJmbWXNlgJ73H0OsBK4LXpuA/A14MPufh5wOXCoatWLjEJ3d/j1bGtr9V87Dn1N25SsSdLTnw/0uHuvuw8QznXbUdamA7grur0aWGhmBlwNbHT3xwHc/SV3P1Kd0kVGp7sbzjgDJkyo/mvPnQv79sELL1T/tUVGI0nozwB2lNzvi5ZVbBOdU3cvMA04C3AzW2dmPzGzT42+ZJHq6O4OP6Sqhfh1NcQjWZMk9K3CsvIvrYO1aQAuBT4QXf+6mS183QrMlplZl5l19ff3JyhJZHTcazNHP6a5+pJVSUK/D5hVcn8msHOwNtE4/hRgd7T8u+7+orvvJ5xc/cLyFbj77e7e7u7tTU1Nw98KkWHq74dXXqld6M+eHYaNFPqSNUlC/xFgrpm1mNkEYDGwpqzNGuDG6Pb1QKe7O7AOmGdmk6IPg3cBm6tTusjI1XLmDoR5+q2tCn3JnoahGrj7YTNbTgjw8cCd7r7JzG4Butx9DXAHcLeZ9RB6+Iuj5+4xs78mfHA4sNbd/6NG2yKSWPxr2VqFfvza+lWuZM2QoQ/g7msJQzOly24uuX0AuGGQ536NMG1TJDO2bQu98TPOqN065syBhx8O+w+s0l4vkRToF7lSSM88E8bdjzuudutobYX9+2HXrtqtQ2S4FPpSSL29tflRVqn49Xt7a7sekeFQ6EshKfSlqBT6UjivvhqGXGod+s3N4VqhL1mi0JfCeeaZcN3SUtv1TJwIM2Yo9CVbFPpSOHEI17qnH69DoS9ZotCXwhnL0G9pUehLtij0pXCeeQZOOgmmTq39ulpb4ac/hYMHa78ukSQU+lI48cydsfjBVGtr+HHWs8/Wfl0iSSj0pXDGYrpmTNM2JWsU+lIoR4+G4Z1az9yJKfQlaxT6UigvvAAHDoxdT/+XfilM3VToS1Yo9KVQxnLmDoT9Bpq2KVmi0JdCiX+YNVahH68rXq9I2hT6Uii9vaH3XctDKpeL5+p7+UlGRVKQKPTNbJGZbTWzHjNbUeHxRjO7P3p8vZk1R8ubzewXZvZYdPk/1S1fZHh6e2HmTGhsHLt1traGUzPu3j126xQZzJAnUTGz8cAq4CrCOW8fMbM17l562sOlwB53n2Nmi4HbgPdFj21z9wuqXLfIiPT2jt3MnVjpDJ5p08Z23SLlkvT05wM97t7r7gPAfUBHWZsO4K7o9mpgoZnOFSTZM5Zz9GOatilZkiT0ZwA7Su73RcsqtnH3w8BeIO7TtJjZo2b2XTN75yjrFRmxX/wCdu4c+9CPv1ko9CULkpwjt1KPvXyX1GBtngdmu/tLZnYR8KCZnefur7zmyWbLgGUAs2fPTlCSyPBt3x6uxzr0J0+G005T6Es2JOnp9wGzSu7PBHYO1sbMGoApwG53P+juLwG4+wZgG3BW+Qrc/XZ3b3f39qampuFvhUgCaUzXjGnapmRFktB/BJhrZi1mNgFYDKwpa7MGuDG6fT3Q6e5uZk3RjmDMrBWYC6i/I6mIe9pjvSM3Xqd6+pIFQ4Z+NEa/HFgHbAEecPdNZnaLmV0XNbsDmGZmPcAngHha52XARjN7nLCD98PurolrkoreXjj++DDUMtZaW+G55+DQobFft0ipJGP6uPtaYG3ZsptLbh8AbqjwvH8B/mWUNYpUxVgeUrlcayscOQI7dqQzvCQS0y9ypTDSmK4Z07RNyQqFvhSCu0JfBBT6UhD9/bBvX3qh/6Y3wYQJCn1Jn0JfCiHN6ZoA48dDc7OmbUr6FPpSCGN9HP1KNG1TskChL4UQh21zc3o16GQqkgUKfSmE3t5w6sJJk9KrobU1HF755ZfTq0FEoS+FkObMnVi8fo3rS5oU+lIIWQp9DfFImhT6knsDA9n4JawOsSxZoNCX3HvuufDjrLRDf8oUmDpVwzuSLoW+5F4WpmvGNINH0qbQl9xT6Isco9CX3OvthcZGOP30tCsJob99ezjipkgaFPqSe729YSfquAz8b29tDcfU/+lP065EiirRn4GZLTKzrWbWY2YrKjzeaGb3R4+vN7Pmssdnm9mrZvZH1SlbJLk49LNA0zYlbUOGfnS6w1XAtUAbsMTM2sqaLQX2uPscYCVwW9njK4H/N/pyRYYvC3P0Ywp9SVuSnv58oMfde919ALgP6Chr0wHcFd1eDSw0C+cnMrP3EM6Lu6k6JYskt2cP7N2bndCfNSsccVPTNiUtSUJ/BrCj5H5ftKxim+icunsJ58ydDPwx8LnRlyoyfFmauQPQ0ACzZ6unL+lJEvqVzijqCdt8Dljp7q++4QrMlplZl5l19ff3JyhJJJmshT5o2qakK0no9wGzSu7PBHYO1sbMGoApwG7g7cAXzGw78HHgf5vZ8vIVuPvt7t7u7u1NTU3D3giRwcThmpUduaDQl3Q1JGjzCDDXzFqAnwKLgfeXtVkD3Aj8ELge6HR3B94ZNzCzzwKvuvuXq1C3SCK9vdDUBCeemHYlx7S2wq5d8OqrcMIJaVcjRTNkTz8ao18OrAO2AA+4+yYzu8XMroua3UEYw+8BPgG8blqnSBq2bcvW0A7AmWeGa/X2JQ1Jevq4+1pgbdmym0tuHwBuGOI1PjuC+kRGpbsbLrss7Spea86ccN3dDfPmpVuLFE8GfqMoUhsHDoRDKschmxVxPT096dYhxaTQl9zq7Q2HVJ47N+1KXuvEE+G000JPX2SsKfQlt+KedNZ6+hBqUk9f0qDQl9yKe9JZ6+lDqEk9fUmDQl9yq6cnnKnqlFPSruT15syBnTth3760K5GiUehLbnV3Z7OXD8fq2rYt3TqkeBT6kls9PdkczwfN4JH0KPQllw4cCCdEz3roa1xfxppCX3LpmWeyOV0zdtJJMH26evoy9hT6kktxDzqrPX0ItamnL2NNoS+5FPegs9rTh1Cbevoy1hT6kkvd3WGq5tSpaVcyuDlzwgnS9+9PuxIpEoW+5FJPT7Z7+aBpm5IOhb7kUnd3tsfzQTN4JB0KfcmdgwfDdM2s9/Q1V1/SoNCX3ImPrpn1nv6UKeGsXurpy1hKFPpmtsjMtppZj5m97qxYZtZoZvdHj683s+Zo+Xwzeyy6PG5mv17d8kVeL8tH1yyno23KWBsy9M1sPLAKuBZoA5aYWVtZs6XAHnefA6wEbouWPwm0u/sFwCLgq9GJ00VqJstH1yyno23KWEvS058P9Lh7r7sPAPcBHWVtOoC7oturgYVmZu6+PzrHLsBEwKtRtMgb6emBk0/O9nTNmKZtylhLEvozgB0l9/uiZRXbRCG/F5gGYGZvN7NNwBPAh0s+BP6HmS0zsy4z6+rv7x/+VoiUiGfumKVdydA0bVPGWpLQr/SnU95jH7SNu6939/OAtwE3mdnE1zV0v93d2929vampKUFJIoN76ik455y0q0jm7LPD9VNPpVuHFEeS0O8DZpXcnwnsHKxNNGY/Bdhd2sDdtwD7gDePtFiRobzyCvT1QVv5XqeMOvvs8I1k8+a0K5GiSBL6jwBzzazFzCYAi4E1ZW3WADdGt68HOt3do+c0AJjZGcDZwPaqVC5SwZYt4bpeQn/SJGhpUejL2BlyJo27Hzaz5cA6YDxwp7tvMrNbgC53XwPcAdxtZj2EHv7i6OmXAivM7BBwFPiIu79Yiw0RgWPhWS+hD6FWhb6MlUTTJ919LbC2bNnNJbcPADdUeN7dwN2jrFEksS1boLEx9J7rRVsbPPQQHD4MDZrQLDWmX+RKrmzeHMbJ6yk829pgYCD8klik1hT6kiubN9fX0A4cq1dDPDIWFPqSG/v2wfbt9Rf68fRShb6MBYW+5MbWreFAa/UW+ieeCLNnK/RlbCj0JTfi0Dz33HTrGIlzz1Xoy9hQ6EtubN4cduDWw9E1y7W1hZlHR46kXYnknUJfcmPz5nAsmwkT0q5k+Nra4MABePbZtCuRvFPoS27U48ydmGbwyFhR6EsuHDwYjlRZr6Ef74eIDyMhUisKfcmFp5+Go0frN/RPOQVOP109fak9hb7kQj0ec6ecjsEjY0GhL7mweTOMGwdnnZV2JSMXh77r/HJSQwp9yYXNm+HMM2Hi607RUz/a2uDVV8P5AERqRaEvubBpU33+KKtUXP+mTenWIfmm0Je6t29fOATD+eenXcnozJsXrh99NN06JN8Shb6ZLTKzrWbWY2YrKjzeaGb3R4+vN7PmaPlVZrbBzJ6IrhdUt3wRePzxMHPnoovSrmR0TjkFWlthw4a0K5E8GzL0zWw8sAq4FmgDlphZ+RyJpcAed58DrARui5a/CPyau7+FcDpFnVBFqi4OyXoPfQjboNCXWkrS058P9Lh7r7sPAPcBHWVtOoC7oturgYVmZu7+qLvHJ1HfBEw0s8ZqFC4S27ABpk+HGTPSrmT0LrooHB76pZfSrkTyKknozwB2lNzvi5ZVbOPuh4G9wLSyNv8LeNTdD46sVJHKNmwIYWmWdiWjF39b+clP0q1D8itJ6Ff6UyqfSfyGbczsPMKQz+9WXIHZMjPrMrOu/v7+BCWJBPv3h+maeRjaAbjwwnCtIR6plSSh3wfMKrk/E9g5WBszawCmALuj+zOBfwN+y923VVqBu9/u7u3u3t7U1DS8LZBCy8tO3NjUqeGk7gp9qZUkof8IMNfMWsxsArAYWFPWZg1hRy3A9UCnu7uZnQz8B3CTu3+/WkWLxPK0EzemnblSS0OGfjRGvxxYB2wBHnD3TWZ2i5ldFzW7A5hmZj3AJ4B4WudyYA7wGTN7LLpMr/pWSGFt2ABNTTBzZtqVVM9FF8Ezz8Du3WlXInnUkKSRu68F1pYtu7nk9gHghgrP+zPgz0ZZo8ig8rQTN1a6M/fKK9OtRfJHv8iVuvWLX4SduO3taVdSXXHod3WlW4fkk0Jf6tbjj4dzyuZpPB+0M1dqS6EvdSuPO3Fj2pkrtaLQl7qVx524Me3MlVpR6EvdyuNO3Jh+mSu1otCXuvTzn4fjzudtJ24sDv3169OtQ/JHoS916XvfCztxL7887UpqY+rUcHz9b3877UokbxT6Upc6O2HCBHjHO9KupHYWLIDvfx8OHEi7EskThb7Upc5OuOQSOP74tCupnSuuCIH/ox+lXYnkiUJf6s6ePeGUggtyfh62yy6DceM0xCPVpdCXuvPd74J7/kP/5JPDDt3OzrQrkTxR6Evd6eyESZNg/vy0K6m9K64IM3j27Uu7EskLhb7Unc5OuPTSsCM37xYsgEOHwg5dkWpQ6Etd+dnPwvz8vA/txC69FBoaNMQj1aPQl7ryne+E6yuuSLWMMTN5Mrz97dqZK9WTKPTNbJGZbTWzHjNbUeHxRjO7P3p8vZk1R8unmdm3zexVM/tydUuXIurshJNOOnYu2SJYsCAcZnnv3rQrkTwYMvTNbDywCrgWaAOWmFlbWbOlwB53nwOsJJwEHeAA8Bngj6pWsRRaZye8611hyKMoFiwI5wH+r/9KuxLJgyQ9/flAj7v3uvsAcB/QUdamA7grur0aWGhm5u773P17hPAXGZVt26Cnpzjj+bGLLw4/QvvmN9OuRPIgSejPAHaU3O+LllVsE51Tdy8wrRoFisTuuy9cv/e96dYx1iZOhF/9VVi9Gg4fTrsaqXdJQr/SgWt9BG0GX4HZMjPrMrOu/v7+pE+TAnGHr389zGaZPTvtasbekiWwa5dm8cjoJQn9PmBWyf2ZwM7B2phZAzAFSHz6B3e/3d3b3b29qakp6dOkQJ54IpwPd8mStCtJx7XXhh3YX/962pVIvUsS+o8Ac82sxcwmAIuBNWVt1gA3RrevBzrdPXFPX2Qo994L48fDDTekXUk6Jk4Mw1r/9m866qaMzpChH43RLwfWAVuAB9x9k5ndYmbXRc3uAKaZWQ/wCeB/pnWa2Xbgr4HfNrO+CjN/RN6Qewj9q64Kp0csqve/H155BdauTbsSqWeJJr65+1pgbdmym0tuHwAq9sHcvXkU9Ynwwx/Cs8/CLbekXUm6rrgCpk8PQzxF25kt1aNf5Erm3XtvGN54z3vSriRdDQ3wG78B3/hG6PGLjIRCXzLt8GF44AH4lV8JOzKLbskSOHgQHnww7UqkXin0JdMefDBMVfzAB9KuJBsuuQRaWuArXwn7OkSGS6EvmXX0aBjHP+ssuO66odsXgRl88pPhFIrf+lba1Ug9UuhLZj34YJif/5nPhOmaEnzoQzBzJnz2s+rty/Ap9CWTjh6Fz30O5s6FxYvTriZbGhvhppvgBz+Ahx9OuxqpNwp9yaR//3fYuDH08ot0RM2kli6FGTPCB6N6+zIcCn3JHPcwlj9nTnEPuzCUuLf/ve/peDwyPAp9yZx/+id47DH4kz9RL/+NxL39T34yTOMUSUKhL5myeTMsXw6XXw4f/GDa1WTbxImwahU8+ih86lNpVyP1QqEvmbF/f/jF6QknhEMNaMbO0Do64GMfgy99KRyMTWQoCn3JjN///dDT/9rX4PTT066mfnzhC/C2t4WpnNu3p12NZJ1CX1LnDn/xF3DnnfDpT4ejaUpyEybA/feHf8eODthZfrYLkRIKfUnVwAD8zu+EmSjvex/86Z+mXVF9amkJxyjatg3mzw87wkUqUehLavr7YdEiuOOOMFPn61/XbJ3RuPpq+P73w6EaLr1UY/xSWaLQN7NFZrbVzHrMbEWFxxvN7P7o8fVm1lzy2E3R8q1mdk31Spd69fLLcPPNcOaZIaT++Z/h85+HceqCjNr558OPfwznnhuOuX/lleF8BCKxIf/MzGw8sAq4FmgDllQ4+9VSYI+7zwFWArdFz20jnF7xPGAR8HfR60nBDAyEQwb8wR+EoYjPfx6uuSZMN/zN30y7unw5/XT47/+GlSvDsYve8Y7wLeDv/76Ox/vvuQeam0PPoLk53JcRSfJlej7Q4+69AGZ2H9ABbC5p0wF8Nrq9GviymVm0/D53Pwg8E51OcT6gvkcOHToEu3eHQyHv2gW9vWE2zubNobf585+HX5K++92hp3/BBWlXnF8TJ8LHPx72l/zt34ZDMS9bFh6bNy98I2hrg3POCR8S06eHU1FOnhyGhzLlnntC8fv3h/vPPntsY3TM7WFLEvozgB0l9/uAtw/Wxt0Pm9leYFq0/Edlz50x4mrfwObN8Nu/XYtXzr/SY7eU3y69HD0aLkeOhICPL/v3h8uhQ69/7UmTQrAsWRJOhLJwYQgWGRuTJ8OKFfDHfwybNoWzbnV2hsvdd7++vVl4zyZNCh/Qxx0XLg0NoZMdX+IPBrPXfkgMdntUHj8PBr792mX7gQ9NgC9WaR0ZcdVVcOuttV1HktCv9NaVH+JpsDZJnouZLQOWAcyePTtBSa/X0ACnnjqipwpv/IcbX8aPP/ZHH4fBccfB8ceHcJk8GaZOPdZrPOMMmD1bY/VZYAZvfnO4rIj2yu3dC93d8LOfhZ3qu3bBq6/Cvn3hMjBw7IP9yJFjH/hHj4bnx52B2GC3R21gkDGpAeDUfH1dPPHE2q8jSej3AbNK7s8Eyt+FuE2fmTUAU4DdCZ+Lu98O3A7Q3t4+ov8uZ50Fa9cO3U5EgilToL097SoSaP5IGNIpd8YZsHb7mJdT75L0wR4B5ppZi5lNIOyYXVPWZg1wY3T7eqDT3T1avjia3dMCzAV+XJ3SRaQQbr01jDeVmjSp9uMgOTVkTz8ao18OrAPGA3e6+yYzuwXocvc1wB3A3dGO2t2EDwaidg8QdvoeBn7P3Y/UaFtEJI/inbWf/jQ891wYM7z1Vu3EHSHzjJ2Bob293bu6utIuQ0SkrpjZBncfcsBOu9hERApEoS8iUiAKfRGRAlHoi4gUiEJfRKRAFPoiIgWi0BcRKRCFvohIgWTux1lm1g9UONBGYqcCL1apnHqi7S4WbXexJNnuM9y9aagXylzoj5aZdSX5VVreaLuLRdtdLNXcbg3viIgUiEJfRKRA8hj6t6ddQEq03cWi7S6Wqm137sb0RURkcHns6YuIyCByE/pmtsjMtppZj5mtSLueWjGzWWb2bTPbYmabzOxj0fKpZvafZtYdXZ+Sdq21YGbjzexRM/tGdL/FzNZH231/dHa3XDGzk81stZk9Fb3vlxTh/TazP4j+jz9pZvea2cS8vt9mdqeZ7TKzJ0uWVXyPLfhSlHUbzezC4awrF6FvZuOBVcC1QBuwxMza0q2qZg4Df+ju5wIXA78XbesK4GF3nws8HN3Po48BW0ru3wasjLZ7D7A0lapq64vAN939HOB8wvbn+v02sxnAR4F2d38z4ax9i8nv+/1PwKKyZYO9x9cSTj07F1gGfGU4K8ooGzX5AAACkUlEQVRF6APzgR5373X3AeA+oCPlmmrC3Z93959Et39OCIAZhO29K2p2F/CedCqsHTObCfwK8A/RfQMWAKujJrnbbjM7CbiMcEpS3H3A3V+mAO834XSux5tZAzAJeJ6cvt/u/l+EU82WGuw97gD+2YMfASeb2elJ15WX0J8B7Ci53xctyzUzawbeCqwHTnP35yF8MADT06usZv4G+BRwNLo/DXjZ3Q9H9/P4vrcC/cA/RsNa/2Bmk8n5++3uPwX+EniOEPZ7gQ3k//0uNdh7PKq8y0voW4VluZ6WZGYnAP8CfNzdX0m7nlozs18Fdrn7htLFFZrm7X1vAC4EvuLubwX2kbOhnEqi8esOoAV4EzCZMKxRLm/vdxKj+n+fl9DvA2aV3J8J7Eyplpozs+MIgX+Pu/9rtPhn8Ve86HpXWvXVyC8D15nZdsLw3QJCz//k6Os/5PN97wP63H19dH814UMg7+/3lcAz7t7v7oeAfwXeQf7f71KDvcejyru8hP4jwNxoz/4Ewg6fNSnXVBPROPYdwBZ3/+uSh9YAN0a3bwT+faxrqyV3v8ndZ7p7M+H97XT3DwDfBq6PmuVxu18AdpjZ2dGihcBmcv5+E4Z1LjazSdH/+Xi7c/1+lxnsPV4D/FY0i+diYG88DJSIu+fiArwbeBrYBnw67XpquJ2XEr7KbQQeiy7vJoxvPwx0R9dT0661hv8GlwPfiG63Aj8GeoD/CzSmXV8NtvcCoCt6zx8ETinC+w18DngKeBK4G2jM6/sN3EvYd3GI0JNfOth7TBjeWRVl3ROEGU6J16Vf5IqIFEhehndERCQBhb6ISIEo9EVECkShLyJSIAp9EZECUeiLiBSIQl9EpEAU+iIiBfL/AYrldUcBdLm8AAAAAElFTkSuQmCC\n", "text/plain": [ "
" ] }, "metadata": { "needs_background": "light" }, "output_type": "display_data" }, { "name": "stdout", "output_type": "stream", "text": [ "1.0903056235633812e-11\n" ] }, { "data": { "image/png": "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\n", "text/plain": [ "
" ] }, "metadata": { "needs_background": "light" }, "output_type": "display_data" } ], "source": [ "from scipy import special\n", "\n", "p0 = 0.5\n", "p1 = 0.7\n", "k=100\n", "\n", "for i in range(1,k):\n", " cs = numpy.linspace(0,i,i+1)\n", " c0 = numpy.zeros(i+1)\n", " for l in range(i):\n", " c0[l] = scipy.special.binom(i,l)*p0**(cs[l])*(1-p0)**(i-cs[l])\n", " plt.plot(cs,c0,color='blue')\n", " plt.scatter(numpy.array([y[i-1]]),numpy.array([0]),color='r')\n", " print(1-numpy.cumsum(c0)[y[i-1]])\n", " plt.show()" ] }, { "cell_type": "code", "execution_count": 117, "metadata": {}, "outputs": [ { "data": { "text/plain": [ "0.53" ] }, "execution_count": 117, "metadata": {}, "output_type": "execute_result" } ], "source": [ "numpy.mean(x)" ] }, { "cell_type": "markdown", "metadata": {}, "source": [ "# $t$-test\n", "\n", "We collect two samples of normally distributed data in an unpaired way. We test whether the difference of expectations is $0$ (null hypothesis)" ] }, { "cell_type": "code", "execution_count": 128, "metadata": {}, "outputs": [], "source": [ "## Import the packages\n", "import numpy\n", "from scipy import stats\n", "\n", "\n", "## Define 2 random distributions\n", "#Sample Size\n", "N = 10\n", "#Gaussian distributed data with mean = 1 and var = 1\n", "a = np.random.randn(N) + 1\n", "#Gaussian distributed data with with mean = 0 and var = 1\n", "b = np.random.randn(N)" ] }, { "cell_type": "code", "execution_count": 129, "metadata": {}, "outputs": [ { "name": "stdout", "output_type": "stream", "text": [ "t = 2.4141807955588708\n", "p = 0.026639355757870353\n", "t = 2.4141807955588717\n", "p = 0.02663935575787036\n" ] }, { "data": { "image/png": "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\n", "text/plain": [ "
" ] }, "metadata": { "needs_background": "light" }, "output_type": "display_data" } ], "source": [ "# calculate the empirical standard deviations with unbiased estimators\n", "\n", "var_a = a.var(ddof=1)\n", "var_b = b.var(ddof=1)\n", "\n", "#std deviation\n", "s = numpy.sqrt((var_a + var_b)/2)\n", "\n", "# Calculate the t-statistics\n", "t = (a.mean() - b.mean())/(s*numpy.sqrt(2/N))\n", "\n", "# Compare with the critical t-value\n", "# Degrees of freedom\n", "df = 2*N - 2\n", "\n", "#p-value after comparison with the t \n", "p = 1 - stats.t.cdf(t,df=df)\n", "\n", "print(\"t = \" + str(t))\n", "print(\"p = \" + str(2*p))\n", "# You can see that after comparing the t statistic with the critical t value (computed internally) we get a good p value of 0.0005 and thus we reject the null hypothesis and thus it proves that the mean of the two distributions are different and statistically significant.\n", "\n", "## Cross Checking with the internal scipy function\n", "t2, p2 = stats.ttest_ind(a,b)\n", "print(\"t = \" + str(t2))\n", "print(\"p = \" + str(p2))\n", "\n", "x = numpy.linspace(-5,5,1000)\n", "\n", "plt.plot(x,stats.t.pdf(x,df=df))\n", "plt.scatter(numpy.array([t]),numpy.array([0]),color='red')\n", "plt.show()" ] }, { "cell_type": "markdown", "metadata": {}, "source": [ "# Bayesian Approach in a Gaussian world -- the Kalman filter" ] }, { "cell_type": "code", "execution_count": 10, "metadata": {}, "outputs": [ { "data": { "image/png": "iVBORw0KGgoAAAANSUhEUgAAAXcAAAD8CAYAAACMwORRAAAABHNCSVQICAgIfAhkiAAAAAlwSFlzAAALEgAACxIB0t1+/AAAADl0RVh0U29mdHdhcmUAbWF0cGxvdGxpYiB2ZXJzaW9uIDMuMC4zLCBodHRwOi8vbWF0cGxvdGxpYi5vcmcvnQurowAAIABJREFUeJzt3WlwXWed5/HvX/u+2ZIteZOTOAkO6UqC4iaErSFNEuhKgAbGqRoaBpgUS6q7q2eoTgYqRYUXLKmBqq7KdAjdmaZpmEDTLAbMmLCkgR4IVjYbx7GjeIllydb1otXa9cyLe45yfX2leySdRbr6fapUvvfcR/f+dXT9u4+e85zzmHMOEREpLEVJFyAiIuFTuIuIFCCFu4hIAVK4i4gUIIW7iEgBUriLiBQghbuISAFSuIuIFCCFu4hIASpJ6oXXrl3r2tvbk3p5EZEV6amnnjrjnGvO1y6xcG9vb6ezszOplxcRWZHM7HiQdhqWEREpQAp3EZECpHAXESlACncRkQKkcBcRKUAKdxGRAqRwFxEpQAp3kSU42T/Kw//+EkdSw0mXInIRhbvIIg1cmOR9D/+Wz//kBd73ld9ybmQi6ZJEZincRRbp0f84ysn+UT737mvpvzDJ3z/RlXRJIrMU7iKLMDPjeGzvy7zl6hbu2rGZP92+ju881c3E1EzSpYkACneRRXnmxHlOD45z53VtALzr+g2cvzDJ3mPnEq5MJE3hLrIIvzp8BjN481UtANx8xVrKiot44lBfwpWJpCncRRah8/g5XrW+jvrKUgCqy0u4YUsDTx5Vz12WB4W7yAJNTs/wzMv93NjeeNH26zY1crB3kLHJ6YQqE3mFwl1kgQ72DnJhYpqO9qaLtl+3qYHJaceBnsGEKhN5hcJdZIH2nxwA0mGe6frN6fvPneiPvSaRbAp3kQU6dGqImvISNjZWXrR9XV0F6+sqZsNfJEkKd5EFeqF3iKvW12Jmlzx25fpaDp8eSqAqkYsp3EUWwDnHC6cGuXp9bc7Hr2ypoatvmOkZF3NlIhdTuIssQO/AGINjU1zdWpfz8W3rahifmqH7/IWYKxO5mMJdZAG6+tJXf9zWUpPz8W3r0j36w6d1lUhJlsJdZAGOnx0BYOva6pyPX+GFvsbdJWkKd5EFOHrmApWlxbTUlud8vK6ilOba8tkPAZGkKNxFFuD42RG2rKnKOVPGt6WpiuNnNeYuyVK4iyzA0bMjtK/JPSTj27ymipfPKdwlWQp3kYCmZxwnzl2gfY7xdt/mpipODY7pGjOSqEDhbma3mdkhM+sys3tzPP5BM0uZ2bPe10fCL1UkWT39o0xOO9rXVM3bbsuaKpyD7vOjMVUmcqm84W5mxcBDwO3AduAuM9ueo+m3nHPXeV//EHKdIok75h0kzd9zTz/+8jkdVJXkBOm57wC6nHNHnHMTwGPAndGWJbL8HPMOkm4J0HMHdFBVEhUk3DcAJzLud3vbsv25me0zs++Y2aZQqhNZRnr6RyktNtbVVszbbk11GdVlxQp3SVSQcM815yv7whk/BNqdc38E/Az4Ws4nMrvbzDrNrDOVSi2sUpGE9fSPsr6+gqKiuadBApgZm5o0Y0aSFSTcu4HMnvhGoCezgXPurHNu3Lv7VeA1uZ7IOfeIc67DOdfR3Ny8mHpFEtPTP0pbfWX+hsDGxip6+nVAVZITJNz3AtvMbKuZlQE7gV2ZDcysNePuHcDB8EoUWR56+sfY0BAs3NsaKhTukqiSfA2cc1Nmdg+wBygGHnXOHTCzB4BO59wu4C/N7A5gCjgHfDDCmkViNzU9w6nBMdoChntrfSWDY1OMjE9RXZ73v5lI6AK965xzu4HdWdvuz7h9H3BfuKWJLB99Q+NMz7jA4d7WkD7o2jswyhUtua/9LhIlnaEqEkDvQHqIxQ/tfNbXpdv19I9FVpPIfBTuIgGc9EI6+Jh7up3/oSASN4W7SAD+wdHWgOG+rq4CM/XcJTkKd5EAevpHqa8spSbgwdGykiLW1pSr5y6JUbiLBNDTPxr4YKqvrb6C3gH13CUZCneRAHr6x2irD3Yw1ddaX6lwl8Qo3EUC6BkYpTXgTBlfa0MFvf2jOJd9tQ6R6CncRfIYn5qm/8Lk7PTGoNrqKxmZmGZwbCqiykTmpnAXyePM8AQAzXMsij2X1owTmUTipnAXyaNvMD1u3pLnUr/ZWuv9cNe4u8RP4S6SR2oofcHThfbc/Q+D1OB4npYi4VO4i+SRGl5cuPvtTw+q5y7xU7iL5NE3OI4ZNFWXLej7KkqLqa8spW9IPXeJn8JdJI/U8DhNVWWUFi/8v8u6unL6htRzl/gp3EXySA2NL3hIxtdSW8FpjblLAhTuInksLdzLZw/IisRJ4S6Sx5LCva6C1NC4zlKV2CncRebhnFtyz31ieob+C5MhVyYyP4W7yDwGR6eYmJ5Z8AlMvpY6bzqkDqpKzBTuIvNIDadDeSkHVCE9nVIkTgp3kXn4odxcs7hwX+f13DXXXeKmcBeZx2LPTvX5PXedpSpxU7iLzMOfxuiPnS9UZVkxteUlmg4psVO4i8wjNTROeUkRtQHXTs2lRWepSgIU7iLz6POmQZrZop+jpbZCB1QldoHC3cxuM7NDZtZlZvfO0+49ZubMrCO8EkWSs5Q57r6WunJNhZTY5Q13MysGHgJuB7YDd5nZ9hztaoG/BJ4Mu0iRpKSGxhc9U8a3ri7dc9dZqhKnID33HUCXc+6Ic24CeAy4M0e7zwJfBNRFkYKRGh5f9MFUX0ttOeNTM1pLVWIVJNw3ACcy7nd722aZ2fXAJufcj0KsTSRRE1MznBuZoLlmcWen+vxhnZSGZiRGQcI915Gk2b8vzawI+DLw3/I+kdndZtZpZp2pVCp4lSIJODuytDnuPv/7dSKTxClIuHcDmzLubwR6Mu7XAq8GnjCzY8BrgV25Dqo65x5xznU45zqam5sXX7VIDBa7dmq22bVUFe4SoyDhvhfYZmZbzawM2Ans8h90zg0459Y659qdc+3A74A7nHOdkVQsEpPZE5hC6rkr3CVOecPdOTcF3APsAQ4C33bOHTCzB8zsjqgLFElKX0g997qKEspKihTuEqtAp90553YDu7O23T9H2zcvvSyR5PlhvKZmYQtjZzMzWmrLNeYusdIZqiJzSA2N01BVSnlJ8ZKfq1nL7UnMFO4icwjjBCZfc43CXeKlcBeZQ9/Q2JJPYPLp4mESN4W7yBxSw2H23Cs4f2GSiamZUJ5PJB+Fu0gOS10YO5v/F8CZYQ3NSDwU7iI5DI9PMTY5E1q4+38BaNxd4qJwF8mhb/YEpqVdV8anE5kkbgp3kRzCuvSAr0ULZUvMFO4iOYQd7muq1XOXeCncRXKYDfeQZsuUlRTRVF2m6ZASG4W7SA59Q+OUFhsNVaWhPadOZJI4KdxFcvDPTl3KwtjZmmvLSWkqpMRE4S6SQ2o4vDnuvpbacvoGFe4SD4W7SA5hnsDk83vuWihb4qBwF8khNTQWSbhPaKFsiYnCXSTL1PQMZ0cmaA7pBCafFsqWOCncRbKcG5nAufDmuPu0ULbESeEukqUv5DnuPi2ULXFSuItkCfvsVJ+uLyNxUriLZEnNXjQs3HDXQtkSJ4W7SBb/RKOwe+5aKFvipHAXyZIaGqe2ooSK0qUvjJ1NC2VLXBTuIln6Ipjj7kv33DUVUqKncBfJkhoaD3283aeeu8RF4S6SJX3pgXBPYPJpoWyJi8JdJEufd0XIKGihbIlLoHA3s9vM7JCZdZnZvTke/6iZ7TezZ83sN2a2PfxSRaI3Mj7FhYnpyMbctVC2xCVvuJtZMfAQcDuwHbgrR3h/0zl3rXPuOuCLwJdCr1QkBlGdwOTTJQgkLkF67juALufcEefcBPAYcGdmA+fcYMbdakDXNJUVyZ/jHtUBVX9YRj13iVpJgDYbgBMZ97uBP85uZGafAP4GKAPeEkp1IjHzF9OIqueuhbIlLkF67rnWGbukZ+6ce8g5dznwt8Cncz6R2d1m1mlmnalUamGVisTAvxxvVOGuhbIlLkHCvRvYlHF/I9AzT/vHgHfmesA594hzrsM519Hc3By8SpGYpIbHKS4ymqrKInsNLZQtcQgS7nuBbWa21czKgJ3ArswGZrYt4+47gBfDK1EkPqmhcdbWlFFUFN7C2Nm0ULbEIe+Yu3NuyszuAfYAxcCjzrkDZvYA0Omc2wXcY2a3AJPAeeADURYtEpW+oXHWRjTH3ddSW86TR0cifQ2RIAdUcc7tBnZnbbs/4/ZfhVyXSCL6BsdZVxdtuGculG0W3V8IsrrpDFWRDKnh8dkVk6Iyu1D2qBbKlugo3EU80zOOs8Pjs3PRozK7ItOwZsxIdBTuIp6zw+PMuOhOYPLpLFWJg8JdxDO7MHbEwzJaKFvioHAX8fgnFsU2LKNwlwgp3EU8/qUHoh6W8RfK1rCMREnhLuLxwzbqee7+QtnquUuUFO4inr6hMeorSyNZGDubltuTqCncRTxRrp2aTQtlS9QU7iKevqHo57j71HOXqCncRTx9g9GfnerTQtkSNYW7COCci3dYRgtlS8QU7iLAwOgkE9MzkS3SkU0LZUvUFO4ivDINsqUunmEZv+euue4SFYW7CPGdwOTTWaoSNYW7CK9coTGuYRktlC1RU7iLEH/PXQtlS9QU7iKkx74rS4upKQ+0OFkotFC2REnhLsIrJzDFuexdS125DqhKZBTuIkDf4FhsQzI+9dwlSgp3EfzrysQzDdKXuVC2SNgU7rLqOec4NTjGupjmuPu0ULZESeEuq97g2BQXJqZprY8/3EELZUs0FO6y6p0eTIfr+oTC3Z+GKRImhbuser0D6XCPu+c+u1C2Lh4mEQgU7mZ2m5kdMrMuM7s3x+N/Y2bPm9k+M/u5mW0Jv1SRaJwaGAWS67lrxoxEIW+4m1kx8BBwO7AduMvMtmc1ewbocM79EfAd4IthFyoSld6BMcyIfbZMXUUJ5VooWyISpOe+A+hyzh1xzk0AjwF3ZjZwzv3SOXfBu/s7YGO4ZYpE59TAGGtryikriXeU0sy0IpNEJsi7eQNwIuN+t7dtLh8GfrKUokTi1DswFvt4u69Za6lKRIKEe67zsXOedWFm/xnoAB6c4/G7zazTzDpTqVTwKkUidGog/jnuvnW1FZwaULhL+IKEezewKeP+RqAnu5GZ3QJ8CrjDOZfz70zn3CPOuQ7nXEdzc/Ni6hUJXe/AaGI997aGSnoHxnSWqoQuSLjvBbaZ2VYzKwN2ArsyG5jZ9cBXSAd7X/hlikTjwsQUg2NTsc+U8bU1VHBhYlpnqUro8oa7c24KuAfYAxwEvu2cO2BmD5jZHV6zB4Ea4F/N7Fkz2zXH04ksK6cSmuPua2uoBOBk/2giry+FK9DFq51zu4HdWdvuz7h9S8h1icTCD/f1dZWJvL4f7j39o2xvq0ukBilMOkNVVrWkzk71tTWkX7dnQD13CZfCXVa1UwldV8a3trqcsuIiDctI6BTusqr1DozSWFVKRWlxIq9fVGS0NlTQ06/pkBIuhbusar39yc1x97XWV9CrnruETOEuq9rJ/lE2NlYlWkNbQyU9CncJmcJdVi3nHCfPj7KxMZmZMr4NDZWcGhxjanom0TqksCjcZdUaHJ1iaHwq8XBva6hkxsFpXUBMQqRwl1XrxPn0hUyXQ7gDGpqRUCncZdXqPp8O06TH3Df4c90V7hIihbusWt3LpOfeWu/33DUdUsKjcJdVq/v8KDXlJdRXliZaR7VXw8n+C/kbiwSkcJdVq/v8KBsaKjHLtWRBvDY2Vs4OE4mEQeEuq1Z6jnuyQzK+LWuqePmseu4SHoW7rFrd5y8sm3Df1FTFifMXmJ7Roh0SDoW7rEoDo5MMjU0lPlPGt6WpmslpN3shM5GlUrjLquTPlNmwTHruW9akP2SOnx1JuBIpFAp3WZWOe+Pbm5uWR8/dr+PEOY27SzgU7rIqHT2T7iG3r61OuJK01voKSops9kNHZKkU7rIqHT0zQnNtOTXlgVaajFxJcREbGys5rp67hEThLqvSsTMjbF2zPHrtvk1NVRqWkdAo3GVVOnZ2hK3LZEjGt2VNlYZlJDQKd1l1hsYmOTM8sWzG232bm6oYGJ1k4MJk0qVIAVC4y6pz7Ey6d7x17fKYKeNr94aJjmo6pIRA4S6rjh+ey63nfnlLDQBdfcMJVyKFQOEuq84xbxrklqblFe5bmqooLTaFu4RC4S6rzrEzI7TWV1BZVpx0KRcpKS6ifU21wl1CESjczew2MztkZl1mdm+Ox99oZk+b2ZSZvSf8MkXC89KZ5TdTxndFSw0vpRTusnR5w93MioGHgNuB7cBdZrY9q9nLwAeBb4ZdoEiYnHN0nR7iynW1SZeS0xUtNRw/O8L41HTSpcgKF6TnvgPocs4dcc5NAI8Bd2Y2cM4dc87tA2YiqFEkNCf7RxmZmGbbupqkS8npipYaZtwrM3pEFitIuG8ATmTc7/a2LZiZ3W1mnWbWmUqlFvMUIkty+PQQAFct05775c2aMSPhCBLuudYgW9SKAs65R5xzHc65jubm5sU8hciSHD6dDs1tyzjczRTusnRBwr0b2JRxfyPQE005ItE6fHqIdXXliS+KPZfKsmI2NlbyYt9Q0qXIChck3PcC28xsq5mVATuBXdGWJRKNw8v4YKrv6vV1HOwdTLoMWeHyhrtzbgq4B9gDHAS+7Zw7YGYPmNkdAGZ2o5l1A+8FvmJmB6IsWmQxpqZn6OobXvbhvr21jiNnRrgwMZV0KbKCBbqYtXNuN7A7a9v9Gbf3kh6uEVm2XkqNMDY5w6s31CVdyryuaavDOTjYO8RrtjQmXY6sUDpDVVaN/ScHALh2Q33ClczvGq++5zU0I0ugcJdV4w8nB6gqK2br2uU5x93XVl9BfWUpz/cMJF2KrGAKd1k19p8c4Jq2OoqLcs3uXT7MjGva6jjQo567LJ7CXVaF6RnH8z2DvHqZD8n4rt1Yzwu9Q4xN6jIEsjgKd1kVuvqGGZ2cXvbj7b4bNjcyMT3DAQ3NyCIp3GVV2HvsHMCKmX1yw+Z0nU8dP59wJbJSKdxlVdh77BzNteVsblpeS+vNpbm2nC1rqhTusmgKd1kVOo+dZ0d7E2bL+2BqptdsbuTpl/txblGXcpJVTuEuBe9k/ygn+0fpaF8ZQzK+G7Y0khoa59hZXf5XFk7hLgXv90fPAnBje1PClSzMzVesBeA3XWcSrkRWIoW7FLxfHT5DU3UZr2pd3pcdyNa+poqNjZX8+rDWPpCFU7hLQZuZcfz74RRv3LZ22Z+8lM3MeMO2tfz2pbNMTmuRM1kYhbsUtH0nBzg3MsGfXN2SdCmL8oZtzQyNT/Hsif6kS5EVRuEuBe2JQ32YpUNyJbr5irWUFhs/PXAq6VJkhVG4S8FyzvHjfb10bGmkqbos6XIWpb6ylDdsa2b3/lOaEikLonCXgnWwd4gX+4a547pFree+bLz92lZO9o9qaEYWROEuBesHz52kpMh4x7WtSZeyJH+6fR2lxcYPntXSxRKcwl0K0sTUDN97+iRvvLJ5xQ7J+OorS7nt1a3829PdWnpPAlO4S0Havb+XvqFx3n/TlqRLCcX7X7uFobEpfviceu8SjMJdCo5zjkf/4yiXNVfzphU6Sybbje2NXL2+lkd/c4yZGR1YlfwU7lJwfnawj33dA3zk9ZdRtMJOXJqLmfHxP7mCQ6eH+OE+9d4lP4W7FJTJ6Rk+t/sglzVX896OjUmXE6o/u7aVq9fX8qXHD2uFJslL4S4F5aFfdnHkzAj/4/ZXUVpcWG/voiLj0+/YzvGzF/jy44eTLkeWucJ698uq9vuj5/i7n7/Iu67fwC3b1yVdTiRev20td+3YzCO/PsITh/qSLkeWMYW7FITnewb5yNf2smVNNQ/ceU3S5UTq0+94FVevr+MT33ia53Rik8whULib2W1mdsjMuszs3hyPl5vZt7zHnzSz9rALFZnL48+f5j995bdUl5fw9Q/voLaiNOmSIlVdXsL//uCNNFaXsfOR3/Hjfb1JlyTLUN5wN7Ni4CHgdmA7cJeZbc9q9mHgvHPuCuDLwBfCLlQk2/7uAT72L0/xX/+5k81rqvjOx17Hxsb41kj9/jMnufnzv2DrvT/m5s//gu8/czK2115fX8F3P/46rlxfyye++TQf+dpeXZ5ALmL5LkZkZjcBn3HO3erdvw/AOfe5jDZ7vDa/NbMS4BTQ7OZ58o6ODtfZ2RnCjyCFbnJ6hoHRSU4NjPFSapj93QP8+sUzHDo9RFVZMR990+Xc/cbLqCgtjq2m7z9zkvu+u5/RjFkrlaXFfO7d1/LO6+O7ls3k9Axf/fURHn7iJQbHpri8uZo3XdnCqzfUceW6Wlpqy2mqLqOkwA4ur2Zm9pRzriNfu5IAz7UBOJFxvxv447naOOemzGwAWAOEvj7Yd5/u5p/+37GLtmV/hDgu/Uy5pE2Oj53sTUGvwreo18/5PC5vm+yNQZ4nV7vcP7/L3ybALgnz9UfGpxiZuHjaX1lJETe2N7Jzx3b+/DUbqUtgGObBPYcuCnaA0clpHtxzKNZwLy0u4uNvvoK/uKmd7z3dzf89cIp/efI4E1OvLO5hBjVlJZSXFlFeUkxZSRGlxYZhZK4XbmaY197/Pr9NYZwtsHx89E2Xc3vE1zwKEu65fq/Z/zWDtMHM7gbuBti8eXOAl75UZWkxa3JcKyR7VftcBV268P2lrbLbBHue9H+CvG0uee6cTxTg9XPUHajGhT9P7hID/KyXbgr082e3qS4vob6ylIaqUppryrmsuYYta6pi7aXn0tM/uqDtUaspL+H9N7Xz/pvamZqe4eiZEV5KDZMaGic1PMHQ2CQTUzOMe1+TUzM43OyHrMP/wE1vcM7flqurIksVx/s3SLh3A5sy7m8Esk+R89t0e8My9cC57Cdyzj0CPALpYZnFFHz7ta2Rf+KJ5NPWUMnJHEHe1lCZQDUXKykuYtu6Wratq026FElQkIG4vcA2M9tqZmXATmBXVptdwAe82+8BfjHfeLvISvfJW6+iMqv3VVlazCdvvSqhikQulrfn7o2h3wPsAYqBR51zB8zsAaDTObcL+Efg62bWRbrHvjPKokWS5o+rP7jnED39o7Q1VPLJW6+KdbxdZD55Z8tERbNlREQWLuhsGc2PEhEpQAp3EZECpHAXESlACncRkQKkcBcRKUAKdxGRAqRwFxEpQAp3EZEClNhJTGaWAo4v8tvXEsEVJ0OguhZGdS3ccq1NdS3MUura4pxrztcosXBfCjPrDHKGVtxU18KoroVbrrWproWJoy4Ny4iIFCCFu4hIAVqp4f5I0gXMQXUtjOpauOVam+pamMjrWpFj7iIiMr+V2nMXEZF5LNtwN7P3mtkBM5sxs46sx+4zsy4zO2Rmt87x/VvN7Ekze9HMvuWtIhV2jd8ys2e9r2Nm9uwc7Y6Z2X6vXeQXsTezz5jZyYza3j5Hu9u8fdhlZvfGUNeDZvaCme0zs++ZWcMc7WLZX/l+fjMr937HXd57qT2qWjJec5OZ/dLMDnrv/7/K0ebNZjaQ8fu9P+q6vNed9/diaX/n7a99ZnZDDDVdlbEfnjWzQTP766w2se0vM3vUzPrM7A8Z25rM7HEvix43s8Y5vvcDXpsXzewDudosiHNuWX4BrwKuAp4AOjK2bweeA8qBrcBLQHGO7/82sNO7/TDwsYjr/Z/A/XM8dgxYG+O++wzw3/O0Kfb23WVAmbdPt0dc19uAEu/2F4AvJLW/gvz8wMeBh73bO4FvxfC7awVu8G7XAodz1PVm4EdxvZ+C/l6AtwM/Ib0++muBJ2Ourxg4RXoeeCL7C3gjcAPwh4xtXwTu9W7fm+t9DzQBR7x/G73bjUupZdn23J1zB51zh3I8dCfwmHNu3Dl3FOgCdmQ2MDMD3gJ8x9v0NeCdUdXqvd77gP8T1WtEYAfQ5Zw74pybAB4jvW8j45z7qXNuyrv7O9KLrSclyM9/J+n3DqTfS2/1fteRcc71Ouee9m4PAQeBlbJ2353AP7u03wENZhbnavZvBV5yzi325Mglc879ivRSo5ky30dzZdGtwOPOuXPOufPA48BtS6ll2Yb7PDYAJzLud3Ppm38N0J8RJLnahOkNwGnn3ItzPO6An5rZU2Z2d4R1ZLrH+9P40Tn+DAyyH6P0IdK9vFzi2F9Bfv7ZNt57aYD0eysW3jDQ9cCTOR6+ycyeM7OfmNk1MZWU7/eS9HtqJ3N3sJLYX751zrleSH94Ay052oS+7/IukB0lM/sZsD7HQ59yzv1grm/LsS17yk+QNoEErPEu5u+13+yc6zGzFuBxM3vB+4RftPnqAv4e+Czpn/mzpIeMPpT9FDm+d8lTp4LsLzP7FDAFfGOOpwl9f+UqNce2yN5HC2VmNcC/AX/tnBvMevhp0kMPw97xlO8D22IoK9/vJcn9VQbcAdyX4+Gk9tdChL7vEg1359wti/i2bmBTxv2NQE9WmzOk/yQs8XpcudqEUqOZlQDvBl4zz3P0eP/2mdn3SA8JLCmsgu47M/sq8KMcDwXZj6HX5R0o+jPgrc4bbMzxHKHvrxyC/Px+m27v91zPpX9yh87MSkkH+zecc9/Nfjwz7J1zu83sf5nZWudcpNdQCfB7ieQ9FdDtwNPOudPZDyS1vzKcNrNW51yvN0zVl6NNN+ljA76NpI83LtpKHJbZBez0ZjJsJf0J/PvMBl5o/BJ4j7fpA8Bcfwks1S3AC8657lwPmlm1mdX6t0kfVPxDrrZhyRrnfNccr7cX2GbpWUVlpP+k3RVxXbcBfwvc4Zy7MEebuPZXkJ9/F+n3DqTfS7+Y6wMpLN6Y/j8CB51zX5qjzXp/7N/MdpD+f3w24rqC/F52AX/hzZp5LTDgD0fEYM6/npPYX1ky30dzZdEe4G1m1ugNo77N27Z4cRxBXswX6VDqBsaB08CejMc+RXqmwyHg9oztu4E27/ZlpEO/C/hXoDyiOv8J+GjWtjZgd0Ydz3lfB0gPT0S9774O7Af2eW+s1uy6vPtvJz0b46WY6uoiPa74rPf1cHZdce6vXD8/8ADpDx+ACu+90+W9ly6LYR+9nvSf4/tiC7bLAAAAj0lEQVQy9tPbgY/67zPgHm/fPEf6wPTrYqgr5+8lqy4DHvL2534yZrlFXFsV6bCuz9iWyP4i/QHTC0x6+fVh0sdpfg686P3b5LXtAP4h43s/5L3XuoD/stRadIaqiEgBWonDMiIikofCXUSkACncRUQKkMJdRKQAKdxFRAqQwl1EpAAp3EVECpDCXUSkAP1/Zku+Vmxhsf8AAAAASUVORK5CYII=\n", "text/plain": [ "
" ] }, "metadata": { "needs_background": "light" }, "output_type": "display_data" }, { "data": { "image/png": "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\n", "text/plain": [ "
" ] }, "metadata": { "needs_background": "light" }, "output_type": "display_data" }, { "data": { "image/png": "iVBORw0KGgoAAAANSUhEUgAAAXcAAAD8CAYAAACMwORRAAAABHNCSVQICAgIfAhkiAAAAAlwSFlzAAALEgAACxIB0t1+/AAAADl0RVh0U29mdHdhcmUAbWF0cGxvdGxpYiB2ZXJzaW9uIDMuMC4zLCBodHRwOi8vbWF0cGxvdGxpYi5vcmcvnQurowAAIABJREFUeJzt3XuQXGd55/HvM/cZaS66jCTrZglbdqy1AcOsgDUQFmwsG0pid0kiV1HhlqjYjUiyZCnsddahTP4JriRV2VWWCEJIqBDZIYbMElGCgLMLBIwG32VZ9li+aCRZGo3mqrnPPPtHnzNqt3o0p2dOd5/u/n2qptR9zjvnPHOm9Zu33z7nPebuiIhIeakqdgEiIhI/hbuISBlSuIuIlCGFu4hIGVK4i4iUIYW7iEgZUriLiJQhhbuISBlSuIuIlKGaYu149erVvmXLlmLtXkSkJP3iF7847+7tC7UrWrhv2bKFrq6uYu1eRKQkmdkrUdppWEZEpAwp3EVEypDCXUSkDCncRUTKUKRwN7OdZnbczLrN7O4s6zeb2SNm9riZPWVmd8ZfqoiIRLVguJtZNbAfuAPYDtxlZtszmv0+8JC73wzsAf487kJFRCS6KD33HUC3u59w90ngILA7o40DLcHjVuB0fCWKiEiuooT7BuBk2vOeYFm6zwMfMbMe4BDw6ViqEymif3rqDI8cP1fsMkQWJUq4W5ZlmTdevQv4mrtvBO4Evm5ml23bzPaaWZeZdfX29uZerUiBPHL8HL/1jcf4+F8d4fFX+4tdjkjOooR7D7Ap7flGLh92+STwEIC7/xRoAFZnbsjdD7h7h7t3tLcvePWsSNE8+POTLK+vobbaePixU8UuRyRnUcL9CLDNzLaaWR2pD0w7M9q8CrwPwMxuIBXu6ppLSZqZdf71xfPcedM63nP9Gn743DncM9+siiTbguHu7tPAPuAwcIzUWTFHzex+M9sVNPs94DfN7Eng74CPuf43SIl69vQQQ+PT3HLtat61bTWnBsY4Mzhe7LJEchJp4jB3P0Tqg9L0ZfelPX4WuCXe0kSK49iZIQDetLGNvosTc8vWtzUWsyyRnOgKVZEMx14borG2ms0rm7h+XQtmqd68SClRuItkOP7aMNeta6aqylheX8PmlU0899pwscsSyYnCXSTDy+cvck37srnnW1Yt45ULF4tYkUjuFO4iaSanZzkzNM6mFU1zy7asauKV86M6Y0ZKisJdJM2ZwTHcYeOKSx+eXr1qGcMT01y4OFnEykRyo3AXSdPTPwbAppWXeu5Xr0o9fuXCaFFqElkMhbtImpNBgKf33DcEj88M6Fx3KR0Kd5E0Pf1jVFcZ61oa5paFj88MjhWrLJGcKdxF0vT0j7K+rYGa6kv/NVoba6mvqeLskHruUjoU7iJpTg+Ms7719VeimhlXtTZoCgIpKQp3kTRnh8dZkzYkE1rb0qCeu5QUhbtImt7hCdY011+2XD13KTUKd5HAyMQ0o5MzWcN9bWsD54YmmJ3VhUxSGhTuIoFzwbDLmpYsPfeWBiZnZrkwqguZpDQo3EUC54ZT0/u2L88+5g5o3F1KhsJdJBCGe7ae+6rlqWWagkBKRaRwN7OdZnbczLrN7O4s6//UzJ4Ivp43s4H4SxXJr94w3LOMua9aXgdA34jCXUrDgndiMrNqYD9wG6mbZR8xs87g7ksAuPt/TWv/aeDmPNQqklfnhsepq66itbH2snWrlqXC/fzIRKHLElmUKD33HUC3u59w90ngILD7Cu3vInUfVZGS0js0QXtzPWZ22bqWhlpqqkzDMlIyooT7BuBk2vOeYNllzOxqYCvww3nW7zWzLjPr6u3tzbVWkbzquzg5N/ySqarKWLmsTsMyUjKihPvl3RiY72TfPcA33X0m20p3P+DuHe7e0d7eHrVGkYLoH51kRVP2cIfUh6rhDbNFki5KuPcAm9KebwROz9N2DxqSkRJ14eLk3Nh6NquW1dGnYRkpEVHC/Qiwzcy2mlkdqQDvzGxkZtcDK4CfxluiSGFcuDjJiiuF+3INy0jpWDDc3X0a2AccBo4BD7n7UTO738x2pTW9CzjoutGklKDxqRlGJ2dYecWeez19OltGSsSCp0ICuPsh4FDGsvsynn8+vrJECqs/mFbgymPudVycnGF8aoaG2upClSayKLpCVYRLFyddueceXMikcXcpAQp3ES713K8Y7sEUBBqakVKgcBfh0pwxK5ddfnVqaKV67lJCFO4iQP9cuF8+r0xoteaXkRKicBch1XM3I+u8MqG24MPWAc3pLiVA4S4CXBidpK2xluqqbBdkpzTX11BlMDA6VcDKRBZH4S4C9F+cuuKHqZCaX6a1sZaBMfXcJfkU7iJA38WJBcMdUufBq+cupUDhLkKq536lC5hCrU21DI4p3CX5FO4iLDwjZKitsVY9dykJCncRYHBsiram+c+UCbU11WnMXUqCwl0q3vjUDBPTs7Rc4TTIUKt67lIiFO5S8cIx9Cud4x5qa6pleHya6ZnZfJclsiQKd6l4YbhHGZYJx+X1oaokncJdKl6uPXeAAYW7JFykcDeznWZ23My6zezuedr8qpk9a2ZHzewb8ZYpkj/hGHqUcA/baNxdkm7Bm3WYWTWwH7iN1P1Uj5hZp7s/m9ZmG3APcIu795vZmnwVLBK33Hru4bCMzpiRZIvSc98BdLv7CXefBA4CuzPa/Caw3937Adz9XLxliuTP3Jh7Y7Tz3EE9d0m+KOG+ATiZ9rwnWJbuOuA6M/uJmf3MzHbGVaBIvg2OpmaEbG5Y+K6Tc2PuCndJuCj3UM02TV7mTbBrgG3Ae4CNwI/M7EZ3H3jdhsz2AnsBNm/enHOxIvkwODaVmvHxCjNChloaajHTtL+SfFF67j3AprTnG4HTWdr8o7tPuftLwHFSYf867n7A3TvcvaO9vX2xNYvEKnV16sJDMpA+M6R67pJsUcL9CLDNzLaaWR2wB+jMaPNt4N8DmNlqUsM0J+IsVCRfBsemIn2YGtL8MlIKFgx3d58G9gGHgWPAQ+5+1MzuN7NdQbPDQJ+ZPQs8AnzW3fvyVbRInAZyDPfWpjr13CXxooy54+6HgEMZy+5Le+zAZ4IvkZIyODbF+rbGyO1TPXeNuUuy6QpVqXhDuQ7LNGnMXZJP4S4Vzd0ZGM19zL3/onrukmwKd6loo5MzTM96bmPujbUMT0wzO5t5RrBIcijcpaJdujo1eri3NNbiDsPj0/kqS2TJFO5S0XKZNCzUpml/pQQo3KWi5TJpWGhuZkhNHiYJpnCXijYX7hFu1BEKw109d0kyhbtUtKFF9NzDycMU7pJkCnepaOHQymKGZRTukmQKd6log2NTVFcZy+sjXawN6G5MUhoU7lLRwknDzBae7jfUUFtNXU3V3JCOSBIp3KWi5Xp1aqitsVbDMpJoCnepaINjU7QsItxbNe2vJJzCXSra0NhUTlenhlrVc5eEU7hLRcv1Rh2htiaFuyRbpHA3s51mdtzMus3s7izrP2ZmvWb2RPD1G/GXKhK/XG/UEWpRz10SbsHzv8ysGtgP3EbqXqlHzKzT3Z/NaPqgu+/LQ40ieTE76znP5R7SsIwkXZSe+w6g291PuPskcBDYnd+yRPJvZHKaWc/tAqZQa2MtIxPTTM/M5qEykaWLEu4bgJNpz3uCZZn+k5k9ZWbfNLNNsVQnkkeDi5gRMhR+CDukaX8loaKEe7arOzLvUvB/gC3u/kbgn4G/zrohs71m1mVmXb29vblVKhKzcFhlUadCNoVXqWpmSEmmKOHeA6T3xDcCp9MbuHufu08ET78MvDXbhtz9gLt3uHtHe3v7YuoVic3Q+OJ77ppfRpIuSrgfAbaZ2VYzqwP2AJ3pDczsqrSnu4Bj8ZUokh+LmREy1NqoG3ZIsi14toy7T5vZPuAwUA181d2Pmtn9QJe7dwK/bWa7gGngAvCxPNYsEovFzOUeUs9dki7SVHjufgg4lLHsvrTH9wD3xFuaSH7Njbk3RJ8RMqRwl6TTFapSsRYz3W9oLtw1v4wklMJdKtbQ2DQtDTU5TfcbqqupoqmuWj13SSyFu1Ssxc4rE9JVqpJkCnepWHGE+4DCXRJK4S4Va7FzuYfUc5ckU7hLxRqKIdx1qz1JKoW7VKyh8RiGZXS2jCSUwl0qkrvrA1Upawp3qUhjUzNMzfiSwr2tqZaxqRkmpzXtrySPwl0q0qWrU5fWc0/flkiSKNylIg0uYdKwUMtcuGvaX0kehbtUpKGx1E02ljrmDuq5SzIp3KUixdFzb2vStL+SXAp3qUhxhHv4vTodUpJI4S4V6dIt9nKfETKkYRlJMoW7VKQwkJuXcLZMOA+8wl2SKFK4m9lOMztuZt1mdvcV2n3YzNzMOuIrUSR+Q2NTNDfUUF2V+3S/oZrqKprraxTukkgLhruZVQP7gTuA7cBdZrY9S7tm4LeBR+MuUiRuQ0u8OjXU0lirG3ZIIkXpue8Aut39hLtPAgeB3VnafQH4IjAeY30iebHUqQdCmoJAkipKuG8ATqY97wmWzTGzm4FN7v6dK23IzPaaWZeZdfX29uZcrEhcBsemlnR1aqitSeEuyRQl3LMNSvrcSrMq4E+B31toQ+5+wN073L2jvb09epUiMYuz564bdkgSRQn3HmBT2vONwOm0583AjcC/mNnLwNuBTn2oKkmmYRkpd1HC/Qiwzcy2mlkdsAfoDFe6+6C7r3b3Le6+BfgZsMvdu/JSsUgMhsanaG2KIdw1LCMJtWC4u/s0sA84DBwDHnL3o2Z2v5ntyneBInGbmJ5hfGo2tp775PQs41MzMVQmEp9Il+e5+yHgUMay++Zp+56llyWSP5em+1381amh9CkI1rVWL3l7InHRFapScYbmph6Ip+cOukpVkkfhLhUnjknDQm2NmhlSkknhLhUnjrncQ5eGZXTDDkkWhbtUnDh77hqWkaRSuEvFGYxzzL1J4S7JpHCXihPeXKMthnBvrq/B7NKHtCJJoXCXitM/OklzfQ011Ut/+VdVGS0NmoJAkkfhLhVnYHSStmVL77WHNAWBJJHCXSrOwNgUK4KbW8dBM0NKEincpeL0j8YzaViotbFWN8mWxFG4S8UZGJ2Mtefe0lirD1QlcRTuUnEGRqdYEcOMkKE2jblLAincpaLMzHow3W98PffwA1V3X7ixSIEo3KWipEKYWHvurY21TM86Fyc17a8kh8JdKko4B0ycY+6agkCSKFK4m9lOMztuZt1mdneW9Z8ys6fN7Akz+7GZbY+/VJGl6w/OaonjLkyhtnAKAp0xIwmyYLibWTWwH7gD2A7clSW8v+HuN7n7m4EvAn8Se6UiMchHzz2co2ZgTDNDSnJE6bnvALrd/YS7TwIHgd3pDdx9KO3pMkCfLEkiheejxz3mDppfRpIlyn3GNgAn0573AG/LbGRmvwV8BqgD3htLdSIx6w967uFNNuKgMXdJoig9d8uy7LKeubvvd/drgM8Bv591Q2Z7zazLzLp6e3tzq1QkBgOjU1QZNMdw/9RQW5PuxiTJEyXce4BNac83Aqev0P4g8KFsK9z9gLt3uHtHe3t79CpFYjIwNklbUx1VVdn6LIuzrK6a6irTFASSKFHC/Qiwzcy2mlkdsAfoTG9gZtvSnn4AeCG+EkXi0z86Fcs87unMTDNDSuIs+N7U3afNbB9wGKgGvuruR83sfqDL3TuBfWZ2KzAF9AMfzWfRIos1MDo5d+pinDQFgSRNpIFHdz8EHMpYdl/a49+JuS6RvBgYnWJdS0Ps221RuEvC6ApVqSgDo1OxXsAUamvStL+SLAp3qSj9MU/3G1q5rI4LF3URkySHwl0qxvjUDKOTM6xcFn+4r15ez/mRCc0MKYmhcJeK0Rf0rFflIdxXLqtjYnqWUc0MKQmhcJeK0TcyAcCq5fWxbzv8g9E3oqEZSQaFu1SMsOeej2GZVcuDcL84Efu2RRZD4S4V48JI/oZlVi1LvRtQz12SQuEuFSPsVYe97DiF7wbUc5ekULhLxei7OElddRXL6+ObNCx0aVhGPXdJBoW7VIy+kUlWLa/DLL5Jw0JNdTU01VVrWEYSQ+EuFePCxcm8fJga0oVMkiQKd6kYfRcn83IaZGhVcCGTSBIo3KVi9I1M5OVMmdCqZXUalpHEULhLxcj3sMwqDctIgijcpSKMTabmlcnHaZChlcvr6Luo+WUkGRTuUhHmznHPY8999bJ6pmac4YnpvO1DJKpI4W5mO83suJl1m9ndWdZ/xsyeNbOnzOwHZnZ1/KWKLN75uatT8/eB6krNLyMJsmC4m1k1sB+4A9gO3GVm2zOaPQ50uPsbgW8CX4y7UJGlODc0DsCalnyeLZMK9wu6SlUSIErPfQfQ7e4n3H0SOAjsTm/g7o+4+2jw9GfAxnjLFFmac8OpwF2bh1vshVYHp1n2DqvnLsUXJdw3ACfTnvcEy+bzSeC72VaY2V4z6zKzrt7e3uhViizRueEJzPI75h6+K+gdHs/bPkSiihLu2a7Vzno6gJl9BOgAHsi23t0PuHuHu3e0t7dHr1JkiXqHx1m1rJ6a6vydQ7BqWT3VVcbZIQ3LSPFFmUGpB9iU9nwjcDqzkZndCtwL/LK769UtiXJ2aII1zfkbbweorjLal9dzdkg9dym+KN2YI8A2M9tqZnXAHqAzvYGZ3Qz8BbDL3c/FX6bI0pwbHs/rh6mhtS31nB1W30aKb8Fwd/dpYB9wGDgGPOTuR83sfjPbFTR7AFgO/L2ZPWFmnfNsTqQozhWg5w6wpqVh7swckWKKNLG1ux8CDmUsuy/t8a0x1yUSm5lZ5/zIRF7PlAmtbamn6+ULed+PyEJ0haqUvb6RCWadgvTc1zY30D86xcT0TN73JXIlCncpe+E57u3Nhei5p/ZxTmfMSJEp3KXsnRvO/9WpoXAf53SuuxSZwl3KXnjeeUGGZYKeu851l2JTuEvZOz0wRpXld+qB0KVwV89dikvhLmXv1MAY61oaqM3j1amhFU211FVX8ZrCXYpM4S5l71T/GOvbGguyLzNjfVsDp/rHCrI/kfko3KXsnRoYY8OKwoQ7wMYVTfQo3KXIFO5S1mZmndcGx9lQoJ47wMYVjZwaULhLcSncpaydGx5netYLNiwDqXDvHZ5gfEoXMknxKNylrIVj34Uclgn3pd67FJPCXcpaGLAbC9pzbwLQuLsUlcJdyloY7oUelgF0xowUlcJdytqp/jHammpZVh9pAtRYrGluoKbK6OkfXbixSJ4o3KWsvdI3ytUrmwq6z+oqY31bo4ZlpKgihbuZ7TSz42bWbWZ3Z1n/bjN7zMymzezD8Zcpsjgvnb/I1tXLCr7fTSsbeeWCeu5SPAuGu5lVA/uBO4DtwF1mtj2j2avAx4BvxF2gyGKNT81wenCMLUUI92val3Pi3AjuWe8lL5J3UXruO4Budz/h7pPAQWB3egN3f9ndnwJm81CjyKK8emEUd4rSc7+mfTnDE9P06n6qUiRRwn0DcDLteU+wTCTRTvReBIoX7gDdvSMF37cIRAt3y7JsUe81zWyvmXWZWVdvb+9iNiES2ct9qXAvxrDMtWtS4f5i8AdGpNCihHsPsCnt+Ubg9GJ25u4H3L3D3Tva29sXswmRyF44O0J7cz0tDbUF3/falnqW1VXz4jn13KU4ooT7EWCbmW01szpgD9CZ37JElu742SF+aV1zUfZtZlyzZjkvalhGimTBcHf3aWAfcBg4Bjzk7kfN7H4z2wVgZv/WzHqAXwH+wsyO5rNokYVMz8zy/NmRooU7pMbdu9VzlyKJdNmeux8CDmUsuy/t8RFSwzUiifBy3yiT07Ncv66laDVct7aZbz1+isHRKVqbCj80JJVNV6hKWTr+2jBAUXvuN25I/WF55vRg0WqQyqVwl7L0zOlBaqps7qyVYrhxfSsAT59SuEvhKdylLD3x6gDb17fQUFtdtBpWLKtj44pGhbsUhcJdys7MrPNkzwA3b2ordinctKGVp3oGil2GVCCFu5Sd588OMzo5w5s3Fz/c33r1Ck5eGOO1wfFilyIVRuEuZafr5QsAvGXziiJXAm/bugqAR1/qK3IlUmkU7lJ2fvTCeTauaGRzgedxz2b7+haa62t49KULxS5FKozCXcrK9MwsP32xj3dtW41ZtmmRCqu6ytixdSU/fuG8pv+VglK4S1l5/OQAwxPTvPPa5Mxd9N4b1vDqhVGeP6urVaVwFO5SVr7z5Gnqa6p493Wri13KnNtuWAvA946+VuRKpJIo3KVsTM/M8k9Pn+G9v7SG5iLMBDmfNS0NvPXqFXQ+eVpDM1IwCncpG48c7+X8yCS737y+2KVc5tc6NvHCuRG6XukvdilSIRTuUja+/KMTbGhr5NZgGCRJPvimq2iur+GvfvJSsUuRCqFwl7Lwk+7z/PylC3z8li3UVCfvZd1UV8PHbtnCoadf4xlNRyAFkLz/BSI5Gp+a4Q86j7J5ZRMfefvVxS5nXr/xrjewoqmWe7/9DNMzupe85FekcDeznWZ23My6zezuLOvrzezBYP2jZrYl7kJFspmZdT73D0/xYu8If/ihG4s6UdhCWhtr+cMP3cSTJwf4wnee1YerklcL3qzDzKqB/cBtpO6nesTMOt392bRmnwT63f1aM9sD/BHwa/koWCR0fmSC//7w03zv2bN89vbrefd1hT23/duPn+KBw8c5PTDG+rZGPnv79Xzo5g1X/J4PvPEqHn91K1/58UuMTMzwB7u2F+Uer1L+otyJaQfQ7e4nAMzsILAbSA/33cDng8ffBP6XmZmrayIxmp6ZpXdkgmdPD/F/n+/lW4+dYmJ6lv/xwe188p1bC1rLtx8/xT0PP83Y1AwApwbGuOfhpwEWDPh7P3ADTfU1/M8fvsAPnjvLh9+ykXdd184N65ppb65PxJW1UvqihPsG4GTa8x7gbfO1cfdpMxsEVgHn4ygy3cOP9fC1f335dcsy/4Q4l/9NuaxNlj87mYui/G3Kvh2P0GbhfV22JKbtxHW8Lv+e3Pedbf/Z2szMOhdGJ+fW1VVXccdN6/j0e6/l2jWFv9vSA4ePzwV7aGxqhgcOH18w3M2Mz9x2He/fvpY/+8EL/M1PX+ErP06dRVNdZbQ11rK8oYbqKqPaLPVvlVFlRinmfgmWnHef+uVruOOmq/K6jyjhnu13k/nfL0obzGwvsBdg8+bNEXZ9ucbaalYtq7tseWZvJ1tBl//HuLxVZpso27Eo28myocu+L2ubzO1k2dcC+17sdrLXk3GcI+1rkdux9MfG6uX1rG2p55r25bxpYxuNdcUbXz89MJbT8mxu3NDKgV/vYHh8imdODXH8tSF6RyYYGJ1iZGKamVl/3ddsCb4RLr2KC6MQnw1FCfceYFPa843A6Xna9JhZDdAKXDYNnrsfAA4AdHR0LOr3fsdNV+X9L57IQta3NXIqS5Cvb2vMeVvNDbW845pVvOOaVXGUJgJEO1vmCLDNzLaaWR2wB+jMaNMJfDR4/GHghxpvl3L22duvpzGj99VYW81nb7++SBWJvN6CPfdgDH0fcBioBr7q7kfN7H6gy907gb8Evm5m3aR67HvyWbRIsYXj6rmeLSNSKFasDnZHR4d3dXUVZd8iIqXKzH7h7h0LtdMVqiIiZUjhLiJShhTuIiJlSOEuIlKGFO4iImVI4S4iUoYU7iIiZUjhLiJShop2EZOZ9QKvLPLbV5OHGSdjoLpyo7pyl9TaVFdullLX1e6+4M0LihbuS2FmXVGu0Co01ZUb1ZW7pNamunJTiLo0LCMiUoYU7iIiZahUw/1AsQuYh+rKjerKXVJrU125yXtdJTnmLiIiV1aqPXcREbmCxIa7mf2KmR01s1kz68hYd4+ZdZvZcTO7fZ7v32pmj5rZC2b2YHAXqbhrfNDMngi+XjazJ+Zp97KZPR20y/sk9mb2eTM7lVbbnfO02xkcw24zu7sAdT1gZs+Z2VNm9i0za5unXUGO10I/v5nVB7/j7uC1tCVftaTtc5OZPWJmx4LX/+9kafMeMxtM+/3el++6gv1e8fdiKX8WHK+nzOwtBajp+rTj8ISZDZnZ72a0KdjxMrOvmtk5M3smbdlKM/t+kEXfN7MV83zvR4M2L5jZR7O1yYm7J/ILuAG4HvgXoCNt+XbgSaAe2Aq8CFRn+f6HgD3B4y8B/znP9f4xcN88614GVhfw2H0e+G8LtKkOjt0bgLrgmG7Pc13vB2qCx38E/FGxjleUnx/4L8CXgsd7gAcL8Lu7CnhL8LgZeD5LXe8BvlOo11PU3wtwJ/BdUvdIfzvwaIHrqwZeI3UeeFGOF/Bu4C3AM2nLvgjcHTy+O9vrHlgJnAj+XRE8XrGUWhLbc3f3Y+5+PMuq3cBBd59w95eAbmBHegMzM+C9wDeDRX8NfChftQb7+1Xg7/K1jzzYAXS7+wl3nwQOkjq2eePu33P36eDpz0jdbL1Yovz8u0m9diD1Wnpf8LvOG3c/4+6PBY+HgWNAqdy7bzfwN57yM6DNzAp5N/v3AS+6+2Ivjlwyd/9/pG41mi79dTRfFt0OfN/dL7h7P/B9YOdSaklsuF/BBuBk2vMeLn/xrwIG0oIkW5s4vQs46+4vzLPege+Z2S/MbG8e60i3L3hr/NV53gZGOY759AlSvbxsCnG8ovz8c22C19IgqddWQQTDQDcDj2ZZ/Q4ze9LMvmtm/6ZAJS30eyn2a2oP83ewinG8Qmvd/Qyk/ngDa7K0if3YLXiD7Hwys38G1mVZda+7/+N835ZlWeYpP1HaRBKxxru4cq/9Fnc/bWZrgO+b2XPBX/hFu1JdwP8GvkDqZ/4CqSGjT2RuIsv3LvnUqSjHy8zuBaaBv51nM7Efr2ylZlmWt9dRrsxsOfAPwO+6+1DG6sdIDT2MBJ+nfBvYVoCyFvq9FPN41QG7gHuyrC7W8cpF7MeuqOHu7rcu4tt6gE1pzzcCpzPanCf1lrAm6HFlaxNLjWZWA/xH4K1X2Mbp4N9zZvYtUkMCSwqrqMfOzL4MfCfLqijHMfa6gg+KPgi8z4PBxizbiP14ZRHl5w/b9AS/51Yuf8sdOzOrJRXsf+vuD2euTw97dz9kZn9uZqvdPa9zqET4veTlNRWNPvt3AAAB4klEQVTRHcBj7n42c0Wxjleas2Z2lbufCYapzmVp00Pqs4HQRlKfNy5aKQ7LdAJ7gjMZtpL6C/zz9AZBaDwCfDhY9FFgvncCS3Ur8Jy792RbaWbLzKw5fEzqQ8VnsrWNS8Y453+YZ39HgG2WOquojtRb2s4817UT+Bywy91H52lTqOMV5efvJPXagdRr6Yfz/UGKSzCm/5fAMXf/k3narAvH/s1sB6n/x315rivK76UT+PXgrJm3A4PhcEQBzPvuuRjHK0P662i+LDoMvN/MVgTDqO8Pli1eIT5BXswXqVDqASaAs8DhtHX3kjrT4ThwR9ryQ8D64PEbSIV+N/D3QH2e6vwa8KmMZeuBQ2l1PBl8HSU1PJHvY/d14GngqeCFdVVmXcHzO0mdjfFigerqJjWu+ETw9aXMugp5vLL9/MD9pP74ADQEr53u4LX0hgIco3eSejv+VNpxuhP4VPg6A/YFx+ZJUh9M/7sC1JX195JRlwH7g+P5NGlnueW5tiZSYd2atqwox4vUH5gzwFSQX58k9TnND4AXgn9XBm07gK+kfe8ngtdaN/DxpdaiK1RFRMpQKQ7LiIjIAhTuIiJlSOEuIlKGFO4iImVI4S4iUoYU7iIiZUjhLiJShhTuIiJl6P8DrsdS2ChhzCwAAAAASUVORK5CYII=\n", "text/plain": [ "
" ] }, "metadata": { "needs_background": "light" }, "output_type": "display_data" }, { "data": { "image/png": "iVBORw0KGgoAAAANSUhEUgAAAXcAAAD8CAYAAACMwORRAAAABHNCSVQICAgIfAhkiAAAAAlwSFlzAAALEgAACxIB0t1+/AAAADl0RVh0U29mdHdhcmUAbWF0cGxvdGxpYiB2ZXJzaW9uIDMuMC4zLCBodHRwOi8vbWF0cGxvdGxpYi5vcmcvnQurowAAHe5JREFUeJzt3X1wHPd93/H3F4c7PBIASYDPlEhJlCzJTioFoyqREyuVrafJSEnqJvSMWyf2WOO2appJqqk0ymg88kw7tiZNplO5jlo7Ttw0suw4DkdDD63Ect2HSBFl68GUTAmiJBMiRYIgnh/vgG//2D3gBB6IxeGe9vbzmuHwbu+H2x8Wi8/98N3d35q7IyIijaWp1h0QEZHyU7iLiDQghbuISANSuIuINCCFu4hIA1K4i4g0IIW7iEgDUriLiDQghbuISANqrtWKe3t7fd++fbVavYhILD3//PPn3L1vrXY1C/d9+/Zx9OjRWq1eRCSWzOztKO1UlhERaUAKdxGRBqRwFxFpQAp3EZEGpHAXEWlACncRkQakcBcRaUAKd5ECA2cn+R/PvE12YbHWXRHZkJpdxCRSb3ILi/zzLz/L6bFZJudyfOZDl9e6SyIl08hdJPTi4Cinx2YBePKlUzXujcjGKNxFQj947RxNBr/1C/s4dmqcselsrbskUjKFu0joRydHed+OLm67dgfu8MOfjtS6SyIlU7iLhAbOTHDVjk28b8em4PnZyRr3SKR0CncRYGI2y6mxWa7Y1snmjgxbOzK8MaRwl/hSuIsAbwxNAXDFtk4ALu/rVLhLrCncRYDBkWkALt3aDsBlfR2cCANfJI4U7iLA6dHgFMid3W0A7O5pY3hqntnsQi27JVIyhbsIcHpslvZMiq7W4Lq+XT1tS8tF4kjhLgKcHpthZ3crZgbAzp7WYPnoTC27JVIyhbsIcGpsdmm0DkFZBuAdhbvElMJdBHh3bIYdXa1Lz3d0B49PjaosI/GkcJfEyy4scnZijp0FI/eW5hQ97WnOTc7VsGcipVO4S+KdnZjDHXZ1t75neW9ni8JdYkvhLol3djwovWzrannP8t7OjMJdYkvhLok3PDkPwNaOleHewrnwNZG4iRTuZna7mR03swEzu7/I65eY2dNm9iMze8nM7ix/V0Uq4/xUEOBbOjLvWd7b2cK5CY3cJZ7WDHczSwGPAncA1wAfM7NrVjT7A+AJd78OOAh8sdwdFamUc1NBgG/tXBnuGSbmcrpKVWIpysj9BmDA3U+4+zzwOHD3ijYOdIWPuwHdxkZi4/zkPO2ZFO2Z9951srczKNMMT6k0I/ETJdx3AycLng+Gywp9Fvi4mQ0Ch4F/U5beiVTB8NT8BSUZWA53lWYkjqKEuxVZ5iuefwz4qrvvAe4EvmZmF7y3md1jZkfN7OjQ0ND6eytSAcNT82ztbLlgeb5MozNmJI6ihPsgsLfg+R4uLLt8CngCwN3/HmgFele+kbs/5u797t7f19dXWo9Fymx4co6tFxu5K9wlhqKE+3PAATPbb2YZggOmh1a0+SlwC4CZXU0Q7hqaSyycn5ovGu75kfuIbpQtMbRmuLt7DrgXOAK8SnBWzDEze9jM7gqb/T7waTN7EfhL4LfcfWXpRqTuuDvDk/Ns6bww3NvSKTKpJkYV7hJDzWs3AXc/THCgtHDZQwWPXwFuKm/XRCpvYi7H/MJi0ZG7mdHdnmZ0WmfLSPzoClVJtLFwVN7TfmG4A2xuT2vkLrGkcJdEG58NgrurNV309Z62DKMzGrlL/CjcJdHGZ3IAS7fXW6lbI3eJKYW7JNpEfuTettrIPc3YjMJd4kfhLok2Ppsfua8S7hq5S0wp3CXRxsNR+aZVyjI97RlmsguaPExiR+EuiTYRjtxXD/dgRK/SjMSNwl0SbXw2S0cmRXOq+K9CT1twiqRKMxI3CndJtInZLJtWqbfD8shdFzJJ3CjcJdHGZ3J0ta1+oXZ3eBbNqMoyEjMKd0m08dnsqmfKQEHNXWUZiRmFuyTaxGxu1YOpsDwtga5SlbhRuEuijc9mV72ACaAjkyKdMk37K7GjcJdEG5+5eFnGzOhuy+hsGYkdhbsklruvWZYB6G5rXrrYSSQuFO6SWDPZBXKLftGyDMCm1vTS7JEicaFwl8RanhHy4uHe1ZZemoNGJC4U7pJY+Rkh1yrLbGptXmorEhcKd0ms8TWm+83rak0vjfJF4kLhLom11o068ro0cpcYUrhLYo0vlWXWrrnP5RaZy2naX4kPhbsk1tKNOi4ytwws1+QndFBVYkThLomVP3d9rbNlFO4SRwp3SayJ2RyZVBOt6dRF2+XDXxcySZwo3CWxgnllLl6SgeWavEbuEicKd0msteaVyct/AOgqVYkThbskVpR5ZaBw5K5wl/hQuEtirTXdb17+PHhdyCRxonCXxIpalunINGOmkbvEi8JdEitqWaapyehsadbkYRIrCndJrKhlGQjnl9HIXWJE4S6JNJ9bZDa7uOa8MnnBzJAauUt8KNwlkSYiziuT19WW1kVMEisKd0mkqPPK5HVp5C4xEynczex2MztuZgNmdv8qbX7DzF4xs2Nm9j/L202R8oo6r0yeau4SN2sOW8wsBTwKfAQYBJ4zs0Pu/kpBmwPAA8BN7j5iZtsq1WGRcsiPwqOWZVRzl7iJMnK/ARhw9xPuPg88Dty9os2ngUfdfQTA3c+Wt5si5bV8F6aoB1TTTMxmcfdKdkukbKKE+27gZMHzwXBZoSuBK83s/5rZM2Z2e7k6KFIJ6y7LtDWz6DA1rxt2SDxEGbZYkWUrhy/NwAHgZmAP8L/N7P3uPvqeNzK7B7gH4JJLLll3Z0XKZbksE33kHnxdls6WaF8jUktRRu6DwN6C53uAU0Xa/I27Z939TeA4Qdi/h7s/5u797t7f19dXap9FNmx8NkuTBVMLRLE8p7vq7hIPUcL9OeCAme03swxwEDi0os23gV8GMLNegjLNiXJ2VKScxmeybGpN09RU7A/TCy3fjUlnzEg8rBnu7p4D7gWOAK8CT7j7MTN72MzuCpsdAYbN7BXgaeA+dx+uVKdFNirqvDJ5+WkKdDqkxEWkvdvdDwOHVyx7qOCxA78X/hOpe+Oz0WaEzNN9VCVudIWqJNL4TC7yaZBQUHNXuEtMKNwlkcZns5EvYILlkbvml5G4ULhLIk3M5tZVlmlNp8ikmlSWkdhQuEsijc9k11WWgfwUBBq5Szwo3CVxFhedyfncusoyEE77q5G7xITCXRJnYi6HO5Fv1JGnkbvEicJdEmdpXpmIt9jL08yQEicKd0mcfECve+TektbIXWJD4S6JszTd77pr7s2aW0ZiQ+EuiVN6WUYjd4kPhbskznqn+83b1NrM1PwCC4u6YYfUP4W7JE6pZZn8qZOTOqgqMaBwl8TJ181LGbmDZoaUeFC4S+JMzGZpz6RoTq1v91+ePEzhLvVP4S6Js97pfvO6NO2vxIjCXRJnvdP95i3fR1XhLvVP4S6JMzG3vul+8zTtr8SJwl0SZ3wmt+6rU2H5vHid6y5xoHCXxBmfza77AibQrfYkXhTukjjrvTl2XjrVRGu6iYk5hbvUP4W7JIq7BzfqKKHmDsFBVdXcJQ4U7pIoM9kFcoteUlkGgtMhVZaROFC4S6KUOq9M3qbWtC5iklhQuEuiLM0IWXJZRiN3iQeFuyTK0qRhJZdlNO2vxIPCXRJlfMNlmWbdJFtiQeEuibLRskxXm0buEg8Kd0mU/Ki7lLllADa1NDObXSS7sFjObomUncJdEmWixBt15OkqVYkLhbskyvhMjkyqiZbm0nb9/IRjupBJ6p3CXRIlmFemGTMr6euXJw/TyF3qm8JdEiWYV6a0kgwUlmU0cpf6pnCXRAnmlSntYCoU3kdVI3epbwp3SZRSp/vN031UJS4U7pIopU73m9elW+1JTEQKdzO73cyOm9mAmd1/kXYfNTM3s/7ydVGkfDYy3S9Ap2ruEhNrhruZpYBHgTuAa4CPmdk1RdptAn4HeLbcnRQpl/HZ7IZG7qkmoyOT0shd6l6UkfsNwIC7n3D3eeBx4O4i7T4HfAGYLWP/RMpmLrfAbHaR7g3U3EE37JB4iBLuu4GTBc8Hw2VLzOw6YK+7P1nGvomU1fhMMNreaLh3tWnaX6l/UcK92NUevvSiWRPwR8Dvr/lGZveY2VEzOzo0NBS9lyJlMDazsel+8za1ppmY08hd6luUcB8E9hY83wOcKni+CXg/8H0zewu4EThU7KCquz/m7v3u3t/X11d6r0VKsNG53PN0ww6Jgyjh/hxwwMz2m1kGOAgcyr/o7mPu3uvu+9x9H/AMcJe7H61Ij0VKlB+5l6PmrnCXerdmuLt7DrgXOAK8Cjzh7sfM7GEzu6vSHRQpl43O5Z63qbVZB1Sl7kU6J8zdDwOHVyx7aJW2N2+8WyLlN16mkXuXRu4SA7pCVRJj+YBq6ee5QzByn19YZDa7UI5uiVSEwl0SY3w2R2u6iZbm1Ibep0s37JAYULhLYoxNZzdckoGCG3ZoCgKpYwp3SYyxDc4rk5cv62jkLvVM4S6JMT5bnpF7d1sGWK7hi9QjhbskxthMucI9eI/R6fkNv5dIpSjcJTHGZjZ2o468nvb00vuJ1CuFuyTGeNlH7gp3qV8Kd0mExUVnYi63ofun5qVTTXS2NCvcpa4p3CURJuZyuG980rC87rY0ozOquUv9UrhLIpRr6oG87jbdsEPqm8JdEqFcc7nn9bSnVZaRuqZwl0Qo98i9pz3NqEbuUscU7pII5ZrLPa+7LaORu9Q1hbskQiXKMmMz87j72o1FakDhLomQn+SrbGWZtjTZBWd6XtP+Sn1SuEsijE5naW4yOjIbm+43b+lCJtXdpU4p3CURRqbn6WnPYGZleb+lKQhUd5c6pXCXRBiZyrK5vTwlGVieGVIXMkm9UrhLIpyfnmdzR6Zs76eRu9Q7hbskwuj0fFlH7vlwV81d6pXCXRJhZDrLlnKO3PNlGY3cpU4p3KXhuTsjU8EB1XJpTTeRSTWp5i51S+EuDW9yLkdu0ctaljEzuts1eZjUL4W7NLyRqSCAN5dx5A7BhUwqy0i9UrhLwxsJ73Va9nBvTy+9t0i9UbhLw1sK9zIeUAXY0pHh/JTCXeqTwl0a3vLIvXw1d4CtnS0MTyrcpT4p3KXhVarmvrUjw8j0PAuLmhlS6o/CXRreyPQ8TVa+6X7ztnRkWPTgAimReqNwl4Y3Mj1Pd1uaVFN5Jg3L29rZAqC6u9Qlhbs0vJHpbNlLMhCUZQDOqe4udUjhLg1veHKurFMP5G3tDN5TI3epRwp3aXjnJufp29RS9vfNf2Ccn5or+3uLbJTCXRreuck5ejsrEO7tKstI/YoU7mZ2u5kdN7MBM7u/yOu/Z2avmNlLZvZ3ZnZp+bsqsn7zuUVGp7MVCffmVBM97WmVZaQurRnuZpYCHgXuAK4BPmZm16xo9iOg391/Bvgm8IVyd1SkFMNhyaQSZRkISjPDKstIHYoycr8BGHD3E+4+DzwO3F3YwN2fdvfp8OkzwJ7ydlOkNOcmglF1b2f5D6gC9HboKlWpT1HCfTdwsuD5YLhsNZ8CvlPsBTO7x8yOmtnRoaGh6L0UKdHQ5CwAvRUduSvcpf5ECfdiV34Uvd7azD4O9AOPFHvd3R9z93537+/r64veS5ES5UfufRWouUNwOuTwpMoyUn+aI7QZBPYWPN8DnFrZyMw+DDwIfMjdtbdLXRiarGzNvbezhZHpLNmFRdIpnXwm9SPK3vgccMDM9ptZBjgIHCpsYGbXAX8C3OXuZ8vfTZHSDE3M0dnSTGs6VZH339HdCsDZCY1npL6sGe7ungPuBY4ArwJPuPsxM3vYzO4Kmz0CdALfMLMXzOzQKm8nUlXnJucqNmoH2N4VvPeZ8dmKrUOkFFHKMrj7YeDwimUPFTz+cJn7JVIWwQVMlTlTBmB7VzByPzOmcJf6oiKhNLShicpcnZq3FO4auUudUbhLQxuaqGxZZkt7hnTKeHdcNXepLwp3aVhTcznGZ3NLBz0roanJ2LaplbMauUudUbhLwzod1sF3dbdVdD3bu1o4M6Fwl/qicJeG9W4Y7pUcuUNQd39XB1SlzijcpWGdGpsBqjFyb+Wsau5SZxTu0rDyo+nt3ZU7oApBuE/M5Ziay1V0PSLroXCXhnV6bIbezgwtzZW5OjVvR/jh8a4OqkodUbhLwzo9NsvOCpdkYLns887ITMXXJRKVwl0a1unR2YofTAXYu6UdgJMj02u0FKkehbs0JHfnp+en2bu5veLr2t7VSjplnDyvkbvUD4W7NKShyTlmsgtcurXy4Z5qMnb3tGnkLnVF4S4N6eT5IGgv2VL5cIegNDOomrvUEYW7NKS3h8Nwr8LIHWDP5nYGz2vkLvVD4S4N6e3hacxgz+bKny0DwXqGp+Z1rrvUDYW7NKST56fZ2dVa8XPc8/b3dgDw5rmpqqxPZC0Kd2lIbw5PVa0kA3DFtk4A3hiarNo6RS5G4S4Nx90ZODPJgW2bqrbOS7e2k2oyBs4q3KU+KNyl4Zwem2ViLseV2zurts6W5hSXbGlXuEvdULhLw3ntzAQAB7ZXb+QOcHlfp8Jd6obCXRrO62eCgL2yyuF+xbZO3hqeYj63WNX1ihSjcJeG89qZCXo7W9jSkanqeq/d1UV2wZf+chCpJYW7NJyX3xnj2l1dVV/vB3Z3L61fpNYU7tJQpuZyvHZmgn+0t6fq6750azubWpsV7lIXFO7SUF5+Z4xFpybhbmZ8YHc3Lw8q3KX2FO7SUF44OQrAz9Yg3CH4UHn19LimIZCaU7hLQ3nmxDCX9XZU/WBq3s9fvpXcovMPb52vyfpF8hTu0jBmsws8c2KYXzzQW7M+9F+6hUyqib9/Y7hmfRABhbs0kOffHmE2u8gvXdlXsz60ZVJcd0kPP3htqGZ9EAGFuzSQp145QybVxI2Xba1pPz5yzXZ+8u4EJzSJmNSQwl0aQnZhkUMvnuIj12yno6W5pn35lZ/ZhRk8+dLpmvZDkk3hLg3hez85y/mpeX79+t217go7ulu5Yd8WvvH8SXILmopAakPhLrHn7nzx+2+wu6etpvX2Qp/84H5Onp/hOz9+t9ZdkYRSuEvsHTl2hhdPjvKvf/kK0qn62KU/cvV2Lu/r4A+/e5zZ7EKtuyMJFOk3wcxuN7PjZjZgZvcXeb3FzL4evv6sme0rd0dFijkzPssffPtlrt7ZxUd/bk+tu7Okqcl4+O7389bwNP/x8Ku17o4k0JpHnswsBTwKfAQYBJ4zs0Pu/kpBs08BI+5+hZkdBD4P/GYlOiySd2Jokk//+VFm5hf4o9/8WTLN1R21f/tH7/DIkeOcGp1hV08b9912Fb963XLN/6YrevnUB/fz5f/zJq2ZFPfdehXNdfKXhTS+KKcV3AAMuPsJADN7HLgbKAz3u4HPho+/CfwXMzN39zL2VRLO3RmZzvLyO2McOfYuf/X8IG2ZFH/62zfwvh3VnQXy2z96hwe+9TIzYcnlndEZHvjWywDvCfgH77yamewCf/K/TvDUsTN8/MZLuemKXi7d2k5rujo375ZkihLuu4GTBc8HgX+8Wht3z5nZGLAVOFeOThb61g8H+er/e+s9y4p9hDgXLlzZrvjXFVkW8TOq5H5EXGfRXmzge1i5pNT+r7asmI30Y2Q6u3QjjExzE//0+t38zi0H2NndFm3lZfTIkeNLwZ43k13gkSPH3xPuTU3Gf/i1D3DzlX388d++zsNPBmMiM+huS9ORaaYtk6K5ycLlhoWvm4Fh4f/SSD7zocu54wM7K7qOKOFebL9a+esXpQ1mdg9wD8All1wSYdUXakun2Fpk3hCzC7tQrFMXNivydUW+MNp7Bb+MkdqtWFbs6yIu2sD3fmG7qO9VvG/l25bF3qu7Pc32rlYObOukf99m2jO1O5/91OjMupbfeu0Obr12B2+dm+LFwVHePDfFyNQ8k3MLTM/nWFgMPkaDDzbHnfB5sY9Xibtq/NUW5bdjENhb8HwPcGqVNoNm1gx0AxfMnOTujwGPAfT395e0z97xgZ0V/8QTWcuunjbeKRLku3ou/lfEvt4O9vV2VKpbIkuiHN15DjhgZvvNLAMcBA6taHMI+ET4+KPA91Rvl0Z2321X0bZi9NWWTnHfbVfVqEci77XmyD2sod8LHAFSwFfc/ZiZPQwcdfdDwJeBr5nZAMGI/WAlOy1Sa/m6+sXOlhGpJavVALu/v9+PHj1ak3WLiMSVmT3v7v1rtdNJtyIiDUjhLiLSgBTuIiINSOEuItKAFO4iIg1I4S4i0oAU7iIiDUjhLiLSgGp2EZOZDQFvl/jlvVRgxskyUL/WR/1av3rtm/q1Phvp16Xuvub9JGsW7hthZkejXKFVberX+qhf61evfVO/1qca/VJZRkSkASncRUQaUFzD/bFad2AV6tf6qF/rV699U7/Wp+L9imXNXURELi6uI3cREbmIug13M/tnZnbMzBbNrH/Faw+Y2YCZHTez21b5+v1m9qyZvW5mXw/vIlXuPn7dzF4I/71lZi+s0u4tM3s5bFfxSezN7LNm9k5B3+5cpd3t4TYcMLP7q9CvR8zsJ2b2kpn9tZn1rNKuKttrre/fzFrCn/FAuC/tq1RfCta518yeNrNXw/3/3xZpc7OZjRX8fB+qdL/C9V7052KB/xxur5fM7Poq9Omqgu3wgpmNm9nvrmhTte1lZl8xs7Nm9uOCZVvM7Kkwi54ys82rfO0nwjavm9knirVZF3evy3/A1cBVwPeB/oLl1wAvAi3AfuANIFXk658ADoaPvwT8ywr39w+Bh1Z57S2gt4rb7rPAv1ujTSrcdpcBmXCbXlPhft0KNIePPw98vlbbK8r3D/wr4Evh44PA16vws9sJXB8+3gS8VqRfNwNPVmt/ivpzAe4EvkNwD/QbgWer3L8U8C7BeeA12V7ALwHXAz8uWPYF4P7w8f3F9ntgC3Ai/H9z+HjzRvpStyN3d3/V3Y8Xeelu4HF3n3P3N4EB4IbCBmZmwD8Bvhku+jPgVyvV13B9vwH8ZaXWUQE3AAPufsLd54HHCbZtxbj7d909Fz59huBm67US5fu/m2DfgWBfuiX8WVeMu5929x+GjyeAV4G43LvvbuDPPfAM0GNm1byb/S3AG+5e6sWRG+buPyC41Wihwv1otSy6DXjK3c+7+wjwFHD7RvpSt+F+EbuBkwXPB7lw598KjBYESbE25fSLwBl3f32V1x34rpk9b2b3VLAfhe4N/zT+yip/BkbZjpX0SYJRXjHV2F5Rvv+lNuG+NEawb1VFWAa6Dni2yMs/b2Yvmtl3zOzaKnVprZ9Lrfepg6w+wKrF9srb7u6nIfjwBrYVaVP2bbfmDbIrycz+FthR5KUH3f1vVvuyIstWnvITpU0kEfv4MS4+ar/J3U+Z2TbgKTP7SfgJX7KL9Qv4r8DnCL7nzxGUjD658i2KfO2GT52Ksr3M7EEgB/zFKm9T9u1VrKtFllVsP1ovM+sE/gr4XXcfX/HyDwlKD5Ph8ZRvAweq0K21fi613F4Z4C7ggSIv12p7rUfZt11Nw93dP1zClw0Cewue7wFOrWhzjuBPwuZwxFWsTVn6aGbNwK8DP3eR9zgV/n/WzP6aoCSwobCKuu3M7L8BTxZ5Kcp2LHu/wgNFvwLc4mGxsch7lH17FRHl+8+3GQx/zt1c+Cd32ZlZmiDY/8Ldv7Xy9cKwd/fDZvZFM+t194rOoRLh51KRfSqiO4AfuvuZlS/UansVOGNmO939dFimOlukzSDBsYG8PQTHG0sWx7LMIeBgeCbDfoJP4H8obBCGxtPAR8NFnwBW+0tgoz4M/MTdB4u9aGYdZrYp/5jgoOKPi7UtlxV1zl9bZX3PAQcsOKsoQ/An7aEK9+t24N8Dd7n79CptqrW9onz/hwj2HQj2pe+t9oFULmFN/8vAq+7+n1ZpsyNf+zezGwh+j4cr3K8oP5dDwL8Iz5q5ERjLlyOqYNW/nmuxvVYo3I9Wy6IjwK1mtjkso94aLitdNY4gl/KPIJQGgTngDHCk4LUHCc50OA7cUbD8MLArfHwZQegPAN8AWirUz68Cn1mxbBdwuKAfL4b/jhGUJyq97b4GvAy8FO5YO1f2K3x+J8HZGG9UqV8DBHXFF8J/X1rZr2pur2LfP/AwwYcPQGu47wyE+9JlVdhGHyT4c/ylgu10J/CZ/H4G3BtumxcJDkz/QhX6VfTnsqJfBjwabs+XKTjLrcJ9aycI6+6CZTXZXgQfMKeBbJhfnyI4TvN3wOvh/1vCtv3Afy/42k+G+9oA8Nsb7YuuUBURaUBxLMuIiMgaFO4iIg1I4S4i0oAU7iIiDUjhLiLSgBTuIiINSOEuItKAFO4iIg3o/wN8Ec6Nx7dPLAAAAABJRU5ErkJggg==\n", "text/plain": [ "
" ] }, "metadata": { "needs_background": "light" }, "output_type": "display_data" }, { "data": { "image/png": "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\n", "text/plain": [ "
" ] }, "metadata": { "needs_background": "light" }, "output_type": "display_data" }, { "data": { "image/png": "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\n", "text/plain": [ "
" ] }, "metadata": { "needs_background": "light" }, "output_type": "display_data" }, { "data": { "image/png": "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\n", "text/plain": [ "
" ] }, "metadata": { "needs_background": "light" }, "output_type": "display_data" }, { "data": { "image/png": "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\n", "text/plain": [ "
" ] }, "metadata": { "needs_background": "light" }, "output_type": "display_data" }, { "data": { "image/png": "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\n", "text/plain": [ "
" ] }, "metadata": { "needs_background": "light" }, "output_type": "display_data" }, { "data": { "image/png": "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\n", "text/plain": [ "
" ] }, "metadata": { "needs_background": "light" }, "output_type": "display_data" }, { "data": { "image/png": "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\n", "text/plain": [ "
" ] }, "metadata": { "needs_background": "light" }, "output_type": "display_data" }, { "data": { "image/png": "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\n", "text/plain": [ "
" ] }, "metadata": { "needs_background": "light" }, "output_type": "display_data" }, { "data": { "image/png": "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\n", "text/plain": [ "
" ] }, "metadata": { "needs_background": "light" }, "output_type": "display_data" }, { "data": { "image/png": "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\n", "text/plain": [ "
" ] }, "metadata": { "needs_background": "light" }, "output_type": "display_data" }, { "data": { "image/png": "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\n", "text/plain": [ "
" ] }, "metadata": { "needs_background": "light" }, "output_type": "display_data" }, { "data": { "image/png": "iVBORw0KGgoAAAANSUhEUgAAAX4AAAD8CAYAAABw1c+bAAAABHNCSVQICAgIfAhkiAAAAAlwSFlzAAALEgAACxIB0t1+/AAAADl0RVh0U29mdHdhcmUAbWF0cGxvdGxpYiB2ZXJzaW9uIDMuMC4zLCBodHRwOi8vbWF0cGxvdGxpYi5vcmcvnQurowAAHmlJREFUeJzt3X2QHPV95/H3d2dmH0ErCS1GSIBEoqIMNjb2luIcuTM520JQKeSLfTlx54TEdqnshLvLpc4VKFeBC/44HOouuVxwQHFUfqgc4Pgwp/OJk7Gxi1Q5EC1P4lFmLbBZBEggaYX2YWZn9nt/dM9qtDsPPTM9Mz3az6tqa2e6fz3z297Zz3z317/pNndHRESWj55Od0BERNpLwS8isswo+EVElhkFv4jIMqPgFxFZZhT8IiLLjIJfRGSZUfCLiCwzCn4RkWUm3ekOlLNmzRrfsGFDp7shItI1nnjiibfdfSRK20QG/4YNGxgbG+t0N0REuoaZ/SJqWw31iIgsMwp+EZFlRsEvIrLMKPhFRJYZBb+IyDKj4BcRWWYU/CIiy4yCX6RFHnhygn2vHu10N0SWUPCLtMCh4zP8yXee4TNff7zTXRFZQsEv0gJPv3YcgGx+nsmZuQ73RuR0Cn6RFnjt6PTC7dePzXSwJyJLKfhFWuCNydmF24eOK/glWRT8Ii3w1olZzuoLzoH41ruzNVqLtJeCX6QFjk/PcdE5gwAa45fEUfCLtMDkzBznreinN92j4JfEUfCLtMDkzBzDgxmGBzJMTiv4JVkU/CItMDkzx/BAhpUDGVX8kjgKfpGYzRXmOZnNs3Kgl5WDGY6r4peEUfCLxOxEWOEPD6QZHujluCp+SRgFv0jMikM7Kwd7GR7ILLwRiCSFgl8kZpMLFX+GlYMZjk3nOtwjkdMp+EVidmI2D8CKgTRn96eZzhUozHuHeyVySs3gN7NdZnbYzJ6rsP4qM5s0s6fDr1tK1m01swNmNm5mN8XZcZGkmskFwT+QSTPUG3x6d2au0MkuiZwmSsX/DWBrjTb/4O4fDL9uAzCzFHAXcA1wKXC9mV3aTGdFusF0Lgj5ob4Ug32pYFk238kuiZymZvC7+6NAI1eT2AyMu/tBd88B9wHbGngcka5SDP6B3tRCxT+VU8UvyRHXGP+vm9kzZvaQmV0WLlsHvFbSZiJcVpaZ7TCzMTMbO3LkSEzdEmm/6XCoZ7A3zWBvUPFPqeKXBIkj+J8ELnL3DwD/A3gwXG5l2lY8wuXuO9191N1HR0ZGYuiWSGcsVPyZFEPhGTqnVfFLgjQd/O5+wt1Phrf3ABkzW0NQ4V9Q0nQ9cKjZ5xNJuplcgb50D6keW6j4i/8FiCRB08FvZueZmYW3N4eP+Q6wD9hkZhvNrBfYDuxu9vlEkm46V1gI/MFeVfySPOlaDczsXuAqYI2ZTQC3AhkAd78b+DTwRTPLAzPAdnd3IG9mNwJ7gRSwy92fb8lPIZIgQfAHf1oa45ckqhn87n59jfV/BfxVhXV7gD2NdU2kO83M5RkIA19j/JJE+uSuSMxOH+oJK36N8UuCKPhFYlYa/MWDvNNZVfySHAp+kZjNlIzxmwUze05qjF8SRMEvErPp3Kkxfgjm88/qXD2SIAp+kZhN5woMZkqCv1fBL8mi4BeJWekYP0B/OsXs3HwHeyRyOgW/SMxmcgUGek/NlO7P9DCbV8UvyaHgF4lRvjBPrjB/WsXfl0kxo3n8kiAKfpEYTYdj+acN9WRSzOY11CPJoeAXidFMybn4i/rTPWR1cFcSRMEvEqOFq2+VjPFrVo8kjYJfJEbFir8/c/qsHl1zV5JEwS8So+Lsnf7MqT+t/kyPpnNKoij4RWKUDQO+L73o4K4qfkkQBb9IjMpV/H2ZFNn8PMFlKkQ6T8EvEqNyFf9AON6f1ZROSQgFv0iMshXG+AF9iEsSQ8EvEqOFij9z+hg/oNM2SGIo+EVitDDGn15a8WtmjySFgl8kRmUr/nC8XzN7JClqBr+Z7TKzw2b2XIX1/87M9odfPzWzD5Sse9XMnjWzp81sLM6OiyRRMdxPr/gV/JIsUSr+bwBbq6x/Bfiou18O3A7sXLT+N939g+4+2lgXRbpHNj9PqsdIp5YGvz69K0mRrtXA3R81sw1V1v+05O5jwPrmuyXSnWbnCqdV+3BqjD+rMX5JiLjH+D8HPFRy34EfmNkTZrYj5ucSSZxsfv608X3QUI8kT82KPyoz+02C4P+NksVXuvshMzsXeNjMXnL3RytsvwPYAXDhhRfG1S2Rtipf8Ws6pyRLLBW/mV0OfB3Y5u7vFJe7+6Hw+2Hge8DmSo/h7jvdfdTdR0dGRuLolkjbla/4NZ1TkqXp4DezC4EHgN9195+VLB8ys7OLt4EtQNmZQSJnitm5An2LK/5wOqc+uStJUXOox8zuBa4C1pjZBHArkAFw97uBW4BzgK+ZGUA+nMHzHuB74bI08D/d/f+14GcQSYxyFX/xalwa6pGkiDKr5/oa6z8PfL7M8oPAB5ZuIXLmKjfGX/wPQEM9khT65K5IjMpV/GZGb7pn4QRuIp2m4BeJUbmKH4oXXFfFL8mg4BeJUa5MxQ/Fi7Go4pdkUPCLxKhixa/r7kqCKPhFYhSM8S/9s+pLq+KX5FDwi8QoqPiXDvWo4pckUfCLxEgVv3QDBb9ITPKFefLzropfEk/BLxKTbL549S1V/JJsCn6RmCxcfavMdE5V/JIkCn6RmCxU/GWmc6rilyRR8IvERBW/dAsFv0hMalb8ugKXJISCXyQmxYq//CkbepjNq+KXZFDwi8SkVsWfy8/j7u3ulsgSCn6RmNQa44dTbw4inaTgF4lJrYof0KmZJREU/CIxiVLx6/KLkgQKfpGYqOKXbqHgF4nJqeBXxS/JpuAXiUl2YahHFb8kW6TgN7NdZnbYzJ6rsN7M7C/NbNzM9pvZh0rW3WBmL4dfN8TVcZGkUcUv3SJqxf8NYGuV9dcAm8KvHcBfA5jZauBW4NeAzcCtZraq0c6KJFl2rkCPQSZlS9ap4pckiRT87v4ocLRKk23AtzzwGLDSzNYCVwMPu/tRdz8GPEz1NxCRrjWbn6cvncJsafAvVPw6bYMkQFxj/OuA10ruT4TLKi0XOeNk5wplz8UPJRW/PsAlCRBX8C8tccCrLF/6AGY7zGzMzMaOHDkSU7dE2iebny87lRNU8UuyxBX8E8AFJffXA4eqLF/C3Xe6+6i7j46MjMTULZH2yebny354C1TxS7LEFfy7gd8LZ/d8BJh09zeAvcAWM1sVHtTdEi4TOePMzhVU8UtXSEdpZGb3AlcBa8xsgmCmTgbA3e8G9gDXAuPANPAH4bqjZnY7sC98qNvcvdpBYpGulQ0P7pajil+SJFLwu/v1NdY78EcV1u0CdtXfNZHuks0Xyn54C06dxkEVvySBPrkrEpPZucoVf0+P0ZvqUcUviaDgF4lJNl95jB/Cq3Cp4pcEUPCLxCQ7N19xHj+E191VxS8JoOAXiUk2P09/haEeCGb26ILrkgQKfpGYzFb55C4EB3hV8UsSKPhFYlJtOicEV+bSGL8kgYJfJCbZvCp+6Q4KfpEYuHvV6Zygil+SQ8EvEoNcofL1dotU8UtSKPhFYlDtQutFqvglKRT8IjEoXlmr0tk5QRW/JIeCXyQGxUpeFb90AwW/SAwWhnpU8UsXUPCLxCCbDyr5flX80gUU/CIxmJ2LXvEHZzEX6RwFv0gMihV/9bNz6mIskgwKfpEYRJnOWVyn4JdOU/CLxCDKdM7iOp2hUzpNwS8Sg0hDPar4JSEU/CIxyEY4uFus+DWzRzpNwS8SgyjTOVXxS1JECn4z22pmB8xs3MxuKrP+z83s6fDrZ2Z2vGRdoWTd7jg7L5IUUaZzquKXpEjXamBmKeAu4BPABLDPzHa7+wvFNu7+n0ra/3vgipKHmHH3D8bXZZHk0Ri/dJMoFf9mYNzdD7p7DrgP2Fal/fXAvXF0TqRbZPPz9Bike6xiG1X8khRRgn8d8FrJ/Ylw2RJmdhGwEXikZHG/mY2Z2WNm9slKT2JmO8J2Y0eOHInQLZHkyObn6c+kMKsc/MWrc6nil06LEvzlXsmVPnO+Hfiuu5eWNBe6+yjwb4G/MLNfKbehu+9091F3Hx0ZGYnQLZHkmJ0rVB3mAehPq+KXZIgS/BPABSX31wOHKrTdzqJhHnc/FH4/CPyE08f/Rc4I2RqXXQRV/JIcUYJ/H7DJzDaaWS9BuC+ZnWNmlwCrgH8sWbbKzPrC22uAK4EXFm8r0u2y+QL9VS60Dqr4JTlqzupx97yZ3QjsBVLALnd/3sxuA8bcvfgmcD1wn59+6sH3AveY2TzBm8wdpbOBRM4UtS60Dqr4JTlqBj+Au+8B9ixadsui+18ps91Pgfc30T+RrpDNFxaCvZI+VfySEPrkrkgMsvn5mgd3Uz1GJmWq+KXjFPwiMShO56ylP62rcEnnKfhFYhBlOicE4/yq+KXTFPwiMQiGempX/H2q+CUBFPwiMYhycBdU8UsyKPhFYhBlOicEY/y6Apd0moJfJAZZjfFLF1Hwi8Rgdm6ewV7N6pHuoOAXaVK+ME+uEG06pyp+SQIFv0iTZsMgH9A8fukSCn6RJs3kwuvtRhjqUcUvSaDgF2lSsYJXxS/dQsEv0qSZOoJfFb8kgYJfpEnFoZ6B3tp/Tv0ZVfzSeQp+kSYVgzzSrJ50UPGfftkKkfZS8Is0qZ6hnv5MCnfIFTTcI52j4Bdp0sLB3SizetK6Cpd0noJfpEnFir8/ytk5M7oKl3Segl+kSTO58ANc9VT8c6r4pXMU/CJNmqnj4G6xTTavil86J1Lwm9lWMztgZuNmdlOZ9b9vZkfM7Onw6/Ml624ws5fDrxvi7LxIEtTzAa5ixT+ril86KF2rgZmlgLuATwATwD4z2+3uLyxqer+737ho29XArcAo4MAT4bbHYum9SALM5AoLF1KvRRW/JEGUin8zMO7uB909B9wHbIv4+FcDD7v70TDsHwa2NtZVkWSamSswkElhVjv4i/8VFI8LiHRClOBfB7xWcn8iXLbYp8xsv5l918wuqHNbka41O1eINL4PLJyzfzqXb2WXRKqKEvzlypjFHzv8P8AGd78c+CHwzTq2DRqa7TCzMTMbO3LkSIRuiSTDzFwh0ukaoDT4NdQjnRPl1ToBXFByfz1wqLSBu7/j7tnw7t8AH466bclj7HT3UXcfHRkZidJ3kUSYnStEmsMPMNgbHFZT8EsnRQn+fcAmM9toZr3AdmB3aQMzW1ty9zrgxfD2XmCLma0ys1XAlnCZyBljJleINIcfYLBPQz3SeTVn9bh73sxuJAjsFLDL3Z83s9uAMXffDfwHM7sOyANHgd8Ptz1qZrcTvHkA3ObuR1vwc4h0zEw9Y/wZDfVI59UMfgB33wPsWbTslpLbNwM3V9h2F7CriT6KJNrM3DwrBzKR2qZTPfSmexT80lH65K5Ik2ZzBfoz0f+UBntTGuqRjlLwizRpKpdnqDfSP88ADPWmVfFLRyn4RZo0nSsw1Bc9+AdU8UuHKfhFmnQym68r+Id6U6r4paMU/CJNmCvMk8vPMxRxOieEFX9WwS+do+AXaUIxwOur+NNMz2moRzpHwS/ShJPhWP1Qnyp+6R4KfpEmTGeLwa9ZPdI9FPwiTThZDP46pnMO9KaY0qwe6SAFv0gTphoZ4+9LMZMr4F72RLUiLafgF2nCVANj/IO9afLzTq6gi7FIZyj4RZow1cBQT/Gc/DMa55cOUfCLNGEqV/9Qjy7GIp2m4BdpwkLFX+dQT+m2Iu2m4BdpwlQ2T4+duoh6FGf3B8F/YlbBL52h4BdpwlS2wFBvGrNyl5cu7+z+4Nz9787OtapbIlUp+EWaMJXNL1xOMarhAVX80lkKfpEmnMzVd2ZOUMUvnafgF2nCdLa+i7AArFgIflX80hkKfpEmTGULdc3oAejP9JDuMU7MqOKXzlDwizRhcmaO4YgXWi8yM87uT6vil46JFPxmttXMDpjZuJndVGb9n5jZC2a238x+ZGYXlawrmNnT4dfuODsv0mknZucWhm7qsWIgwwmN8UuH1BycNLMUcBfwCWAC2Gdmu939hZJmTwGj7j5tZl8E/gz4N+G6GXf/YMz9FkmERip+QBW/dFSUin8zMO7uB909B9wHbCtt4O4/dvfp8O5jwPp4uymSPHOFeaZzBVY0EPwr+jMa45eOiRL864DXSu5PhMsq+RzwUMn9fjMbM7PHzOyTlTYysx1hu7EjR45E6JZIZxWDWxW/dJso89DKfSSx7InEzewzwCjw0ZLFF7r7ITO7GHjEzJ51958veUD3ncBOgNHRUZ2oXBKv+AGsFQP1TeeEYC6/5vFLp0Sp+CeAC0rurwcOLW5kZh8Hvgxc5+7Z4nJ3PxR+Pwj8BLiiif6KJMZkWPE3dHC3P6NP7krHRAn+fcAmM9toZr3AduC02TlmdgVwD0HoHy5ZvsrM+sLba4ArgdKDwiJdq9mhnpPZPIV5/XMr7Vfzf1R3z5vZjcBeIAXscvfnzew2YMzddwN3AmcBfx+erOqX7n4d8F7gHjObJ3iTuWPRbCCRrrVQ8TdycHfg1GkbVg72xtovkVoiDU66+x5gz6Jlt5Tc/niF7X4KvL+ZDook1fHpHAArB+sP/lXhNsemFfzSfvrkrkiD3j4ZBP+qBoJ79VCwzdGpbI2WIvFT8Is06OhUjuGBDJlU/X9G5wz1AfBO+OYh0k4KfpEGHZ3Kcc5QY8M0q88qVvwKfmk/Bb9Ig96ZynLOWY0Ff/EN4x0Fv3SAgl+kQUencgtj9fXqz6QY7E2p4peOUPCLNOidkzlWh2P1jVg91Kvgl45Q8Is0IF+Y59h0jjUNDvUAjJzdx5F3NatH2k/BL9KAw+9mmXdYOzzQ8GOsHe7njcmZGHslEo2CX6QBb0zOAkF4N+q8FQO8MTmLu07bIO2l4BdpwJth8J/XRPCvHe5nOlfg3axO1ibtpeAXaUBxiKapij/ctvgmItIuCn6RBrwxOUt/pqehM3MWFd80Xj+ucX5pLwW/SAN+8c4UF60eIjwbbUMuPGcweKy3p+LqlkgkCn6RBhw8MsXFI0NNPcbIWX2c3ZfmFQW/tJmCX6ROc4V5fnl0mo1rmgt+M2PjyBAHFfzSZgp+kTq9dnSa/Lw3HfwAG9cMcfCIgl/aS8EvUqdnX58E4NLzVzT9WJedv4LXj8/wzkl9glfaR8EvUqenfnmcgUyKS95zdtOP9YH1KwF4ZuJ4048lEpWCX6ROT/7yGO9fN0y6gQuwLPa+dcOkeoyxV4/F0DORaBT8InV4c3KW/ROTfPSSkVgeb6gvzehFq/jRi4djeTyRKCIFv5ltNbMDZjZuZjeVWd9nZveH6x83sw0l624Olx8ws6vj67pI+31//yEAtlz6ntgec8tl53HgrXd56c0TsT2mSDU1g9/MUsBdwDXApcD1ZnbpomafA465+68Cfw58Ndz2UmA7cBmwFfha+HgiXedkNs/X/+EVNm9czaYa4/sPPvU6V97xCBtv+r9ceccjPPjU6xXb/vYV6xjsTfHff/hy3F0WKStKxb8ZGHf3g+6eA+4Dti1qsw34Znj7u8DHLPhI4zbgPnfPuvsrwHj4eCJd5ZW3p/jCt5/g8Luz/OnWS6q2ffCp17n5gWd5/fgMTnBKhpsfeLZi+K8a6uUPr/oVHnruTf7LnheZnJ5rwU8gcko6Qpt1wGsl9yeAX6vUxt3zZjYJnBMuf2zRtusa7m0Nv3P3PzKbLyxZXu2st075lVW3qbCu2sl1Gzn1buXnqfxYcfatao+r7tP6nqf6NtWeJ77fXTXTuTzHpufoTfVwx6cu58MXra7a/s69B5iZO/11ODNX4M69B/jkFeVf/l+86leZODbDPY8e5J5HD7J2uJ+hvjR96R56qpwWotoZIxo/mYR0yvBgL9/6bOtr4yjBX+71s/jPp1KbKNsGD2C2A9gBcOGFF0bo1lKrhjLk8uVHkqqdU6XSmuqnYSm/spE/xOrbNPA8FdZVeqxwZd2r4t6ncT5P9Z+nvn2aSfWw6dyz+K3L13Luitpn4zxU4aRrlZYDpHqMOz51OZ/5yEX85MBhXnl7mtm5ArNzhVjfSCXZmjnpXz2iBP8EcEHJ/fXAoQptJswsDQwDRyNuC4C77wR2AoyOjjb0ur3nd0cb2UwkVuevHCh7xs3zV9a+Wtf71g3zvnXDreiWyIIoY/z7gE1mttHMegkO1u5e1GY3cEN4+9PAIx6UI7uB7eGsn43AJuCf4um6SDJ96epLGMic/p/nQCbFl66ufmxApF1qVvzhmP2NwF4gBexy9+fN7DZgzN13A38LfNvMxgkq/e3hts+b2XeAF4A88EfuvnQQXuQMUhzHv3PvAQ4dn+H8lQN86epLKo7vi7SbJfF6n6Ojoz42NtbpboiIdA0ze8LdI41365O7IiLLjIJfRGSZUfCLiCwzCn4RkWVGwS8isswo+EVElhkFv4jIMqPgFxFZZhL5AS4zOwL8osHN1wBvx9iduKhf9VG/6qN+1edM7NdF7h7p0nCJDP5mmNlY1E+vtZP6VR/1qz7qV32We7801CMisswo+EVElpkzMfh3droDFahf9VG/6qN+1WdZ9+uMG+MXEZHqzsSKX0REqujK4Dezf21mz5vZvJmNLlp3s5mNm9kBM7u6wvYbzexxM3vZzO4PrywWdx/vN7Onw69XzezpCu1eNbNnw3YtvwiBmX3FzF4v6du1FdptDffhuJnd1IZ+3WlmL5nZfjP7npmtrNCuLfur1s8fXlXu/nD942a2oVV9KXnOC8zsx2b2Yvj6/49l2lxlZpMlv99bWt2v8Hmr/l4s8Jfh/tpvZh9qQ58uKdkPT5vZCTP740Vt2rK/zGyXmR02s+dKlq02s4fDHHrYzFZV2PaGsM3LZnZDuTZ1c/eu+wLeC1wC/AQYLVl+KfAM0AdsBH4OpMps/x1ge3j7buCLLe7vfwVuqbDuVWBNG/fdV4D/XKNNKtx3FwO94T69tMX92gKkw9tfBb7aqf0V5ecH/hC4O7y9Hbi/Db+7tcCHwttnAz8r06+rgO+36/UU9fcCXAs8BBjwEeDxNvcvBbxJMNe97fsL+BfAh4DnSpb9GXBTePumcq95YDVwMPy+Kry9qtn+dGXF7+4vuvuBMqu2Afe5e9bdXwHGgc2lDczMgH8JfDdc9E3gk63qa/h8vwPc26rnaIHNwLi7H3T3HHAfwb5tGXf/gbvnw7uPAetb+Xw1RPn5txG8diB4LX0s/F23jLu/4e5PhrffBV4EuuV6jtuAb3ngMWClma1t4/N/DPi5uzf6wdCmuPujBJelLVX6GqqUQ1cDD7v7UXc/BjwMbG22P10Z/FWsA14ruT/B0j+Mc4DjJSFTrk2c/jnwlru/XGG9Az8wsyfMbEcL+1HqxvDf7V0V/r2Msh9b6bME1WE57dhfUX7+hTbha2mS4LXVFuHQ0hXA42VW/7qZPWNmD5nZZW3qUq3fS6dfU9upXHx1Yn8BvMfd34DgTR04t0ybluy3mhdb7xQz+yFwXplVX3b3/11pszLLFk9bitImkoh9vJ7q1f6V7n7IzM4FHjazl8LqoGHV+gX8NXA7wc98O8Ew1GcXP0SZbZue/hVlf5nZl4E88HcVHib2/VWuq2WWtex1VC8zOwv4X8Afu/uJRaufJBjOOBkev3kQ2NSGbtX6vXRyf/UC1wE3l1ndqf0VVUv2W2KD390/3sBmE8AFJffXA4cWtXmb4N/MdFiplWsTSx/NLA38NvDhKo9xKPx+2My+RzDM0FSQRd13ZvY3wPfLrIqyH2PvV3jg6reAj3k4wFnmMWLfX2VE+fmLbSbC3/MwS/+Vj52ZZQhC/+/c/YHF60vfCNx9j5l9zczWuHtLz0sT4ffSktdURNcAT7r7W4tXdGp/hd4ys7Xu/kY47HW4TJsJguMQResJjm025Uwb6tkNbA9nXGwkeOf+p9IGYaD8GPh0uOgGoNJ/EM36OPCSu0+UW2lmQ2Z2dvE2wQHO58q1jcuicdV/VeH59gGbLJj91Evwb/LuFvdrK/CnwHXuPl2hTbv2V5SffzfBaweC19Ijld6s4hIeQ/hb4EV3/28V2pxXPNZgZpsJ/sbfaXG/ovxedgO/F87u+QgwWRzmaIOK/3V3Yn+VKH0NVcqhvcAWM1sVDstuCZc1p9VHs1vxRRBYE0AWeAvYW7LuywQzMg4A15Qs3wOcH96+mOANYRz4e6CvRf38BvCFRcvOB/aU9OOZ8Ot5giGPVu+7bwPPAvvDF97axf0K719LMGvk523q1zjBWObT4dfdi/vVzv1V7ucHbiN4YwLoD1874+Fr6eI27KPfIPg3f3/JfroW+ELxdQbcGO6bZwgOkv+zNvSr7O9lUb8MuCvcn89SMhuvxX0bJAjy4ZJlbd9fBG88bwBzYXZ9juCY0I+Al8Pvq8O2o8DXS7b9bPg6Gwf+II7+6JO7IiLLzJk21CMiIjUo+EVElhkFv4jIMqPgFxFZZhT8IiLLjIJfRGSZUfCLiCwzCn4RkWXm/wPirdj+/c0s8AAAAABJRU5ErkJggg==\n", "text/plain": [ "
" ] }, "metadata": { "needs_background": "light" }, "output_type": "display_data" }, { "data": { "image/png": "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\n", "text/plain": [ "
" ] }, "metadata": { "needs_background": "light" }, "output_type": "display_data" }, { "data": { "image/png": "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\n", "text/plain": [ "
" ] }, "metadata": { "needs_background": "light" }, "output_type": "display_data" }, { "data": { "image/png": "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\n", "text/plain": [ "
" ] }, "metadata": { "needs_background": "light" }, "output_type": "display_data" }, { "data": { "image/png": "iVBORw0KGgoAAAANSUhEUgAAAX4AAAD8CAYAAABw1c+bAAAABHNCSVQICAgIfAhkiAAAAAlwSFlzAAALEgAACxIB0t1+/AAAADl0RVh0U29mdHdhcmUAbWF0cGxvdGxpYiB2ZXJzaW9uIDMuMC4zLCBodHRwOi8vbWF0cGxvdGxpYi5vcmcvnQurowAAH3hJREFUeJzt3X2QXFd95vHvM909L3qxJVtj/CZZ8uL1YhIwMKs4OLuYALZMZS2yy7JyZROHl1KFxNnNbi21dlFlKFNbReLKUsXGxCigMqQS2wnBRMuKGCWGmCwxsQy2jI2FB2FW4zFItt49Mz3TM7/9o2+PWqPu6dsvM33H83yquqb73nO7z9zpefr0Oefeq4jAzMyWj55uV8DMzBaXg9/MbJlx8JuZLTMOfjOzZcbBb2a2zDj4zcyWGQe/mdky4+A3M1tmHPxmZstMvtsVqGXdunWxcePGblfDzGzJePzxx1+KiME0ZTMZ/Bs3bmTv3r3droaZ2ZIh6Sdpy7qrx8xsmXHwm5ktMw5+M7NlxsFvZrbMOPjNzJYZB7+Z2TLj4DczW2Yc/GYd9OiBl3n8J0e6XQ2zeWXyAC6zpWrbjkcBeO5/3Egh53aVZZPfmWYdcujkxOz9g0fGulgTs/k5+M06ZPTY6eD/8UuvdLEmZvNz8Jt1yE+Pnw7+wyeLXayJ2fwc/GYdcvjU6bB/+ZXJLtbEbH4NB3cl7QR+BTgUET9XY/1HgF+rer7XAYMRcUTS88BJYBooRcRQpypuljUnxqcAKOTEy6cc/JZdaVr89wJb6q2MiLsi4uqIuBq4Hfj7iKiez/b2ZL1D317VTkxM0Zvr4cJz+zk65uC37GoY/BHxCJB2YvLNwH1t1chsiTo5UeKcgTyr+gqcKpa6XR2zujrWxy9pBeVvBn9VtTiAr0t6XNL2Tr2WWRadnCixur/Aqr4cpyYc/JZdnTyA698A/3dON8+1ETEq6QJgj6Rnk28QZ0k+GLYDbNiwoYPVMlscJ8anOKc/z6q+vAd3LdM6OatnG3O6eSJiNPl5CHgQ2Fxv44jYERFDETE0OJjqspFmmXJyYorV/QVW9uXd4rdM60jwSzoXeBvw11XLVkpaXbkPXA98vxOvZ5ZFJyZKrO7Ps7o/7z5+y7Q00znvA64D1kkaAT4GFAAi4p6k2K8CX4+I6sMVXwM8KKnyOn8eEX/TuaqbZcvJiSnO6S+wstfBb9nWMPgj4uYUZe6lPO2zetkB4I2tVsxsqTmZtPhX9ecZm5xmeibI9ajb1TI7i4/cNeuA6ZlgbHKaVcngLsArk271WzY5+M06YHxqGoCVvVXB7+4eyygHv1kHjE+Wg7+/N8fKJPg9s8eyysFv1gETSYt/oJBjVX85+E+6xW8Z5eA364CxpMW/ojfHikIOgIlkmVnWOPjNOmC8qsU/0Js7Y5lZ1jj4zTpgto+/kKO/0uKfmulmlczqcvCbdcBsH39vjoGCW/yWbQ5+sw6ohPyK3hx9hfK/1YSD3zLKwW/WAZXB3YHC6Ra/g9+yysFv1gGVFv+ZffwOfssmB79ZB1Smbg705ijkesj3yH38llkOfrMOqJ7OCeWWv2f1WFY5+M06YGxymt58z+zZOPsLObf4LbMc/GYdMDE1PdvaB+gv9LiP3zLLwW/WAeOT0/QXTv87DRRyDn7LLAe/WQcUS9Ozs3mgPMg77nP1WEY5+M06oFiaoS9/+t+pP+/BXcuuhsEvaaekQ5JqXihd0nWSjkt6IrndUbVui6T9koYl3dbJiptlyWRpht7q4O/14K5lV5oW/73AlgZlvhURVye3OwEk5YC7gRuBq4CbJV3VTmXNsqrc4q8a3M17cNeyq2HwR8QjwJEWnnszMBwRByJiErgf2NrC85hl3uTcrh4P7lqGdaqP/xclPSnpa5Jenyy7BDhYVWYkWVaTpO2S9krae/jw4Q5Vy2xxFEvTZ3T19OV7mCy5j9+yqRPB/13gsoh4I/C/gK8ky1WjbNR7kojYERFDETE0ODjYgWqZLZ65g7u9+R4mpx38lk1tB39EnIiIU8n93UBB0jrKLfz1VUUvBUbbfT2zLCoP7p7u4+/L5yh6Vo9lVNvBL+lCSUrub06e82XgMeAKSZsk9QLbgF3tvp5ZFs1t8fcVeii6xW8ZlW9UQNJ9wHXAOkkjwMeAAkBE3AO8F/iwpBIwDmyLiABKkm4FHgJywM6IeHpBfguzLjurqydX7uOPCJJ2kVlmNAz+iLi5wfo/Av6ozrrdwO7Wqma2dJw1uJucvqFYmjnjiF6zLPCRu2YdMHcef2+u/K/lAV7LIge/WZsi4qwjd/uSVr4HeC2LHPxmbaq06s8Y3M1Vunp8EJdlj4PfrE2VA7XmzuqpXmeWJQ5+szYVawV/vueMdWZZ4uA3a9Pp4K8a3M27xW/Z5eA3a1Ml3M88V08yuOvgtwxy8Ju1qTKAO/dcPdXrzLLEwW/Wptotfnf1WHY5+M3aVKuP3109lmUOfrM2zU7nLJzd1eMWv2WRg9+sTZV+/MppGqB6Oqf7+C17HPxmbaqclqFWi99dPZZFDn6zNlVO2VCrxe+uHssiB79Zm063+M8+gMstfssiB79Zm4o1TtLWm3PwW3Y5+M3aVJxKBnergl8SffkeD+5aJjUMfkk7JR2S9P06639N0r7k9m1Jb6xa97ykpyQ9IWlvJytulhW1TtIG5Q8Cn4/fsihNi/9eYMs8638MvC0i3gB8AtgxZ/3bI+LqiBhqrYpm2TZ75G7uzH+nvnzOV+CyTEpzzd1HJG2cZ/23qx4+ClzafrXMlo5icvWtuRdV73OL3zKq0338HwS+VvU4gK9LelzS9g6/llkmTJZmzurmgXLwu8VvWdSwxZ+WpLdTDv5fqlp8bUSMSroA2CPp2Yh4pM7224HtABs2bOhUtcwWXLE0XTP4y338Hty17OlIi1/SG4DPAVsj4uXK8ogYTX4eAh4ENtd7jojYERFDETE0ODjYiWqZLYpiaeaME7RVlGf1uMVv2dN28EvaAHwZ+PWI+GHV8pWSVlfuA9cDNWcGmS1lk0kf/1x9+ZyP3LVMatjVI+k+4DpgnaQR4GNAASAi7gHuAM4HPpMMbpWSGTyvAR5MluWBP4+Iv1mA38Gsq+br6hmbLHWhRmbzSzOr5+YG6z8EfKjG8gPAG8/ewuzVZb7B3WPjbvFb9vjIXbM2Fet09fgALssqB79ZmyY9uGtLjIPfrE31Wvwe3LWscvCbtWneefw+SZtlkIPfrE31p3P2uMVvmeTgN2tTsc6snl738VtGOfjN2lR/cDdHaSaYnoku1MqsPge/WZvmm84Jvu6uZY+D36xN9QZ3+2avu+sBXssWB79ZG2ZmgqnpqD24W3CL37LJwW/WhsnZC62f3cfvC65bVjn4zdpQ73q7AH2FXFLGXT2WLQ5+szZUQr3ePP5yGbf4LVsc/GZtqJyErd48fnDwW/Y4+M3aUOnjn7fF7zN0WsY4+M3acLrFX/vsnIAvuG6Z4+A3a8PsrJ5C7bNzAr7gumWOg9+sDZVQ78vV7+pxi9+yJlXwS9op6ZCkmhdLV9mnJQ1L2ifpzVXrbpH0XHK7pVMVN8uC2emcNVr8ve7jt4xK2+K/F9gyz/obgSuS23bgjwEknUf54uy/AGwGPiZpbauVNcuaylG5vbnaJ2kDz+qx7EkV/BHxCHBkniJbgS9G2aPAGkkXATcAeyLiSEQcBfYw/weI2ZIyX4t/tqvHB3BZxnSqj/8S4GDV45FkWb3lZq8Kk9NJH7/n8dsS0qngV41lMc/ys59A2i5pr6S9hw8f7lC1zBZWpf9+vnn8PkmbZU2ngn8EWF/1+FJgdJ7lZ4mIHRExFBFDg4ODHaqW2cKa7yRt+VwPPXKL37KnU8G/C/iNZHbPNcDxiHgReAi4XtLaZFD3+mSZ2avCfC1+KH8g+CRtljX5NIUk3QdcB6yTNEJ5pk4BICLuAXYD7waGgTHg/cm6I5I+ATyWPNWdETHfILHZklIJ9Vp9/FAe9HVXj2VNquCPiJsbrA/gd+qs2wnsbL5qZtlXLM3QI8j31BrOKp+T3109ljU+ctesDcXkQutS7eDvKzj4LXsc/GZtKE5N15zDX9Gbc1ePZY+D36wNk9Mzdfv3wYO7lk0OfrM2FKdmak7lrHBXj2WRg9+sDeU+/vm7ehz8ljUOfrM2FEvz9/H3FXIOfsscB79ZGyqzeurpy3tw17LHwW/WhnIf/zxdPfkeD+5a5jj4zdpQLE3XPV0DlFv8vhCLZY2D36wNjQZ3+/I9vvSiZY6D36wNjfv4c77YumWOg9+sDcWpabf4bclx8Ju1oViamf+UDfnyPP7yeQzNssHBb9aGyRTTOSNgatrBb9nh4DdrQ+PB3fKHgrt7LEsc/GYtmpmJ5CRt9Vv8sxdc9wCvZYiD36xFs9fbne+UDZXg99G7liGpgl/SFkn7JQ1Luq3G+k9JeiK5/VDSsap101XrdnWy8mbdVDkwq9GRu4BP22CZ0vDSi5JywN3Au4AR4DFJuyLimUqZiPgvVeV/F3hT1VOMR8TVnauyWTZUTsUw/5G7uaSsg9+yI02LfzMwHBEHImISuB/YOk/5m4H7OlE5syyrhHmjWT3gFr9lS5rgvwQ4WPV4JFl2FkmXAZuAh6sW90vaK+lRSe9puaZmGVNp8afp6vGJ2ixLGnb1ALWuIl1vUvI24EsRUf0u3xARo5IuBx6W9FRE/OisF5G2A9sBNmzYkKJaZt01kaKP34O7lkVpWvwjwPqqx5cCo3XKbmNON09EjCY/DwDf5Mz+/+pyOyJiKCKGBgcHU1TLrLtmu3oK8116MZnH7+C3DEkT/I8BV0jaJKmXcrifNTtH0pXAWuAfq5atldSX3F8HXAs8M3dbs6VospRiVk/OXT2WPQ27eiKiJOlW4CEgB+yMiKcl3QnsjYjKh8DNwP1x5klJXgd8VtIM5Q+ZT1bPBjJbytL08Vfm+Lurx7IkTR8/EbEb2D1n2R1zHn+8xnbfBn6+jfqZZVaaWT2nW/wOfssOH7lr1qLTffxu8dvS4uA3a1Hl/DupTtLm4LcMcfCbtajSim90zd1yWQ/uWnY4+M1a1FQfvy+4bhni4DdrUZpZPT09opCTz8dvmeLgN2tRmrNzltfn3OK3THHwm7WoWJqhN9+DVOusJqf15Xvcx2+Z4uA3a9Fkg8suVvQXcrPn9THLAge/WYuKpel5B3Yr+gs9TPjSi5YhDn6zFjW60HrFQG+OcQe/ZYiD36xFqYO/kHOL3zLFwW/WovHJafrnOSVzRX/BLX7LFge/WYsmpqYZ6E0Z/JMOfssOB79Zi8anphlI0eIfKOR8kjbLFAe/WYvSd/X0uMVvmeLgN2tR2q6eAffxW8Y4+M1aND41zYo0Lf5ez+qxbHHwm7VovIkWf7E0w8xMNCxrthhSBb+kLZL2SxqWdFuN9b8p6bCkJ5Lbh6rW3SLpueR2Sycrb9ZNzUznBJjw+XosIxpec1dSDrgbeBcwAjwmaVeNi6Y/EBG3ztn2POBjwBAQwOPJtkc7UnuzLpmZCYqlmdSzegAmpmZY0bvQNTNrLE2LfzMwHBEHImISuB/YmvL5bwD2RMSRJOz3AFtaq6pZdlQGawd60x25W72NWbelCf5LgINVj0eSZXP9O0n7JH1J0vomtzVbUmaDP0WLv3LBdU/ptKxIE/y1TjY+d5TqfwMbI+INwN8CX2hi23JBabukvZL2Hj58OEW1zLqnEuJp+vhPd/U4+C0b0gT/CLC+6vGlwGh1gYh4OSKKycM/Ad6Sdtuq59gREUMRMTQ4OJim7mZdMzHb1ZMi+Hsd/JYtaYL/MeAKSZsk9QLbgF3VBSRdVPXwJuAHyf2HgOslrZW0Frg+WWa2pDXT1eM+fsuahrN6IqIk6VbKgZ0DdkbE05LuBPZGxC7gP0m6CSgBR4DfTLY9IukTlD88AO6MiCML8HuYLapKV0+a4O+vmtVjlgUNgx8gInYDu+csu6Pq/u3A7XW23QnsbKOOZpkz3kRXT79b/JYxPnLXrAUt9fF7Vo9lhIPfrAXN9PH3J1fp8pG7lhUOfrMWjDXRx19p8Xsev2WFg9+sBbPz+NP08efLZcYc/JYRDn6zFkw00dXT0yMGCjnGJksLXS2zVBz8Zi0Yn5om3yMKuXT/Qiv78pwqusVv2eDgN2vB+GS6M3NWrOrL8UrRLX7LBge/WQvGJkuppnJWrOrPO/gtMxz8Zi04VSyxqj/V8Y8ArOzNc9LBbxnh4DdrwaliiVV96YN/VZ9b/JYdDn6zFrxSLLGyt4kWv4PfMsTBb9aCU8VpVjbR4vesHssSB79ZC14plljV51k9tjQ5+M1a8Eqx1FSLf1VfgfGpaUrTPjWzdZ+D36wFTc/qSb4dvOLTNlgGOPjNmjQ1PUOxNMOqJgZ3KzOA3N1jWeDgN2tSJbybHdyt3tasmxz8Zk06lYR3s/P4q7c166ZUwS9pi6T9koYl3VZj/X+V9IykfZL+TtJlVeumJT2R3HbN3dZsqXklmZbZ1OBuv4PfsqPhO1dSDrgbeBcwAjwmaVdEPFNV7HvAUESMSfow8AfAf0jWjUfE1R2ut1nXnJiYAuCcgeZO2QDu6rFsSNPi3wwMR8SBiJgE7ge2VheIiG9ExFjy8FHg0s5W0yw7jo+Vg//cgULqbU539XhWj3VfmuC/BDhY9XgkWVbPB4GvVT3ul7RX0qOS3lNvI0nbk3J7Dx8+nKJaZt1xbLyF4E+6ek4m3xbMuinNd1XVWBY1C0r/ERgC3la1eENEjEq6HHhY0lMR8aOznjBiB7ADYGhoqObzm2XB8RaC/5wk+CvbmnVTmhb/CLC+6vGlwOjcQpLeCXwUuCkiipXlETGa/DwAfBN4Uxv1Neu6Sniv7k8f/PlcD6v78xwbc/Bb96UJ/seAKyRtktQLbAPOmJ0j6U3AZymH/qGq5Wsl9SX31wHXAtWDwmZLzonxKVb358n11PoyXN/aFb0cG5tcoFqZpdewqyciSpJuBR4CcsDOiHha0p3A3ojYBdwFrAL+UhLA/4uIm4DXAZ+VNEP5Q+aTc2YDmS05x8enWLMifWu/Ys2KAkfd4rcMSDUfLSJ2A7vnLLuj6v4762z3beDn26mgWdYcG5tsqn+/Ys2K3tmBYbNu8pG7Zk06Pj7VWvAPFNzVY5ng4Ddr0pFXJlm7orfp7dasKHhw1zLBwW/WpJdOTTK4uq/p7das6OXExBTTM56tbN3l4DdrwsTUNKeKJdataj74163qJQJefqXYuLDZAnLwmzXh8MlyaA+2EPwXrO4H4NAJB791l4PfrAkvnSqH9rrVzffxX3BO+cPi0MmJjtbJrFkOfrMmvHSqPCtncFV/09tekIwLuMVv3ebgN2tCpaunlRZ/ZUD40EkHv3WXg9+sCSNHx8j3aLa/vhl9+RxrVxTc1WNd5+A3a8LBo+NcvGag6fP0VFy8ZoAXjo53uFZmzXHwmzVh5OgY688baHn7jeev5CcvjzUuaLaAHPxmTTh4ZJz1a1e0vP1l56/g4NExStMzHayVWXMc/GYpnZyY4qVTRdaf117wT00Ho8fcz2/d4+A3S+nZn54E4KqLzmn5OTatWwXAc4dOdqROZq1w8Jul9MzoCQCuurj14H/9xefQI9g3crxT1TJrmoPfLKV9I8c5f2Xv7IFYrVjZl+e1F6xi38ixDtbMrDkOfrMUIoJHnjvMNf/sfJKrzLXsLZetZe/zRymWpjtUO7PmpAp+SVsk7Zc0LOm2Guv7JD2QrP+OpI1V625Plu+XdEPnqm62eJ4cOc7hk0Wu++eDbT/Xu656DSeLJb49/HIHambWvIbBLykH3A3cCFwF3CzpqjnFPggcjYjXAp8Cfj/Z9irKF2d/PbAF+EzyfGZLyue+dYBVfXlu+LkLU5X/yvde4NpPPsym2/4P137yYb7yvRdm11372nWcv7KXz//Dj4nwuflt8aVp8W8GhiPiQERMAvcDW+eU2Qp8Ibn/JeAdKn8f3grcHxHFiPgxMJw8n9mSMFma4XPfOsBX973I+6/dyDn9jS+5+JXvvcDtX36KF46NE8ALx8a5/ctPzYZ/Xz7Hb7/9tfzD8Et8as8PGZ90l48trjQXW78EOFj1eAT4hXplIqIk6ThwfrL80TnbXtJybRt43z3/yMScftN6Darg7BV1y9ZYXq+d1kwLrvbz1t6+3TrUrdUCPW/bv1uTDeGFqtuxsSmKpRne8S8u4Hd/+YpUdbnrof2MT535Phyfmuauh/bznjeV3/7vf+tGnn7hOJ9+eJh7/v4AF57bz0AhR0+PyPVAj0R7Iwm2FJ27opcvfmDh28Zpgr/W+2/uf0m9Mmm2LT+BtB3YDrBhw4YU1Trb2pUFJktn9yTVG4yrtbT+uN3ZK+qVbeZ5a/171y1bY3ndeKhZtt7z1qhD3bIL87z1f43u7vfV/Xne+tp1vO2KQXpSnp9n9Fjtc/FUL+/pEX/4vjfyvn+5nm/uP8zosXGKpWmmZ2Amghl3AS1L5w40/kbZCWmCfwRYX/X4UmC0TpkRSXngXOBIym0BiIgdwA6AoaGhlt71n/31oVY2M+uoi9cM8EKN8L94zZnn+JHENZefzzWXn79YVTMD0vXxPwZcIWmTpF7Kg7W75pTZBdyS3H8v8HCUv3vvArYls342AVcA/9SZqptl00duuJKBwpnfPAcKOT5yw5VdqpHZmRq2+JM++1uBh4AcsDMinpZ0J7A3InYBnwf+VNIw5Zb+tmTbpyX9BfAMUAJ+JyI8kmWvapV+/Lse2s/osfJpnD9yw5Wzy826TVmcTjY0NBR79+7tdjXMzJYMSY9HRKr+bh+5a2a2zDj4zcyWGQe/mdky4+A3M1tmHPxmZsuMg9/MbJlx8JuZLTMOfjOzZSaTB3BJOgz8pMXN1wEvdbA6neJ6Ncf1ao7r1ZxXY70ui4hUVwrKZPC3Q9LetEevLSbXqzmuV3Ncr+Ys93q5q8fMbJlx8JuZLTOvxuDf0e0K1OF6Ncf1ao7r1ZxlXa9XXR+/mZnN79XY4jczs3ksyeCX9O8lPS1pRtLQnHW3SxqWtF/SDXW23yTpO5Kek/RAcmWxTtfxAUlPJLfnJT1Rp9zzkp5Kyi34RQgkfVzSC1V1e3edcluSfTgs6bZFqNddkp6VtE/Sg5LW1Cm3KPur0e+fXFXugWT9dyRtXKi6VL3meknfkPSD5P3/n2uUuU7S8aq/7x0LXa/kdef9u6js08n+2ifpzYtQpyur9sMTkk5I+r05ZRZlf0naKemQpO9XLTtP0p4kh/ZIWltn21uSMs9JuqVWmaZFxJK7Aa8DrgS+CQxVLb8KeBLoAzYBPwJyNbb/C2Bbcv8e4MMLXN8/BO6os+55YN0i7ruPA/+tQZlcsu8uB3qTfXrVAtfreiCf3P994Pe7tb/S/P7AbwP3JPe3AQ8swt/uIuDNyf3VwA9r1Os64KuL9X5K+3cB3g18DRBwDfCdRa5fDvgp5bnui76/gH8NvBn4ftWyPwBuS+7fVus9D5wHHEh+rk3ur223PkuyxR8RP4iI/TVWbQXuj4hiRPwYGAY2VxeQJOCXgS8li74AvGeh6pq83vuA+xbqNRbAZmA4Ig5ExCRwP+V9u2Ai4usRUUoePgpcupCv10Ca338r5fcOlN9L70j+1gsmIl6MiO8m908CPwCWyvUctwJfjLJHgTWSLlrE138H8KOIaPXA0LZExCOUL0tbrfo9VC+HbgD2RMSRiDgK7AG2tFufJRn887gEOFj1eISz/zHOB45VhUytMp30r4CfRcRzddYH8HVJj0vavoD1qHZr8nV7Z52vl2n240L6AOXWYS2Lsb/S/P6zZZL30nHK761FkXQtvQn4To3VvyjpSUlfk/T6RapSo79Lt99T26jf+OrG/gJ4TUS8COUPdeCCGmUWZL81vNh6t0j6W+DCGqs+GhF/XW+zGsvmTltKUyaVlHW8mflb+9dGxKikC4A9kp5NWgctm69ewB8Dn6D8O3+CcjfUB+Y+RY1t257+lWZ/SfooUAL+rM7TdHx/1apqjWUL9j5qlqRVwF8BvxcRJ+as/i7l7oxTyfjNV4ArFqFajf4u3dxfvcBNwO01Vndrf6W1IPsts8EfEe9sYbMRYH3V40uB0TllXqL8NTOftNRqlelIHSXlgX8LvGWe5xhNfh6S9CDlboa2giztvpP0J8BXa6xKsx87Xq9k4OpXgHdE0sFZ4zk6vr9qSPP7V8qMJH/nczn7q3zHSSpQDv0/i4gvz11f/UEQEbslfUbSuohY0PPSpPi7LMh7KqUbge9GxM/mrujW/kr8TNJFEfFi0u11qEaZEcrjEBWXUh7bbMurratnF7AtmXGxifIn9z9VF0gC5RvAe5NFtwD1vkG0653AsxExUmulpJWSVlfuUx7g/H6tsp0yp1/1V+u83mPAFSrPfuql/DV51wLXawvw34GbImKsTpnF2l9pfv9dlN87UH4vPVzvw6pTkjGEzwM/iIj/WafMhZWxBkmbKf+Pv7zA9Urzd9kF/EYyu+ca4Hilm2MR1P3W3Y39VaX6PVQvhx4Crpe0NumWvT5Z1p6FHs1eiBvlwBoBisDPgIeq1n2U8oyM/cCNVct3Axcn9y+n/IEwDPwl0LdA9bwX+K05yy4GdlfV48nk9jTlLo+F3nd/CjwF7EveeBfNrVfy+N2UZ438aJHqNUy5L/OJ5HbP3Hot5v6q9fsDd1L+YALoT947w8l76fJF2Ee/RPlr/r6q/fRu4Lcq7zPg1mTfPEl5kPyti1Cvmn+XOfUScHeyP5+iajbeAtdtBeUgP7dq2aLvL8ofPC8CU0l2fZDymNDfAc8lP89Lyg4Bn6va9gPJ+2wYeH8n6uMjd83MlplXW1ePmZk14OA3M1tmHPxmZsuMg9/MbJlx8JuZLTMOfjOzZcbBb2a2zDj4zcyWmf8PbQpJdaR6B7MAAAAASUVORK5CYII=\n", "text/plain": [ "
" ] }, "metadata": { "needs_background": "light" }, "output_type": "display_data" }, { "data": { "image/png": "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\n", "text/plain": [ "
" ] }, "metadata": { "needs_background": "light" }, "output_type": "display_data" }, { "data": { "image/png": "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\n", "text/plain": [ "
" ] }, "metadata": { "needs_background": "light" }, "output_type": "display_data" }, { "data": { "image/png": "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\n", "text/plain": [ "
" ] }, "metadata": { "needs_background": "light" }, "output_type": "display_data" }, { "data": { "image/png": "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\n", "text/plain": [ "
" ] }, "metadata": { "needs_background": "light" }, "output_type": "display_data" }, { "data": { "image/png": "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\n", "text/plain": [ "
" ] }, "metadata": { "needs_background": "light" }, "output_type": "display_data" }, { "data": { "image/png": "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\n", "text/plain": [ "
" ] }, "metadata": { "needs_background": "light" }, "output_type": "display_data" }, { "data": { "image/png": "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\n", "text/plain": [ "
" ] }, "metadata": { "needs_background": "light" }, "output_type": "display_data" }, { "data": { "image/png": "iVBORw0KGgoAAAANSUhEUgAAAXcAAAD8CAYAAACMwORRAAAABHNCSVQICAgIfAhkiAAAAAlwSFlzAAALEgAACxIB0t1+/AAAADl0RVh0U29mdHdhcmUAbWF0cGxvdGxpYiB2ZXJzaW9uIDMuMC4zLCBodHRwOi8vbWF0cGxvdGxpYi5vcmcvnQurowAAGgpJREFUeJzt3X2MHPd93/HPd/d2lw+iRFE8WRRJmXQiq1WCAJavjGO1gRAn1gMCK0mdggZaK3YKIWmExmhrVKoLJ3X+iW0kAYw4EpRYsJ0athrHUdhUrqxEbty0kGpS1qMpWic/QEfSJEWJz3f7+O0fM3tc7s3szBz34cf1+wUcuLszu/u7ueXnvved38yYuwsAMF1Kkx4AAGD4CHcAmEKEOwBMIcIdAKYQ4Q4AU4hwB4ApRLgDwBQi3AFgChHuADCFZib1xps3b/YdO3ZM6u0B4JK0b9++19x9Nmu9iYX7jh07tHfv3km9PQBckszsB3nWoy0DAFOIcAeAKUS4A8AUItwBYAoR7gAwhQh3AJhChDsATCHCHViFRquj//rkD3Su0Zr0UIBEhDuwCn//nWP6z4+8oP+y59uTHgqQiHAHVmH+6BlJ0g9PLU14JEAywh1YhYMnzkmSTi81JzwSIBnhDqzCG2ejUD9yqj7hkQDJCHdgFd4415AknaJyR6AId2AV3jgXhfrppZbaHZ/waICVCHdgFU4tnq/YzywxHRLhIdyBVVhstrWuWpZEawZhItyBVThbb+nqDTVJUWsGCA3hDhTU7rjqrY42ra9Kiqp4IDSZ4W5m283s62a238xeNLPfTljHzOxTZjZvZs+Z2U2jGS4wed0w37Q+qtyXCHcEKM81VFuS/r27P21mGyTtM7PH3b33uOvbJV0ff/20pPvjf4Gp0z2fzFXdyr1BuCM8mZW7ux9296fj26cl7Ze0tW+1OyV93iNPStpoZluGPlogAN0w33QZbRmEq1DP3cx2SHqbpKf6Fm2V9GrP/QWt/AUATIVzcbhfRc8dAcsd7mZ2maS/lPQhdz/VvzjhKSuO7DCzu81sr5ntPXbsWLGRAoHohvvmy+i5I1y5wt3MKoqC/Qvu/pWEVRYkbe+5v03Sof6V3P1Bd59z97nZ2dnVjBeYuOW2TFy5n6PnjgDlmS1jkj4jab+7/2HKanskvT+eNfMOSSfd/fAQxwkEo7tDdRM7VBGwPLNlbpb0ryQ9b2bPxI/9J0nXSZK7PyDpUUl3SJqXdE7SB4Y/VCAM3R77umpZayol2jIIUma4u/s/KLmn3ruOS/qtYQ0KCNnZejfcZ7S2UmaHKoLEEapAQd22zNpqWeuqM7RlECTCHSio3upIktZWorbMOSp3BIhwBwrqhnulbFpbLWuJyh0BItyBgprtjqrlksyMnjuCRbgDBTVaHVVnov86awh3BIpwBwrqDffaTEmNuE0DhIRwBwpqtjuqlKPZwVXCHYEi3IGCeiv3armkRptwR3gId6CgRrujSjkOdyp3BIpwBwpqtKLZMhLhjnAR7kBBjXZHteW2TJlwR5AId6CgZl9bpk7PHQEi3IGCLtihGrdlonPnAeEg3IGC+ue5S1KzTbgjLIQ7UFCj7efbMvG/TIdEaAh3oKBGq31BWyZ6jHBHWAh3oKBm2y+YCikR7ggP4Q4UdME89/jfeouThyEshDtQUKPdoS2D4BHuQEHN1oXz3KXzF/AAQkG4AwXVkyp3ZssgMIQ7UIC7XzjPvUxbBmEi3IECWp3oYKVqz/ncJcId4SHcgQK6Ic4OVYSOcAcKaMa99RXz3Om5IzCEO1BAt0KvzPSdfoDKHYEh3IECulMeOUIVoSPcgQKW2zJ9PXfO6Y7QEO5AAY2+nnutXI4ep3JHYAh3oABmy+BSQbgDBXTbMv2nHyDcERrCHSig3le5l0umcsnUaHNWSISFcAcK6F5OrxvuUtR/p3JHaAh3oIBG31RI6fxFsoGQEO5AAf07VLu3OUIVoSHcgQL6d6hKURXP+dwRmsxwN7OHzOyomb2QsvwWMztpZs/EXx8d/jCBMCRV7jXaMgjQTI51PivpjyV9fsA6/9vdf3EoIwICVm/Tc8elIbNyd/dvSHp9DGMBgtdM26FKzx2BGVbP/WfM7Fkz+6qZ/cSQXhMITqO9si1TKZeWe/FAKPK0ZbI8LenN7n7GzO6Q9Iik65NWNLO7Jd0tSdddd90Q3hoYr2bSbJlySc2WT2pIQKKLrtzd/ZS7n4lvPyqpYmabU9Z90N3n3H1udnb2Yt8aGLtGu6OSRUemdlVmSpwVEsG56HA3s2vMzOLbu+LXPH6xrwuEqPfi2F1R5U64IyyZbRkz+6KkWyRtNrMFSb8jqSJJ7v6ApPdK+k0za0lalLTb3fkbFVOp0e5cMMddkqozxg5VBCcz3N39fRnL/1jRVElg6jVaHdX6Knd2qCJEHKEKFNBodS6YBilx4jCEiXAHCmi2O8sXx+6qzFC5IzyEO1BAo51cuXNuGYSGcAcKaLR85WwZKncEiHAHCkicLVMuLV/EAwgF4Q4U0Gi1V1TulXJJ7Y6r3SHgEQ7CHSig2faVPfc47GnNICSEO1BA0hGqlXJ0KgJ2qiIkhDtQQOI8dyp3BIhwBwpImufeDXsOZEJICHeggHpC5d6dPUPljpAQ7kABzXbCWSFpyyBAhDtQQHSEql3wWLdyZ4cqQkK4AwUkzZapLVfuzHNHOAh3oIBmwhGqFXaoIkCEO5BTp+PRQUz03HEJINyBnLpXW0o7iInKHSEh3IGcupV52lRILrWHkBDuQE7dyjxthyqVO0JCuAM5dWfDcBATLgWEO5BTtzJfcT53dqgiQIQ7kFOj3ZaUtEOVtgzCQ7gDOTVaUVsmrXJvcBATAkK4Azl1Z8PUOCskLgGEO5BT2mwZeu4IEeEO5NQN7/62TLlkKhmVO8JCuAM5pVXuUhT4VO4ICeEO5NRIOUJVigKfU/4iJIQ7kNP5yt1WLKtSuSMwhDuQ03K4l8srllVnCHeEhXAHclreoZpQuVfKJXaoIiiEO5BTVs+dKzEhJIQ7kFPWbBl2qCIkhDuQUyNlnrtEzx3hIdyBnM7vUE0I97LRc0dQCHcgp+ji2KZSKXmHKpU7QkK4Azk1Wp3ElowUtWW4zB5CkhnuZvaQmR01sxdSlpuZfcrM5s3sOTO7afjDBCav0eok7kyVmAqJ8OSp3D8r6bYBy2+XdH38dbek+y9+WEB4Gm0fWLnTlkFIMsPd3b8h6fUBq9wp6fMeeVLSRjPbMqwBAqFotDqJO1OlaCcrbRmEZBg9962SXu25vxA/toKZ3W1me81s77Fjx4bw1sD4NNqdFRfq6KqWS2q2OIgJ4RhGuK+cOiAlfsrd/UF3n3P3udnZ2SG8NTA+zQE7VCszRuWOoAwj3Bckbe+5v03SoSG8LhCURjt9h2q1XFaTHaoIyDDCfY+k98ezZt4h6aS7Hx7C6wJBaQ4I98qMqU7ljoDMZK1gZl+UdIukzWa2IOl3JFUkyd0fkPSopDskzUs6J+kDoxosMEn1VnQQU5Lu+dzdXWbJ6wDjlBnu7v6+jOUu6beGNiIgUI1WR5evrSQuq5ZLcpdaHU/9BQCME0eoAjk12x1VU4K7ErdrmOuOUBDuQE6DjlDtzn9nOiRCQbgDOTXa6QcxdSv3ers9ziEBqQh3IKdB89xr3cqdqzEhEIQ7kNOgee7d66py8jCEgnAHcqo3O6rNlBOXVcrsUEVYCHcgp/rAI1Sjx6ncEQrCHcjB3dVopZ84rLtDlfPLIBSEO5BDN7RrlcE7VKncEQrCHcihPuDi2BIHMSE8hDuQQ7cir1WSd6hW2aGKwBDuQA7dyr2WVrnTlkFgCHcgh3ozOvI0redeXd6hykFMCAPhDuSwvEOVqZC4RBDuQA71ZrxDNeMIVXruCAXhDuRwvnIfvEOVyh2hINyBHLIrd2bLICyEO5BDvRXvUM3oudep3BEIwh3IodtuybxYB5U7AkG4Azksz3NP6bmXSqaZklG5IxiEO5DD8hGqKZW7JK2plJd788CkEe5ADt2ee1pbRoqCv7seMGmEO5BDPWflvkTljkAQ7kAO9YwdqlJ0aoIlKncEgnAHcsg65a8krZmh545wEO5ADt2rMJlZ6jq1Cj13hINwB3Kot9oDWzJSVLkvNQl3hIFwB3Kotzqpc9y71lRKzHNHMAh3IIdBF8fuqlG5IyCEO5BDPUe4r6mUmAqJYBDuQA6NPD33SpkdqggG4Q7kkKdyr81QuSMchDuQQ72ZZ4cqPXeEg3AHcmi0O5ltmVqlrHqrI3cuko3JI9yBHOqtdq4dqtG6tGYweYQ7kEO92VGtkj0VsrsuMGm5wt3MbjOzA2Y2b2b3Jiz/NTM7ZmbPxF//evhDBSZnsdnWmkr2QUySOHkYgjCTtYKZlSV9WtIvSFqQ9E0z2+Pu3+5b9WF3v2cEYwQmbqnZ1tqscKdyR0DyVO67JM27+3fdvSHpS5LuHO2wgLAsNrLDvUbljoDkCfetkl7tub8QP9bvn5vZc2b2ZTPbPpTRAQFwdy0221pbzVe5Mx0SIcgT7knnOO2f6/XfJe1w95+S9LeSPpf4QmZ3m9leM9t77NixYiMFJqTR7qjjytFzj9syzJZBAPKE+4Kk3kp8m6RDvSu4+3F3r8d3/1TS25NeyN0fdPc5d5+bnZ1dzXiBsesedZrZc++2ZajcEYA84f5NSdeb2U4zq0raLWlP7wpmtqXn7nsk7R/eEIHJ6oZ1VlumttyWoXLH5GXOlnH3lpndI+kxSWVJD7n7i2b2MUl73X2PpH9rZu+R1JL0uqRfG+GYgbFabMThnrNy5+RhCEFmuEuSuz8q6dG+xz7ac/s+SfcNd2hAGBbjyj1vz53KHSHgCFUgw2Lutgw9d4SDcAcyLMVtmTVZ55apMhUS4SDcgQx5K/d1cVvmXINwx+QR7kCG5XDP6LnPlEuqzZR0ttEax7CAgQh3IEN3tkzWDlVJWl+b0bk6lTsmj3AHMizlnC0jSeuqZSp3BIFwBzKcjSv3y2rZM4fXV6ncEQbCHchwtt5Syc4fpDTIuhqVO8JAuAMZTi+1tL42I7Okc+hdaH11htkyCALhDmQ4W2/laslIcc+9TuWOySPcgQxnG1Hlnsf6GpU7wkC4AxnO1Nu5w31dtaxz9NwRAMIdyHBmqakNhcKdyh2TR7gDGc7W21pfy57jLknr4h2qnU7/xcqA8SLcgQxn6vl77t0dr0yHxKQR7kCGs438s2UuXxutd2qJcMdkEe7AAO6uswUq9yvWViRJJ881RzksIBPhDgxQb3XUbHv+yn1NHO6LhDsmi3AHBjgVh3S3Is9yebzeqSXCHZNFuAMDvBG3Vzauyxfuy20ZKndMGOEODPDGuYYk6cp11VzrL1fuhDsmjHAHBjhRsHLfUJuRGeGOySPcgQFOxJX7xpyVe6lk2lCboS2DiSPcgQG6Pfcrc1buknTFugrhjokj3IEBTiw2VC2XMi+O3euq9TUdP9sY4aiAbIQ7MMCJs01tXFfJdaGOrtkNNR07XR/hqIBshDswwGtn6tq0Pl+/vetqwh0BINyBAQ6fXNK1G9cWes7shqgt02x3RjQqIBvhDgxw5NSS3nT5mkLPmd1QkyQdP0PfHZNDuAMplpptHT/b0JYrCob7ZVG405rBJBHuQIqjp6JwvqZguG+5ImrjHDxxbuhjAvIi3IEUh04uSpKuKdiW2bF5nSTpe68R7pgcwh1IMX/0jCTpLbPrCz1vw5qKNl9W0/dfOzuKYQG5EO5AipePnNb6allbC86WkaSdm9fpe4Q7JohwB1IcOHJab71mQ6EDmLp+/OoNeumHp7hQNiaGcAcSNNsdvXDwlG7ccvmqnj/35it1aqml7xw9PeSRAfnkCnczu83MDpjZvJndm7C8ZmYPx8ufMrMdwx4oME77fvCGztRb+tm3zq7q+bt2bpIk/d/548McFpBbZribWVnSpyXdLulGSe8zsxv7Vvt1SW+4+49L+iNJHx/2QIFxeuRbB1WdKemdP3bVwHVu/v0ntPPe/6Gbf/8JPfKtg8vLtl25Vv/omg36i30Lcqc1g/HLU7nvkjTv7t9194akL0m6s2+dOyV9Lr79ZUnvstU0KoEJc3c98dIRfXnfgnb/k+3asCb5VL+PfOug7vvK8zp4YlEu6eCJRd33leeXA97M9IGbd2j/4VO6/+9fUYtTEWDM8lzSfaukV3vuL0j66bR13L1lZiclXSXptWEMstc/vPyaPvHYS4nL0gokV3rlNKioSn+9Qc8pXqUNHEPKuw1+Ttr7DNgO6S+XunA122HwcwYNYRXbYRUF82KzrdfPNnTDmzbo3/3CW1PX++RjB7TYbK947icfO6BfettWSdKvvn27/m7/UX3ifx7Q/V9/RVdfXtOaSlndssd0vv45/xh+FPzKTdt01zt3jPQ98oR70uet/79NnnVkZndLuluSrrvuuhxvvVJ1pqSrBpylL+0PhkH/aQb/jZHyegOek7Zo8HPSF6Y9b1Wvt4pxR+81vO26mvcZ+LxVbIe0t5kpmX5q20b98tu2am01/Rzuh04sZj5eKpke+Jdv19e+/UP9n/njOn62rnozquB7/3N0fxHSvPnRMeizNSx5wn1B0vae+9skHUpZZ8HMZiRdIen1/hdy9wclPShJc3Nzq/os79q5Sbt27lrNU4GhuXbjWh1MCPj+M0iWSqbbfnKLbvvJLeMaGiApX8/9m5KuN7OdZlaVtFvSnr519ki6K779XklPOHuRMMU+fOsNK67OtLZS1odvvWFCIwIulFm5xz30eyQ9Jqks6SF3f9HMPiZpr7vvkfQZSX9uZvOKKvbdoxw0MGndvvonHzugQycWde3GtfrwrTcsPw5Mmk2qwJ6bm/O9e/dO5L0B4FJlZvvcfS5rPY5QBYApRLgDwBQi3AFgChHuADCFCHcAmEKEOwBMIcIdAKYQ4Q4AU2hiBzGZ2TFJP1jl0zdrBGecHIJQxyWFOzbGVQzjKmYax/Vmd8+8iszEwv1imNnePEdojVuo45LCHRvjKoZxFfOjPC7aMgAwhQh3AJhCl2q4PzjpAaQIdVxSuGNjXMUwrmJ+ZMd1SfbcAQCDXaqVOwBggGDD3cx+1cxeNLOOmc31LbvPzObN7ICZ3Zry/J1m9pSZvWxmD8dXkRr2GB82s2fir++b2TMp633fzJ6P1xv5SezN7HfN7GDP2O5IWe+2eBvOm9m9YxjXJ83sJTN7zsz+ysw2pqw3lu2V9f2bWS3+Gc/Hn6UdoxpLz3tuN7Ovm9n++PP/2wnr3GJmJ3t+vh8d9bh63nvgz8Yin4q32XNmdtMYxnRDz7Z4xsxOmdmH+tYZyzYzs4fM7KiZvdDz2CYzezzOosfN7MqU594Vr/Oymd2VtE4h7h7kl6R/LOkGSf9L0lzP4zdKelZSTdJOSa9IKic8/79J2h3ffkDSb454vH8g6aMpy74vafMYt93vSvoPGeuU4233FknVeJveOOJxvVvSTHz745I+Pqntlef7l/RvJD0Q394t6eEx/Oy2SLopvr1B0ncSxnWLpL8Z1+epyM9G0h2SvqrosuXvkPTUmMdXlvRDRXPBx77NJP2spJskvdDz2Cck3Rvfvjfpcy9pk6Tvxv9eGd++8mLGEmzl7u773f1AwqI7JX3J3evu/j1J85IuuGK2mZmkn5P05fihz0n6pVGNNX6/fyHpi6N6jxHYJWne3b/r7g1JX1K0bUfG3b/m7q347pOKLrY+KXm+/zsVfXak6LP0rvhnPTLuftjdn45vn5a0X9KldO2+OyV93iNPStpoZuO8Ovi7JL3i7qs9QPKiuPs3FF1qtFfv5ygti26V9Li7v+7ub0h6XNJtFzOWYMN9gK2SXu25v6CVH/6rJJ3oCZKkdYbpn0k64u4vpyx3SV8zs31mdvcIx9HrnvjP4odS/gzMsx1H6YOKKrwk49heeb7/5XXiz9JJRZ+tsYjbQG+T9FTC4p8xs2fN7Ktm9hPjGpOyfzaT/lztVnqRNalt9iZ3PyxFv7wlXZ2wztC3W+YFskfJzP5W0jUJiz7i7n+d9rSEx/qn/ORZJ5ecY3yfBlftN7v7ITO7WtLjZvZS/Bt+1QaNS9L9kn5P0ff8e4paRh/sf4mE51701Kk828vMPiKpJekLKS8z9O2VNNSEx0b2OSrKzC6T9JeSPuTup/oWP62o7XAm3p/yiKTrxzEuZf9sJrnNqpLeI+m+hMWT3GZ5DH27TTTc3f3nV/G0BUnbe+5vk3Sob53XFP05OBNXXEnrDGWMZjYj6VckvX3AaxyK/z1qZn+lqCVwUWGVd9uZ2Z9K+puERXm249DHFe8o+kVJ7/K42ZjwGkPfXgnyfP/ddRbin/MVWvkn99CZWUVRsH/B3b/Sv7w37N39UTP7EzPb7O4jP4dKjp/NSD5XOd0u6Wl3P9K/YJLbTNIRM9vi7ofjFtXRhHUWFO0X6NqmaH/jql2KbZk9knbHMxl2Kvrt+/96V4hD4+uS3hs/dJektL8ELtbPS3rJ3ReSFprZejPb0L2taKfiC0nrDktfj/OXU97vm5Kut2hWUVXRn7N7Rjyu2yT9R0nvcfdzKeuMa3vl+f73KPrsSNFn6Ym0X0jDEvf0PyNpv7v/Yco613R7/2a2S9H/4+OjHFf8Xnl+NnskvT+eNfMOSSe7LYkxSP0LelLbLNb7OUrLosckvdvMrozbqO+OH1u9Ue89Xu2XolBakFSXdETSYz3LPqJopsMBSbf3PP6opGvj229RFPrzkv5CUm1E4/yspN/oe+xaSY/2jOPZ+OtFRe2JUW+7P5f0vKTn4g/Wlv5xxffvUDQb45UxjWteUV/xmfjrgf5xjXN7JX3/kj6m6JePJK2JPzvz8WfpLWPYRv9U0Z/jz/Vspzsk/Ub3cybpnnjbPKtox/Q7Rz2uQT+bvrGZpE/H2/R59cx0G/HY1ikK6yt6Hhv7NlP0y+WwpGacX7+uaD/N30l6Of53U7zunKQ/63nuB+PP2rykD1zsWDhCFQCm0KXYlgEAZCDcAWAKEe4AMIUIdwCYQoQ7AEwhwh0AphDhDgBTiHAHgCn0/wFP4gPyQBI6ZgAAAABJRU5ErkJggg==\n", "text/plain": [ "
" ] }, "metadata": { "needs_background": "light" }, "output_type": "display_data" }, { "data": { "image/png": "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\n", "text/plain": [ "
" ] }, "metadata": { "needs_background": "light" }, "output_type": "display_data" }, { "data": { "image/png": "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\n", "text/plain": [ "
" ] }, "metadata": { "needs_background": "light" }, "output_type": "display_data" }, { "data": { "image/png": "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\n", "text/plain": [ "
" ] }, "metadata": { "needs_background": "light" }, "output_type": "display_data" }, { "data": { "image/png": "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\n", "text/plain": [ "
" ] }, "metadata": { "needs_background": "light" }, "output_type": "display_data" }, { "data": { "image/png": "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\n", "text/plain": [ "
" ] }, "metadata": { "needs_background": "light" }, "output_type": "display_data" }, { "data": { "image/png": "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\n", "text/plain": [ "
" ] }, "metadata": { "needs_background": "light" }, "output_type": "display_data" }, { "data": { "image/png": "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\n", "text/plain": [ "
" ] }, "metadata": { "needs_background": "light" }, "output_type": "display_data" }, { "data": { "image/png": "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\n", "text/plain": [ "
" ] }, "metadata": { "needs_background": "light" }, "output_type": "display_data" }, { "data": { "image/png": "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\n", "text/plain": [ "
" ] }, "metadata": { "needs_background": "light" }, "output_type": "display_data" }, { "data": { "image/png": "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\n", "text/plain": [ "
" ] }, "metadata": { "needs_background": "light" }, "output_type": "display_data" }, { "data": { "image/png": "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\n", "text/plain": [ "
" ] }, "metadata": { "needs_background": "light" }, "output_type": "display_data" }, { "data": { "image/png": "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\n", "text/plain": [ "
" ] }, "metadata": { "needs_background": "light" }, "output_type": "display_data" }, { "data": { "image/png": "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\n", "text/plain": [ "
" ] }, "metadata": { "needs_background": "light" }, "output_type": "display_data" }, { "data": { "image/png": "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\n", "text/plain": [ "
" ] }, "metadata": { "needs_background": "light" }, "output_type": "display_data" }, { "data": { "image/png": "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\n", "text/plain": [ "
" ] }, "metadata": { "needs_background": "light" }, "output_type": "display_data" }, { "data": { "image/png": "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\n", "text/plain": [ "
" ] }, "metadata": { "needs_background": "light" }, "output_type": "display_data" }, { "data": { "image/png": "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\n", "text/plain": [ "
" ] }, "metadata": { "needs_background": "light" }, "output_type": "display_data" }, { "data": { "image/png": "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\n", "text/plain": [ "
" ] }, "metadata": { "needs_background": "light" }, "output_type": "display_data" }, { "data": { "image/png": "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\n", "text/plain": [ "
" ] }, "metadata": { "needs_background": "light" }, "output_type": "display_data" }, { "data": { "image/png": "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\n", "text/plain": [ "
" ] }, "metadata": { "needs_background": "light" }, "output_type": "display_data" }, { "data": { "image/png": "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\n", "text/plain": [ "
" ] }, "metadata": { "needs_background": "light" }, "output_type": "display_data" }, { "data": { "image/png": "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\n", "text/plain": [ "
" ] }, "metadata": { "needs_background": "light" }, "output_type": "display_data" }, { "data": { "image/png": "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\n", "text/plain": [ "
" ] }, "metadata": { "needs_background": "light" }, "output_type": "display_data" }, { "data": { "image/png": "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\n", "text/plain": [ "
" ] }, "metadata": { "needs_background": "light" }, "output_type": "display_data" }, { "data": { "image/png": "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\n", "text/plain": [ "
" ] }, "metadata": { "needs_background": "light" }, "output_type": "display_data" }, { "data": { "image/png": "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\n", "text/plain": [ "
" ] }, "metadata": { "needs_background": "light" }, "output_type": "display_data" }, { "data": { "image/png": "iVBORw0KGgoAAAANSUhEUgAAAXcAAAD8CAYAAACMwORRAAAABHNCSVQICAgIfAhkiAAAAAlwSFlzAAALEgAACxIB0t1+/AAAADl0RVh0U29mdHdhcmUAbWF0cGxvdGxpYiB2ZXJzaW9uIDMuMC4zLCBodHRwOi8vbWF0cGxvdGxpYi5vcmcvnQurowAAG3hJREFUeJzt3X2MZNV95vHvUy89M8CYAaZjYBg8sEZow2ptSAvjeHdF4hcwsoyTJbvj1cbEdoTiNVpbyjoLsUSyzl+OtY7k2GtEAjL2WjZZm5BZC4uQtS3bmzWhwcObB5Y264hhMDQvMwPMS3VV/faPe6unurqq696eqq4z7ecjtaaq7umqM7eqn/71ueeeq4jAzMzWl8qkO2BmZqPncDczW4cc7mZm65DD3cxsHXK4m5mtQw53M7N1yOFuZrYOOdzNzNYhh7uZ2TpUm9QLb926NXbs2DGplzczOyE98MADL0TE9LB2Ewv3HTt2MDs7O6mXNzM7IUn6xyLtPCxjZrYOOdzNzNYhh7uZ2TrkcDczW4eGhrukjZL+QdJDkh6T9F/6tNkg6Q5Jc5Luk7RjHJ01M7NiilTuR4Ffj4g3AW8GrpR0WU+bDwMvR8QbgT8DPj3abpqZWRlDwz0yr+Z36/lX7+WbrgZuz29/A3i7JI2sl2ZmVkqhMXdJVUm7geeBeyPivp4m24CnASKiCRwAzujzPNdJmpU0Oz8/f3w9N5uwJ597hb+fe2HS3TDrq1C4R0QrIt4MnANcKumf9TTpV6UvuzhrRNwSETMRMTM9PfQEK7OkvfPPvs+/+8veOscsDaVmy0TEfuB7wJU9m/YC2wEk1YBTgZdG0D+z5B1ZaE26C2bLFJktMy1pS357E/AO4PGeZruAa/Pb1wDfiYhllbvZevTsgSOT7oLZMkXWljkLuF1SleyXwV9FxLckfQqYjYhdwK3AVyTNkVXsO8fWY7MENFvtxduvHFmYYE/M+hsa7hHxMHBxn8dv6rp9BPit0XbNLF2vNY4Nxbx21MMylh6foWq2Cq8dbS7ePrzQXKGl2WQ43M1WoTvcXblbihzuZqvQPSxzqOHK3dLjcDdbhe5AP9Rw5W7pcbibrUKjeWy2jMPdUuRwN1uF7nDvvm2WCoe72So0uua5L7Qc7pYeh7vZKrhyt9Q53M1WoTvQXblbihzuZqvQGZapVrRkiMYsFQ53s1XoVO6nbKjRaHqNPEuPw91sFTrV+ikbaq7cLUkOd7NV6FTuJ01VWfABVUuQw91sFRrNNrWK2FivunK3JDnczVah0WwzVatQr8pTIS1JDnezVVhoZeE+Vau4crckOdzNVqHRalOvVqhXK67cLUkOd7NVONpsM1WtsKFW8UlMliSHu9kqNJptNtRcuVu6HO5mq9A5oDrlyt0S5XA3W4XOAVVX7pYqh7vZKnQOqGazZbz8gKXH4W62Co38gOpUtUKj6SsxWXqGhruk7ZK+K2mPpMckfaxPm8slHZC0O/+6aTzdNUvD0jF3V+6WnlqBNk3g9yPiQUmbgQck3RsRP+lp94OIeM/ou2iWnqPdZ6j6gKolaGjlHhHPRsSD+e1XgD3AtnF3zCxlC63OsEyVVjtotV29W1pKjblL2gFcDNzXZ/NbJT0k6duSLhpB38ySlR1QFfWaAF+NydJTONwlnQJ8E/h4RBzs2fwg8IaIeBPw58BdA57jOkmzkmbn5+dX22eziWu1IpstU81+hI56OqQlplC4S6qTBftXI+LO3u0RcTAiXs1v3w3UJW3t0+6WiJiJiJnp6enj7LrZ5Cy0g1pVTNWyHyFX7paaIrNlBNwK7ImIzw5oc2beDkmX5s/74ig7apaSVjuoVSrUKtmPUNMzZiwxRWbLvA34beARSbvzx/4QOBcgIm4GrgE+IqkJHAZ2RoQ/7bZuLbTaVCuiVvWYu6VpaLhHxA8BDWnzeeDzo+qUWeqarcgOqObh3vRsGUuMz1A1W4VWO6hVu4dlXLlbWhzuZquw0M6uoVpfHJZx5W5pcbibldRuBxEsPaDaduVuaXG4m5W0kAd5rdp9QNWVu6XF4W5WUmfaYzYs4zF3S5PD3aykzsyY7ICqZ8tYmhzuZiV1qvRaRdSqPkPV0uRwNyuptVi5d81z95i7JcbhblbSQh7udc+WsYQ53M1K6gzLVD3P3RLmcDcrqdk1LNMZc3flbqlxuJuVdGwq5LHZMq7cLTUOd7OSml0nMR2b5+5wt7Q43M1K6gR5vesMVQ/LWGoc7mYldYK8WqlQz2fLNHyZPUuMw92spMXKvdJduXtYxtLicDcrqRPk3Vdi8toylhqHu1lJ3WvLdIZlPFvGUuNwNyupU6XXq6JSEdWKfEDVkuNwNyupU6VX8znutYo8FdKS43A3K6mzcFhnjnu9WvGwjCXH4W5W0uJJTJ3KvephGUuPw92spO7lBzr/unK31DjczUrqXn4AsgOrngppqRka7pK2S/qupD2SHpP0sT5tJOlzkuYkPSzpkvF012zyFrquoQqdYRlX7paWWoE2TeD3I+JBSZuBByTdGxE/6WrzbuCC/OstwBfzf83WnVbXPHfILtrhy+xZaoZW7hHxbEQ8mN9+BdgDbOtpdjXw5cj8CNgi6ayR99YsAZ0g7wzL1KqeCmnpKTXmLmkHcDFwX8+mbcDTXff3svwXgNm6sFi5L85zr3i2jCWncLhLOgX4JvDxiDjYu7nPtywrZSRdJ2lW0uz8/Hy5npolYnH5gUpnnrs8W8aSUyjcJdXJgv2rEXFnnyZ7ge1d988B9vU2iohbImImImamp6dX01+ziVscllk8oOrK3dJTZLaMgFuBPRHx2QHNdgEfyGfNXAYciIhnR9hPs2S02kFFUOlafsCVu6WmyGyZtwG/DTwiaXf+2B8C5wJExM3A3cBVwBxwCPjg6LtqloaFVizOlIFs+YFDjeYEe2S23NBwj4gf0n9MvbtNAB8dVafMUtZqtxeHZMDz3C1NPkPVrKSFViwNdy8/YAlyuJuV1Gy3F1eEBC8/YGlyuJuV1GrH4lru0Jkt48rd0uJwNytpoRVLK/eKvPyAJcfhblbS8srdyw9YehzuZiUttNqL68pANizjyt1S43A3K6nZCuoVD8tY2hzuZiU1fUDVTgAOd7OSsqmQx8K9Xq14zN2S43A3K6n3gGq9Kha8cJglxuFuVlJ2QPXYj06tUiHi2DrvZilwuJuV1GzFkmGZzswZH1S1lDjczUrKDqguXX6g87hZKhzuZiU1223qPQuHAV5fxpLicDcrqdlafkAV8MqQlhSHu1lJzfbStWU6B1d9qT1LicPdrKRm7/IDeRXvue6WEoe7WUm9Z6h2qnjPlrGUONzNSupdW6bm2TKWIIe7WUnNdvQMy7hyt/Q43M1KavZcIHtxnrvH3C0hDnezkpqtWLr8gGfLWIIc7mYlLavcK57nbulxuJuVlFXuS9dz7zxuloqh4S7pNknPS3p0wPbLJR2QtDv/umn03TRLQ0RkB1T7zJbxsr+WklqBNl8CPg98eYU2P4iI94ykR2YJ6yzru3RYxpW7pWdo5R4R3wdeWoO+mCWvM5d96QHVzmwZV+6WjlGNub9V0kOSvi3pokGNJF0naVbS7Pz8/Ihe2mztNPtV7ovDMq7cLR2jCPcHgTdExJuAPwfuGtQwIm6JiJmImJmenh7BS5utrU513vckpqYrd0vHcYd7RByMiFfz23cDdUlbj7tnZgnqTHfsOyzjA6qWkOMOd0lnSlJ++9L8OV883uc1S1EnwOt9Fw7zsIylY+hsGUlfAy4HtkraC/wRUAeIiJuBa4CPSGoCh4GdEeFPua1LzX6Ve8UHVC09Q8M9It4/ZPvnyaZKmq17ncXBui+QXa91lh9wTWPp8BmqZiV0hl66r8RUr3hYxtLjcDcroVO5d0+F9Dx3S5HD3ayEztBLvc+Yu+e5W0oc7mYl9JvnLolaRa7cLSkOd7MSFue5V5b+6NSq8gFVS4rD3ayEzpj7VE1LHq9XKr7MniXF4W5WQuckpr6Vu2fLWEIc7mYlHFt+YGnlXqtWvPyAJcXhblZCs888d8iWI/A8d0uJw92shGPDMn0qd4+5W0Ic7mYl9DtDFbJhGs9zt5Q43M1KOLa2TO+wjCt3S4vD3ayEficxde57toylxOFuVsLisMyyqZAVD8tYUhzuZiUsHlCt9p7E5OUHLC0Od7MSBs9z97CMpcXhblbC4gHVnmGZerXCgk9isoQ43M1KaLaCakVUeue5V1y5W1oc7mYlLLTby05ggvyAqsfcLSEOd7MSmq1YNscdsmuqOtwtJQ53sxKarfayg6mQrRLp9dwtJQ53sxIW2rFsuV/wbBlLj8PdrISFZpupPpW7L9ZhqRka7pJuk/S8pEcHbJekz0mak/SwpEtG302zNDTbQa3fmHvNl9mztBSp3L8EXLnC9ncDF+Rf1wFfPP5umaVpYYUxd1fulpKh4R4R3wdeWqHJ1cCXI/MjYIuks0bVQbOUNFux7AQmyGbLeMzdUjKKMfdtwNNd9/fmj5mtO832gMrdl9mzxIwi3Jd/0qFvCSPpOkmzkmbn5+dH8NJma6sxaJ57fpm9CFfvloZRhPteYHvX/XOAff0aRsQtETETETPT09MjeGmztdVstakPqNwBWj6oaokYRbjvAj6Qz5q5DDgQEc+O4HnNktNsDZ7nDnjGjCWjNqyBpK8BlwNbJe0F/gioA0TEzcDdwFXAHHAI+OC4Oms2aQvtNhvqy39sOgdZF1ptNtara90ts2WGhntEvH/I9gA+OrIemSVs0Noyi5W7Z8xYInyGqlkJC63Bq0ICXtPdkuFwNythodWmXus/WwZcuVs6HO5mJTTbsRjk3TqVu8PdUuFwNyuh2Rqwtkw+5u5hGUuFw92shIVB89wrrtwtLQ53sxKaK6znDnjxMEuGw92shIVme+Bl9sAnMVk6HO5mJSy0hw3LuHK3NDjczUrIDqj2my3TGZZx5W5pcLibFdRuRzYVsu+wTF65e7aMJcLhblZQIx9ymepzElPNJzFZYhzuZgUdbWbhvqG2fGGwTuXe8Ji7JcLhblZQo7lC5e6FwywxDnezgjpV+Ya+wzIec7e0ONzNCjq60AL6h/tUZ1VIV+6WCIe7WUGLB1RXXM/dlbulweFuVtDRhXxYpr7C8gM+Q9US4XA3K+hY5d5ntozPULXEONzNCurMllmpcvdsGUuFw92soKPN7IBqvzH3ui+zZ4lxuJsVtOI8d5+haolxuJsVdOwM1eU/NtWKZ8tYWhzuZgUdXaFyl8RUrcJRh7slwuFuVtBKwzIAm+pVjjRaa9kls4EKhbukKyU9IWlO0g19tv+OpHlJu/Ov3x19V80ma6WFwwA21iscWXDlbmmoDWsgqQp8AXgnsBe4X9KuiPhJT9M7IuL6MfTRLAmNFcbcATbWqxxpunK3NBSp3C8F5iLiqYhoAF8Hrh5vt8zSs9JUSMiHZRYc7paGIuG+DXi66/7e/LFe/1rSw5K+IWl7vyeSdJ2kWUmz8/Pzq+iu2eQ0mtn1UyuV5ZfZA9hQr3LYwzKWiCLh3u+T3DuZ938COyLinwN/B9ze74ki4paImImImenp6XI9NZuwRrM9sGoH2FiruHK3ZBQJ971AdyV+DrCvu0FEvBgRR/O7fwH8ymi6Z5aOo802G+r9D6ZCNuZ+1OFuiSgS7vcDF0g6T9IUsBPY1d1A0lldd98L7BldF83ScGShNfBgKnTG3D0sY2kYOlsmIpqSrgfuAarAbRHxmKRPAbMRsQv4j5LeCzSBl4DfGWOfzSbi0EKLTVMrVe4VDrtyt0QMDXeAiLgbuLvnsZu6bt8I3Djarpml5XCjxclTg39kNnq2jCXEZ6iaFXSo0RxSuTvcLR0Od7OCDjVanDQs3Jsec7c0ONzNChoe7hUazTYtX2rPEuBwNyvocKPFSUPG3OHYmaxmk+RwNyvotUZzxcp9Ux7ung5pKXC4mxV0qDF8KiTg6ZCWBIe7WQGtdtBotlecCnnyhmzba0eba9Uts4Ec7mYFHGpkgb3SsMzmjXUAXjmysCZ9MluJw92sgEP5FZZWGpbZvDGr3A8eceVuk+dwNyugE+4rVe6vy8P9FYe7JcDhblbAsWGZwWPuHpaxlDjczQo4eDgL980bVgp3V+6WDoe7WQH7DzUA2HLS1MA2m+pVqhW5crckONzNCnj5UBbYp51cH9hGEps31ly5WxIc7mYFvJxX7qetULkDDndLhsPdrICXX2uwsV5ZXD9mkM0b6h6WsSQ43M0KePnQAqcPqdoBtpxU56XXGmvQI7OVOdzNCth/qLHiwdSO179uI88dPDq0ndm4OdzNCnj5UGPFg6kdWbgfoe013W3CHO5mBcy/epQzTt4wtN1Zp26k2Q5e9NCMTZjD3WyIVjt4dv8Rtp22aWjb179uIwDPHTwy7m6ZrcjhbjbEcweP0GwH27YMD/czT83C/ecHHO42WQ53syHmnn8VgH8yfcrQtudtPRmAJ557Zax9MhumULhLulLSE5LmJN3QZ/sGSXfk2++TtGPUHTWblMf2HQTgwjM3D2176qY65289md1P7x93t8xWNHgVpJykKvAF4J3AXuB+Sbsi4iddzT4MvBwRb5S0E/g08G/H0WGztfZ/nnqR86dP5vSTl06FvOvHz/CZe55g3/7DnL1lE5+44kLed/E23rR9Cz94cp5mq02t6j+ObTKKfPIuBeYi4qmIaABfB67uaXM1cHt++xvA2yVpdN00m4xHnznA/557gSsuOnPJ43f9+BluvPMRntl/mACe2X+YG+98hLt+/AxXXHQmL7za4M4Hn5lMp80oULkD24Cnu+7vBd4yqE1ENCUdAM4AXhhFJ7v98MkX+NN7Hl/2eAyYVhz03zCw/cDnGdS+3HzmkfWzZH8G9nLMzz/u92WQUfQzCF54tcHWU6b40NvOW7LtM/c8sexC2IcXWnzmnif4wR/8Gm/evoU/+ObDfOF7c2zeWEMsrXX6lT6uhn5x/OYl53Dtr+4Y62sUCfd+n7neH4UibZB0HXAdwLnnnlvgpZebqlU44+T+ZwoO+mNh0A/N4L8tBjzPgPZln7/3B31o+5LPU/Lhke23UT3/4P6v/ftyxilT/PvL3sD05qVz3PftP9z3OfbtP0ylIv77776F2//+Zzzx81d4teeC2f1+8fiUp18sK12ucVSKhPteYHvX/XOAfQPa7JVUA04FXup9ooi4BbgFYGZmZlWf50vPO51Lz7t0Nd9qNjJnb9nEM30C/ux8uuQpG2p89NfeuNbdMltUZMz9fuACSedJmgJ2Art62uwCrs1vXwN8J8qOV5idQD5xxYVs6lkhclO9yieuuHBCPTJbamjlno+hXw/cA1SB2yLiMUmfAmYjYhdwK/AVSXNkFfvOcXbabNLed/E2gL6zZcxSoEkV2DMzMzE7OzuR1zYzO1FJeiAiZoa18yRcM7N1yOFuZrYOOdzNzNYhh7uZ2TrkcDczW4cc7mZm65DD3cxsHXK4m5mtQxM7iUnSPPCPq/z2rYxhxckRSLVfkG7f3K9y3K9y1mO/3hAR08MaTSzcj4ek2SJnaK21VPsF6fbN/SrH/SrnF7lfHpYxM1uHHO5mZuvQiRrut0y6AwOk2i9It2/uVznuVzm/sP06IcfczcxsZSdq5W5mZitINtwl/ZakxyS1Jc30bLtR0pykJyRdMeD7z5N0n6QnJd2RX0Vq1H28Q9Lu/OtnknYPaPczSY/k7ca+iL2kP5b0TFffrhrQ7sp8H85JumEN+vUZSY9LeljSX0vaMqDdmuyvYf9/SRvy93gu/yztGFdful5zu6TvStqTf/4/1qfN5ZIOdL2/N427X12vveJ7o8zn8n32sKRL1qBPF3bti92SDkr6eE+bNdlnkm6T9LykR7seO13SvXkW3SvptAHfe23e5klJ1/ZrU0pEJPkF/FPgQuB7wEzX478MPARsAM4DfgpU+3z/XwE789s3Ax8Zc3//K3DTgG0/A7au4b77Y+A/DWlTzffd+cBUvk9/ecz9ehdQy29/Gvj0pPZXkf8/8B+Am/PbO4E71uC9Owu4JL+9Gfi/ffp1OfCttfo8lXlvgKuAb5Ndn/wy4L417l8V+DnZXPA132fAvwIuAR7teuxPgRvy2zf0+9wDpwNP5f+elt8+7Xj6kmzlHhF7IuKJPpuuBr4eEUcj4v8Bc8CSK2ZLEvDrwDfyh24H3jeuvuav92+Ar43rNcbgUmAuIp6KiAbwdbJ9OzYR8bcR0czv/ojsYuuTUuT/fzXZZweyz9Lb8/d6bCLi2Yh4ML/9CrAHOJGu3Xc18OXI/AjYIumsNXz9twM/jYjVniB5XCLi+2SXGu3W/TkalEVXAPdGxEsR8TJwL3Dl8fQl2XBfwTbg6a77e1n+4T8D2N8VJP3ajNK/BJ6LiCcHbA/gbyU9IOm6Mfaj2/X5n8W3DfgzsMh+HKcPkVV4/azF/iry/19sk3+WDpB9ttZEPgx0MXBfn81vlfSQpG9Lumit+sTw92bSn6udDC6yJrXPXh8Rz0L2yxv4pT5tRr7fhl4ge5wk/R1wZp9Nn4yIvxn0bX0e653yU6RNIQX7+H5WrtrfFhH7JP0ScK+kx/Pf8Ku2Ur+ALwJ/QvZ//hOyIaMP9T5Fn+897qlTRfaXpE8CTeCrA55m5PurX1f7PDa2z1FZkk4Bvgl8PCIO9mx+kGzY4dX8eMpdwAVr0S+GvzeT3GdTwHuBG/tsnuQ+K2Lk+22i4R4R71jFt+0FtnfdPwfY19PmBbI/B2t5xdWvzUj6KKkG/CbwKys8x7783+cl/TXZkMBxhVXRfSfpL4Bv9dlUZD+OvF/5gaL3AG+PfLCxz3OMfH/1UeT/32mzN3+fT2X5n9wjJ6lOFuxfjYg7e7d3h31E3C3pv0naGhFjX0OlwHszls9VQe8GHoyI53o3THKfAc9JOisins2HqJ7v02Yv2XGBjnPIjjeu2ok4LLML2JnPZDiP7LfvP3Q3yEPju8A1+UPXAoP+Ejhe7wAej4i9/TZKOlnS5s5tsoOKj/ZrOyo9Y5y/MeD17gcuUDaraIrsz9ldY+7XlcB/Bt4bEYcGtFmr/VXk/7+L7LMD2WfpO4N+IY1KPqZ/K7AnIj47oM2ZnbF/SZeS/Ry/OM5+5a9V5L3ZBXwgnzVzGXCgMySxBgb+BT2pfZbr/hwNyqJ7gHdJOi0fRn1X/tjqjfvo8Wq/yEJpL3AUeA64p2vbJ8lmOjwBvLvr8buBs/Pb55OF/hzwP4ANY+rnl4Df63nsbODurn48lH89RjY8Me599xXgEeDh/IN1Vm+/8vtXkc3G+Oka9WuObFxxd/51c2+/1nJ/9fv/A58i++UDsDH/7Mzln6Xz12Af/QuyP8cf7tpPVwG/1/mcAdfn++YhsgPTvzrufq303vT0TcAX8n36CF0z3cbct5PIwvrUrsfWfJ+R/XJ5FljI8+vDZMdp/hfwZP7v6XnbGeAvu773Q/lnbQ744PH2xWeompmtQyfisIyZmQ3hcDczW4cc7mZm65DD3cxsHXK4m5mtQw53M7N1yOFuZrYOOdzNzNah/w8Ylq+ciyHHYAAAAABJRU5ErkJggg==\n", "text/plain": [ "
" ] }, "metadata": { "needs_background": "light" }, "output_type": "display_data" }, { "data": { "image/png": "iVBORw0KGgoAAAANSUhEUgAAAXcAAAD8CAYAAACMwORRAAAABHNCSVQICAgIfAhkiAAAAAlwSFlzAAALEgAACxIB0t1+/AAAADl0RVh0U29mdHdhcmUAbWF0cGxvdGxpYiB2ZXJzaW9uIDMuMC4zLCBodHRwOi8vbWF0cGxvdGxpYi5vcmcvnQurowAAG1pJREFUeJzt3WuMZOV95/Hvry49zIDDbdrhNniwjMji1SbgFsZhY7HGay6yTDYhK3hhE9u7o3iD1payViCWSJa8iWOtI3ntmCULMraQTdYm7KwXi5DFkW1FYDf3y0AYY2MGMAyD58bMdNflvy/Oqe6a6qquc2qqu55p/z5Sa6rqPF311OmaXz/9f55zjiICMzNbWyqT7oCZmY2fw93MbA1yuJuZrUEOdzOzNcjhbma2BjnczczWIIe7mdka5HA3M1uDHO5mZmtQbVIvvHHjxti8efOkXt7M7Kj00EMPvR4R08PaTSzcN2/ezOzs7KRe3szsqCTphSLtXJYxM1uDHO5mZmuQw93MbA1yuJuZrUEOdzOzNWhouEs6RtIPJT0m6SlJ/7VPm3WS7pS0XdKDkjavRGfNzKyYIiP3OeB9EfHrwG8Al0m6sKfNx4FfRMQ7gL8CPjvebpqZWRlDwz0y+/O79fyr99p8VwK357e/CVwiSWPrpVmC/vnVfXz/uZ2T7oZZX4Vq7pKqkh4FXgPui4gHe5qcDrwIEBFNYA9w8jg7apaa3/3rf+LDt/6QA/PNSXfFbIlC4R4RrYj4DeAM4AJJ/7KnSb9R+pIrb0vaImlW0uzOnR7x2NFt31wW6j95/c0J98RsqVKrZSJiN/CPwGU9m3YAmwAk1YDjgTf6fP8tETETETPT00NPjWB2VNh3yCN3S0+R1TLTkk7Ib68H3g8809NsK3Btfvsq4P6IWDJyN1uLHO6WoiInDjsVuF1SleyXwd9GxLcl3QTMRsRW4Fbga5K2k43Yr16xHpslZt+hxqS7YLbE0HCPiMeB8/o8fmPX7UPA7423a2bparbaC7c9crcU+QhVsxEcai6G+/45h7ulx+FuNoKD862F23ON1jItzSbD4W42gkNdgT7XNYo3S4XD3WwEB7vC/ZBH7pYgh7vZCA7Me+RuaXO4m43gkEfuljiHu9kIGl1LIT1ytxQ53M1G0GwtHoDtkbulyOFuNoLOyH3DVNUjd0uSw91sBM12NnLfMFXzyN2S5HA3G0H3yL0T9GYpcbibjaBTc19frzLvsowlyOFuNoJWPlpf75G7JcrhbjaCRnuxLNO9LNIsFQ53sxF0l2W6l0WapcLhbjaCzmh9/VSVeY/cLUEOd7MRdOrs6+suy1iaHO5mI2h2L4V0WcYS5HA3G0EjD/RjXJaxRDnczUbQbLepVcRUtXLY9VTNUuFwNxtBsxXUqqJWqdCOxXXvZqlwuJuNoNEK6pUK9Zry+x69W1oc7mYjaLbb1KpZWQYc7pYeh7vZCBqtoFqpUKto4b5ZSoaGu6RNkr4raZukpyR9sk+biyXtkfRo/nXjynTXLA2tdpt6VdRr2X8hT6paamoF2jSBP4qIhyW9BXhI0n0R8XRPu+9HxAfH30Wz9HQmVOuVLNy9HNJSM3TkHhGvRMTD+e19wDbg9JXumFnKGu3eCVWXZSwtpWrukjYD5wEP9tn8HkmPSfqOpHcO+P4tkmYlze7cubN0Z81S0WxlE6r1qssylqbC4S7pOOBbwKciYm/P5oeBt0XErwP/Hbi733NExC0RMRMRM9PT06P22WziGq2gVqlQc1nGElUo3CXVyYL9joi4q3d7ROyNiP357XuAuqSNY+2pWUKa+YTqVF6W8fllLDVFVssIuBXYFhGfH9DmlLwdki7In3fXODtqlpJsQnVx5O517paaIqtlLgI+DDwh6dH8sT8BzgSIiJuBq4BPSGoCB4GrI8JDGVuzGq3s3DKdmrvLMpaaoeEeET8ANKTNF4EvjqtTZqlrtoP19arLMpYsH6FqNoLOahmXZSxVDnezEXRWy9QXzi3jkbulxeFuNoLOapl61WeFtDQ53M1G0GwF1a4JVYe7pcbhbjaCZjuoVyvUqp5QtTQ53M1G0OxZCtloe+RuaXG4m42g0e4cxOSRu6XJ4W42gmYrm1CtdU4c5muoWmIc7mYjaC6cOKwzcndZxtLicDcbQaPdGbnn4e6RuyXG4W42gt4rMXkppKXG4W5WUkTQbGdlmUpFVAQtj9wtMQ53s5I6JZjO0am1SsWnH7DkONzNSuose+yslKlV5QlVS47D3aykzgFLnZUytYo8oWrJcbiblbQwcs/DvV6t0PQRqpYYh7tZSZ0STKcsU63IR6hachzuZiX1TqjWq55QtfQ43M1KWizLdE2ouixjiXG4m5W0MKGaj9yrnlC1BDnczUrqjNw7p/utVypeCmnJcbibldQ51cDCUsiqJ1QtPQ53s5IWJ1Q7NfcKDZdlLDFDw13SJknflbRN0lOSPtmnjSR9QdJ2SY9LOn9lums2eYtLIRcPYmp5QtUSUyvQpgn8UUQ8LOktwEOS7ouIp7vaXA6cnX+9G/hy/q/ZmtPoXS1TkZdCWnKGjtwj4pWIeDi/vQ/YBpze0+xK4KuReQA4QdKpY++tWQI6yx6717l7QtVSU6rmLmkzcB7wYM+m04EXu+7vYOkvALM1oe+Jw1xzt8QUDndJxwHfAj4VEXt7N/f5liWfdklbJM1Kmt25c2e5npolYslqGZ9+wBJUKNwl1cmC/Y6IuKtPkx3Apq77ZwAv9zaKiFsiYiYiZqanp0fpr9nEdUbpta7zufsIVUtNkdUyAm4FtkXE5wc02wp8JF81cyGwJyJeGWM/zZKxEO7dpx/wyN0SU2S1zEXAh4EnJD2aP/YnwJkAEXEzcA9wBbAdOAB8dPxdNUtDZ/L0sAlV19wtMUPDPSJ+QP+aenebAP5wXJ0yS1nvhGp2yl+XZSwtPkLVrKTOicPqCxfrkI9QteQ43M1KWrIU0icOswQ53M1KavSefsDr3C1BDnezkhZOHNZ1+gGvlrHUONzNSlpy4jBfINsS5HA3K2nxxGH5hGp+4rBs0ZhZGhzuZiU1221qFZEd37c4seqyu6XE4W5WUrMVCyUZyNa5w+JEq1kKHO5mJTVasTCZCotHqnrFjKXE4W5WUrPdpto1cu+cY8Zr3S0lDnezkprtWAh0WFw145G7pcThblZSs9VeKMVA98jd4W7pcLibldQ7odq57QlVS4nD3aykRtsTqpY+h7tZSc1Wu2cpZPbfqOWjVC0hDnezkhqtwydU6wvr3D1yt3Q43M1KarZ7JlSrnlC19DjczUrKJlSXLoVsuCxjCXG4m5XUaLUXThoGiycQa3lC1RLicDcrqdkO6t0j97z+7qWQlhKHu1lJvatlFpZCuuZuCXG4m5XUu1qmU393WcZS4nA3K6lzPveOmk/5awkaGu6SbpP0mqQnB2y/WNIeSY/mXzeOv5tm6Wi2+59+wEeoWkpqBdp8Bfgi8NVl2nw/Ij44lh6ZJa7Z8oSqpW/oyD0ivge8sQp9MTsqNHuWQnYmVF1zt5SMq+b+HkmPSfqOpHeO6TnNktRoH34QU+cye14tYykpUpYZ5mHgbRGxX9IVwN3A2f0aStoCbAE488wzx/DSZquv93zunRKNj1C1lBzxyD0i9kbE/vz2PUBd0sYBbW+JiJmImJmenj7SlzabiGbvUkiP3C1BRxzukk6RpPz2Bflz7jrS5zVLVWPQicNcc7eEDC3LSPo6cDGwUdIO4E+BOkBE3AxcBXxCUhM4CFwdEf6U25q15EpMCyN3l2UsHUPDPSKuGbL9i2RLJc3WvIjwBbLtqOAjVM1K6AT4YROqXuduCXK4m5XQmTTtXgpZqQjJ69wtLQ53sxI6yx27D2KCbPTuy+xZShzuZiW0OiP3nnCvVeUJVUuKw92shIWRe/Xw/zq1ijyhaklxuJuV0Km5d0+oQhb2TR+haglxuJuVsDChWukzcnfN3RLicDcrYbEs0zOhWvWEqqXF4W5WwmJZpmfkXpXLMpYUh7tZCZ0DlXpXy1Q9oWqJcbiblbB4hOrh/3XqlYqXQlpSHO5mJXQCvLfmnq1z98jd0uFwNyuhMWi1TLXisowlxeFuVkJn0nTJOveKJ1QtLQ53sxL6nTgMsnD3UkhLicPdrIRBq2XqVU+oWloc7mYldE7r229C1af8tZQ43M1KaLQHn37AZRlLicPdrIRO6WXphKpPHGZpcbiblTBwQtXr3C0xDnezEuYHjNzrXuduiXG4m5XQKctM9YzcqxVficnS4nA3K6Ex4KyQ9aoWJlvNUuBwNythftC5ZXziMEvM0HCXdJuk1yQ9OWC7JH1B0nZJj0s6f/zdNEtD5yCmeqVPWcYjd0tIkZH7V4DLltl+OXB2/rUF+PKRd8ssTc1WUKuIypIjVL1axtIyNNwj4nvAG8s0uRL4amQeAE6QdOq4OmiWkkarvaTeDr5AtqVnHDX304EXu+7vyB9bQtIWSbOSZnfu3DmGlzZbXfOt9pJ6O0A9P0I1wqN3S8M4wn3pJx36fsIj4paImImImenp6TG8tNnqarTaS5ZBAlTzGrzL7paKcYT7DmBT1/0zgJfH8LxmyWm2YkBZJhvjNLxixhIxjnDfCnwkXzVzIbAnIl4Zw/OaJWe+1aZe61OWycPdK2YsFbVhDSR9HbgY2ChpB/CnQB0gIm4G7gGuALYDB4CPrlRnzSat0YolyyBh8SyRXutuqRga7hFxzZDtAfzh2HpklrDmwNUyHrlbWnyEqlkJjQFlmcWRu8Pd0uBwNythvhVLLtQBnlC19DjczUpoNPsvhexMqPpSe5YKh7tZCc12/7JMZ527j1K1VDjczUqYH7DOvV7plGU8crc0ONzNSmg02wNq7p5QtbQ43M1KaLTaTPVbLdOZUHVZxhLhcDcrodnuX5ZZlz8233S4Wxoc7mYlzDf7H8Q0Vcsem3O4WyIc7mYlZOdzX1qWWVerAh65Wzoc7mYlDCrLdEbuDndLhcPdrITGkLLMfKu12l0y68vhblbCoCsxeeRuqXG4m5Uw6EpMU14tY4lxuJsV1Gy1aQesq3m1jKXP4W5WUCe4Oytjuq1zuFtiHO5mBS2Ee91lGUufw92soLlmthKmX1mmUhH1qpj3+dwtEQ53s4LmGoPLMpCN3j1yt1Q43M0KWqy59/9vM1VzuFs6HO5mBS2UZfrU3MHhbmlxuJsVtNxqGcjD3TV3S4TD3aygxZr7gJF7tbIwujebtELhLukySc9K2i7p+j7bf1/STkmP5l//YfxdNZusxdUy/Ufu62pVl2UsGbVhDSRVgS8B/xbYAfxI0taIeLqn6Z0Rcd0K9NEsCcutc4esLOODmCwVRUbuFwDbI+L5iJgHvgFcubLdMkvPcuvcwROqlpYi4X468GLX/R35Y71+V9Ljkr4padNYemeWkGHr3Nd5QtUSUiTcl57fFHov8f5/gM0R8a+AfwBu7/tE0hZJs5Jmd+7cWa6nZhM2dJ27D2KyhBQJ9x1A90j8DODl7gYRsSsi5vK7fwO8q98TRcQtETETETPT09Oj9NdsYoatcz+mXuVQw6tlLA1Fwv1HwNmSzpI0BVwNbO1uIOnUrrsfAraNr4tmaeiUZfqdzx1g/VSVg/MOd0vD0NUyEdGUdB1wL1AFbouIpyTdBMxGxFbgP0v6ENAE3gB+fwX7bDYRc802tYqoDQj3DVNVDnjkbokYGu4AEXEPcE/PYzd23b4BuGG8XTNLy1yzNbDeDtnI/YBH7pYIH6FqVtBcs826ev+VMgDr69lBTK1273oDs9XncDcr6OB8i2OWGblvmMqC/8B8c7W6ZDaQw92soDfnmxy7bnAlc/1Utu2g6+6WAIe7WUH751psWCbcN+QlG6+YsRQ43M0KOjDX5Lh1g2vui2UZh7tNnsPdrKD9c002TC1XlnG4Wzoc7mYFvTnf5LjlyjKdmrvD3RLgcDcr6MBci2MLlWW8WsYmz+FuVtD+uSbHFijLeLWMpcDhblZAs9VmrtledilkZ+TusoylwOFuVsCbeWB3AryfTs19/5zLMjZ5DnezAt7MA3u5CdW3rKtREew92FitbpkN5HA3K6AzSbpcWaZSEcevr/OLAw53mzyHu1kB+w51wn1wWQbghA1T7PbI3RLgcDcr4I035wE4+dh1y7Y7fn2d3QfmV6NLZstyuJsVsGt/Hu7HTS3b7sQNdXa7LGMJcLibFfD6m9klgoeN3LOyjEfuNnkOd7MCdu2f59ip6sKBSoMcv77O7jc9crfJc7ibFbBr/xwnH7f8qB3gxA1T7Jtr0mi1V6FXZoM53M0K2PXm/NB6O8Apx2e/AH6+59BKd8lsWQ53swJ27ptjY4GR+5knHQvAC7sOrHSXzJblcDcbIiJ4YdcBNp24YWjbt52ctXnhjTdXultmy3K4mw3x872HONhosXnj8HD/1V85hqlqhZ955G4TVijcJV0m6VlJ2yVd32f7Okl35tsflLR53B01m5QnX9oLwLmn/srQttWKOGvjsTz9yt6V7pbZsoaGu6Qq8CXgcuBc4BpJ5/Y0+zjwi4h4B/BXwGfH3VGzSXng+V1M1Sq887TjD3v87kde4qK/uJ+zrv+/XPQX93P3Iy8BcNE7NvLgT97wqX9tooqM3C8AtkfE8xExD3wDuLKnzZXA7fntbwKXSNL4umk2GTv3zXH3Iy/x3rOnD1vjfvcjL3HDXU/w0u6DBPDS7oPccNcT3P3IS7zv197KfLPN13/4s8l13H7pDT7F3aLTgRe77u8A3j2oTUQ0Je0BTgZeH0cnu/3gudf5y3ufWbZNxPLPEQxpUOA5ir1Okeco0urI+pH15cjf8zjeT6F3W+j9HFk/ivbltb1ztCL45CVnH/b45+59dskVlw42Wnzu3mf5wR//G37r7I3c9O2n+co//ZQNU1WqFVGtCI94DOB3zj+Da39z84q+RpFw7/d57P1/UaQNkrYAWwDOPPPMAi+91FStwsnHDl9vPOwPhyL/yYr97THkdQo8xzj6UiQ2CvVlDK8zrEmx91vg/Qx9jgKvM2T78evrXPPuM/m1Uw6vt7+8+2Df9i/vPogk/seH38UdD/yMx3bsZq7Zpt0OWmP4RW5rw7AjncehSLjvADZ13T8DeHlAmx2SasDxwBu9TxQRtwC3AMzMzIz0Sb/grJO44KwLRvlWs7E57YT1vNQn4E87YT2QXZXpP7737avdLbMFRWruPwLOlnSWpCngamBrT5utwLX57auA+2Mc9QazRH360nNYXz989LW+XuXTl54zoR6ZHW7oyD2voV8H3AtUgdsi4ilJNwGzEbEVuBX4mqTtZCP2q1ey02aT9tvnnQ5ktfeXdx/ktBPW8+lLz1l43GzSNKkB9szMTMzOzk7ktc3MjlaSHoqImWHtfISqmdka5HA3M1uDHO5mZmuQw93MbA1yuJuZrUEOdzOzNcjhbma2BjnczczWoIkdxCRpJ/DCiN++kRU44+QYpNovSLdv7lc57lc5a7Ffb4uI6WGNJhbuR0LSbJEjtFZbqv2CdPvmfpXjfpXzy9wvl2XMzNYgh7uZ2Rp0tIb7LZPuwACp9gvS7Zv7VY77Vc4vbb+Oypq7mZkt72gduZuZ2TKSDXdJvyfpKUltSTM9226QtF3Ss5IuHfD9Z0l6UNJzku7MryI17j7eKenR/Ounkh4d0O6nkp7I2634Sewl/Zmkl7r6dsWAdpfl+3C7pOtXoV+fk/SMpMcl/Z2kEwa0W5X9Nez9S1qX/4y355+lzSvVl67X3CTpu5K25Z//T/Zpc7GkPV0/3xtXul9dr73sz0aZL+T77HFJ569Cn87p2hePStor6VM9bVZln0m6TdJrkp7seuwkSfflWXSfpBMHfO+1eZvnJF3br00pEZHkF/AvgHOAfwRmuh4/F3gMWAecBfwYqPb5/r8Frs5v3wx8YoX7+9+AGwds+ymwcRX33Z8B/2VIm2q+794OTOX79NwV7tcHgFp++7PAZye1v4q8f+A/ATfnt68G7lyFn92pwPn57bcA/9ynXxcD316tz1OZnw1wBfAdsmuPXwg8uMr9qwI/J1sLvur7DHgvcD7wZNdjfwlcn9++vt/nHjgJeD7/98T89olH0pdkR+4RsS0inu2z6UrgGxExFxE/AbYDh10xW5KA9wHfzB+6Hfjtlepr/nr/Hvj6Sr3GCrgA2B4Rz0fEPPANsn27YiLi7yOimd99gOxi65NS5P1fSfbZgeyzdEn+s14xEfFKRDyc394HbAOOpmv3XQl8NTIPACdIOnUVX/8S4McRMeoBkkckIr5HdqnRbt2fo0FZdClwX0S8ERG/AO4DLjuSviQb7ss4HXix6/4Oln74TwZ2dwVJvzbj9FvAqxHx3IDtAfy9pIckbVnBfnS7Lv+z+LYBfwYW2Y8r6WNkI7x+VmN/FXn/C23yz9Iess/WqsjLQOcBD/bZ/B5Jj0n6jqR3rlafGP6zmfTn6moGD7Imtc9+NSJegeyXN/DWPm3Gvt+GXiB7JUn6B+CUPps+ExH/e9C39Xmsd8lPkTaFFOzjNSw/ar8oIl6W9FbgPknP5L/hR7Zcv4AvA39O9p7/nKxk9LHep+jzvUe8dKrI/pL0GaAJ3DHgaca+v/p1tc9jK/Y5KkvSccC3gE9FxN6ezQ+TlR325/MpdwNnr0a/GP6zmeQ+mwI+BNzQZ/Mk91kRY99vEw33iHj/CN+2A9jUdf8M4OWeNq+T/TlYy0dc/dqMpY+SasDvAO9a5jlezv99TdLfkZUEjiisiu47SX8DfLvPpiL7cez9yieKPghcEnmxsc9zjH1/9VHk/Xfa7Mh/zsez9E/usZNUJwv2OyLirt7t3WEfEfdI+mtJGyNixc+hUuBnsyKfq4IuBx6OiFd7N0xynwGvSjo1Il7JS1Sv9Wmzg2xeoOMMsvnGkR2NZZmtwNX5SoazyH77/rC7QR4a3wWuyh+6Fhj0l8CRej/wTETs6LdR0rGS3tK5TTap+GS/tuPSU+P8dwNe70fA2cpWFU2R/Tm7dYX7dRnwx8CHIuLAgDartb+KvP+tZJ8dyD5L9w/6hTQueU3/VmBbRHx+QJtTOrV/SReQ/T/etZL9yl+ryM9mK/CRfNXMhcCeTkliFQz8C3pS+yzX/TkalEX3Ah+QdGJeRv1A/tjoVnr2eNQvslDaAcwBrwL3dm37DNlKh2eBy7sevwc4Lb/9drLQ3w78L2DdCvXzK8Af9Dx2GnBPVz8ey7+eIitPrPS++xrwBPB4/sE6tbdf+f0ryFZj/HiV+rWdrK74aP51c2+/VnN/9Xv/wE1kv3wAjsk/O9vzz9LbV2Ef/WuyP8cf79pPVwB/0PmcAdfl++Yxsonp31zpfi33s+npm4Av5fv0CbpWuq1w3zaQhfXxXY+t+j4j++XyCtDI8+vjZPM0/w94Lv/3pLztDPA/u773Y/lnbTvw0SPti49QNTNbg47GsoyZmQ3hcDczW4Mc7mZma5DD3cxsDXK4m5mtQQ53M7M1yOFuZrYGOdzNzNag/w+IUJdBdz4CawAAAABJRU5ErkJggg==\n", "text/plain": [ "
" ] }, "metadata": { "needs_background": "light" }, "output_type": "display_data" }, { "data": { "image/png": "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\n", "text/plain": [ "
" ] }, "metadata": { "needs_background": "light" }, "output_type": "display_data" }, { "data": { "image/png": "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\n", "text/plain": [ "
" ] }, "metadata": { "needs_background": "light" }, "output_type": "display_data" }, { "data": { "image/png": "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\n", "text/plain": [ "
" ] }, "metadata": { "needs_background": "light" }, "output_type": "display_data" }, { "data": { "image/png": "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\n", "text/plain": [ "
" ] }, "metadata": { "needs_background": "light" }, "output_type": "display_data" }, { "data": { "image/png": "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\n", "text/plain": [ "
" ] }, "metadata": { "needs_background": "light" }, "output_type": "display_data" }, { "data": { "image/png": "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\n", "text/plain": [ "
" ] }, "metadata": { "needs_background": "light" }, "output_type": "display_data" }, { "data": { "image/png": "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\n", "text/plain": [ "
" ] }, "metadata": { "needs_background": "light" }, "output_type": "display_data" }, { "data": { "image/png": "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\n", "text/plain": [ "
" ] }, "metadata": { "needs_background": "light" }, "output_type": "display_data" }, { "data": { "image/png": "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\n", "text/plain": [ "
" ] }, "metadata": { "needs_background": "light" }, "output_type": "display_data" }, { "data": { "image/png": "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\n", "text/plain": [ "
" ] }, "metadata": { "needs_background": "light" }, "output_type": "display_data" }, { "data": { "image/png": "iVBORw0KGgoAAAANSUhEUgAAAXcAAAD8CAYAAACMwORRAAAABHNCSVQICAgIfAhkiAAAAAlwSFlzAAALEgAACxIB0t1+/AAAADl0RVh0U29mdHdhcmUAbWF0cGxvdGxpYiB2ZXJzaW9uIDMuMC4zLCBodHRwOi8vbWF0cGxvdGxpYi5vcmcvnQurowAAGyJJREFUeJzt3X2MZNV95vHvUy/dMzATwEw7xvPiwTLLJo52A2mNsb0vyDgGI8t4s2QzjhQT26tRvEZrS4m1EEsk6/zlWOuVLHtNcEDGlmWTYMLOWngJWTtrWwqEBg9vHlgaYkIzBIYBhplh6Omq+u0fdW9PdXVVddXt6q4zxfORWlNV91TV6ds1T5/+3XPPVURgZmbjpTTqDpiZ2fA53M3MxpDD3cxsDDnczczGkMPdzGwMOdzNzMaQw93MbAw53M3MxpDD3cxsDFVG9cZbtmyJnTt3jurtzcxOSffdd98LETG1UruRhfvOnTuZmZkZ1dubmZ2SJD3VTzuXZczMxpDD3cxsDDnczczGkMPdzGwMOdzNzMaQw93MbAw53M3MxpDD3WwVIoJb75vj8PGFUXfFbAmHu9kq/O+H/4k/+MsH+LP/+8Sou2K2hMPdbBWefOEYAPWGLzRvaXG4m63CQr0BONwtPQ53s1U4Nl8D4MhrtRH3xGwph7vZKhydrwP4gKolx+Futgr5yP34Qn3EPTFbyuFutgp5uM/XHO6WFoe72Sq8loX6fK0x4p6YLeVwN1uFhXpzlsxrCw53S4vD3WwV8imQLstYahzuZqtQy+a5z3vkbolxuJutQl6W8cjdUuNwN1uFxbKMR+6WGIe72SosNLKyjGfLWGJWDHdJGyT9vaQHJD0i6b92aDMp6RZJs5LukbRzLTprlppaVpY5UW8Q4fVlLB39jNzngfdExL8EfhW4TNJFbW0+DrwUEW8D/jvw+eF20yxNrQuG5fV3sxSsGO7RdDS7W82+2j/FVwA3Z7dvBS6RpKH10ixR+aqQALWGSzOWjr5q7pLKkvYBzwN3RcQ9bU22Ak8DREQNOAyc3eF19kiakTRz8ODB1fXcLAE1j9wtUX2Fe0TUI+JXgW3ALkm/0tak0yh92Sc9Im6IiOmImJ6amhq8t2aJqdUblEtavG2WioFmy0TEy8DfApe1bZoDtgNIqgBnAC8OoX9mSas1gg2V0uJts1T0M1tmStKZ2e2NwHuBR9ua7QWuym5fCfwgPHXAXgdqjWDjRBlYWn83G7VKH23OAW6WVKb5y+AvIuJ7kj4HzETEXuBG4JuSZmmO2HevWY/NElKrN9hQzcPd4xlLx4rhHhEPAhd0ePy6ltuvAb853K6Zpa3RCBoBG7Nwd83dUuIzVM0KymvsJ8syHrlbOhzuZgXl89rzsoznuVtKHO5mBeUjddfcLUUOd7OC8qUHNlazqZCuuVtCHO5mBeVhvtEjd0uQw92soIX2A6quuVtCHO5mBdXbau41j9wtIQ53s4LykbrnuVuKHO5mBdXaZ8t4bRlLiMPdrKBa28h9wZfas4Q43M0Kykfu+QFVn8RkKXG4mxXUfoaqp0JaShzuZgUtjtx9QNUS5HA3KyhfOGxD1RfrsPQ43M0KWvAZqpYwh7tZQfnaMht8JSZLkMPdrKB8pD5Z8cJhlh6Hu1lB+WyZarlEtSyfxGRJcbibFZSXZSolUSmVPHK3pDjczQrKyzLVcolKWT6gaklZMdwlbZf0Q0n7JT0i6VMd2lws6bCkfdnXdZ1ey2yc1LOyTLkkquWSz1C1pFT6aFMDfj8i7pe0GbhP0l0R8bO2dj+OiA8Mv4tmacpH6pWyqJTEQs0jd0vHiiP3iHg2Iu7Pbh8B9gNb17pjZqnLa+zVUikbuTvcLR0D1dwl7QQuAO7psPmdkh6Q9H1Jbx9C38ySlod5uSzKJS2WacxS0E9ZBgBJm4DvAp+OiFfaNt8PvCUijkq6HLgdOK/Da+wB9gDs2LGjcKfNUpCHe7XUPKDqkbulpK+Ru6QqzWD/VkTc1r49Il6JiKPZ7TuAqqQtHdrdEBHTETE9NTW1yq6bjVZelslr7r7MnqWkn9kyAm4E9kfEF7u0eVPWDkm7stc9NMyOmqWm1j7P3SN3S0g/ZZl3A78DPCRpX/bYHwI7ACLieuBK4BOSasBxYHdE+JNuY61WD8olISkry7jmbulYMdwj4ieAVmjzZeDLw+qU2algodGgXGr+16iUtHjGqlkKfIaqWUH1elBdDPeSV4W0pDjczQqqNYJKuflfqFL2yN3S4nA3K2ih3qCSjdzLJa8tY2lxuJsVVG8ElbJr7pYmh7tZQQv1oFLKyzKeCmlpcbibFVRrNJaM3L2eu6XE4W5WUK0RizX3SrnksowlxeFuVlCt3qCaz5YpiQWfxGQJcbibFZSfoQrZAVXPlrGEONzNCmqf5+4LZFtKHO5mBdUajcUzVMueCmmJcbibFbS0LFPybBlLisPdrKBaI5YcUPU8d0uJw92soFq9ZVVIn8RkiXG4mxXUHLn7JCZLk8PdrKDakuUHRCOg4dG7JcLhblbQQqNBuWXkDrg0Y8lwuJsVVG+0XKwjO7Dq6ZCWCoe7WUG1estJTFnIewkCS4XD3ayg9ot1AF6CwJLhcDcraMnFOrIRvGvulooVw13Sdkk/lLRf0iOSPtWhjSR9SdKspAclXbg23TVLR3PkvrQsU3NZxhJR6aNNDfj9iLhf0mbgPkl3RcTPWtq8Hzgv+3oH8NXsX7OxtWQ99zzcXZaxRKw4co+IZyPi/uz2EWA/sLWt2RXAN6LpbuBMSecMvbdmCWlfFTJ/zCwFA9XcJe0ELgDuadu0FXi65f4cy38BmI2V5sU6Ti4cBlB3WcYS0Xe4S9oEfBf4dES80r65w1OWDWEk7ZE0I2nm4MGDg/XULCGNRtAIllysA5oXzTZLQV/hLqlKM9i/FRG3dWgyB2xvub8NONDeKCJuiIjpiJiempoq0l+zJOTll3xVyMWpkC7LWCL6mS0j4EZgf0R8sUuzvcBHslkzFwGHI+LZIfbTLCn5rJg81KueCmmJ6We2zLuB3wEekrQve+wPgR0AEXE9cAdwOTALvAp8dPhdNUtHXn5pP4nJK0NaKlYM94j4CZ1r6q1tAvjksDpllrp6W1nGs2UsNT5D1ayAfITeepm95uMOd0uDw92sgJMHVPPlB3yGqqXF4W5WQG2x5t62/IBH7pYIh7tZAfnSvpW2k5hcc7dUONzNCsgPqLZeZg9clrF0ONzNClioLx25+yQmS43D3ayAWts896pny1hiHO5mBeS19XxVyLLLMpYYh7tZAfk89+riyN0nMVlaHO5mBeQhXl62/IDD3dLgcDcroL0s42uoWmoc7mYF5GWZ5ZfZc83d0uBwNytgcVXIZcsPeORuaXC4mxWwbFVIT4W0xDjczQpov1hH9o+voWrJcLibFZCXZfKTlyRRLctlGUuGw92sgHrbwmHQHMU73C0VDnezAtovswfNUbxr7pYKh7tZAYtTIcsn/wuVy/LyA5YMh7tZASdPYjo5cq+USi7LWDJWDHdJN0l6XtLDXbZfLOmwpH3Z13XD76ZZWhbDvdQa7vJJTJaMSh9tvg58GfhGjzY/jogPDKVHZqeAk2eotpRlfEDVErLiyD0ifgS8uA59MTtldBq5V8vyxTosGcOqub9T0gOSvi/p7UN6TbNk1epBSVAqtU2F9GwZS0Q/ZZmV3A+8JSKOSrocuB04r1NDSXuAPQA7duwYwlubjUatEUtmykBzKQLPlrFUrHrkHhGvRMTR7PYdQFXSli5tb4iI6YiYnpqaWu1bm41Mrd5YUpIBj9wtLasOd0lvkqTs9q7sNQ+t9nXNUlZrxLJwr5Q9FdLSsWJZRtK3gYuBLZLmgD8CqgARcT1wJfAJSTXgOLA7IvwJt7FWazQWV4TMVUo+icnSsWK4R8SHV9j+ZZpTJc1eN2r1WFwRMldxWcYS4jNUzQpYqMfykbtXhbSEONzNCqg3GkuWHgAoe/kBS4jD3ayAhcbysky1JF+sw5LhcDcroFZvLF6oI+epkJYSh7tZAfVGLCvLVD0V0hLicDcrYKG+fJ572atCWkIc7mYF1BqNZcsPeLaMpcThblZArcPI3fPcLSUOd7MCah1q7p4KaSlxuJsV0Fw4rH1VSC8/YOlwuJsV0GnhsHJJ1F2WsUQ43M0KqNU9FdLS5nA3K2Chw2yZsleFtIQ43M0KqHcoy1R9gWxLiMPdrIBah1Uhy6USEfgi2ZYEh7tZASfqDarl9isxNe+7NGMpcLibFVCrd74SU3ObR+42eg53swKaa8ssP6AKuO5uSXC4mxWw0KEsk4/kXXO3FDjczQqoNTodUM3LMq652+g53M0G1GhEl/XcXZaxdKwY7pJukvS8pIe7bJekL0malfSgpAuH302zdCxks2E6TYUEH1C1NPQzcv86cFmP7e8Hzsu+9gBfXX23zNK1kIX38pq7p0JaOlYM94j4EfBijyZXAN+IpruBMyWdM6wOmqUmr6l3rbm7LGMJGEbNfSvwdMv9uewxs7GUj9yXXYnJZRlLyDDCXR0e6/jplrRH0oykmYMHDw7hrc3W30I+cu9wJSZwWcbSMIxwnwO2t9zfBhzo1DAiboiI6YiYnpqaGsJbm62/2mLNva0s49kylpBhhPte4CPZrJmLgMMR8ewQXtcsSSeykfuyqZAln8Rk6ais1EDSt4GLgS2S5oA/AqoAEXE9cAdwOTALvAp8dK06a5aCvOwy0eWA6oJPYrIErBjuEfHhFbYH8Mmh9cgscbUuB1TzqZAeuVsKfIaq2YC6lWXKXhXSEuJwNxtQHt7tZZn8AKsPqFoKHO5mA8pPYmq/zJ4XDrOUONzNBpSXZaqVLhfr8MjdEuBwNxvQ4jz3tot1VBbLMh652+g53M0GtHiGaqXLGao+oGoJcLibDWghK7u0X2av4qmQlhCHu9mATq4K2fmA6oLD3RLgcDcb0EKXJX+ri6tCuuZuo+dwNxvQySV/l47cJ7LZM15+wFLgcDcbUB7e7Scx5eE+v+Bwt9FzuJsNqNvaMpWSKOnkPHizUXK4mw3oRJczVCUxUSkxX3O42+g53M0GdCIL78nK8v8+E+XS4nazUXK4mw1ovtZgslJCWn6FyclqmflafQS9MlvK4W42oPlaveOoHZojd5dlLAUOd7MBzdcaTFbLHbdNVh3ulgaHu9mA5hcaPUfurrlbChzuZgPqVZZp1twd7jZ6DnezATUPqHYpy5RLnPABVUuAw91sQM2ae7eRu2vuloa+wl3SZZIekzQr6ZoO239X0kFJ+7Kv/zj8rpqlYX6h92wZ19wtBZWVGkgqA18Bfh2YA+6VtDciftbW9JaIuHoN+miWlPlagzM2Vjtum6w63C0N/YzcdwGzEfFkRJwAvgNcsbbdMktXfhJTJ57nbqnoJ9y3Ak+33J/LHmv37yU9KOlWSds7vZCkPZJmJM0cPHiwQHfNRm++Vu86z32i4pG7paGfcF9+jjW0X2rmfwE7I+JfAH8D3NzphSLihoiYjojpqampwXpqlohe89wnK15+wNLQT7jPAa0j8W3AgdYGEXEoIuazu18Dfm043TNLT8+yjEfuloh+wv1e4DxJ50qaAHYDe1sbSDqn5e4Hgf3D66JZWponMXWZ5+4lfy0RK86WiYiapKuBO4EycFNEPCLpc8BMROwF/rOkDwI14EXgd9ewz2Yj1Wue+0SlRK0R1BuxeMFss1FYMdwBIuIO4I62x65ruX0tcO1wu2aWnojgRI+yTD6in6/VOW2ir/9eZmvCZ6iaDWB+8UIdncsyp082H3/1hA+q2mg53M0GkIf7RJeRez5af3Xe4W6j5XA3G8CrJ2oAnDbReeS+KRu5H52vrVufzDpxuJsN4FgW2psmO9fTF0fuJxzuNloOd7MBHHmtd7ifnj3ukbuNmsPdbADHslr6pg3dwt0HVC0NDnezARydXwDg9C7THPPHj3nkbiPmcDcbwNFs5L6568jd4W5pcLibDeDoa9nIvesB1WZZ5pjLMjZiDnezAeShndfW201WSlRK8mwZGzmHu9kAjrxWY6Jc6nqGqiROmygvHng1GxWHu9kAjs3Xus6UyZ0+WXHN3UbO4W42gKPzta4lmdwvbKhy+PjCOvXIrDOHu9kAjry2wKbJzhfHzm3ZPMELR+d7tjFbaw53swEcOnaCs0+f6Nlmy6ZJDjrcbcQc7mYDOHT0BFs29Q73qU2TvHDkxDr1yKwzh7vZAA4dnefsTZM922zZPMnxhboPqtpIOdzN+vTKawscO1HnjZtXCPcs/A8ecWnGRsfhbtanZ146DsC2s07r2W4qC3/X3W2UHO5mffrHF18FYNtZG3u2e+uW0wF4/Lmja94ns276CndJl0l6TNKspGs6bJ+UdEu2/R5JO4fdUbNRe/TZI0hw3i9u6tlu21kb+YUNFR4+cHidema23IqXZ5dUBr4C/DowB9wraW9E/Kyl2ceBlyLibZJ2A58HfmstOmw2KjNPvcjbpjYtXm3p9p8+wxfufIwDLx/nzWdu5DOXns+HLtiKJH5l6xk8OPfyiHtsr2f9jNx3AbMR8WREnAC+A1zR1uYK4Obs9q3AJZI0vG6ajdZTh47xd08c4j3//I1AM9ivve0hnnn5OAE88/Jxrr3tIW7/6TMA/Nt/NsXDz7zCT//xpRH22l7PVhy5A1uBp1vuzwHv6NYmImqSDgNnAy8Mo5OtfvL4C/zpnY923BbR+TlBlw29ntP9KT1eDaLXEwftQ5F+93yfHq838Ibum4q8T+/9PdyfX/fX6v6kF46dYGO1zFXv2gnAF+58jOMLSxcHO75Q5wt3PsaHLtjKb79jB1/78T/wW392N285+zSq5RLlkih5yGPAb1y4bfGztFb6CfdOH8f2/wX9tEHSHmAPwI4dO/p46+UmKqWeZwh2+4Oh1/+p7n9jdH9Wr79Lum3q/Zwu/e71nC7bur1WtnHgTb3+CCv2vQ7vfXpt7LUfuu+7zjZvqPLb79jOm89sHkw98PLxju3yxzdvqHL7J9/FN//uKX5+6Bj1RlBv9Po1Za8nGyd6r080DP2E+xywveX+NuBAlzZzkirAGcCL7S8UETcANwBMT08X+pzvOvcN7Dp3V5Gnmg3Nm8/cyDMdAj4Pf2hOmbz28l9az26ZLeqn5n4vcJ6kcyVNALuBvW1t9gJXZbevBH4QReoTZqeIz1x6PhurS0dfG6tlPnPp+SPqkdlSK47csxr61cCdQBm4KSIekfQ5YCYi9gI3At+UNEtzxL57LTttNmofumArQMfZMmYp0KgG2NPT0zEzMzOS9zYzO1VJui8ipldq5zNUzczGkMPdzGwMOdzNzMaQw93MbAw53M3MxpDD3cxsDDnczczGkMPdzGwMjewkJkkHgacKPn0La7Di5BCk2i9It2/u12Dcr8GMY7/eEhFTKzUaWbivhqSZfs7QWm+p9gvS7Zv7NRj3azCv5365LGNmNoYc7mZmY+hUDfcbRt2BLlLtF6TbN/drMO7XYF63/Tola+5mZtbbqTpyNzOzHpINd0m/KekRSQ1J023brpU0K+kxSZd2ef65ku6R9LikW7KrSA27j7dI2pd9/VzSvi7tfi7poazdmi9iL+mPJT3T0rfLu7S7LNuHs5KuWYd+fUHSo5IelPRXks7s0m5d9tdK37+kyexnPJt9lnauVV9a3nO7pB9K2p99/j/Voc3Fkg63/HyvW+t+tbx3z5+Nmr6U7bMHJV24Dn06v2Vf7JP0iqRPt7VZl30m6SZJz0t6uOWxN0i6K8uiuySd1eW5V2VtHpd0Vac2A4mIJL+AXwLOB/4WmG55/JeBB4BJ4FzgCaDc4fl/AezObl8PfGKN+/vfgOu6bPs5sGUd990fA3+wQptytu/eCkxk+/SX17hf7wMq2e3PA58f1f7q5/sH/hNwfXZ7N3DLOvzszgEuzG5vBv5fh35dDHxvvT5Pg/xsgMuB79O81vhFwD3r3L8y8E8054Kv+z4D/g1wIfBwy2N/ClyT3b6m0+ceeAPwZPbvWdnts1bTl2RH7hGxPyIe67DpCuA7ETEfEf8AzAJLrpgtScB7gFuzh24GPrRWfc3e7z8A316r91gDu4DZiHgyIk4A36G5b9dMRPx1RNSyu3fTvNj6qPTz/V9B87MDzc/SJdnPes1ExLMRcX92+wiwHziVrt13BfCNaLobOFPSOev4/pcAT0RE0RMkVyUifkTzUqOtWj9H3bLoUuCuiHgxIl4C7gIuW01fkg33HrYCT7fcn2P5h/9s4OWWIOnUZpj+NfBcRDzeZXsAfy3pPkl71rAfra7O/iy+qcufgf3sx7X0MZojvE7WY3/18/0vtsk+S4dpfrbWRVYGugC4p8Pmd0p6QNL3Jb19vfrEyj+bUX+udtN9kDWqffaLEfEsNH95A2/s0Gbo+23FC2SvJUl/A7ypw6bPRsT/7Pa0Do+1T/npp01f+uzjh+k9an93RByQ9EbgLkmPZr/hC+vVL+CrwJ/Q/J7/hGbJ6GPtL9HhuaueOtXP/pL0WaAGfKvLywx9f3XqaofH1uxzNChJm4DvAp+OiFfaNt9Ps+xwNDuecjtw3nr0i5V/NqPcZxPAB4FrO2we5T7rx9D320jDPSLeW+Bpc8D2lvvbgANtbV6g+edgJRtxdWozlD5KqgC/Afxaj9c4kP37vKS/olkSWFVY9bvvJH0N+F6HTf3sx6H3KztQ9AHgksiKjR1eY+j7q4N+vv+8zVz2cz6D5X9yD52kKs1g/1ZE3Na+vTXsI+IOSf9D0paIWPM1VPr42azJ56pP7wfuj4jn2jeMcp8Bz0k6JyKezUpUz3doM0fzuEBuG83jjYWdimWZvcDubCbDuTR/+/59a4MsNH4IXJk9dBXQ7S+B1Xov8GhEzHXaKOl0SZvz2zQPKj7cqe2wtNU4/12X97sXOE/NWUUTNP+c3bvG/boM+C/AByPi1S5t1mt/9fP976X52YHmZ+kH3X4hDUtW078R2B8RX+zS5k157V/SLpr/jw+tZb+y9+rnZ7MX+Eg2a+Yi4HBeklgHXf+CHtU+y7R+jrpl0Z3A+ySdlZVR35c9VtxaHz0u+kUzlOaAeeA54M6WbZ+lOdPhMeD9LY/fAbw5u/1WmqE/C/wlMLlG/fw68Httj70ZuKOlHw9kX4/QLE+s9b77JvAQ8GD2wTqnvV/Z/ctpzsZ4Yp36NUuzrrgv+7q+vV/rub86ff/A52j+8gHYkH12ZrPP0lvXYR/9K5p/jj/Ysp8uB34v/5wBV2f75gGaB6bftdb96vWzaeubgK9k+/QhWma6rXHfTqMZ1me0PLbu+4zmL5dngYUsvz5O8zjN/wEez/59Q9Z2Gvjzlud+LPuszQIfXW1ffIaqmdkYOhXLMmZmtgKHu5nZGHK4m5mNIYe7mdkYcribmY0hh7uZ2RhyuJuZjSGHu5nZGPr/uKOl0QV7S+sAAAAASUVORK5CYII=\n", "text/plain": [ "
" ] }, "metadata": { "needs_background": "light" }, "output_type": "display_data" }, { "data": { "image/png": "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\n", "text/plain": [ "
" ] }, "metadata": { "needs_background": "light" }, "output_type": "display_data" }, { "data": { "image/png": "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\n", "text/plain": [ "
" ] }, "metadata": { "needs_background": "light" }, "output_type": "display_data" }, { "data": { "image/png": "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\n", "text/plain": [ "
" ] }, "metadata": { "needs_background": "light" }, "output_type": "display_data" }, { "data": { "image/png": "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\n", "text/plain": [ "
" ] }, "metadata": { "needs_background": "light" }, "output_type": "display_data" }, { "data": { "image/png": "iVBORw0KGgoAAAANSUhEUgAAAXcAAAD8CAYAAACMwORRAAAABHNCSVQICAgIfAhkiAAAAAlwSFlzAAALEgAACxIB0t1+/AAAADl0RVh0U29mdHdhcmUAbWF0cGxvdGxpYiB2ZXJzaW9uIDMuMC4zLCBodHRwOi8vbWF0cGxvdGxpYi5vcmcvnQurowAAG/lJREFUeJzt3X2MZNV95vHvU1XdM8M7eNrLMAwerBB2bSsY3BrjsC/IsOZFDngTEo21G+OXaBRvkI2UtQKxRBIireJY60iOHaPxgowty5AFQiYOiOCA5fgPMA0eBvCAGXvx0jBhmpcZGGbo7qr67R/3VndNdVVX1e3qrjPt5yO1pqruqaozt6ufPv27556riMDMzFaX0rA7YGZmg+dwNzNbhRzuZmarkMPdzGwVcribma1CDnczs1XI4W5mtgo53M3MViGHu5nZKlQZ1huvX78+Nm/ePKy3NzM7Kj366KMvR8RYt3ZDC/fNmzczMTExrLc3MzsqSfpFL+26lmUkrZX0I0mPS3pK0p+1afNxSVOSduZfv1ek02ZmNhi9jNyngQ9GxEFJI8APJd0bEQ+1tLs9Iq4ZfBfNzKxfXcM9smUjD+Z3R/IvLyVpZpawnmbLSCpL2gnsA+6PiIfbNPstSbsk3SFpU4fX2SZpQtLE1NTUErptZmaL6SncI6IWEe8FTge2SHpPS5N/ADZHxK8B3wNu7fA62yNiPCLGx8a6Huw1M7OC+prnHhH7ge8Dl7Y8/kpETOd3vw68byC9MzOzQnqZLTMm6aT89jrgYuDpljYbmu5eAeweZCfNzKw/vYzcNwAPStoFPEJWc/+upBslXZG3+Uw+TfJx4DPAx5enu2Zp2X9ohr/f+cKwu2G2QC+zZXYB57Z5/Iam29cD1w+2a2bp+8xtO/nBT6c474yT2XTKMcPujtkcry1jtgRP730dgP2HZofcE7MjOdzNlmC2Vgdg6uBbQ+6J2ZEc7mZLsHakDMDLB2eG3BOzIznczZZA+b+HZ2pD7YdZK4e72RI01uF4a9bhbmlxuJstQT2yeH9rtj7knpgdyeFutgSztTzcqx65W1oc7mZL0Ki1uyxjqXG4mxUUERyebYS7yzKWFoe7WUHT1flAn/bI3RLjcDcrqFqfv2aNa+6WGoe7WUHV2vzIfabqsoylxeFuVlDzyL0xa8YsFQ53s4JqTeFerXvkbmlxuJsVNNtUlpmteuRuaXG4mxXUPHKf9cjdEuNwNyuoueZedc3dEuNwNyuoOdCbSzRmKXC4mxXUOIhaKemIUbxZCrqGu6S1kn4k6fH8Ith/1qbNGkm3S9oj6WFJm5ejs2YpaYzc146Uj5jzbpaCXkbu08AHI+Ic4L3ApZLOb2nzKeC1iPgV4K+ALwy2m2bpaYzW146UPc/dktM13CNzML87kn+1fpKvBG7Nb98BXCRJmK1ijdky60ZLrrlbcnqquUsqS9oJ7APuj4iHW5psBJ4HiIgqcAB42yA7apaaRilm3UjZNXdLTk/hHhG1iHgvcDqwRdJ7Wpq0G6Uv+LRL2iZpQtLE1NRU/701S8iRZRmP3C0tfc2WiYj9wPeBS1s2TQKbACRVgBOBV9s8f3tEjEfE+NjYWKEOm6WiMVtmbaXsee6WnF5my4xJOim/vQ64GHi6pdkO4Or89lXAAxHhT7utanOzZUY9crf0VHposwG4VVKZ7JfB30bEdyXdCExExA7gZuBbkvaQjdi3LluPzRIxd0B1pES1HkQEnkdgqega7hGxCzi3zeM3NN1+C/jtwXbNLG2zc+FeBrIa/EjZ4W5p8BmqZgXVGjX3Rri77m4JcbibFdR8hip4ZUhLi8PdrKDmqZDgkbulxeFuVlC1pebuGTOWEoe7WUGNM1TXjmQ/Rg53S4nD3ayg+bVlXJax9DjczQqaq7lXGlMhPXK3dDjczQqaK8uMNmruHrlbOhzuZgXNj9yzHyOXZSwlDnezgqq1oCQYzcN9xgdULSEOd7OCqvWgUioxUm6M3B3ulg6Hu1lBtXqdSllUStl6Mr5gh6XE4W5W0GwtKJdEpex57pYeh7tZQbV6MFIuza0E6QOqlhKHu1lB1Xo+ci+V5u6bpcLhblZQtVanUhKVfORec7hbQhzuZgXV6kGlLMpzB1Rdc7d0ONzNCpptTIUs+SQmS4/D3aygWr1OuSTKLstYghzuZgVVa5HV3POyjK/EZCnpGu6SNkl6UNJuSU9J+mybNhdKOiBpZ/51Q7vXMltNqnnNvRHuHrlbSio9tKkCfxgRj0k6HnhU0v0R8ZOWdv8SER8efBfN0tRYfqDimrslqOvIPSL2RsRj+e03gN3AxuXumFnqGlMhGzV3z5axlPRVc5e0GTgXeLjN5g9IelzSvZLe3eH52yRNSJqYmprqu7NmKZk/iclry1h6eg53SccBdwLXRsTrLZsfA94REecAfw3c3e41ImJ7RIxHxPjY2FjRPpslobH8wFzN3WUZS0hP4S5phCzYvx0Rd7Vuj4jXI+JgfvseYETS+oH21Cwx1Vo+FXJutozD3dLRy2wZATcDuyPiSx3anJq3Q9KW/HVfGWRHzVJTrQcjZSFlAV9zzd0S0stsmQuA3wWekLQzf+yPgTMAIuIm4Crg05KqwGFga0R4GGOrWi2vuQNUSnLN3ZLSNdwj4oeAurT5CvCVQXXK7GgwW6vPTYOslOSauyXFZ6iaFdRYOAyg7JG7JcbhblZQ40pMACPlkue5W1Ic7mYF1eoxNw0yO6Dqkbulw+FuVlC2tsx8zX3WNXdLiMPdrKBqvT43cq+USx65W1Ic7mYF1WpxxGwZH1C1lDjczQqqts6WqfmAqqXD4W5WUDW/EhNkZRmP3C0lDnezgqr1YKTpDFXX3C0lDnezAmr1IALKec29XBKzLstYQhzuZgU0Tlhq1Nw9crfUONzNCmgE+fxUSM+WsbQ43M0KaJywNL8qZMmzZSwpDnezAhoj95HyfM3dZRlLicPdrIDGKH1+4TCXZSwtDnezAqotNXeP3C01DnezAuYOqM4tHFbyVEhLisPdrIBGkDfPlvHI3VLicDcrYH7k7isxWZq6hrukTZIelLRb0lOSPtumjSR9WdIeSbsknbc83TVLQ2vNvVISVa/nbgnpeoFsoAr8YUQ8Jul44FFJ90fET5raXAaclX+9H/ha/q/ZqlSdm+ee19y9cJglpuvIPSL2RsRj+e03gN3AxpZmVwLfjMxDwEmSNgy8t2aJaL/8gA+oWjr6qrlL2gycCzzcsmkj8HzT/UkW/gIwWzXaTYV0WcZS0nO4SzoOuBO4NiJeb93c5ikLPumStkmakDQxNTXVX0/NElJdsPyAD6haWnoKd0kjZMH+7Yi4q02TSWBT0/3TgRdbG0XE9ogYj4jxsbGxIv01S0Lr8gO+hqqlppfZMgJuBnZHxJc6NNsBfCyfNXM+cCAi9g6wn2ZJma0fufxApaS5x8xS0MtsmQuA3wWekLQzf+yPgTMAIuIm4B7gcmAPcAj4xOC7apaOWl6WGWm6WEcE1OtBqdSuSmm2srqGe0T8kPY19eY2AfzBoDpllrpqvXXhsFL+eDDqcLcE+AxVswKqbc5QBVx3t2Q43M0KWHAlpvxf190tFQ53swIaV2KqNM5QbYzcPdfdEuFwNyug1nKGarmp5m6WAoe7WQHtFg7LHndZxtLgcDcroN0Zqs2Pmw2bw92sgGrrlZjKni1jaXG4mxVQbbkSU2PpX5dlLBUOd7MCGiP3BWUZj9wtEQ53swIWLBzmmrslxuFuVkCjLNNYaaBRc/fI3VLhcDcroFoPRsoiWzR1vubuqzFZKhzuZgVU6zFXbwcYcVnGEuNwNyugWou5pQfAC4dZehzuZgXU6vW5OjvM19xnHe6WCIe7WQGz9ZibIQPzC4i55m6pcLibFVDrUJZxzd1S4XA3K6D1gKqnQlpqHO5mBVRba+4lL/lraXG4mxVQXVBzb8yWcc3d0tA13CXdImmfpCc7bL9Q0gFJO/OvGwbfTbO0VGv1tjX3WdfcLRGVHtp8A/gK8M1F2vxLRHx4ID0yOwrUOtTcPc/dUtF15B4RPwBeXYG+mB01GssPNLjmbqkZVM39A5Iel3SvpHd3aiRpm6QJSRNTU1MDemuzlVettYzc56ZCuuZuaRhEuD8GvCMizgH+Gri7U8OI2B4R4xExPjY2NoC3NhuObLZMU83dZRlLzJLDPSJej4iD+e17gBFJ65fcM7OEZWvLNC8c5rKMpWXJ4S7pVOXrnkrakr/mK0t9XbOUtZ7E5IXDLDVdZ8tI+g5wIbBe0iTwJ8AIQETcBFwFfFpSFTgMbI0If8JtVavVY+4qTDBfc591zd0S0TXcI+KjXbZ/hWyqpNkvjdla/YiRe6kkSvLI3dLhM1TNCqi1TIWEbDqka+6WCoe7WQHZSUxH/viUS/JUSEuGw92sgNl6/YjZMpCdpeqRu6XC4W5WQOtUSMgOqrrmbqlwuJsVMFuLI05iAiiXSl44zJLhcDcroFqvtzmgKi/5a8lwuJsVUG25zB645m5pcbibFTBbaz9y9zVULRUOd7MCqvU44jJ7kE2F9AFVS4XD3axPEUGtvrAsM1IuUXXN3RLhcDfrU6Ou3lqW8cjdUuJwN+tTo67eOhWyUpKnQloyHO5mfZrNSy8Lz1AteeRuyXC4m/VpbuReWliWcc3dUuFwN+tTY3GwdmUZT4W0VDjczfo02+GAaqXsJX8tHQ53sz7Njdxbz1D1bBlLiMPdrE+zc7NlFtbcfZk9S4XD3axPjYOmI21q7h65Wyq6hrukWyTtk/Rkh+2S9GVJeyTtknTe4Ltplo5Os2U8FdJS0svI/RvApYtsvww4K//aBnxt6d0yS9f8GaptTmLyVEhLRNdwj4gfAK8u0uRK4JuReQg4SdKGQXXQLDXzUyHbLD/gqZCWiEHU3DcCzzfdn8wfM1uV5g6oLlg4zOu5WzoGEe5q81jbT7ikbZImJE1MTU0N4K3NVt78AVUvHGbpGkS4TwKbmu6fDrzYrmFEbI+I8YgYHxsbG8Bbm628zguHlTwV0pIxiHDfAXwsnzVzPnAgIvYO4HXNkjRb67BwmEfulpBKtwaSvgNcCKyXNAn8CTACEBE3AfcAlwN7gEPAJ5ars2Yp6DRbpuyauyWka7hHxEe7bA/gDwbWI7PEzXaYLVMpOdwtHT5D1axPHU9iKmUnMWXjHbPhcrib9akxW6bdkr+A6+6WBIe7WZ8a89xHWi/WkZdpXJqxFDjczfrUGJl3Grk73C0FDnezPnU+oJr9OHkJAkuBw92sT3NTIVsv1pGHvRcPsxQ43M36NFPtvPwA+ICqpcHhbtanmWqdckkLau6NkbyXILAUONzN+jRTqzNaXvijM1LJyzKuuVsCHO5mfZqp1hmtLPzRGS2X57abDZvD3axP0x3CfU3+2HS1ttJdMlvA4W7Wp5lq+7JMI/A9crcUONzN+jRdrc2N0ps53C0lDnezPnWquc+XZRzuNnwOd7M+zdTqi47cHe6WAoe7WZ86j9yz2TI+oGopcLib9albWcY1d0uBw92sT51OYpoLd5+haglwuJv1aXq2w0lMjZr7rMPdhs/hbtan7IBqecHjox65W0J6CndJl0p6RtIeSde12f5xSVOSduZfvzf4rpqlofPyAx65Wzoq3RpIKgNfBf4zMAk8ImlHRPykpentEXHNMvTRLCmdlh+olEuUS2Km5tkyNny9jNy3AHsi4ucRMQPcBly5vN0yS9dMtdb2gCpkB1U9crcU9BLuG4Hnm+5P5o+1+i1JuyTdIWlTuxeStE3ShKSJqampAt01G75OJzFBVnd3zd1S0Eu4q81jrQtW/wOwOSJ+DfgecGu7F4qI7RExHhHjY2Nj/fXULAER0bEsA1nd3fPcLQW9hPsk0DwSPx14sblBRLwSEdP53a8D7xtM98zSUq0HEXQcua8ZKXn5AUtCL+H+CHCWpDMljQJbgR3NDSRtaLp7BbB7cF00S0djVO6Ru6Wu62yZiKhKuga4DygDt0TEU5JuBCYiYgfwGUlXAFXgVeDjy9hns6GZC/eOB1TLXlvGktA13AEi4h7gnpbHbmi6fT1w/WC7ZpaeQ7NZcB+zpv2PzmjFZRlLg89QNevDoekqAMeOtg/3dSNlDs945G7D53A368PBPNyPWbNw+QGAY9dU5tqYDZPD3awPh/JReaeR+3FrynNtzIbJ4W7WhzcbI/fRziP3Nz1ytwQ43M36MDdy73BA1WUZS4XD3awPb840Dqh2GLmPVpiu1ql6CQIbMoe7WR8OTS8+FfLY/EDrm66725A53M360Bi5rxvpXHMHXHe3oXO4m/Xh0EyNdSNlyqV26+nNh/uhGYe7DZfD3awPb05X50ov7RyXbzs47bKMDZfD3awPh2ZqHNNhjjvMz393WcaGzeFu1oc33qp2nOMOrrlbOhzuZn3Yf2iGk48Z7bj9hLUjWbvDsyvVJbO2HO5mfXj10AynHNs53MeOXwPA1BvTHduYrQSHu1kf9h+a5eRjRzpuXzda5vg1FYe7DZ3D3axH1Vqd1w7NcMoiZRmAsRPWONxt6BzuZj3a98Y0EXDqiesWbTd23Br2vfHWCvXKrD2Hu1mP9h44DMCGE9cu2u7tJ6xln0fuNmQOd7Me/b9XDwFw+smLj9xPPWEN/3rgLS8eZkPVU7hLulTSM5L2SLquzfY1km7Ptz8safOgO2o2bD996SAjZbF5/bGLtnvPxhOZrtb56UsHV6hnZgt1DXdJZeCrwGXAu4CPSnpXS7NPAa9FxK8AfwV8YdAdNRu2R3/xGv/21BMYKWc/Nnf/+AUu+IsHOPO6f+SCv3iAu3/8AgDnbjoZgB8//9rQ+mrWy8h9C7AnIn4eETPAbcCVLW2uBG7Nb98BXCSp/cpKZkeh515+k4nnXuU//eoYkAX79Xc9wQv7DxPAC/sPc/1dT3D3j19g0ynr2HDiWu58dJJaPYbbcful1XmRjHkbgeeb7k8C7+/UJiKqkg4AbwNeHkQnm/3w2Zf5y/uebrstFvk5CjpvXPR5HbYt9iMbi73gos9bZFvR/nd8ziKv1/nlFt1YdJ8s/rxOzxns97Oblw9Oc+yaCv/t/HcA8MX7nuHw7JGLgx2erfHF+57hI+du5NqLz+KP7nyC9//Pf+btx69BgpKEBB712G+edzpX//rmZX2PXsK93Wex9UeklzZI2gZsAzjjjDN6eOuFRisl3rbIGYKL/cGw2A/V4n9ntN+42HOKvpcWeeaizyvymoX7P/h9XOT9CnzL8k397+Pj1lT4r+8/g1PzmTIv7j/ctl3j8d8Z38QxoxUefHofr781S0T2A1Ev+tvFVpV1i6xPNCi9hPsksKnp/unAix3aTEqqACcCr7a+UERsB7YDjI+PF/qUbznzFLacuaXIU80G5rST1vFCm4A/7aRsJo0kfuOc0/iNc05b6a6ZAb3V3B8BzpJ0pqRRYCuwo6XNDuDq/PZVwANRtDZhdhT43CVnL7ga07qRMp+75Owh9cjsSF1H7nkN/RrgPqAM3BIRT0m6EZiIiB3AzcC3JO0hG7FvXc5Omw3bR87dCGS19xf3H+a0k9bxuUvOnnvcbNg0rAH2+Ph4TExMDOW9zcyOVpIejYjxbu18hqqZ2SrkcDczW4Uc7mZmq5DD3cxsFXK4m5mtQg53M7NVyOFuZrYKOdzNzFahoZ3EJGkK+EXBp69nGVacHIBU+wXp9s396o/71Z/V2K93RMRYt0ZDC/elkDTRyxlaKy3VfkG6fXO/+uN+9eeXuV8uy5iZrUIOdzOzVehoDfftw+5AB6n2C9Ltm/vVH/erP7+0/Toqa+5mZra4o3XkbmZmi0g23CX9tqSnJNUljbdsu17SHknPSLqkw/PPlPSwpGcl3Z5fRWrQfbxd0s786zlJOzu0e07SE3m7ZV/EXtKfSnqhqW+Xd2h3ab4P90i6bgX69UVJT0vaJenvJJ3Uod2K7K9u/39Ja/Lv8Z78s7R5ufrS9J6bJD0oaXf++f9smzYXSjrQ9P29Ybn71fTei35vlPlyvs92STpvBfp0dtO+2CnpdUnXtrRZkX0m6RZJ+yQ92fTYKZLuz7Pofkknd3ju1XmbZyVd3a5NXyIiyS/g3wFnA98HxpsefxfwOLAGOBP4GVBu8/y/Bbbmt28CPr3M/f1fwA0dtj0HrF/BffenwP/o0qac77t3AqP5Pn3XMvfrQ0Alv/0F4AvD2l+9/P+B/w7clN/eCty+At+7DcB5+e3jgZ+26deFwHdX6vPUz/cGuBy4l+wS5ecDD69w/8rAv5LNBV/xfQb8R+A84Mmmx/4SuC6/fV27zz1wCvDz/N+T89snL6UvyY7cI2J3RDzTZtOVwG0RMR0R/xfYAxxxxWxJAj4I3JE/dCvwkeXqa/5+vwN8Z7neYxlsAfZExM8jYga4jWzfLpuI+KeIqOZ3HyK72Pqw9PL/v5LsswPZZ+mi/Hu9bCJib0Q8lt9+A9gNHE3X7rsS+GZkHgJOkrRhBd//IuBnEVH0BMkliYgfkF1qtFnz56hTFl0C3B8Rr0bEa8D9wKVL6Uuy4b6IjcDzTfcnWfjhfxuwvylI2rUZpP8AvBQRz3bYHsA/SXpU0rZl7Eeza/I/i2/p8GdgL/txOX2SbITXzkrsr17+/3Nt8s/SAbLP1orIy0DnAg+32fwBSY9LulfSu1eqT3T/3gz7c7WVzoOsYe2zfxMReyH75Q28vU2bge+3rhfIXk6Svgec2mbT5yPi7zs9rc1jrVN+emnTkx77+FEWH7VfEBEvSno7cL+kp/Pf8IUt1i/ga8Cfk/2f/5ysZPTJ1pdo89wlT53qZX9J+jxQBb7d4WUGvr/adbXNY8v2OeqXpOOAO4FrI+L1ls2PkZUdDubHU+4GzlqJftH9ezPMfTYKXAFc32bzMPdZLwa+34Ya7hFxcYGnTQKbmu6fDrzY0uZlsj8HK/mIq12bgfRRUgX4TeB9i7zGi/m/+yT9HVlJYElh1eu+k/R14LttNvWyHwfer/xA0YeBiyIvNrZ5jYHvrzZ6+f832kzm3+cTWfgn98BJGiEL9m9HxF2t25vDPiLukfQ3ktZHxLKvodLD92ZZPlc9ugx4LCJeat0wzH0GvCRpQ0TszUtU+9q0mSQ7LtBwOtnxxsKOxrLMDmBrPpPhTLLfvj9qbpCHxoPAVflDVwOd/hJYqouBpyNist1GScdKOr5xm+yg4pPt2g5KS43zv3R4v0eAs5TNKhol+3N2xzL361Lgj4ArIuJQhzYrtb96+f/vIPvsQPZZeqDTL6RByWv6NwO7I+JLHdqc2qj9S9pC9nP8ynL2K3+vXr43O4CP5bNmzgcONEoSK6DjX9DD2me55s9Rpyy6D/iQpJPzMuqH8seKW+6jx0W/yEJpEpgGXgLua9r2ebKZDs8AlzU9fg9wWn77nWShvwf4P8CaZernN4Dfb3nsNOCepn48nn89RVaeWO599y3gCWBX/sHa0Nqv/P7lZLMxfrZC/dpDVlfcmX/d1Nqvldxf7f7/wI1kv3wA1uafnT35Z+mdK7CP/j3Zn+O7mvbT5cDvNz5nwDX5vnmc7MD0ry93vxb73rT0TcBX8336BE0z3Za5b8eQhfWJTY+t+D4j++WyF5jN8+tTZMdp/hl4Nv/3lLztOPC/m577yfyztgf4xFL74jNUzcxWoaOxLGNmZl043M3MViGHu5nZKuRwNzNbhRzuZmarkMPdzGwVcribma1CDnczs1Xo/wObpQNRmD8vkAAAAABJRU5ErkJggg==\n", "text/plain": [ "
" ] }, "metadata": { "needs_background": "light" }, "output_type": "display_data" }, { "data": { "image/png": "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\n", "text/plain": [ "
" ] }, "metadata": { "needs_background": "light" }, "output_type": "display_data" }, { "data": { "image/png": "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\n", "text/plain": [ "
" ] }, "metadata": { "needs_background": "light" }, "output_type": "display_data" }, { "data": { "image/png": "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\n", "text/plain": [ "
" ] }, "metadata": { "needs_background": "light" }, "output_type": "display_data" }, { "data": { "image/png": "iVBORw0KGgoAAAANSUhEUgAAAXcAAAD8CAYAAACMwORRAAAABHNCSVQICAgIfAhkiAAAAAlwSFlzAAALEgAACxIB0t1+/AAAADl0RVh0U29mdHdhcmUAbWF0cGxvdGxpYiB2ZXJzaW9uIDMuMC4zLCBodHRwOi8vbWF0cGxvdGxpYi5vcmcvnQurowAAG0tJREFUeJzt3XuMZOV95vHvU9XdczEjBph2GObiwTKLFkdZg1sTHJIIGa+5rAXemKzG0sb4plGcoNhS1lqIVyQh/8Sx1pEse43GCzJYyODFhEy8IIKDvY6lBbsZDwN4wLS9tmjA0DDMwHhudfntH3Wqp6b61FTV6equd8rPR2pNddXbVb85ffrpt3/nPacUEZiZ2WgpDbsAMzMbPIe7mdkIcribmY0gh7uZ2QhyuJuZjSCHu5nZCHK4m5mNIIe7mdkIcribmY2gsWG98Lp162LLli3Denkzs1PSY4899kpETHYbN7Rw37JlC9PT08N6eTOzU5KkX/Qyzm0ZM7MR5HA3MxtBDnczsxHkcDczG0EOdzOzEdQ13CWtlPQDSY9LekrSX+eM+bCkOUm7s4+PL025ZmbWi16WQh4F3h0RByWNA9+X9EBEPNI27u6IuH7wJZqZWb+6ztyj4WD26Xj24ffmMwP2HzrGP+5+fthlmC3QU89dUlnSbuBl4KGIeDRn2Ack7ZF0j6RNHZ5nu6RpSdNzc3OLKNssDX9y5y4+edduXjxweNilmJ2gp3CPiFpEvAPYCGyV9JttQ/4J2BIRvwV8G7i9w/PsiIipiJianOx69qxZ8nY/tx+AA4crQ67E7ER9rZaJiP3Ad4Er2u5/NSKOZp9+BXjnQKozS9yRSg2Afb86NuRKzE7Uy2qZSUlrs9urgPcAT7eNWd/y6dXA3kEWaZaqleNlAPYf8szd0tLLapn1wO2SyjR+GXwjIr4l6WZgOiJ2An8m6WqgCuwDPrxUBZulZLxcAmpuy1hyuoZ7ROwBLsy5/6aW2zcCNw62NLP0RTQWjh2r1odcidmJfIaq2SJk2c7Ram24hZi1cbibLUI9S/ejFc/cLS0Od7NFqNSycHdbxhLjcDcrqFqrc6zWCPXmkkizVDjczQpqBjt45m7pcbibFVSpHr/Ekg+oWmoc7mYFeeZuKXO4mxXUGu7uuVtqHO5mBVVaZuvVmq+CbWlxuJsVVGmZuVfqDndLi8PdrKDWtky15p67pcXhblZQpaUV47aMpcbhblZQsy0jQaXumbulxeFuVlDzSpCrx8vU3HO3xDjczQpq9txXTYyd0KIxS4HD3ayg5lLI1RNlH1C15DjczQpqztZXT5Spui1jiXG4mxVUzQ6irhwvn7Dm3SwFDnezgprLH1eOl7wU0pLTNdwlrZT0A0mPS3pK0l/njFkh6W5JM5IelbRlKYo1S0lzhczK8fL8LN4sFb3M3I8C746Ifwe8A7hC0sVtYz4GvBYRbwP+HvjsYMs0S0+zz75irOSeuyWna7hHw8Hs0/Hso31Pvga4Pbt9D3CZJA2sSrME1Vp67m7LWGp66rlLKkvaDbwMPBQRj7YN2QA8BxARVeAAcFbO82yXNC1pem5ubnGVmw1Zc7a+cswHVC09PYV7RNQi4h3ARmCrpN9sG5I3S18wlYmIHRExFRFTk5OT/VdrlpDjPXe3ZSw9fa2WiYj9wHeBK9oemgU2AUgaA04H9g2gPrNkVVsOqNbqQYQD3tLRy2qZSUlrs9urgPcAT7cN2wlcl92+Fng4vKfbiGuelbpivAzgSxBYUsZ6GLMeuF1SmcYvg29ExLck3QxMR8RO4Fbga5JmaMzYty1ZxWaJaF0tA/jiYZaUruEeEXuAC3Puv6nl9hHgDwdbmlnaavWgpOPhXqnXWUV5yFWZNfgMVbOCqvVgrFRirNRYT+DlkJYSh7tZQbV6UC6JsXLjx8hXhrSUONzNCqrWgrGSGC83Zu5+k2xLicPdrKBavU65LMZKnrlbehzuZgU1eu5iLJu5+0QmS4nD3ayg+Z77/Mzd4W7pcLibFVSpZatlmj13t2UsIQ53s4Jq9TrllgOqbstYShzuZgVV68GYD6haohzuZgXV2g6o+toylhKHu1lB1XpQLpUYb57E5Lfas4Q43M0Kas7cyyX33C09DnezgqrZUshxL4W0BDnczQqq1esnzNxrbstYQhzuZgVVas0Lh7ktY+lxuJsVVMuWQh6fuTvcLR0Od7OCmtdzd8/dUuRwNytovude9szd0tPLG2RvkvQdSXslPSXpkzljLpV0QNLu7OOmvOcyGyXVZs/dSyEtQb28QXYV+POI2CVpDfCYpIci4sdt4/41It43+BLN0rSw5+7VMpaOrjP3iHgxInZlt98A9gIblrows9TVsjNUPXO3FPXVc5e0BbgQeDTn4XdJelzSA5LePoDazJJWbTtD1T13S0kvbRkAJJ0GfBP4VES83vbwLuAtEXFQ0lXAfcB5Oc+xHdgOsHnz5sJFm6VgwZt1ONwtIT3N3CWN0wj2OyPi3vbHI+L1iDiY3b4fGJe0LmfcjoiYioipycnJRZZuNlyVWvsZqg53S0cvq2UE3ArsjYjPdxhzdjYOSVuz5311kIWapeb4zD3ruXuduyWkl7bMJcAfAU9I2p3d9xfAZoCIuAW4FviEpCpwGNgWEd7TbaRV68F4uUSpJCSvlrG0dA33iPg+oC5jvgh8cVBFmZ0KmjN3gLGS3HO3pPgMVbOCqtkZqgBlh7slxuFuVtCJM/eSe+6WFIe7WUHNde4AY2W5525JcbibFVCvBxFQzta4u+duqXG4mxVQyWbpzTfqKJfkde6WFIe7WQHNID+h5+5wt4Q43M0KaAZ562oZz9wtJQ53swJqtRPD3T13S43D3ayAZpCXy40focbM3atlLB0Od7MCajltGa9zt5Q43M0KqGaz9PIJ69wd7pYOh7tZAQtn7l4tY2lxuJsVUF2wFNIzd0uLw92sgOr8apnjB1QrNR9QtXQ43M0KWNBz98zdEuNwNyugGeTjZV/y19LkcDcroL3nPl4ueeZuSXG4mxVwfLXM8Z67Z+6WEoe7WQHNA6pZtmc9dx9QtXR0DXdJmyR9R9JeSU9J+mTOGEn6gqQZSXskXbQ05Zql4XjP3TN3S1PXN8gGqsCfR8QuSWuAxyQ9FBE/bhlzJXBe9vHbwJezf81GklfLWOq6ztwj4sWI2JXdfgPYC2xoG3YNcEc0PAKslbR+4NWaJSL3DFVfW8YS0lfPXdIW4ELg0baHNgDPtXw+y8JfAGYjo1LzGaqWtp7DXdJpwDeBT0XE6+0P53zJgj1d0nZJ05Km5+bm+qvULCELVsuU3XO3tPQU7pLGaQT7nRFxb86QWWBTy+cbgRfaB0XEjoiYioipycnJIvWaJaHa9h6qXi1jqelltYyAW4G9EfH5DsN2Ah/KVs1cDByIiBcHWKdZUnKv5+6ZuyWkl9UylwB/BDwhaXd2318AmwEi4hbgfuAqYAY4BHxk8KWapcNXhbTUdQ33iPg++T311jEB/OmgijJL3cIzVL1axtLiM1TNCsibuVfdc7eEONzNCqhl124fa3mbvXpA3a0ZS4TD3ayA+Zl7y2oZgFo43C0NDnezAqo5Z6gCPqhqyXC4mxXQfkC1GfJeDmmpcLibFXD8PVSPr3MHqHnFjCXC4W5WQK1eR4JSywFVwCtmLBkOd7MCqvWYn7VDy8zdbRlLhMPdrIBaPeYDHdxzt/Q43M0KaMzcj//4eLWMpcbhblaAZ+6WOoe7WQGVWr1Dz90HVC0NDnezAjrN3CteCmmJcLibFVCtB+Pl1p67V8tYWhzuZgUsmLmX3XO3tDjczQpoX+c+Nr9axj13S4PD3ayAWr2ev1rGPXdLhMPdrIBq7cS2jHvulhqHu1kBtXrM99nBPXdLT9dwl3SbpJclPdnh8UslHZC0O/u4afBlmqWlUo/5s1LBZ6haerq+QTbwVeCLwB0nGfOvEfG+gVRkdgqo1ettB1Q9c7e0dJ25R8T3gH3LUIvZKaNa63RVSK+WsTQMquf+LkmPS3pA0tsH9JxmyVrQc/fM3RLTS1umm13AWyLioKSrgPuA8/IGStoObAfYvHnzAF7abDiq9WB1yWeoWroWPXOPiNcj4mB2+35gXNK6DmN3RMRURExNTk4u9qXNhqbW4SQmr3O3VCw63CWdLUnZ7a3Zc7662Oc1S1m17fID5bJn7paWrm0ZSV8HLgXWSZoF/hIYB4iIW4BrgU9IqgKHgW0R4T3cRlqn1TIVH1C1RHQN94j4YJfHv0hjqaTZrw2foWqp8xmqZgW0Xzhs3D13S4zD3ayAxlLIltUy7rlbYhzuZgVUfYaqJc7hblZA+5t1+AxVS43D3ayA9p57WZ65W1oc7mYF1GonXhWyVBIluedu6XC4mxVQbbu2DDTOUvXM3VLhcDcroL3nDo2+u2fulgqHu1kBlbbVMtBYMeN17pYKh7tZn+r1IIKFM/eyvFrGkuFwN+tTs68+Xj7xx2esJPfcLRkOd7M+NfvqeT13t2UsFQ53sz5Vs9bLwp67V8tYOhzuZn062czdPXdLhcPdrE/N2fmCmXvZPXdLh8PdrE/HZ+4LD6h6nbulwuFu1qdKLb/nXnbP3RLicDfrU6eeu2fulhKHu1mf5nvu5ZylkA53S0TXcJd0m6SXJT3Z4XFJ+oKkGUl7JF00+DLN0nHymbtXy1gaepm5fxW44iSPXwmcl31sB768+LLM0nWs2gjwibYzVH0Sk6Wka7hHxPeAfScZcg1wRzQ8AqyVtH5QBZqlpnlAdcHlB8ruuVs6BtFz3wA81/L5bHbfApK2S5qWND03NzeAlzZbfpVa/rVlvFrGUjKIcFfOfbl7eETsiIipiJianJwcwEubLb/q/Mzdq2UsXYMI91lgU8vnG4EXBvC8Zkk61gz3sYU992bLxmzYBhHuO4EPZatmLgYORMSLA3hesyQ12zLtB1Qnym7LWDrGug2Q9HXgUmCdpFngL4FxgIi4BbgfuAqYAQ4BH1mqYs1S0OmA6nhZ8ytpzIata7hHxAe7PB7Anw6sIrPEVTr03MfLJbdlLBk+Q9WsT51Wy0yMOdwtHQ53sz51bsuU3JaxZDjczfrUqS0zMVaaX0ljNmwOd7M+NWfn7UshJ8ql+ZaN2bA53M361Gkp5Hi5RK0ePpHJkuBwN+tTx577mE543GyYHO5mfarW6pS08JK/zZm8++6WAoe7WZ+O1YKx8sIfnYmsB1/xihlLgMPdrE+VWn1Bvx2Ot2k8c7cUONzN+lSp1Rcsg4Tj4V6p+oCqDZ/D3axPjXDv3JbxzN1S4HA369OxauSHe9mrZSwdDnezPlXr9flZeqv5nrsPqFoCHO5mferac/fM3RLgcDfr07FqMFZyz93S5nA361OlVl9wXRlwW8bS4nA361NjnfvCtszEfFvGSyFt+BzuZn2q1jqslhlzz93S4XA369PRaq3DahkvhbR09BTukq6Q9IykGUk35Dz+YUlzknZnHx8ffKlmaThcqbFqvLzg/uZs/qh77paArm+QLakMfAn498As8ENJOyPix21D746I65egRrOkHKnUWZkT7m7LWEp6mblvBWYi4mcRcQy4C7hmacsyS9fhSo2V43lnqHq1jKWjl3DfADzX8vlsdl+7D0jaI+keSZvynkjSdknTkqbn5uYKlGs2fEcqtdyZ+6qJxn2HK7XlLslsgV7CfeGaL2hf6/VPwJaI+C3g28DteU8UETsiYioipiYnJ/ur1CwRRzu0ZVaMlZDgyDGHuw1fL+E+C7TOxDcCL7QOiIhXI+Jo9ulXgHcOpjyztNTqwbFaPfeAqiRWjZc9c7ck9BLuPwTOk3SupAlgG7CzdYCk9S2fXg3sHVyJZuk4kgV3Xs8dYNV4mUOeuVsCuq6WiYiqpOuBB4EycFtEPCXpZmA6InYCfybpaqAK7AM+vIQ1mw1Nc1aeN3OHRt/dM3dLQddwB4iI+4H72+67qeX2jcCNgy3NLD3NmfuKTuE+XuawZ+6WAJ+hataH420Zz9wtbQ53sz4cqTTWsHdsy7jnbolwuJv14XC3A6oT5fnZvdkwOdzN+nCkywHV1RPuuVsaHO5mfWi2ZTr13Fe6LWOJcLib9eFwlwOqq92WsUQ43M360Ly0gE9istQ53M368MbRKgBrVoznPr5qYozDlRoRfqs9Gy6Hu1kf3jhSAeC0lfnn/632lSEtEQ53sz68caTKmybKlEt5F0uFtasaM/r9hyrLWZbZAg53sz68frjCmpX5LRmAM940AcC+Xx1brpLMcjnczfrwxpEqazq0ZADOzML9tUMOdxsuh7tZH/YdOjY/O89zxmrP3C0NDnezPuz71THOOkm4z8/cHe42ZA53sz68evDofIDnOX3VOBLs8wFVGzKHu1mPjlXr7D9c4azTVnQcUy6JtavGPXO3oXO4m/XolweOEAEbz1h10nHrTlvBL18/skxVmeVzuJv1aPa1QwBsXHvycH/bm0/j2ZfeWI6SzDrqKdwlXSHpGUkzkm7IeXyFpLuzxx+VtGXQhZoN20+ywH7bm0876bjzz17DL/Yd8qV/bai6hrukMvAl4ErgAuCDki5oG/Yx4LWIeBvw98BnB12o2bDtfm4/605bweSaRs/9vh89zyV/+zDn3vC/ueRvH+a+Hz0PwPm/sYYIeMazdxuiXmbuW4GZiPhZRBwD7gKuaRtzDXB7dvse4DJJ+ednm52CDhyq8C9Pv8zv/5t1SOK+Hz3Pjfc+wfP7DxPA8/sPc+O9T3Dfj55nasuZlAT3PPbcsMu2X2OdT7U7bgPQupfOAr/daUxEVCUdAM4CXhlEka2+/+wr/N2DT3cd1+tF+YLeBvb8fD2M6/V6gYO+suDQtklPz9Xja/b2kj0P7PX5Xjl4lCOVGh/73XMB+NyDzyy4ONjhSo3PPfgM779wA//54rdwx//9Bf/nJ3OsXTVBv1Mdz4xG2x9ctJHrfmfLkr5GL+Get5+1/0z0MgZJ24HtAJs3b+7hpReaGCud9CSSttfrbVyPr937D2j3gb0+16BrU4/P2PPzDfJ1B75NBvf9f9OKMh+4aCNvP+d0AF7Yfzh3XPP+//YfLmDzmavZM3tg/kqSvfLFgkffqon8N3sZpF7CfRbY1PL5RuCFDmNmJY0BpwP72p8oInYAOwCmpqYK7cNbzz2TreduLfKlZgNzztpVPJ8T8OdkK2kmxkp8/Pfeutxlmc3rpef+Q+A8SedKmgC2ATvbxuwErstuXws8HH63Ahthn778/AVvkr1qvMynLz9/SBWZnajrzD3roV8PPAiUgdsi4ilJNwPTEbETuBX4mqQZGjP2bUtZtNmwvf/CDUCj9/7C/sOcs3YVn778/Pn7zYZNw5pgT01NxfT09FBe28zsVCXpsYiY6jbOZ6iamY0gh7uZ2QhyuJuZjSCHu5nZCHK4m5mNIIe7mdkIcribmY0gh7uZ2Qga2klMkuaAXxT88nUswRUnByDVuiDd2lxXf1xXf0axrrdExGS3QUML98WQNN3LGVrLLdW6IN3aXFd/XFd/fp3rclvGzGwEOdzNzEbQqRruO4ZdQAep1gXp1ua6+uO6+vNrW9cp2XM3M7OTO1Vn7mZmdhLJhrukP5T0lKS6pKm2x26UNCPpGUmXd/j6cyU9KulZSXdn7yI16BrvlrQ7+/i5pN0dxv1c0hPZuCW/iL2kv5L0fEttV3UYd0W2DWck3bAMdX1O0tOS9kj6B0lrO4xblu3V7f8vaUX2PZ7J9qUtS1VLy2tukvQdSXuz/f+TOWMulXSg5ft701LX1fLaJ/3eqOEL2TbbI+miZajp/JZtsVvS65I+1TZmWbaZpNskvSzpyZb7zpT0UJZFD0k6o8PXXpeNeVbSdXlj+hIRSX4A/xY4H/guMNVy/wXA48AK4Fzgp0A55+u/AWzLbt8CfGKJ6/3vwE0dHvs5sG4Zt91fAf+ly5hytu3eCkxk2/SCJa7rvcBYdvuzwGeHtb16+f8DfwLckt3eBty9DN+79cBF2e01wE9y6roU+NZy7U/9fG+Aq4AHaLzv+MXAo8tcXxn4JY214Mu+zYDfBy4Cnmy57++AG7LbN+Tt98CZwM+yf8/Ibp+xmFqSnblHxN6IeCbnoWuAuyLiaET8P2AGOOEds9V42/t3A/dkd90OvH+pas1e7z8BX1+q11gCW4GZiPhZRBwD7qKxbZdMRPxzRFSzTx+h8Wbrw9LL//8aGvsONPaly7Lv9ZKJiBcjYld2+w1gL3AqvXffNcAd0fAIsFbS+mV8/cuAn0ZE0RMkFyUivkfjrUZbte5HnbLocuChiNgXEa8BDwFXLKaWZMP9JDYAz7V8PsvCnf8sYH9LkOSNGaTfA16KiGc7PB7AP0t6TNL2Jayj1fXZn8W3dfgzsJftuJQ+SmOGl2c5tlcv///5Mdm+dIDGvrUssjbQhcCjOQ+/S9Ljkh6Q9Pblqonu35th71fb6DzJGtY2+42IeBEav7yBN+eMGfh26/oG2UtJ0reBs3Me+kxE/GOnL8u5r33JTy9jetJjjR/k5LP2SyLiBUlvBh6S9HT2G76wk9UFfBn4Gxr/57+h0TL6aPtT5HztopdO9bK9JH0GqAJ3dniagW+vvFJz7luy/ahfkk4Dvgl8KiJeb3t4F422w8HseMp9wHnLURfdvzfD3GYTwNXAjTkPD3Ob9WLg222o4R4R7ynwZbPAppbPNwIvtI15hcafg2PZjCtvzEBqlDQG/AHwzpM8xwvZvy9L+gcaLYFFhVWv207SV4Bv5TzUy3YceF3ZgaL3AZdF1mzMeY6Bb68cvfz/m2Nms+/z6Sz8k3vgJI3TCPY7I+Le9sdbwz4i7pf0PySti4glv4ZKD9+bJdmvenQlsCsiXmp/YJjbDHhJ0vqIeDFrUb2cM2aWxnGBpo00jjcWdiq2ZXYC27KVDOfS+O37g9YBWWh8B7g2u+s6oNNfAov1HuDpiJjNe1DSmyStad6mcVDxybyxg9LW4/yPHV7vh8B5aqwqmqDx5+zOJa7rCuC/AldHxKEOY5Zre/Xy/99JY9+Bxr70cKdfSIOS9fRvBfZGxOc7jDm72fuXtJXGz/GrS1lX9lq9fG92Ah/KVs1cDBxotiSWQce/oIe1zTKt+1GnLHoQeK+kM7I26nuz+4pb6qPHRT9ohNIscBR4CXiw5bHP0Fjp8AxwZcv99wPnZLffSiP0Z4D/BaxYojq/Cvxx233nAPe31PF49vEUjfbEUm+7rwFPAHuyHWt9e13Z51fRWI3x02Wqa4ZGX3F39nFLe13Lub3y/v/AzTR++QCszPadmWxfeusybKPfpfHn+J6W7XQV8MfN/Qy4Pts2j9M4MP07S13Xyb43bbUJ+FK2TZ+gZaXbEte2mkZYn95y37JvMxq/XF4EKll+fYzGcZp/AZ7N/j0zGzsF/M+Wr/1otq/NAB9ZbC0+Q9XMbASdim0ZMzPrwuFuZjaCHO5mZiPI4W5mNoIc7mZmI8jhbmY2ghzuZmYjyOFuZjaC/j/8L/K+Ud+GqgAAAABJRU5ErkJggg==\n", "text/plain": [ "
" ] }, "metadata": { "needs_background": "light" }, "output_type": "display_data" }, { "data": { "image/png": "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\n", "text/plain": [ "
" ] }, "metadata": { "needs_background": "light" }, "output_type": "display_data" }, { "data": { "image/png": "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\n", "text/plain": [ "
" ] }, "metadata": { "needs_background": "light" }, "output_type": "display_data" }, { "data": { "image/png": "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\n", "text/plain": [ "
" ] }, "metadata": { "needs_background": "light" }, "output_type": "display_data" }, { "data": { "image/png": "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\n", "text/plain": [ "
" ] }, "metadata": { "needs_background": "light" }, "output_type": "display_data" }, { "data": { "image/png": "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\n", "text/plain": [ "
" ] }, "metadata": { "needs_background": "light" }, "output_type": "display_data" }, { "data": { "image/png": "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\n", "text/plain": [ "
" ] }, "metadata": { "needs_background": "light" }, "output_type": "display_data" }, { "data": { "image/png": "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\n", "text/plain": [ "
" ] }, "metadata": { "needs_background": "light" }, "output_type": "display_data" }, { "data": { "image/png": "iVBORw0KGgoAAAANSUhEUgAAAXcAAAD8CAYAAACMwORRAAAABHNCSVQICAgIfAhkiAAAAAlwSFlzAAALEgAACxIB0t1+/AAAADl0RVh0U29mdHdhcmUAbWF0cGxvdGxpYiB2ZXJzaW9uIDMuMC4zLCBodHRwOi8vbWF0cGxvdGxpYi5vcmcvnQurowAAG31JREFUeJzt3X+M5HWd5/Hnq6p6mOHHMsC0Os4PB5Ujt252BftG1LsLUVeQM+DtsrtjLiv+ymS9JavJnjlYE3bX/ePONeclRk+CJxGNp7iI7KyBIK4a10tAGhyGHwNLy2loYKEVZ2Bg6Omqet8f32/1VFd/a/pb36nu+kz5eiSdqa7vp6s+8+3qV7/7XZ/v96uIwMzMxktt1BMwM7Phc7ibmY0hh7uZ2RhyuJuZjSGHu5nZGHK4m5mNIYe7mdkYcribmY0hh7uZ2RhqjOqJN23aFDt27BjV05uZnZDuueeeX0TE5ErjRhbuO3bsYHp6elRPb2Z2QpL08zLj3JYxMxtDDnczszHkcDczG0MOdzOzMeRwNzMbQw53M7Mx5HA3MxtDDnez47Bv9gD3/PxXo56G2TIjO4jJbBxc+tn/C8DP/vt/GPFMzJZy5W5W0ZFme9RTMOvL4W5W0YtHmqOegllfDnezig4vtEY9BbO+HO5mFR0+cjTc3aKx1DjczSrqrtxfmHeLxtKyYrhLWi/px5Luk/SgpL8uGPM+SXOS9uYfH1qd6Zqlo7tyn3flbokpsxRyHnhrRBySNAH8SNJtEXFnz7gbI+LK4U/RLE3dlbvbMpaaFcM9IgI4lH86kX/Eak7K7ESwpOfe8purlpZSPXdJdUl7gWeAOyLiroJhvy9pn6SbJG0b6izNEvRSV7XutoylplS4R0QrIl4PbAV2SvqtniH/AOyIiN8GvgvcUPQ4knZLmpY0PTc3dzzzNhu5VvtooLstY6kZaLVMRBwAfgBc3HP/LyNiPv/0C8Ab+nz9dRExFRFTk5MrXt/VLGnN1tHu5ELLnUpLS5nVMpOSNua3NwBvBx7uGbO569NLgf3DnKRZilrto4Huyt1SU2a1zGbgBkl1sl8G34iIb0v6BDAdEXuAP5N0KdAEngXet1oTNktFszvc/YaqJabMapl9wHkF91/Tdftq4OrhTs0sba7cLWU+QtWsou7K3atlLDUOd7OKvFrGUuZwN6uou3Lvvm2WAoe7WUWtlsPd0uVwN6uoO9BbLbdlLC0Od7OKml09d1fulhqHu1lF7rlbyhzuZhW1WsFEXdlth7slxuFuVlGzHaxv1AFYcM/dEuNwN6uo1Q4mGjVqcuVu6XG4m1XUbAf1mmjUau65W3Ic7mYVtdptGjVRr4mm2zKWGIe7WUWLlXtdrtwtOQ53s4pa7aBRE42ally4wywFDnezijqVe909d0uQw92solYraNRqTNS15AyRZilwuJtVdLRyd1vG0lPmGqrrJf1Y0n2SHpT01wVjTpJ0o6QZSXdJ2rEakzVLSbPdplHPe+5uy1hiylTu88BbI+J3gNcDF0u6oGfMB4FfRcRrgf8JfHK40zRLT2txtUzNBzFZclYM98gcyj+dyD96X8mXATfkt28C3iZJQ5ulWYKarWCiVqNRk08/YMkp1XOXVJe0F3gGuCMi7uoZsgV4HCAimsBB4KxhTtQsNa2unrsrd0tNqXCPiFZEvB7YCuyU9Fs9Q4qq9GWvdkm7JU1Lmp6bmxt8tmYJWey5170U0tIz0GqZiDgA/AC4uGfTLLANQFIDOB14tuDrr4uIqYiYmpycrDRhs1Qs9txrWnLhDrMUlFktMylpY357A/B24OGeYXuAK/LblwPfiwiXMjbWmvkRql4KaSlqlBizGbhBUp3sl8E3IuLbkj4BTEfEHuCLwFckzZBV7LtWbcZmiehU7hN1Mb/gyt3SsmK4R8Q+4LyC+6/puv0S8AfDnZpZ2rLKvUa9VmOh3Rr1dMyW8BGqZhUtVu41n37A0uNwN6uoueR87u65W1oc7mYVNVs+n7uly+FuVlGzHfm5ZXz6AUuPw92sIq9zt5Q53M0qarba+WoZ99wtPQ53s4oWL7Pn0w9YghzuZhU120E9P5+7e+6WGoe7WUWtrtMP+JS/lhqHu1kFEZFfZq9zDVVX7pYWh7tZBZ0szyp399wtPQ53swo6Sx8Xl0K6LWOJcbibVdBpw3R67u3IWjVmqXC4m1XQacN0KnfAfXdLisPdrILOQUuNmqjXs3B3391S4nA3q2Cx516vuXK3JDnczSpY2nPPfoxcuVtKHO5mFSxpy2SFuyt3S0qZC2Rvk/R9SfslPSjpIwVjLpR0UNLe/OOaoscyGxeLlXtd1Ou1JfeZpaDMBbKbwJ9HxL2STgPukXRHRDzUM+6fIuJdw5+iWXqOrpap0ahl/XeHu6Vkxco9Ip6KiHvz288D+4Etqz0xs5T1rnMHfE53S8pAPXdJO4DzgLsKNr9J0n2SbpP0uiHMzSxZvUeogit3S0uZtgwAkk4Fvgl8NCKe69l8L/CqiDgk6RLgFuCcgsfYDewG2L59e+VJm41aceXucLd0lKrcJU2QBftXI+Lm3u0R8VxEHMpv3wpMSNpUMO66iJiKiKnJycnjnLrZ6HQfodoJ97bD3RJSZrWMgC8C+yPi033GvCIfh6Sd+eP+cpgTNUvJ0cr96EFMrtwtJWXaMm8B/hi4X9Le/L6/ALYDRMS1wOXAhyU1gcPArvBZlGyMdda517sOYnLP3VKyYrhHxI8ArTDms8BnhzUps9R13lBt1OXK3ZLkI1TNKujuudcWV8t4KaSlw+FuVkGr6/QDR5dCjnJGZks53M0qaHa9oeqDmCxFDnezCrrPLeODmCxFDnezCrqPUPVBTJYih7tZBUVHqHb68GYpcLibVVB0hGrLh3ZYQhzuZhUsPULVBzFZehzuZhUUVe7uuVtKHO5mFbTyRe1L17l7KaSlw+FuVsFi5V7vqtz9hqolxOFuVkGzaLWM2zKWEIe7WQWtrp57w6tlLEEOd7MKmq3lpx9w5W4pcbibVdBqt5E6lXv2Y+Seu6XE4W5WQbMdi+2YPNtduVtSHO5mFbTasdiOWazcHe6WkDLXUN0m6fuS9kt6UNJHCsZI0mckzUjaJ+n81ZmuWRqyyj378Vm8QLbfULWElLmGahP484i4V9JpwD2S7oiIh7rGvBM4J/94I/D5/F+zsbS0cvc6d0vPipV7RDwVEffmt58H9gNbeoZdBnw5MncCGyVtHvpszRLRbLe7eu5C8hGqlpaBeu6SdgDnAXf1bNoCPN71+SzLfwGYjY3uyh2gLrnnbkkpHe6STgW+CXw0Ip7r3VzwJcte6ZJ2S5qWND03NzfYTM0S0mwdXS0DWd/dq2UsJaXCXdIEWbB/NSJuLhgyC2zr+nwr8GTvoIi4LiKmImJqcnKyynzNktBqB/X60XBvONwtMWVWywj4IrA/Ij7dZ9ge4L35qpkLgIMR8dQQ52mWlIWu1TKQVe5uy1hKyqyWeQvwx8D9kvbm9/0FsB0gIq4FbgUuAWaAF4H3D3+qZulotdtLeu6Nes2VuyVlxXCPiB9R3FPvHhPAnw5rUmap6+251/yGqiXGR6iaVdBqB41lPXcvhbR0ONzNKmi2g3pPz73lbLeEONzNKmi1l7ZlGnVX7pYWh7tZBc2eN1S9WsZS43A3q2BZ5e517pYYh7tZBc2e0w94tYylxuFuVkFRz73tcLeEONzNKmi2elfL1Fy5W1Ic7mYVuOduqXO4m1Ww0G4vOXFYtlrGSyEtHQ53swp6K/e6XLlbWhzuZhVk55Y5+uOTHcTkcLd0ONzNKlhWubvnbolxuJtV0Cy4WIdXy1hKHO5mFbS6LpANrtwtPQ53swp6j1D1uWUsNQ53swqW99xrPkLVkuJwN6ug93zu7rlbaspcIPt6Sc9IeqDP9gslHZS0N/+4ZvjTNEuLV8tY6spcIPtLwGeBLx9jzD9FxLuGMiOzxEUErZ6ee8NHqFpiVqzcI+KHwLNrMBezE0KnQl9ygWxX7paYYfXc3yTpPkm3SXrdkB7TLEmd3nrvOneHu6WkTFtmJfcCr4qIQ5IuAW4BzikaKGk3sBtg+/btQ3hqs7XXLKjcvRTSUnPclXtEPBcRh/LbtwITkjb1GXtdRExFxNTk5OTxPrXZSLRanXBfulrGlbul5LjDXdIrJCm/vTN/zF8e7+OaparzxmmjvnSduyt3S8mKbRlJXwMuBDZJmgX+EpgAiIhrgcuBD0tqAoeBXRHhV7mNrU6FvvQIVVy5W1JWDPeIeM8K2z9LtlTS7NdCcc+9RqsdRAT5H7JmI+UjVM0GdLRyX9pzB3DxbqlwuJsNqN9qmWybD2SyNDjczQbUygO81nOEarbNpbulweFuNqCFfCnkRGHl7nC3NDjczQbU7Kxzrx/98emEe2cNvNmoOdzNBrRQsM59sS3jVcCWCIe72YCai22Z7so9u+2eu6XC4W42oGarf+XunrulwuFuNqCFPMAn6ktP+QvuuVs6HO5mA1qs3AsOYvI6d0uFw91sQIsHMdWXL4Vs+w1VS4TD3WxAzT6n/AX33C0dDnezARWf8jcPd/fcLREOd7MBLRQshewEvSt3S4XD3WxARUshj65z9xuqlgaHu9mAFgreUO0sizzSdOVuaXC4mw2oU7l3t2Um8vPMeCmkpWLFcJd0vaRnJD3QZ7skfUbSjKR9ks4f/jTN0nH0xGHdlXv2o7TQcrhbGspU7l8CLj7G9ncC5+Qfu4HPH/+0zNLVXDxCtbtyz4J+watlLBErhntE/BB49hhDLgO+HJk7gY2SNg9rgmapOXqEqit3S9cweu5bgMe7Pp/N7zMbSwuL11BdHu5e526pGEa4F13qvfAVLmm3pGlJ03Nzc0N4arO112y1magLaflZIY+4crdEDCPcZ4FtXZ9vBZ4sGhgR10XEVERMTU5ODuGpzdZesx1LqnaAdQ1X7paWYYT7HuC9+aqZC4CDEfHUEB7XLEkLrfaSZZBwtHJ3z91S0VhpgKSvARcCmyTNAn8JTABExLXArcAlwAzwIvD+1ZqsWQqarViyDBJgouE3VC0tK4Z7RLxnhe0B/OnQZmSWuGa7veTi2HD0gCYvhbRU+AhVswE1W8FET8+9s8696crdEuFwNxtQsx3LKve6e+6WGIe72YAWWu1lPXdJrKvXFtfAm42aw91sQFlbZvmPTqMuFpqu3C0NDnezAWVvqC4/dm+iXvPFOiwZDnezAS20lvfcIXtT1UeoWioc7mYDarbby1bLQF65O9wtEQ53swEttJaffgDynrvXuVsiHO5mA2q1Y8m53Dsm6jUvhbRkONzNBnSk2eakRkG41xzulg6Hu9mA5pstTpooCPeGfFZIS4bD3WxA88026wraMo1azatlLBkOd7MBZW2Z+rL717nnbglxuJsNaL7ZXrw4R7dG3W0ZS4fD3WxA8wut4jdUXblbQhzuZgM60moXv6FaF0dcuVsiHO5mA2i1g4VWsK5e0HNv1DjSbI1gVmbLOdzNBnAkP+tjUeW+fqLOSwtuy1gaSoW7pIslPSJpRtJVBdvfJ2lO0t7840PDn6rZ6C2Ge0HPff1EnXlX7paIMhfIrgOfA34XmAXulrQnIh7qGXpjRFy5CnM0S0YnvItWy6xvuHK3dJSp3HcCMxHxWEQcAb4OXLa60zJL0/xi5b68575+osZLC67cLQ1lwn0L8HjX57P5fb1+X9I+STdJ2lb0QJJ2S5qWND03N1dhumaj1anc+7Vlmu3waX8tCWXCffm5TaF3vdc/ADsi4reB7wI3FD1QRFwXEVMRMTU5OTnYTM0S0KncC9sy+ZusL/lSe5aAMuE+C3RX4luBJ7sHRMQvI2I+//QLwBuGMz2ztMyv8IYq4NaMJaFMuN8NnCPpbEnrgF3Anu4BkjZ3fXopsH94UzRLx/zCMXruDYe7pWPF1TIR0ZR0JXA7UAeuj4gHJX0CmI6IPcCfSboUaALPAu9bxTmbjUznrI+FbZl1DndLx4rhDhARtwK39tx3Tdftq4Grhzs1s/TMLxzjDdX8Pi+HtBT4CFWzAXR67uv7HKEKrtwtDQ53swF0jlAtOrfM0XB35W6j53A3G8DhvCovrtw7bRlX7jZ6DnezAbx4pAnAKSctf7tqsXL3+WUsAQ53swG8MJ8F94aJYy2FdFvGRs/hbjaAF+abnLKuTq22/MDt9evclrF0ONzNBvDCkRYnF7RkAE5el93/wnxzLadkVsjhbjaAF49klXuRU9bVadTEwcMLazwrs+Uc7mYDeGG+uVih95LExpMnOOBwtwQ43M0GcPDwAqdvmOi7/Tc2TLhytyQ43M0GsFK4b9wwwcEXHe42eg53swEcePHY4X76hgkOHD6yhjMyK+ZwNxvAwcMLnH7yMSr3k9e5LWNJcLiblXRovsl8s82Zp6zrO+b0DRMccFvGEuBwNyvpmedeAuDlv3FS3zGnb5jg+Zeavo6qjZzD3aykp5/LriT5stPW9x2z+fRs21MHX1qTOZn143A3K+nxX70IwJaNG/qOec3LTgVgZu7QmszJrJ9S4S7pYkmPSJqRdFXB9pMk3Zhvv0vSjmFP1GzUHpt7gYm62HpG/3B/7WQW7j99xuFuo7XiZfYk1YHPAb8LzAJ3S9oTEQ91Dfsg8KuIeK2kXcAngT9ajQmbjcp9jx/gX738NBr1rCa65SdP8KnbH+HJA4d55cYNfOyic3n3eVs485R1/PPTz494tvbrrkzlvhOYiYjHIuII8HXgsp4xlwE35LdvAt4maflp88xOUP9y8CWmf/4sb3ntJiAL9qtvvp8nDhwmgCcOHObqm+/nlp88wb/ZcQbfe3iOQz6BmI1QmQtkbwEe7/p8FnhjvzER0ZR0EDgL+MUwJtntR4/+gr+9/eEVx0WUe7yg3MDSj1diXMmHIso+aenHKzlu2Puk1GOVfM5yT1l6YNnHm3t+nprEf3rjdgA+dfsji1dl6ji80OJTtz/Cp//wd/jOQ3fy5v/2j2w942QkqElI4IrHAH7v/K1c8eYdq/ocZcK96PXY+zNRZgySdgO7AbZv317iqZdb16hx1jHWGfc8X7lxJZ+7/N8iKw8s+1jDnptKPmLpxxvm8w59nwzv+3/q+gZ/NLWNV511CgBPHjhcOO7JA4d546vP4v986AK+9ZNZnn3hCBHZD0N7yL+s7cS1oc+ZRYepTLjPAtu6Pt8KPNlnzKykBnA68GzvA0XEdcB1AFNTU5Ve6TvPPpOdZ++s8qVmQ/PKjRt4oiDgX5mvpHnTa87iTa85a62nZbaoTM/9buAcSWdLWgfsAvb0jNkDXJHfvhz4Xgy7p2CWkI9ddO6yS+1tmKjzsYvOHdGMzJZasXLPe+hXArcDdeD6iHhQ0ieA6YjYA3wR+IqkGbKKfddqTtps1N593haAwtUyZinQqArsqampmJ6eHslzm5mdqCTdExFTK43zEapmZmPI4W5mNoYc7mZmY8jhbmY2hhzuZmZjyOFuZjaGHO5mZmPI4W5mNoZGdhCTpDng5xW/fBOrcMbJIUh1XpDu3DyvwXhegxnHeb0qIiZXGjSycD8ekqbLHKG11lKdF6Q7N89rMJ7XYH6d5+W2jJnZGHK4m5mNoRM13K8b9QT6SHVekO7cPK/BeF6D+bWd1wnZczczs2M7USt3MzM7hmTDXdIfSHpQUlvSVM+2qyXNSHpE0kV9vv5sSXdJelTSjflVpIY9xxsl7c0/fiZpb59xP5N0fz5u1U9iL+mvJD3RNbdL+oy7ON+HM5KuWoN5fUrSw5L2SfqWpI19xq3J/lrp/y/ppPx7PJO/lnas1ly6nnObpO9L2p+//j9SMOZCSQe7vr/XrPa8up77mN8bZT6T77N9ks5fgzmd27Uv9kp6TtJHe8asyT6TdL2kZyQ90HXfmZLuyLPoDkln9PnaK/Ixj0q6omjMQCIiyQ/gXwPnAj8Aprru/03gPuAk4Gzgp0C94Ou/AezKb18LfHiV5/s/gGv6bPsZsGkN991fAf9lhTH1fN+9GliX79PfXOV5vQNo5Lc/CXxyVPurzP8f+M/AtfntXcCNa/C92wycn98+DfjngnldCHx7rV5Pg3xvgEuA28iuO34BcNcaz68O/AvZWvA132fAvwfOBx7ouu9vgavy21cVve6BM4HH8n/PyG+fcTxzSbZyj4j9EfFIwabLgK9HxHxE/D9gBlhyxWxll71/K3BTftcNwLtXa6758/0h8LXVeo5VsBOYiYjHIuII8HWyfbtqIuI7EdHMP72T7GLro1Lm/38Z2WsHstfS2/Lv9aqJiKci4t789vPAfuBEunbfZcCXI3MnsFHS5jV8/rcBP42IqgdIHpeI+CHZpUa7db+O+mXRRcAdEfFsRPwKuAO4+Hjmkmy4H8MW4PGuz2dZ/uI/CzjQFSRFY4bp3wFPR8SjfbYH8B1J90javYrz6HZl/mfx9X3+DCyzH1fTB8gqvCJrsb/K/P8Xx+SvpYNkr601kbeBzgPuKtj8Jkn3SbpN0uvWak6s/L0Z9etqF/2LrFHts5dHxFOQ/fIGXlYwZuj7bcULZK8mSd8FXlGw6eMR8ff9vqzgvt4lP2XGlFJyju/h2FX7WyLiSUkvA+6Q9HD+G76yY80L+DzwN2T/578haxl9oPchCr72uJdOldlfkj4ONIGv9nmYoe+voqkW3Ldqr6NBSToV+Cbw0Yh4rmfzvWRth0P5+ym3AOesxbxY+Xszyn22DrgUuLpg8yj3WRlD328jDfeIeHuFL5sFtnV9vhV4smfML8j+HGzkFVfRmKHMUVID+D3gDcd4jCfzf5+R9C2ylsBxhVXZfSfpC8C3CzaV2Y9Dn1f+RtG7gLdF3mwseIyh768CZf7/nTGz+ff5dJb/yT10kibIgv2rEXFz7/busI+IWyX9L0mbImLVz6FS4nuzKq+rkt4J3BsRT/duGOU+A56WtDkinspbVM8UjJkle1+gYyvZ+42VnYhtmT3Arnwlw9lkv31/3D0gD43vA5fnd10B9PtL4Hi9HXg4ImaLNko6RdJpndtkbyo+UDR2WHp6nP+xz/PdDZyjbFXROrI/Z/es8rwuBv4rcGlEvNhnzFrtrzL//z1krx3IXkvf6/cLaVjynv4Xgf0R8ek+Y17R6f1L2kn2c/zL1ZxX/lxlvjd7gPfmq2YuAA52WhJroO9f0KPaZ7nu11G/LLodeIekM/I26jvy+6pb7XePq36QhdIsMA88Ddzete3jZCsdHgHe2XX/rcAr89uvJgv9GeDvgJNWaZ5fAv6k575XArd2zeO+/ONBsvbEau+7rwD3A/vyF9bm3nnln19Cthrjp2s0rxmyvuLe/OPa3nmt5f4q+v8DnyD75QOwPn/tzOSvpVevwT76t2R/ju/r2k+XAH/SeZ0BV+b75j6yN6bfvNrzOtb3pmduAj6X79P76VrptspzO5ksrE/vum/N9xnZL5engIU8vz5I9j7NPwKP5v+emY+dAv5319d+IH+tzQDvP965+AhVM7MxdCK2ZczMbAUOdzOzMeRwNzMbQw53M7Mx5HA3MxtDDnczszHkcDczG0MOdzOzMfT/AXxN6EpHubhIAAAAAElFTkSuQmCC\n", "text/plain": [ "
" ] }, "metadata": { "needs_background": "light" }, "output_type": "display_data" }, { "data": { "image/png": "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\n", "text/plain": [ "
" ] }, "metadata": { "needs_background": "light" }, "output_type": "display_data" }, { "data": { "image/png": "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\n", "text/plain": [ "
" ] }, "metadata": { "needs_background": "light" }, "output_type": "display_data" }, { "data": { "image/png": "iVBORw0KGgoAAAANSUhEUgAAAXcAAAD8CAYAAACMwORRAAAABHNCSVQICAgIfAhkiAAAAAlwSFlzAAALEgAACxIB0t1+/AAAADl0RVh0U29mdHdhcmUAbWF0cGxvdGxpYiB2ZXJzaW9uIDMuMC4zLCBodHRwOi8vbWF0cGxvdGxpYi5vcmcvnQurowAAGvhJREFUeJzt3X+Q3HWd5/Hnq3t+JPwMISOEEAy6FHe45QpOZVHvrihR+VEWeC67F/9YQd1KrbfUatWedbBWsbvsH1eudV6dpycVF06kLMFDlstaoVg8tVz/ABliCGDgGDmVIZyMhAQSwsx09/v+6G9POj3fznR/0z39SfN6VE2lu7+f6X7nOz2vvPPpz/f7VURgZmbDpTToAszMrPcc7mZmQ8jhbmY2hBzuZmZDyOFuZjaEHO5mZkPI4W5mNoQc7mZmQ8jhbmY2hEYG9cLr1q2LTZs2DerlzcxOSI899thvI2JiuXEDC/dNmzYxNTU1qJc3MzshSfpVJ+M8LWNmNoQc7mZmQ8jhbmY2hBzuZmZDyOFuZjaEHO5mZkPI4W5mNoQc7mbH4We/foW7f/rrQZdhtsTADmIyGwY3/I9HOXB4gQ9edBZnnjI+6HLMFrlzNzsOBw4vAHBorjrgSsyO5nA364FD85VBl2B2FIe7WQ+87nC3xDjczXrg9XlPy1haHO5mPeA5d0uNw92soEq1tnjb0zKWGoe7WUHzTeE+V6kdY6TZynO4mxVUqcXi7YWqw93Ssmy4S1ol6aeSHpf0lKS/yRlzg6RZSbuyrz/pT7lm6ahWj4T7vDt3S0wnR6jOAe+PiIOSRoGfSHogIh5uGXdPRNzY+xLN0nR05x7HGGm28pYN94gI4GB2dzT78jvZ3vSqnpaxhHU05y6pLGkX8BLwUEQ8kjPsDyTtlnSvpI1tnmerpClJU7Ozs8dRttngVWpHAt3hbqnpKNwjohoR7wLOBTZL+t2WIf8IbIqIdwLfB+5s8zzbImIyIiYnJiaOp26zgWvu3Ocd7paYrlbLRMR+4EfAlS2PvxwRc9ndrwPv7kl1Zgk7as694plKS0snq2UmJK3Jbq8GPgA83TJmfdPda4A9vSzSLEWec7eUdbJaZj1wp6Qy9X8MvhMR35N0KzAVEduBP5d0DVAB9gE39Ktgs1RUqg53S1cnq2V2AxfnPH5L0+2bgZt7W5pZ2jznbinzEapmBTWvlvFBTJYah7tZQZ5zt5Q53M0Kal4t0xz0ZilwuJsV1BzoFYe7JcbhblZQ81SMO3dLjcPdrKBGoI+PlDznbslxuJsVVGkKd3fulhqHu1lBjUBfNVr2nLslx+FuVlClKdzduVtqHO5mBVWzg5jGR0pHnYrALAUOd7OCGoE+Puo5d0uPw92soCOrZcpHnYrALAUOd7OCvFrGUuZwNyuoeZ27V8tYahzuZgVVmqZl3LlbahzuZgUtrpYZdedu6XG4mxXkOXdLWSfXUF0l6aeSHpf0lKS/yRkzLukeSdOSHpG0qR/FmqWkWm1aLeNzy1hiOunc54D3R8TvAe8CrpR0acuYTwGvRMTvAP8F+EJvyzRLT6NzH3PnbglaNtyj7mB2dzT7an0nXwvcmd2+F7hcknpWpVmCqrWgXBIjZbHgcLfEdDTnLqksaRfwEvBQRDzSMmQD8DxARFSAA8CZOc+zVdKUpKnZ2dnjq9xswCqNcC/Jnbslp6Nwj4hqRLwLOBfYLOl3W4bkdelL3u0RsS0iJiNicmJiovtqzRJSrdUYKYlyqT4tE+GAt3R0tVomIvYDPwKubNk0A2wEkDQCnA7s60F9ZslaqNY799FSvbdx924p6WS1zISkNdnt1cAHgKdbhm0Hrs9uXwf8INzG2JCr1qLeuZfr4e617paSkQ7GrAfulFSm/o/BdyLie5JuBaYiYjtwO3CXpGnqHfuWvlVsloj6nHuJEXfulqBlwz0idgMX5zx+S9PtN4A/7G1pZmlrnnMHd+6WFh+halZQpRaMlOXO3ZLkcDcraHHOvdSYc/dRqpYOh7tZQc3r3MGdu6XF4W5WULUajJRKRzp3X0fVEuJwNyuo0nT6AXDnbmlxuJsVVK3VGCl7tYylyeFuVpDn3C1lDnezgrxaxlLmcDcrqLVz9weqlhKHu1lB9c69abWMp2UsIQ53s4Ianftouf5r5Dl3S4nD3aygI+eW8Zy7pcfhblZQperVMpYuh7tZQdXsxGGec7cUOdzNCjpyPvfsICavlrGEONzNCqpkc+5ZtntaxpLicDcrqLo45+7VMpaeTq6hulHSDyXtkfSUpM/kjLlM0gFJu7KvW/Key2yYVFqOUK36ssGWkE6uoVoB/iIidko6FXhM0kMR8fOWcf8cER/ufYlmaaouuRKTl0JaOpbt3CPixYjYmd1+DdgDbOh3YWapq7QeoeoPVC0hXc25S9pE/WLZj+Rsfo+kxyU9IOkdbb5/q6QpSVOzs7NdF2uWkmp2hGrZ69wtQR2Hu6RTgO8Cn42IV1s27wTeGhG/B/w34P6854iIbRExGRGTExMTRWs2S0JjtcyI17lbgjoKd0mj1IP9WxFxX+v2iHg1Ig5mt3cAo5LW9bRSs8S0du41f6BqCelktYyA24E9EfGlNmPOzsYhaXP2vC/3slCz1DRWy/ggJktRJ6tl3gf8MfCEpF3ZY38JnAcQEbcB1wGfllQBDgNbItzG2PCq1YIIKJdKlH0NVUvQsuEeET8BtMyYrwBf6VVRZqlrzK+PlEVZnnO39PgIVbMCGl360atlvM7d0uFwNyugce72kaNO+TvIisyO5nA3K6C5cy+VhOTO3dLicDcrYHHOPevaR0rynLslxeFuVsCRzr3+K1SSvFrGkuJwNyvAnbulzuFuVkC1emTOvfGnO3dLicPdrICFxmqZ7ACmkXLJ4W5JcbibFdC8Wqbxp6dlLCUOd7MCGueRacy5lyUvhbSkONzNCqgufqBa/xVy526pcbibFdA4QrW8OOcuag53S4jD3ayAastSSHfulhqHu1kBlZYPVEe8FNIS43A3K6B1zr0kd+6WFoe7WQFLOveyO3dLi8PdrIBq0yl/oX6OGYe7paSTa6hulPRDSXskPSXpMzljJOnLkqYl7ZZ0SX/KNUtDpeo5d0tbJ9dQrQB/ERE7JZ0KPCbpoYj4edOYq4ALsq/fB76W/Wk2lKpNl9mDxmoZH8Rk6Vi2c4+IFyNiZ3b7NWAPsKFl2LXAN6PuYWCNpPU9r9YsEa1nhSz7lL+WmK7m3CVtAi4GHmnZtAF4vun+DEv/ATAbGq3nc/cHqpaajsNd0inAd4HPRsSrrZtzvmXJO13SVklTkqZmZ2e7q9QsIUs6d8+5W2I6CndJo9SD/VsRcV/OkBlgY9P9c4G9rYMiYltETEbE5MTERJF6zZLQWC1T9sU6LFGdrJYRcDuwJyK+1GbYduDj2aqZS4EDEfFiD+s0S4o7d0tdJ6tl3gf8MfCEpF3ZY38JnAcQEbcBO4CrgWngdeATvS/VLB0+n7ulbtlwj4ifkD+n3jwmgD/rVVFmqVuotp7yt+SzQlpSfISqWQHV1lP+unO3xDjczQrwnLulzuFuVkA15/QDPkLVUuJwNyugtXMvuXO3xDjczQqo1oJySdRXCvvEYZYeh7tZAZUs3Bu8FNJS43A3K6Baqy1OyYA7d0uPw92sgKWde8mduyXF4W5WQLUWR3Xu5RLu3C0pDnezAuqd+5Ffn8Zl9uoHa5sNnsPdrIBqNZbMuQO4ebdUONzNCshbLVN/3AcyWRoc7mYFVGu1xeunwpHO3fPulgqHu1kB7Tt3h7ulweFuVsDS1TLZnLvD3RLhcDcroHW1zIg7d0uMw92sgKWde2nxcbMUdHIN1TskvSTpyTbbL5N0QNKu7OuW3pdplpbWOXd37paaTq6h+g3gK8A3jzHmnyPiwz2pyOwEUKkefW6ZUmO1TNXhbmlYtnOPiB8D+1agFrMTRrvOveojVC0RvZpzf4+kxyU9IOkdPXpOs2RVa3HUOvfy4jp3H8RkaehkWmY5O4G3RsRBSVcD9wMX5A2UtBXYCnDeeef14KXNBqNSC0a8WsYSdtyde0S8GhEHs9s7gFFJ69qM3RYRkxExOTExcbwvbTYwredzXzyIyXPulojjDndJZyu71pikzdlzvny8z2uWsko1/whVL4W0VCw7LSPp28BlwDpJM8BfAaMAEXEbcB3waUkV4DCwJXzeUxtybefc/da3RCwb7hHxsWW2f4X6UkmzN43qkiNUfRCTpcVHqJoVUKkFTY2759wtOQ53swIq1Roj5abOvew5d0uLw92sgIVaMNrUupfki3VYWhzuZgXUTz+wdJ17zR+oWiIc7mYFVNqslvGcu6XC4W5WQKX1Atmec7fEONzNCqjUWj5Q9ekHLDEOd7MuRQQL1WC0tPQDVXfulgqHu1mXGvl9dOfug5gsLQ53sy4tVOvLHY86t4zn3C0xDnezLjXm1ZvXuXvO3VLjcDfrUiXr3JvXuftiHZYah7tZl/I697LcuVtaHO5mXWocqNT8garn3C01DnezLuV9oDrii3VYYhzuZl2q5k3L+ANVS4zD3axLjTM/jvhiHZawZcNd0h2SXpL0ZJvtkvRlSdOSdku6pPdlmqVjobq0c2/M0Lhzt1R00rl/A7jyGNuvAi7IvrYCXzv+sszS1fhAtfkye5IYKWlxmaTZoC0b7hHxY2DfMYZcC3wz6h4G1kha36sCzVKzOC3TfJ297L47d0tFL+bcNwDPN92fyR4zG0qL69xLR//6jJZLzFfcuVsaehHuynkst32RtFXSlKSp2dnZHry02cprLIVs7dzHyiVfZs+S0YtwnwE2Nt0/F9ibNzAitkXEZERMTkxM9OClzVbe4kFMpaPDfbRcYqHiaRlLQy/CfTvw8WzVzKXAgYh4sQfPa5akxnLH5iNUAUZHtNjVmw3ayHIDJH0buAxYJ2kG+CtgFCAibgN2AFcD08DrwCf6VaxZChanZXI693mHuyVi2XCPiI8tsz2AP+tZRWaJO3LisJbOvVRy527J8BGqZl3KO7cMNKZlPOduaXC4m3Up79wy9fvu3C0dDnezLuWd8hcc7pYWh7tZlxayteyjpaXr3D0tY6lwuJt1qX3n7qWQlg6Hu1mXGqtlWj9QHfHpBywhDnezLjXO/Nj6geqY59wtIQ53sy4tLIZ73rSM59wtDQ53sy7NV2pI+Ueo+nzulgqHu1mX5io1xkdKSK0HMZWYd+duiXC4m3VprlJjrLz0V8dz7pYSh7tZl+YqNcZHy0se91JIS4nD3axL82069xF37pYQh7tZl+YqVcZHl/7qjGZHqNZPlGo2WA53sy6169zHsnXvvki2pcDhbtal9nPu9V8nT81YChzuZl2ar9QYz+ncF8Pd11G1BHQU7pKulPSMpGlJN+Vsv0HSrKRd2def9L5UszS0nXMfqT/mS+1ZCjq5hmoZ+CrwQWAGeFTS9oj4ecvQeyLixj7UaJaU+WqNM/I69+yIVYe7paCTzn0zMB0Rz0XEPHA3cG1/yzJL19xCLbdzbzzmM0NaCjoJ9w3A8033Z7LHWv2BpN2S7pW0sSfVmSVovpq/WmZ19iHr4fnqSpdktkQn4a6cx1o/MfpHYFNEvBP4PnBn7hNJWyVNSZqanZ3trlKzRMwt1BgfWbpaZlUj3Bcc7jZ4nYT7DNDciZ8L7G0eEBEvR8RcdvfrwLvznigitkXEZERMTkxMFKnXbODmqzXGRpb+6pw0Vv8I6w2HuyWgk3B/FLhA0vmSxoAtwPbmAZLWN929BtjTuxLN0jK3UGU8J9w9LWMpWXa1TERUJN0IPAiUgTsi4ilJtwJTEbEd+HNJ1wAVYB9wQx9rNhuodp376rH6Y56WsRQsG+4AEbED2NHy2C1Nt28Gbu5taWbpqdWChWp4zt2S5yNUzbrQWMOe27ln4e45d0uBw92sC3MLxwj3Mc+5Wzoc7mZdeKNSD+7VOScOWzXiaRlLh8PdrAuH5ioAnDy+NNxLJTE+UnK4WxIc7mZdeD2bcmmsaW+1eqzMG56WsQQ43M26sNi5jy3t3KE+XePO3VLgcDfrwmLnPt6mcx8tL44xGySHu1kXDs0v07mPlb0U0pLgcDfrQmNa5lid+6E5h7sNnsPdrAuvHq6H+2mr8sN9zUlj7D+8sJIlmeVyuJt1Yf/hecolcUqbzv3Mk8fYd2gud5vZSnK4m3XhwOEFTls1gpR3mQM44+QxXjm0QIQvkm2D5XA368KBwxVOXz3advvak0eZr9Y4mM3Nmw2Kw92sC/sOzXHGyWNtt689eRyAVw553t0Gy+Fu1oWXXp3jrFNXtd2+9uR6V7/v9fmVKsksl8PdrAsvvTbHW04bb7u90bn7Q1UbNIe7WYdee2OBA4cXWH/66rZjzsqCf+/+N1aqLLNcHYW7pCslPSNpWtJNOdvHJd2TbX9E0qZeF2o2aM/NHgLg7RMntx1z1qmrWD1a5hezB1eqLLNcy15mT1IZ+CrwQWAGeFTS9oj4edOwTwGvRMTvSNoCfAH4d/0o2GxQdr9wAIB/cfZpANz/sxf44oPPsHf/Yc5Zs5rPXXEhH7l4AxeefSpPZmPNBqWTzn0zMB0Rz0XEPHA3cG3LmGuBO7Pb9wKXq91CYLMTUESwY/eLbFizmo1rV3P/z17g5vue4IX9hwnghf2Hufm+J7j/Zy/w3refyc5f7+fXL78+6LLtTayTC2RvAJ5vuj8D/H67MRFRkXQAOBP4bS+KbPaTZ3/L3z34dEdjuz2OJOj8G7p57u7r6Oa5+3OwTDL7rrsyutof3Tz34fkqLx54g1s+fBGS+OKDzyw5te/hhSpffPAZ7t56KXc9/Cuu+q8/ZuPakxgbKeFOx5p99JJzuf69m/r6Gp2Ee977svX3opMxSNoKbAU477zzOnjppcZGSpx5jHXGOa/Z1fN3M7q7p+6yji6G96tm9bPmPtbRzfBOh5ZL4r1vP5M/mtwIwN79h3PH7d1/mI1rT+Kere/hrod/ycsH5xcvqm3WsLrNWUV7qZNwnwE2Nt0/F9jbZsyMpBHgdGBf6xNFxDZgG8Dk5GShlnPz+WvZfP7mIt9q1jPnrFnNCzkBf86a+kqai845jf/00XeudFlmizqZc38UuEDS+ZLGgC3A9pYx24Hrs9vXAT8In1zDhtjnrrhwyUWyV4+W+dwVFw6oIrOjLdu5Z3PoNwIPAmXgjoh4StKtwFREbAduB+6SNE29Y9/Sz6LNBu0jF28AyF0tY5YCDarBnpycjKmpqYG8tpnZiUrSYxExudw4H6FqZjaEHO5mZkPI4W5mNoQc7mZmQ8jhbmY2hBzuZmZDyOFuZjaEHO5mZkNoYAcxSZoFflXw29fRhzNO9kCqdUG6tbmu7riu7gxjXW+NiInlBg0s3I+HpKlOjtBaaanWBenW5rq647q682auy9MyZmZDyOFuZjaETtRw3zboAtpItS5ItzbX1R3X1Z03bV0n5Jy7mZkd24nauZuZ2TEkG+6S/lDSU5JqkiZbtt0saVrSM5KuaPP950t6RNKzku7JriLV6xrvkbQr+/qlpF1txv1S0hPZuL6fxF7SX0t6oam2q9uMuzLbh9OSblqBur4o6WlJuyX9g6Q1bcatyP5a7u8vaTz7GU9n76VN/aql6TU3SvqhpD3Z+/8zOWMuk3Sg6ed7S7/ranrtY/5sVPflbJ/tlnTJCtR0YdO+2CXpVUmfbRmzIvtM0h2SXpL0ZNNjayU9lGXRQ5LOaPO912djnpV0fd6YrkREkl/AvwQuBH4ETDY9fhHwODAOnA/8AijnfP93gC3Z7duAT/e53v8M3NJm2y+BdSu47/4a+A/LjCln++5twFi2Ty/qc10fAkay218AvjCo/dXJ3x/498Bt2e0twD0r8LNbD1yS3T4V+D85dV0GfG+l3k/d/GyAq4EHqF97/FLgkRWurwz8P+prwVd8nwH/BrgEeLLpsb8Dbspu35T3vgfWAs9lf56R3T7jeGpJtnOPiD0R8UzOpmuBuyNiLiL+LzANHHXFbEkC3g/cmz10J/CRftWavd4fAd/u12v0wWZgOiKei4h54G7q+7ZvIuKfIqKS3X2Y+sXWB6WTv/+11N87UH8vXZ79rPsmIl6MiJ3Z7deAPcCJdO2+a4FvRt3DwBpJ61fw9S8HfhERRQ+QPC4R8WPqlxpt1vw+apdFVwAPRcS+iHgFeAi48nhqSTbcj2ED8HzT/RmWvvnPBPY3BUnemF7618BvIuLZNtsD+CdJj0na2sc6mt2Y/bf4jjb/DexkP/bTJ6l3eHlWYn918vdfHJO9lw5Qf2+tiGwa6GLgkZzN75H0uKQHJL1jpWpi+Z/NoN9XW2jfZA1qn50VES9C/R9v4C05Y3q+35a9QHY/Sfo+cHbOps9HxP9q9205j7Uu+elkTEc6rPFjHLtrf19E7JX0FuAhSU9n/8IXdqy6gK8Bf0v97/y31KeMPtn6FDnfe9xLpzrZX5I+D1SAb7V5mp7vr7xScx7r2/uoW5JOAb4LfDYiXm3ZvJP6tMPB7POU+4ELVqIulv/ZDHKfjQHXADfnbB7kPutEz/fbQMM9Ij5Q4NtmgI1N988F9raM+S31/w6OZB1X3pie1ChpBPgo8O5jPMfe7M+XJP0D9SmB4wqrTvedpK8D38vZ1Ml+7Hld2QdFHwYuj2yyMec5er6/cnTy92+Mmcl+zqez9L/cPSdplHqwfysi7mvd3hz2EbFD0n+XtC4i+n4OlQ5+Nn15X3XoKmBnRPymdcMg9xnwG0nrI+LFbIrqpZwxM9Q/F2g4l/rnjYWdiNMy24Et2UqG86n/6/vT5gFZaPwQuC576Hqg3f8EjtcHgKcjYiZvo6STJZ3auE39Q8Un88b2Sssc579t83qPAheovqpojPp/Z7f3ua4rgf8IXBMRr7cZs1L7q5O//3bq7x2ov5d+0O4fpF7J5vRvB/ZExJfajDm7MfcvaTP13+OX+1lX9lqd/Gy2Ax/PVs1cChxoTEmsgLb/gx7UPss0v4/aZdGDwIcknZFNo34oe6y4fn96XPSLeijNAHPAb4AHm7Z9nvpKh2eAq5oe3wGck91+G/XQnwb+JzDepzq/Afxpy2PnADua6ng8+3qK+vREv/fdXcATwO7sjbW+ta7s/tXUV2P8YoXqmqY+r7gr+7qtta6V3F95f3/gVur/+ACsyt4709l76W0rsI/+FfX/ju9u2k9XA3/aeJ8BN2b75nHqH0y/t991Hetn01KbgK9m+/QJmla69bm2k6iH9elNj634PqP+j8uLwEKWX5+i/jnN/waezf5cm42dBP6+6Xs/mb3XpoFPHG8tPkLVzGwInYjTMmZmtgyHu5nZEHK4m5kNIYe7mdkQcribmQ0hh7uZ2RByuJuZDSGHu5nZEPr/Q27r7QjQ0bwAAAAASUVORK5CYII=\n", "text/plain": [ "
" ] }, "metadata": { "needs_background": "light" }, "output_type": "display_data" }, { "data": { "image/png": "iVBORw0KGgoAAAANSUhEUgAAAXcAAAD8CAYAAACMwORRAAAABHNCSVQICAgIfAhkiAAAAAlwSFlzAAALEgAACxIB0t1+/AAAADl0RVh0U29mdHdhcmUAbWF0cGxvdGxpYiB2ZXJzaW9uIDMuMC4zLCBodHRwOi8vbWF0cGxvdGxpYi5vcmcvnQurowAAGyhJREFUeJzt3W2QXOV55vH/1T0zkngxAjQOQghLrCmydjY2eErBZrPFGq8RlAvyQrbkDzF+SaniDRW7KvEG4iqSkC9xXOtUOfaalRfW4HLZOJgQmYgl8mKvQ+2CPWAhwIJlIHYxCJsxIIGENJruvvfDOa3p6eme6T7TM/2ouX5VU+qXp7vvOdO65pm7n3OOIgIzMxsspX4XYGZmvedwNzMbQA53M7MB5HA3MxtADnczswHkcDczG0AOdzOzAeRwNzMbQA53M7MBNNSvF163bl1s2rSpXy9vZnZCevjhh38REaOLjetbuG/atInx8fF+vbyZ2QlJ0k87Gee2jJnZAHK4m5kNIIe7mdkAcribmQ0gh7uZ2QByuJuZDSCHu5nZAHK4my1BrRbc+fAkB1+f6XcpZnM43M2W4Nt79/PHf/co/+P//Eu/SzGbw+FutgTPTh0Gshm8WUoc7mZLIGX/OtotNQ53syWYrtTm/GuWCoe72RIcOVYF4NB0pc+VmM3lcDdbgnq4H3a4W2Ic7mZL8PpMPdyrfa7EbC6Hu9kSHDmWzdhnqu65W1oc7mZLcHQmC3WHu6XG4W62BJWaw93StGi4S1ot6QeSHpX0hKS/aDHmw5KmJO3Jv35veco1S0s133npmJdCWmI6OYfqNPDeiDgkaRh4QNK9EfFg07g7IuK63pdolq5KPdyr3o3J0rJouEdEAIfyq8P5l9/JZszO3N2WsdR01HOXVJa0B3gR2B0RD7UY9tuS9kq6U9LGnlZplqhK1W0ZS1NH4R4R1Yh4J3AOsEXSrzQN+TawKSJ+FfgOcFur55G0XdK4pPGpqaml1G2WBM/cLVVdrZaJiAPA94CtTbe/FBHT+dUvA+9q8/gdETEWEWOjo6MFyjVLi1fLWKo6WS0zKmltfnkN8D7gyaYx6xuuXgXs62WRZqmqz9x94DBLTSerZdYDt0kqk/0y+GZE3CPpJmA8InYCfyjpKqACvAx8eLkKNktJxW0ZS1Qnq2X2Ahe2uP3Ghss3ADf0tjSz9Hmdu6XKe6iaLUF95l4Ln43J0uJwN1uCakOgV8PhbulwuJstQaWh1171zN0S4nA3W4LGQK843C0hDnezJajUgpGh7L9R1ceXsYQ43M2WoFoLVuXhXt+hySwFDnezgiKCSi1YNVQG3HO3tDjczQqqZ/nszN3hbulwuJsVVG/DrBrOw909d0uIw92soHobpt6Wcc/dUuJwNyuo3oZZnc/c3XO3lDjczQqqL310z91S5HA3K6jS1JbxzN1S4nA3K2i25+6Zu6XH4W5W0OxqmfrM3R+oWjoc7mYFzZu5eymkJcThblZQpSnc3XO3lHRyDtXVkn4g6VFJT0j6ixZjVkm6Q9KEpIckbVqOYs1S0rzOfcbhbgnpZOY+Dbw3It4BvBPYKunipjEfA16JiLcCfwN8prdlmqWn3oZZdXydu3vulo5Fwz0yh/Krw/lX8xTlauC2/PKdwGWS1LMqzRJUn7mvru+h6p67JaSjnruksqQ9wIvA7oh4qGnIBuA5gIioAAeBM1s8z3ZJ45LGp6amlla5WZ81H1vGPXdLSUfhHhHViHgncA6wRdKvNA1pNUuf906PiB0RMRYRY6Ojo91Xa5YQr3O3lHW1WiYiDgDfA7Y23TUJbASQNAScBrzcg/rMkuU9VC1lnayWGZW0Nr+8Bngf8GTTsJ3Atfnla4D7I3wqeBtsnrlbyoY6GLMeuE1SmeyXwTcj4h5JNwHjEbETuAX4qqQJshn7tmWr2CwRx2fuXi1jCVo03CNiL3Bhi9tvbLh8FPid3pZmlrZ6mM8ez90zd0uH91A1K6jSfMhfL4W0hDjczQpyz91S5nA3K2j2TEw+KqSlx+FuVlC16QNVz9wtJQ53s4LmrXN3z90S4nA3K6jehhlxz90S5HA3K6ge5sMlUS7Je6haUhzuZgXVw7ych7tn7pYSh7tZQfV17UOlEkMlebWMJcXhblbQ8Zl7OZu5z/gDVUuIw92soHobZqikfObucLd0ONzNCqq3YcolMVQuHT95h1kKHO5mBdVn7mWJsjxzt7Q43M0KqtaCkqB0fClkvysym+VwNyuoUguGStl/oaGyV8tYWhzuZgVVa0G5lJ0+2OvcLTUOd7OCKtVgqB7u7rlbYjo5h+pGSd+VtE/SE5I+0WLMpZIOStqTf93Y6rnMBkm1VqNcnp25O9wtJZ2cQ7UC/FFEPCLpVOBhSbsj4sdN4/45Ij7Q+xLN0pT13LNwz3ruDndLx6Iz94h4ISIeyS+/BuwDNix3YWapm9tzL7nnbknpqucuaRPZybIfanH3uyU9KuleSW9v8/jtksYljU9NTXVdrFlKGlfLlIVn7paUjsNd0inAt4BPRsSrTXc/ArwlIt4B/C1wd6vniIgdETEWEWOjo6NFazZLQuPMfajkPVQtLR2Fu6RhsmD/WkTc1Xx/RLwaEYfyy7uAYUnrelqpWWIae+7lknC2W0o6WS0j4BZgX0R8rs2Ys/JxSNqSP+9LvSzULDXVWm125l6WZ+6WlE5Wy1wC/C7wmKQ9+W1/CpwLEBE3A9cAH5dUAY4A2yLCDUgbaJXq3J2Y3HO3lCwa7hHxAKBFxnwB+EKvijI7EVRrwVB5dicmr5axlHgPVbOCKrWgXF8t45m7JcbhblZQ1TsxWcIc7mYFVRo+UC2XSg53S4rD3aygxpl7WbjnbklxuJsVVGk6/IBn7pYSh7tZQXN67v5A1RLjcDcrKFvnnq+WKXsppKXF4W5W0Nyeu0+zZ2lxuJsVVGk6WYdn7pYSh7tZQc0995rD3RLicDcraM5qGffcLTEOd7OCvFrGUuZwNytozrFlfOAwS4zD3aygOatl8pB3391S4XA3K6hSnXuyDvAhCCwdDnezgqpNp9mr32aWAoe7WUGVWsyuc1d95u4dmSwNnZxDdaOk70raJ+kJSZ9oMUaSPi9pQtJeSRctT7lm6Wg1c3e2Wyo6OYdqBfijiHhE0qnAw5J2R8SPG8ZcAZyff/0a8KX8X7OBFBFzVsvM9tyd7paGRWfuEfFCRDySX34N2AdsaBp2NXB7ZB4E1kpa3/NqzRJRb627526p6qrnLmkTcCHwUNNdG4DnGq5PMv8XAJK2SxqXND41NdVdpWYJqc/Qy6XmnrvD3dLQcbhLOgX4FvDJiHi1+e4WD5n3Lo+IHRExFhFjo6Oj3VVqlpD6DN0zd0tVR+EuaZgs2L8WEXe1GDIJbGy4fg6wf+nlmaWpPkNvXufucLdUdLJaRsAtwL6I+FybYTuBD+WrZi4GDkbECz2s0ywp1WrzzD37r+S2jKWik9UylwC/CzwmaU9+258C5wJExM3ALuBKYAJ4HfhI70s1S8fxmXs5Xy3jtowlZtFwj4gHaN1TbxwTwB/0qiiz1DX33EveickS4z1UzQpoXi0z5J2YLDEOd7MC5q2W8U5MlhiHu1kB81bLuOduiXG4mxUwO3OfPVkHeLWMpcPhblZApTp35j574DCHu6XB4W5WQHPP3SfrsNQ43M0KOL5apjx3Jyb33C0VDnezAuatlnHP3RLjcDcroHm1zOyBw7wU0tLgcDcroHm1zOyBw/pWktkcDnezAtrN3L0Tk6XC4W5WQL39MuSdmCxRDnezAprXuZf8gaolxuFuVsDxnnt57jp378RkqXC4mxVQaXOaPc/cLRUOd7MCqsc/UK2frMM7MVlaHO5mBcybubvnbonp5Byqt0p6UdLjbe6/VNJBSXvyrxt7X6ZZWqpNJ+sou+duienkHKpfAb4A3L7AmH+OiA/0pCKzE8BM0wmyh9xzt8QsOnOPiO8DL69ALWYnjKoPP2CJ61XP/d2SHpV0r6S3txskabukcUnjU1NTPXpps5VX8ck6LHG9CPdHgLdExDuAvwXubjcwInZExFhEjI2Ojvbgpc36o9p0yN9SSUheLWPpWHK4R8SrEXEov7wLGJa0bsmVmSWsebVM/bLD3VKx5HCXdJaU/U0qaUv+nC8t9XnNUlZtOvxA/bLD3VKx6GoZSV8HLgXWSZoE/gwYBoiIm4FrgI9LqgBHgG0R4Xe4DbTjR4VU48y95J67JWPRcI+IDy5y/xfIlkqavWFUa0FJWa+9ruSeuyXEe6iaFVCpxfGVMnVD5ZLD3ZLhcDcroFqrzem3Q9Zzd1vGUuFwNysgm7nPDfdstYx3YrI0ONzNCqhU4/gx3OtK8szd0uFwNyugUqsxVG7uucsHDrNkONzNCpipBsPuuVvCHO5mBVSqLWbu3onJEuJwNyugUnPP3dLmcDcroFJtsVqm7Jm7pcPhblZApVabtxNTueSdmCwdDnezAmaqwXC51Tp3h7ulweFuVkCrpZDZahnvxGRpcLibFTDToudelmfulg6Hu1kBlWqN4RY7MTncLRUOd7MCqi2WQvpkHZYSh7tZAVlbZv5OTF7nbqlYNNwl3SrpRUmPt7lfkj4vaULSXkkX9b5Ms7RUarV5q2VKEpWqw93S0MnM/SvA1gXuvwI4P//aDnxp6WWZpa1SjXnHcx8eKnm1jCVj0XCPiO8DLy8w5Grg9sg8CKyVtL5XBZqlaKY2/wPVkXKJGc/cLRG96LlvAJ5ruD6Z32Y2sFodfmC4LGaqnrlbGnoR7mpxW8vpi6TtksYljU9NTfXgpc36Iztw2PxzqDrcLRW9CPdJYGPD9XOA/a0GRsSOiBiLiLHR0dEevLRZf2Tr3OfOa0bKJY5VHO6Whl6E+07gQ/mqmYuBgxHxQg+e1yxZlRZLIYfLXgpp6RhabICkrwOXAuskTQJ/BgwDRMTNwC7gSmACeB34yHIVa5aKmRZLIYfdlrGELBruEfHBRe4P4A96VpHZCaDVCbKH8tUyEYHU6qMos5XjPVTNuhQRVGpBudS8FDILdC+HtBQ43M26VO+rN58gu77u3TsyWQoc7mZdqh8crHkpZD3cZyqeuVv/OdzNulT/0HT+B6rZ9WP+UNUS4HA361L94GDz91DNZ+4Od0uAw92sSzN5T71dW8ZHhrQUONzNulQP73ltmaHsv5PbMpYCh7tZl2bbMk0z91J9KaTD3frP4W7Wpdm2jHvuli6Hu1mXZtsyTTP3oXq4u+du/edwN+vSdKUKwKqh+QcOA8/cLQ0Od7MuTeeH9V01VJ5zu9sylhKHu1mX6sdsXzXcZg9Vh7slwOFu1qXF2zLuuVv/OdzNujQ9k83MR4aajwrpmbulw+Fu1qV2Pfchh7slxOFu1qVF2zI+KqQloKNwl7RV0lOSJiRd3+L+D0uakrQn//q93pdqlobjH6i2a8v4eO6WgE7OoVoGvgj8B2AS+KGknRHx46ahd0TEdctQo1lSjrdlhlsvhayHv1k/dTJz3wJMRMSzEXEM+AZw9fKWZZaueriPNO2hujoP+6MzDnfrv07CfQPwXMP1yfy2Zr8taa+kOyVt7El1ZgmanqkizT8qZL1Nc2Sm2o+yzOboJNxbnca9+ROjbwObIuJXge8At7V8Imm7pHFJ41NTU91VapaI6WqNVUMlpLn/NUolsXq4xFGHuyWgk3CfBBpn4ucA+xsHRMRLETGdX/0y8K5WTxQROyJiLCLGRkdHi9Rr1nfTM7V5yyDr1gyXHe6WhE7C/YfA+ZI2SxoBtgE7GwdIWt9w9SpgX+9KNEvLdKU2bwemujXDZY4cc7hb/y26WiYiKpKuA+4DysCtEfGEpJuA8YjYCfyhpKuACvAy8OFlrNmsr6Yr1XnLIOtWj5Tdc7ckLBruABGxC9jVdNuNDZdvAG7obWlmaZqu1NqGu9sylgrvoWrWpWOVhXvunrlbChzuZl2artTmHe63bs2Ie+6WBoe7WZemZ6rzdmCqWz1c5oh3YrIEONzNunS0Uju+N2oz99wtFQ53sy4dnq5wyqrWaxFWD5fclrEkONzNunR4usJJIwvM3CsOd+s/h7tZlw5PVzi53czdH6haIhzuZl2ICA4fq3LyqvYz9+lKjVrNJ+yw/nK4m3VhulKjWou2M/eTR7LbDx+rrGRZZvM43M26cGg6C+12H6iuPWkYgAOvz6xYTWatONzNulAP7dPWDLe8/4yTRwB4+fCxFavJrBWHu1kXDh7JQnvtSSMt7z+9Hu6vO9ytvxzuZl2oz9zXtpu556H/imfu1mcOd7MuvLJIW+Z0t2UsEQ53sy68+NpRAN78plUt73/T6iHKJTncre8c7mZd+NnBo5y6eoiTRlqvlpHE6SeN8Ip77tZnDnezLuw/cJT1p61ecMwvvWkV+w8cXaGKzFrrKNwlbZX0lKQJSde3uH+VpDvy+x+StKnXhZql4JmpQ5y37pQFx5w3egrPTB1aoYrMWls03CWVgS8CVwBvAz4o6W1Nwz4GvBIRbwX+BvhMrws167dXj87w05cOc8FZpwJw94+e55K/up/N1/8jl/zV/dz9o+cB+OWzTmXylSPuu1tfdTJz3wJMRMSzEXEM+AZwddOYq4Hb8st3ApdJUu/KNOu///n4z6gFXPLWddz9o+e54a7HeP7AEQJ4/sARbrjrMe7+0fO851+dCcA9e/f3t2B7Q+vkBNkbgOcark8Cv9ZuTERUJB0EzgR+0YsiGz3w9C/46/uenHd7tDlOU9D6jnbjF36uduO7O0hUr2rttp4Fq1zm1+jZ99zl8bgW+tl0U2sQvPjaNP9mw2mMveV0fv2OPfPOlXpkpspn73uKB/7k37Nl8xnc+A9PcPv//SlrhsuUPNWxBr910Tlc+55Ny/oanYR7q7dl89u/kzFI2g5sBzj33HM7eOn5RoZKnHly670D2/2x0O7/1cJ/W7R5rjaP6fY11O3zd/k8Xd6cv0Zvtl/78d09f/vvoTc/m4UfM/+OdaeO8NFLNlMqif0HjrR83P4DR5DElz80xn/738/w7NRhpivVhX+x2hvOmjbnA+ilTsJ9EtjYcP0coPnvzfqYSUlDwGnAy81PFBE7gB0AY2Njhd7vWzafwZbNW4o81Kxnzl67hudbBPzZa9cA2U5O/3nrL690WWbHddJz/yFwvqTNkkaAbcDOpjE7gWvzy9cA90e3vQqzE8inLr+ANU3nUV0zXOZTl1/Qp4rM5lp05p730K8D7gPKwK0R8YSkm4DxiNgJ3AJ8VdIE2Yx923IWbdZvv3HhBgA+e99T7D9whLPXruFTl19w/HazflO/JthjY2MxPj7el9c2MztRSXo4IsYWG+c9VM3MBpDD3cxsADnczcwGkMPdzGwAOdzNzAaQw93MbAA53M3MBpDD3cxsAPVtJyZJU8BPCz58HctwxMkeSLUuSLc219Ud19WdQazrLRExutigvoX7Ukga72QPrZWWal2Qbm2uqzuuqztv5LrcljEzG0AOdzOzAXSihvuOfhfQRqp1Qbq1ua7uuK7uvGHrOiF77mZmtrATdeZuZmYLSDbcJf2OpCck1SSNNd13g6QJSU9JurzN4zdLekjS05LuyM8i1esa75C0J//6iaQ9bcb9RNJj+bhlP4i9pD+X9HxDbVe2Gbc134YTkq5fgbo+K+lJSXsl/b2ktW3Grcj2Wuz7l7Qq/xlP5O+lTctVS8NrbpT0XUn78vf/J1qMuVTSwYaf743LXVfDay/4s1Hm8/k22yvpohWo6YKGbbFH0quSPtk0ZkW2maRbJb0o6fGG286QtDvPot2STm/z2GvzMU9LurbVmK5ERJJfwL8GLgC+B4w13P424FFgFbAZeAYot3j8N4Ft+eWbgY8vc73/BbixzX0/Adat4Lb7c+CPFxlTzrfdecBIvk3ftsx1vR8Yyi9/BvhMv7ZXJ98/8J+Am/PL24A7VuBntx64KL98KvD/WtR1KXDPSr2fuvnZAFcC95Kdl/xi4KEVrq8M/IxsLfiKbzPg3wEXAY833PbXwPX55etbve+BM4Bn839Pzy+fvpRakp25R8S+iHiqxV1XA9+IiOmI+BdgAphzxmxJAt4L3JnfdBvwG8tVa/56/xH4+nK9xjLYAkxExLMRcQz4Btm2XTYR8U8RUcmvPkh2svV+6eT7v5rsvQPZe+my/Ge9bCLihYh4JL/8GrAPOJHO3Xc1cHtkHgTWSlq/gq9/GfBMRBTdQXJJIuL7ZKcabdT4PmqXRZcDuyPi5Yh4BdgNbF1KLcmG+wI2AM81XJ9k/pv/TOBAQ5C0GtNLvw78PCKebnN/AP8k6WFJ25exjkbX5X8W39rmz8BOtuNy+ijZDK+VldhenXz/x8fk76WDZO+tFZG3gS4EHmpx97slPSrpXklvX6maWPxn0+/31TbaT7L6tc1+KSJegOyXN/DmFmN6vt0WPUH2cpL0HeCsFnd9OiL+od3DWtzWvOSnkzEd6bDGD7LwrP2SiNgv6c3AbklP5r/hC1uoLuBLwF+Sfc9/SdYy+mjzU7R47JKXTnWyvSR9GqgAX2vzND3fXq1KbXHbsr2PuiXpFOBbwCcj4tWmux8hazscyj9PuRs4fyXqYvGfTT+32QhwFXBDi7v7uc060fPt1tdwj4j3FXjYJLCx4fo5wP6mMb8g+3NwKJ9xtRrTkxolDQG/BbxrgefYn//7oqS/J2sJLCmsOt12kr4M3NPirk62Y8/ryj8o+gBwWeTNxhbP0fPt1UIn3399zGT+cz6N+X9y95ykYbJg/1pE3NV8f2PYR8QuSf9V0rqIWPZjqHTws1mW91WHrgAeiYifN9/Rz20G/FzS+oh4IW9RvdhizCTZ5wJ155B93ljYidiW2Qlsy1cybCb77fuDxgF5aHwXuCa/6Vqg3V8CS/U+4MmImGx1p6STJZ1av0z2oeLjrcb2SlOP8zfbvN4PgfOVrSoaIftzducy17UV+BPgqoh4vc2YldpenXz/O8neO5C9l+5v9wupV/Ke/i3Avoj4XJsxZ9V7/5K2kP0/fmk568pfq5OfzU7gQ/mqmYuBg/WWxApo+xd0v7ZZrvF91C6L7gPeL+n0vI36/vy24pb70+OiX2ShNAlMAz8H7mu479NkKx2eAq5ouH0XcHZ++Tyy0J8A/g5YtUx1fgX4/abbzgZ2NdTxaP71BFl7Yrm33VeBx4C9+RtrfXNd+fUryVZjPLNCdU2Q9RX35F83N9e1ktur1fcP3ET2ywdgdf7emcjfS+etwDb6t2R/ju9t2E5XAr9ff58B1+Xb5lGyD6bfs9x1LfSzaapNwBfzbfoYDSvdlrm2k8jC+rSG21Z8m5H9cnkBmMnz62Nkn9P8L+Dp/N8z8rFjwH9veOxH8/faBPCRpdbiPVTNzAbQidiWMTOzRTjczcwGkMPdzGwAOdzNzAaQw93MbAA53M3MBpDD3cxsADnczcwG0P8HZBYC53QfEOcAAAAASUVORK5CYII=\n", "text/plain": [ "
" ] }, "metadata": { "needs_background": "light" }, "output_type": "display_data" }, { "data": { "image/png": "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\n", "text/plain": [ "
" ] }, "metadata": { "needs_background": "light" }, "output_type": "display_data" }, { "data": { "image/png": "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\n", "text/plain": [ "
" ] }, "metadata": { "needs_background": "light" }, "output_type": "display_data" }, { "data": { "image/png": "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\n", "text/plain": [ "
" ] }, "metadata": { "needs_background": "light" }, "output_type": "display_data" }, { "data": { "image/png": "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\n", "text/plain": [ "
" ] }, "metadata": { "needs_background": "light" }, "output_type": "display_data" }, { "data": { "image/png": "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\n", "text/plain": [ "
" ] }, "metadata": { "needs_background": "light" }, "output_type": "display_data" }, { "data": { "image/png": "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\n", "text/plain": [ "
" ] }, "metadata": { "needs_background": "light" }, "output_type": "display_data" }, { "data": { "image/png": "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\n", "text/plain": [ "
" ] }, "metadata": { "needs_background": "light" }, "output_type": "display_data" }, { "data": { "image/png": "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\n", "text/plain": [ "
" ] }, "metadata": { "needs_background": "light" }, "output_type": "display_data" }, { "data": { "image/png": "iVBORw0KGgoAAAANSUhEUgAAAXcAAAD8CAYAAACMwORRAAAABHNCSVQICAgIfAhkiAAAAAlwSFlzAAALEgAACxIB0t1+/AAAADl0RVh0U29mdHdhcmUAbWF0cGxvdGxpYiB2ZXJzaW9uIDMuMC4zLCBodHRwOi8vbWF0cGxvdGxpYi5vcmcvnQurowAAHClJREFUeJzt3X+Q5HV95/Hnq3tmfyDIrx1l2R8ulhR3mkoEpzao94MSFeQs1kvwslYlgpraixcqWpXzDmIVScgfV8SKuTJ6UmugBGMJHhJuQ0ERjBBjVVgZcFnAhWM0IsNy7sCyCyu7O9Pd7/vj++3Znp5v7/R8u2f6s8PrUTW13f39TM97vtP7mvd8+vP9fhURmJnZ8lIZdAFmZtZ/Dnczs2XI4W5mtgw53M3MliGHu5nZMuRwNzNbhhzuZmbLkMPdzGwZcribmS1DQ4P6wmvWrIlNmzYN6submZ2QHnnkkRcjYmS+cQML902bNjE2NjaoL29mdkKS9Gw34zwtY2a2DHUd7pKqkn4k6e6CbSsl3S5pXNJOSZv6WaSZmS3MQjr3zwB7Omz7FPByRLwN+Evghl4LMzOz8roKd0nrgf8A/HWHIVuAW/LbdwAXS1Lv5ZmZWRnddu7/E/hvQKPD9nXAcwARUQMOAmf2XJ2ZmZUyb7hL+jCwLyIeOd6wgsfmXAVE0jZJY5LGJicnF1CmmZktRDed+3uByyX9DLgNeJ+kv2kbMwFsAJA0BJwK7G9/oojYHhGjETE6MjLvMk0zMytp3nCPiGsjYn1EbAK2At+LiN9uG7YDuDK/fUU+xtfvs9eFw1N1/uahZ3ltqjboUsxmlD6ISdL1wFhE7ABuAr4haZysY9/ap/rMkvc/7t3Drf/8LG9+4yo+8PY3D7ocM2CB4R4RDwIP5reva3n8CPDRfhZmdqLYe+AwgDt3S4qPUDXrWbae4LWp+oDrMDvG4W7WJw53S4nD3axPjkw73C0dDnezHtUb2bF9nnO3lDjczXp0ZLoZ7u7cLR0Od7MeHc6nYzwtYylxuJv1qBnqUzUft2fpcLib9Wi6nk3L1BqdzqtntvQc7mY9mq5H/q/D3dLhcDfrUS0PdU/LWEoc7mY9mm5koe5pGUuJw92sR83pGE/LWEoc7mY9qs3MuXtaxtLhcDfr0ZQ7d0uQw92sRzWHuyXI4W7Wg3ojyN9PnZmeMUtBNxfIXiXph5Iek/SkpD8tGHOVpElJu/KP312ccs3S0tqtT7lzt4R0cyWmo8D7IuKQpGHgB5LujYiH2sbdHhFX979Es3TVGse6dU/LWErmDff8QteH8rvD+Yf//jQDpmvHAt3TMpaSrubcJVUl7QL2AfdHxM6CYb8pabekOyRt6GuVZomabjlwyZ27paSrcI+IekS8E1gPbJb0K21D/g7YFBG/CnwXuKXoeSRtkzQmaWxycrKXus2S0FzbvmKoQr3hzt3SsaDVMhFxAHgQuLTt8Zci4mh+92vAuzp8/vaIGI2I0ZGRkRLlmqWlnof7qqHKrPl3s0HrZrXMiKTT8turgfcDT7WNWdty93JgTz+LNEtVPZqde9WduyWlm9Uya4FbJFXJfhl8OyLulnQ9MBYRO4A/kHQ5UAP2A1ctVsFmKWleP3XlUIVXjzjcLR3drJbZDZxf8Ph1LbevBa7tb2lm6Wu+h7rSc+6WGB+hataD5ml+V+Rz7hEOeEuDw92sB82VkCuGsv9Kbt4tFQ53sx7MdO7Vyqz7ZoPmcDfrQSOfhlk5nHfuznZLhMPdrAfNUw64c7fUONzNetBc575yqJrd96S7JcLhbtaDZpg331D1UaqWCoe7WQ9qbeHecLhbIhzuZj1ohvlKd+6WGIe7WQ/aO3fPuVsqHO5mPWh4zt0S5XA360EzzFdWm527l0JaGhzuZj04dhBTcynkIKsxO8bhbtYDH8RkqXK4m/Wg3nb6Ab+haqlwuJv1YOYgpqrfULW0dHOZvVWSfijpMUlPSvrTgjErJd0uaVzSTkmbFqNYs9T4ICZLVTed+1HgfRHxa8A7gUslXdg25lPAyxHxNuAvgRv6W6ZZmo4dxJS9oerO3VIxb7hH5lB+dzj/aH8FbwFuyW/fAVwsSX2r0ixRPojJUtXVnLukqqRdwD7g/ojY2TZkHfAcQETUgIPAmf0s1CxFPojJUtVVuEdEPSLeCawHNkv6lbYhRV36nFe5pG2SxiSNTU5OLrxas8TU2t5Q9Zy7pWJBq2Ui4gDwIHBp26YJYAOApCHgVGB/wedvj4jRiBgdGRkpVbBZStqvxOTO3VLRzWqZEUmn5bdXA+8HnmobtgO4Mr99BfC98GXg7XWg/SAmn37AUjHUxZi1wC2SqmS/DL4dEXdLuh4Yi4gdwE3ANySNk3XsWxetYrOENA9i8py7pWbecI+I3cD5BY9f13L7CPDR/pZmlr56o0G1IqoV5fcd7pYGH6Fq1oN6A6oVMeRwt8Q43M16UG80GGrp3D0tY6lwuJv1oNYIqhJDFR/EZGlxuJv1oNEIqlWRZ7s7d0uGw92sB+2duw9islQ43M160IiYtVrGnbulwuFu1oNaPdpWy/ggJkuDw92sB3V37pYoh7tZD+qN2eFerzvcLQ0Od7MezIR7fvmCuk+pZIlwuJv1oJ6vlqlUREXHTiRmNmgOd7Me1PLOHbLTELhzt1Q43M160GgEQ9Vj4e517pYKh7tZD5oHMQFUJa+WsWQ43M160DyICfJpGYe7JcLhbtaD5kFM4HC3tHRzmb0Nkh6QtEfSk5I+UzDmIkkHJe3KP64rei6z5aY+q3Ov+A1VS0Y3l9mrAX8YEY9KOgV4RNL9EfHjtnH/FBEf7n+JZumqN4Lh/OLY1YoPYrJ0zNu5R8QLEfFofvtVYA+wbrELMzsRZAcxZf+Nhty5W0IWNOcuaRPZ9VR3Fmx+t6THJN0r6R0dPn+bpDFJY5OTkwsu1iw12UFM2e1KxRfrsHR0He6STga+A3w2Il5p2/wo8JaI+DXgr4C7ip4jIrZHxGhEjI6MjJSt2SwZtfbO3eFuiegq3CUNkwX7NyPizvbtEfFKRBzKb98DDEta09dKzRLUaMTM6X4rcudu6ehmtYyAm4A9EfHFDmPOyschaXP+vC/1s1CzFNUajZnVMu7cLSXdrJZ5L/A7wOOSduWP/RGwESAibgSuAD4tqQYcBrZG+J0lW/4awUy4Vyo+QtXSMW+4R8QPAM0z5svAl/tVlNmJYnbnLhruaSwRPkLVrAeNhjt3S5PD3awHtUZj5sRhQz4rpCXE4W7Wg3oDqtXWs0L6AtmWBoe7WQ/qLZ17dj73ARdklnO4m/Wg3nYlJnfulgqHu1kP6i0HMWWX2RtwQWY5h7tZD+ZcQ9WduyXC4W7Wg7lXYhpwQWY5h7tZD2Z17nLnbulwuJuV1GgE0XL6gWrVl9mzdDjczUpqXphjZimkHO6WDoe7WUnNIG8exDRUka/EZMlwuJuVNBPuOnZuGV9D1VLhcDcraWZapuLO3dLjcDcrqdmlt54V0nPulopursS0QdIDkvZIelLSZwrGSNKXJI1L2i3pgsUp1ywdzdP7DrV27g53S0Q3V2KqAX8YEY9KOgV4RNL9EfHjljEfAs7NP34d+Gr+r9my1ZiZlsl6pIp8PndLx7yde0S8EBGP5rdfBfYA69qGbQFujcxDwGmS1va9WrOENIO8mv8v8vncLSULmnOXtAk4H9jZtmkd8FzL/Qnm/gIwW1Yajdmde9VXYrKEdB3ukk4GvgN8NiJead9c8ClzXuWStkkakzQ2OTm5sErNEtPeuVd9DVVLSFfhLmmYLNi/GRF3FgyZADa03F8P7G0fFBHbI2I0IkZHRkbK1GuWjLo7d0tYN6tlBNwE7ImIL3YYtgP4eL5q5kLgYES80Mc6zZLTfhBTtSIi8Ly7JaGb1TLvBX4HeFzSrvyxPwI2AkTEjcA9wGXAOPAa8In+l2qWlmOd+7Fzy0B2cFOlcKbSbOnMG+4R8QOK59RbxwTw+/0qyuxEMCfc83PM1BvBcHVgZZkBPkLVrLTm9VKH2jt3T8tYAhzuZiU12s4t0/zXb6paChzuZiXV6sXh7jdULQUOd7OSis4KCe7cLQ0Od7OS2t9QrTQ7dx/IZAlwuJuV1B7u7twtJQ53s5LmXIlJnnO3dDjczUqa07lX3blbOhzuZiXNmXP3OndLiMPdrKS5q2Wy/04Od0uBw92spHrbZfaap/51uFsKHO5mJU3Xm+HePOWvO3dLh8PdrKR6fm6Z5gnDmh183evcLQEOd7OSmqtihtsOYmqGvtkgOdzNSup0EFPd2W4JcLiblVRrm3NvLoWsuXO3BHRzmb2bJe2T9ESH7RdJOihpV/5xXf/LNEvPTOdenX0Qk7PdUtDNZfa+DnwZuPU4Y/4pIj7cl4rMThC1tqWQ7twtJfN27hHxfWD/EtRidkKp5ZPr7XPuPiukpaBfc+7vlvSYpHslvaNPz2mWtPbOfeZKTHWHuw1eN9My83kUeEtEHJJ0GXAXcG7RQEnbgG0AGzdu7MOXNhuceiOoVoTUdiUmd+6WgJ4794h4JSIO5bfvAYYlrekwdntEjEbE6MjISK9f2myganm4N/kaqpaSnsNd0lnKWxdJm/PnfKnX5zVLXb3RmJmSgWPh7tMPWArmnZaR9C3gImCNpAngj4FhgIi4EbgC+LSkGnAY2Brhv0tt+ZvTufuUv5aQecM9Ij42z/Yvky2VNHtdqTfCnbsly0eompWUde7H/gs53C0lDnezkmr12XPuPiukpcThblZS+5x7xZ27JcThblZSvREMV+d27j6IyVLgcDcrqdM6d3fulgKHu1lJ9XrMnO4XYDi/iOq0TxxmCXC4m5XU3rl7WsZS4nA3K6neaMycwx1aTxzmzt0Gz+FuVlJ75y6J4aqY9py7JcDhblZSrT77CFXILrnnzt1S4HA3Kyk7/cDs/0JDVTHtOXdLgMPdrKRa25w7ZCtmfJk9S4HD3ayketucO2QrZrxaxlLgcDcrqdaYO+c+XK14WsaS4HA3K6moc69WRN3TMpYAh7tZSdP1RvEbql4KaQmYN9wl3Sxpn6QnOmyXpC9JGpe0W9IF/S/TLD1Fnfuwl0JaIrrp3L8OXHqc7R8Czs0/tgFf7b0ss/TVGjFntcxQ1W+oWhrmDfeI+D6w/zhDtgC3RuYh4DRJa/tVoFmq2i+zBzBUrXhaxpLQjzn3dcBzLfcn8sfMlrX2y+wBDFfkaRlLQj/CXQWPFbYukrZJGpM0Njk52YcvbTY4xZ27p2UsDf0I9wlgQ8v99cDeooERsT0iRiNidGRkpA9f2mxwavXG3DdUqxWfz92S0I9w3wF8PF81cyFwMCJe6MPzmiWt6CAmH6FqqRiab4CkbwEXAWskTQB/DAwDRMSNwD3AZcA48BrwicUq1iwl0/UGK4ba17lXmPacuyVg3nCPiI/Nsz2A3+9bRWYngEYjmK7HzKX1moYq8jVULQk+QtWshOa8elHnXnO4WwIc7mYlNE8OtqI6dymkp2UsBQ53sxKmalmAD/sIVUuUw92shGZ3vmKoOuvxIV+swxLhcDcroVPnnk3LuHO3wXO4m5VwrHMveEPVc+6WAIe7WQlTzXBvXwrp87lbIhzuZiVM17IAb1/n7vO5Wyoc7mYlNDv34TnTMqIR2UFOZoPkcDcrofmG6pxpmfxcMz55mA2aw92shGNvqM5eLdN8g9UrZmzQHO5mJTTDvX3OfWW+7r3Z2ZsNisPdrISZaZmh9nDP7h+Zri95TWatHO5mJUx16tyHs/tH3bnbgDnczUrodOKwVfm0zNGaO3cbLIe7WQmdjlCd6dyn3bnbYHUV7pIulfS0pHFJ1xRsv0rSpKRd+cfv9r9Us3QcO7dM8RuqnnO3QevmMntV4CvAB8guhv2wpB0R8eO2obdHxNWLUKNZco6tlpm9FHKV59wtEd107puB8Yj4aURMAbcBWxa3LLO0TXWalpmZc3e422B1E+7rgOda7k/kj7X7TUm7Jd0haUPRE0naJmlM0tjk5GSJcs3SMDMtUyleCuk3VG3Qugl3FTzWfvjd3wGbIuJXge8CtxQ9UURsj4jRiBgdGRlZWKVmCZmuNxiqiEpl9n+PY3Pu7txtsLoJ9wmgtRNfD+xtHRARL0XE0fzu14B39ac8szRN12POm6nQOufuzt0Gq5twfxg4V9I5klYAW4EdrQMkrW25ezmwp38lmqXnyHR9Jshbzcy5u3O3AZt3tUxE1CRdDdwHVIGbI+JJSdcDYxGxA/gDSZcDNWA/cNUi1mw2cIen6qwers553EeoWirmDXeAiLgHuKftsetabl8LXNvf0szSdXi6zqoVBeHuc8tYInyEqlkJR6aLO3dJrBiquHO3gXO4m5VwuEO4Q9a9+w1VGzSHu1kJh6fqrC6YlgFYNVx1524D53A3K+HwdINVx+ncPedug+ZwNyvh8FSt47TMSSuqvHbU4W6D5XA3K+HQ0TonrypebHbq6mEOHp5e4orMZnO4m5Xw6pFpTllZHO5vXOVwt8FzuJst0FStwdFag1PcuVvCHO5mC3ToaA2Akzt17quHecXhbgPmcDdboENH8nBfNVy4/dTVw7x6tEa90X7yVLOl43A3W6ADh6eALMSLvDF//NUj7t5tcBzuZgv00qEs3NecvKJwezP0Pe9ug+RwN1ugFw9lly5Yc/LKwu0Od0uBw91sgV7MO/czO3Tup5+UhftLv5xasprM2jnczRbouZdf44w3rOCkFcWrZdaffhIAEy8fXsqyzGbpKtwlXSrpaUnjkq4p2L5S0u359p2SNvW7ULNU/Pyl19hwxkkdt7/plJWsGKrw85d+uYRVmc0278U6JFWBrwAfILue6sOSdkTEj1uGfQp4OSLeJmkrcAPwW4tRsNkgRQRP7D3IB9/+Zu760fN84b6n2XvgMGeftprPXXIeHzl/HZWK+FdnncITz78y6HLtdaybzn0zMB4RP42IKeA2YEvbmC3ALfntO4CLJQmzZWbnv+znwGvTDFUqXHvn4zx/4DABPH/gMNfe+Th3/eh5ADZvOoNHnn2Zfa8eGWzB9rrVzWX21gHPtdyfAH6905j8mqsHgTOBF/tRZKsfPPMif37fU3Mejw7HiwRzN3Qc2/E5isYu7ACVvtTX8bkLnqNjId0/3Ol77FxHp+fu/WdQPHbp69v/yylGTlnJA0/v43DbaX0PT9f5wn1P85Hz17F180Zu/ednufgv/pGNZ5xEpaXXaW173AG9Pv3GBeu58j2bFvVrdBPuRa+/9pd+N2OQtA3YBrBx48YuvvRcK4YqnPmG4lUKnf5YKHq0898VHZ6j4OFOT9HpubWQ517gcxQ93Lm+3vdTP56787eS7s9g9fAQV71nE//+Cw8UPs/eA9mbqG9708nc9p8v5LYf/nxmdQ3M/oXk41dfvzpd6KWfugn3CWBDy/31wN4OYyYkDQGnAvvbnygitgPbAUZHR0u9tjefcwabz9lc5lPN+ubs01bz/IG5q2HOPm31zO0LNp7OBRtPX8qyzGZ0M+f+MHCupHMkrQC2AjvaxuwArsxvXwF8LxY6b2F2AvncJefNuVjH6uEqn7vkvAFVZDbbvJ17Pod+NXAfUAVujognJV0PjEXEDuAm4BuSxsk69q2LWbTZoH3k/HUAhatlzFKgQTXYo6OjMTY2NpCvbWZ2opL0SESMzjfOR6iamS1DDnczs2XI4W5mtgw53M3MliGHu5nZMuRwNzNbhhzuZmbLkMPdzGwZGthBTJImgWdLfvoaFuGMk32Qal2Qbm2ua2Fc18Isx7reEhEj8w0aWLj3QtJYN0doLbVU64J0a3NdC+O6Fub1XJenZczMliGHu5nZMnSihvv2QRfQQap1Qbq1ua6FcV0L87qt64Scczczs+M7UTt3MzM7jmTDXdJHJT0pqSFptG3btZLGJT0t6ZIOn3+OpJ2SnpF0e34VqX7XeLukXfnHzyTt6jDuZ5Iez8ct+knsJf2JpOdbarusw7hL8304LumaJajrC5KekrRb0t9KOq3DuCXZX/N9/5JW5j/j8fy1tGmxamn5mhskPSBpT/76/0zBmIskHWz5+V632HW1fO3j/myU+VK+z3ZLumAJajqvZV/skvSKpM+2jVmSfSbpZkn7JD3R8tgZku7Ps+h+SYXXXpR0ZT7mGUlXFo1ZkIhI8gP418B5wIPAaMvjbwceA1YC5wA/AaoFn/9tYGt++0bg04tc718A13XY9jNgzRLuuz8B/us8Y6r5vnsrsCLfp29f5Lo+CAzlt28AbhjU/urm+wf+C3BjfnsrcPsS/OzWAhfkt08B/m9BXRcBdy/V62khPxvgMuBesmuXXwjsXOL6qsD/I1sLvuT7DPh3wAXAEy2P/TlwTX77mqLXPXAG8NP839Pz26f3UkuynXtE7ImIpws2bQFui4ijEfEvwDgw64rZkgS8D7gjf+gW4COLVWv+9f4T8K3F+hqLYDMwHhE/jYgp4DayfbtoIuLvI6KW332I7GLrg9LN97+F7LUD2Wvp4vxnvWgi4oWIeDS//SqwBziRrt23Bbg1Mg8Bp0lau4Rf/2LgJxFR9gDJnkTE98kuNdqq9XXUKYsuAe6PiP0R8TJwP3BpL7UkG+7HsQ54ruX+BHNf/GcCB1qCpGhMP/1b4BcR8UyH7QH8vaRHJG1bxDpaXZ3/WXxzhz8Du9mPi+mTZB1ekaXYX918/zNj8tfSQbLX1pLIp4HOB3YWbH63pMck3SvpHUtVE/P/bAb9utpK5yZrUPvszRHxAmS/vIE3FYzp+36b9wLZi0nSd4GzCjZ9PiL+T6dPK3isfclPN2O60mWNH+P4Xft7I2KvpDcB90t6Kv8NX9rx6gK+CvwZ2ff8Z2RTRp9sf4qCz+156VQ3+0vS54Ea8M0OT9P3/VVUasFji/Y6WihJJwPfAT4bEa+0bX6UbNrhUP5+yl3AuUtRF/P/bAa5z1YAlwPXFmwe5D7rRt/320DDPSLeX+LTJoANLffXA3vbxrxI9ufgUN5xFY3pS42ShoDfAN51nOfYm/+7T9Lfkk0J9BRW3e47SV8D7i7Y1M1+7Htd+RtFHwYujnyyseA5+r6/CnTz/TfHTOQ/51OZ+yd330kaJgv2b0bEne3bW8M+Iu6R9L8krYmIRT+HShc/m0V5XXXpQ8CjEfGL9g2D3GfALyStjYgX8imqfQVjJsjeF2haT/Z+Y2kn4rTMDmBrvpLhHLLfvj9sHZCHxgPAFflDVwKd/hLo1fuBpyJiomijpDdIOqV5m+xNxSeKxvZL2xznf+zw9R4GzlW2qmgF2Z+zOxa5rkuB/w5cHhGvdRizVPurm+9/B9lrB7LX0vc6/ULql3xO/yZgT0R8scOYs5pz/5I2k/0/fmkx68q/Vjc/mx3Ax/NVMxcCB5tTEkug41/Qg9pnudbXUacsug/4oKTT82nUD+aPlbfY7x6X/SALpQngKPAL4L6WbZ8nW+nwNPChlsfvAc7Ob7+VLPTHgf8NrFykOr8O/F7bY2cD97TU8Vj+8STZ9MRi77tvAI8Du/MX1tr2uvL7l5GtxvjJEtU1TjavuCv/uLG9rqXcX0XfP3A92S8fgFX5a2c8fy29dQn20b8h+3N8d8t+ugz4vebrDLg63zePkb0x/Z7Frut4P5u22gR8Jd+nj9Oy0m2RazuJLKxPbXlsyfcZ2S+XF4DpPL8+RfY+zT8Az+T/npGPHQX+uuVzP5m/1saBT/Rai49QNTNbhk7EaRkzM5uHw93MbBlyuJuZLUMOdzOzZcjhbma2DDnczcyWIYe7mdky5HA3M1uG/j+wak17wBtVLQAAAABJRU5ErkJggg==\n", "text/plain": [ "
" ] }, "metadata": { "needs_background": "light" }, "output_type": "display_data" }, { "data": { "image/png": "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\n", "text/plain": [ "
" ] }, "metadata": { "needs_background": "light" }, "output_type": "display_data" }, { "data": { "image/png": "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\n", "text/plain": [ "
" ] }, "metadata": { "needs_background": "light" }, "output_type": "display_data" }, { "data": { "image/png": "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\n", "text/plain": [ "
" ] }, "metadata": { "needs_background": "light" }, "output_type": "display_data" } ], "source": [ "import matplotlib.pyplot as plt\n", "import numpy\n", "\n", "mu = 1\n", "sigmaX = 1\n", "sigmaZ = 1\n", "n = 100\n", "\n", "X = numpy.random.normal(mu,sigmaX,1)*numpy.ones(n)\n", "Z = numpy.random.normal(0,sigmaZ,n)\n", "Y = X + Z\n", "Xhat = numpy.ones(n)\n", "\n", "for i in range(n):\n", " x = numpy.linspace(-10,10,1000)\n", " l = numpy.exp(-0.5*(Y[i]-x)**2/sigmaZ**2)*numpy.exp(-0.5*(x-mu)**2/sigmaX**2)\n", " l = l/numpy.sum(l)*1000/20\n", " plt.plot(x,l)\n", " plt.scatter(numpy.array([X[0]]),numpy.array([0]))\n", " plt.show()\n", " Xhat[i] = numpy.sum(l*x)*20/1000\n", " helper=(mu/sigmaX**2+Y[i]/sigmaZ**2)\n", " sigmaX = numpy.sqrt(1/(1/sigmaX**2+1/sigmaZ**2))\n", " mu = sigmaX**2*helper" ] }, { "cell_type": "code", "execution_count": 11, "metadata": {}, "outputs": [ { "data": { "image/png": "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\n", "text/plain": [ "
" ] }, "metadata": { "needs_background": "light" }, "output_type": "display_data" } ], "source": [ "plt.plot(X)\n", "plt.plot(Y)\n", "plt.plot(Xhat)\n", "plt.show()" ] }, { "cell_type": "code", "execution_count": null, "metadata": {}, "outputs": [], "source": [] } ], "metadata": { "kernelspec": { "display_name": "Python 3", "language": "python", "name": "python3" }, "language_info": { "codemirror_mode": { "name": "ipython", "version": 3 }, "file_extension": ".py", "mimetype": "text/x-python", "name": "python", "nbconvert_exporter": "python", "pygments_lexer": "ipython3", "version": "3.7.7" } }, "nbformat": 4, "nbformat_minor": 2 }